You are on page 1of 434

.

A WORKBOOK OF
SCIENCE & TECHNOLOGY
+ ENVIRONMENT
For Civil Services Prelims Examination

WORKBOOK

Forum Learning Centre


DELHI PATNA HYDERABAD
2nd Floor, IAPL House, 2nd Floor, AG Palace, East 1st & 2nd Floor, SM Plaza,
19, Pusa Road, Opp. Metro Boring Canal Road Patna - RTC X Rd, Indira Park
Pillar 95-96, Karol Bagh - 800001 Road, Jawahar Nagar,
110005 Telangana - 500020

https://blog.forumias.com | www.academy.forumias.com | 9821711605


.
THE WORKBOOKS
Dear Aspirants,

On the basis of the success of the previous series of workbooks as most of the UPSC CSE Prelims
2022 questions came from our workbook, we are glad to inform you that we have come up with the
2nd series of Workbooks to aid your Prelims preparations. The Workbooks are a series of question
banks of MCQs aimed at covering the entire syllabus holistically. How to use the workbook?
The workbook of each subject, consist of a set of units intended to cover the entire subject. Each
unit consist of 2 parts:
1. Previous Year Questions: This part contains all the PYQs asked by the Commission in that
specific Unit.
2. Practise Questions: This section contains MCQs for your practise, which are divided into
sub-sections. This will help the students to practise questions on specific sub-topics.
The combination of both these sections will help our students to check the questions asked by the
commission and at the same time practise similar question. You can also use these workbooks to
solve questions as and when you prepare small portions of the syllabus.
No preparation is complete unless you have understood the concept and applied them in solving
questions. Solving questions is the only way to validate your knowledge. Therefore, the workbooks
would be the perfect friend which will test your conceptual clarity and challenge your knowledge
base.
Happy Learning and Happy Testing!!!
– Your team @ForumIAS

No part of this publication may be reproduced or transmitted, in any form or by any means,
electronic, mechanical, photocopying, recording or otherwise, or stored in any retrieval system of
any nature without the written permission of the copyright holder and the publisher, application
for which shall be made to the publisher.
©Forum (Flaviant Network Pvt. Ltd.)

This book/material is sold subject to the conditions that it shall not, by way of trade or otherwise,
be lent, re-sold, hired out or otherwise circulated without the publisher’s prior consent in any
form of binding or cover other than that in which it is published and without a similar condition
including this condition being imposed on the subsequent purchaser.

We are continuously working on improving our quality. If you are unhappy or happy with the
quality of this study material, want to report an error, or want to leave us feedback, you can email
us at ravi@forumias.com

© 2023 2nd Print Edition | For feedback email us : ravi@forumias.com


.
INDEX
SECTION - A
UNIT 1. GENERAL SCIENCE .....1 – 91
1.1. Previous Years Questions .....1
1.2. General Physics .....4
1.3. General Chemistry .....10
1.4. General Biology .....16
SOLUTIONS .....29

UNIT 2. APPLIED SCIENCE .....92 – 191


2.1. Previous Years Questions .....92
2.2. Biotechnology .....104
2.3. Science in Environment and Agriculture .....107
2.4. Health and Medicine .....109
2.5. Information Technology .....113
2.6. Astronomy and Space .....117
2.7. Energy .....122
2.8. Defence Technology .....125
2.9. New Technology and Miscellaneous .....126
SOLUTIONS .....130

SECTION - B

UNIT 3. ENVIRONMENT AND ECOLOGY .....192 – 226


3.1. Previous Years Questions .....192
3.2. Ecology and Ecosystem .....194
3.3. Functions of ecosystem .....198
3.4. Terrestrial Ecosystem .....200
3.5. Aquatic Ecosystem .....201
SOLUTIONS .....205

UNIT 4. ENVIRONMENTAL POLLUTION .....227 – 259


4.1. Previous Years Questions .....227
4.2. Pollution and Major pollutants .....230
4.3. Environmental Issues .....235
.
4.4. EIA .....236
4.5. Health impact of pollution .....238
4.6. Renewable energy and energy efficiency .....238
SOLUTIONS .....240

UNIT 5. BIODIVERSITY .....260 – 317


5.1. Previous Years Questions .....260
5.2. Biodiversity in India .....265
5.3. Animal diversity in india .....269
5.4. Plant diversity of India .....273
5.5. Marine organism .....274
5.6. Protected Area network .....275
SOLUTIONS .....281

UNIT 6. CLIMATE CHANGE .....318 – 344


6.1. Previous Years Questions .....318
6.2. Global warming, GHG and climate forcing .....319
6.3. Ocean acidification .....322
6.4. Ozone depletion .....323
6.5. Mitigation strategies .....324
SOLUTIONS .....328

UNIT 7. GOVERNMENT EFFORTS AND INITIATIVES .....345 – 430


7.1. Previous Years Questions .....345
7.2. Schemes, Projects and Measures .....353
7.3. Acts and Policies .....358
7.4. Indian Environmental bodies, organisations and reports .....363
7.5. International Environmental Organisations, Conventions And Initiatives .....366
SOLUTIONS .....380
.
SCIENCE & TECHNOLOGY

SCIENCE & TECHNOLOGY


GENERAL SCIENCE
5. When the bark of a tree is removed in a circular
1.1. Previous Years Questions fashion all around near its base, it gradually dries
1. The surface of a lake is frozen in severe winter, but up and dies because? [UPSC CSE Pre. 2011]
the water at its bottom is still liquid. What is the (a) Water from soil cannot rise to aerial parts.
reason? [UPSC CSE Pre. 2011] (b) Roots are starved of energy.
(a) Ice is a bad conductor of heat (c) Tree is infected by soil microbes.
(b) Since the surface of the lake is at the same (d) Roots do not receive oxygen for respiration.
temperature as the air, no heat is lost.
(c) The density of water is maximum at 4°C. 6. With reference to street-lighting, how do sodium
(d) None of the statements (a), (b) and (c) given is lamps differ from led lamps?
correct. [UPSC CSE Pre. 2011]
1. Sodium lamps produce light in 360 degrees but it
2. A married couple adopted a male child. A few is not so in the case of LED lamps.
years later, twin boys were born to them. The blood
group of the couple is AB positive and O negative. 2. As street-lights, sodium lamps have longer life
The blood group of the three sons is A positive, B span than LED lamps.
positive, and O positive. The blood group of the 3. The spectrum of visible light from sodium lamps
adopted son is? [UPSC CSE Pre. 2011] is almost monochromatic while LED lamps offer
(a) O positive significant colour advantages in street-lighting.
(b) A positive Select the correct answer using the code given below.
(c) B positive (a) 3 only
(d) Cannot be determined on the basis of the given (b) 2 only
data (c) 1 and 3 only
3. Regular intake of fresh fruits and vegetables is (d) 1, 2 and 3
recommended in the diet since they are a good
7. Water can dissolve more substances than any other
source of antioxidants. How do antioxidants help
liquid because [UPSC CSE Pre. 2011]
a person maintain health and promote longevity?
[UPSC CSE Pre. 2011] (a) It is dipolar in nature
(b) It is a good conductor of heat
(a) They activate the enzymes necessary for vitamin
(c) It has high value of specific heat
synthesis in the body and help prevent vitamin
deficiency (d) It is an oxide of hydrogen
(b) They prevent excessive oxidation of 8. The surface of a lake is frozen in severe winter, but
carbohydrates, fats and proteins in the body and the water at its bottom is still liquid. What is the
help avoid unnecessary wastage of energy reason? [UPSC CSE Pre. 2011]
(c) They neutralize the free radicals produced in the (a) Ice is a bad conductor of heat.
body during metabolism
(b) Since the surface of the lake is at the same
(d) They activate certain genes in the cells of the
body and help delay the ageing process temperature as the air, no heat is lost.
(c) The density of water is maximum at 4°C.
4. Which of the following statements is/are correct (d) None of the statements (a), (b) and (c) given is
regarding vegetative propagation of plants? correct.
[UPSC CSE Pre. 2011]
9. The function of heavy water in a nuclear reactor is
1. Vegetative propagation produces clonal
to [UPSC CSE Pre. 2011]
population.
(a) Slow down the speed of neutrons
2. Vegetative propagation helps in eliminating the
virus. (b) Increase the speed of neutrons
(c) Cool down the reactor
3. Vegetative propagation can be practiced most of
(d) Stop the nuclear reaction
the year.
Select the correct answer using the code given below. 10. Electrically charged particles from space travelling
(a) 1 only at speeds of several hundred km/sec can severely
(b) 2 and 3 only harm living beings if they reach the surface of the
(c) 1 and 3 only Earth. What prevents them from reaching the
(d) 1, 2 and 3 surface of the Earth? [UPSC CSE Pre. 2012]

1 Workbook
.
SCIENCE & TECHNOLOGY

(a) The Earth’s magnetic field diverts them towards 3. Viruses are transmitted from one organism to
its poles another by biological vectors only.
(b) Ozone layer around the Earth reflects them back Select the correct answer using the codes given
to outer space below.
(c) Moisture in the upper layers of atmosphere (a) 1 only
prevents them from reaching the surface of the (b) 2 and 3 only
Earth (c) 1 and 3 only
(d) None of the statements (a), (b) and (c) given (d) 1, 2 and 3
above is correct
16. Ball bearings are used in bicycles, cars, etc.,
11. Consider the following statements: because [UPSC CSE Pre. 2013]
[UPSC CSE Pre. 2012] (a) the actual area of contact between the wheel and
If there were no phenomenon of capillarity axle is increased
1. it would be difficult to use a kerosene lamp (b) the effective area of contact between the wheel
and axle is increased
2. one would not be able to use a straw to consume
(c) the effective area of contact between the wheel
a soft drink and axle is reduced
3. he blotting paper would fail to function (d) None of the above statements is correct
4. the big trees that we see around would not have
grown on the Earth 17. The known forces of nature can be divided into
four classes, viz., gravity, electromagnetism, weak
Which of the statements given above are correct? nuclear force and strong nuclear force. With
(a) 1, 2 and 3 only reference to them, which one of the following
(b) 1, 3 and 4 only statements is not correct? [UPSC CSE Pre. 2013]
(c) 2 and 4 only (a) Gravity is the strongest of the four
(d) 1, 2, 3 and 4 (b) Electromagnetism acts only on particles with an
electric charge
12. Fruits stored in a cold chamber exhibit longer
(c) Weak nuclear force causes radioactivity
storage life because [UPSC CSE Pre. 2013]
(d) Strong nuclear force holds protons and neutrons
(a) exposure to sunlight is prevented inside the nucleus of an atom
(b) concentration of carbon dioxide in the
environment is increased 18. Rainbow is produced when sunlight falls on drops
(c) rate of respiration is decreased of rain. [UPSC CSE Pre. 2013]
(d) there is an increase in humidity Which of the following physical phenomena are
13. Many transplanted seedlings do not grow because responsible for this?
[UPSC CSE Pre. 2013] 1. Dispersion
(a) the new soil does not contain favorable minerals 2. Refraction
(b) most of the root hair grip the new soil too hard 3. Internal reflection
(c) most of the root hairs are lost during Select the correct answer using the codes below.
transplantation (a) 1 and 2 only
(d) leaves get damaged during transplantation (b) 2 and 3 only
14. Consider the following minerals: (c) 1 and 3 only
[UPSC CSE Pre. 2013] (d) 1, 2 and 3
1. Calcium 19. Graphene is frequently in news recently. What is
2. Iron its importance? [UPSC CSE Pre. 2013]
3. Sodium 1. It is a two-dimensional material and has good
Which of the minerals given above is/are required by electrical conductivity.
human body for the contraction of muscles? 2. It is one of the thinnest but strongest materials
tested so far.
(a) 1 only
(b) 2 and 3 only 3. It is entirely made of silicon and has high optical
transparency.
(c) 1 and 3 only
(d) 1, 2 and 3 4. It can be used as ‘conducting electrodes’ required
for touch screens, LCDs and organic LEDs.
15. Which of the following statements is/are correct? Which of the statements given above are correct?
[UPSC CSE Pre. 2013] (a) 1 and 2 only
1. Viruses lack enzymes necessary for the (b) 3 and 4 only
generation of energy. (c) 1, 2 and 4 only
2. Viruses can be cultured in any synthetic medium. (d) 1, 2, 3 and 4

Workbook 2
.
SCIENCE & TECHNOLOGY

20. Consider the following phenomena: Which of the pairs given above is/are correctly
[UPSC CSE Pre. 2013] matched?
1. Size of the sun at dusk. (a) 1 only
2. Color of the sun at dawn (b) 2 and 3 only
3. Moon being visible at dawn (c) 1 and 3 only
(d) 1, 2 and 3
4. Twinkle of stars in the sky
5. Polestar being visible in the sky 25. Consider the following statements:
Which of the above are optical illusions? [UPSC CSE Pre. 2018]
(a) 1, 2 and 3 1. Light is affected by gravity.
(b) 3, 4 and 5 2. The Universe is constantly expanding.
(c) 1, 2 and 4 3. Matter warps its surrounding space-time.
(d) 2, 3 and 5
Which of the above is/are the prediction/predictions
21. Which one of the following is the process involved of Albert Einstein’s General Theory of Relativity,
in photosynthesis? [UPSC CSE Pre. 2014] often discussed in media?
(a) Potential energy is released to form free energy (a) 1 and 2 only
(b) Free energy is converted into potential energy (b) 3 only
and stored (c) 1 and 3 only
(c) Food is oxidized to release carbon dioxide and (d) 1, 2 and 3
water
(d) Oxygen is taken, and carbon dioxide and water 26. What is the reason for Non-Metals to be a bad
vapor are given out conductor of electricity? [UPSC CSE Pre. 2018]
(a) High melting point at normal temperature
22. Among the following organisms, which one does
(b) Excessive free protons around nucleus
not belong to the class of other three?
(c) Absence of readily available free electrons
[UPSC CSE Pre. 2014] (d) Low resistance to external factors
(a) Crab
(b) Mite 27. With reference to the recent developments in
(c) Scorpion science, which one of the following statements is
(d) Spider not correct? [UPSC CSE Pre. 2019]
(a) Functional chromosomes can be created by
23. Which of the following is/are the example/
joining segments of DNA taken from cells of
examples of chemical change?
different species.
[UPSC CSE Pre. 2014] (b) Pieces of artificial functional DNA can be created
1. Crystallization of sodium chloride in laboratories.
2. Melting of ice (c) A piece of DNA taken out from an animal cell
3. Souring of milk can be made to replicate outside a living cell in
Select the correct answer using the code given below. a laboratory.
(a) 1 and 2 only (d) Cells taken out from plasma and animals can be
(b) 3 only made to undergo cell division in laboratory petri
(c) 1, 2 and 3 dishes.
(d) None 28. Which one of the following is a reason why
24. Consider the following pairs: astronomical distances are measured in light-
[UPSC CSE Pre. 2017] years? [UPSC CSE Pre. 2021]
(a) Distances among stellar bodies do not change.
Commonly used/ Unwanted or (b) Gravity of stellar bodies does not change.
consumed materials controversial (c) Light always travels in straight line.
chemicals likely to (d) Speed of light is always same.
produce found in 29. Consider the following statements in respect of
them probiotics: [UPSC CSE Pre. 2022]
1. Lipstick Lead 1. Probiotics are made of both bacteria and yeast.
2. The organisms in probiotics are found in foods
2. Soft Drinks Brominated vegetable we ingest but they do not naturally occur in our
oils gut.
3. Chinese fast food Monosodium 3. Probiotics help in the digestion of milk sugars.
glutamate Which of the statements given above is/are correct?

3 Workbook
.
SCIENCE & TECHNOLOGY

(a) 1 only (a) It is formed out of the Earth’s magnetosphere.


(b) 2 only (b) The Geotail region provides better conditions for
(c) 1 and 3 space observations.
(d) 2 and 3 (c) The region exists as a result of the interactions
between the Sun and Earth.
30. Which one of the following statements best (d) All of the above.
describes the role of B cells and T cells in the
human body? [UPSC CSE Pre. 2022] 35. Consider the following statements:
(a) They protect the body from environmental 1. Planets always have more lithium content than
allergens. their Stars within their respective galaxies.
(b) They alleviate the body’s pain and inflammation. 2. Earth has more lithium content than the Sun.
(c) They act as immunosuppressants in the body. 3. Stars have a tendency to destroy lithium as they
(d) They protect the body from the diseases caused evolve.
by pathogens.
Which of the statements given above is/are correct?
Practice Question (a) 1 and 2 only
(b) 2 and 3 only
(c) 1 and 3 only
1.2. General Physics (d) 1, 2 and 3
31. Which of the following events is/are associated 36. Regarding the Bose-Einstein Condensate (BEC),
with the phenomenon of ‘Solar Minimum’? consider the following statements:
1. Reduction in sun spots and solar flares. 1. At Bose-Einstein Condensate, each atom
2. Weakening of the sun’s magnetic field. behaves as a separate individual entity.
3. Non-occurrence of total solar eclipse. 2. BECs can be easily studied on Earth, but not in
Select the correct answer using the code given below: outer space.
(a) 1 and 2 only 3. Recently, scientists have observed BEC for the
(b) 1 only first time in space.
(c) 2 and 3 only Which of the statements given above is/are correct?
(d) 1, 2 and 3 (a) 2 and 3 only
32. Consider the following statements: (b) 1 and 3 only
1. Black Holes are the regions in the space having (c) 3 only
almost negligible gravitational pull. (d) 1, 2 and 3
2. Dark Matter doesn’t interact with baryonic 37. In the context of space research, the term ‘Ceres’
matter and it is completely invisible to light. was recently seen in news. Which of the following
3. Solar Storm can be caused by a sudden release of statements is/are correct regarding ‘Ceres’?
high energy particles from the Sun. 1. It is an icy exoplanet.
Which of the statements given above is/are correct? 2. It is located in the asteroid belt.
(a) 1 and 3 only Select the correct answer using the code given below:
(b) 2 and 3 only (a) 1 only
(c) 2 only (b) 2 only
(d) 1, 2 and 3
(c) Both 1 and 2
33. ‘Raman Spectroscopy’ has the potential (d) Neither 1 nor 2
applications in which of the following?
38. Recently, the third unit of the Kakrapar Atomic
1. Monitoring and quality control in industrial Power Project (KAPP-3) achieved its first
food processing. criticality. What is/are the significance(s) of this
2. In advanced nanotechnology including cancer unit?
detection. 1. It is India’s biggest indigenously developed
3. In the analysis of drugs. variant of the Fast Breeder Reactor.
Select the correct answer using the codes given 2. It is the first nuclear power project located in
below: Northeast India.
(a) 1 and 2 only Select the correct the answer using the code given
(b) 3 only below:
(c) 1 and 3 only
(a) 1 only
(d) 1, 2 and 3
(b) 2 only
34. Which of the following statements is/are correct (c) Both 1 and 2
regarding ‘Geotail’, sometimes seen in the news? (d) Neither 1 nor 2

Workbook 4
.
SCIENCE & TECHNOLOGY

39. Suppose by some means, we start using ‘electron (a) 1 only


gas with ultra-high mobility’ in the manufacturing (b) 2 only
of electronic devices. Then, which of the following (c) Both 1 and 2
benefit/benefits is/are likely to accrue to us? (d) Neither 1 nor 2
1. Data storage and memory of the device might
increase. 44. Which of the following statements is/are correct
2. The device would not require any input energy regarding to the superconductivity?
to operate. 1. Superconductivity is the complete disappearance
Select the correct answer using the code given below: of electrical resistance in various solids when they
are heated above a characteristic temperature.
(a) 1 only
(b) 2 only 2. The main advantages of devices made from
(c) Both 1 and 2 superconductors are low power dissipation,
(d) Neither 1 nor 2 high-speed operation, and high sensitivity.
40. Which of the following is/are the result of Select the correct answer using the code given below:
refraction of light? (a) 1 only
1. Formation of rainbow. (b) 2 only
2. A swimming pool looking shallower than it (c) Both 1 and 2
actually is. (d) Neither 1 nor 2
3. Formation of mirage on the road on a hot day. 45. The splitting of incoming white light by a prism
Select the correct answer using the code given below: into its constituent seven colours is known as?
(a) 1 only (a) Refraction
(b) 2 and 3 only (b) Dispersion
(c) 1 and 3 only (c) Total internal reflection
(d) 1, 2 and 3 (d) Tyndal Effect
41. They are the orphaned moons that have escaped 46. Consider the following pairs:
the bonds of their planetary parents. These
objects begin as moons around large planets, but Disease Lens used to correct
eventually move out on their own. They are also
1. Myopia Concave lens
known as tidally detached exomoons.
What is being referred to in the above passage? 2. Hypermetropia Cylindrical lens
(a) Exoplanet 3. Astigmatism Convex lens
(b) Dwarf planet Which of the above pair(s) is/are correctly matched?
(c) Ploonet (a) 1 only
(d) Red Dwarf star (b) 1 and 2 only
42. Consider the following statements: (c) 2 and 3 only
1. Sonar uses ultrasonic waves to measure the (d) 1, 2 and 3
direction of underwater objects. 47. It is a naturally occurring optical phenomenon in
2. Lidar uses light in the form of a pulsed laser to which light rays bend via refraction to produce a
measure variable distances to the Earth. displaced image of distant objects or the sky, and
3. Lidar assists in emergency operations during fog the land is mistaken for a lake or sheet of water.
and snow. What is the phenomenon being described here?
(a) Looming
Which of the statements given above is/are correct? (b) Mirage
(a) 1 and 2 only (c) Dispersion
(b) 3 only (d) Tyndall Effect
(c) 1 and 3 only
(d) 1, 2 and 3 48. Consider the following statements:
1. The convex lens is a lens that converges rays of
43. Recently, the spectrum auction for 4G airwaves light that convey parallel to its principal axis.
was conducted. In this context, consider the 2. Concave mirrors reflect light inward to one focal
following statements: point.
1. Spectrum refers to a range of radio-waves that 3. The image by a concave mirror for an object kept
are used for communication purposes. at its focus will be formed at infinity.
2. Lower airwaves bands provide a good indoor 4. The image by a convex mirror for an object kept
coverage while higher bands have a good at infinity will be formed at the focus, behind the
carrying capacity. mirror.
Which of the statements given above is/are correct? Which of the above statements are correct?

5 Workbook
.
SCIENCE & TECHNOLOGY

(a) 1, 2 and 3 only which of the following does this nanosheet has
(b) 1, 2 and 4 only applications?
(c) 2, 3 and 4 only 1. Production of electric current
(d) 1, 2, 3 and 4 2. Check the presence of contaminants in drinking
49. Which of the following best describes the term water
‘Milankovitch cycle’? 3. Detection of alcohol in breath
(a) It refers to the circadian rhythms that govern our Select the correct answer using the code given below:
sleep patterns.
(b) It refers to the annual animal migrations across (a) 1 and 2 only
the globe using Earth’s magnetic field (b) 2 and 3 only
(c) It refers to the Water circulation in the deep (c) 1 and 2 only
oceans affecting fish population near coastal (d) 1, 2 and 3
areas 54. A person standing waist-deep in a swimming pool
(d) It refers to how the tilt of the Earth, the shape appears to have short legs. Which of the following
of its orbit and the direction where its axis is phenomena explains this condition?
pointed collectively influence its climate.
(a) Reflection of light
50. With reference to black holes, consider the (b) Dispersion of light
following statements: (c) Diffraction of light
1. Event Horizon Telescope is a group of radio (d) Refraction of light
telescopes which recently captured image of
55. With reference to Superconductivity, consider the
black hole.
following statements:
2. Primordial Black Holes have masses comparable
1. It is a state in which a material shows absolutely
to or less than that of the Earth.
zero electrical resistance.
3. Black holes are always stationary and does not
2. It could be easily achieved at high temperatures
move from their places.
in the range of 100°C and above.
Which of the statements given above are correct? 3. It can be used to run high-speed magnetic-
(a) 1 and 2 only levitation trains.
(b) 1 and 3 only
Which of the statements given above are correct?
(c) 2 and 3 only
(d) 1, 2 and 3 (a) 1 and 2 only
(b) 2 and 3 only
51. Consider the following statements: (c) 1 and 3 only
1. Wolf–Rayet stars are the coldest stars, having (d) 1, 2 and 3
very low luminosity. 56. What are the possible reasons behind the
2. The origin of a Supernova cannot be traced to launching of satellites from the eastern coast near
a star. the Equator?
3. Neutron stars are formed when a massive star 1. This will give an initial boost equal to the velocity
collapses. of Earth surface.
Which of the statements given above is/are correct? 2. To take the advantage of angular momentum of
(a) 1 and 2 only Earth’s rotation.
(b) 2 and 3 only 3. In case of failure, satellite does not fall on built-
(c) 3 only up hinterland.
(d) 1 and 3 only Which of the statements given above is/are correct?
52. “This theory proposed that the laws of physics are (a) 1 and 2 only
the same for all non-accelerating observers. It also (b) 2 and 3 only
showed that the speed of light within a vacuum is (c) 1 and 3 only
the same no matter the speed at which an observer (d) 1, 2 and 3
travel.” 57. Consider the following statements:
Which one of the following is described in the above 1. Sun produces its energy through Nuclear Fission
paragraph? only.
(a) Big-Bang Theory 2. The outermost part of Sun’s atmosphere is called
(b) Theory of Special Relativity Corona.
(c) Quantum Theory of Matter 3. Sunspots are dark area occurring in Sun’s
(d) String Theory photosphere.
53. Recently, some scientists have discovered a new 4. The giant bubble surrounding the Sun and the
method of soft ionization using Nanosheets solar system is called Heliosphere.
prepared from molybdenum disulphide. In Which of the statements given above is/are correct?

Workbook 6
.
SCIENCE & TECHNOLOGY

(a) 1, 2, 3 and 4 62. Which of the following is/are the characteristic (s)
(b) 2, 3 and 4 only of the Sunspots, found on the surface of the Sun?
(c) 2 and 4 only 1. They appear brighter than the surrounding
(d) 3 and 4 only regions.
58. With reference to electromagnetic radiation, 2. They are relatively warmer than the nearby
which of the following statements is/are incorrect? regions.
1. All electromagnetic radiation reach the earth’s 3. Sun’s magnetic fields are particularly strong in
surface. these regions.
2. In an Electromagnetic spectrum, as one moves Select the correct answer using the code given below:
from gamma rays to radio waves the wavelength (a) 1 and 2 only
goes on increasing. (b) 2 only
3. Radio waves can pass through metal and water. (c) 3 only
(d) 1, 2 and 3
Select the correct answer using the code given below:
(a) 1 and 2 only 63. Which of the following is the correct reason
(b) 2 only behind the reddish colour of the sky during sunset
(c) 1 and 3 only and sunrise?
(d) 1,2 and 3 (a) Light rays don’t get the opportunity to scatter
properly as they do during the day time.
59. What does the term Exoplanets, often seen in news, (b) The high concentration of aerosols in the air
mean? during evening causes reddish colours.
(a) Cosmic aggregations of dust, ice, frozen gases (c) The high angle of sun emits low wavelengths
and rocks orbiting the sun in highly elliptical during evening causing reddish colours.
orbits. (d) Light rays have to travel larger distances and
(b) Celestial objects with round shape orbiting the scatter multiple times.
sun but unable to clear their orbital path.
64. Consider the following statements:
(c) Planets orbiting a star other than our sun.
(d) Free Floating planets orbiting around a galactic 1. The opposite poles of different magnet always
centre and not around any star. attract each other.
2. The similar poles of different magnet always
60. It is a dying star in the last stages of stellar repel each other.
evolution. When hydrogen fuel at the centre of a Which of the following statements is/are correct?
star is exhausted, nuclear reactions will start to
(a) 1 only
move outwards into its atmosphere and burn the (b) 2 only
hydrogen that’s in a shell surrounding the core. (c) Both 1 and 2
As a result, the outside of the star starts to expand (d) Neither 1 nor 2
and cool, turning much redder. Over time, the star
will change into a new celestial body and grow to 65. In Which of the following medium Sound cannot
more than 400 times its original size. travel-
Identify the celestial body or phenomenon from the 1. Air
above paragraph. 2. Water
(a) Red dwarf 3. Vacuum
(b) Black hole 4. Diamond
(c) Neutron star Which of the above statements is/are correct in the
(d) Red giant above context?
61. Consider the following statements: (a) 1 and 2 only
(b) 3 only
1. White colour of clouds is caused due to scattering (c) 3 and 4 only
of light. (d) 1, 3 and 4 only
2. Diffraction is the phenomenon of slight bending
of light as it passes around the edge of an object. 66. Which of the following phenomenon involves the
3. Rainbows are the result of the refraction and process of evaporation?
reflection of light. 1. Drying of clothes
Which of the statements given above are correct? 2. Crystallization
3. Preparation of common salt
(a) 1 and 2 only
(b) 1 and 3 only 4. Preparation of Juice/ Milk concentrates
(c) 2 and 3 only 5. Melting of ice cube
(d) 1, 2 and 3 Select the correct answer using the code given below:

7 Workbook
.
SCIENCE & TECHNOLOGY

(a) 1, 2 and 3 only (c) Echo


(b) 1, 3, 4 and 5 only (d) Diffraction
(c) 1, 3 and 4 only
72. Which of the following phenomena involves the
(d) 1, 2, 3, 4 and 5 capillary action?
67. Which of the following statements best describes 1. Uptake of water in paper and plaster
the term Van der Waals forces? 2. The quick drying of paint between the hairs of
(a) These forces are a result of attraction between a paintbrush
opposite charged particles 3. The movement of water through sand
(b) These forces arise from the interactions between 4. Absorption of water by sponge
uncharged atoms or molecules. Select the correct answer using the codes given
(c) These forces develop in a metallic body which below:
does not show magnetism. (a) 1, 2 and 3 only
(d) These forces help in changing the state of matter (b) 2, 3 and 4 only
from liquid to gaseous state. (c) 1, 3 and 4 only
68. Consider the following statements: (d) 1, 2, 3 and 4
1. The loudness or softness of a sound is determined 73. Consider the following statements with reference
by its frequency. to the various types of electromagnetic radiations:
2. A loud sound has more frequency than a soft 1. Infrared waves cannot be seen by human eyes.
sound. 2. Microwaves can be used in remote sensing.
Which of the above statements is/are correct? 3. Radio waves have the shortest wavelengths in the
(a) 1 only electromagnetic spectrum.
(b) 2 only Which of the statements given above is/are correct?
(c) Both 1 and 2 (a) 1 and 2 only
(d) Neither 1 nor 2 (b) 2 and 3 only
(c) 1 and 3 only
69. Which of the following statements correctly (d) 1, 2, and 3
explains why shoulder bags are provided with
broad straps and not thin straps? 74. “Place a rigid card on an empty glass tumbler and
(a) Broad straps apply more pressure on the then keep a coin on that card. Now try to flick the
shoulders than thin straps. card speedily with your finger. You will observe
(b) Broad straps are stronger to carry load than thin that the coin will fall vertically into the glass
straps. tumbler while the card flows side away.”
(c) Force exerted per unit area by broad straps is less Which of the following laws is being observed in the
on the shoulders than thin straps. above instance?
(d) Broad straps do not allow the bag to slide down (a) Law of Inertia
from the shoulders. (b) Law of conservation of energy
70. Why do pendulum clocks run slower in summer as (c) Newton’s second law of motion
compared to winter? (d) Law of impulse
(a) The temperature coefficient of metal wire is high 75. “When a gun is fired, it exerts a forward force on
in summer. the bullet. The bullet exerts an equal and opposite
(b) The resistivity coefficient of wire decreases with reaction force on the gun. This results in the recoil
increase in temperature. of the gun” . Which of the following principle is
(c) The length of metal wire decreases in summer. being obeyed in the above instance?
(d) The acceleration due to gravity increases with (a) Conservation of momentum.
increase in temperature. (b) Newton’s first law of motion.
(c) Newton’s second law of motion.
71. Consider the following phenomenon:
(d) Conservation of energy.
It is the collection of reflected sounds from the
surfaces in a closed hall or auditorium. It is a 76. Consider the following statements with reference
desirable property of auditoriums to a certain extent. to the General Relativity and Quantum Mechanics:
However, if it is excessive, it makes the sounds 1. In general relativity, events are continuous and
run together with loss of articulation - the sound deterministic.
becomes muddy, garbled. 2. In quantum mechanics, events produced are
Which among the following has been described with probabilistic rather than definite outcomes.
above? 3. The theory of quantum mechanics describes the
(a) Resonance behaviour of light as both particles and as waves.
(b) Reverberation Which of the statements given above is/are correct?

Workbook 8
.
SCIENCE & TECHNOLOGY

(a) 3 only 2. When the density of an object is more than the


(b) 1 and 2 only density of water, then the object sinks.
(c) 2 and 3 only Which of the statements given above is/are correct?
(d) 1, 2 and 3
(a) 1 only
77. During lightening, a person would be safest: (b) 2 only
(a) Under a tree (c) Both 1 and 2
(b) In open area (d) Neither 1 nor 2
(c) Inside a closed car 83. Which of the following statements is/are correct
(d) Along concrete walls regarding characteristics of Translucent and
78. If we see an image of a tree in the water of a nearby luminous objects?
pond or a lake, what phenomenon would it be? 1. Translucent objects reflect most of the light
(a) Refraction received from luminous objects.
(b) Reflection 2. If a translucent object is placed in path of
(c) Total Internal Reflection luminous object, it results in formation of
(d) Shadow formation shadows.
79. Consider the following statements: Which of the statements given above is/are correct?
1. The weight of an object differs on poles and (a) 1 only
equator due to the shape of the earth. (b) 2 only
2. The weight of an object placed in a moving lift (c) Both 1 and 2
increases whenever the lift moves upwards. (d) Neither 1 nor 2
Which of the statements given above is/are correct? 84. With reference to ‘Ignition temperature’, which of
(a) 1 only the following statement is correct?
(b) 2 only (a) It is the lowest temperature where fluid can
(c) Both 1 and 2 evaporate to form a combustible concentration
(d) Neither 1 nor 2 of gas.
80. It is the phenomenon when abnormal refraction (b) It is the lowest temperature at which a substance
increases the apparent elevation of distant objects catches fire.
— often lifting the object above the horizon. This (c) It is the lowest temperature at which a volatile
is an optical illusion seen at sea shores in winter combustible substance continues to burn in air
evening, due to which a ship is seen formed in air after its vapours catches fire.
in sea-sky and the actual ship is nowhere visible. (d) It is the temperature at which complete
combustion process occurs without any work or
The above paragraph is describing about which of heat transfer.
the following phenomenon?
(a) Looming 85. If two conducting bodies are connected and the
(b) Dispersion electric current does not flow in between the two
(c) Atmospheric Refraction conducting bodies then it means
(d) Pillars (a) both have the same quantity of charge.
(b) their potential difference is zero.
81. Consider the following statements. (c) their temperature difference is zero.
1. If two objects are rubbed together, both get (d) they have the same ratio of potential per unit
charged due to process of convection. charge.
2. Whenever the two objects are rubbed, one
86. Suppose a Girl go to the school by riding on a
gets neutral and the other becomes positively
bicycle. At the time of reaching her school she
charged. stops pedaling. When she stops pedalling, the
3. . An attractive force develops between the bicycle begins to slow down.
rubbing objects.
What may be the possible Reason/Explanation for
Which of the statements given above is/are correct? this?
(a) 1 and 2 only (a) Newton’s first law of motion.
(b) 2 only (b) force of friction from the ground.
(c) 3 only (c) Both a and b
(d) 1 and 3 only (d) None of the above
82. Consider the following statements 87. With reference to the pitch of the sound, consider
1. When the buoyant force is equal to the weight the following statements:
of the object, then the object rises to the surface 1. Higher the frequency of the sound waves, higher
and floats. is its pitch.

9 Workbook
.
SCIENCE & TECHNOLOGY

2. Lion makes a low-pitched roar whereas bird 92. Consider the following pairs:
makes a high-pitched sound.
Events Phenomena
Which of the statement given above is/are correct?
Responsible
(a) 1 only
(b) 2 only 1. Appearance of Rayleigh’s scattering
(c) Both 1 and 2 blue sky
(d) Neither 1 nor 2 2. Twinkling of stars Atmospheric
88. Why is it advised to tie any luggage kept on the reflection
roof of a bus with a rope? 3. Formation of Refraction and
(a) While the bus is moving, luggage tends to rainbow dispersion of sunlight
remain in inertia of motion state and can move
4. Mirage Refraction of light
backward.
through a uniform
(b) When the bus stops, the luggage tends to resist medium
the change and due to inertia of motion it moves
forward. Which of the pairs given above is/are correct?
(c) Both a and b are true (a) 1 and 3 only
(d) None of the above is true (b) 1, 2 and 4 only
(c) 2 and 3 only
89. With reference to echo, which of the following (d) 2, 3 and 4 only
statements is/are correct?
93. Regarding the fermions and bosons, which of the
1. To hear a distinct echo the time interval between
following statements is not correct?
the original sound and the reflected one must be
at least 0.1s. (a) Fermions are particles that make up matter while
bosons are particles that carry forces.
2. The minimum distance of reflecting object (b) Fermions include electrons and neutrons while
should be around 17 m to hear echo. bosons include photons.
3. Echo is heard sooner in cold temperatures as (c) The wave function of fermions is symmetric
compared to hot temperatures. while that of boson is antisymmetric.
Select the correct answer using the code given below: (d) Fermion particles have half integer spin while
(a) 1 and 2 only boson particles have integer spin.
(b) 2 only 94. The roads are bent inwards on curves. Which of
(c) 2 and 3 only the following is the correct reason for this bending
(d) 1 and 3 only of the roads?
90. A superconductor is a material that achieves (a) To provide the required centrifugal force.
superconductivity. Which among the following (b) To provide the necessary centripetal force.
best describes the term ‘superconductivity’? (c) To counter Newton’s First Law of Motion
(a) It is a state of matter that has very low electrical (d) To counter the effect of gravitational force.
resistance and allows magnetic fields to
penetrate. 1.3. General Chemistry
(b) It is a state of matter that has high electrical 95. In recent times, hydrogen has been increasingly
resistance and does not allow magnetic fields to mooted as an alternate source of energy, what is/
penetrate. are the advantage/advantages of using hydrogen
(c) It is a state of matter that has high electrical as fuel?
resistance and allows magnetic fields to
1. It can be generated from renewable energy
penetrate.
resources.
(d) It is a state of matter that has no electrical
resistance and does not allow magnetic fields to 2. As a fuel, it is more efficient than petrol.
penetrate. 3. Use of hydrogen can lead to zero vehicular
emissions.
91. Which of the following is the reason that a gun
Select the correct answer using the code given below:
recoils when a bullet is fired?
(a) 1 and 2 only
(a) The force of the bullet is more than the force of
(b) 1 and 3 only
the gun
(c) 2 and 3 only
(b) Total final momentum of the system increases to
(d) 1, 2 and 3
more than zero.
(c) The force exerted by a gun is more than force 96. A material called Cadmium (Cd) doped Silver
exerted by the bullet. Antimony Telluride (AgSbTe2) was recently seen
(d) For Conservation of linear momentum. in news. What is the use of this material?

Workbook 10
.
SCIENCE & TECHNOLOGY

1. It can efficiently convert waste heat into 3. Assaying Centre’s identification number
electricity, which can power small home 4. Jeweller’s identification mark.
equipment. Select the correct answer using the code given below:
2. It is a lead-free thermoelectric material which (a) 1, 2 and 3 only
has the potential to be used for mass-market (b) 2, 3 and 4 only
applications. (c) 1, 3 and 4 only
Select the correct answer using the code given below: (d) 1, 2, 3 and 4
(a) 1 only 101. Recently, some researchers have discovered a gas
(b) 2 only called phosphine in Venus’ atmosphere. Regarding
(c) Both 1 and 2 this gas, consider the following statements:
(d) Neither 1 nor 2 1. It is considered as a biomarker of life.
97. ‘Gas hydrates’ are considered as a potentially 2. It cannot be produced through industrial
valuable energy source. However, there are some processes.
concerns regarding their use. Why? Which of the statements given above is/are correct?
1. Decomposition of gas hydrates could aid to (a) 1 only
global warming. (b) 2 only
2. Most gas hydrates are formed from methane, the (c) Both 1 and 2
release of which can cause submarine landslides. (d) Neither 1 nor 2
3. So far, no gas hydrate deposits have been found 102. In the context of transmission of viruses, how
in India. are the aerosols different from the respiratory
Which of the statements given above is/are correct? droplets?
(a) 2 and 3 only 1. Aerosols are smaller in size than the respiratory
(b) 1 and 2 only droplets.
(c) 1 and 3 only 2. Aerosols remain suspended in air for longer
(d) 1, 2 and 3 period than the respiratory droplets.
98. In the context of Vanadium, consider the following Select the correct answer using the code given below:
statements: (a) 1 only
1. Vanadium alloys have an application in nuclear (b) 2 only
reactors. (c) Both 1 and 2
2. Vanadium is used in the manufacturing of steel (d) Neither 1 nor 2
alloys. 103. Which of the following has/have a bearing on the
3. India is the largest producer and exporter of safeguards against chemical disasters in India?
vanadium in the world. 1. The Environment (Protection) Act, 1986
Which of the statements given above is/are correct? 2. The Public Liability Insurance Act, 1991
(a) 1 and 3 only 3. The National Environment Appellate Authority
(b) 1 and 2 only Act, 1997
(c) 2 and 3 only Select the correct answer using the code given below:
(d) 1, 2 and 3
(a) 2 and 3 only
99. Which of the following statements is/are correct (b) 1 only
regarding styrene? (c) 1 and 3 only
1. Inhaling styrene does not cause any health (d) 1, 2 and 3
complications.
104. With reference to Organisation for the Prohibition
2. It is a raw material used in the synthesis of of Chemical Weapons (OPCW), which of the
polystyrene. following statements is/are correct?
Select the correct answer from the codes given 1. It is an informal group formed under United
below: Nations framework in 1992.
(a) 1 only 2. It provides assistance and protection to state
(b) 2 only Parties against chemical threats.
(c) Both 1 and 2 3. It can perform testing of sites and victims of
(d) Neither 1 nor 2 suspected chemical weapons attacks.
100. The Hallmarked Gold Jewellery contains which of 4. OPCW reports to the UN through the office of
the following identification mark? the Secretary-General.
1. BIS mark. Select the correct answer using the codes given
2. Purity in carat. below:

11 Workbook
.
SCIENCE & TECHNOLOGY

(a) 1, 2 and 3 only 2. It is a known cancer-causing agent.


(b) 1, 3 and 4 only 3. It is used to increase shelf life of fresh foods.
(c) 2, 3 and 4 only
(d) 1, 2, 3 and 4 Select the correct answer using the code given below:
(a) 1 and 2 only
105. They are different structural forms of the same (b) 2 and 3 only
element and can exhibit quite different physical (c) 1 and 3 only
properties and chemical behaviours. The change (d) 1, 2 and 3
between their different forms is triggered by
the same forces that affect other structures, i.e., 110. Which of the following substances turn blue
pressure, light, and temperature. The concept litmus paper to red?
of was proposed by Swedish scientist Jons Jakob 1. rain water
Berzelius in 1841. 2. pure water
What is being referred to in the above passage? 3. milk of magnesia
(a) Isotopes
Select the correct answer using the code given below:
(b) Allotropes
(c) Isobars (a) 1 only
(d) Isotones (b) 1 and 2 only
(c) 2 and 3 only
106. In the context of solar cells, consider the following (d) 1, 2 and 3
statements:
1. A solar cell is semi-conducting device made up 111. Which of the following chemical compounds are
of silicon. used in cloud seeding?
2. Solar Photovoltaic cells produce direct electricity 1. Dry ice
from solar radiation. 2. Silver Iodide
3. Operation and maintenance cost of these cells 3. Potassium Iodide
is very high which makes them unattractive for
Select the correct answer using the code given below:
residential purposes.
(a) 1 and 2 only
Which of the statements given above is/are correct?
(b) 2 and 3 only
(a) 1 and 3 only (c) 1 and 3 only
(b) 2 and 3 only (d) 1, 2 and 3
(c) 1 and 2 only
(d) 1, 2 and 3 112. Which one of the following is the best explanation
107. Which of the following statements best describes of the process of deposition in chemistry?
“Elastocaloric effect”? (a) It is the process of conversion of solid substances
(a) Effect of increase of caloric intake on human directly to gas without passing through liquid
health. state.
(b) Effect of decrease of caloric intake on human (b) It is the process of conversion of a gas directly
health. into a solid without passing through the liquid
(c) Cooling effect produced by twisting and state.
untwisting of rubber bands. (c) It is the process by which a solid forms, where
(d) Heating effect produced by twisting and the atoms or molecules are highly organized into
untwisting of rubber bands. a structure known as a crystal.
(d) occurs when evapotranspiration exceeds
108. Syngas, which can be produced through Coal
Gasification, is a mixture consisting primarily of: precipitation causing the upward movement of
dissolved alkaline salts from the groundwater.
1. Carbon monoxide
2. Ammonium 113. Which of the following statements is/are correct
3. Carbon dioxide regarding to the bleaching powder:
4. Hydrogen 1. Bleaching powder is produced by the action of
Which of the statements given above is/are correct? chlorine on dry slaked lime [Ca(OH)2 ].
(a) 2 and 4 only 2. It is represented as CaOCl2
(b) 1, 2 and 3 only 3. It is used for disinfecting drinking water to make
(c) 1, 3 and 4 only it free of germs.
(d) 1, 2, 3 and 4 Which of the above statements are correct?
109. Which of the following statements is/are correct (a) 1 and 2 only
regarding to the Formalin? (b) 1 and 3 only
1. Formalin is a chemical derived from (c) 2 and 3 only
formaldehyde. (d) 1, 2 and 3

Workbook 12
.
SCIENCE & TECHNOLOGY

114. With reference to thorium, consider the following 118. Graphene is said to be a disruptive technology;
statements: one that could open up new markets and even
1. India has around 25 percentage of the world’s replace existing technologies or materials. Which
thorium reserves. of the following can be the potential applications
of Graphene?
2. Thorium is itself a fissile material which split and
release energy. 1. As carriers of drugs and antigens in the human
body.
Which of the statements given above is/are correct? 2. Can be used to enhance the strength of other
(a) 1 only materials.
(b) 2 only 3. Making heat-spreading solutions, such as heat
(c) Both 1 and 2 sinks or heat dissipation films.
(d) Neither 1 nor 2 4. In batteries and supercapacitors to store more
115. With reference to Arsenic, consider the following energy and charge faster.
statements: 5. In anti-corrosion coatings and paints.
1. Inorganic arsenic is harmful which is mainly Select the correct answer using the code given below:
sourced from groundwater. (a) 1, 2 ,3 and 5 only
2. Organic arsenic found in seafood is less harmful (b) 2, 3, 4 and 5 only
to health. (c) 1, 2, 3 and 4 only
3. Arsenic is colourless but has pungent odour and (d) 1, 2, 3, 4 and 5
salty taste. 119. Which of the following statements is incorrect
4. Muktoshri is a newly developed rice variety that about properties of atoms?
is resistant to arsenic. (a) Most of the atoms do not exist independently.
Select the correct answer using the code given below: (b) A group of atoms carrying a charge is known as
(a) 1, 2 and 3 only a polyatomic ion.
(b) 1, 2 and 4 only (c) Atoms are destructible and loses their identity in
chemical reactions.
(c) 1, 3 and 4 only
(d) Atoms which has imbalanced outer shells are
(d) 2, 3 and 4 only
highly reactive.
116. Which of the following statements correctly
120. Consider the following statements regarding ions:
explains the effect of altitude on boiling point of
water? 1. They are always positively charged.
(a) As altitude increases, the boiling point of water 2. They can be formed by breaking up of covalent
increases. bond between two atoms.
(b) As altitude increases, the boiling point of water 3. They are important in preventing blood cells
decreases. from bursting.
(c) There is no change in boiling point of water with Which of the statements given above is/are correct?
respect to change in altitude. (a) 1 only
(d) The boiling point of water decreases with (b) 2 and 3 only
increase in altitude due to increase in pressure. (c) 2 only
117. Consider the following pairs (d) 3 only

Polymeric Function 121. Which of the following statements is incorrect in


carbohydrate regard to oxytocin?
(a) It plays an important role in the reduction of
1. Starch Main storage carbohydrate maternal deaths.
source in plants
(b) Though Oxytocin is a natural hormone, it can
2. Glycogen Important structural component also be chemically manufactured.
of cell wall in plants. (c) It finds application in the dairy industry.
(d) It can be manufactured by both the public and
3. Cellulose Energy storage polymer in private sector industries for domestic use.
animal cells.
Which of the pairs given above is/are correctly 122. Nylon is a synthetic polymer fibre. Why there is/
matched? are concern/concerns over the use of nylon?
(a) 1 only 1. Manufacturing and processing of nylon is
(b) 2 only energy-intensive.
(c) 1 and 2 only 2. Its production process aids to the depletion of
(d) 1, 2 and 3 the ozone layer.

13 Workbook
.
SCIENCE & TECHNOLOGY

3. Improper disposal of nylon products leads Which of the above statements is/are correct?
to accumulation of microplastic in aquatic (a) 1 only
ecosystem. (b) 1 and 2 only
Select the correct answer using the code given below: (c) 3 only
(a) 1 and 2 only (d) 2 and 3 only
(b) 2 only 128. When an ant bites, it injects formic acid into
(c) 1 and 3 only the skin causing pain. The effect of acid can be
(d) 1, 2 and 3 neutralized by rubbing which of the following
123. Take two identical tin cans. Paint the outer surface solutions on the affected area?
of one black and of the other white. Pour equal (a) Solution of Sodium Hydrogen Carbonate
amounts of water in each and leave them in the (b) Solution of Hydrogen Peroxide
mid-day sun for about an hour. Which of the (c) Solution of Tranquilizer
following is the correct observation regarding (d) Solution of Chloramphenicol
these two cans after one hour?
129. Consider the following statements with reference
(a) The water in the black-coloured tin can will be to the Heavy water:
warmer than that in the white-coloured tin can.
1. Heavy water can be used as a moderator in
(b) The water in both the tin cans will be coolers
pressurized heavy water reactors.
because of the evaporation of water.
(c) The level of water in the black-coloured tin can 2. It helps to enhance the thermal stability of
will be higher than that in the white-coloured tin certain type of vaccines
can. 3. Heavy water is relatively less viscous than H2O
(d) The level and temperature of the water in both and has lower melting and boiling points.
the cans will not change as tin is a bad conductor Which of the statements given above is/are correct?
of heat. (a) 1 and 2 only
124. There are many high-tech devices that are used to (b) 2 and 3 only
measure pH in laboratories. The pH scale measures (c) 1 and 3 only
how acidic an object is. Which of the following is (d) 1, 2 and 3
an easy way to measure pH in our daily lives? 130. With reference to Acids and Bases, consider the
(a) Using Blotting paper. following statements:
(b) Using Filter paper. 1. Acids are sour whereas bases taste bitter.
(c) Using Litmus paper.
(d) Using Sand paper. 2. Acids change blue litmus to red whereas Bases
change red litmus to blue.
125. Which of the following is the correct reason for Which of the statements above is/are incorrect?
the use of aluminium to wrap the food items?
(a) 1 only
(a) It is the only metal that exists in solid state at (b) 2 only
room temperature. (c) Both 1 and 2
(b) It is a non-ductile metal. (d) Neither 1 nor 2
(c) It is brittle material which can be pounded into
sheets. 131. Consider the following statements:
(d) It is a malleable metal 1. Galvanization is a process of depositing a layer
of zinc on iron.
126. When a piece of wood is burnt, it releases a
combination of oxygen, smoke, ashes, water vapor 2. Crystallisation is a process of solidification
and other gases. Burning of wood is an example of: of chemical solution into a solid organised
structure.
(a) Archimedes principle.
(b) Law of Conservation of Momentum. 3. Galvanisation is a physical change whereas
(c) Law of Conservation of Mass. Crystallisation is a chemical change.
(d) Bernoulli’s principle Which of the above statements is/are correct?
127. Consider the following statements with reference (a) 1 and 2 only
to isotopes and allotropes of an element: (b) 2 and 3 only
(c) 1 and 3 only
1. Unlike Allotropes, Isotopes of an element have
(d) 1, 2 and 3
different atomic mass.
2. Isotopes of an element have similar chemical 132. Consider the following fuels and arrange in
properties but their physical properties are decreasing order of their calorific value?
different. 1. Kerosene
3. Diamond and graphite are allotropes of Carbon. 2. Hydrogen

Workbook 14
.
SCIENCE & TECHNOLOGY

3. Liquified Petroleum Gas Which of the statements above is/are incorrect?


4. Compressed Natural Gas (a) 1 and 3 only
Select the correct answer using the code given below: (b) 2 only
(a) 2-1-3-4 (c) 2 and 3 only
(b) 4-3-2-1 (d) 3 only
(c) 2-3-4-1 137. With reference to reaction of metals with different
(d) 3-4-2-1 medium, which of following statements is/are
correct?
133. Consider the following statements with reference
to Pasteurization: 1. Metals on reaction with oxygen produces Metal
Oxides.
1. It is a process in which fat globules in milk are
broken down to prevent the formation of creamy 2. All Metals react with water to produce hydrogen.
layers. Select the correct answer using the code given below:
2. Pasteurization involves heating milk usually (a) 1 only
below its boiling point. (b) 2 only
3. Pasteurization kills organisms responsible for (c) Both 1 and 2
typhoid fever, tuberculosis and diphtheria. (d) Neither 1 nor 2
Which of the statements given above is/are correct? 138. Sodium and Potassium are not kept in open air but
(a) 1 and 2 only immersed in kerosene oil. Which of the following
(b) 1 only could be the possible reason for this?
(c) 2 and 3 only (a) It reacts vigorously in open air due to high
(d) 3 only reactivity.
(b) It may lose its shine when kept open.
134. Consider the following:
(c) A thin layer of oxide gets deposited due to
1. Fuel Cell Production oxidation.
2. Aluminum Ore Processing (d) It gets rust when kept in open for long time.
3. Petroleum Refining
139. Why Nitrogen gas is used in the tyres of vehicles
4. Manufacturing of Glass and airplanes?
Which of the above is/are the uses/benefits of (a) It rapidly decreases the inflation pressure of the
Caustic Soda? tires.
(a) 1, 2 and 3 only (b) It has a high volumetric energy density.
(b) 3 and 4 only (c) It has less water and oxygen content.
(c) 1, 2 and 4 only (d) It has small air molecules with high binding
(d) 1, 2, 3 and 4 capacity
135. With reference to Nylon and Polyester, consider 140. Which of the following processes is/are the result
the following statements: of Oxidation process?
1. Nylon is a polymer with ‘amide’ functional group 1. Rancidity of food items
whereas Polyesters are polymers with the ‘ester’ 2. Estimation of alcohol content in breath
functional group. 3. Hydrogenation of vegetable oil
2. Nylon is thermoplastic whereas Polyesters can Select the correct option using the code given below.
be thermoplastic or thermoset. (a) 1 only
3. Unlike Nylon, Polyesters get easily damaged by (b) 1 and 2 only
acids in cold conditions. (c) 2 and 3 only
Which of the following statements is/are correct? (d) 1, 2 and 3
(a) 1 and 2 only 141. Which among the following methods can be used
(b) 2 and 3 only to remove the permanent hardness in water?
(c) 1 and 3 only 1. Treatment with washing soda
(d) 1, 2 and 3 2. Treatment with lime
136. With reference to Dry Ice, consider the following 3. Ion-exchange method
statements: 4. Treatment with synthetic resins
1. It is formed by freezing liquefying carbon 5. Boiling of water
dioxide & then compressing it to a solid ice. Select the correct answer using the code given below:
2. The liquefication of dry ice causes waterlogging (a) 1, 2 and 3 only
in the storage space. (b) 1, 3 and 4 only
3. It helps in prevention of hair loss of cancer (c) 2, 3, 4 and 5 only
patients undergoing chemotherapy treatment. (d) 1, 3, 4 and 5 only

15 Workbook
.
SCIENCE & TECHNOLOGY

142. Which of the following are the applications of 147. There are limitations in the adoption of Single cell
gypsum? proteins (SCP) as protein supplements in human
1. As a coagulant in Food industry. foods on a large scale. What are these limitations?
2. As a traditional medicinal product 1. High concentration of nucleic acid in some
3. As a Heat Sink material. microbial biomass may lead to gastrointestinal
problems or kidney stones.
Select the correct answer using the code given below:
2. Cultivation of microorganisms has a much
(a) 1 and 2 only higher carbon footprint than agricultural food
(b) 2 and 3 only production.
(c) 1 and 3 only
3. Some microorganisms may produce toxic
(d) 1, 2 and 3
substances which can be detrimental to human
143. Which of the following chemical reactions are health.
exothermic in nature? Select the correct answer using the code given below:
1. The decomposition of vegetable matter into (a) 1 and 2 only
compost.
(b) 2 and 3 only
2. Burning of natural gas. (c) 1 and 3 only
3. Decomposition of calcium carbonate (d) 1, 2 and 3
4. Respiration
5. Evaporation of liquid water 148. Consider the following statements:
1. Cord blood is the blood found in the nervous
Select the correct answer using the code given below:
tissues of the spinal cord.
(a) 1, 2, 3 and 4 only
(b) 1, 2 and 4 only 2. Cord blood can be collected to extract
(c) 2, 3 and 4 only hematopoietic stem cells that can be used to treat
(d) 1, 2 and 5 only certain diseases.
Which of the statements given above is/are correct?
144. Which of the following statements is correct about
aqua regia? (a) 1 only
(b) 2 only
(a) It is a mixture of concentrated sulphuric acid and
concentrated Hydrobromic acid. (c) Both 1 and 2
(b) It can dissolve the noble metals including gold. (d) Neither 1 nor 2
(c) It is generally stored in cast iron containers 149. In the context of human genetics, how is the RNA
(d) It is used in the manufacturing of car batteries. different from the DNA?
1. Unlike DNA, RNA is not capable of undergoing
1.4. General Biology mutations.
145. Many different strains of Influenza A virus have 2. Chemically, DNA is less reactive and structurally
been detected till now. What is/are the likely more stable when compared to RNA.
reason/reasons behind the presence of so many Select the correct answer using the code given below:
strains of this virus? (a) 1 only
1. Influenza viruses can swap genetic material (b) 2 only
through reassortment. (c) Both 1 and 2
2. It is a DNA virus with an unsegmented genome. (d) Neither 1 nor 2
3. It is susceptible to constant mutations.
150. ‘G4’ virus was recently mentioned in the news with
Select the correct answer using the code given below:
reference to which one of the following diseases?
(a) 1 and 3 only
(b) 2 and 3 only (a) Yellow fever
(c) 1 only (b) Japanese encephalitis
(d) 1, 2 and 3 (c) Swine flu
(d) Bird flu
146. Which one of the following statements correctly
describes the meaning of ‘Cytokine storm’? 151. Which of the following statements is/are correct
(a) An abnormal rise in the sea-water level during a regarding the ‘Kala-Azar’:
cyclonic storm. 1. It is a neglected tropical disease (NTD).
(b) Overproduction of certain signaling proteins 2. In India, it is mostly prevalent in the Western
which can cause harm to the body. and Southern regions.
(c) A swarm of locusts which leads to the largescale
destruction of crops. 3. It is transmitted to humans by the bite of infected
(d) Sudden rise in the prices of essential female sand-flies.
commodities. Select the correct answer using the code given below:

Workbook 16
.
SCIENCE & TECHNOLOGY

(a) 1 and 3 only blood cells are stored there. It also helps fight
(b) 2 only certain kinds of bacteria that cause pneumonia
(c) 3 only and meningitis. It varies in size and shape between
(d) 1, 2 and 3 people, but it’s commonly fist-shaped.
152. Which of the following microorganisms is Which is the organ referred to in the above passage?
considered as non-living? (a) Liver
(a) Bacteria (b) Heart
(b) Virus (c) Spleen
(c) Fungi (d) Pancreas
(d) Algae 157. Consider the following statements:
153. Which of the following is/are found in animal 1. Xylem is the complex tissue of plants, responsible
cells? for transporting water and other nutrients to the
1. Cell Wall plants.
2. Mitochondria 2. Phloem is living tissue in plants, responsible for
3. Ribosomes transporting food and other organic materials.
Select the correct answer using the code given below: 3. Xylem and phloem are also found in some
(a) 1 and 2 only animal species.
(b) 1 and 3 only Which of the above statements are correct?
(c) 2 and 3 only
(a) 1 and 2 only
(d) 1, 2 and 3
(b) 2 and 3 only
154. With reference to DNA, consider the following (c) 1 and 3 only
statements? (d) 1, 2 and 3
1. DNA stands for Deoxyribonucleic Acid, a
hereditary material found in all living things. 158. Which of the following statements is/are correct
regarding to the human brain?
2. Most DNA is located in the cell nucleus (called
nuclear DNA) but some small amount of 1. The human brain is the largest brain of all
DNA can be found in Mitochondria (called vertebrates relative to body size.
mitochondrial DNA) 2. Visual processing takes place in the occipital
3. Viruses do not have DNA but only RNA. lobe, near the back of the skull.
3. The temporal lobe processes sound and language.
Which of the above statements are correct?
(a) 1 and 2 only Which of the above statements are correct?
(b) 1 and 3 only (a) 1 and 2 only
(c) 2 and 3 only (b) 2 and 3 only
(d) 1, 2 and 3 (c) 1 and 3 only
(d) 1, 2 and 3
155. Which of the following statements is/are correct
regarding to the Vitamin A? 159. With reference to the human eye, consider the
following statements:
1. Vitamin A is a fat soluble vitamin found mainly
in dairy food sources. 1. The outermost layer, called the sclera, is what
gives most of the eyeball its white color.
2. An early symptom of vitamin A deficiency is
night blindness, which is caused by a disorder of 2. During eye transplant, only the iris and retina
the retina. are the parts that are transplanted.
3. Vitamin A deficiency also compromises the 3. Iris is a dark muscular diaphragm that controls
immune system and hence increases the the size of the pupil.
vulnerability to covid disease. 4. The cornea’s main function in the human eye is
to refract, or bend, light.
Select the correct answer using the code given below:
(a) 1 and 2 only Which of the above statements are correct?
(b) 2 and 3 only (a) 1, 2 and 4 only
(c) 1 and 3 only (b) 1, 3 and 4 only
(d) 1, 2 and 3 (c) 1, 2 and 3 only
(d) 1, 2, 3 and 4
156. This organ of the human body plays multiple
supporting roles in the body. It acts as a filter for 160. Consider the following statements:
blood as part of the immune system. Old red blood 1. In a human cell, Deoxyribonucleic acid (DNA) is
cells are recycled in it, and platelets and white found only inside the nucleus.

17 Workbook
.
SCIENCE & TECHNOLOGY

2. Mitochondrial diseases are caused by faulty (b) They are malicious software that targets the host
genes on X-chromosome in the human cells. device by corrupting the hard disk.
3. Mitochondrial DNA in humans is inherited only (c) They are a group of plant metabolites that
from Mothers. provide health benefits to human body.
(d) They are a type of cyanobacteria which can
Which of the statements given above is/are correct? survive in extreme conditions.
(a) 1 only
(b) 1 and 2 only 166. Candida Auris is a multidrug-resistant pathogen
(c) 3 only which poses a serious global threat to human
(d) 2 and 3 only health. Recently, in which part of India has it been
identified for the first time in the environment?
161. Consider the following pairs: (a) Western Himalayas
(b) Hills of Southern India
Diseases Causes (c) Andaman and Nicobar Islands
1. Kala-azar Fungi (d) Tropical rain forests of Northeast
2. Ringworms Protozoa 167. Recently, the state of Odisha reported its first-
3. Typhoid Bacteria ever case of ‘Harlequin ichthyosis’. Consider the
following statements regarding the disease:
Which of the pairs given above is/are correctly 1. It is transmitted from person-to-person by the
matched? same mosquito that transmits dengue.
(a) 1 and 2 only 2. It primarily affects the development of skin in
(b) 2 and 3 only humans.
(c) 3 only
Which of the statements given above is/are correct?
(d) 1, 2 and 3
(a) 1 only
162. Which of the following set of diseases are all (b) 2 only
caused by viruses? (c) Both 1 and 2
(a) Common cold, influenza, typhoid, Cholera (d) Neither 1 nor 2
(b) Influenza, AIDS, kala-azar, chicken pox 168. Which of the following statements correctly
(c) Kala-azar, tuberculosis, anthrax, small pox defines the term C-peptide?
(d) Influenza, dengue, chikungunya, polio
(a) Human blood samples that contain antibodies
163. Which of the following diseases/infections are against hepatitis B virus.
examples of zoonotic infection? (b) A substance that is created when the hormone
1. West Nile virus insulin is produced and released into the body.
(c) Monoclonal antibody used for the treatment of
2. Brucellosis
mild to moderate COVID-19.
3. Middle East Respiratory Syndrome (d) Material used in the modification of a gene
4. Nipah virus product.
5. Blastomycosis
169. With reference to different kinds of fats, consider
Select the correct answer using the code given below: the following statements:
(a) 1, 2, 3 and 4 only 1. Unsaturated fats are beneficial fats for body
(b) 1, 3 and 4 only which is found mostly in plants.
(c) 2, 4 and 5 only 2. Most animal foods contain saturated fat.
(d) 1, 2, 3, 4 and 5
3. Trans-fat are a form of saturated fat which leads
164. Which one of the following statements is incorrect, to increased risk of heart attacks.
with reference to disease causing microorganisms? Which of the statements given above is/are correct?
(a) Infection is the invasion and multiplication of (a) 1 and 2 only
pathogenic microbes in an individual. (b) 2 and 3 only
(b) Both protozoa and worms are treated as (c) 3 only
parasites. (d) 1, 2 and 3
(c) A virus cannot replicate outside the host cell.
170. With reference to the beginning of life on earth,
(d) For a long-term survival, a pathogen must
consider the following statements:
rapidly kill every host it infects.
1. Prokaryotes were the earliest life forms on earth.
165. Which one of the following statements correctly 2. Early volcanic eruptions was responsible for the
describes the ‘Flavonoids’? creation of Earth’s second atmosphere.
(a) They are a group of pulsars that produces 3. Blue-green algae reduced the carbon dioxide
gamma-ray emissions. and increased oxygen content in the atmosphere.

Workbook 18
.
SCIENCE & TECHNOLOGY

Which of the statements given above is/are correct? (a) 1 only


(a) 1 only (b) 2 only
(b) 1 and 2 only (c) Both 1 and 2
(c) 2 and 3 only (d) Neither 1 nor 2
(d) 1, 2 and 3 176. With reference to Chlorophyll, which one of the
171. Consider the following statements with reference following is not correct?
to avian influenza: (a) Chlorophyll is present in all the algae.
1. H1N1 is the most common strain of avian (b) The leaves other than green also contain
influenza. chlorophyll.
2. The avian influenza virus has very high human (c) Carbon fertilization can promote plant
to human transmission. chlorophyll synthesis.
(d) Chlorophyll is located in cytoplasm within plant
Which of the statements given above is/are correct? cells.
(a) 1 only
(b) 2 only 177. Consider the following statements:
(c) Both 1 and 2 1. Ingestion is the breakdown of complex
(d) Neither 1 nor 2 components of food into simpler substances.
2. Egestion is the process of taking food into the
172. Which of the following statements is incorrect body.
with regard to stem cells?
3. Digestion is the removal of faecal matter from
(a) Embryonic stem cells can form any cell types in the body.
the body.
(b) Bone marrow contains stem cells that can give Which of the statements given above is/are incorrect?
rise to red blood cells, white blood cells and (a) 2 only
platelets. (b) 1 and 2 only
(c) Somatic stem cells remain in our body (c) 1 and 3 only
throughout our lifetime. (d) 1, 2 and 3
(d) Induced Pluripotent stem cells are produced by 178. Consider the following statements about African
genetically manipulating embryonic stem cells Swine flu (ASF):
to produce somatic stem cells.
1. It is a severe viral disease that affects wild and
173. Which of the following statement about cell domestic pigs.
potency is correct? 2. It is a type of zoonotic disease and lethal for
(a) Pluripotency is restricted only to the early cells Humans.
of fertilized eggs. 3. There has been no incidence of outbreak in
(b) Totipotent cells formed during sexual and India.
asexual reproduction are spores and zygotes Which of the statements given above is/are correct?
respectively. (a) 1 only.
(c) A totipotent has potential to be differentiated (b) 1 and 2 only
into any type of cell. (c) 2 only
(d) Pluripotent cell can develop into a placenta. (d) 1, 2 and 3
174. Which of the following statement is incorrect, with 179. Which of the following are type of White Blood
respect to disease causing microorganisms? Cells (WBC)?
(a) Infection is the invasion and multiplication of 1. Neutrophils
pathogenic microbes in an individual.
2. Lymphocytes
(b) Both protozoa and worms are treated as
parasites. 3. Erythrocytes
(c) A virus cannot replicate outside the host cell. 4. Basophils
(d) For a long-term survival a pathogen must rapidly Select the correct answer using the code given below:
kill every host it infects. (a) 1, 2 and 3 only
175. Consider the following statements regarding the (b) 1, 2 and 4 only
difference between Phenotype and Genotype of an (c) 1, 3 and 4 only
organism: (d) 2, 3 and 4 only
1. Phenotype is the physical appearance while 180. With reference to antimicrobial resistance,
Genotype is genetic composition of an organism. consider the following statements:
2. Phenotype are partly inherited while Genotype 1. Lack of clean water and sanitation leads to
is not inherited during the reproduction process. antimicrobial resistance.
Which of the following statements given above is/are 2. Due to antimicrobial resistance, our ability to
correct? treat common infections decreases.

19 Workbook
.
SCIENCE & TECHNOLOGY

3. It affects old age people while young ones are Select the correct answer using the code given below.
generally immune. (a) 1 and 2 only
Which of the statements given above are correct? (b) 2, 3 and 4 only
(a) 1 and 2 only (c) 1, 3 and 4 only
(d) 1, 2, 3 and 4
(b) 2 and 3 only
(c) 1 and 3 only 186. Consider the following statements:
(d) 1, 2 and 3 1. Prokaryotes do not have a nuclear membrane.
181. Which of the following is correct regarding the 2. Eukaryotes include only multicellular organisms
helminth infection, which was in news recently? Which of the statements given above is/are correct?
(a) It is an air borne disease caused by sneezing. (a) 1 only
(b) India currently has no active cases of helminth (b) 2 only
infection. (c) Both 1 and 2
(c) Infected children are nutritionally and physically (d) Neither 1 nor 2
impaired. 187. Which of the following diseases are caused by
(d) Currently there are no effective medicines to viruses?
control the disease. 1. Hepatitis C
182. Consider the following statement: 2. Chicken Pox
1. Heart is the largest gland in the human body. 3. Common Cold
2. Gall bladder is responsible for secretion of bile in 4. Malaria
the human body. Select the correct answer using the code given below.
3. The pancreas is a both exocrine and endocrine (a) 1, 2 and 3 only
organ of the human body. (b) 2 and 4 only
4. The liver is the only organ in the human body (c) 1, 3 and 4 only
that can regenerate itself. (d) 2, 3 and 4 only
Select the correct answer using the code given below: 188. Which of the following are the beneficial uses of
(a) 1 and 2 only bacteria?
(b) 1 and 3 only 1. Sewage treatment
(c) 3 and 4 only 2. Production of medicines
(d) 2, 3 and 4 only 3. Increasing soil fertility
183. Consider the following properties of the Virus: 4. Vaccine development
1. It is metabolically inactive outside the host cell. Select the correct answer using the code given below.
2. They are known to infect all kinds of cells except (a) 1 and 3 only
bacteria. (b) 2, 3 and 4 only
(c) 1, 2 and 3 only
3. All Coronaviruses are RNA virus. (d) 1, 2, 3 and 4
Which of the statements given above is/are correct?
189. Consider the following statements:
(a) 1 and 3 only
(b) 2 and 3 only 1. The changes that occur at adolescence are
(c) 2 only controlled by hormones.
(d) 3 only 2. All the hormones in our body are secreted by the
pituitary gland.
184. It is also called tinea pedis or ringworm of the foot Which of the statements given above is/are correct?
and is a contagious fungal infection that affects (a) 1 only
the skin on the feet. It requires a warm and moist (b) 2 only
environment and can also spread to the toenails (c) Both 1 and 2
and the hands. The disease mentioned here is: (d) Neither 1 nor 2
(a) Mycetoma
(b) Athlete’s foot 190. Consider the following pairs of organisms and
(c) Elephant foot their mode of nutrition:
(d) Blastomycosis
Organism Mode of nutrition
185. Which of the following is/are the negative 1. Mushroom Saprotrophic
consequences of noise pollution on human health?
1. Hypertension 2. Insectivorous plants Parasitic
2. Hearing disability 3. Algae Autotrophic
3. Sleeping disorders Which of the pairs given above is/are correctly
4. Anxiety matched?

Workbook 20
.
SCIENCE & TECHNOLOGY

(a) 1 and 3 only Which of the statements given above is/are correct?
(b) 1 and 2 only (a) 1 only
(c) 3 only (b) 2 only
(d) 1, 2 and 3 (c) Both 1 and 2
191. With reference to ‘Bile’, consider the following (d) Neither 1 nor 2
statements: 196. Consider the following statements:
1. It plays an important role in digestion of fats. 1. 22 out of 23 pairs chromosomes in the humans
2. It is secreted by the Gallbladder. are exactly same between males and females.
3. It makes the acidic food coming from the 2. XY pair of chromosomes is found in females and
stomach alkaline. XX pair of chromosomes is found in males.
Which of the statements given above is/are Which of the statements given above is/are correct?
correct? (a) 1 only
(a) 1 only (b) 2 only
(b) 2 and 3 only (c) Both 1 and 2
(c) 1, 2 and 3 (d) Neither 1 nor 2
(d) 1 and 3 only
197. Digestive juices in our body are secreted by
192. Consider the following statements: 1. Small intestine
1. Methanogens are bacteria that produce methane 2. Diaphragm
by growing aerobically on cellulosic material. 3. Liver
2. Biogas is a mixture of gases predominantly 4. Pancreas
containing methane.
Select the correct answer using the code given below.
3. Methanogens play an important role in the
(a) 1 and 2 only
nutrition of cattle.
(b) 2, 3 and 4 only
Which of the statements given above is/are correct? (c) 1, 3 and 4 only
(a) 1 and 2 only (d) 1, 2, 3 and 4
(b) 1 and 3 only
(c) 2 and 3 only 198. Which of the following conditions is/ are
(d) 1, 2 and 3 compulsory for the Germination of a seed?
1. Presence of light
193. Consider the following statements: 2. Correct range of temperature
1. Fungi lives in symbiosis with the plant and 3. Presence of moisture
passes phosphorus from the soil to the plant.
4. Availability of oxygen
2. Azotobacter and Azospirillum are free-living
bacteria that fix atmospheric nitrogen in the soil. Select the correct option using the code given below:
(a) 1, 2 and 3 only
Which of the statements given above is/are correct?
(b) 2, 3 and 4 only
(a) 1 only (c) 1, 3 and 4 only
(b) 2 only (d) 1, 2, 3 and 4
(c) Both 1 and 2
(d) Neither 1 nor 2 199. With respect to the characteristics of Prey and
Predator, consider the following statements:
194. Arrange the following organisms chronologically
1. Features like long ears and eyes on the side are
in terms of their evolutionary periods:
found in animals of prey.
1. Fishes
2. Features like camouflage are exclusively found in
2. Birds animals of prey.
3. Corals 3. Predators generally have blunt claws and hoofs
4. Flowering plants that enable them to run fast.
Select the correct answer using the code given below. Which of the statements given above is/ are correct?
(a) 3-1-2-4 (a) 1 only
(b) 3-1-4-2 (b) 2 and 3 only
(c) 1-3-2-4 (c) 1 and 3 only
(d) 1-3-4-2 (d) 1, 2 and 3
195. Consider the following statements: 200. With reference to the common characteristics
1. Chromosomal disorders are caused by absence of Living Organisms, consider the following
of one or more chromosomes. statements:
2. Down’s syndrome and Turner’s syndrome are 1. Not all living organisms need to obtain food for
examples of chromosomal disorders. energy.

21 Workbook
.
SCIENCE & TECHNOLOGY

2. All living organisms respond to external stimuli. Which of the following acid is responsible for
3. All living organisms grow throughout their life. creating the effect being described in the lines above?
Which of the statements given above is/ are correct? (a) Acetic Acid
(a) 1 and 2 only (b) Hydrochloric Acid
(b) 2 and 3 only (c) Amino Acid
(c) 1 and 3 only (d) Lactic Acid
(d) 1, 2 and 3 205. With reference to the circulatory system of the
201. With reference to the excretory system in plants, human body, consider the following pairs:
consider the following statements:
Blood Vessels Function
1. Like humans, plants have a specialised system
for excretion. 1. Arteries Carry oxygen poor
2. Many plant secretions like rubber, gums and blood to the upper half
resins are actually waste products. of the body
Which of the statements given above is/ are correct? 2. Veins Carrying deoxygenated
(a) 1 only blood towards the heart
(b) 2 only 3. Capillaries Connects the arteries
(c) Both 1 and 2 and vessels
(d) Neither 1 nor 2 Which of the pairs given above is/ are correctly
202. With reference to the special features and matched?
variations found among different types of plants, (a) 1 and 2 only
consider the following statements: (b) 2 and 3 only
1. Carotenoids are responsible for the bright red (c) 1 and 3 only
colour on application of henna leaves. (d) 1, 2 and 3
2. Volatile organic Compounds (VOCs) are 206. With reference to the transport of water and
responsible for the scent in flowers. minerals in a plant, consider the following
Which of the statements given above is/ are correct? statements:
(a) 1 only 1. Xylem transports food whereas Phloem
(b) 2 only transports water in plants.
(c) Both 1 and 2 2. Unlike transport of water, transport of food
(d) Neither 1 nor 2 occurs with the expense of energy.
203. Which of the following organisms practice 3. Xylem is made of dead cells whereas Phloem is
anaerobic respiration? made of living cells.
1. Yeast 4. Flow in Xylem is unidirectional while flow in
2. Earthworms Phloem is bidirectional.
3. Clostridium Select the correct answer using the code given below:
4. Button Mushrooms (a) 1, 2 and 3 only
5. Methanogenic Archaea (b) 1, 2 and 4 only
Select the correct option using the code given below: (c) 1, 3 and 4 only
(a) 1, 2 and 3 only (d) 2, 3 and 4 only
(b) 2, 3 and 4 only 207. Consider the following statements with reference
(c) 1, 3 and 5 only to transpiration in plants:
(d) 1, 2, 3, 4 and 5 1. Water reaches great heights in the trees because
204. “During heavy exercise it is often observed that of transpiration.
people draw heavy breaths. This is because the 2. Plants growing on high altitudes will show
energy being required for the exertion exceeds that higher rate of transpiration.
which is available with the given amount of oxygen. 3. Transpiration does not occur at night due to
In the short span of following time, to compensate non-availability of sunlight.
for the energy deficit the cells of the muscle being
4. Transpiration helps in cooling of plants.
exercised begin to respire anaerobically to produce
extra energy, while also creating an acid as a by- Which of the statements given above is/are correct?
product. Due to the accumulation of this acid, the (a) 1,2 and 4 only
person starts getting extremely fatigued and often (b) 1,2 and 3 only
the muscles cramp occurred, causing them to stop (c) 2,3 and 4 only
exercising.” (d) 1,2,3 and 4

Workbook 22
.
SCIENCE & TECHNOLOGY

208. Consider the following statements with reference Which of the statements given above is/are
to vegetative propagation: incorrect?
1. Vegetative propagation is a type of sexual (a) 1 only
reproduction in which new plants is obtained (b) 3 only
without production of seeds. (c) 2 and 3 only
2. Potato sprouting from an ‘eye’ is a form of (d) 1,2 and 3
vegetative propagation. 213. With reference to the cell biology, consider the
Which of the statements given above is/are correct? following statements:
(a) 1 only 1. Plant cells have a cell wall which is absent in
(b) 2 only animals cells.
(c) Both 1 and 2 2. Both plants and animals cells have chloroplasts.
(d) Neither 1 nor 2 3. Plant cells have smaller vacuoles compared to
209. Which of the following statement is correct about the animal cells.
the process of pollination? Which of the statement given above is/are incorrect?
(a) Seeds can only be produced when pollen is (a) 1 and 3 only
transferred between flowers of the same species. (b) 2 only
(b) It involves transfer of pollen from the stigma to (c) 3 only
the anther of a flower. (d) 2 and 3 only
(c) Self-pollination process needs a vector for its
occurrence. 214. Consider the following statements:
(d) Cross-pollination occurs when the pollen lands 1. In External fertilization, fusion of a male and a
on the flower of a different species. female gamete takes place outside the body of
the female
210. Consider the following statements with reference 2. External fertilization is very common in plants
to pituitary gland: as a method of sexual reproduction.
1. The pituitary gland is an exocrine gland that
secrets the growth hormone. Which of the statements given above is/are correct?
2. The pituitary gland is the largest gland in the (a) 1 only
human body. (b) 2 only
(c) Both 1 and 2
3. The pituitary gland controls the secretion from (d) Neither 1 or 2
the Thyroid and adrenals.
4. The pituitary gland is located in the neck region. 215. Consider the following pairs:
Which of the statements given above is/are correct? Reproductive part Description
(a) 1 and 3 only
(b) 3 only 1. Embryo Early stage of
(c) 2 and 3 only development of
(d) 3 and 4 only organisms taking place
in uterus.
211. Consider the following statements:
2. Zygote Fusion of egg and
1. The fruit is the ripened ovary. sperm cells.
2. All fruits contain the seed.
3. Foetus Stage of the embryo in
3. Plants produced from seeds take less time to bear
which all the body parts
fruits than those produced from by vegetative can be identified.
propagation.
Which of the pairs given above is/are correctly
Which of the statements given above is/are correct?
matched?
(a) 1 only
(a) 1 and 2 only
(b) 3 only
(b) 1 and 3 only
(c) 1 and 2 only
(c) 2 and 3 only
(d) 2 and 3 only
(d) 1,2 and 3
212. Cell is considered as the basic unit of life, in this 216. Consider the following statements with reference
context consider the following statements: to inbreeding:
1. The size of the cells is directly related to the size 1. Inbreeding refers to the mating of animals within
of the body of the animal. the same breed but having no common ancestors
2. All cells of a particular organism are always the on either side for up to 4-6 generations.
same. 2. Inbreeding increases the productivity of inbred
3. Viruses have the smallest cell. population.

23 Workbook
.
SCIENCE & TECHNOLOGY

3. Inbreeding reduces fertility if continued for long Which of the statements given above is/are correct?
time. (a) 1 and 2 only
Which of the statements given above is/are correct? (b) 2 and 3 only
(a) 1 and 2 only (c) 1 and 3 only
(b) 1 and 3 only (d) 1, 2 and 3
(c) 2 and 3 only 221. Some species have diverged after origin from
(d) 1,2 and 3 common ancestor giving rise to new species
217. Consider the following pairs. adapted to new habitats and ways of life. The
species has likely undergone which form of
Animal Breeding Feature evolution?
1. Out-Crossing Animal breeding (a) Adaptive radiation
between same breed (b) Divergent evolution
having common (c) Convergent evolution
ancestors in recent. (d) Mutation
2. Cross-Breeding Animal breeding 222. Consider the following diseases:
between different 1. Typhoid
breed. 2. Parkinson’s disease
3. Interspecific Animal breeding 3. Color Blindness
Hybridization between different Which of the diseases mentioned above can be
species. diagnosed by Widal Test?
Which of the pairs given above is/are correct? (a) 1 and 2 only
(a) 1 and 2 Only (b) 1 only
(b) 2 and 3 Only (c) 2 and 3 only
(c) 1 and 3 Only (d) 2 only
(d) 1, 2 and 3
223. In the context of hereditary diseases, consider the
218. With reference to the nephrons in the excretory following statements:
organ system of the living beings, consider the 1. Passing on mitochondrial diseases from parent
following statements. to child can be prevented by mitochondrial
1. Nephron is involved in removal of waste replacement therapy either before or after in
products from the blood. vitro fertilization of egg.
2. Each Kidney consists of one Nephron unit. 2. A child inherits mitochondrial diseases entirely
3. Nephron is only found in human beings. from mother and not from father.
Which of the statements given above is/are correct? Which of the statements given above is/are correct?
(a) 1 Only (a) 1 only
(b) 2 and 3 Only (b) 2 only
(c) 1 and 3 Only (c) Both 1 and 2
(d) 1, 2 and 3 (d) Neither 1 nor 2
219. This is the molecule which is immediately 224. Which one of the following is the most probable
synthesized using the energy released during the reason for a human being to sneeze?
cellular respiration. It is the source of energy for (a) Rupture of our throat.
use and storage at the cellular level. This refers to (b) Antigens present in the environment called
which of the following?
allergy.
(a) Glucose (c) Exposure of the diffused sunlight.
(b) Ribonucleic Acid (d) Absence of the mucus membrane underneath
(c) Deoxyribonucleic Acid the nasal cavity.
(d) Adenosine Triphosphate
225. Consider the following :
220. With reference to the differences between
prokaryotes and eukaryotes, consider the 1. Hepatitis C
following statements. 2. Tobacco and other related narcotics.
1. Unlike Prokaryotes, nuclear membrane is 3. Genetics
present in eukaryotes. 4. Transformation of normal cells into neoplastic
2. Unlike eukaryotes, prokaryotes consist of single cells
chromosome. Which of the following is/are likely to cause cancer
3. Cell Division in eukaryotes are done through in human beings?
meiosis whereas it is done through binary fission (a) 2 and 3 only
in prokaryotes. (b) 1,3 and 4 only

Workbook 24
.
SCIENCE & TECHNOLOGY

(c) 2 and 4 only (c) White Blood Cells are produced only in the bone
(d) 1,2,3 and 4 marrow.
(d) White blood cells help the immune system to
226. Consider the following statements with reference fight with infection.
to the mitochondria:
1. Mitochondria are the power-generating units of 231. With reference to the reproduction in plant,
the cell. consider the following statements:
2. Mitochondria are found in nearly all eukaryotic 1. If the pollen lands on the stigma of the same
cells. flower of the same plant, it is called self-
3. Each mitochondrion is a single membrane pollination.
bound structure. 2. Flowers which contain both stamens and pistil
are called bisexual flowers.
Which of the statements given above is/are correct?
Which of the statements given above is/are correct?
(a) 1 and 2 only
(b) 2 and 3 only (a) 1 only
(c) 1 and 3 only (b) 2 only
(d) 1, 2 and 3 (c) Both 1 and 2
(d) Neither 1 nor 2
227. Consider the following statements regarding
‘Bryum Bharatiensis’: 232. Which of the following are the examples of
external fertilization?
1. It is a new species of moss found in Antarctica by
Indian Scientist. 1. Starfish
2. The unique feature of this moss is that it does not 2. Cow
require sunlight for their growth. 3. Frog
3. It predominantly grows in areas where penguins 4. Sea Urchins
breed in large numbers. Select the correct answer using the codes given
Which of the statements given above is/are correct? below:
(a) 1 and 2 only (a) 1, 2 and 4 only
(b) 2 and 3 only (b) 2 and 4 only
(c) 1 and 3 only (c) 1, 3 and 4 only
(d) 1, 2 and 3 (d) 1, 2, 3 and 4

228. With reference to Trans Fats, consider the 233. Which of the followings are the functions of the
Cell Membrane?
following statements:
1. It transports nutrients into the cell.
1. These are a type of saturated fat
2. It transports toxic substances out of the cell.
2. Trans-fats are associated with increased risk of
heart attacks 3. It provides interaction of one cell with another
cell.
Which of the statements given above is/ are correct? 4. It provides structural support to the cell.
(a) 1 only
(b) 2 only Select the correct answer using the codes given
(c) Both 1 and 2 below:
(d) Neither 1 nor 2 (a) 1 and 2 only
(b) 2 and 4 only
229. Which of the following vitamins are fat-soluble? (c) 1 and 3 only
1. Vitamin C (d) 1, 2, 3 and 4
2. Vitamin D 234. The plant cells can withstand much greater
3. Vitamin A changes in the surrounding environment than
4. Vitamin K animal cells because:
Select the correct answer using the code given below: (a) plant cells consist of an active element called
(a) 1, 2 and 3 only chlorophyll.
(b) 2, 3 and 4 only (b) plant cells have an extra cell wall.
(c) 3 and 4 only (c) plant cells are primarily made of cellulose.
(d) 1, 2, 3, and 4 (d) plant cells show active osmosis.

230. Which among the following statements is incorrect 235. Which of the following is/are the examples of
in context of the different blood cells in body? Involuntary muscles?
(a) Red Blood Cell carries oxygen to different parts 1. Visceral muscles
of the body. 2. Cardiac muscles
(b) Platelets plays active role in blood clotting 3. Skeletal muscles
during injury. 4. Pectoral muscles

25 Workbook
.
SCIENCE & TECHNOLOGY

Select the correct answer using the codes given 2. Reducing the impact of the oil spills in oceans
below: 3. Decomposition of petroleum products
(a) 1 and 2 only 4. Cutting plastic waste
(b) 2 and 3 only Select the correct answer using the codes given
(c) 3 and 4 only below:
(d) 1 and 4 only
(a) 1, 2 and 3 only
236. With reference to the diversity in living world, (b) 2, 3 and 4 only
consider the following statements: (c) 1, 3 and 4 only
1. Viruses do not show any molecular movement (d) 1, 2, 3 and 4
in them until they infect some cell.
241. Which of the following are metabolic changes that
2. Fungi, are similar to plants as they also are increases the risk of non-communicable diseases?
autotrophs.
1. Abdominal obesity
Which of the statements given above is/are correct? 2. Insulin sensitivity
(a) 1 only 3. Hyperglycemia
(b) 2 only
4. Dyslipidemia
(c) Both 1 and 2
(d) Neither 1 nor 2 Select the correct answer using the code given below:
(a) 1, 2 and 3 only
237. With reference to the human body, Mitochondria (b) 2, 3 and 4 only
are known as the powerhouses of the cell because: (c) 1, 3 and 4 only
(a) they contain ribosomes necessary for the (d) 1, 2, 3 and 4
manufacturing of proteins.
(b) they help in the manufacture of fat molecules or 242. Consider the following statements:
lipids, important for cell function. 1. All cells are microscopic in size.
(c) they play a crucial role in detoxifying many 2. The size of a living organism does not affect the
poisons and drugs. size of its cells.
(d) they are the sites for ATP (Adenosine Which of the statements given above is/are correct?
triphopshate) generating chemical reactions.
(a) 1 only
238. Consider the following difference between plants (b) 2 only
and animals: (c) Both 1 and 2
1. Plants have a large quantity of dead cells while (d) Neither 1 nor 2
most of the tissues in animals are living. 243. Which of the following statements correctly define
2. The growth in plants is limited to certain regions, the relation between plastids and chlorophyll?
while this is not so in animals. (a) Plastids containing the pigment chlorophyll are
Which of the statements given above is/are correct? known as chromoplasts.
(a) 1 only (b) Chloroplasts are green plastids containing the
(b) 2 only pigment chlorophyll.
(c) Both 1 and 2 (c) Chlorophyll pigments in plastids help in storing
(d) Neither 1 nor 2 carbohydrates and proteins.
(d) Chlorophyll pigments in plastids trap light
239. With reference to the Cell cycle and Cell division, energy in leaves required for photosynthesis.
consider the following statements:
1. Meiosis is the process in sexually-reproducing 244. Consider the following statements with reference
animals that reduces the number of to Prokaryotes and Eukaryotes cells:
chromosomes in a cell before reproduction. 1. Eukaryotic cells have no mitochondria whereas
every Prokaryotic cell has mitochondria.
2. Mitosis is a type of cell division is required for
the growth and repair of the body. 2. Eukaryotic cells have completely different
chemical compositions in comparison to
Which of the statements given above is/are correct? Prokaryotic cells.
(a) 1 only 3. Eukaryotic cells have membrane-bound
(b) 2 only organelles whereas Prokaryotic cells have no
(c) Both 1 and 2 membrane-encased organelles.
(d) Neither 1 nor 2
Which of the above statements is/are correct?
240. Which of the following is/are the applications of (a) 2 only
bacteria? (b) 3 only
1. Can be used to convert milk into useful dairy (c) 1 and 3 only
products (d) 1, 2 and 3

Workbook 26
.
SCIENCE & TECHNOLOGY

245. Consider the following diseases: Which of the statements given above is/are correct?
1. Klinefelter’s syndrome (a) 1 only
2. Sickle cell Anaemia (b) 2 only
3. Hypertension (c) 1 and 2 only
Which of the diseases given above is/are a genetic (d) 1, 2 and 3
disease/disorder? 250. DNA can be extracted from which of the following
(a) 1 only specimens?
(b) 2 and 3 only 1. Fingernails
(c) 1 and 2 only
2. Skin
(d) 1, 2 and 3
3. Hair
246. Consider the following pairs 4. Blood
Deficiency Diseases/Disorders 5. Saliva
Select the correct answer using the code given
1. Vitamin A Xerophthalmia
below:
2. Calcium Rickets (a) 1, 2, 3 and 4 only
3. Vitamin D Osteoporosis (b) 1, 2, 3 and 5 only
(c) 2, 3, 4 and 5 only
4. Vitamin B12 Anemia (d) 1, 2, 3, 4 and 5
Which of the statements given above are correct?
251. Consider the following pairs:
(a) 1 and 2 Only
(b) 2 and 3 Only Diseases Pathogens that cause
(c) 1 and 4 Only the disease
(d) 3 and 4 Only
1. Kyasanur Forest Virus
247. Which of the following statements is incorrect disease
regarding ‘Darwinian Theory of Evolution’?
(a) The existing living forms share varying degree of 2. Leishmaniasis Fungi
similarities with life forms that existed millions 3. Brucellosis Protozoa
of years ago.
(b) The organisms with disadvantageous variants 4. Typhoid Bacteria
are eliminated from nature during struggle of Which of the pairs given above is/are correctly
existence. matched?
(c) The species in the absence of environmental (a) 1 only
checks tends to increase very fast. (b) 1 and 4 only
(d) The new species arise due to sudden changes (c) 2, 3 and 4 only
after catastrophic event. (d) 1 and 3 only
248. Which of the following criteria must be fulfilled 252. Consider the following statements regarding
by a molecule to act as a genetic material?
Innate immunity in humans:
1. It should be able to generate its replica.
1. Tears contain antimicrobial compounds which
2. It should provide the scope for slow changes that
prevent microbial growth.
are required for evolution.
2. Saliva plays an important role in physico-
3. It should be chemically and structurally stable.
chemical defence of tooth surfaces.
Select the correct answer using the code given below
3. Hair follicles prevents microorganisms by
(a) 1 only creating an acidic environment.
(b) 2 and 3 only
(c) 2 only Which of the statements given above is/are correct?
(d) 1, 2 and 3 (a) 1 and 2 only
(b) 2 only
249. Consider the following statements with reference
(c) 1 and 3 only
to fluids found in the body and their functions:
(d) 1, 2 and 3
1. Thrombocytes play a key role in the coagulation
of the blood. 253. With reference to Microorganisms, consider the
2. Plasma contains minerals like amino acids and following statements:
lipids. 1. The presence of Coliform bacteria in drinking
3. Plasma constitutes nearly fifty per cent of the water indicates that disease-causing organisms
blood in the body. could be in the water system.

27 Workbook
.
SCIENCE & TECHNOLOGY

2. Lactobacillus microorganism is found in milk 1. Innate immunity is present at the time of birth.
and they convert milk to the curd. 2. Acquired immunity is developed with the help
3. Lactic Acid Bacteria originally found on plants. of lymphocytes in blood.
Which of the statements given above is/are correct? 3. In vaccination weakened pathogen is introduced
(a) 1 and 2 only into the body.
(b) 2 and 3 only Which of the statements given above is/are correct?
(c) 1 and 3 only (a) 1 and 2 only
(d) 1, 2 and 3 (b) 2 and 3 only
(c) 1 and 3 only
254. Which of the following statements regarding the (d) 1, 2 and 3
differences between B-cells and T-cells is correct?
(a) While B-cells are present in the bloodstream, 258. Consider the following statements with
T-cells are located in the lymphatic system. reference to the Yeast:
(b) While B-cells are produced in the liver, T-cells 1. Yeast is a multicellular fungus.
are produced in the bone marrow. 2. Asexual mode of Yeast reproduction includes
(c) Both B-cells and T-cells are involved with budding and binary fission.
antibody production. 3. Yeast can survive either in the presence or
(d) Unlike T-cells, B-cells are only involved in absence of oxygen.
destroying the infected cells.
Which of the statements given above are correct?
255. Consider the following statements: (a) 1 and 2 only
Assertion(A): Colour Blindness is a sex-linked (b) 2 and 3 only
recessive disorder found more in men. (c) 1 and 3 only
Reason(R): The breakdown of proteins in the lens of (d) 1, 2 and 3
male’s eyes is faster than in female’s eyes. 259. Which of the following statements correctly
Which of the option given below is correct based on defines the features of a gene?
the above given statements? 1. The number of genes in an organism varies
(a) Both (A) and (R) are true and (R) is the between various species.
correct explanation of (A) 2. All genes produce proteins to carry various
(b) Both (A) and (R) are true but (R) is not the instructions in living organisms.
correct explanation of (A)
3. All genes are made of deoxyribonucleic acid
(c) (A) is true but (R) is false (DNA) to store genetic information.
(d) (A) is false but (R) is true
Select the correct answer using the code given below:
256. Consider the following statements with reference (a) 1 only
to different type of respiration in living organisms: (b) 2 and 3 only
1. Frogs can also breathe through their skin. (c) 1 and 3 only
2. Cockroaches breathe through small openings on (d) 3 only
the sides of its body called spiracles.
3. Certain prokaryotes can perform respiration in 260. Regarding the features of chromosomes, which of
the absence of oxygen. the following statements is not correct?
(a) It keeps DNA molecules tightly wrapped around
Which of the statements given above is/are correct?
histones.
(a) 2 only (b) It ensures DNA is accurately copied and
(b) 3 only distributed in the vast majority of cell divisions.
(c) 2 and 3 only
(c) All living things contain the same number of
(d) 1, 2 and 3
chromosomes though the size can vary.
257. With reference to immunity in human bodies, (d) Reproductive cells are the only human cells that
consider the following statements: do not contain chromosomes.

Workbook 28
.
SCIENCE & TECHNOLOGY

SOLUTIONS

5. Solution: (b)
1.1. Previous Years Questions
Exp) Option b is correct.
1. Solution (a) When the bark of a tree is removed in a circular fashion
Exp) Option a is correct. all around near its base, it gradually dries up and dies - the
Warm water generally gets more dense as it gets colder, phloem layer of tissue just below the bark is responsible
and therefore sinks. However, Colder than 4° Celsius (39° for carrying food produced in the leaves by photosynthesis
Fahrenheit), water begins expanding and becomes less to the roots. Without food, the roots ultimately die and
stop sending water and minerals to the leaves. There is
dense as it gets colder. As a result, close to freezing, colder
no water supply to the leaves, hence no photosynthesis.
water floats to the top and the warmer water sinks to the
Therefore plant has no energy and it dies.
bottom. The density of water as a function of temperature
can be seen in the plot on the right. Eventually, the coldest 6. Solution: (c)
water, which has floated to the top of the lake in wintry Exp) Option c is correct.
conditions, freezes to form a layer of ice. Right when the
water freezes to ice, the ice becomes significantly less Statement 1 is correct. Sodium lamps produce light in
dense than the water and continues to float on the lake’s 360 degrees but it is not so in the case of LED lamps.
surface. LED lamps have 180-degree bulb angle to preserve light
efficiency and allow for target lighting over areas.
2. Solution (a) Statement 2 is incorrect. As street lights, sodium lamps
Exp) Option a is correct. have on an average shorter lifespan than LED lamps.
High Pressure Sodium lamps tend to have a lifespan
Each biological parent donates one of two ABO genes to
averaging 24,000 hours. Whereas, LED lamps lifespan
their child. The A and B genes are dominant and the O
ranges from 25,000 to 200,000 hours.
gene is recessive. For example, if an O gene is paired with
an A gene, the blood type will be A. Statement 3 is correct. The spectrum of visible light
from sodium lamps is almost monochromatic as they are
Couple with AB+ blood group consist of A+ and B+ gene known for their warm yellow glow. On the other hand,
with O negative blood group consist of O- gene. A+ gene LED lamps offer significant colour advantages in street-
is dominant over O gene which results in A+ blood group lighting. LEDs offer variety of colour options providing
in one child. B+ gene is dominant over O- gene which better lighting solutions than High Pressure Sodium
results in B+ blood group in other child. Therefore, the counterpart.
blood group of the adopted son would be O+.
7. Solution: (a)
3. Solution: (c)
Exp) Option a is correct.
Exp) Option c is correct.
Water is called the “universal solvent” because it is capable
Free radicals are atoms that contain an unpaired electron. of dissolving more substances than any other liquid. This
Due to this lack of a stable number of outer shell electrons, is important to every living thing on earth. It means that
they are in a constant search to bind with another electron wherever water goes, either through the air, the ground,
to stabilize themselves—a process that can cause damage or through our bodies, it takes along valuable chemicals,
to DNA and other parts of human cells. Antioxidants minerals, and nutrients.
neutralize the free radicals produced in the body during It is water’s chemical composition and physical attributes
metabolism. that make it such an excellent solvent. Water molecules
4. Solution: (c) have a polar arrangement of oxygen and hydrogen
atoms—one side (hydrogen) has a positive electrical
Exp) Option c is correct. charge and the other side (oxygen) had a negative charge.
Vegetative reproduction is a method wherein new plants This allows the water molecule to become attracted to
are produced asexually by vegetative parts of a plant many other different types of molecules.
called propagules. The vegetative parts mean the leaf, stem Water can become so heavily attracted to a different
and root. compound, like salt (NaCl), that it can disrupt the
Vegetative propagation does not help in eliminating virus, attractive forces that hold the sodium and chloride in the
rather it is a very effective method for perpetuating and salt compound together and, thus, dissolve it.
spreading viruses. Vegetative propagation produces exact
copies of parent cell; thus it produces clonal population. It 8. Solution: (a)
can be practised most of the year. Exp) Option a is correct.

29 Workbook
.
SCIENCE & TECHNOLOGY

Water generally gets more dense as it gets colder, and 14. Solution (d)
therefore sinks. However, when the water is colder than Exp) Option d is correct.
4° Celsius (39° Fahrenheit), water begins expanding and
Sodium and potassium play essential roles in muscle
becomes less dense. As a result, close to freezing, colder contraction because of their importance in nerve
water floats to the top and the warmer water sinks to the function. Nervous system communicates with muscles
bottom. Eventually, the coldest water, which has floated through structures called neuromuscular junctions, and
to the top of the lake in wintry conditions, freezes to form the activation of a nerve triggers muscle contraction.
a layer of ice. Right when the water freezes to ice, the
Calcium and magnesium work together to control muscle
ice becomes significantly less dense than the water and
contraction. Both minerals interact with the proteins actin
continues to float on the lake’s surface.
and myosin -- structural proteins that shorten with each
9. Solution: (a) muscle contraction, then lengthen to relax your muscles.
Exp) Option a is correct. Iron also helps your muscles contract. Each muscle
contraction uses up energy as well as oxygen. Iron helps
Moderator is required in a Nuclear reactor to slow down support continued muscle function by supporting energy
the neutrons produced during the fission reaction so that production in your muscle cells, so that your muscle fibers
the chain reaction can be sustained. Heavy Water is an always have the energy they need to contract properly.
excellent moderator due to its high moderating ratio and
low absorption cross section for neutrons. 15. Solution: (a)
Exp) option a is correct.
10. Solution (c)
Viruses lack metabolic enzymes and thus they cannot
Exp) Option c is correct. generate their own energy. They cannot be cultured in
Fruits are influenced by physiological processes, which any synthetic medium as they require a growth medium
are respiration and transpiration. Rate of respiration containing living cell.
increases with increase in surrounding temperature – this A mechanical vector carries a pathogen on its body from
decreases shelf life of the fruit. Similarly, Fruits stored in one host to another, not as an infection. A biological
a cold chamber exhibit longer storage life because rate of vector carries a pathogen from one host to another after
respiration is decrease. becoming infected itself. Thus, Viruses are transmitted
from one organism to another not only by biological
11. Solution (b) vectors but also by mechanical vectors
Exp) Option b is correct.
16. Solution: (c)
Capillary rise or capillarity is a phenomenon in which
liquid ‘spontaneously’ rises or falls in a narrow space Exp) Option c is correct.
such as a thin tube or in the voids of a porous material. A ball bearing is a type of rolling-element bearing that
Surface tension is an important factor in the phenomenon serves three main functions while it facilitates motion:
of capillarity. it carries loads, reduces friction and positions moving
The sipping action occurs when you lower the air pressure machine parts.
in your mouth, which allows the atmospheric pressure to Ball bearings are used in bicycles, cars, etc., because the
push the liquid up the straw. Thus it is not a capillary rise. effective area of contact between the wheel and axle is
reduced, this reduces friction.
12. Solution (a)
17. Solution: (a)
Exp) Option a is correct.
Exp) Option a is correct.
The solar wind or other electrically charged particles from
space do not impact Earth directly but are deflected, like There are four fundamental forces in nature which
a stream diverted around a boulder by Earth’s magnetic are Gravitational force, Strong force, Weak force and
field, which forms a tear-drop-shaped cavity in the solar Electromagnetic force. These forces prevent the drifting of
wind flow. This cavity called the magnetosphere, contains matter in the universe and helps keep it together.
highly dilute plasmas whose behaviour is controlled by Gravitational force (Gravity) is the weakest force but has
Earth’s magnetic field and its interaction with the solar an infinite range.
wind. Strong force: This force holds the nucleus of an atom
together. It is the strongest of the forces, but it is very short
13. Solution (c) ranged.
Exp) Option c is correct. Weak Force: This force is weak compared to the strong
Transplanting is the process of moving a plant from one force as the name implies and has the shortest range.
growing medium to another. Electromagnetic Force: This is the second strongest force
Many transplanted seedlings do not grow because most of after the strong force and it acts on electrically charged
the root hairs are lost during transplantation particles.

Workbook 30
.
SCIENCE & TECHNOLOGY

18. Solution: (d) 20. Solution: (c)


Exp) Option d is correct. Exp) Option c is correct.
An optical illusion is a visual stimuli that is perceived by
the eyes and then comprehended by the brain in a way
that is different from reality.
Most of the stars are shining with a steady light. The
movement of air (sometimes called turbulence) in the
atmosphere of Earth causes the starlight to get slightly
bent as it travels from the distant star through the
atmosphere down to us on the ground. The twinkling of
a star is due to atmospheric refraction of starlight. The
starlight, on entering the earth’s atmosphere, undergoes
refraction continuously before it reaches the earth. The
atmospheric refraction occurs in a medium of gradually
changing refractive index.
This means that some of the light reaches us directly and
When sunlight falls on the drops of rain formation of a some gets bent slightly away. To our eyes, this makes the
Rainbow occurs in the sky. These droplets work as a prism star seem to twinkle.
for the rays coming from the sun. When the sunlight
gets through these droplets, it results in refraction and 21. Solution (b)
dispersion in seven colours known as VIBGYOR. Then Exp) Option b is correct.
it goes to total internal reflection. This happens due to
Photosynthesis is the process by which plants use sunlight,
combined phenomena of Dispersion, Refraction, and
water, and carbon dioxide to create oxygen and energy in
Total Internal reflection in the atmosphere. the form of sugar.
Statement 1 is correct. Dispersion is the process of
splitting lights into different colours. 22. Solution: (a)
Example: Dispersion done by Prism. Each individual Exp) Option a is correct.
droplet of water acts as a tiny prism that both disperses Arachnids are a class of joint-legged invertebrates, with
the light and reflects it back to our eye. eight legs.
Statement 2 is correct. Dispersion is the process of Mite, Scorpion and spider are Arachnids, whereas Crab is
splitting lights into different colours. Since water is denser a Crustacean.
than air, light is refracted as it enters the drop.
23. Solution: (b)
Statement 3 is correct. Some of the light will reflect off
the back of the drop if the angle is larger than the critical Exp) Option b is correct.
angle (48° for water). Total Internal reflection is the Souring of milk is a chemical change as acidification takes
phenomenon that occurs based on 2 conditions:- place and soured milk is produced. This
1. When light moves from a denser medium to a less sour milk has different chemical properties from fresh
dense medium. milk.
2. When the angle of incidence is greater than the critical Crystallisation of sodium chloride and melting of ice are
angle. physical changes.
19. Solution: (c) 24. Solution: (d)
Exp) Option c is correct. Exp) Option d is correct.
Graphene is a one-atom-thick layer of carbon atoms (not Pair 1 is correctly matched. Lead is a common impurity
silica) arranged in a hexagonal lattice. It is the building- found in cosmetic lip products, such as lipsticks, and
block of Graphite. Graphene is the thinnest compound other cosmetics, such as eye shadows, blushes, compact
known to man at one atom thick, the lightest material powders, shampoos, and body lotions.
known, the strongest compound discovered (between Pair 2 is correctly matched. Brominated vegetable oil is
100-300 times stronger than steel). a food additive sometimes used to keep citrus flavouring
In the field of optics, transparency is the physical from separating out in some sodas and soft drinks. It’s
property of allowing light to pass through the material banned as a food additive in Europe and Japan.
without appreciable scattering of light. Pair 3 is correctly matched. MSG, is a glutamate, or
Graphene has high optical transparency and can uniformly salt of glutamic acid, a “non-essential” amino acid. It is a
absorb light across the visible and near-infrared parts of neurotransmitter - transporting messages from one nerve
the spectrum and thus it is potentially suitable for use in cell to another. Because it is said to enhance flavours,
spin transport. some scientists believe it “excites nerve endings” and

31 Workbook
.
SCIENCE & TECHNOLOGY

exhibits “neuro-excitatory properties”, that is the ability to adding or removing specific molecules, such as hormones
stimulate neurons. It is commonly used in Chinese fast or growth factors, can be explored.
food.
28. Solution: (d)
25. Solution (d) Exp) Option d is correct.
Exp) Option d is correct. A light-year is a measurement of distance in space. The
• According to Albert Einstein’s General Theory of astronomical distances are measured in light-years
Relativity, Light will be affected the same way matter because, the speed of light is constant throughout the
is affected by gravity. universe and is known to high precision. In a vacuum,
• Based on this theory, Hubble found that distant light travels at 670,616,629 mph (1,079,252,849 km/h). To
galaxies move away faster than those that are relatively find the distance of a light-year, you multiply this speed
nearby and the universe is indeed expanding. by the number of hours in a year (8,766). The result: One
• According to this theory, gravity is caused by a light-year equals 5,878,625,370,000 miles (9.5 trillion km).
warping of space and time. Gravity is the curvature At first glance, this may seem like an extreme distance, but
of the universe, caused by massive bodies, which the enormous scale of the universe dwarfs this length.
determines the path that objects travel. Measuring in light-years also allows astronomers to
determine how far back in time they are viewing. Because
26. Solution: (c) light takes time to travel to our eyes, everything we view in
Exp) Option c is correct the night sky has already happened. In other words, when
Conducting takes place due to the movement of the you observe something 1 light-year away, you see it as it
electrons. Most of the metals are considered as good appeared exactly one year ago.
conductors of electricity. This is because when metal atoms
combine with each other, the outermost electrons lose 29. Solution: (c)
contact with their parent atoms. The remaining positively Exp) Option c is the correct answer.
charged atomic centers form an ordered structure while Probiotics are live microorganisms that are intended to
the outer electrons move freely around the whole sample. have health benefits when consumed or applied to the
On the other hand, in case of non-metals the body. They can be found in yogurt and other fermented
conduction electrons are held tightly by the nucleus. foods, dietary supplements, and beauty products.
So, they cannot move easily which means, they do
not have readily available free electrons. Hence, the Statement 1 is correct: Probiotics are a combination of
non-metals are not good conductors of electricity. live beneficial bacteria and/or yeasts.
However, non-metals can also become good conductors Statement 2 is incorrect: Probiotics live naturally in
of electricity if external energy is applied to them, which your body. L. acidophilus is a probiotic bacteria that
makes the conduction electrons feasible. naturally occurs in the human gut. The most common
Therefore, Non-metals are considered as insulators place linked to beneficial microbes is your gut (mostly
because they have extremely high resistance to the flow of large intestines), but there are other places like mouth,
charge through them. The atoms hold onto their electrons urinary tract, skin , lungs etc where these probiotics are
tightly and so they cannot conduct electricity. found.
Statement 3 is correct: Yogurt is a well-known food
27. Solution: (a)
source of probiotics. Certain strains of bacteria in yogurt
Exp) Option a is correct. have ß-D-galactosidase, which is an enzyme that helps
Option a is incorrect. The joining of DNA segments break down lactose in dairy products into the sugars
from two cells of different species is possible but making a glucose and galactose. Thus Probiotics help in the
Functional chromosome out of it has not been discovered digestion of milk sugars.
yet.
30. Solution: (d)
Option b is correct. Unlike DNA synthesis in living cells,
artificial gene synthesis does not require template DNA, Exp) Option d is the correct answer
allowing virtually any DNA sequence to be synthesized in Lymphocytes are a type of white blood cells. They play a
the laboratory. critical role in keeping us healthy. Without them, we can’t
Option c is correct. A piece of DNA from an animal survive.
cell can be made to replicate outside a living cell in a T cells protect us from infection. In our daily lives,
laboratory. we’re constantly exposed to pathogens, such as bacteria,
Option d is correct. Given appropriate surroundings, viruses and fungi. Without T lymphocytes, also called T
most plant and animal cells can live, multiply, and even cells, every exposure could be life-threatening. T cells can
express differentiated properties in a tissue-culture wipe out infected or cancerous cells. They also direct the
dish. The cells can be watched continuously under the immune response by helping B lymphocytes to eliminate
microscope or analyzed biochemically, and the effects of invading pathogens.

Workbook 32
.
SCIENCE & TECHNOLOGY

B cells create antibodies. B lymphocytes, also called B and solar flares. Thus, Solar storms can be cause by a
cells, create a type of protein called an antibody. These sudden release of high energy particles from the Sun.
antibodies bind to pathogens or to foreign substances,
such as toxins, to neutralize them. For example, an 33. Solution (d)
antibody can bind to a virus, which prevents it from Exp) Option d is correct.
entering a normal cell and causing infection. B cells can Statement 1 is correct. Raman spectroscopy is the
also recruit other cells to help destroy an infected cell. measurement of the intensity and wavelength of
inelastically scattered light from molecules. The Raman
1.2. General Physics scattered light occurs at wavelengths that are shifted from
the incident light by the energies of molecular vibrations.
31. Solution (a)
A Raman spectrometer coupled with a fiber optics probe
Exp) Option a is correct has great potential in applications such as monitoring
Statement 1 is correct. Solar Minimum is the period and quality control in industrial food processing, food
of least solar activity in the 11-year solar cycle of the safety in agricultural plant production, and convenient
Sun. During this time, sunspot and solar flare activity inspection of pharmaceutical products, even though
diminishes and often does not occur for days at a time. different types of packing.
Statement 2 is correct. During solar minimum, the sun’s Statements 2 is correct. Raman spectroscopy is used in
magnetic field weakens and provides less shielding from advanced nanotechnology including cancer detection,
cosmic rays. It thus may cause health risks to astronauts analysing various substances in forensic science, and
travelling through space. to identify and characterize the original and decayed
Statement 3 is incorrect. Solar minimum does not mean compounds present in objects of art and archaeology.
the non- occurrence of total solar eclipse. But, during Other application areas are biotechnology, mineralogy,
a total solar eclipse it is possible to see clear, naked eye environmental monitoring, food and beverages, medical
evidence of where the Sun is in its cycle. Totality—when and clinical chemistry, diagnostics, pharmaceutical,
the Moon completely blocks the Sun’s bright disk—affords material science, surface analysis, etc.
a brief view of the Sun’s corona, its hot outer atmosphere. Statement 3 is correct. Raman spectroscopy is very
During solar minimum the corona is relatively small and useful in drug analysis due to advantages such as ease
tightly bound to the surface. During solar maximum, the
of use, minimal sample handling, and the significant
Sun’s corona is typically flared and stretching away into
differences in scattering strength between packaging
space.
materials, tablet excipients, and active drug components.
32. Solution (b) It can also be used to identify isomers and to determine
Exp) Option b is correct. energy difference between isomers. These advantages,
in combination with fiber optics and microscopes, have
Statement 1 is incorrect. A black hole is a place in space enabled the use of Raman spectroscopy as a quality
where gravitational pull is so immense that even light control tool in the pharmaceutical industry.
cannot get out. The gravity is so strong because matter
has been squeezed into a tiny space. Because no light 34. Solution (d)
can get out, people can’t see black holes. Black holes are Exp) Option d is correct.
thought to be formed when very massive stars collapse at
the end of their life cycle. Geotail is a region around the earth that exists as a result
of the interactions between the Sun and Earth. (Option
Statement 2 is correct. Dark matter is an invisible mass
c is correct)
thought to be non-baryonic (not composed primarily of
baryons protons, neutrons and all the objects composed Our Sun emits a continuous outflowing stream of
of them) in nature. They are possibly composed of some electrons and protons into the solar system, called the
as-yet undiscovered subatomic particles. Scientists have solar wind. The solar wind plasma which has charged
not yet observed dark matter directly. It doesn’t interact particles embedded in the extended magnetic field of the
with baryonic matter and it’s completely invisible to Sun, moves at speeds of a few hundred km per second.
light and other forms of electromagnetic radiation, It interacts with solar system bodies including Earth and
making dark matter impossible to detect with current its moon. Since the Earth has a global magnetic field, it
instruments. obstructs the solar wind plasma and this interaction results
Statement 3 is correct. A solar storm is a disturbance in the formation of a magnetic envelope around Earth,
on the Sun, which can emanate outward across the called the magnetosphere. The Earth’s magnetosphere is
heliosphere, affecting the entire Solar System, including compressed into a region approximately three to four
Earth. Solar flares, coronal mass ejections, high-speed times the Earth radius (~22000 km above the surface)
solar wind, and solar energetic particles are all forms of on the side facing the Sun, but is stretched into a long
solar storms. Solar energetic particles are high-energy tail on the opposite side that goes beyond the orbit of
charged particles, primarily thought to be released by Moon. It is this tail that is called the geotail.(Option a
shocks formed at the front of coronal mass ejections is correct)

33 Workbook
.
SCIENCE & TECHNOLOGY

The geotail region is comparatively free of Solar charged Statement 3 is correct. Recently, Scientists at NASA
particles and hence allows better observation conditions have observed Bose-Einstein Condensate (BEC) for the
to observe the moon when it is in the geotail region. first time in space offering unprecedented insight that
The geotail is a region in space that allows the best could help solve some of the quantum universe’s most
observations. The region exists as a result of the intractable conundrums.
interactions between the Sun and Earth. (Option b is
correct) 37. Solution (b)
Thus, all of the options given above are correct Exp) Option b is correct.
Statement 1 is incorrect. Ceres is a dwarf planet. It is the
only dwarf planet located in the inner solar system.
Statement 2 is correct. It is the largest object in the
asteroid belt between Mars and Jupiter. According to the
data collected by NASA’s Dawn spacecraft, dwarf planet
Ceres reportedly has salty water underground. These
findings have given Ceres the status of an ocean world as
it has a big reservoir of salty water underneath its frigid
surface.
38. Solution (d)
Exp) Option d is correct.
Statement 1 is incorrect. It is the biggest indigenously
developed variant of the Pressurised Heavy Water
Reactor (PHWR).
35. Solution (b) The PHWRs, which use natural uranium as fuel and
Exp) Option b is correct. heavy water as moderator, are the mainstay of India’s
nuclear reactor fleet. Until now, the biggest reactor size
Statement 1 is incorrect. Planets generally have more of indigenous design was the 540 MWe PHWR, two of
lithium content than their stars since stars, as per known which have been deployed in Tarapur, Maharashtra.
mechanisms of evolution, actually destroy lithium as they
evolve into red giants. However, there are exceptions as Statement 2 is incorrect. It is a nuclear power station
some stars were found that were lithium-rich. Recently, located in Gujarat.
researchers found that when stars grow beyond their Red 39. Solution (a)
Giant stage into what is known as the Red Clump stage,
they produce lithium in what is known as a Helium Flash Exp) Option a is correct.
and this is what enriches them with lithium. Statement 1 is correct. Scientist at Institute of Nano
Statement 2 is correct. Earth has more lithium content Science and Technology (INST), Mohali (Punjab) have
produced electron gas with ultra-high mobility, which
than the Sun. The Sun has about a factor of 100 lower
can speed up transfer of quantum information and
amount of lithium than the Earth.
signal from one part of a device to another and increase
Statement 3 is correct. Stars have a tendency to destroy data storage and memory. Due to the high mobility of the
lithium as they evolve into red giants via low-temperature electron gas, electrons do not collide inside the medium
nuclear burning. for a long distance and hence do not lose the memory and
information. Hence, such a system can easily remember
36. Solution (c) and transfer its memory for a long time and distance.
Exp) Option c is correct. Statement 2 is incorrect. Due to the high mobility of the
Statement 1 is incorrect. Bose-Einstein Condensate electron gas, electrons would collide less during their flow
(BEC) is the fifth state of matter whose existence was and thus their resistance is very low, and hence they don’t
predicted by Albert Einstein and Indian mathematician dissipate energy as heat. Since such devices don’t dissipate
Satyendra Nath Bose in the early 1920s. It is formed when energy as heat, such devices do not heat up easily and
atoms of certain elements are cooled to near absolute zero need less input energy to operate.
(0 Kelvin/-273.15 Celsius). At this point, atoms become
a single entity with quantum properties wherein each 40. Solution (d)
particle also functions as a wave of matter. Exp) Option d is correct.
Statement 2 is incorrect. BECs are extremely fragile. The The light does not travel in the same direction in all
slightest interaction with the external world is enough media. It appears that when travelling obliquely from
to warm them past their condensation threshold. This one medium to another, the direction of propagation of
makes them nearly impossible for scientists to study on light in the second medium changes. This phenomenon is
Earth, where gravity interferes with the magnetic fields known as refraction of light. Following are the examples
required to hold them in place for observation. of refraction of light:

Workbook 34
.
SCIENCE & TECHNOLOGY

Option 1 is correct. Formation of a rainbow is an which support different propagation characteristics for
example of refraction as the sun rays bend through the transmission of voice and data. Lower bands like 800
raindrops resulting in the rainbow. MHz and 900 MHz support a good indoor coverage
Option 2 is correct. A swimming pool always looks while higher bands like 2300 MHz have a good carrying
shallower than it really is because the light coming capacity.
from the bottom of the pool bends at the surface due to
refraction of light. Telecom spectrum starts from 800MHz, and goes
up to 2300MHz. Commonly used bands for cellular
Option 3 is correct. Refraction can also be observed in
communication are 800MHz, 900MHz, 1800MHz,
the form of mirage, while driving down a road on a hot
day. 2100MHz, and 2300MHz. According to the GSM Alliance,
the most suitable spectrum for telecommunication is in
41. Solution (c) the 400MHz to 4GHz range, and these bands are used
Exp) Option c is correct globally for various telecommunications purposes.
Ploonet is nothing but an orphaned moon that has 44. Solution (b)
escaped the gravitational bond of its parent planet.
According to the definition adopted by the IAU in 2006, Exp) Option b is correct
a dwarf planet is, “a celestial body orbiting a star that Statement 1 is incorrect.
is massive enough to be rounded by its own gravity but Superconductivity, complete disappearance of electrical
has not cleared its neighbouring region of planetesimals
resistance in various solids when they are cooled below a
and is not a satellite. Examples include pluto. There are
currently five dwarf planets in our solar system, Ceres, characteristic temperature. This temperature, called the
Pluto, Haumea, Makemake and Eris. With the exception transition temperature, varies for different materials but
of Ceres, which is located in the asteroid belt, the other generally is below 20 K (−253 °C).
dwarf planets are found in the outer solar system. A superconducting material can be returned to the
An exoplanet is any planet beyond our solar system. They normal (i.e., non superconducting) state either by passing
orbit a star other than our sun. a sufficiently large current through it or by applying a
A dwarf star is a star of relatively small size and low sufficiently strong magnetic field to it.
luminosity. Important subclasses of dwarf stars are white Statement 2 is correct. Suggested uses for
dwarfs and red dwarfs. superconducting materials include medical magnetic-
42. Solution (a) imaging devices, magnetic energy-storage systems,
motors, generators, transformers, computer parts, and
Exp) Option a is correct
very sensitive devices for measuring magnetic fields,
Statement 1 is correct. The acronym SONAR stands for voltages, or currents.
SOund Navigation And Ranging. Sonar is a device that
uses ultrasonic waves to measure the distance, direction 45. Solution (b)
and speed of underwater objects.
Exp) Option b is correct
Statement 2 is correct. Lidar, which stands for Light
Detection and Ranging, is a remote sensing method that Refraction is the bending of light (it also happens
uses light in the form of a pulsed laser to measure ranges with sound, water and other waves) as it passes from
(variable distances) to the Earth. These light pulses— one transparent substance into another. This bending
combined with other data recorded by the airborne system by refraction makes it possible for us to have lenses,
— generate precise, three-dimensional information about magnifying glasses, prisms and rainbows.
the shape of the Earth and its surface characteristics. Refractive index, also called index of refraction, measure
Statement 3 is incorrect. Lidar can’t perform well in of the bending of a ray of light when passing from one
fog, rain, snow and dusty weather. It also struggles to medium into another. It is defined as the ratio of the
detect a glass wall or door, which is why smartphone sine of the angle of incidence to the sine of the angle of
manufacturers and self-driving cars makers use LiDAR refraction; i.e., n = sin i / sin r
along with secondary cameras and sensors.
43. Solution (c)
Exp) Option c is correct.
Statement 1 is correct. Spectrum refers to a range of
radio-waves that are used for communication purposes.
Spectrum relates to the radio frequencies allocated to the
mobile industry and other sectors for communication
over the airwaves.
Statement 2 is correct. Airwaves come in different bands

35 Workbook
.
SCIENCE & TECHNOLOGY

Example - When a torch is switched on in a foggy


environment, the path of the light becomes visible. In this
scenario, the water droplets in the fog are responsible for
the light scattering.\

Dispersion: When white light is passed through a glass


prism it splits into its spectrum of colours (in order
violet, indigo, blue, green, yellow, orange and red) and
this process of white light splitting into its constituent
colours is termed as dispersion.

46. Solution (a)


Exp) Option a is correct.
Pair 1 is correctly matched. Myopia or near-sightedness
is a condition in which close objects appear clearly, but far
ones don’t. Myopia occurs if the eyeball is too long or the
cornea (the clear front cover of the eye) is too curved. As
a result, the light entering the eye isn’t focused correctly,
and distant objects look blurred. If you are hypermetropic,
the image of a faraway object is formed before the retina.
It can be corrected using a concave lens.
Total internal reflection is the phenomenon of complete
reflection of light back into the medium when light travels Pair 2 is incorrectly matched. Hypermetropia or far-
from a medium having more density to a medium having sightedness is a condition in which faraway objects appear
less density. The phenomenon occurs if the angle of clearly, but near ones don’t. If you are hypermetropic, the
incidence is greater than a certain limiting angle, called image of a nearby object is formed behind the retina.
Farsightedness occurs when your eyeball is shorter than
the critical angle.
normal or your cornea is curved too little. The effect is the
Following are the two conditions of total internal opposite of nearsightedness.
reflection:
It can be corrected using a convex lens.
1. The light ray moves from a more dense medium to a
less dense medium. Pair 3 is incorrectly matched. Astigmatism is a common
vision problem caused by an error in the shape of the
2. The angle of incidence must be greater than the cornea. With astigmatism, the lens of the eye or the cornea,
critical angle. which is the front surface of the eye, has an irregular
Examples - mirage, diamond sparkling, optical fibres, etc. curve. This can change the way light passes, or refracts, to
your retina. The cause of astigmatism is unclear; however
it is believed to be partly related to genetic factors.
It can be corrected using a cylindrical lens.

Tyndall effect, also called Tyndall phenomenon is the


scattering of a beam of light by a medium containing
small suspended particles—e.g., smoke or dust in a room,
which makes visible a light beam entering a window. The
effect is named for the 19th-century British physicist John
Tyndall, who first studied it extensively. 47. Solution (b)

Workbook 36
.
SCIENCE & TECHNOLOGY

Exp) Option b is correct. rays towards the principal axis) which is relatively thick
Mirage and looming both are results of total internal across the middle and thin at the lower and upper edges.
reflection of light. However, mirage takes place in hot The edges are curved outward rather than inward. It is
regions and in looming takes place in cold regions. used to correct hypermetropia.
In mirage, light from distant objects appears to be A concave lens is a lens that possesses at least one surface
reflected from ground and looming distant objects appear that curves inwards. It is a diverging lens, meaning that it
to be flying. For mirage to form, refractive index goes on spreads out light rays that have been refracted through it.
increasing as we go up and for looming, refractive index A concave lens is thinner at its centre than at its edges, and
goes on decreasing. is used to correct short-sightedness (myopia).
A superior mirage, also called looming, can be produced While plane mirrors always produce virtual images,
when warm air exists over cold water. Again, using the concave mirrors are capable of producing both real and
pattern from Greenler, the vertical scale and the curvature virtual images. A real image occurs where rays converge,
are greatly exaggerated to show the effect. Such mirages whereas a virtual image occurs where rays only appear to
are often seen at great distances in the arctic region when diverge.
the air is significantly warmer than the water. Since the
geometry of the mirage images depends on the details of A concave mirror has a reflective surface that is curved
the temperature contour, a great variety of mirage images inward and away from the light source. Concave mirrors
can be formed. reflect light inward to one focal point. Unlike convex
mirrors, the image formed by a concave mirror shows
different image types depending on the distance between
the object and the mirror.
A convex mirror is a diverging mirror in which the
reflective surface bulges towards the light source. They are
not used to focus light as they reflect light outwards. The
image formed by convex mirrors is smaller than the object
but gets larger as they approach the mirror. They are used
as rear view mirrors in vehicles.
Statement 3 is correct: When the object is placed at
principal focus (F) of a concace mirror, a highly enlarged
In optics, dispersion refers to light that is separated by imaged is formed at infinity.
individual colors as it passes through an object. Rainbows Statement 4 is correct: For a convex mirror, if the object
are the most common example of this event. The rainbow is at infinity the image will be a dot on te focal point. As
forms when white light is separated into components of the object moves from infinity toward the mirror, the
different wavelengths (or colors). image moves along the principal axis toward the mirror.

Tyndall effect, also called Tyndall phenomenon,


scattering of a beam of light by a medium containing
small suspended particles—e.g., smoke or dust in a room,
which makes visible a light beam entering a window. The
effect is named for the 19th-century British physicist John
Tyndall, who first studied it extensively.
48. Solution (d)
Exp) Option d is correct
All the statements are correct.
The convex lens is a lens that converges rays of light that
are parallel to its principal axis (i.e. converges the incident

37 Workbook
.
SCIENCE & TECHNOLOGY

Statement 1 is correct. Event Horizon Telescope is a


group of 8 radio telescopes which recently captured first
optical image of black hole in 2019.

Statement 2 is correct. Primordial Black Holes have


masses comparable to or less than that of the Earth.
These purely hypothetical objects could have been formed
through the gravitational collapse of regions of high
density at the time of the Big Bang.
Statement 3 is incorrect. Black holes are not always
stationary and move from its place. Scientists have
discovered the first moving supermassive black hole
whose mass is about three million times that of our Sun.
However, majority of supermassive black holes are not
expected to be moving and they’re usually content to just
49. Solution (d) sit around at their place.
Exp) Option d is correct.
51. Solution (c)
The Milankovitch Cycles describe how the tilt of
Exp) Option c is correct.
the Earth, the shape of its orbit and where its axis is
pointing collectively influence climate. Statement 1 is incorrect. Wolf–Rayet stars are highly
luminous objects and one of the hottest kinds of stars.
A century ago, Serbian scientist Milutin Milankovitch The surface temperature of WR stars is a thousand times
hypothesized the long-term, collective effects of changes more than the Sun. They are massive stars. Further, they
in Earth’s position relative to the Sun are a strong driver have completely lost their outer hydrogen.
of Earth’s long-term climate, and are responsible for
triggering the beginning and end of glaciation periods Statement 2 is incorrect. A supernova is a powerful and
(Ice Ages). luminous stellar explosion. A supernova is the biggest
explosion that humans have ever seen. Each blast is the
Specifically, he examined how variations in three types extremely bright, super-powerful explosion of a star.
of Earth orbital movements affect how much solar A supernova is the explosion of a star. It is the largest
radiation (known as insolation) reaches the top of Earth’s explosion that takes place in space. Supernovas are often
atmosphere as well as where the insolation reaches. These seen in other galaxies. But supernovas are difficult to see in
cyclical orbital movements, which became known as our own Milky Way galaxy because dust blocks our view.
the Milankovitch cycles, cause variations of up to 25 Further, a supernova happens where there is a change in
percent in the amount of incoming insolation at Earth’s the core or center of a star.
mid-latitudes (the areas of our planet located between Statement 3 is correct. Neutron stars are formed when
about 30 and 60 degrees north and south of the equator). a massive star runs out of fuel and collapses. A neutron
The Milankovitch cycles include: star is the collapsed core of a massive supergiant star,
1. The shape of Earth’s orbit, known as eccentricity; which had a total mass of between 10 and 25 solar masses,
2. The angle Earth’s axis is tilted with respect to Earth’s possibly more if the star was especially metal-rich. Except
orbital plane, known as obliquity; and for black holes, and some hypothetical objects (e.g. white
holes, quark stars, and strange stars), neutron stars are
3. The direction Earth’s axis of rotation is pointed,
the smallest and densest currently known class of stellar
known as precession.
objects
A Climate Time Machine - The small changes set in
motion by Milankovitch cycles operate separately and 52. Solution (b)
together to influence Earth’s climate over very long Exp) Option b is correct.
timespans, leading to larger changes in our climate Albert Einstein, in his theory of special relativity,
over tens of thousands to hundreds of thousands of determined that the laws of physics are the same for all
years. Milankovitch combined the cycles to create a non-accelerating observers, and he showed that the speed
comprehensive mathematical model for calculating of light within a vacuum is the same no matter the speed
differences in solar radiation at various Earth latitudes at which an observer travel.
along with corresponding surface temperatures. The
Special relativity argued that space and time are
model is sort of like a climate time machine: it can be run
inextricably connected. It found that space and time were
backward and forward to examine past and future climate
interwoven into a single continuum known as space-time.
conditions.
Option a is incorrect.
50. Solution (a) Big Bang Theory states that all of the current and past
Exp) Option a is correct. matter in the Universe came into existence at the same

Workbook 38
.
SCIENCE & TECHNOLOGY

time, roughly 13.8 billion years ago. At this time, all (it also happens with sound, water and other waves) as
matter was compacted into a very small ball with infinite it passes from one transparent substance into another
density and intense heat called a Singularity. Suddenly, caused by its change in speed. When light travels from air
the Singularity began expanding, and the universe as we into water, it slows down, causing it to change direction
know began. slightly. This change of direction is called refraction.
Option c is incorrect.
Quantum theory is the theoretical basis of modern
physics that explains the nature and behaviour of
matter and energy on the atomic and subatomic level.
The nature and behaviour of matter and energy at that
level is sometimes referred to as quantum physics and
quantum mechanics.
Option d is incorrect.
String theory is a theoretical framework in which the
point-like particles of particle physics are replaced by one-
dimensional objects called strings. String theory describes
how these strings propagate through space and interact
with each other.
53. Solution (d) 55. Solution (c)
Exp) Option d is correct. Exp) Option c is correct.
Statement 1 is correct. Recently, some researchers Statement 1 is correct. Superconductivity is a state
have developed a new method of soft ionisation using in which a material shows absolutely zero electrical
nanosheets which eliminates the need for external resistance. In a superconducting state, the material offers
energy sources completely. The team used a simple no resistance at all. All the electrons align themselves in
filter paper as a support material and coated this with
a particular direction and move without any obstruction
2D molybdenum disulphide (MoS2). This molybdenum
coherently.
disulphide was created as a thin sheet and spread on a
paper. This modified paper produces an electric current Statement 2 is incorrect. Superconductivity is a
when liquids flow over it. phenomenon that has been so far possible only at
extremely low temperatures, in the range of 100°C
The team of scientists observed that a flow of pure
below zero (-100°C)
methanol over the nanosheet generates a record-high
current of 1.3 microampere. Statement 3 is correct. Superconductors have the
potential for use in, an enormous variety of applications.
Statement 2 is correct. The nanosheet so made has
many more applications: check glucose in blood, toxic Examples include high-speed magnetic-levitation
chemicals in any liquid, pesticide or any contaminants trains, magnetic-resonance-imaging (MRI) equipment,
in drinking water. The main plus point is that it can ultra-high-speed computer chips, high-capacity digital
even detect very low levels. The current wet chemical or memory chips, alternative energy storage systems,
reagent-based methods are lengthy and require skilled radio-frequency (RF) filters, radio-frequency amplifiers,
labour. The new method is cost-effective, and all you need sensitive visible-light and infrared detectors, miniaturized
is simple paper coated with nanostructures. wireless transmitting antennas, systems to detect
submarines and underwater mines, and gyroscopes for
Statement 3 is correct. The nanosheet can be used as earth-orbiting satellites.
a self-energised disposable sensor for breath alcohol
detection. The device was modified in such a way that the 56. Solution (d)
breath of a drunken person can interact directly with the
Exp) Option d is correct.
flowing liquid on the MoS2-coated surface. A volunteer
who had consumed 650 mL of beer containing 9% (v/v) Statement 1 is correct. A launch near the equator
alcohol blew over the paper, and the corresponding towards the east direction will get an initial boost equal
current was measured. While standard breath analysers to the velocity of Earth’s surface. The surface velocity of
detect breath alcohol concentration above 5%, this device rotation varies from point to point on the Earth. It is about
could detect even less than 3%. 1600 km per hour or about 460 meters in a second near
the equator.
54. Solution (d)
Statement 2 is correct. If satellites are launched from the
Exp) Option d is correct. west, they have to battle against the angular momentum
Option d is correct. A person standing waist-deep in a of Earth’s rotation. Thus, launching from the east would
swimming pool appears to have short legs due to effect give an advantage and satellites could take advantage of
of refraction of light. Refraction is the bending of light this momentum.

39 Workbook
.
SCIENCE & TECHNOLOGY

Statement 3 is correct. Another characteristic of surface, the wave will bounce back as electrons vibrate and
launching satellites is that the launching stations are do not allow to pass it.
generally located near the eastern coastline so that, just
in case of failure of the launch, the satellite does not fall 59. Solution (c)
on the built-up hinterland. Exp) Option c is correct.
57. Solution (b) Statement a is incorrect: Comets are cosmic aggregations
of rocky dust, frozen gases orbiting our sun in the
Exp) Option b is correct. Kuiper Belt or beyond that in highly elliptical orbits.
Statement 1 is incorrect. The Sun is a star, and thus When they come near the sun, they develop a trail of
generates its most of the energy through nuclear fusion melted ice and gas, called a tail.
of hydrogen nuclei into helium and further on. Statement b is incorrect: Dwarf planets are planet-like
Statement 2 is correct: Our Sun is surrounded by a celestial bodies orbiting around our sun, with enough
jacket of gases called an atmosphere. The corona is the gravity to assume a round shape, but not enough to clear
outermost part of the Sun’s atmosphere. their orbit of other celestial bodies and moons. For e.g.
Statement 3 is correct. Sunspots are areas that appear Ceres, Pluto, Makemake
dark on the surface of the Sun. Sunspots are temporary Statement c is correct: Exoplanet is a planet like
phenomena on the Sun’s photosphere. They are regions celestial body orbiting a star other than our sun in
of reduced surface temperature caused by concentrations another star system or another galaxy. They are made of
of magnetic field flux that inhibit convection. elements similar to the planets in our solar system but the
Statement 4 is correct. The sun sends out a constant proportion of their mix is varying.
flow of charged particles called the solar wind. The solar Statement d is incorrect: Rogue Planets are free floating
wind travels past all the planets to some three times the planet-like celestial bodies that are not tethered to an
distance to Pluto before being impeded by the interstellar orbit around any star (as is normal), but rather orbit
medium. This forms a giant bubble around the sun and around a galaxy centre.
its planets, known as the heliosphere.
60. Solution (d)
58. Solution (c)
Exp) Option d is correct.
Exp) Option c is correct.
Option a is incorrect. The faintest (less than 1/1000th the
Statement 1 is incorrect. Not all electromagnetic
brightness of the Sun) main sequence stars are called the
radiation reaches the earth’s surface. Radio frequencies,
red dwarfs. They have low luminosity and hence are not
visible light and some ultraviolet light reach up to
visible to the naked eye. They are quite small compared
sea level. The Earth’s atmosphere stops most types of
to the sun and have a surface temperature of about 4000
electromagnetic radiation from space from reaching
degree celcius. According to some estimates, red dwarfs
Earth’s surface.
make up three-quarters of the stars in the Milky Way.
Statement 2 is correct. In an Electromagnetic spectrum, Proxima Centauri is a red dwarf.
as one moves from gamma rays, X rays to radio waves the
Option b is incorrect. A black hole is an object in space
wavelength goes on increasing while the frequency goes
that is so dense and has such strong gravity that no
on decreasing.
matter or light can escape its pull. Because no light can
escape, it is black and invisible. There’s a boundary at the
edge of a black hole called the event horizon, which is
the point of no return — any light or matter that crosses
that boundary is sucked into the black hole. It would need
to travel faster than the speed of light to escape, which
is impossible. Anything that crosses the event horizon is
destined to fall to the very centre of the black hole and be
squished into a single point with infinite density, called
the singularity. While black holes are very massive, that
doesn’t mean they take up a lot of space. Because they’re
so dense, they’re actually quite small.
Option c is incorrect. Neutron stars are formed when
a massive star runs out of fuel and collapses. The very
central region of the star, the core collapses, crushing
together every proton and electron into a neutron. If
the core of the collapsing star is between about 1 and
Statement 3 is incorrect. Radio waves cannot pass 3 solar masses, these newly-created neutrons can stop
through metal and water as both are electrical conductor the collapse, leaving behind a neutron star. (Stars with
and contain free electrons. So, when radio waves hit the higher masses will continue to collapse into stellar-mass

Workbook 40
.
SCIENCE & TECHNOLOGY

black holes.) Since neutron stars began their existence as Statement 3 is correct. They are formed at areas where
stars, they are found scattered throughout the galaxy in magnetic fields are particularly strong. These magnetic
the same places where we find stars. And like stars, they fields are so strong that they keep some of the heat within
can be found by themselves or in binary systems with a the Sun from reaching the surface. These spots are the
companion. visible markers of the Sun’s magnetic field which forms
Option d is correct. A red giant star is a dying star in a blanket that protects the solar system from harmful
the last stages of stellar evolution. In only a few billion cosmic radiation.
years, our own sun will turn into a red giant star. When
63. Solution: (d)
hydrogen fuel at the centre of a star is exhausted, nuclear
reactions will start move outwards into its atmosphere Exp) Option d is correct.
and burn the hydrogen that’s in a shell surrounding the Statement a is incorrect: Light rays scatter much more
core. As a result, the outside of the star starts to expand and repeatedly (not insufficiently) during the hours of
and cool, turning much redder. Over time, the star will sunrise and sunset, as the sun is at or below the horizon
change into a red giant and grow to more than 400 times and sun rays can’t reach the observer directly rather due
its original size. to their low angle, they strike the atmosphere which
scatters these rays’ multiple times before they reach the
61. Solution (d) observer. This repeated scattering (called Rayleigh
Exp) Option d is correct. Scattering) causes loss of energy, and so low frequency
Statement 1 is correct. Scattering of light is the and high wavelength radiation (like red and orange) are
phenomenon of re-emitting of light in all directions by visible, while high energy low frequency radiation (like
the atoms and molecules of different gases that absorbed blue) is absorbed.
the light. Statement b is incorrect: The concentration of pollutants
Applications of Scattering is dependent on many factors other than the time of the
day. So the presence of pollutants is not the sole reason
• Blue colour of the sky
for the scattering of light rays and its subsequent reddish
• Red sky and sun on the time of Sunset or Sunrise hue.
• White colour of clouds Statement c is incorrect: The sun emits the entire
• Tyndall Effect: The phenomenon of scattering of light spectrum of visible light containing all wavelengths at all
by dust, particles, smoke and water droplets suspended times of the day. It is the wavelengths which get absorbed
in air in colloidal form is known as “Tyndall effect” or and those which get reflected from our atmosphere that
“Tyndall Scattering”. change with the time of the day and are responsible for the
Statement 2 is correct. Diffraction is the slight bending of various colours of the sky.
light as it passes around the edge of an object. The amount Statement d is correct: During the time of sunset
of bending depends on the relative size of the wavelength and sunrise, the sun is at or just below the horizon.
of light to the size of the opening. If the opening is much During sunsets, the sun rays are at a very low angle
larger than the light’s wavelength, the bending will be and cannot reach the earth directly. They in fact fall on
almost unnoticeable. However, if the two are closer in the atmosphere, where they get scattered (Rayleigh
size or equal, the amount of bending is considerable, and Scattering) repeatedly. Also, the rays have to travel
easily seen with the naked eye. through a much thicker length of atmosphere to reach,
Statement 3 is correct. Rainbows are the result of the so they lose a lot of their energy. Low energy radiation
refraction and reflection of light. Both refraction and has low frequency and high wavelength (like yellow,
reflection are phenomena that involve a change in a wave’s orange and red). Also, short wavelengths like violet and
direction. A refracted wave may appear “bent”, while blue are scattered the most. So, when the sun rays travel
a reflected wave might seem to “bounce back” from a a long distance at a low angle from the Sun almost at the
surface or other wavefront. horizon, most of the blue light gets scattered away from
Light entering a water droplet is refracted. It is then our eyes and lost. So, the sky appears reddish during
reflected by the back of the droplet. As this reflected light sunrise and sunsets.
leaves the droplet, it is refracted again, at multiple angles.
62. Solution (c)
Exp) Option c is correct.
Statement 1 is incorrect. Sunspots are areas that appear
dark on the surface of the Sun. They appear dark because
they are cooler than other parts of the Sun’s surface.
Statement 2 is incorrect. They are relatively cooler than
the surrounding regions.

41 Workbook
.
SCIENCE & TECHNOLOGY

64. Solution: (c) • The melting of an ice cube is an example of evaporation.


Exp) Option c is correct. • Evaporation of acetone which is used for removing
Magnetism is a physical phenomenon arising from the nail paint is another everyday example of evaporation.
force caused by magnets, objects that produce fields • Distillation is the process in which two mixtures are
raised from motion of electric charges that attract or separated with the help of evaporation as boiling of
repel objects. It is one of the aspects combined with these mixtures is involved.
electromagnetic force. • The concentration of samples in chromatography
Statement 1 is correct. When the unlike poles of a bar are obtained with the help of rotary evaporators and
magnet face each other the magnetic lines of force are in centrifugal evaporators.
the same direction and hence unlike poles attract each • Another everyday example of evaporation is the
other. natural drying of the body after the shower.
Statement 2 is correct. When two magnets are placed • Drying of wet floors is also an example of evaporation.
with their like poles facing each other, the lines of forces
are in opposite directions and hence like poles repel each 67. Solution: (b)
other. Exp) Option b is the correct answer.
65. Solution: (b) Van der Waals forces, relatively weak electric forces that
attract neutral molecules to one another in gases, in
Exp) Option b is correct liquefied and solidified gases, and in almost all organic
Sound is a mechanical wave and needs a material liquids and solids. The forces are named for the Dutch
medium such as a solid, liquid, or gas physicist Johannes Diderik van der Waals, who in 1873
for its propagation. first postulated these intermolecular forces in developing
a theory to account for the properties of real gases.
Sound cause particles of the medium to vibrate parallel
to the direction of wave travel. The vibrations can travel Solids that are held together by van der Waals forces
characteristically have lower melting points and are softer
through solids, liquids or gases. The speed of sound
than those held together by the stronger ionic, covalent,
depends on the medium through which it is travelling.
and metallic bonds.
Option 3 is incorrect A vacuum is an area without any
Van der Waals forces are weak intermolecular forces
air, like space. So, sound cannot travel through space
that are dependent on the distance between atoms or
because there is no matter for the vibrations to work in. molecules. These forces arise from the interactions
Option 1 is correct. Air is a mixture of gases. So, sound between uncharged atoms/molecules.
can travel through it.
An Image below shows the induced formation of a dipole
Option 2 is correct. Water is a liquid. So, sound can in an atom/molecule due to a fluctuating dipole in another
travel through it. atom/molecule. The adsorption of gaseous molecules
Option 4 is correct. Diamond is a solid. So, sound can to the surface of an adsorbent and the cohesion of
travel through it. condensed phases can be accounted for by Van der
Waals forces.
66. Solution: (d)
Exp) Option d is the correct answer. 68. Solution: (d)
Evaporation takes place when the particles of the liquid at Exp) Option d is correct.
surface changes into gaseous phase. Sound is a pressure wave which is created by a vibrating
It is a surface phenomenon as only the liquid particles at object. These vibrations set particles in the surrounding
the surface gain enough energy to break free from the rest medium (typical air) in vibrational motion, thus
of the liquid to escape into a gaseous phase. transporting energy through the medium.
Following are the examples of evaporation in daily life Statement 1 is incorrect. The loudness or softness of a
with explanation: sound is determined by its amplitude. The amplitude of a
sound wave is defined as the maximum displacement of
• Drying of clothes in the sun: The water present in the wave from the equilibrium position. Mathematically, it
the clothes when they are washed is removed by the is the distance between a crest, trough and the equilibrium
process of evaporation. position of a sound wave.
• Crystallization: Crystallization is the process of
The amplitude of the sound wave depends upon the force
obtaining crystals from the “mother” liquid. It takes
with which an object is made to vibrate. If we strike a table
place due to evaporation.
lightly, we hear a soft sound because we produce a sound
• The evaporation process is used in food processing wave of less energy (amplitude). If we hit the table hard we
industries to process milk, pasta, and other fruit juice hear a louder sound. A sound wave spreads out from its
concentrates. source. As it moves away from the source its amplitude as
• Salt crystals obtained either from a natural process or well as its loudness decreases. Louder sound can travel a
from an industrial process, evaporation is used. larger distance as it is associated with higher energy.

Workbook 42
.
SCIENCE & TECHNOLOGY

Statement 2 is incorrect. Fig. 12.10 shows the wave greater amplitude at a specified frequency of operation.
shapes of a loud and a soft sound of the same frequency. Option b is correct: Reverberation is the collection
The number of oscillations of a sound wave per unit time of reflected sounds from the surfaces in an enclosure.
is the frequency of the sound wave. Thus a loud sound or a Reverberation, in acoustics and psychoacoustics, is
soft sound can have same frequency as loudness/softness the persistence of a sound after the sound is made. A
of sound is not dependent on the frequency. reverberation, or reverb, is produced when a sound
or signal is reflected producing a large number of
Wave
distrubance
reflections to form up and then decay as the sound is
absorbed by the objects in the space – which could contain
people and material objects including air.
Time
Option c is incorrect: Echo is the reflection of a sound
wave off a surface. On the other hand, Reverberation is
created by the reflection of sound waves created by the
Soft Sound
superposition of echoes. Echo can be heard in both open
Wave and enclosed space unlike reverberation.
distrubance Option d is incorrect: Diffraction refers to the bending
of waves around small obstacles and the spreading out of
waves beyond such small openings.
72. Solution: (d)
Time
Exp) Option d is correct.
Capillary action is defined as the spontaneous flow of
a liquid into a narrow tube or porous material. This
movement does not require the force of gravity to occur.
Louder Sound
Figure: Soft sound has small amplitude and
In fact, it often acts in opposition to gravity. Capillary
louder has larger amplitude action is sometimes called capillary motion, capillarity, or
wicking.
• Capillary action is caused by the combination of
69. Solution: (c)
cohesive forces of the liquid and the adhesive forces
Exp) Option c is correct. between the liquid and tube material. Cohesion and
The force acting on a unit area of a surface is called adhesion are two types of intermolecular forces.
pressure. These forces pull the liquid into the tube. In order for
Pressure = force / area on which it acts. wicking to occur, a tube needs to be sufficiently small
in diameter.
The area is in the denominator in the above expression.
So, the smaller the area, larger the pressure on a surface • Examples of capillary action include the uptake of
for the same force. This explains why shoulder bags are water in paper and plaster (two porous materials),
provided with broad straps and not thin strap. Broad the quick drying of paint between the hairs of a
straps cover more area and hence apply less pressure paintbrush, and the movement of water through
on the shoulders than thin straps. In other words, force sand.
exerted per unit area by broad straps is less on the • Capillary rise is the height to which the water rises
shoulders than thin straps. within the tube, and decreases as the width of the tube
increases. Thus, the narrower the tube, the water will
70. Solution: (a) rise to a greater height. Capillary action is the same
Exp) Option a is correct effect that causes porous materials, such as sponges,
In summer, the temperature is high and thus the length to soak up liquids.
of metal wire increases significantly (as the temperature 73. Solution: (a)
coefficient of metal wire is high). That’s why the time
period of a pendulum increases and this makes the Exp) Option a is the correct answer.
clock slower whereas in winter the temperature is low Statement 1 is correct: Infrared waves, or infrared
so the metal wire contracts and this causes to decrease light, are part of the electromagnetic spectrum. People
the time period of a pendulum and makes the pendulum encounter Infrared waves every day; the human eye
faster. cannot see it, but humans can detect it as heat. Its
wavelengths longer than those of visible light and hence
71. Solution: (b) invisible to naked eye.
Exp) Option b is the correct answer. Statement 2 is correct: Microwaves are a portion or
Option a is incorrect: Resonance is a physical “band” found at the higher frequency end of the radio
phenomenon in which an external force or a vibrating spectrum, but they are commonly distinguished from
system forces another system around it to vibrate with radio waves because of the technologies used to access

43 Workbook
.
SCIENCE & TECHNOLOGY

them. Microwaves can penetrate through clouds, dust, subatomic particles happen in jumps ( quantum leaps),
smoke, snow, and rain and hence are useful in remote with probabilistic rather than definite outcomes.
sensing. Statement 3 is correct: Quantum mechanics tells us that
Statement 3 is incorrect: Radio waves have the longest light can behave simultaneously as a particle or a wave.
(not shortest) wavelengths in the electromagnetic Quantum theory describes that matter, and light consists
spectrum. Radio waves are a type of electromagnetic of minute particles that have properties of waves that are
radiation best-known for their use in communication associated with them.
technologies, such as television, mobile phones and
radios. 77. Solution: (c)
Exp) Option c is the correct answer.
74. Solution: (a)
Option c is the correct. During a lightning a person is
Exp) Option a is correct. safe in the car. Cars are safe from lightning because of
When you try to flick the card speedily with your finger, the metal cage surrounding the people inside the vehicle.
The inertia of the coin tries to maintain its state of rest This may sound counter-intuitive because metal is a good
even when the card flows off. A momentary force acts conductor of electricity, but the metal cage of a car directs
only on the card and thus the coin fall vertically into the the lightning charge around the vehicle occupants and
glass tumbler due to its inertia. So here Law of Inertia is safely into the ground. The car essentially becomes a
being observed in this situation. Faraday cage and protects anyone inside.
Inertia is the natural tendency of an object to resist a Options a, b and d are incorrect. During a thunderstorm
change in its state of motion or of rest. The mass of an and lightening it is unsafe to lie on concrete floors or
object is a measure of its inertia. leaning on concrete walls. Lightning can travel through
Law of inertia, also called Newton’s first law, postulate in any metal wires or bars in concrete walls or flooring.
physics that, if a body is at rest or moving at a constant During a thunderstorm, never shelter under an isolated
speed in a straight line, it will remain at rest or keep tree and avoid open vehicles such as convertibles,
moving in a straight line at constant speed unless it is motorcycles, and golf carts. Be sure to avoid open
acted upon by a force. structures and stay away from open spaces.
75. Solution: (a)
78. Solution: (b)
Exp) Option a is correct.
Exp) Option b is the correct answer.
Momentum is a conserved quantity, it cannot be created
Reflection is the change in direction of a wavefront
or destroyed (momentum before = momentum after).
at an interface between two different media so that
It can only be transferred between objects. Momentum
the wavefront returns into the medium from which it
is conserved because of Newton’s third law of motion.
originated. For example, we all use mirrors at home. You
When one object exerts a force on a second object for a
look into the mirror and see your own face inside the
certain amount of time, the second object exerts an equal
mirror. What you see is a reflection of your face in the
but oppositely directed force on the first object for exactly
mirror. We also see reflections of other objects that are
the same amount of time. The momentum lost by the
in front of the mirror. Sometimes, we see reflections of
first object is exactly equal to the momentum gained by
trees, buildings and other objects in the water of a pond
the second object. Momentum is transferred from the
or a lake.
first object to the second object.
If a gun exerts a force on a bullet when firing it forward
then the bullet will exert an equal force in the opposite
direction on the gun causing it to move backward or
recoil. Although the action and reaction forces are equal
in size the effect on the gun and the bullet are not the same
since the mass of the gun is far greater than the mass of
the bullet. The acceleration of the bullet while moving
along the gun barrel would be much greater than the
acceleration of the gun. Thus, the conservation of
momentum is demonstrated along with newtons 3rd law
of motion.
76. Solution: (d) 79. Solution: (c)
Exp) Option d is the correct answer. Exp) Option c is the correct answer.
Statement 1 and 2 is correct: In general relativity, events Statement 1 is correct: The weight of an object differs on
are continuous and deterministic, meaning that every poles and equator. There are two reasons for this
cause matches up to a specific, local effect. In quantum a) Earth is not perfectly spherical. It has less radius on
mechanics, events produced by the interaction of poles and More on equator. So due to less radius on

Workbook 44
.
SCIENCE & TECHNOLOGY

poles the gravity experienced is more as the surface is one substance and a negative charge to the other. Charges
closer to the centre of earth compared to the equator remain on the surfaces of the products until they can flow
- if radius of earth increase, the gravitational force on or discharge. Whenever there is friction between materials
body decreases. that differ in their ability to give up or gain electrons, the
b) Earth is spinning with more angular velocity at electrons can be transferred in this manner, i.e. through
equator and it decrease as we move towards the poles. friction.
This increased angular velocity at equator produces a Statement 2 is incorrect . Whenever the two objects are
force a outward centrifugal force which decreases the rubbed, the objects acquire equal and opposite charges.
effect of g. The object that loses electrons becomes positively charged.
Statement 2 is correct: The weight of an object is the force The other that gains electrons becomes negatively charged.
that it exerts on the surface on which it is kept. When a Charging by conduction involves the contact of a
person is normally standing on earth, his weight will be charged object to a neutral object. Suppose that a
where m is mass, g is the acceleration due to gravity. Thus positively charged aluminum plate is touched to a neutral
weight, W= mass(m) x gravity(g) ie W=mg metal sphere. The neutral metal sphere becomes charged
When the g will change, or due to some other factor the as the result of being contacted by the charged aluminium
normal force by person changes, the weight will change, plate.
though the mass will remain the same. The changed Statement 3 is correct. As the objects acquire equal
weight is often termed as apparent weight. and opposite charges and an attractive force develops
Apparent Weight is when a body is moving with constant between them. When a piece of amber, plastic, polythene,
acceleration upward or downward, then there is a change or hard rubber is rubbed with fur, electrons are transferred
in weight is observed due to force of gravity acting on an from fur to the other material. Fur acquires net positive
object is not balanced by an equal and opposite normal charge, since it has fewer electrons than protons. Similarly,
force. This new weight may be less or more than the the amber, plastic, or hard rubber acquires a net negative
actual/true weight. charge since they have excess electrons.

80. Solution: (a) 82. Solution: (b)


Exp) Option a is the correct answer. Exp) Option b is correct.
When abnormal refraction increases the apparent elevation Statement 1 is incorrect. If the buoyant force is greater
of distant objects — often lifting above the horizon things than the object’s weight, the object rises to the surface and
normally below it — the process is described as ‘looming.’ floats. If the buoyant force is less than the object’s weight,
Because we associate a certain apparent altitude with a the object sinks. If the buoyant force equals the object’s
certain distance, this phenomenon generally makes the weight, the object can remain suspended at its present
objects seem nearer than they really are. depth.
It is due to total internal reflection. In cold evening over Statement 2 is correct. The object sinks in water because
seabed sea water becomes too cold. Air layer in its contact of the difference in their densities. The density of a
is cold and denser. As we go up air layers become less and substance is defined as the mass per unit volume. When
less cold and hence rarer. Rays from invisible ship going the density of object is less than the density of water
upward move from denser to rarer air layers. They are which means that the upthrust of water on the object or
totally reflected downwards and received by an observer the buoyant force is greater than the weight of the object.
at sea-shore. The observer sees an virtual image of a ship So it floats.
hanging in the sky. And when the density of an object is more than the
density of water. This means that the upthrust of water on
the object is less than the weight of the object. So it sinks.
83. Solution: (b)
Exp) Option b is correct.
When the light falls on the surface of the objects it may
be transmitted, absorbed or reflected. Based on the
ability to transmit light, objects can be broadly classified
into three types- Translucent (allow part of light to pass);
Transparent (allow light to pass) and Opaque objects (do
not allow light to pass).
81. Solution: (c)
Statement 1 is incorrect. Translucent substances allow a
Exp) Option c is correct. part of the light to pass through, e.g. ground glass, oiled
Statement 1 is incorrect. If two objects are rubbed paper, etc. Whereas luminous objects are the ones that
together, both get charged but not due to the process of emits light. Objects generally both reflect and refract
convection, but the frictions developed between them light rays at the same time but in different proportions.
allows electrons to travel. This gives a positive charge to Opaque objects reflect most of the light rays, translucent

45 Workbook
.
SCIENCE & TECHNOLOGY

objects refract (and not reflect) most of the light rays. we will find that the potential will be zero for no charge
Statement 2 is correct. If an opaque or translucent body because I = 0. Therefore, there is no flow of electric charges
is placed in the path of light spreading from a light source even between two charged particles due to the presence of
(luminous objects) then on the other side of that light- the same potential in them.
blocking body will form a black figure like it, which is 86. Solution: (b)
called its shadow. While a translucent material makes a
faint shadow and opaque body makes a dark shadow. Exp) Option b is correct
Frictional force is generated when two surfaces are either
84. Solution: (b) contacting or sliding against each other.
Exp) Option b is correct. Frictional force depends upon the force impelling them
The ignition temperature is the lowest temperature at together and to the surface texture.
which the substance can catch fire. Ignition temperature When she stops pedaling, the bicycle begins to slow down.
is an important property of any fuel because the The reason is friction starts acting opposite to direction of
combustion reaction of the fuel becomes self-sustaining motion which leads to slowing down the bicycle.
only above this temperature. It is generally lower The friction force of the ground is an external force which
for higher volatile matter content fuel. Therefore, opposes the motion of the bicycle.
the combustion temperature of petrol is low while
the combustion temperature of coal is high. Ignition In order to keep the bicycle moving, She have to start
temperature, however, is not necessarily a unique pedalling again.
property of a fuel because it depends on several other It thus appears that an object maintains its motion under
factors like oxygen, partial pressure, particle size, rate of the continuous application of an unbalanced force.
heating, and a particle’s thermal surroundings.
87. Solution: (c)
Statement a is incorrect. The flash point of a chemical
substance is the lowest temperature where enough fluid Exp) Option c is correct
can evaporate to form a combustible concentration Pitch is defined as the characteristic of sound which is
of gas. The flash point is an indication of how easy a used for differentiating between the shrill and flat sound.
chemical may burn. Materials with higher flash points are Statement 1 is correct: Loudness depends on the
less flammable or hazardous than chemicals with lower amplitude of the sound wave. It is based on the energy
flash points. of wavelength and is measured in decibel. The frequency
Statement c is incorrect. The lowest temperature at which determines the shrillness or pitch of a sound. If the
a volatile combustible substance continues to burn in air frequency of vibration is higher, we say that the sound is
after its vapours have been ignited (as when heating is shrill and has a higher pitch. If the frequency of vibration
continued after the flash point has been determined) is is lower, we say that the sound has a lower pitch.
known as fire point. Statement 2 is correct: As said in the first statement, pitch
Statement d is incorrect. The Adiabatic flame is not about loudness. Lion can make a low-pitched
temperature is the temperature that results from a roar which is highly audible whereas sound of bird is
complete combustion process that occurs without any high pitched but feeble. For example, a drum vibrates
work, heat transfer or changes in kinetic or potential with a low frequency. Therefore, it produces a low-pitched
energy. The Adiabatic Flame temperature depends on the sound. On the other hand, a whistle has a high frequency
heating value of the fuel, the temperature and amount of and therefore, produces a sound of higher pitch
oxidizer, and final combustion product composition.
88. Solution: (c)
85. Solution: (b)
Exp) Option c is correct
Exp) Option b is correct
Inertia is the characteristic of an object which resists
If two charged bodies having equal potential are connected changes to the state of the motion of the object. It is an
through a conducting wire, then the Current will not flow. intrinsic characteristic of the object which is related to its
The flow of current is defined as the rate of flow of electric mass which tells us how much force it will take to create
charge. When the electric charge flows in one direction,
the particular acceleration on the object. Newton’s First
the current flows in the opposite direction. The current
law of motion defines the term “inertia” which states that
only flows when there is a potential difference between
the object remains in steady state or in motion until any
two bodies.
external force is applied to it.
For the flow of electric current in any substance, there
must be the presence of two different potentials in Luggage on the top of the bus is a loose fixture and not a
that substance. This is because the tendency of the compact part of the bus.
flow of electric charge is from higher potential to Option a is correct. When some luggage is put on the
the lower potential. Since, the question is about zero roof of a bus that is initially at rest, the acceleration of
flow of charges, this can only happen if there is no the bus in the forward direction will exert a force (in
potential diference in both the conducting bodies. the backward direction) on the luggage. As the luggage
As we know that the potential is measured as V = IR so continues in the same state because of inertia, and

Workbook 46
.
SCIENCE & TECHNOLOGY

hence is left behind relative to the bus, such that it falls Option d is correct: The principle of impulse and the
backward. conservation of momentum explains the problems
Option b is correct. When the bus stops, the luggage associated with a recoil of a gun. Recoil of a gun is a case
tends to resist the change and due to inertia of motion it of conservation of linear momentum. Before firing, the
moves forward and may fall off. gun is loaded with the bullet and both are in a state of
Hence, To avoid the falling of luggage, it is tied with a rest. The initial velocity of bullet as well as that of the gun
rope. is zero. Hence, the initial total momentum of the system
is zero. Upon firing, the bullet moves in one direction
89. Solution: (a) and exerts force while the gun moves in the opposite
Exp) Option a is correct. direction. By principle of conservation of momentum,
An echo is a sound that is repeated because the sound the momentum of the bullet will be equal and opposite
waves are reflected back. to that of the gun, because the total final momentum of
the system shall also be zero. Thus, in order to conserve
Statement 1 is correct. To hear a distinct echo the time the linear momentum a gun always recoils when a bullet
interval between the original sound and the reflected one
is fired.
must be at least 0.1s. It means that the sound must go to
the obstacle and reach back the ear of the listener on 92. Solution: (a)
reflection after 0.1s.
Exp) Option a is correct.
Statement 2 is correct. The total distance covered by
the sound from the point of generation to the reflecting Pair 1 is correct: A phenomenon called Rayleigh’s
surface and back should be at least (344 m/s) × 0.1 s = Scattering causes light to scatter when it passes
34.4 m. Thus, for hearing distinct echoes, the minimum through particles that have a diameter one-tenth that
distance of the obstacle from the source of sound must be of the wavelength (colour) of light. Sunlight is made up
half of this distance, that is, 17.2 m. of different colours, but because of the elements in the
Statement 3 is incorrect. Higher temperatures lower atmosphere, the colour blue is scattered more efficiently
the density of a medium, which puts less resistance on than other colours making the sky appear blue. The
the sound waves and thus echo is heard sooner. Echo is molecules of air and other fine particles in the atmosphere
heard sooner in hot temperatures as compared to cold have size smaller than the wavelength of visible light. These
temperatures. are more effective in scattering light of shorter wavelengths
at the blue end than light of longer wavelengths at the red
90. Solution: (d) end. The red light has a wavelength about 1.8 times greater
Exp) Option d is correct. than blue light. Thus, when sunlight passes through the
Superconducting materials show zero electrical atmosphere, the fine particles in air scatter the blue
resistance at low temperatures. It allows them to conduct colour (shorter wavelengths) more strongly than red. If
‘supercurrents’ without heat dissipation. the earth had no atmosphere, there would not have been
any scattering. Then, the sky would have looked dark.
Option d is correct. A superconductor is a material
that achieves superconductivity. It is a state of matter The sky appears dark to passengers flying at very high
that has no electrical resistance and does not allow altitudes, as scattering is not prominent at such heights.
magnetic fields to penetrate. An electric current in a Pair 2 is incorrect: The twinkling of a star is due
superconductor can persist indefinitely. Superconductivity to atmospheric refraction (and not atmospheric
can only typically be achieved at very low temperatures. reflection) of starlight. The starlight, on entering the
Superconductors have a wide variety of everyday earth’s atmosphere, undergoes refraction continuously
applications, from MRI machines to super-fast maglev before it reaches the earth. The atmospheric refraction
trains that use magnets to levitate the trains off the track occurs in a medium of gradually changing refractive
to reduce friction. index. Since the atmosphere bends starlight towards
Superconducting electromagnets can be used in maglev the normal, the apparent position of the star is slightly
trains, experimental nuclear fusion reactors and high- different from its actual position. The star appears slightly
energy particle accelerator laboratories. Superconductors higher (above) than its actual position when viewed
can also be used to power railguns and coilguns, cell phone near the horizon. Further, this apparent position of the
base stations, fast digital circuits and particle detectors. star is not stationary, but keeps on changing slightly,
since the physical conditions of the earth’s atmosphere
91. Solution: (d) are not stationary. Since the stars are very distant, they
Exp) Option d is correct. approximate point-sized sources of light. As the path
Newton’s Third Law of Motion states that whenever one of rays of light coming from the star goes on varying
object exerts a force on another object, the second object slightly, the apparent position of the star fluctuates and
exerts an equal and opposite on the first. Thereby, this law the amount of starlight entering the eye flickers – the
is manifested in the form of the principle of impulse and star sometimes appears brighter, and at some other time,
the conservation of momentum. fainter, which is the twinkling effect.

47 Workbook
.
SCIENCE & TECHNOLOGY

• Fermions include electrons, protons, and neutrons


while bosons include photons.
• The wave function which describes a collection of
fermions must be antisymmetric with respect to
the exchange of identical particles, while the wave
function for a collection of bosons is symmetric (and
not vice-versa).
• Fermions are particles which have half-integer spin
while bosons are particles which have integer spin.
The energy distribution of bosons is described by
Bose-Einstein statistics.
• At low temperatures, bosons can behave very
differently than fermions because an unlimited
number of them can collect into the same energy
state. The collection into a single state is called
condensation, or Bose-Einstein condensation. It is
responsible for the phenomenon of superfluidity in
Pair 3 is correct: A rainbow is a natural spectrum
liquid helium.
appearing in the sky after a rain shower. It is caused by
dispersion of sunlight by tiny water droplets, present • Coupled particles can also act effectively as bosons.
in the atmosphere. A rainbow is always formed in a In the Bardeen–Cooper–Schrieffer (BCS) theory of
direction opposite to that of the Sun. The water droplets superconductivity, coupled pairs of electrons act like
act like small prisms. They refract and disperse the bosons and condense into a state which demonstrates
incident sunlight, then reflect it internally, and finally zero electrical resistance.
refract it again when it comes out of the raindrop. Due
94. Solution: (b)
to the dispersion of light and internal reflection, different
colours reach the observer’s eye. Exp) Option b is correct.
Pair 4 is incorrect: A mirage is an optical phenomenon Option b is correct: The roads are bent inwards on the
that creates the illusion and results from the refraction curves and at the turns because the bends avoid moving
of light through a non-uniform medium. Mirages are object like vehicles to fall outside of the roads and
most commonly observed on sunny days. On hot summer prevent the accidents. Thus, it helps in keeping the vehicle
days, the air near the ground becomes hotter than the on road. When an object turns in a circle, it is influenced
air at higher levels. The refractive index of air increases by centrifugal force which pushes it away from the
with its density. Hotter air is less dense, and has smaller centre of the circle. Thus, the roads are bent inwards to
refractive index than the cooler air. If the air currents are provide the necessary centripetal force (force required
small, that is, the air is still, the optical density at different for circular motion). The centripetal force pulls inward
layers of air increases with height. As a result, light from a and ensures the car to have a curved path. The design of a
tall object such as a tree, passes through a medium whose curved track has an inclination to the horizontal because
refractive index decreases towards the ground. Thus, a there is an extra force to enhance the magnitude of the
ray of light from such an object successively bends away centripetal force.
from the normal and undergoes total internal reflection, if Also, according to Newton’s first law of motion, an object
the angle of incidence for the air near the ground exceeds maintains its state unless a force acts on it. Therefore, a
the critical angle. To a distant observer, the light appears moving car does not change its direction and keeps its
to be coming from somewhere below the ground. This speed unless a force acts on it. A car can turn along the
phenomenon is called mirage. Example: It is especially corner on a flat road if there is a frictional force which
common in hot deserts. While moving in a bus or a provides the necessary centripetal force for a circular
car during a hot summer day, a distant patch of road, motion. That is why roads are bend inwards on curve.
especially on a highway, appears to be wet. But you do not
find any evidence of wetness when you reach that spot. 1.3. General Chemistry
The appearance of the water is simply an illusion.
95. Solution (d)
93. Solution: (c)
Exp) Option d is correct.
Exp) Option c is correct.
Statement 1 is correct. Hydrogen can be generated from
Option c is correct: Particles come in two types: Fermions renewable energy resources. Depending on the nature of
and Bosons. the method of its extraction, hydrogen is categorised into
• The particles that make up matter are known as three categories, namely, grey, blue and green. Hydrogen
‘fermions’, and the particles that carry forces are produced from fossil fuels is called grey hydrogen; this
known as ‘bosons’. constitutes the bulk of the hydrogen produced today.

Workbook 48
.
SCIENCE & TECHNOLOGY

Hydrogen generated from fossil fuels with carbon capture are formed from methane (CH4). Sudden release
and storage options is called blue hydrogen; hydrogen of pressurized methane gas may cause submarine
generated entirely from renewable power sources is landslides, which in turn can trigger tsunamis.
called green hydrogen. On Earth, gas hydrates occur naturally in some marine
Green hydrogen has specific advantages. One, it is a sediments and within and beneath permafrost. They are
clean burning molecule, which can decarbonise a range also speculated to form on other planets.
of sectors including iron and steel, chemicals, and Statement 3 is incorrect. A recent study conducted
transportation. Two, renewable energy that cannot be by researchers at the Agharkar Research Institute (an
stored or used by the grid can be channelled to produce autonomous institute of the Department of Science and
hydrogen. Technology) have found that the methane hydrate
Statement 2 is correct. Hydrogen is two-three times deposits are located in the Krishna-Godavari (KG)
more efficient than burning petrol. The energy in 1 basin. These deposits are of biogenic origin. The
kilogram of hydrogen gas is about the same as the energy methane hydrate deposit in this basin is a rich source
in 2.8 kilograms of petrol. that will ensure adequate supplies of methane, a natural
Statement 3 is correct. Hydrogen is one of the cleanest gas.
fuels, which on being burnt in air produces only water
98. Solution (b)
as a by-product and no carbon-based emissions are
released, thereby leading to zero vehicular emissions. Exp) Option b is correct.
Statement 1 is correct. Vanadium alloys are used in
96. Solution (c) nuclear reactors because of vanadium’s low neutron-
Exp) Option c is correct. absorbing properties. Vanadium pentoxide is used as a
Statement 1 is correct. Scientist from Jawaharlal Nehru catalyst for the production of sulfuric acid. The vanadium
Centre for Advanced Scientific Research (JNCASR), redox battery for energy storage may be an important
Bengaluru, an autonomous institution of the Department application in the future.
of Science & Technology (DST), have identified a new Statement 2 is correct. Vanadium is mainly used to
lead-free material called Cadmium (Cd) doped Silver produce specialty steel alloys such as high-speed tool
Antimony Telluride (AgSbTe2). The material can steels, and some aluminium alloys. Thus, Vanadium is
efficiently allow recovery of electricity from ‘waste heat’ used in the manufacturing of steel alloys.
marking a paradigm shift in the thermoelectric puzzle. Statement 3 is incorrect. The largest deposits of vanadium
It can efficiently convert waste heat to power our small of the world are in China, followed by Russia and South
home equipment and automobiles. Africa. China, which produces 57% of the world’s
Statement 2 is correct. Cadmium (Cd) doped Silver vanadium consumed 44% of the metal in 2017. India is a
Antimony Telluride (AgSbTe2) is a lead-free material significant consumer of vanadium, but is not a primary
and thus has the potential to be used for mass market producer of the strategic metal. India consumed 4% of
applications. Thermoelectric energy conversion allows about 84,000 tonnes of vanadium produced across the
generation of electrical voltage when one end of a globe in 2017.
material is heated while keeping the other side cold. The
efficient material to realize this needs three seemingly 99. Solution (b)
different properties into a single material-- high electrical Exp) Option b is correct.
conductivity of metals, high thermoelectric sensitivity
of semiconductors, and low thermal conductivity Styrene was the gas responsible for the Visakhapatnam
of glasses. Most efficient thermoelectric materials gas leak, an industrial accident that occurred at
developed by scientists so far use Lead (Pb) as a major the LG Polymers chemical plant on the outskirts of
constituent element, restricting their use for mass-market Visakhapatnam.
applications. But with this material being lead free, mass Statement 1 is incorrect. Styrene is a colourless and
market application is feasible. inflammable gas. Inhaling it for a short period of time
causes irritation in the eye, respiratory problems,
97. Solution (b) nausea, unsteady gait, loss of consciousness and
Exp) Option b is correct. gastrointestinal effects. Also, studies have shown that it
Statement 1 is correct. Gas hydrates could be dangerous produces lung cancer in rodents.
as their decomposition can release large amounts of Statement 2 is correct. Styrene is the main raw material
methane, which is a greenhouse gas that could aid to used for synthesis of polystyrene. It is also used in the
global warming and impact Earth’s climate. manufacturing of fiberglass, rubber, and latex. It is also
Statement 2 is correct. Gas hydrates are ice-like crystalline found in vehicle exhaust, cigarette smoke, and in natural
minerals that form when low molecular weight gas (such foods like fruits and vegetables.
as methane, ethane, or carbon dioxide) combines with
water and freezes into a solid under low temperature 100. Solution (d)
and moderate pressure conditions. Most gas hydrates Exp) Option d is correct.

49 Workbook
.
SCIENCE & TECHNOLOGY

Hallmarking is the accurate determination and official set standards and inspect industrial units. Two rules have
recording of the proportionate content of precious metal been notified under the Environment (Protection)
in precious metal articles. Hallmarks are thus official Act, 1986 for ensuring chemical safety, namely, the
marks used in India as a guarantee of purity or fineness of Manufacture, Storage and Import of Hazardous Chemicals
precious metal articles. Rules, 1989 (MSIHC) and the Chemical Accidents
In India, at present two precious metals namely gold (Emergency, Planning, Preparedness, and Response)
and silver have been brought under the purview of Rules, 1996 (EPPR).
Hallmarking. Statement 2 is correct. Public Liability Insurance Act,
Gold Hallmarked Jewellery consists of following four 1991 mandates hazardous units to procure an insurance
marks: policy and deposit an equal amount in the Environment
Relief Fund to provide immediate relief to victims of
BIS Purity in carat Assay centre’s Jewellers chemical accidents.
Mark and fineness for identification identifica- Statement 3 is correct. The National Environment
gold (e.g.22K916, mark / num- tion mark/ Appellate Authority Act, 1997 establishes the National
18K750, ber number Environment Appellate Authority. It can hear appeals
regarding the restriction of areas in which any industries,
14K585 )
operations or processes shall or shall not be carried out
101. Solution (a) subject to certain safeguards under EPA, 1986.
Exp) Option a is correct. 104. Solution (c)
Statement 1 is correct. Scientist discovered a gas called Exp) Option c is correct.
phosphine in Venus’ atmosphere that indicates microbes
Statement 1 is incorrect. OPCW is an intergovernmental
may inhabit the planet. Phosphine gas is considered a
organisation and the implementing body for the Chemical
biomarker of life. A molecule of phosphine gas consists of
Weapons Convention. This convention is in force since
a phosphorus atom surrounded by three hydrogen atoms,
1997 and 193 members. OPCW is not a United Nations
just like ammonia consists of a nitrogen atom surrounded
(UN) organization.
by three hydrogen atoms.
Statement 2 is correct. The objectives of the organisation
It is also produced by some anaerobic bacteria, which live
are:
in oxygen-sparse environments such as sewers, landfills,
or even animal guts. If you can rule out the production of 1. Destroying all existing chemical weapons under
the gas through chemistry, it is the biochemical processes international verification.
that form a source of the gas — the anaerobic bacteria — 2. Monitoring chemical industry to prevent chemical
hence it is considered a biomarker in astronomy. weapons from re-emerging.
Statement 2 is incorrect. Phosphine can be produced 3. Providing assistance and protection to States
through industrial processes. Apart from being Parties against chemical threats.
produced in industrial processes, phosphine, a 4. Fostering international cooperation to strengthen
colourless, smelly gas, is known to be made only by some implementation of the Convention and promote the
species of bacteria that survive in the absence of oxygen. peaceful use of chemistry.
Statement 3 is correct. The OPCW is authorized to
102. Solution (c) perform inspections to verify that signatory states are
Exp) Option c is correct. complying with the convention. It can also perform
Statement 1 is correct. Aerosol is a term used to broadly testing of sites and victims of suspected chemical
refer to particles suspended in the air; they could include weapons attacks.
fine dust, mist, or smoke. In the context of transmission Statement 4 is correct. By the 2001 Relationship
of viruses, aerosols are read as micro droplets, much Agreement between the OPCW and the UN, the OPCW
smaller (5 microns or less) than respiratory droplets. reports on its inspections and other activities to the UN
Statement 2 is correct. Aerosols will be expelled by people through the office of the Secretary-General.
breathing, laughing or singing, as against respiratory
105. Solution (b)
droplets that are expelled with forceful acts such as
sneezing or coughing. Exp) Option b is correct
Aerosols take a longer time to drop to the floor and The term allotrope refers to one or more forms of a
remain suspended in the air for longer periods than chemical element that occur in the same physical state.
respiratory droplets. The different forms arise from the different ways atoms
may be bonded together. The concept of allotropes was
103. Solution (d) proposed by Swedish scientist Jons Jakob Berzelius in
Exp) Option d is correct. 1841. The ability for elements to exist in this way is called
Statement 1 is correct. The Environment Protection allotropism.
Act, 1986, gives powers to the central government to Allotropes may display very different chemical and
undertake measures for improving the environment and physical properties. For example, graphite and diamond

Workbook 50
.
SCIENCE & TECHNOLOGY

are both allotropes of carbon that occur in the solid state. Syngas or synthetic gas, is a gas mixture consisting
Graphite is soft, while diamond is extremely hard. primarily of hydrogen (H2), carbon monoxide (CO)
Fullerene from another class of carbon allotropes. and very often some carbon dioxide (CO2). Syngas is a
building block in chemical industry, which corresponds
Isotopes are atoms with the same number of protons
to 2% of the total primary energy consumption.
(atomic mass) but differing numbers of neutrons. Isobars
are atoms of different chemical elements having equal Syngas is usually a product of gasification and can be
values for atomic mass. Isotopes have the same atomic produced from many sources, including natural gas, coal
number. Isobars have different atomic numbers. and biomass, by reaction with steam or oxygen.
Example - Carbon-12 and carbon-14 are isotopes because The syngas can be used in a variety of applications such
they have same atomic number-12 but different mass as in the production of electricity and making chemical
number-12 and 14 respectively. products such as fertilisers. The hydrogen obtained from
(Mass number is equal to the number of total protons and coal gasification can be used for various purposes such
neutron in an atom). as making ammonia, powering a hydrogen economy or
upgrading fossil fuels. Methane or natural gas extracted
An example of a series of isobars would be 40S, 40Cl, 40Ar, from coal gasification can be converted into LNG for
40
K, and 40Ca. direct use as fuel in the transport sector.
Isotones are atomic species that share the same number of 109. Solution (d)
neutrons, and differ in the number of protons. Examples of
isotones include carbon-12, nitrogen-13 and oxygen-14. Exp) Option d is correct.
These atoms all have six neutrons and six, seven and eight Statement 1 is correct.
protons respectively. Formalin is a chemical derived from formaldehyde, a
106. Solution (c) 37% aqueous (water) solution of formaldehyde, a pungent
gas, with the chemical formula HCHO, used as an
Exp) Option c is correct antiseptic, disinfectant, and especially today as a fixative
Statement 1 is correct. Photovoltaic (PV) solar panels for histology (the study of tissues under the microscope).
are made up of many solar cells. Solar cells are made of Statement 2 is correct. In excess quantities and if
silicon, like semiconductors. They are constructed with consumed over a long period of time, it has been found
a positive layer and a negative layer, which together create to cause cancer.
an electric field, just like in a battery.
Statement 3 is also correct. It is used to increase shelf
Statement 2 is correct. Solar photovoltaic (SPV) cells life of food, especially that of fishes by fishing vessels in
convert solar radiation (sunlight) into electricity, which the sea.
can be used to power equipment or to recharge a battery.
Statement 3 is incorrect. Photovoltaic cells generate clean 110. Solution (a)
and green energy as they do not emit greenhouse gases. Exp) Ossption a is correct.
The operation and maintenance cost of cells are very
Blue litmus paper turns red under acidic conditions
low. The cost of solar panel incurred is only the initial cost
and red litmus paper turns blue under basic or alkaline
i.e., purchase and installation.
conditions.
107. Solution (c) Since pH of rainwater is less than 7 which corresponds to
Exp) Option c is correct. acidic solutions, it will turn the blue litmus paper to red.
When rubbers bands are twisted and untwisted, Pure water has pH=7 and hence it will have no effect on
it produces a cooling effect. This is called the the colour of the litmus paper.
“elastocaloric” effect. Milk of magnesia is a basic solution with pH>7 and hence
Researchers from multiple universities, including Nankai it will also not react with the litmus paper.
University in China, have found that the elastocaloric Acidic solutions are those that have pH value less than 7
effect, if harnessed, may be able to do away with the need and basic solutions or bases have pH more than 7.
of fluid refrigerants used in fridges and air-conditioners
111. Solution (d)
In the elastocaloric effect, the transfer of heat works much
the same way as when fluid refrigerants are compressed Exp) Option d is correct.
and expanded. Cloud seeding is a way to artificially tweak rain. It is also
When a rubber band is stretched, it absorbs heat from its known by other terms such as man-made precipitation
environment, and when it is released, it gradually cools enhancement, artificial weather modification, rainmaking
down. and so on.
The technology sprays particles of salts like silver iodide
108. Solution (c) and chloride on clouds using a special aircraft, rockets or
Exp) Option c is correct. from dispersion devices located on the ground.

51 Workbook
.
SCIENCE & TECHNOLOGY

These salt particles act as a core (cloud condensation and linen in the textile industry, for bleaching wood
nuclei or ice-nucleating particles) which draw water pulp in paper factories and for bleaching washed clothes
vapour within the cloud towards them and the moisture in laundry; (ii) as an oxidising agent in many chemical
latches on, condensing into water droplets leading to the industries; and (iii) for disinfecting drinking water to
formation of raindrops. make it free of germs.
The goal of cloud seeding is to alter the natural 114. Solution (a)
development of the cloud to enhance precipitation,
suppress hail, dissipate fog, or reduce lightning. Exp) Option a is correct.
The most common chemicals used for cloud seeding Thorium is a weakly radioactive metallic chemical
include silver iodide, potassium iodide and dry ice element. Thorium has high thermal conductivity and
(solid carbon dioxide). Liquid propane, which expands higher melting point.
into a gas, has also been used. Statement 1 is correct. Nearly 25 per cent of the world’s
thorium ore is available in India. India’s reserves of
thorium are at least three times larger than its uranium
reserves. India’s thorium deposits, estimated at 360,000
tonnes, far outweigh its natural uranium deposits at
70,000 tonnes.
Statement 2 is incorrect. Thorium itself will not split and
release energy. Thorium is fertile rather than fissile,
which means reactions can be stopped when necessary.
It can only be used as a fuel in conjunction with a fissile
material such as recycled plutonium. It needs to undergo
transmutation to U-233 in a reactor fuelled by other
fissile material. It produces waste products that are less
radioactive, and generates more energy per ton.
115. Solution (b)
Exp) Option b is correct.
Statement 1 is correct. Human exposure to elevated
levels of inorganic arsenic occurs mainly through the
112. Solution (b) consumption of groundwater containing naturally high
Exp) Option b is correct levels of inorganic arsenic. Food prepared with this water
and food crops irrigated with high-arsenic water sources
In chemistry, deposition refers to the process in which also causes exposure of Humans to arsenic. Arsenic is
a gas changes directly to a solid without going through highly toxic in its inorganic form.
the liquid state. Examples of deposition in nature include
frost forming on the ground and cirrus clouds forming Statement 2 is correct. Organic arsenic compounds,
high in the atmosphere. which are abundant in seafood, are less harmful to
health and are rapidly eliminated by the body.
Option a refers to the process called Sublimation.
Example is evaporation of camphor and dry ice directly Statement 3 is incorrect. Arsenic is tasteless, colorless,
to gas. and odorless and thus testing is needed for detection.
Sublimation and deposition chemistry are examples of Statement 4 is correct. The new rice variety, Muktoshri
— also called IET 21845 is resistant to arsenic and is
phase changes.
developed West Bengal’s Agriculture Department and
Option c refers to the process of Crystallisation. the National Botanical Research Institute, Lucknow.
Option d refers to the process of Calcification in soil, i.e.,
formation of calcium in soils. 116. Solution (b)
Exp) Option b is correct.
113. Solution (d)
As altitude increases and atmospheric pressure decreases,
Exp) Option d is correct the boiling point of water decreases. There’s more air
All the three statements are correct. above a point at sea level, which means the atmospheric
Chlorine gas is used for the manufacture of bleaching pressure is greater here than at higher altitudes where
powder. Bleaching powder is produced by the action there’s less air bearing down above. At a higher elevation,
of chlorine on dry slaked lime [Ca(OH)2 ]. Bleaching the lower atmospheric pressure means heated water
powder is represented as CaOCl2 , though the actual reaches its boiling point more quickly. i.e., at a lower
composition is quite complex. temperature. Water at sea level boils at 212 degrees
Fahrenheit; at 5,000 feet above sea level, the boiling point
Ca(OH)2 + Cl2 CaOCl2 + H2O is 203 degrees F. Up at 10,000 feet, water boils at 194
Bleaching powder is used – (i) for bleaching cotton degrees F.

Workbook 52
.
SCIENCE & TECHNOLOGY

117. Solution (a) 119. Solution (c)


Exp) Option a is correct. Exp) Option c is correct.
Pair 1 is correctly matched: Starch is the polysaccharide Statement a is correct. Most of the atoms do not exist
synthesized by plants as the main storage carbohydrate independently but, the atoms of noble gases which are
source. It occurs in grains, fruits. etc. It is a polymeric highly inert can exist independently. For example, Helium
carbohydrate consisting of numerous glucose units joined (He), Neon (Neon) and Argon (Ar).
by glycosidic bonds. This polysaccharide is produced by Statement b is correct. Compounds composed of metals
most green plants for energy storage. Worldwide, it is and nonmetals contain charged species. The charged
the most common carbohydrate in human diets, and is species are known as ions. An ion is a charged particle
contained in large amounts in staple foods like potatoes, and can be negatively or positively charged. A negatively
maize (corn), rice, wheat and cassava (manioc). charged ion is called an ‘anion’ and the positively charged
Pair 2 is incorrectly matched: Glycogen is an energy ion, a ‘cation’. A group of atoms carrying a charge is known
storage polymer found in animal cells and bacteria. It as a polyatomic ion.
is broken down into glucose when needed. It is present Statement c is incorrect. As per Daltons atomic theory
in the form of granules in the cytosol of the cell. On atoms are indestructible and retain their identity
hydrolysis it forms glucose. in all chemical reactions. This is basically the Law of
Pair 3 is incorrectly matched: Cellulose is a structural Conservation of Matter, stating that an equal amount of
polymer found in plants. Cellulose is the important mass exists before and after a reaction.
component of plant cell wall; cellulose occurs in the form Statement d is correct. Atoms which have imbalanced
of fibers. It is the most abundant organic compound on outer shells show high reactivity. Imbalanced outer
the earth, it is also used in making paper, artificial threads shells mean low number of electrons (one or two) or a
and strings. higher but not full number (six or seven). The most highly
reactive elements are halogens, which are looking to gain
118. Solution (d) one more electron, and alkali metals, which are looking to
Exp) Option d is correct. lose the single electron in their outer shells
Graphene is a one-atom-thick sheet of carbon atoms 120. Solution (b)
arranged in a honeycomb-like pattern. Graphene is
Exp) Option b is correct.
considered to be the world’s thinnest, strongest and
most conductive material - of both electricity and heat. Statement 1 is incorrect. Ions can be positively charged
Graphene has the potential to revolutionize entire or negatively charged. Atoms that lose electrons become
industries - in the fields of electricity, conductivity, energy cations and that gain electron becomes anions.
generation, batteries, sensors and more. Statement 2 is correct. Ions can be formed by breaking
Statement 1 is correct. Carbon nanotubes (CNTs) are up of covalent bond between two atoms. It can also be
cylindrical molecules that consist of rolled-up sheets formed by the addition of electrons to, or the removal of
of single-layer carbon atoms (graphene). Carbon electrons from, neutral atoms or molecules or other ions
Nanotubes can be used as carriers of drugs and antigens or by combination of ions with other particles.
in the human body. The main applications of CNTs in Statement 3 is correct. Ions make up only about 1% by
pharmacy and medicine include drug, biomolecule, weight of human plasma. However, they are the major
gene delivery to cells or organs, tissue regeneration, and contributors to plasma molarity, since their molecular
biosensor diagnostics and analysis. weights are much less than those of proteins. Thus, ions
Statement 2 is correct. Graphene is the world’s strongest are important in preventing blood cells from bursting
by taking up excess water in osmosis. Sodium chloride
material, and can be used to enhance the strength of
(NaCl) constitutes more than 65% of the plasma ions.
other materials.
Statement 3 is correct. Graphene is the most heat 121. Solution (d)
conductive material found till date. As graphene is also Exp) Option d is correct.
strong and light, it means that it is a great material for
Statement a is correct. Oxytocin also known as the
making heat-spreading solutions, such as heat sinks or ‘love hormone’ is a hormone secreted by the pituitary
heat dissipation films. glands during sex, childbirth, lactation or social bonding.
Statement 4 is correct. Being world’s thinnest material Oxytocin helps contract the uterus and induce delivery,
with extremely high surface-area to volume ratio, makes control bleeding, and promote the release of breast milk.
graphene a very promising material for use in batteries Its use is especially crucial to prevent new mothers from
and supercapacitors. Graphene may enable batteries excessively bleeding after giving birth which is a common
and supercapacitors (and even fuel-cells) to store more cause of maternal deaths.
energy - and charge faster, too. Statement b is correct. Oxytocin can also be chemically
Statement 5 is correct. Graphene has a lot of promise for manufactured. It is administered either as an injection or
applications like anti-corrosion coatings and paints. a nasal solution.

53 Workbook
.
SCIENCE & TECHNOLOGY

Statement c is correct. Oxytocin was used in the dairy 124. Solution: (c)
industry where livestock is injected with Oxytocin to Exp) Option c is correct
increase milk secretion and production.
Acid Rain and the pH Scale
Statement d is incorrect. The Ministry of Family and
Health Welfare restricted the manufacture of Oxytocin The pH scale measures how acidic an object is. Objects
formulations for domestic use to public sector only. that are not very acidic are called basic. The scale has
It banned sell of oxytocin by private retail chemists. values ranging from zero (the most acidic) to 14 (the most
It has also banned the import of Oxytocin and its basic).
formulations. Government has banned private firms from
manufacturing and selling oxytocin.
122. Solution (d)
Exp) Option d is correct.
Statement 1 is correct. Manufacturing and processing
of nylon is energy-intensive, which causes emission of
greenhouse gases leading to global warming. Nylon tops
the list of synthetic materials that have the highest impact
on the environment, according to the Pulse of Fashion
Report, 2017.
Statement 2 is correct. The production process of Nylon
releases nitrous oxide, a greenhouse gas 300 times more
potent than carbon dioxide, which depletes the ozone.
Also, Waste water generated during the production of
nylon contains the unreacted monomer, caprolactam,
which is polluting. Its untreated discharge through factory
wastewater causes harm to a range of aquatic organisms. As you can see from the pH scale above, pure water has a
pH value of 7. This value is considered neutral—neither
Statement 3 is correct. Improper disposal of nylon
acidic nor basic.
products also leads to accumulation of microplastic
in the aquatic ecosystem. Even if properly disposed, One easy way that you can measure pH is with a strip of
microscopic pieces of fiber slowly break down and litmus paper.
contribute to marine pollution. When you touch a strip of litmus paper to something, the
Conventional nylon is non-biodegradable; it remains on paper changes color depending on whether the substance
the earth for hundreds of years, either in a landfill or an is acidic or basic. If the paper turns red, the substance is
ocean. acidic, and if it turns blue, the substance is basic.
Known for its strength, durability and elasticity, nylon 125. Solution: (d)
was the world’s first entirely synthetic polymer fibre
Exp) Option d is correct.
marketed to women in 1938. Its unique selling point was
the longevity of stockings as compared to that of silk and Option d is correct: The material used for wrapping food
rayon. items should be flexible, foldable, and non-reactive with
the food. In this case, aluminium is a soft and malleable
123. Solution: (a) metal. The property of metals by which they can be
Exp) Option a is correct. beaten into thin sheets is called malleability. It can be
easily beaten into thin sheets to form wrapping foils and
Dark surfaces absorb more heat whereas light-coloured
also does not react with food items. Hence, aluminium
surfaces reflect most of the heat that falls on them.
foils are used to wrap food items.
Therefore, the black-coloured tin can will absorb more
heat of the Sun and thus water in it will be warmer than The property of metal by which it can be drawn into wires
the water in the white-coloured tin can. is called ductility.
Black is the ultimate heat absorber. It absorbs all light on Mercury is the only metal that exists in liquid state at
the visual spectrum, creating a void of light. As a result room temperature.
of absorbing all light wavelengths, black is the hottest Non-metals are brittle and therefore, cannot be rolled into
possible colour. White is the opposite. wires or pounded into sheets.
White light is the sum of all wavelengths, so when some 126. Solution: (c)
people view a white object, they are really viewing all
visible light hitting the object’s surface and reflecting back. Exp) Option c is correct.
Some heat is still absorbed based on the nature of the The law of conservation of mass states that matter
object’s material, but minimal additional heat is absorbed, cannot be created or destroyed but can change forms in a
making white the coolest possible colour. chemical reaction.

Workbook 54
.
SCIENCE & TECHNOLOGY

According to the law of conservation of mass, the mass of universally to treat typhoid fever from 1948 until the
the reactants must be equal to the mass of the products for 1970s, when widespread resistance occurred. Ampicillin
a low energy thermodynamic process. and trimethoprim-sulfamethoxazole (TMP-SMZ) then
Burning and other changes in matter do not destroy became treatments of choice .
matter.
129. Solution: (a)
For example, when wood burns, the mass of the soot,
Exp) Option a is correct.
ashes, and gases equals the original mass of the wood and
the oxygen when it first reacted. Heavy Water (D2O), discovered by Dr. H. C. Urey in 1932,
earned him a Nobel Prize but he could not have imagined
Law of Conservation of Mass Examples
the enormous interest this molecule would generate in
127. Solution: (d) industry, biology and medicine. Heavy water (D2O) is
Exp) Option d is correct. also called as deuterium oxide. It is water composed of
deuterium, the hydrogen isotope with a mass double that
The atomic number of an atom is defined as the total of ordinary hydrogen, and oxygen. (Ordinary water has a
number of protons present in the nucleus of an atom. composition represented by H2O.)
The mass number of an atom is defined as the total Statement 1 is correct: Heavy water (D2O) exhibits about
number of protons and neutrons present in the nucleus 2600 times’ neutron-moderating ability compared to
of an atom. H2O. Heavy water is extensively used as a moderator
Statement 1 is correct and 2 is correct. Isotopes are in nuclear reactors. Heavy water can be prepared by
defined as the atoms of the same element, having the exhaustive electrolysis of water or as a by-product in some
same atomic number but different mass numbers. There fertilizer industries.
are three isotopes of hydrogen atom, namely protium, Statement 2 is correct: A unique property of Heavy
deuterium and tritium. The chemical properties of water relates to the enhancement of thermal stability
isotopes are similar but their physical properties are of macromolecules, cells and tissues. This property
different.
has been utilized to enhance the thermo stability
Allotropes are different forms of the same element. This of certain vaccines e.g., oral polio vaccine or other
happens due to the different bonding of the atoms which macromolecules which need freezing temperature. All
are arranged in sequence, thus forming a new structure. of Enterovirus Research Centre, an ICMR Institution
All allotropes have the same element. So, the proton and at Mumbai in collaboration with the Heavy Water
neutron numbers are the same and so the atomic mass is Board (HWB) have improved the process and a distinct
also the same. advantage of D2O based OPV has been demonstrated.
Statement 3 is correct. Allotropes of Carbon are Thus, Oral Polio Vaccine remains biologically active
diamond and graphite. They are formed by carbon atoms even if the cold chain is disturbed for a while. This
but the difference lies in the manner in which the technology has enormous potential in tropical countries
carbon atoms are bonded to one another. In diamond, and can serve as a support system for cold chain in far
each carbon atom is bonded to four other carbon atoms remote areas where it is difficult to maintain the desired
forming a rigid three-dimensional structure. In graphite, temperature. Likewise, several tissues/organs e.g., kidneys
each carbon atom is bonded to three other carbon atoms and other biological material can be preserved without
in the same plane giving a hexagonal array. degradation or decay.
128. Solution: (a) Statement 3 is incorrect: Heavy water (D2O) is relatively
more viscous than H2O and has higher melting and
Exp) Option a is correct. boiling points. Heavy Water exhibits different chemical
Ant venom contains irritants and toxins inflicted by ants. and physical properties compared to normal water due
Most ants spray or inject venom, the main constituent of to difference in the zero-point energy arising out of the
which is formic acid. isotope effects. The isotope effect gives rise to difference
Option a is correct. When an ant bites, it injects the in bond energies and this leads to the different chemical
acidic liquid called formic acid into the skin. The effect of behaviour of heavy water. Due to the difference in bond
the acid can be neutralised by rubbing moist baking soda energy D2O exhibit different deuterium bonding effects
or sodium hydrogen carbonate or calamine solution, also (hydrogen bonding in case of normal water) giving
which contains zinc carbonate. rise to changes in composite molecular structure and
behaviour.
Option b is incorrect. Hydrogen peroxide is a topical
antiseptic used in wound cleaning which kills pathogens 130. Solution: (a)
through oxidation burst and local oxygen production. Exp) Option a is correct
Option c is incorrect. Tranquilizer is a drug that is used Acids have a pH below 7. The more H+ ions, the more
to reduce anxiety, fear, tension, agitation, and related acidic it is and the lower the pH will be. Bases have a pH
states of mental disturbance. above 7. pH 7 is said to be neutral – this means there is a
Option d is incorrect. Chloramphenicol was used balance of H+ and OH- ions.

55 Workbook
.
SCIENCE & TECHNOLOGY

Statement 1 is incorrect. An acid is a substance that hydrocarbon gases, most commonly propane, butane,
donates hydrogen ions. Because of this, in an acidic and propylene. However, the latter two typically
solution there are more hydrogen ions than hydroxide comprise 5% or less of the mixture. LPG is used as
ions. This means the more acidic a food is, the more fuel gas in heating appliances, cooking equipment,
hydrogen ions are available to trigger the sour taste and vehicles.
receptors. Bases are bitter to taste. A base is a substance 3. The Calorific value of CNG is 50000 kJ/kg. However,
that accepts hydrogen ions. When a base is dissolved in that of biogas is 35000-40000 kJ/kg. CNG, is an auto
water, the balance between hydrogen ions and hydroxide fuel in Gaseous state. Mainly comprising of Methane
ions shifts the opposite way so the hydroxide ions (80% to 90%) CNG is low density and is compressed
dominate. up to 200 bar pressure so that it can be stored in a
Statement 2 is correct. Acids change blue litmus to red larger capacity in the fuel tank. Hence, it is named
as the pigment in blue litmus reacts with H+ ions and Compressed Natural Gas
changes chemically so the bonds are ‘tuned’ to reflect a 4. The Calorific value (CV) of Kerosene is 45000
longer wavelength of light to appear red to our eyes. The kJ/kg. It is equal to Diesel and Petrol. Kerosene is
pigment is absorbing the blue to green wavelengths when typically pale yellow or colourless and has a not-
it reacts with the free hydrogen ions in acid. While, bases unpleasant characteristic odour. It is obtained from
changes red litmus to blue because the same pigment petroleum and is used for burning in kerosene lamps
in base reacts differently with OH− ions and becomes and domestic heaters or furnaces, as a fuel or fuel
‘tuned’ to reflect the short wavelength to appear blue. component for jet engines, and as a solvent for greases
Now it absorbs the green and red wavelengths having and insecticides.
reacted with the hydroxyl ions in the base.
133. Solution: (c)
131. Solution: (a)
Exp) Option c is the correct answer.
Exp) Option a is correct.
Pasteurization is defined as the process of heating
Whenever the appearance of an object changes but the products to a particular temperature and holding it at
substance remains the same as it was before, this change is that temperature for a particular time till the pathogenic
called physical change. Conversely, when the appearance (disease causing) micro-organisms are destroyed causing
of an object changes or does not change, but the substance minimum change in composition, flavor and nutritive
changes, that change is called chemical change. value of products such as milk.
Statements 1 and 2 are correct. The process of depositing Statement 1 is incorrect. Homogenization is the process
a layer of zinc on iron is called galvanisation. It provides in which fat globules in milk are broken down to a size
it with several advantages like- rust resistance; low initial small enough to prevent the formation of a cream layer.
cost; longer life; sacrificial anode keeping steel safe etc.
When milk is properly homogenized, the cream will
Crystallisation, is the process of atoms or molecules not rise to the top. The process involves forcing the
arranging into a well-defined, rigid crystal lattice in order milk through small openings under high pressure, thus
to minimize their energetic state. breaking up the fat globules.
Statement 3 is incorrect. Both crystallisation and
Cream and other food products, such as peanut butter,
galvanisation are physical changes. Galvanisation is a
may be homogenized to produce a stable emulsion—one
physical change as there is only deposition of zinc on
in which fats or oils will not separate from other elements.
the surface without forming any new substance. While in
crystallisation crystals of salt or any matter is obtained by Statement 2 is correct. Pasteurization is a process in
evaporation of liquid only. Hence a physical change. which packaged and non-packaged foods (such as milk
and fruit juices) are treated with mild heat, usually to
132. Solution: (c) less than 100 °C. Milk’s boiling point is nearly close
Exp) Option c is correct to that of water, 100 degrees Centigrade (212 degrees
Fahrenheit). It means under pasteurization, the milk
The amount of heat energy produced on complete
is boiled below its boiling point to eliminate pathogens
combustion of 1 kg of a fuel is called its calorific value.
and extend shelf life.
The calorific value of a fuel is expressed in a unit called
kilojoule per kg (kJ/kg). The correct order of the above There are two methods of pasteurization (of milk) in
fuels in decreasing order of their calorific value is - general use. One is low temperature holding (LTH)
1. The Calorific value of Hydrogen is 150000 kJ/kg. method in which milk is heated to 62.8°C (145F) for 30
Hydrogen is a clean fuel that, when consumed in minutes in commercial pasteurizers (or) large closed vats
a fuel cell, produces only water. Hydrogen can be which are heated by steam coils, hot water jackets etc.
produced from a variety of domestic resources, such The other method (i.e.) high temperature short time
as natural gas, nuclear power, biomass, and renewable (HTST) method in which the milk is heated to 71.7°C
power like solar and wind. (161F) for 15 seconds.
2. The Calorific value of LPG is 55000 kJ/kg. Statement 3 is correct. Pasteurization only destroys the
Liquefied petroleum gas, is a flammable mixture of vegetative forms of the bacteria. Pasteurization kills

Workbook 56
.
SCIENCE & TECHNOLOGY

harmful organisms responsible for such diseases as While prolonged exposure to boiling hydrochloric acid
listeriosis, typhoid fever, tuberculosis, diphtheria, Q destroys the fibres, and 96% sulfuric acid and causes
fever, and brucellosis. Pasteurization conditions are not disintegration of the fibres. Whereas, Nylon has good
sufficient to destroy thermo-resistant spores (reproductive resistance to hot acids.
part of microorganisms). Thus, Pasteurization does not
sterilize the products but kills only those organisms that 136. Solution: (b)
grow most readily at low temperatures. The surviving Exp) Option b is correct.
organisms must be kept from multiplying by constant Dry Ice is the common name for solid carbon dioxide
refrigeration. (CO2) which does not melt into a liquid when heated;
134. Solution: (a) instead it changes directly into a gas.
Exp) Option a is correct. Statement 1 is correct. Dry ice is made by liquefying
carbon dioxide and injecting it into a holding tank, where
Sodium hydroxide (NaOH) also called as Caustic Soda or
it’s frozen at a temperature of -109° F or –78 °C (freezing
Iye is an inorganic corrosive white crystalline solid that
readily absorbs moisture until it dissolves. point) and compressed into solid ice.
Statement 1 is correct. In the energy sector, sodium Statement 2 is incorrect. Dry ice sublimes or passes
hydroxide is used in fuel cell production as conventional directly to the vapour state. Hence there is no moisture
Alkaline Fuel Cell uses aqueous KOH/NaOH solution as and waterlogging in the insulation of the storage space.
the liquid electrolyte to avoid water loss from the alkaline Statement 3 is correct. Dry ice can help cancer patients
electrolyte solution undergoing chemotherapy treatment. Patients have
Statement 2 is correct. Sodium hydroxide is used to extract found the use of super cooled “cold caps” to be an
alumina from naturally occurring minerals(bauxite) at effective way to reduce or prevent hair loss. Super cooled
high temperatures and pressures using Bayer process. caps (-20 F to -30F) are worn before, during and after the
This alumina is used to make aluminium and a variety of actual intravenous chemotherapy treatment.
products including foil, cans, kitchen utensils, beer kegs
and airplane parts. 137. Solution: (a)
Statement 3 is correct. Sodium hydroxide solutions are Exp) Option a is correct
used in petroleum refining to remove hydrogen sulfide The reactivity series of metals is a chart listing metals
(H2S) and mercaptans from various hydrocarbon in order of decreasing reactivity. In general, the more
streams. The resulting sulfide-laden waste stream is called reactive a metal is: the more vigorously it reacts with
spent-sulfidic caustic. other substances, the more easily it loses electrons to
Statement 4 is incorrect. Glass is made from natural and form positive ions
abundant raw materials which include sand, soda ash Statement 1 is correct. Almost all metals combine with
(Sodium Carbonate) and limestone that are melted at very oxygen to form metal oxides. For example, when copper
high temperature to form a new material(glass). is heated in air, it combines with oxygen to form copper
(II) oxide, a black oxide. However, Different metals show
135. Solution: (a) different reactivities towards oxygen.
Exp) Option a is correct. Statement 2 is incorrect. Metals react with water and
Synthetic fibres are categorized as man-made fibers produce a metal oxide and hydrogen gas. Metal oxides
produced by using different types of chemicals under that are soluble in water dissolve in it to further form
controlled conditions. Examples of synthetic fibres are metal hydroxide. But all metals do not react with water.
nylon and polyester. Metals like aluminium, iron and zinc do not react either
Statement 1 is correct. Nylon is a polyamide polymer with cold or hot water. But they react with steam to
characterized by the presence of amide groups (CO–NH) form the metal oxide and hydrogen. Metals such as lead,
in the main polymer chain. While Polyester is made up of copper, silver and gold do not react with water at all.
the repeating units of a chemical called an ester. Esters are
the chemicals which give fruits their smell. 138. Solution: (a)
Statement 2 is correct. Nylon is an example of Exp) Option a is correct
thermoplastic (substances that gets deformed easily Alkali metals are the chemical elements found in Group
on heating) as it can be melted and processed into 1 of the periodic table. The alkali metals include: lithium,
fibers on heating. Whereas Polyesters can occur in both sodium, potassium, rubidium, caesium, and francium.
thermoplastic and thermoset (plastics which when Option a is correct. Potassium and Sodium are highly
moulded once cannot be softened by heating) versions as reactive metals. If kept in open air, they react with
they can also retain their form and stay solid under heat oxygen and catches fire. The reaction is very vigorous.
once cured. Both of these metals are in Group 1 on the periodic table.
Statement 3 is incorrect. Polyesters have good resistance Both of these do not react with Kerosene. Therefore they
to acids in cold conditions and even at room temperature. are kept in kerosene.

57 Workbook
.
SCIENCE & TECHNOLOGY

Option b is incorrect. Sodium and Potassium are so Oxidation reactions are reactions in which atoms of the
soft that they can be cut with a knife. They have low elements involved in the reaction lose electrons.
densities and low melting points. However, they don’t Statement 1 is correct. Food material cooked/fried in
shine normally. So, there is no question of losing shine. oil/fat when left for a long time smells unpleasant and
Option c is incorrect. At ordinary temperature, the tastes bad because the fats and oils of food items undergo
surfaces of metals such as magnesium, aluminium, oxidation and become rancid. This change is called
zinc, lead, etc., are covered with a thin layer of oxide. rancidity.
The protective oxide layer prevents the metal from further Statement 2 is correct. Cops often use breathalysers or
oxidation. This is more prominent in Aluminium and breath analysers to estimate the content of alcohol in a
protects it from getting corroded. It is not the reason for driver’s breath. The breathalysers employ the chemical
immersing sodium and potassium in kerosene. principle of redox reactions (in which oxidation and
Option d is incorrect. Sodium and Potassium cannot be reduction both takes place) where ethanol gets oxidized as
kept in open air or even in presence of water vapor. So, it loses electrons (H atoms) and the potassium dichromate
the question of rusting will not appear. Also, rusting gets reduced as it picks up some electrons.
is a phenomenon generally associated with long time, Statement 3 is incorrect. Hydrogenation (an addition
and it is unlikely for that to be the reason for storage reaction) is a process in which unsaturated hydrocarbons
in kerosene. (i.e. alkenes and alkynes) are converted to alkanes by the
139. Solution: (c) addition of hydrogen in the presence of a catalyst. This
process is used in preparation of vegetable ghee from
Exp) Option c is correct. edible oils by converting unsaturated fats to saturated
Nitrogen(N2) is a diatomic gas that is colorless, odorless ones.
gas and is insoluble in water.
141. Solution: (b)
Statement a is incorrect. The regular air has tendency
Exp) Option b is the correct answer.
to seep out of microscopic pores in the tires, this will
cause a decreasing inflation pressure. When air gets Permanent Hardness is due to the presence of soluble salts
pressurized, the moisture in it turns into a liquid. of magnesium and calcium in the form of chlorides and
When this air is pumped into your tire, this liquid will sulphates in water. Temporary hardness is a type of water
come with it as well. When you are driving, your tire hardness caused by the presence of dissolved bicarbonate
will heat up which will turn the liquid into a gas. This minerals (calcium bicarbonate and magnesium
bicarbonate).
makes it expand which increases the tire pressure.
Option 1 is correct: Washing soda can remove
Statement b is incorrect. The Volumetric energy
temporary and permanent hardness from water.
density refers to the amount of energy that can be Sodium carbonate is soluble but calcium carbonate and
contained within a given volume. Increasing the magnesium carbonate are insoluble. The water is softened
volumetric energy density of batteries allows electric because it no longer contains dissolved calcium ions and
vehicles (EVs) to travel further without increasing the magnesium ions.
size of the battery pack.
Option 2 is incorrect: Treatment with lime removes out
Statement c is correct Nitrogen is drier in nature calcium carbonate and magnesium hydroxide which can
(less water and oxygen content) than compressed be filtered off and thus remove temporary hardness not
air which not only helps regulate the tyre pressure permanent hardness.
and temperature on hot afternoons and extremely Option 3 is correct: Ion-exchange method is also called
long drives but also causes lesser rubber damage and zeolite/permutit process. Hydrated sodium aluminium
corrosion of the metal tyre rims. Since, Compressed silicate is zeolite/permutit. When this is added in hard
air has more water and oxygen which reacts quickly water, exchange reactions take place. Permutit/zeolite is
with the changing temperature in the tyres, leaving said to be exhausted when all the sodium in it is used up. It
more chances of tyre blowouts in summers which is is regenerated for further use by treating with an aqueous
not the case with nitrogen. sodium chloride solution.
Statement d is incorrect. Nitrogen molecules are larger Option 4 is correct: Hard water is softened by using
than air molecules which means it moves through synthetic cation exchangers. This method is more efficient
tires at a slower rate and can offer more consistent air than zeolite process. Cation exchange resins contain
pressure. It is also an unreactive gas because it has a large organic molecule with - SO3H group and are water
triple covalent bond between the nitrogen atoms in N insoluble. Ion exchange resin (RSO3H) is changed to RNa
molecules that makes it less prone to binding up with by treating it with NaCl. The resin exchanges Na + ions
2
other atoms. with Ca 2+ and Mg 2+ ions present in hard water to make
the water soft.
140. Solution: (b) Option 5 is incorrect: Permanent hardness is not
Exp) Option b is correct. removed by boiling. Boiling removes magnesium and

Workbook 58
.
SCIENCE & TECHNOLOGY

calcium hydrogen carbonates and thus remove temporary broken down to form glucose. This glucose combines with
hardness. oxygen in the cells of our body and provides energy. This
reaction is respiration. So, respiration is an exothermic
142. Solution: (a) process.
Exp) Option a is the correct answer. Statement 5 is incorrect: Evaporating liquid water is an
Gypsum composed of hydrated calcium sulphate endothermic process as water molecules must absorb
(CaSO4·2H2O). Natural gypsum applications are divided heat from the surroundings to increase their kinetic
into general applications and their deep processing energy.
applications. The general applications of natural gypsum
mainly include: construction, art, ceramics, food 144. Solution: (b)
additives, pharmaceuticals, and medical gypsum boards. Exp) Option b is the correct answer.
Statement 1 is correct: In food, gypsum can coagulate Option a is incorrect: Aqua regia is a mixture of
soybean milk into tofu, and can also be used as a concentrated hydrochloric acid (HCl) and concentrated
coagulant in canned tomatoes. Gypsum can also be used nitric acid (HNO3) at a ratio of either 3:1 or 4:1. It is a
as an animal feed additive
reddish-orange or yellowish-orange fuming liquid.
Statement 2 is correct: Gypsum can be used as medicine.
Option b is correct: Aqua regia has ability to dissolve the
It is widely used in traditional Chinese medicine. It is
noble metals gold, platinum, and palladium.
also called fine stone and water cold stone in Chinese
medicine. It is a heat-clearing and purging gunpowder. It Glassware may also be washed with aqua regia to remove
mainly treats high fever, polydipsia, lung heat, cough, organic compounds only in trace amounts. But aqua regia
and stomach fire. Pain and other symptoms; hemihydrate will not dissolve all noble metals. For example, iridium
gypsum is mainly used for external use, which can and tantalum are not dissolved.
cause muscle sores. It can be taken alone, but is usually Option c is incorrect:
employed with herbs such as indigo, coptis, ephedra and
anemarrhea. Aqua regia solutions are extremely corrosive. It will melt
some plastics and corrode most metals.
Statement 3 is incorrect: Gypsum has a very low thermal
conductivity. Thus, it cannot be used in heat sink Aqua regia is generally not stored. Aqua regia quickly
applications as the materials for heat sink applications loses its effectiveness due to oxidation of its reactive
should have high thermal conductivity in order to components. The solution could build up pressure and
absorb more heat energy. Aluminium alloys are the most explode.
common heat sink material. Option d is incorrect:
143. Solution: (b) Aqua regia is not used in the manufacturing of car
Exp) Option b is the correct answer. batteries. Sulphuric acid is used in car batteries.
Reactions in which heat is released along with the
formation of products are called exothermic chemical 1.4. General Biology
reactions.
145. Solution (a)
Endothermic reactions are chemical reactions in which
the reactants absorb heat energy from the surroundings Exp) Option a is correct.
to form products. These reactions create a cooling effect. Statements 1 and 3 are correct. Influenza A virus has
Statement 1 is correct: The decomposition of vegetable so many strains because of constant mutations that it
matter into compost is also an example of an exothermic undergoes. Reassortment is one of the reasons behind
reaction. the constant mutation of the viruses. Reassortment is the
process by which influenza viruses swap gene segments.
Statement 2 is correct: Burning of natural gas i. e. CH4(g)
+ 2O2 (g) → CO2 (g) + 2H2O is an exothermic process. This genetic exchange is possible due to the segmented
nature of the viral genome and occurs when two differing
Statement 3 is incorrect: Decomposition of calcium influenza viruses co-infect a cell.
carbonate is an endothermic reaction because it needs
energy to decompose into carbon dioxide and calcium When a cell happens to be infected with two different
oxide. Decomposition of calcium carbonate to calcium flu A viruses, their genes can easily get mixed up. This
oxide and carbon dioxide on heating is an important mixing, known as reassortment, is a viral version of sex.
decomposition reaction used in various industries. Statement 2 is incorrect. Another reason behind Influenza
Calcium oxide is called lime or quick lime. When a A virus mutating constantly is: It is an RNA virus with
decomposition reaction is carried out by heating, it is a segmented genome, i.e. it has eight separate strands,
called thermal decomposition. which makes its copying prone to errors or mutations.
Statement 4 is correct: During digestion, food is broken This ‘antigenic drift’ results in slight but continuous
down into simpler substances. For example, rice, potatoes mutations in the surface proteins, which is the reason why
and bread contain carbohydrates. These carbohydrates are flu vaccines have to be updated regularly.

59 Workbook
.
SCIENCE & TECHNOLOGY

146. Solution (d) having shorter life span mutate and evolve faster.
Exp) Option b is correct. Statement 2 is correct. 2’-OH group present at every
An immune reaction triggered by the body to fight an nucleotide in RNA is a reactive group and makes RNA
infection is known as a cytokine storm when it turns labile and easily degradable. RNA is also now known to
severe. Cytokines are signaling proteins that are be catalytic, hence reactive. Therefore, DNA chemically
released by cells. During a cytokine storm, the body is less reactive and structurally more stable when
releases too many cytokines into the blood too quickly. compared to RNA.
While normally they regulate immune responses, in 150. Solution (c)
this case they cause harm and can even cause death. A
cytokine storm is characterized by the overproduction Exp) Option c is correct.
of immune cells which can be harmful since an excess of The scientists have discovered a new strain of swine flu
immune cells can attack healthy tissues as well. virus (H1N1) from pigs in China, which has been named
G4. The new strain, also called G4 EA H1N1, is genetically
147. Solution (c) descended from the H1N1 strain that caused a pandemic
Exp) Option c is correct. in 2009.
Single-cell proteins (SCP) refers to edible unicellular It was identified through surveillance of influenza viruses
microorganisms. They are increasingly used as protein in pigs through a study carried out from 2011 to 2018
supplements in human foods. But there are some in ten provinces of China. Scientist have found that this
limitations in their adoption on a large scale. strain of influenza virus is infecting Chinese pigs and has
the potential of triggering a pandemic. The experiments
Statement 1 is correct. Fast growing microorganisms
indicate the transmission of G4 virus from animals to
such as bacteria and yeast have a high concentration
humans. But there is no evidence yet that it can be passed
of nucleic acid. Ingestion of such biomass leads to from human to human.
increased plasma levels of uric acid, which can cause
gastrointestinal problems or kidney stones. Also similar 151. Solution (a)
to plant cells, the cell wall of some microorganisms such Exp) Option a is correct.
as algae and yeast contain non-digestible components,
such as cellulose. Statement 1 is correct. Kala azar is one of the most
dangerous neglected tropical diseases (NTDs). Kala
Statement 2 is incorrect. Cultivation of microorganisms azar is endemic in 76 countries with approximately 200
has a much lower carbon footprint than agricultural million people at risk of infection. Kala-azar or visceral
food production. Also, cultivation of microorganisms leishmaniasis (VL) is a tropical disease characterised by
does not require fertile soil and therefore does not irregular fever, weight loss, anaemia and swelling of the
compete with agriculture. spleen and liver. It is caused by a protozoan Leishmania
Statement 3 is correct. Some microorganisms may parasite.
produce toxic substances such as mycotoxins or Statement 2 is incorrect. India accounts for about two-
cyanotoxins. Also, some yeast and fungal proteins tend thirds of the total global cases, and the disease is endemic
to be deficient in methionine which is an essential amino to Bihar, Jharkhand, Uttar Pradesh and West Bengal.
acid in humans.
Statement 3 is correct. It is transmitted to humans by
148. Solution (b) the bite of infected female sand-flies. According to the
World Health Organisation (WHO), globally, about 7 to
Exp) Option b is correct.
10 lakh new cases occur annually.
Statement 1 is incorrect. Cord blood is the blood that is
left in the umbilical cord and placenta after the birth of 152. Solution (b)
a child. Cord blood banking is the process of collecting Exp) Option b is correct.
the cord blood and extracting and cryogenically freezing
Only those organisms who has ability to reproduce are
its stem cells and other cells of the immune system for considered as living beings. Bacteria, algae and fungi
potential future medical use. are all living beings as they can reproduce. However,
Statement 2 is correct. Cord blood contains special type viruses on the other hand isn’t considered “alive”
of cells called hematopoietic stem cells that can be used by scientists because they lack many of the properties
to treat certain diseases. Cord blood can be collected, associated with living organisms. Primarily, they lack the
and stem cells can be extracted and stored for potential ability to reproduce without the aid of the host cell and
future medical use. do not use a typical cell division approach to replication.
149. Solution (b) Bacteria are microscopic, single-celled organisms
(prokaryotes) that thrive in diverse environments. These
Exp) Option b is correct. organisms can live in soil, the ocean and inside the human
Statement 1 is incorrect. Both DNA and RNA are able gut. Sometimes bacteria lend us a helping hand, such as by
to mutate. In fact, RNA being unstable, mutate at a faster curdling milk into yogurt or helping with our digestion.
rate. Consequently, viruses having RNA genome and In other cases, bacteria are destructive, causing diseases

Workbook 60
.
SCIENCE & TECHNOLOGY

like pneumonia and methicillin-resistant Staphylococcus


aureus (MRSA).
A fungus is any member of the group of eukaryotic
organisms that includes microorganisms such as yeasts
and molds, as well as the more familiar mushrooms. Fungi
are a group of living organisms which are classified in their
own kingdom. This means they are not animals, plants,
or bacteria. Fungi, once considered plant-like organisms,
are more closely related to animals than plants. Fungi
are not capable of photosynthesis: they are heterotrophic
because they use complex organic compounds as sources
of energy and carbon. 154. Solution (a)
Algae are a diverse group of aquatic eukaryotic organisms Exp) Option a is correct.
that have the ability to conduct photosynthesis. Certain Relevance) Human genetics and biotechnology forms
algae are familiar to most people; for instance, seaweeds important area in prelims.
(such as kelp or phytoplankton), pond scum or the Statement 1 is correct. DNA is the chemical name
algal blooms in lakes. They can range in size from the for the molecule that carries genetic instructions in all
microscopic (microalgae), to large seaweeds (macroalgae), living things. The DNA molecule consists of two strands
that wind around one another to form a shape known
such as giant kelp more than one hundred feet in length.
as a double helix. Each strand has a backbone made of
A virus is the smallest type of parasite to exist, that cannot alternating sugar (deoxyribose) and phosphate groups.
reproduce by itself. Once it infects a susceptible cell, It is made up of 23 pairs of chromosomes and provides
however, a virus can direct the cell machinery to produce instructions for building an entire organism and the
proteins. The information in DNA is stored as a code
more viruses. Most viruses have either RNA or DNA as made up of four chemical bases: adenine (A), guanine
their genetic material. The nucleic acid may be single- (G), cytosine (C), and thymine (T).
or double-stranded. The entire infectious virus particle, Statement 2 is correct. Nuclear DNA can be inherited
called a virion, consists of the nucleic acid and an outer from both parents while mitochondrial DNA is inherited
shell of protein. only from the mother. Also, nuclear DNA has lower
mutation rate than mitochondrial DNA.
153. Solution (c) Statement 3 is incorrect. Most viruses have either RNA
Exp) Option c is correct. or DNA as their genetic material. The nucleic acid may
be single- or double-stranded. The entire infectious virus
Plant cells have a cell wall, as well as a cell membrane. particle, called a virion, consists of the nucleic acid and an
In plants, the cell wall surrounds the cell membrane. This outer shell of protein. The simplest viruses contain only
gives the plant cell its unique rectangular shape. Animal enough RNA or DNA to encode four proteins.
cells simply have a cell membrane, but no cell wall.
155. Solution (d)
Mitochondria and Ribosomes are present in both plant Exp) option d is correct
and animal cells. Statement 1 is correct. Vitamin A is the name of a group
Mitochondria are membrane-bound cell organelles of fat-soluble retinoids, including retinol, retinal, and
(mitochondrion, singular) that generate most of the retinyl esters. Concentrations of preformed vitamin A are
chemical energy needed to power the cell’s biochemical highest in liver and fish oils. Other sources of preformed
vitamin A are milk and eggs.
reactions. Chemical energy produced by the mitochondria
Statement 2 is correct. vitamin A deficiency can also lead
is stored in a small molecule called adenosine triphosphate
to night blindness. Vitamin A, also called retinol, plays
(ATP). a role in transforming nerve impulses into images in the
Ribosomes are minute particles consisting of RNA and retina.
associated proteins that function to synthesize proteins. Statement 3 is correct. Vitamin A is involved in
Proteins are needed for many cellular functions such immune function, vision, reproduction, and cellular
communication.
as repairing damage or directing chemical processes.
Ribosomes can be found floating within the cytoplasm or 156. Solution (c)
attached to the endoplasmic reticulum. Exp) Option c is correct.

61 Workbook
.
SCIENCE & TECHNOLOGY

The spleen is the largest organ in the lymphatic system. It It weighs about 1.5 kilograms and makes up about 2
is an important organ for keeping bodily fluids balanced, percent of a human’s body weight.
but it is possible to live without it. The spleen is located The cerebrum makes up 85 percent of the brain’s weight.
under the rib cage and above the stomach in the left upper It contains about 86 billion nerve cells (neurons) — the
quadrant of the abdomen. “gray matter.”
The spleen acts as a blood filter; it controls the amount It contains billions of nerve fibers (axons and dendrites)
of red blood cells and blood storage in the body, and — the “white matter.”
helps to fight infection. If the spleen detects potentially
dangerous bacteria, viruses, or other microorganisms in These neurons are connected by trillions of connections,
or synapses.
the blood, it — along with the lymph nodes — creates
white blood cells called lymphocytes, which act as The largest part of the human brain is the cerebrum,
defenders against invaders. The lymphocytes produce which is divided into two hemispheres.
antibodies to kill the foreign microorganisms and stop Like all vertebrate brains, the human brain develops
infections from spreading. from three sections known as the forebrain, midbrain
and hindbrain. Each of these contains fluid-filled cavities
called ventricles. The forebrain develops into the cerebrum
and underlying structures; the midbrain becomes part of
the brainstem; and the hindbrain gives rise to regions of
the brainstem and the cerebellum.
The cerebral cortex is greatly enlarged in human brains
and is considered the seat of complex thought. Visual
processing takes place in the occipital lobe, near the
back of the skull. The temporal lobe processes sound and
language, and includes the hippocampus and amygdala,
which play roles in memory and emotion, respectively.
The parietal lobe integrates input from different senses
and is important for spatial orientation and navigation.
159. Solution (b)
Exp) Option b is correct
Statement 1 is correct. The sclera is the white part of the
157. Solution (a)
eye that surrounds the cornea. In fact, the sclera forms
Exp) Option a is correct more than 80 percent of the surface area of the eyeball. The
Statement 1 and 2 are correct. sclera begins to turn yellow in individuals experiencing
Xylem and Phloem are two different types of vascular liver failure. This condition is called jaundice.
tissues, which are mainly involved in the transportation Statement 2 is incorrect. Only the cornea is transplanted
process. These tissues form a vascular bundle and these during an eye transplant. The retina is a thin layer of
work together as a unit. The movement of xylem is tissue that lines the back of the eye on the inside. It is
unidirectional, while the movement of phloem is located near the optic nerve. The purpose of the retina is
bidirectional. to receive light that the lens has focused, convert the light
Xylem and phloem facilitate the transportation of water, into neural signals, and send these signals on to the brain
minerals and food throughout the plant. Xylem transports for visual recognition.
soluble mineral nutrients and water molecules from Statement 3 is correct. Iris is the colored part of the eye
the roots to the aerial parts of the plant while Phloem which helps regulate the amount of light entering the eye.
transports food and other nutrients including sugar and When there is bright light, the iris closes the pupil to let in
amino acids from leaves to storage organs and growing less light so that the retina does not get damaged.
parts of the plant. Statement 4 is correct. The cornea acts as the eye’s
Statement 3 is incorrect. Xylem and phloem are only outermost lens. It functions like a window that controls
found in plants and not animals. and focuses the entry of light into the eye. It focuses light
into our eye so that a clear image is formed at the retina
158. Solution (d)
of the object.
Exp) Option d is correct
All the statements are correct. 160. Solution (c)
The human brain has the same basic structure as other Exp) Option c is correct.
mammal brains but is larger in relation to body size than Statement 1 is incorrect. While most of DNA is found in
any other brains. cell nucleus, some DNA is also found in the mitochondria,
The human brain is the largest brain of all vertebrates it is called mitochondrial DNA (mtDNA).
relative to body size. Statement 2 is incorrect. Mitochondrial diseases are

Workbook 62
.
SCIENCE & TECHNOLOGY

caused by faulty Mitochondrial DNA, not due to faulty Statement 5 is incorrect. Blastomycosis is an infection
genes on the X-Chromosome. Certain disorders caused caused by a fungus called Blastomyces. The fungus lives
due to mtDNA dysfunction are diabetes, respiratory in the environment, particularly in moist soil and in
disorders, Huntington’s disease, Parkinson’s disease, decomposing matter such as wood and leaves. People
Alzheimer’s disease etc. There is currently no cure for can get blastomycosis after breathing in the microscopic
mitochondrial diseases. fungal spores from the air. Blastomycosis is not spread
Statement 3 is correct. Mitochondria, which are structures person-to-person or animal-to-person. Thus, it is not a
in each cell that convert molecules into energy, each contain zoonotic disease.
a small amount of DNA. Because only egg cells contribute 164. Solution (d)
mitochondria to the developing embryo, only females can
pass on mitochondrial mutations to their children. Exp) Option d is correct.
Statement a is correct. Different diseases are caused by
161. Solution (c) different types of micro-organisms. An infection is the
Exp) Option c is correct invasion and multiplication of pathogenic microbes in
an individual or population. Disease is when the infection
Pair 1 is incorrectly matched. Kala-azar disease is
causes damage to the individual’s vital functions or
caused by protozoan microbes (and not by fungi).
systems. An infection does not always result in disease.
Other diseases caused by protozoa are malaria, Dysentery,
Sleeping Sickness etc. Statement b is correct. The agents that cause disease fall
into five groups: viruses, bacteria, fungi, protozoa, and
Pair 2 is incorrectly matched. Ringworms are caused
helminths (worms). Protozoa and worms are usually
by fungi and transmitted by skin to skin contact. Many
grouped together as parasites, and are the subject of the
common skin infections are also caused by different kinds
discipline of parasitology.
of fungi.
Statement c is correct. Intracellular pathogens must
Pair 3 is correctly matched. Diseases like typhoid fever, invade host cells in order to replicate. Such pathogens
cholera, tuberculosis and anthrax are caused by bacteria. can be subdivided further into those that replicate freely
162. Solution (d) in the cell, such as viruses and certain bacteria and those,
such as the mycobacteria, that replicate in cellular vesicles.
Exp) Option d is correct As viruses are obligate intracellular pathogens they cannot
Option d is correct. Common examples of diseases replicate without the machinery and metabolism of a host
caused by viruses are the common cold, influenza, den- cells.
gue fever, AIDS, Chicken pox, Small pox, Chikungunya,
Statement d is incorrect. Rapidly killing every host that
Ebola, polio etc.
it infects is no better for the long-term survival of a
pathogen than being wiped out by the immune response
163. Solution (a) before it has had time to infect another individual. Thus,
Exp) Option a is correct. for the long-term survival of pathogen killing every host
it infects does no good.
Zoonotic diseases are diseases that are transmitted
from animals to humans. 165. Solution (c)
Statement 1 is correct. West Nile fever is a zoonosis Exp) Option c is correct.
(an animal disease affecting humans). West Nile virus
Flavonoids are a group of plant metabolites thought to
(WNV) is the leading cause of mosquito-borne disease.
provide health benefits through cell signalling pathways
It is most commonly spread to people by the bite of an and antioxidant effects. These molecules are found in a
infected mosquito. There are no vaccines to prevent or variety of fruits and vegetables.
medications to treat WNV in people.
Flavonoids are important antioxidants and promote
Statement 2 is correct. Brucellosis is an infectious disease several health effects. Aside from antioxidant activity,
caused by bacteria. People can get the disease when they these molecules provide beneficial effects like Anti-viral,
are in contact with infected animals or animal products Anti-cancer, Anti-inflammatory, Anti-allergic. Almost all
contaminated with the bacteria. Animals that are most fruits, vegetables, and herbs contain a certain amount of
commonly infected include sheep, cattle, goats, pigs, and flavonoids. They can also be found in other food sources
dogs, among others. including dry beans, grains, red wine and green and black
Statement 3 is correct. Middle East Respiratory teas.
Syndrome MERS-CoV is a zoonotic virus, which means
it is a virus that is transmitted between animals and 166. Solution (c)
people. Exp) Option c is correct.
Statement 4 is correct. Nipah is an example of zoonotic Candida Auris, or C.auris, is a multidrug resistant
disease as the virus can be transmitted to humans from pathogen that presents “a serious global threat to human
animals (pigs, bats). Fruit bats are the natural host of health.” The Candida Auris, has for the first time been
Nipah virus. identified in the environment, off the coast of South

63 Workbook
.
SCIENCE & TECHNOLOGY

Andaman district in the Andaman and Nicobar Islands. natural or industrial sources. Trans-fats are associated
Candida Auris was first identified in 2009 in a patient in with increased risk of heart attacks and death from
Japan. The fungus emerged in more than 40 countries coronary heart disease. It raises bad (LDL) cholesterol
across five continents in the last decade. levels and lower the good (HDL) cholesterol levels. It
is also associated with a higher risk of developing type 2
The fungus is mostly found in tropical marshes and diabetes. TFAs have an adverse effect on the brain and
marine environments, outside hospital environments. nervous system.
167. Solution (b) 170. Solution (d)
Exp) Option b is correct. Exp) Option d is correct.
Statement 1 is incorrect. Harlequin ichthyosis is a genetic Statement 1 is correct. Prokaryotes were the earliest life
disorder. The disease is caused due to mutations in the forms, simple creatures that fed on carbon compounds
ABCA12 gene inherited from the parents. The ABCA12 that were accumulating in Earth’s early oceans. Slowly,
protein plays a major role in transporting fats in cells that other organisms evolved that used the Sun’s energy, along
make up the outermost layer of skin. with compounds such as sulphides, to generate their own
Statement 2 is correct. Harlequin ichthyosis is a severe energy.
genetic disorder that mainly affects the skin. The disease Statement 2 is correct. Early volcanic eruptions led to the
creates large diamond-shaped plates across the body that Earth’s second atmosphere, which led to Earth’s modern
are separated by deep cracks (fissures). The skin becomes atmosphere by releasing carbon dioxide and other gases
dry and scaly almost like fish skin and hence the term into the air. Without volcanoes, most of Earth’s water
‘icthyosis’, derived from ‘ikthus’, Greek for fish. would still be trapped in the crust and mantle. It also
Severe mutations in the gene lead to the absence or partial released water vapour that eventually cooled to form the
production of the ABCA12 protein. This results in lack oceans. The atmosphere slowly became more oxygen-rich
of lipid transport and as a result, the skin development as solar radiation split water molecules and cyanobacteria
is affected by varying degrees according to the severity of began the process of photosynthesis. Besides water and
the mutation. air, volcanoes are responsible for land, another necessity
for many life forms.
168. Solution (b) Statement 3 is correct. Once photosynthetic organisms
Exp) Option b is correct. evolved, the organisms, blue-green algae, fed off
C-peptide is a substance that is created when the atmospheric carbon dioxide and converted much of
hormone insulin is produced and released into the body. it into marine sediments consisting of the shells of sea
creatures. While photosynthetic life reduced the carbon
Measuring the amount of C-peptide in blood indicates dioxide content of the atmosphere, it also started to
how much insulin is being produced. In mammals, produce oxygen. Once oxygen had been produced,
including humans, insulin is synthesised as a pro- ultraviolet light split the molecules, producing the ozone
hormone (like a pro-enzyme, the pro-hormone also UV shield as a by-product.
needs to be processed before it becomes a fully mature
and functional hormone) which contains an extra stretch 171. Solution (d)
called the C peptide. This C peptide is not present in the Exp) Option d is correct.
mature insulin and is removed during maturation into Statement 1 is incorrect. Bird flu or avian influenza is
insulin. Generally, high C-peptide production indicates the name used to describe a viral infection that is reported
high insulin production, and vice versa. mostly in birds but has the potential to affect humans and
169. Solution (a) other animals. The most common strain of the virus that
causes severe respiratory disease in birds is H5N1; various
Exp) Option a is correct. other strains like H7, H8 too, cause infection. H1N1 virus
Statement 1 is correct. Unsaturated fats, which are strain is known to cause swine flu.
liquid at room temperature, are considered beneficial Statement 2 is incorrect. Almost all cases of H5N1
fats because they can improve blood cholesterol levels, infection in people have been associated with close contact
ease inflammation, stabilize heart rhythms, and play with infected live or dead birds, or H5N1 contaminated
a number of other beneficial roles. Unsaturated fats are environments. The virus does not infect humans easily,
predominantly found in foods from plants, such as and spread from person to person appears to be
vegetable oils, nuts, and seeds. unusual.
Statement 2 is correct. Saturated fat is solid at room
temperature and is mostly found in animal foods, such 172. Solution (d)
as milk, cheese, and meat. Poultry and fish have less Exp) Option d is correct.
saturated fat than red meat. Saturated fat is also in tropical Option a is correct. Stem cells are the foundation for
oils, such as coconut oil, palm oil, and cocoa butter. every organ and tissue in your body. Stem cells are a class
Statement 3 is incorrect. Trans-fat, or trans-fatty acids of undifferentiated cells that are able to differentiate into
(TFA), are unsaturated fatty acids that come from either specialized cell types.

Workbook 64
.
SCIENCE & TECHNOLOGY

Types of Stem Cells - There are many different types of Option b is incorrect. It is correct that totipotent cells
stem cells that come from different places in the body or are formed during sexual and asexual reproduction but
are formed at different times in our lives. These include zygotes are a product of sexual reproduction whereas
embryonic stem cells that exist only at the earliest stages spores are result of asexual reproduction.
of development and various types of tissue-specific (or Option c is correct. Single totipotent cell has the potential
adult or somatic) stem cells that appear during fetal to develop into an embryo with all the specialized cells
development and remain in our bodies throughout life. that make up a living being, as well as into the placental
Embryonic stem cells are derived from the inner cell mass support structure necessary for foetal development. Thus,
of an embryo and these are capable of forming any cell each totipotent cell is a self-contained entity that can give
types of the body. rise to the whole organism.
Embryonic stem cells are pluripotent, meaning they can Option d is incorrect. Pluripotency is the ability of a cell
give rise to every cell type in the fully formed body, but not to develop into the three primary germ cell layers of the
the placenta and umbilical cord. These cells are incredibly early embryo and therefore into all cells of the adult body,
valuable because they provide a renewable resource for but not extra-embryonic tissues such as the placenta.
studying normal development and disease, and for testing
drugs and other therapies. Human embryonic stem cells Totipotent vs Pluripotent vs Multipotent
have been derived primarily from blastocysts created by Comparison Chart
in vitro fertilization (IVF) for assisted reproduction that Totipotent Pluripotent Multipo-
were no longer needed. tent
Option b is correct. Blood-forming (or hematopoietic) R e l a t i v e High Medium Low
stem cells in the bone marrow can give rise to red blood potency
cells, white blood cells and platelets.
Cell types Differenti- Differenti- Differen-
Option c is correct. Adult stem cells are also known as
capable of ate into any ate into cells tiate into
somatic stem cells. Tissue-specific stem cells (also referred generating cell type from any of a limited
to as somatic or adult stem cells) are more specialized the three range of cell
than embryonic stem cells. Typically, these stem cells can germ layers types
generate different cell types for the specific tissue or organ
in which they live. Terminol- Toti = Pluri = Multi =
ogy Whole Many Several
They generate cells to replace those that are lost through
normal repair, disease, or injury. ASCs are found Examples Zygote, ear- Embryonic Ha e m at o -
throughout one’s lifetime in tissues such as the umbilical ly morula stem cells, p o i e t i c
cord, placenta, bone marrow, muscle, brain, fat tissue, I n d u c e d stem cells,
skin, gut, etc. pluripotent n e u r a l
stern cells stem
Option d is incorrect. Induced Pluripotent stem cells
are produced by genetically manipulating somatic cells cells, mes-
to produce embryonic-like stem cells. e n c hy r n a l
stem cells
Induced Pluripotent stem cells are cells that have been
engineered in the lab by converting tissue-specific cells, Found Early cells Inner mass In many tis-
such as skin cells, into cells that behave like embryonic of fertilised cells of the sues
stem cells. egg blastocyst
While iPS cells share many of the same characteristics of Expression +++ ++ +
embryonic stem cells, including the ability to give rise to of pluripo-
all the cell types in the body, they aren’t exactly the same. tency genes
Scientists are exploring what these differences are and Expression + + +++
what they mean. For one thing, the first iPS cells were of line-
produced by using viruses to insert extra copies of genes age-specific
into tissue-specific cells. Researchers are experimenting genes
with many alternative ways to create iPS cells so that they
can ultimately be used as a source of cells or tissues for Pros of use Easy to Easy to Less ethical
medical treatments. in research isolate and isolate and issues, less
grow grow chance of
173. Solution (c) immune
Exp) Option c is correct. rejection if
Option a is incorrect. It is totipotency which is restricted taken from
only to the early cells of fertilized eggs (and not in same pa-
pluripotency). tient

65 Workbook
.
SCIENCE & TECHNOLOGY

174. Solution (d) photosynthesis. In plant cells, chloroplasts are responsible


Exp) Option d is correct. for trapping light energy essential for photosynthesis.
Option a is correct. Different diseases are caused by Option a is correct. Algae contain chlorophyll which gives
different types of micro-organisms. An infection is the them the green colour. All algae contain chlorophyll. But
invasion and multiplication of pathogenic microbes in not all algae are green. Some algae have pigments that
an individual or population. Disease is when the infection mask the green chlorophyll. Algae exhibits different color
causes damage to the individual’s vital functions or variations because of the other photosynthetic pigments
systems. An infection does not always result in disease. they contain.
Option b is correct. The agents that cause disease fall Option b is correct. The leaves other than green also
into five groups: viruses, bacteria, fungi, protozoa, and have chlorophyll. The large amount of red, brown and
helminths (worms). Protozoa and worms are usually other pigments masks the green colour. Photosynthesis
grouped together as parasites and are the subject of the takes place in these leaves also.
discipline of parasitology. Option c is correct. The plants respond to higher
Option c is correct. Intracellular pathogens must invade CO2 concentration by showing increased rates of
host cells in order to replicate, such pathogens can be photosynthesis leading to higher productivity has been
subdivided further into those that replicate freely in the used for some greenhouse crops such as tomatoes and
cell, such as viruses and certain bacteria and those, such bell pepper. They are allowed to grow in carbon dioxide
as the mycobacteria, that replicate in cellular vesicles. enriched atmosphere that leads to higher yields.
Option d is incorrect. Rapidly killing every host it infects 177. Solution (d)
is no better for the long-term survival of a pathogen than
being wiped out by the immune response before it has Exp) Option d is correct.
had time to infect another individual. Statement 1 is incorrect. The components of food such
as carbohydrates are complex substances. These complex
substances cannot be utilised as such. So, they are broken
down into simpler substances. The breakdown of
complex components of food into simpler substances
is called Digestion. Digestion is a form of catabolism
that is often divided into two processes based on how
food is broken down: mechanical and chemical digestion.
The term mechanical digestion refers to the physical
breakdown of large pieces of food into smaller pieces
175. Solution (a) which can subsequently be accessed by digestive enzymes.
In chemical digestion, enzymes break down food into
Exp) Option a is correct. small molecules the body can use.
Statement 1 is correct. The phenotype is the physical Statement 2 is incorrect. Food is taken into the body
appearance of an organism, while the Genotype is through the mouth. The process of taking food into
the genetic composition of an organism. Phenotype is the body is called Ingestion. Ingestion can also include
observable and are the expression of the genes of an
absorbing of food by a microorganism.
individual. We can notice one’s hair colour, eye colour,
height, weight, skin colour, etc. but cannot look at genes Statement 3 is also incorrect. The large intestine is wider
responsible for these characters, so the observable and shorter than the small intestine. It is about 1.5 metre
physical look is the phenotype while the unnoticed genes in length. Its function is to absorb water and some salts
responsible for such characters present in the DNA of from the undigested food material. The remaining waste
cell of the individual is genotype. passes into the rectum and remains there as semi-solid
Statement 2 is incorrect. Genotype are partly inherited faeces. The faecal matter is removed through the anus
from an individual to the offspring as one of the two from time-to-time. This is called Egestion.
alleles, during the reproduction process. The hereditary 178. Solution (a)
characters of the organisms, which may or may not get
expressed in the next generation. The same genotype Exp) Option a is correct.
produces the same phenotype in a particular environment. Statement 1 is correct. ASF is a severe viral disease that
Phenotype is not inherited. It appears outside the body, affects wild and domestic pigs typically resulting in an
as physical appearance. acute hemorrhagic fever. The disease has a case fatality rate
(CFR) of almost 100 per cent. Its routes of transmission
176. Solution (d) include direct contact with an infected or wild pig (alive or
Exp) Option d is correct. dead), indirect contact through ingestion of contaminated
Option d is incorrect. The chloroplasts contain material such as food waste, feed or garbage, or through
chlorophyll and carotenoid pigments which are biological vectors such as ticks.
responsible for trapping light energy essential for Statement 2 is incorrect. African Swine Fever (ASF) does

Workbook 66
.
SCIENCE & TECHNOLOGY

not affect humans but can be catastrophic for pigs. It is Option d is incorrect. Currently there are several
a notifiable, non-zoonotic disease for which no effective effective medicines to control the disease. The WHO
treatment or vaccine currently exists. recommended medicines –albendazole (400 mg) and
Statement 3 is incorrect. The current outbreak of ASF in mebendazole (500 mg) – are effective, inexpensive and
India (in Assam) is the first time that the disease has been
easy to administer.
reported in the country.
179. Solution (b) 182. Solution (c)
Exp) Option b is correct. Exp) Option c is correct.
Option 1, 2 and 4 are correct. White Blood Cells, also Statement 1 is incorrect. Liver is the largest gland of the
called leukocytes, are an important part of the immune body weighing about 1.2 to 1.5 kg in an adult human. It is
system. These cells help fight infections by attacking
bacteria, viruses, and germs that invade the body. They situated in the abdominal cavity, just below the diaphragm
are colourless due to lack of haemoglobin. and has two lobes. On the other hand, average weight of
White blood cells originate in the bone marrow but heart is 250- 300 gm.
circulate throughout the bloodstream. There are five Statement 2 is incorrect. Liver is responsible for
major types of white blood cells: production of a digestive fluid known as bile. Liver
1. neutrophils releases Bile into Gall Bladder which is a small, pear-
2. lymphocytes shaped organ located just below your liver in the upper
3. eosinophils right side of your abdomen.
4. monocytes
5. basophils Statement 3 is correct. The pancreas is a both exocrine
and endocrine organ of the body. The pancreas has
Option 3 is incorrect. Erythrocytes or red blood cells
(RBC) are the most abundant of all the cells in blood. the dual function of secreting hormones into blood
RBCs are formed in the red bone marrow in the adults. (endocrine) and secreting enzymes through ducts
(exocrine).
180. Solution (a)
Statement 4 is correct. The liver is the only organ in
Exp) Option a is correct
the human body that can regenerate. After the diseased
Statement 1 is correct. Lack of clean water, sanitation
and inadequate infection prevention and control portion of the liver is removed, fibrinogen and blood
promotes the spread of microbes. This can lead to platelets accumulate in the remaining liver, which triggers
antimicrobial resistance. the earliest stages of regeneration.
Statement 2 is correct. The emergence and spread
of drug-resistant pathogens that have acquired new 183. Solution (a)
resistance mechanisms, leads to antimicrobial resistance. Exp) Option a is correct
This threatens our ability to treat common infections.
Statement 1 is correct: Viruses are typically described as
Statement 3 is incorrect. Antibiotic resistance has the obligate intracellular parasites. It is an acellular infectious
potential to affect people at any stage of life, as well
as the healthcare, veterinary, and agriculture industries. agent that requires the presence of a host cell to multiply.
Antimicrobial resistance thus, is one of the world’s most Outside the host cell, they are inert or metabolically
urgent public health problems. inactive.
181. Solution (c) Statement 2 is incorrect: Viruses have been found
Exp) Option c is correct. to infect all types of cells- humans, animals,
plants, bacteria, yeast, archaea, protozoans etc. Eg.
Option a is incorrect. Helminth infections are Soil-
transmitted and are caused by different species of Bacteriophages or simply phages are viruses that infect
parasitic worms. bacteria.
Option b is incorrect. India accounts for a quarter of the Statement 3 is correct. Coronavirus is an RNA virus
world’s STH-infected children; 64% of them are younger with ribonucleic acid (RNA) as its genetic material
than 14 years. instead of deoxyribonucleic acid (DNA).
Option c is correct. Infected children are nutritionally All viruses contain nucleic acid, either DNA or RNA (but
and physically impaired. Heavier infections can cause
not both), and a protein coat, which encases the nucleic
a range of symptoms including intestinal manifestations
(diarrhoea and abdominal pain), malnutrition, general acid. Some viruses are also enclosed by an envelope of fat
malaise and weakness, and impaired growth and physical and protein molecules. In its infective form, outside the
development. cell, a virus particle is called a virion.

67 Workbook
.
SCIENCE & TECHNOLOGY

186. Solution (a)


Exp) Option a is correct
Statement 1 is correct. The cells having nuclear material
without nuclear membrane are termed prokaryotic
cells. The organisms with these kinds of cells are called
prokaryotes. Examples are bacteria and blue green algae.
The nucleus of the bacterial cell is not well-organised like
the cells of multicellular organisms.
Statement 2 is incorrect. Eukaryotes, can be unicellular
or multicellular and include animals, plants, fungi and
protists. The cells having well-organised nucleus with a
nuclear membrane are designated as eukaryotic cells.
187. Solution (a)
Exp) Option a is correct
184. Solution (b)
Options 1 and 2 are correct. Hepatitis C and Chicken
Exp) Option b is correct. Pox are caused by viruses.
Option a is incorrect. Mycetoma is a disease caused by Option 3 is correct. Common cold is mostly caused
certain types of bacteria and fungi found in soil and water. by a number of different types of viruses. More than
These bacteria and fungi may enter the body through a 200 different types of viruses are known to cause the
break in the skin, often on a person’s foot. The resulting common cold, with most common being the rhinovirus
infection causes firm, usually painless but debilitating causing approximately 30%-40% of all adult colds.
masses under the skin that can eventually affect the Other commonly implicated viruses include coronavirus
underlying bone. (COVID-19), adenovirus, respiratory syncytial virus, and
Option b is correct. Athlete’s foot is a fungal infection parainfluenza virus.
that affects the upper layer of the skin of the foot, Option 4 is incorrect. Malaria is caused by Protozoa
especially when it is warm, moist, and irritated. It is also and not by virus.
known as tinea Pedi and ringworm of the foot. The
fungus that causes athlete’s foot is called Trichophyton and 188. Solution (d)
is commonly found on floors and in clothing. The disease Exp) Option d is correct
is common around swimming pools and showers as these Statement 1 is correct. The microorganisms like aerobic
places are warm and humid. The disease is contagious. and anaerobic bacteria are used in sewage treatment
Option c is incorrect. Elephantiasis is also known as plants. They breakdown the organic matter, thereby
lymphatic filariasis. It’s caused by parasitic worms and reducing the organic content of the sewage along with the
can spread from person to person through mosquitoes. Biological Oxygen Demand of the treated sewage water.
Elephantiasis causes swelling of the scrotum, legs, or The treated sewage water then becomes fit to be released
breasts. to the natural water bodies.
Option d is incorrect. Blastomycosis is an infection Cleaning of environment - In nature, microorganisms
caused by a fungus called Blastomyces. The fungus lives decompose dead organic waste of plants and animals
in the environment, particularly in moist soil and in converting them into simple substances. These substances
decomposing matter such as wood and leaves. People are again used by other plants and animals. Thus,
can get blastomycosis after breathing in the microscopic microorganisms can be used to degrade the harmful and
fungal spores from the air. smelly substances and thereby clean up the environment.
185. Solution (d) Statement 2 is correct. Many medicines are sourced
from microorganisms such as bacteria and fungi.
Exp) Option d is correct
• Such medicines called, antibiotics, kill or stop the
Noise pollution refers to the presence of excessive noise growth of the disease-causing microorganisms.
in the surroundings. Noise pollution may cause many
health-related problems such as: • These days a number of antibiotics are being produced
from bacteria and fungi. Streptomycin, tetracycline
• Hypertension (high blood pressure). Hence, Option and erythromycin are some of the commonly known
1 is correct. antibiotics which are made from fungi and bacteria.
• Hearing disability - A person who is exposed to a • The antibiotics are manufactured by growing specific
loud sound continuously may get temporary or even microorganisms and are used to cure a variety of
permanent impairment of hearing. Hence, Option 2 diseases.
is correct.
Statement 3 is correct. Bacteria can be used for
• Sleeping disorders. Hence Option 3 is correct. increasing the fertility of soils as some bacteria are able
• Anxiety. Hence, Option 4 is correct. to fix nitrogen from the atmosphere to enrich soil with

Workbook 68
.
SCIENCE & TECHNOLOGY

nitrogen and increase its fertility. These microbes are 190. Solution (a)
commonly called biological nitrogen fixers. Exp) Option a is correct
Statement 4 is correct. Vaccination development is Pair 1 is correctly matched. Mushroom is a Saprotroph,
one of the most important uses of bacteria. These days which means it takes its nutrition from dead and decaying
vaccines are made on a large scale from microorganisms to organic matter. It falls under the category of fungus.
protect humans and other animals from several diseases.
Pair 2 is incorrectly matched. Insectivorous plants are
• Several diseases, including cholera, tuberculosis, partial heterotrophs whereas other plants are autotrophs.
smallpox and hepatitis can be prevented by Insectivores plants are not the examples of parasitic
vaccination. A vaccine works by training the immune nutrition in plants.
system to recognize and combat pathogens, either
Parasitic plants cannot photosynthesise and hence, are
viruses or bacteria.
depended on other living organisms for their nutrition.
• To do this, certain molecules from the pathogen must Insectivorous plants can photosynthesise but lack only
be introduced into the body to trigger an immune in nitrogen. So,to fulfil this requirement, they consume
response. These molecules are called antigens, and the digestive nutrients in the insects.
they are present on all viruses and bacteria. Pair 3 is correctly matched. Plants as well as Algae are
called autotrophs.
189. Solution (a)
Algae contains chlorophyll can also prepare its own
Exp) Option a is correct food by photosynthesis. The mode of nutrition in which
Statement 1 is correct. The changes which occur at organisms make food themselves from simple substances
adolescence are controlled by hormones. is called autotrophic nutrition.
• The male hormone or testosterone begins to be 191. Solution (d)
released by the testes at the onset of puberty. This
Exp) Option d is correct
causes changes in boys, for example, the growth of
facial hair. Statement 1 is correct. Bile is a complex fluid containing
water, electrolytes and organic molecules like bile acids,
• Once puberty is reached in girls, ovaries begin to cholesterol, phospholipids and bilirubin that flows
produce the female hormone or estrogen which through the biliary tract into the small intestine.
makes the breasts develop. Milk secreting glands or
Bile contains bile acids, which are critical for digestion
mammary glands develop inside the breasts.
and absorption of fats and fat-soluble vitamins in the
• The production of these hormones is under the control small intestine.
of another hormone secreted from an endocrine gland Statement 2 is incorrect. Bile is secreted by the liver of
called pituitary gland. most vertebrates, that aids in the digestion of lipids in the
Statement 2 is incorrect. Hormones are chemical small intestine. In humans, it is secreted by the liver, and
substances which are secreted from endocrine glands, stored and concentrated in the gallbladder.
or endocrine system. Pituitary gland is just one of the Statement 3 is correct. The food coming from the
glands of endocrine system. Thus, all the hormones stomach is acidic and has to be made alkaline for the
in our body are not secreted by the pituitary gland. pancreatic enzyme to act. Bile accomplishes this. Bile
Hormones are produced by glands and sent into the salts break- down larger globules of fats into smaller
bloodstream to the various tissues in the body. They send globules increasing the efficiency of enzyme action.
signals to those tissues to tell them what they are supposed
192. Solution (c)
to do. When the glands do not produce the right amount
of hormones, diseases develop that can affect many Exp) Option c is correct
aspects of life. Statement 1 is incorrect. Methanogens are microbes
that produce methane by growing anaerobically (
not aerobically) on cellulosic material. In addition
to methane, some amounts of Carbon dioxide and
Hydrogen gas are also produced. One such microbe is
Methanobacterium.
Methanogens belong to the domain archaea, which is
distinct from bacteria. Both bacteria and Archaea are
prokaryotic organisms.
Statement 2 is correct. Biogas is a mixture of gases
predominantly containing methane. It is produced by
the microbial activity and may be used as fuel.
Statement 3 is correct. Methonogens like
Methanobacterium are present in the rumen of (a part of

69 Workbook
.
SCIENCE & TECHNOLOGY

stomach) of cattle. A lot of cellulosic material present in Chronologically, the following organisms evolved in
the food of cattle is also present in the rumen. In rumen, the sequence given below -
these microbes help in the breakdown of cellulose and 1. Corals
play an important role in the nutrition of cattle. 2. Fishes
Thus, the excreta of cattle (dung) is rich in these microbes. 3. Birds
Dung can be used for generation of biogas, commonly
called gobar gas. 4. Flowering plants
These Methanogens are also found in the anaerobic sludge 195. Solution (c)
during sewage treatment. Exp) Option c is correct.
193. Solution (c) Statement 1 is correct. Chromosomal disorders
Exp) Option c is correct. are caused due to absence or excess or abnormal
arrangement of one or more chromosomes.
Statement 1 is correct. Fungi are known to form
symbiotic associations with plants (mycorrhiza) where Statement 2 is correct. Chromosomal Disorders -
the fungi absorbs phosphorus from soil and passes it to Sometimes, either an additional copy of a chromosome
the plant. may be included in an individual or an individual may
Plants having such associations show other benefits also, lack one of any one pair of chromosomes. These situations
such as resistance to root-borne pathogens, tolerance are known as trisomy or monosomy of a chromosome,
to salinity and drought, and an overall increase in plant respectively. Down’s syndrome, Turner’s syndrome,
growth and development. Klinefelter’s syndrome are common examples of
chromosomal disorders.
Statement 2 is correct. In addition to nitrogen-fixing
bacteria like Rhizobium that live in the plant roots, there In the Down’s syndrome, a person has an additional copy
are some free-living bacteria in the soil like Azospirillum of chromosome number 21. In the Turner’s syndrome, a
and Azotobacter that fix atmospheric nitrogen in the person has an absence of one of the X chromosomes. In
soil. These bacteria fix atmospheric nitrogen into organic Klinefelter’s syndrome, a person has an additional copy of
forms, which is used by the plant as nutrient. X chromosome.
194. Solution (a) 196. Solution (a)
Exp) Option a is correct Exp) Option a is correct
Statement 1 is correct. The total number of chromosomes
in a normal human cell is 46 (23 pairs). Out of these 22
pairs are autosomes and one pair of chromosomes are
sex chromosome. The autosomes are exactly the same
between males and females in the humans.
Statement 2 is incorrect. XX pair of chromosomes are
found in the females and XY chromosomes are found in
the males.
197. Solution (c)
Exp) Option c is correct
The digestive juices convert complex substances of food
into simpler ones. The digestive tract and the associated
glands together constitute the digestive system.
Digestive juices in our body are secreted by
• Inner walls of the stomach and the small intestine.
Hence, statement 1 is correct.
• Various glands associated with the canal such as
salivary glands, the liver and the pancreas. Hence,
statements 3 and 4 are correct.
The diaphragm is a thin skeletal muscle that sits at
the base of the chest and separates the abdomen from
the chest. It contracts and flattens when we inhale. This
creates a vacuum effect that pulls air into the lungs. When
we exhale, the diaphragm relaxes and the air is pushed out
of lungs. It does not secrete digestive juices in our body.
Hence, statement 2 is incorrect.

Workbook 70
.
SCIENCE & TECHNOLOGY

198. Solution: (b) found mostly in animals of prey, like herbivores (deer,
Exp) Option b is correct. zebra, etc). These features help the prey to detect the
presence of any predator lying in wait to catch and kill
An important phase in the life cycle of all plants is the them.
process of germination. It refers to the biological process
through which a plant embryo emerges out of the seed to Statement 2 is incorrect: Characteristics like Camouflage
become a seedling that ultimately grows into a plant. are not exclusive to animals of prey. They are found in
both animals of prey and in predators. Camouflage refers
Option 1 is incorrect: Generally, most plants require to the quality of having skin patterns and coloration
a particular wavelength of light in order to germinate which help an animal blend with its surroundings, thus
and grow. However, it is not always compulsory for giving the impression of invisibility.
the germination of all plant seeds. The seeds of some
plants are not affected by the absence of light as far as It is used by both (prey and predator) in their own ways.
their germination is considered. Such plants are called Animals of prey like the deer and zebra use it to hide in
‘non photoblastic’. So, light may not be termed as a their surroundings so that they may fool the predators
compulsory condition of germination of seeds of all and save their lives.
plants. Predators use this to hide in their surroundings so that
Option 2 is correct: The correct range of temperature is they can surprise the prey and catch and kill it.
a condition essential for germination of all plants. It must Statement 3 is incorrect: Predators have sharper teeth
be noted that this range is different for different plants’ and claws compared to animals of prey. This is because
seeds. It may be high for some and it may be low for some. they have to hunt and kill. On the other hand, animals
If the temperature is higher than the ideal range, the of prey are generally herbivores, so they have blunt teeth
enzymes and cell contents of the seed will get destroyed, for chewing vegetation and hoofs that enable them to
and if it is lower than the ideal range, the seed will go into run fast.
dormancy and not germinate.
200. Solution: (a)
Option 3 is correct: The Presence of Moisture is an
absolutely essential condition for the germination of Exp) Option a is correct.
seeds. Moisture plays following vital roles: Objects in the world can be roughly categorised into two
• Makes the external coat of the seed soft, allowing the groups - living and non living things. All living things
plant to break out and grow. have certain common characteristics which may be used
• The softened seed coat allows exchange of nutrients to determine their status. Some of these characteristics
and air between the environment and the plant have been discussed below.
embryo, allowing it to grow. Statement 1 is correct: All living organisms need energy
• It activates the protoplasm of the cells. for various life functions. Most living organisms derive
this energy from the chemical energy that is stored
• Mobilises food material by turning them from in food. However, not all organisms need to obtain
insoluble to soluble.
food to derive this energy. Autotrophs, i.e. organisms
• Many growth essential enzymes become active only that manufacture their own food, like green plants
in the presence of water. (via photosynthesis), use solar energy and nutrients to
Option 4 is correct: Adequate supply of oxygen is manufacture and store food, whose chemical energy they
absolutely essential for the germination of a seed. This is use for their life functions.
because oxygen is required for the process of respiration Statement 2 is correct: All living organisms react to any
in which the plant embryo utilises stored food and breaks change in external or internal environment (known as
it down into energy which is utilised for the growth of stimulus). Like all organisms, plants detect and respond
the embryo into a seedling. to stimuli in their environment. For example if a human
199. Solution: (a) being touches a hot object they immediately withdraw
their hands or cry with pain. However if non living
Exp) Option a is correct. objects like a dress are set on fire, then it will not react in
All organisms interact with one another. These any manner. Stimuli can be any change in an organism’s
interactions can be categorised into multiple categories environment like the level of light, the temperature, touch,
like mutualism, predation, parasitism, competition, smell, etc.
ammensalism and commensalism. Statement 3 is incorrect: Most living organisms grow,
Predation is an interspecies interaction in which one i.e. increase in length/ size only from their birth until
species (predator) hunts and kills another species (prey) their adolescence and roughly remain the same after
to eat it as food. Some common features of prey and attaining adulthood. The body cells may regrow to
predators are discussed below. replace old worn ones or to repair damaged ones, but
Statement 1 is correct: Defensive characteristics like there is generally no increase in length (i.e. growth). An
long ears (which help pick even faint sounds) and eyes exception is plants. Most plants grow throughout their
on the side (which gives a very wide field of vision) are lifespan.

71 Workbook
.
SCIENCE & TECHNOLOGY

201. Solution: (b) 203. Solution: (c)


Exp) Option b is correct. Exp) Option c is correct.
All organisms obtain food in order to carry out life Respiration is the process of using food substrates to create
processes like respiration, metabolism, etc to derive energy within a living organism. Most of the organisms,
energy for various functions like growth, movement, especially the higher order, complex organisms need
reproduction, etc. No organism can absorb all the food it oxygen to complete this process. However, some lower
ingests. Some of it always turns to waste inside the body. order organisms can grow and survive in the absence of
This waste product is toxic and needs to be eliminated by oxygen as well - this type of respiration is called anaerobic
the living organism in order to stay healthy. This process respiration.
of expulsion of waste products by living organisms is Option 1 is correct: Yeast is a type of fungi that ferments
called excretion. Both animals and plants excrete, but starchy substrates in the absence of oxygen (anaerobic
have different mechanisms for doing so. process) to create alcohols as a by-product while deriving
Statement 1 is incorrect: Plants do not have a specialised energy for itself (a process called glycolysis).
system for excretion. So, it is not a complex process. Most Option 2 is incorrect: Earthworms are complex
waste products are removed by plants using structures organisms of a higher order called Annelida. They have
that already are performing other functions. a complex organisation of cells and tissues and cannot
respire without oxygen. So, they carry out aerobic
For example: Carbon dioxide produced during
respiration using their food materials and oxygen to
respiration and oxygen produced during photosynthesis,
derive energy.
excess water is expelled through the stomata in the leaves;
Option 3 is correct: Clostridium is a type of bacteria
The plants often store its waste products in old xylem
which is an obligate anaerobic (is harmed by the
tissues in barks, or in old leaves which are eventually presence of oxygen). These bacteria are responsible for
shed. botulism (food poisoning) and are also found in deep
Statement 2 is correct: Plants often find ways to excrete sea sediments.
waste products through various secretions like the latex Option 4 is incorrect: Mushrooms are a type of
dripping from the barks of rubber trees, the gums and fungi. They use dead and decaying organic matter as
resins in the barks of trees, oils of trees like eucalyptus, substrate to derive energy and nutrients to grow. Button
etc. Thus, plant secretions like rubber, gums and resins are mushrooms are aerobes, i.e. they require oxygen in order
actually waste products. to extract energy from food and grow. They are widely
used in cooking.
202. Solution: (b)
Option 5 is correct: Methanogenic Archaea is a type of
Exp) Option b is correct.
simple and single celled prokaryotic organism that is
Statement 1 is incorrect: A strong pigment named often found near deep sea hydrothermal vents. It creates
Lawsone (not carotenoids) is responsible for the bright methane as a by-product of its life processes. It operates at
red colour on application of henna. Henna/ Mehendi is great depths within anoxic environments, so it practises
a form of body art widely prevalent in South Asia, Middle anaerobic respiration.
East and North Africa. The flowering plant from which
henna is derived is called Lawsonia inermis. The leaves 204. Solution: (d)
of henna when crushed do not stain the skin. They Exp) Option d is correct.
will stain only after the release of lawsone molecules Respiration is the process by which living organisms
(2-hydroxy-1, 4-naphthoquinone) present in the leaves, turn food into energy for carrying out life processes.
after smashing them with something mildly acidic liquid Respiration is of two kinds - aerobic (using oxygen)
like tea. and anaerobic (without oxygen). Anaerobic cellular
Carotenoids are a type of phytonutrient (plant respiration creates lactic acid in the body as a by-product.
pigments) that give the characteristic red, orange and Option a is incorrect: Acetic Acid is not found
yellow colour seen in many fruits and vegetables like naturally in the human body. It is an organic
carrot, tomato, red chilli, red bell pepper, etc. They help
chemical compound best recognized for giving
the plant absorb light energy for photosynthesis. They
have antioxidant and anti-inflammatory properties and vinegar its sour taste and pungent smell. It is
are good for human health. one of the simplest carboxylic acids and has
the chemical formula CH3COOH. In nature,
Statement 2 is correct: The unique smell that emanates
from each type of flower (called the floral scent/ aroma) is Acetic acid is produced and excreted by acetic
due to low weight Volatile Organic Compounds (VOCs) acid bacteria, notably the genus Acetobacter and
that are created and emitted by floral tissues in the flower Clostridium acetobutylicum. These bacteria are
petals. The smell is a result of a complex combination of found universally in foodstuffs, water, and soil,
hundreds of organic compounds. Its main purpose is to and acetic acid is produced naturally as fruits and
repel herbivores and attract pollinators. other foods spoil.

Workbook 72
.
SCIENCE & TECHNOLOGY

Option b is incorrect: Hydrochloric Acid is a Pair 2 is correct: Veins are the blood vessels that carry
strong inorganic acid present within the stomach deoxygenated blood (rich in carbon dioxide, deficient in
of human beings. It is necessary to help along oxygen) from the entire body to the heart. Blood flows at
the process of food digestion and absorption. lower pressure in veins, so they are comparatively thin
walled.
It is required to catalyse the reaction to convert
the protein in food to smaller particles called Pair 3 is correct: Capillaries are extremely thin (at many
peptides. It causes proteins from food to lose their points just one cell thick) blood vessels that connect
the arteries and the veins to complete the circuit of
characteristic folded structure (or denature). This the circulatory system in the body. The arteries keep
exposes the protein’s peptide bonds. The chief branching out till they become capillaries which join and
cells of the stomach secrete enzymes for protein group again to form veins. They are also important for
breakdown (inactive pepsinogen and renin). delivering oxygen and collecting wastes from tissues and
Gastric acid activates pepsinogen into the enzyme cells in extremely crowded and thin spacing.
pepsin, which then helps digestion by breaking
the bonds linking amino acids, a process known 206. Solution: (d)
as proteolysis. In addition, many microorganisms Exp) Option d is correct.
have their growth inhibited by such an acidic Like all living organisms, plants need food to create
environment, which is helpful to prevent infection. energy and create waste products through respiration,
and need nutrients and water. To get these to all parts
Option c is incorrect: Amino Acids are a group
within a plant, there is a system of special tissues called
of organic molecules that consist of a basic amino vascular tissues that form the transportation system of a
group (-NH2), an acidic carboxyl group (-COOH), plant. There are two types of vascular tissues in a plant -
and an organic R group (or side chain). These are Xylem and Phloem.
the building blocks of proteins in the body of living Statement 1 is incorrect: Xylem transports water (not
organisms. Proteins are one of the 6 vital nutrients food), while Phloem transports food (not water) like
in the body and are responsible for a number of vital carbohydrates and amino acids in a plant.
functions (like building the contents of cells together, Statement 2 is correct: Transport of water occurs by
catalysing important biochemical reactions within the process of transpiration pull and root pressure. So,
the body, providing immunity, transporting oxygen, there is no expense of energy in this process. However, in
etc). the translocation of food, energy in the form of ATP is
Option d is correct: Lactic Acid is the acid utilised to push food from one cell to an adjacent cell.
produced in the body as a result of anaerobic Statement 3 is correct: Xylem is made of dead cells with
cellular respiration. The accumulation of this no cytoplasm and no end walls in between, thus forming
acid near the muscles carrying out the anaerobic a continuous hollow tube. On the other hand, Phloem
respiration causes fatigue and results in a cramp. is made of living cells having cell walls and cytoplasm.
Statement 4 is correct: Xylem which transports water
has unidirectional flow (from roots to tips) as all water
is absorbed from soil by roots only and by no other plant
part.
On the other hand, In Phloem food moves from source
to sink. Generally the source is leaves which manufacture
the food while sink is the root and stem and other parts
of the plant which need energy. However this reverses
with seasons. Suppose in cold season, when the leaves
205. Solution: (b) shed, the plant is forced to utilise food stored in roots
Exp) Option b is correct. (now the source). So flow in Phloem is bi-directional.
The Circulatory system of the human body including 207. Solution: (a)
the heart and various blood and lymph vessels create the
Exp) Option a is correct.
system for transport of various substances throughout
the human body, like nutrients for energy creation, waste Transpiration is a process that involves loss of water
products, etc. Arteries, Veins and Capillaries are the 3 vapour through the stomata of plants.
types of blood vessels in human body. Statement 1 is correct. Plants absorb mineral nutrients
Pair 1 is incorrect: Arteries are the blood vessels that and water from the soil. The water evaporates through the
carry oxygen rich (not poor) blood from the heart to the stomata present on the surface of the leaves by the process
entire (not just upper half) body. They are comparatively of transpiration. The evaporation of water from leaves
thick walled as the blood has to be carried under high generates a suction pull which can pull water to great
pressure. heights in the tall trees.

73 Workbook
.
SCIENCE & TECHNOLOGY

Statement 2 is correct. Reduction of atmospheric 209. Solution: (a)


pressure reduces the density of external atmosphere thus Exp) Option a is correct
permitting more rapid diffusion of water. Plants growing
on high altitudes will show higher rate of transpiration Pollination is the act of transferring pollen grains from the
hence they develop xerophytic characters. male anther of a flower to the female stigma.
Option a is correct. Seeds can only be produced when
pollen is transferred between flowers of the same species.
Option b is incorrect. The transfer of pollen from the
anther to the stigma of a flower is called pollination.
Option c is incorrect. If the pollen lands on the stigma
of the same flower or another flower of the same plant, it
is called self-pollination. In Self-pollinating, the plant do
not need a vector.
Vectors are the agents that help in the movement of
pollens.
In cross-pollinating, the plant needs a vector (a pollinator
or the wind) to get the pollen to another flower of the
same species.
Statement 3 is incorrect. Transpiration takes place both Option d is incorrect. When the pollen of a flower lands
during day and night. The rate of transpiration slows on the stigma of a flower of a different plant of the same
down during night due to decrease in rate of evaporation. species, it is called cross-pollination.
Statement 4 is correct. Transpiration cools the plant.
The loss of water vapour from the plant cools the plant
down when the weather is very hot.

208. Solution: (c)


Exp) Option c is correct.
There are several ways by which plants produce their
offspring. These are categorised into two types: (i) asexual
and (ii) sexual reproduction. In asexual reproduction 210. Solution: (b)
plants can give rise to new plants without seeds, whereas
Exp) Option b is correct.
in sexual reproduction, new plants are obtained from
seeds.
Statement 1 is incorrect. Vegetative propagation is
a type of asexual reproduction in which new plants
are produced from roots, stems, leaves and buds. Since
reproduction is through the vegetative parts of the plant,
it is known as vegetative propagation.
Statement 2 is correct. Potato sprouting from an ‘eye’
is a form of vegetative propagation. You can plant these
eyes and you can grow potatoes through vegetative
propagation method.

Hormones are chemical substances. These are secretions


from endocrine glands, or endocrine system.
Statement 1 is incorrect. Growth hormone (GH) is
secreted by the anterior pituitary gland. But the pituitary
gland is endocrine gland and not exocrine gland.

Workbook 74
.
SCIENCE & TECHNOLOGY

Statement 2 is incorrect. The liver is the largest gland in Even though all living organisms are made of cells that
the human body. contain similar structures, there are differences between
Statement 3 is correct. Pituitary gland secretes hormones the structures of the cells of plants and animals.
that make other glands such as the testes, ovaries, thyroids Statement 1 is correct. Plant cells have a cell wall which
and adrenals secrete hormones. is absent in the animals cells do not. Cell walls provide
Statement 4 is incorrect. The pituitary gland is attached support and give shape to plants.
to the brain. Thyroid gland is located in the neck region. Statement 2 is incorrect. Plant cells have chloroplasts,
but animal cells do not. Chloroplasts enable plants to
211. Solution: (c) perform photosynthesis to make food.
Exp) Option c is correct. Statement 3 is incorrect. Plant cells usually have one or
There are several ways by which plants produce their more large vacuole(s), while animal cells have smaller
offspring. vacuoles, if any are present. Large vacuoles help provide
shape and allow the plant to store water and food for future
These are categorised into two types: (i) asexual and (ii) use. The storage function plays a lesser role in animal cells,
sexual reproduction. In asexual reproduction plants can therefore the vacuoles are smaller.
give rise to new plants without seeds, whereas in sexual
Asexual reproduction In asexual reproduction new plants 214. Solution: (a)
are obtained without production of seeds. Exp) Option a is corect.
Statement 1 is correct. In plants, after fertilisation, the The process of fertilisation is the meeting of an egg
ovary grows into a fruit and other parts of the flower fall cell from the mother and a sperm cell from the father.
off. The fruit is the ripened ovary. The seeds develop from Fertilisation is categorized into two groups – Internal and
the ovules. The seed contains an embryo enclosed in a External. Fertilisation which takes place inside the female
protective seed coat. Fruit is the mature ovary whereas body is called internal fertilisation. Examples including
ovule develops into a seed, which contains the developing humans, cows, dogs and hens.
embryo. Statement 1 is correct. External fertilization is the
Statement 2 is correct. A Fruit is the seed-bearing part process of fusion of a male and a female gamete outside
of a flowering plant. So all fruits contain the seed. the body of the female. In some animals, fertilisation takes
Statement 3 is incorrect. Plants produced by vegetative place in water. When the male and female come together
propagation take less time to grow and bear flowers and in water, the female lays hundreds of eggs. A layer of
fruits earlier than those produced from seeds. The new jelly holds the eggs together and provides protection to
plants are exact copies of the parent plant, as they are the eggs. The sperms come in contact with the eggs. This
produced from a single parent. Plants produce seeds as a results in fertilisation. This type of fertilisation is called
result of sexual reproduction. external fertilisation. It is very common in aquatic animals
such as fish, starfish, etc.
212. Solution: (d) Statement 2 is incorrect. Most sexual plants use internal
Exp) Option d is correct. fertilization to reproduce. In plants, sperm and egg meet
Cells are considered the basic units of life in part because inside the ovary. Sperm comes from pollen. Stigma
they come in discrete and easily recognizable packages. catches the pollen and directs it to the ovary.
Statement 1 is incorrect. The size of the cells has no 215. Solution: (d)
relation with the size of the body of the animal or plant. Exp) Option d is correct.
It is not necessary that the cells in the elephant be much
bigger than those in a rat. The first step in the process of reproduction is the fusion
of a sperm and an ovum. The fusion of the egg and the
Statement 2 is incorrect. Cells differ in size even in a sperm is called fertilisation.
particular organism. The size of the cell is related to its
function. For example, nerve cells, both in the elephant Pair 1 is correct. Zygote begins to develop into an embryo.
and rat, are long and branched. The zygote divides repeatedly to give rise to a ball of cells.
The cells then begin to form groups that develop into
Statement 3 is incorrect. A virus is a tiny, infectious different tissues and organs of the body. This developing
particle that can reproduce only by infecting a host cell. structure is termed an embryo. It is unicellular in nature.
Viruses “commandeer” the host cell and use its resources The embryo gets embedded in the wall of the uterus for
to make more viruses, basically reprogramming it to further development.
become a virus factory. Because they can’t reproduce by
themselves (without a host), viruses are not considered Pair 2 is correct. During fertilisation, the nuclei of the
living. Nor do viruses have cells: they’re very small, much sperm and the egg fuse to form a single nucleus. This
smaller than the cells of living things, and are basically results in the formation of a fertilised egg or zygote.
just packages of nucleic acid and protein. Pair 3 is correct. The embryo continues to develop in the
uterus. It gradually develops body parts such as hands,
213. Solution: (d) legs, head, eyes, ears etc. The stage of the embryo in which
Exp) Option d is correct. all the body parts can be identified is called a foetus.

75 Workbook
.
SCIENCE & TECHNOLOGY

have been developed by this approach. For example,


Hisardale is a new breed of sheep developed in Punjab by
crossing Bikaneri ewes and Marino rams.
Pair 3 is correct. Interspecific hybridisation involves
breeding between two species. It is an animal breeding
method in which male and female animals of two
216. Solution: (c) different related species are mated. In this practice,
Exp) Option c is correct. the progeny may combine desirable features of both the
Animal breeding aims at increasing the yield of animals parents, which may help in increasing economic value of
and improving the desirable qualities of the produce. the animal in considerable way. For example, A mule is
When breeding is between animals of the same breed it is
called inbreeding, while crosses between different breeds produced by the interspecific hybridisation between male
are called out breeding. donkey and a female horse (mare).
Statement 1 is incorrect. Inbreeding refers to the mating
of more closely related individuals within the same breed 218. Solution: (a)
for 4- 6 generations. Out-crossing is the practice of Exp) Option a is correct
mating of animals within the same breed, but having no
common ancestors on either side of their pedigree up to The excretory system which is composed of a pair
4-6 generations. The offspring of such a mating is known of kidneys, a pair of ureters, a urinary bladder and a
as an out-cross. urethra. In this system, nephron is one of the important
Statement 2 is correct. Inbreeding exposes harmful component of kidney which is involved in making urine
recessive genes that are eliminated by selection. It also
helps in accumulation of superior genes and elimination as described below:
of less desirable genes. Therefore, inbreeding increases the Statement 1 is correct. Nephron is the functional unit
productivity of inbred population.
of the Kidney which is involved in producing urine. It is
Statement 3 is correct. Continued inbreeding, especially
done through the process of removing waste and excess
close inbreeding, usually reduces fertility and even
productivity. This is called inbreeding depression. substances from the blood. In human beings, excretory
Whenever this becomes a problem, selected animals of products in the form of soluble nitrogen compounds are
the breeding population should be mated with unrelated removed by the nephrons in the kidneys.
superior animals of the same breed.
217. Solution: (b)
Exp) Option b is correct
Animal’s breeding is an important aspect in the field
of animal husbandry. It aims at increasing the yield of
animals and improving the desirable qualities of the
produce. Inbreeding is a practice of breeding related
animals i.e. animals of same breed, and Out-breeding
is the practice of breeding of the unrelated animals.
Outbreeding is further subdivided into following animal
breeding practices:
Pair 1 is incorrect. Out-crossing is an animal breeding
practice between animals of same breed having no
common ancestors in recent. In this mating of animals
is done within the same breed, but having no common
ancestors on either side of their pedigree up to 4-6 Statement 2 is incorrect. Each kidney has large numbers
generations. The offspring of such a mating is known
as an out-cross. It is the considered as the best breeding of these filtration, known as nephrons, packed close
method for animals that are below average in productivity together. There are about 1,000,000 nephrons in each
in milk production, growth rate in beef cattle, etc. human kidney. Further, the basic filtration unit in the
Pair 2 is correct. Cross-breeding involves breeding of kidneys, like in the lungs, is a cluster of very thin-walled
animals of different breeds. In this, superior males of one
blood capillaries. Each capillary cluster in the kidney is
breed are mated with superior females of another breed.
Cross-breeding allows the combination of desirable associated with the cup-shaped end of a coiled tube called
qualities of two different breeds. Many new animal breeds Bowman’s capsule that collects the filtrate.

Workbook 76
.
SCIENCE & TECHNOLOGY

219. Solution: (d)


Exp) Option d is correct
Adenosine Triphosphate (ATP) is a molecule which is the
is the source of energy for use and storage at the cellular
level. In general, it is formed during the respiration
process. More specifically, the energy released during
the process of respiration is used to make an Adenosine
Triphosphate (ATP) molecule from ADP and inorganic
phosphate. Adenosine Triphosphate (ATP) molecule is
then further used to fuel all other activities inside the
cell. In the processes inside the cell, ATP is broken down
giving rise to a fixed amount of energy which can drive the
endothermic reactions which keep taking place in the cell.
For instance, The ATP can be used in the cells for
various activities like the contraction of muscles, protein
synthesis, conduction of nervous impulses and many
other activities.
220. Solution: (d)
Statement 3 is incorrect. Nephrons are found in living
beings other than human beings. The most primitive Exp) Option d is correct
nephrons are found in the kidneys of primitive fish, The cells based upon certain characters can be categorized
amphibian larvae, and embryos of more advanced as Prokaryotes and Eukaryotes. Common differences
vertebrates. The most advanced nephrons occur in the between them are as follows.
adult kidneys of land vertebrates, such as reptiles, birds,
and mammals.

Statements Characteristic Prokaryotes Eukaryotes


Statement Nucleus and No nuclear membrane In eukaryotic cells, the membrane that
1 is correct Membrane- or nucleoli (nucleoid). surrounds the nucleus commonly called
enclosed Also, Membrane-enclosed the nuclear membrane is present. It
organelles organelles are Absent in partitions DNA from the cell’s protein
Prokaryotes. synthesis machinery, which is located in
the cytoplasm.
Statement Chromosome Single circular chromosome is Such organism consists of Multiple linear
2 is correct (DNA) present. It also lacks histones chromosomes with histones
arrangement
Statement Cell division Prokaryotes propagate It is done through Mitosis. It is a process
3 is correct by binary fission, such as of cell division by which most of the cells
bacteria. divide for growth. In this process, each
cell called mother cell

77 Workbook
.
SCIENCE & TECHNOLOGY

221. Solution: (a) A wide range of organisms belonging to bacteria, viruses,


Exp) Option a is correct. fungi, protozoans, helminths, etc., could cause diseases
in man. Such disease-causing organisms are called
pathogens. The Widal test is a serological test that is
performed for typhoid fever diagnosis.
Statement 1 is correct.Typhoid fever could be confirmed
by Widal Test. Salmonella typhi is a pathogenic bacterium
which causes typhoid fever in human beings. These
pathogens generally enter the small intestine through
food and water contaminated with them and migrate to
other organs through blood.. In this test bacteria causing
typhoid fever is combined with a serum containing
specific antibodies obtained from an infected person. A
positive test result indicates the presence of a high titre of
the antigen for the disease.
Statement 2 is incorrect. No specific test exists to
diagnose Parkinson’s disease. A neurologist will diagnose
Parkinson’s disease based on the medical history, a review
Evolutionary Biology is the study of history of of your signs and symptoms, and a neurological and
life forms on earth. The term evolution refers to physical examination.
an orderly change from one form or condition to Statement 3 is incorrect. Ishihara Test is used for the
another resulting in descendants becoming different diagnosis of ‘Color-Blindness’ in people. The test contains
from ancestors. Thus, it is history and development a number of colored plates, called Ishihara plates. Each of
of race along with variations. these plates contain a circle of dots appearing randomized
Statement a is correct. Adaptive radiation is a in color and size.
process that gave rise to a variety of species that 223. Solution: (c)
originated from its original species. It is the
Exp) Option c is correct.
evolution of an animal or plant group into a wide
Statement 1 is correct. Mitochondrial replacement
variety of types adapted to specialized modes of life.
therapy (MRT) is a form of reproductive invitro
Adaptive radiations are best exemplified in closely fertilization (IVF) which works on the principle of
related groups that have evolved in a relatively short replacing a women’s mitochondrial DNA (mt-DNA) with
time. Darwin’s finches show adaptive radiation. the donor’s healthy one. MRT include different techniques
They developed different beak varieties from the like spindles transfer (ST), pronuclear transfer (PNT) or
seed-eating variety. polar body transfer (PBT).
Statement b is incorrect. Divergent evolution is the Pronuclear transfer technique is a significant approach
process whereby groups from the same common of MRT administered after fertilization, in which two
zygotes are raised in vitro.
ancestor evolve and accumulate differences,
resulting in the formation of new species. Divergent Maternal spindle transfer (MST) technique is executed
before fertilization is a form of selective reproduction
evolution may occur as a response to changes in
similar to prenatal diagnosis and pre-implantation genetic
abiotic factors, such as a change in environmental diagnosis (PGD)
conditions, or when a new niche becomes available.
Statement 2 is correct. Mitochondria are the organelle
Statement c is incorrect. Convergent evolution is which contain DNA in circular form, and in animals it is
the process when more than one adaptive radiation the only organelle in addition to nucleus which contain
occurs in a single geological isolated area. It gives DNA and gene. The sperm contains very low number
rise to analogous organs. They do not show common of mitochondria and mitochondrial gene. So, in the
ancestry but they show evolution. offspring the mitochondrial genes are inherited from the
mother. Thus, a father with mitochondrial gene defect
Statement d is incorrect. Mutation is the change cannot transmit the disease to his offspring.
in our DNA base pair sequence due to various
environmental factors such as UV light, or mistakes 224. Solution: (b)
during DNA replication. Exp) Option b is correct.
Sneezing is your body’s way of removing irritants from
222. Solution: (b) your nose or throat. A sneeze is a powerful, involuntary
Exp) Option b is correct. expulsion of air. Sneezing often happens suddenly

Workbook 78
.
SCIENCE & TECHNOLOGY

and without warning. Another name for sneezing is Option 4 is correct. Transformation of normal cells into
sternutation. cancerous neoplastic cells may be induced by physical,
Option a is incorrect. Rupture of the throat does not chemical or biological agents. These agents are called
cause sneezing. It is the halting of a sneeze that can carcinogens.
rupture your throat, burst an ear drum, or pop a blood 226. Solution: (a)
vessel in your brain Many people when they feel a sneeze
coming on—block all the exits, essentially swallowing the Exp) Option a is correct.
sneeze’s explosive force. Halting sneezing by blocking the Mitochondria are membrane-bound cell organelles
nostrils and mouth should be avoided. (mitochondrion, singular) that generate most of
Option b is correct. Allergies are an extremely common the chemical energy needed to power the cell’s
condition caused by your body’s response to foreign biochemical reactions. Chemical energy produced by
organisms. The exaggerated response of the immune the mitochondria is stored in a small molecule called
system to certain antigens present in the environment is adenosine triphosphate (ATP). Mitochondria contain
called allergy. The substances to which such an immune their own small chromosomes.
response is produced are called allergens. Common Statement 1 is correct. Mitochondria are the power-
examples of allergens are mites in dust, pollens, animal generating units of the cell and are abundant where
dander, etc. Symptoms of allergic reactions include energy-requiring processes take place, such as skeletal
sneezing, watery eyes, running nose and difficulty in and cardiac muscle. The primary role of mitochondria is
breathing. Allergy is due to the release of chemicals like to convert the products of carbohydrate, protein, and fat
histamine and serotonin from the mast cells. metabolism to CO2 and water, using key enzymes of the
electron transport chain.
Option c is incorrect. Sneezing tends to occur in the
bright sunlight and not in the diffused sunlight. Scientists Statement 2 is correct. Mitochondria are found in
believe the UV rays of the sun irritate the nose lining of nearly all eukaryotic cells. Some cells may have single
these people so they sneeze. However, it is not always true. large mitochondrion, but more often a cell has hundreds
or even thousands of mitochondria at variable location
Option d is incorrect. The mucus membrane lines the in cells depending upon the cell function. Mitochondria
nose, sinuses and throat. It moistens and warms the air we possess recognizable morphological characteristics
breathe in. It also forms a sticky mucus that prevents dust despite considerable variability in appearance. Typical
and other small particles into the nose. It does not have mitochondria are sausage shaped. In electron micrographs,
anything to do with sneezing. they appear mostly as rod-shaped or cylindrical.
225. Solution: (d) Statement 3 is incorrect. Each mitochondrion is a
Exp) Option d is correct. double membrane bound structure i.e., outer and the
inner membrane, each consisting of phospholipid’s bilayer
Cancer is one of the most dreaded diseases of human with proteins. The outer membrane is smooth, but the
beings and is a major cause of death all over the globe. inner membrane has infolding’s called cristae (singular:
A disease in which abnormal cells divide uncontrollably crista) that provides it comparatively larger surface area.
and destroy body tissue. It can start almost anywhere in The inner membrane divides the mitochondrion into
the human body, which is made up of trillions of cells. two internal compartments. The narrow region between
Normally, human cells grow and divide to form new cells the inner and outer membrane is the peri-mitochondrial
as the body needs them. When cells grow old or become space, and the innermost compartment lined by inner
damaged, they die and new cells take their place. membrane is called mitochondrial matrix.
Option 1 is correct. Hepatitis C is a liver disease. The
virus can cause both acute and chronic hepatitis. Hepatitis 227. Solution: (c)
C is a major cause of liver cancer. Antiviral medicines can Exp) Option c is correct.
cure more than 95% of persons with hepatitis C infection, A team of Indian scientists has discovered a new plant
thereby reducing the risk of death from cirrhosis and liver species in Antarctica named Bryum Bharatiensis.
cancer, but access to diagnosis and treatment is low. There Statement 1 is correct. Bryum bharatiensis is a species
is currently no effective vaccine against hepatitis C. of moss. It was discovered by a team of researchers
However research in this area is ongoing. from the Central University of Punjab. The species has
Option 2 is correct. The chemical carcinogens present in been named after Bharati, the Indian research station in
tobacco smoke have been identified as a major cause of Antarctica. The station itself was named after the Hindu
lung cancer. goddess Bharati.
Option 3 is correct. One of the major causes of cancer is Statement 2 is incorrect. Mosses are small, non-vascular
genetics as well. Cancer causing viruses called oncogenic flowerless plants in the taxonomic division Bryophyta.
viruses have genes called viral oncogenes. Furthermore, Scientists believes that during the colder months, moss
several genes called cellular oncogenes (c-onc) or proto is likely to dry up to the point that it is almost dormant
oncogenes have been identified in normal cells which, and then germinate again in September when sunlight
when activated under certain conditions, could lead to returns. When the snow melts, the dried-up moss absorbs
oncogenic transformation of the cells. the water.

79 Workbook
.
SCIENCE & TECHNOLOGY

Statement 3 is correct. The moss predominantly grows foods such as liver, butter, fatty fish (fish containing high
in areas where penguins breed in large numbers. This levels of cholesterol or fatty acids as glycerides), and egg
is because Penguin poop has nitrogen and plants need yolks. A deficiency of Vitamin D leads to diseases like
nitrogen along with potassium, phosphorus, sunlight, and Rickets in children and osteomalacia and osteoporosis
water to survive. in adults.
228. Solution: (b) Option 3 is correct: Vitamin A is a fat soluble vitamin.
It is also known as carotenoids and retinoids. VItamin
Exp) Option b is the correct answer.
A is integral to bone formation, tooth formation, and
Trans Fats are a type of fatty molecule that consists of vision. It contributes to immune and cellular function
an unsaturated bond (double or triple bond) but still while keeping the intestines working properly. It occurs
are solid like saturated fats at room temperature. Their naturally only in foods of animal origin, such as liver,
consumption has been identified by WHO as a health risk butter, whole milk, and egg yolks, but the body converts
to humans. certain carotenoids, especially β-carotene, to vitamin A.
Statement 1 is incorrect: Trans Fats are compounds that Carotenoids are present in dark-green, leafy vegetables
are a type of unsaturated fat. These are uncommon in and in yellow and orange vegetables and fruits like
nature and found only in traces in animal fats. However carrots, tomatoes, pumpkin, etc. A deficiency of Vitamin
they are manufactured on a large scale in industries to A leads to diseases like Night Blindness, etc.
create hyperpalatable junk food.
Option 4 is correct: Vitamin K is a fat soluble vitamin.
Statement 2 is correct: Like Saturated Fats (and unlike
It is central to blood clotting (helps create blood clotting
unsaturated fats), Trans Fats cause great harm to the
human body. They increase the level of LDL (Low factors like prothrombin, etc) and also keeps bones
Density Lipoprotein), also known as Bad Cholesterol healthy. Vitamin K1, (phylloquinone) is synthesized by
(which causes hypertension, cardiac issues, diabetes, plants, whereas vitamin K2 homologs (menaquinones)
etc) and reduces the levels of HDL (High Density are synthesized by bacteria. The human body can obtain
Lipoprotein) also known as Good Cholesterol. Trans-fats vitamin K from dietary sources as well as through
are associated with increased risk of heart attacks and synthesis by the gut microflora.
death from coronary heart disease. As per the World Larger amounts of vitamin K are present in dark-green
Health Organisation (WHO), approximately 5.4 lakh leafy vegetables; lower levels are found in cereals, dairy
deaths take place each year globally because of intake of products, meats, and fruits. A deficiency of Vitamin
industrially produced trans fatty acids. The WHO has also K causes excessive blood loss upon wounding due to
called for global elimination of trans-fats by 2023. inability to form blood clots.
229. Solution: (b) 230. Solution: (c)
Exp) Option b is the correct answer.
Exp) Option c is the correct answer.
Vitamins are nutrients that the body needs in small
amounts to stay healthy and work the way it should. Blood is the fluid connective tissue and it plays a very
A vitamin that can dissolve in fats and oils. They are vital role in the human body. There are four primary
absorbed into the body in different ways and excreted components of blood, which are red blood cells, white
from the body at different rates. Fat-soluble vitamins are blood cells, platelets and plasma.
absorbed along with fats in the diet and are stored in the Option a is correct: Red blood cell, also called erythrocyte,
body’s fatty tissue and in the liver. Vitamins A, D, E, and give the blood its characteristic colour and carry oxygen
K are fat-soluble. from the lungs to the tissues. The mature human red
Option 1 is incorrect: Vitamin C is a water (not fat) blood cell is small, round, and biconcave. Hemoglobin,
soluble vitamin. This means that it is dissolved in or Hb, is a protein molecule found in red blood cells
the watery components of our tissues and fluids for (erythrocytes).
immediate use and is not stored. So it needs to be Option b is correct: Platelets plays active role in blood
regularly replenished. It gets excreted with urine. clotting during injury. Any damage in the blood vessels
Vitamin C is central to iron absorption and collagen sends signals to the platelets. The platelets rush to the
synthesis. It aids in wound healing and bone formation site of damage and form clots to repair the damage. The
while improving overall immune function. It is found activated platelets stick together to form a platelet plug
in foods such as Indian gooseberry (amla), lemons, lime, which in turn activates the coagulation factor.
pineapple, oranges, watermelon, green peppers, tomatoes,
cabbage, turnip, carrots, green mint, guava, etc. A Option c is incorrect: White Blood Cells are produced in
deficiency of Vitamin C leads to diseases like Scurvy, etc. the red bone marrow, lymph nodes, and spleen. While,
Option 2 is correct: Vitamin D is a fat soluble vitamin. It Red cells are produced continuously in the marrow of
aids in the development of teeth and bone by encouraging certain bones.
the absorption and metabolism of phosphorus and Option d is correct: White blood cells circulate around the
calcium. Vitamin D occurs naturally only in animal blood and help the immune system fight off infections.

Workbook 80
.
SCIENCE & TECHNOLOGY

These cells circulate through your bloodstream and tissues Statement 4 is correct. The cell membrane also provides
to respond to injury or illness by attacking any unknown some structural support for a cell. Cell membrane also
organisms that enter your body. helps in maintaining the shape of the cell.
231. Solution: (c)
Exp) Option c is the correct answer.
Statement 1 is correct. If the pollen lands on the stigma
of the same flower or another flower of the same plant, it is
called self-pollination. When the pollen of a flower lands
on the stigma of a flower of a different plant of the same
kind, it is called cross-pollination.
Statement 2 is correct. Flowers which contain either
only pistil or only stamens are called unisexual flowers.
Flowers which contain both stamens and pistil are called
bisexual flowers. Corn, papaya and cucumber produce
unisexual flowers, whereas mustard, rose and petunia
have bisexual flowers.
234. Solution: (b)
232. Solution: (c) Exp) Option b is the correct answer.
Exp) Option c is the correct answer. A major difference between plant and animal cells is the
Type of fertilisation in which the fusion of a male and a presence of a cell wall. It surrounds the cell membrane
female gamete takes place outside the body of the female (also called the plasma membrane) of plant cells. The
is called external fertilisation. cell wall provides tensile strength and protection against
mechanical and osmotic stress. It helps cells to develop
Option 2 is incorrect. Fertilisation which takes place turgor pressure.
inside the female body is called internal fertilisation.
Option a is incorrect: Chlorophyll is found in virtually
Internal fertilisation occurs in many animals including
all photosynthetic organisms, including green plants,
humans, cows, dogs and hens. cyanobacteria, and algae. It absorbs energy from light;
Option 1, 3 and 4 are correct. External Fertilisation is this energy is then used to convert carbon dioxide to
very common in aquatic animals such as fish, starfish, etc. carbohydrates. However, it does not explain the reason
Other examples of external fertilization: Nearly all fish that plant cells can withstand much greater changes.
spawn, as do crustaceans (such as crabs and shrimp), Option b is correct: Plant cell can withstand much greater
mollusks (such as oysters), squid, and echinoderms (such changes in the surrounding medium than animal cell this
as sea urchins and sea cucumbers), frogs, starfish. is because plant cell has an extra cell wall above it which
gives them support during absorption or emission
233. Solution: (d) for of water. It holds the organelle inside the cell intact
Exp) Option d is the correct answer. and prohibits bursting. Plants require protection from
osmotic stress and mechanical pressure. These functions
The basic components of a cell are cell membrane, are provided by this additional layer.
cytoplasm and nucleus. The cytoplasm and nucleus are Option c is incorrect: Plant cell walls are primarily made of
enclosed within the cell membrane, also called the plasma cellulose. Cellulose is the most abundant macromolecule
membrane. on Earth. Its fibers are long, linear polymers of glucose
The plasma membrane, or the cell membrane, provides molecules. However, it does not explain the reason that
protection for a cell. It also provides a fixed environment plant cells can withstand much greater changes.
inside the cell, and that membrane has several different Option d is incorrect: Plant cells show active osmosis.
functions. For example, if a plant cell is surrounded by a solution
Statement 1 and 2 are correct. One of the functions of that contains a higher concentration of water molecules
cell membrane is to transport nutrients into the cell and than the solution inside the cell, water will enter the cell
by osmosis and the plant cell will become turgid (firm).
also to transport toxic substances out of the cell.
However, it does not explain the reason that plant cells can
Statement 3 is correct. The cell membrane, also called withstand much greater changes.
the plasma membrane, is found in all cells and separates
the interior of the cell from the outside environment. 235. Solution: (a)
Plasma membrane, have proteins on it which interact with Exp) Option a is the correct answer.
other cells. And those proteins which stick outside of the Muscle is a specialized tissue of mesodermal origin.
plasma membrane will allow for one cell to interact with About 40-50 per cent of the body weight of a human adult
another cell. is contributed by muscles. They have special properties

81 Workbook
.
SCIENCE & TECHNOLOGY

like excitability, contractility, extensibility and elasticity. Statement 2 is correct: Fungi are more like animals
Option 1 is correct: Visceral muscles are located in the because they are heterotrophs, as opposed to autotrophs,
inner walls of hollow visceral organs of the body like like plants, that make their own food. Fungi have to obtain
the alimentary canal, reproductive tract, etc. They do not their food, nutrients and glucose, from outside sources.
exhibit any striation and are smooth in appearance. Hence, Also, Fungi are more complicated organisms than viruses
they are called smooth muscles (nonstriated muscle). and bacteria—they are “eukaryotes,” which means they
Their activities are not under the voluntary control of the have cells.
nervous system and are therefore known as involuntary 237. Solution: (d)
muscles. They assist, for example, in the transportation of
food through the digestive tract and gametes through the Exp) Option d is the correct answer.
genital tract. Statement a is incorrect: The ribosomes, which
Option 2 is correct: Cardiac muscles are the muscles are present in all active cells, are the sites of protein
of the heart. Many cardiac muscle cells assemble in manufacture.
a branching pattern to form a cardiac muscle. Based Statement b is incorrect: The smooth endoplasmic
on appearance, cardiac muscles are striated. They are reticulum (SER) helps in the manufacture of fat
involuntary in nature as the nervous system does not molecules, or lipids, important for cell function.
control their activities directly.Option 3 is incorrect: Statement c is incorrect: In the liver cells, SER plays a
Skeletal muscles are closely associated with the skeletal crucial role in detoxifying many poisons and drugs.
components of the body. They have a striped appearance Statement d is correct: Mitochondria are known as the
under the microscope and hence are called striated powerhouses of the cell. Mitochondria are known as the
muscles. As their activities are under the voluntary powerhouse of the cell because it is responsible for the
control of the nervous system, they are known as extracting energy from food through cellular respiration.
voluntary muscles too. They are primarily involved The energy required for various chemical activities needed
in locomotory actions and changes of body postures. for life is released by mitochondria in the form of ATP
Option 4 is incorrect: Voluntary muscles are the ones (Adenosine triphopshate) molecules.
that move or contract under the conscious control of a
person. Pectoral muscles, hamstrings, biceps, triceps, 238. Solution: (c)
quadriceps, abdominals, etc. are some of the examples of Exp) Option c is the correct answer.
voluntary muscles. Pectorals muscle is any of the muscles
that connect the front walls of the chest with the bones Statement 1 is correct: Plants are stationary or fixed –
they don’t move. Since they have to be upright, they have
of the upper arm and shoulder. There are two such
a large quantity of supportive tissue. The supportive
muscles on each side of the sternum (breastbone) in the
tissue generally has dead cells. Animals on the other
human body: pectorals major and pectorals minor.
hand move around in search of food, mates and shelter.
They consume more energy as compared to plants. Most
of the tissues they contain are living.
Statement 2 is correct: The growth in plants is limited
to certain regions, while this is not so in animals. There
are some tissues in plants that divide throughout their
life. These tissues are localized in certain regions. Cell
growth in animals is more uniform. So, there is no such
demarcation of dividing and nondividing regions in
animals.
239. Solution: (c)
Exp) Option c is the correct answer.
Cell cycle is the sequence of events by which a cell
duplicates its genome, synthesises the other constituents
of the cell and eventually divides into two daughter cells.
236. Solution: (a) Cell division is a very important process in all living
organisms. During the division of a cell, DNA replication
Exp) Option a is the correct answer. and cell growth takes place. All these processes, i.e., cell
Using visible movement as the defining characteristic of division, DNA replication, and cell growth, hence, have to
life is not enough. Movements over very small scales will take place in a coordinated way to ensure correct division
be invisible to the naked eye – movements of molecules, and formation of progeny cells containing intact genomes.
for example. Statement 1 is correct: The Meiosis is the process
Statement 1 is correct: In fact, viruses do not show any in eukaryotic, sexually-reproducing animals that
molecular movement in them until they infect some reduces the number of chromosomes in a cell before
cell, and that is partly why there is a controversy about reproduction. Many organisms package these cells into
whether they are truly alive or not. gametes, such as egg and sperm. The gametes can then

Workbook 82
.
SCIENCE & TECHNOLOGY

meet, during reproduction, and fuse to create a new Option 2 is correct: After the catastrophe of the BP oil
zygote. Because the number of alleles (It is a variant form spill in 2010, scientists observed many species of bacteria
of a gene) was reduced during meiosis, the combination of that broke down the oil, including one previously
two gametes will yield a zygote with the same number of unknown to science. The oil spill was the largest in history,
alleles as the parents. dumping almost 5m barrels of oil into the Gulf of Mexico.
The production of offspring by sexual reproduction The bacteria that feasted on the oil compounds played
includes the fusion of two gametes, each with a complete an instrumental role in reducing the impact of the spill
haploid set of chromosomes (Haploid is a cell that contains and could be used if any future disasters occur.
a single set of chromosomes). Gametes are formed from Option 3 is correct: Microbial decomposition of
specialised diploid cells.(Diploid is a cell or organism that petroleum products by hydrocarbon-oxidizing bacteria
has paired chromosomes). This specialised kind of cell and fungi is of considerable ecological importance.
division that reduces the chromosome number by half The microbial decomposition of petroleum is an aerobic
results in the production of haploid daughter cells. This process, which is prevented if the oil settles to the layer
kind of division is called meiosis. of anaerobic sediment at the bottom (natural oil deposits
Statement 2 is correct: Mitosis or the equational in anaerobic environments are millions of years old).
division is usually restricted to the diploid cells only. Hydrocarbon-oxidizing bacteria attach to floating
However, in some lower plants and in some social insects’ oil droplets on the water surface, where their action
haploid cells also divide by mitosis. It is essential to eventually decomposes the oil to carbon dioxide. It is
understand the significance of this division in the life of becoming a common practice to spray such bacteria
an organism. and their growth factors onto oil spills to enhance the
Mitosis usually results in the production of diploid rate of degradation of petroleum products.
daughter cells with identical genetic complement. The Option 4 is correct: In 2016, Japanese scientists identified
growth of multicellular organisms is due to mitosis. Cell a species of bacterium that feeds on Polyethylene
growth results in disturbing the ratio between the nucleus terephthalate (PET)- the plastic which is used to make
and the cytoplasm. It therefore becomes essential for the disposable drink bottles. The bacteria use two enzymes
cell to divide to restore the nucleo-cytoplasmic ratio. A to break the PET down into two components that can
very significant contribution of mitosis is cell repair. then be used to synthesise new PET, breaking the reliance
The cells of the upper layer of the epidermis, cells of the on fossil fuels and reducing plastic waste. The scientists
lining of the gut, and blood cells are being constantly are currently working on optimising these enzymes to
replaced. Mitotic divisions in the meristematic tissues – make recycling PET more efficient.
the apical and the lateral cambium, result in a continuous
growth of plants throughout their life 241. Solution: (c)
240. Solution: (d) Exp) Option c is the correct answer.
Exp) Option d is the correct answer. 67% of all disease loads are caused by non-communicable
diseases (NCDs) globally. Metabolic and behavioural risk
Bacteria are small single-celled organisms. Bacteria are
factors are the most common causes of noncommunicable
found almost everywhere on Earth and are vital to the
diseases. A group of these risk factors is called Metabolic
planet’s ecosystems. Some species can live under extreme
conditions of temperature and pressure. Syndromes which have contributed significantly to the
rise in NCDs.
Scientists have discovered a way of using bacteria to
produce graphene – a nanomaterial composed of a single Option 1 is correct: Abdominal obesity is defined as
layer of carbon atoms with extraordinary properties and a waist circumference of more than 88 cm (35 inches)
they have also found a genetically modified bacteria to in women and more than 102 cm (40 inches) in men.
detect tumours in mice. All obesities do not bring an equal risk of developing a
disease, which reflects the heterogeneity of this condition.
Option 1 is correct: Certain bacteria convert milk into The regional distribution of body fat is now recognized
useful dairy products, such as buttermilk, yogurt, and as a very important component of the obesity-related
cheese. Commercially cultured buttermilk is prepared health hazards. Epidemiological studies have shown
from milk injected with a microorganism, which are
that abdominal obesity, that is, a preponderance of
deliberately used to assist the beginning of fermentation
fat in the abdominal area, is a better predictor of both
of Lactococcus. Yogurt and other fermented milk
cardiovascular disease and type 2 diabetes than obesity
products are produced in a similar manner using different
cultures of bacteria. Many cheeses are likewise made It is associated with high levels of LDL (“bad”) cholesterol
through the action of bacteria. Growth in milk of an acid- and triglycerides and low levels of HDL (“good”)
producing bacterium such as L. lactis causes the casein to cholesterol. It brings metabolic changes like impaired
precipitate as curd. Following the removal of moisture and body responsiveness to insulin, raising of blood sugar
the addition of salt, the curd is allowed to ripen through and insulin levels. It contributes to major NCDs causing
the action of other microorganisms. Different bacteria death and disability, including heart attacks, strokes,
impart different flavours and characteristics to foods. e.g., high blood pressure, cancer, diabetes, osteoarthritis,
probiotic milk [yakult], flavoured yoghurt fatty liver, and depression.

83 Workbook
.
SCIENCE & TECHNOLOGY

Option 2 is incorrect: Insulin sensitivity refers to how Option b is correct: Plastids containing chlorophyll are
sensitive the body’s cells are in response to insulin. It known as the chloroplasts (green coloured plastids).
is not good or bad in itself. If a person has low insulin Majority of the chloroplasts of the green plants are
sensitivity, his body needs higher amounts of insulin to found in the mesophyll cells of the leaves. These are
clear the glucose from their bloodstream which may lead lens-shaped, oval, spherical, discoid or even ribbon-like
to disorders like hyperglycemia leading to NCDs like organelles having variable length (5-10μm) and width (2-
Type 2 diabetes. Presence of high amounts of insulin in 4μm). The chloroplasts are also double membrane bound.
blood has been associated with damage to blood vessels,
high blood pressure, heart disease and heart failure, Option c is incorrect: The leucoplasts are the colourless
obesity, osteoporosis and even cancer. plastids of varied shapes and sizes with stored nutrients:
Amyloplasts store carbohydrates (starch), e.g., potato;
Option 3 is correct: Hyperglycemia is the technical term
elaioplasts store oils and fats whereas the aleuroplasts
for high blood glucose (blood sugar). High blood sugar
happens when the body has too little insulin or when store proteins.
the body can’t use insulin properly. This occurs due to Option d is incorrect: Chloroplasts contain carotenoid
a disbalance in the body’s capability to metabolise food to pigments (not chlorophyll pigments) which are
maintain the levels of glucose in the blood. It may lead to responsible for trapping light energy essential for
insulin induced coma or ketoacidosis (accumulation of photosynthesis. In contrast, chloroplasts are green
toxic metabolites in blood). It is associated with the NCD coloured plastids which provides green colour to the
and often results in hospitalisation and sometimes death. leaves. The stroma of the chloroplast contains enzymes
Option 4 is correct: Dyslipidemia is defined as having required for the synthesis of carbohydrates and proteins.
blood lipid levels that are too high or low. A certain level
of lipids/ fats (like HDL cholesterol, etc) are essential for 244. Solution: (b)
bodily functions. If it is too low many body functions like Exp) Option b is the correct answer.
production of hormones, menstrual cycle in women, All living organisms can be sorted into one of two groups
etc get disrupted. If they are too high (especially bad depending on the fundamental structure of their cells: the
cholesterol like LDL) then they lead to atherosclerosis prokaryotes and the eukaryotes.
- coronary blockages increasing the risk of NCDs like
heart attack, stroke, etc. Eukaryotic cells contain a nucleus and organelles bound
by plasma membranes. Fungi, plants, and animals are
242. Solution: (b) made of eukaryotic cells (eukaryotes). Prokaryotic cells
Exp) Option b is the correct answer. do not have a membrane-bound nucleus or organelles. All
Cells in the living organisms are basic structural units and bacteria and members of Archaea are made of prokaryotic
complex living structures. cells (prokaryotes).
Statement 1 is incorrect: The size of cells in living
organisms may be as small as a millionth of a metre
(micrometre or micron) or may be as large as a few
centimetres. However, most of the cells are microscopic
in size and are not visible to the unaided eye. They need
to be enlarged or magnified by a microscope. The smallest
cell is 0.1 to 0.5 micrometre in bacteria. The largest cell
measuring 170 mm ×130 mm, is the egg of an ostrich.
Statement 2 is correct. The size of the cells has no
relation with the size of the body of the animal or plant.
It is not necessary that the cells in the elephant be much
bigger than those in a rat. The size of the cell is related to
its function. For example, nerve cells, both in the elephant
and rat, are long and branched. They perform the same
Differences in cellular structure of prokaryotes and
function that of transferring messages.
eukaryotes include the presence of mitochondria
243. Solution: (b) and chloroplasts, the cell wall, and the structure of
Exp) Option b is the correct answer. chromosomal DNA.
Plastids are found in all plant cells and in euglenoides. Statement 1 is incorrect. No prokaryotic cell has a
These are easily observed under the microscope as they are nucleus; every eukaryotic cell has a nucleus. Prokaryotic
large. They bear some specific pigments, thus imparting cells have no mitochondria; nearly every eukaryotic cell
specific colours to the plants. has mitochondria.
Option a is incorrect: In the chromoplasts fat soluble Mitochondria are rod-shaped organelles that are
carotenoid pigments like carotene, xanthophylls and considered the power generators of the cell. During
others are present. This gives the part of the plant a yellow, cellular respiration, mitochondria convert glucose and
orange or red colour. oxygen to produce adenosine triphosphate (ATP), which

Workbook 84
.
SCIENCE & TECHNOLOGY

is the biochemical energy “currency” of the cell to do any Vitamin B6 Involved in Microcytic
cellular activities.
greater than 100 anemia, scaling
Statement 2 is incorrect. Both prokaryotic and eukaryotic enzyme reactions of the lips
cells have similar chemical compositions in the body and and cracks in
For example, both types of cells use and/or contain: involved in protein the corners
• Carbohydrates metabolism of the mouth,
• Lipids swollen tongue,
depression, and
• Proteins (long chains of amino acids)
confusion (17).
• DNA and RNA (long chains of nucleic acids)
Vitamin B12 Involved in Megaloblastic
Statement 3 is correct. Prokaryotes are organisms made
red blood cell anemia, fatigue,
up of cells that lack a cell nucleus or any membrane-
formation, weakness,
encased organelles. Eukaryotes are organisms made up of
cells that possess a membrane-bound nucleus that holds neurological constipation,
genetic material as well as membrane-bound organelles. function, and DNA loss of appetite
synthesis. and weight loss
245. Solution: (d) (13).
Exp) Option d is the correct answer. Vitamin C Involved in the Development
Genetic diseases are pathological conditions that are formation of of scurvy
caused or influenced by abnormalities in the genetic collagen, certain which would
material (genome) of a person and these are caused neurotransmitters, include fatigue,
by gene mutations and chromosomal abnormalities. and protein inflammation
It is categorized into- Single Gene Genetic diseases; synthesis, of the gums,
Polygenic genetic diseases; Chromosomal diseases and and weakened
Mitochondrial DNA disorders. connective
Statement 1 is correct. Klinefelter syndrome is a tissue. (14).
chromosomal disease caused by the loss or gain of one
or more chromosomes or by alterations in chromosome Vitamin D Promotes calcium Development
structure. absorption and of rickets in
proper bone children or
Statement 2 is correct. Sickle cell anaemia is a monogenic
formation, involved osteomalacia
genetic disease or Mendelian genetic disorders caused
by changes or mutations that occur in the DNA sequence in cell growth, in adults, and
of a single gene. Single gene disorders are passed on immune function, fatigue (18).
between generations from parents to offspring. and reduces
inflammation.
Statement 3 is correct. Hypertension is a Polygenic
genetic disease or multifactorial genetic disorder caused Calcium Involved in muscle Development
due to the interaction of multiple genes and interaction function nerve of osteoporosis
between genes and environmental factors. transmission, (19).
and proper bone
246. Solution: (c)
formation.
Exp) Option c is the correct answer
Folate Involved in the Megaloblastic
Deficiency of different vitamins and minerals may impact synthesis of RNA anemia (20).
our health. It results in certain diseases or disorders.
and DNA and
However, if a proper or balanced diet is taken, all
deficiency diseases can be prevented. is required for
cell division and
Option 1 and 4 are correctly matched. the prevention
Nutrient Function Signs and of Neural Tube
Defects.
symptoms of
deficiency Iodine A component of Stunted growth
thyroid hormones and neuro
Vitamin A Involved in Night
that regulate developmental
immune function blindness and
protein synthesis deficits (7).
vision cell xerophthalmia
metabolism, and
growth and cell (16).
enzyme activity
communication.

85 Workbook
.
SCIENCE & TECHNOLOGY

Iron A component of Microcytic, environmental resources are limited. During struggle


for existence, organisms with advantageous variations
hemoglobin and hypochromic are protected and allowed to reproduce while the
therefore important anemia; disadvantageous variants are eliminated from nature.
in the transfer of impaired This is what was termed natural selection by Darwin.
oxygen from the cognitive Statement c is correct. The idea of Darwin’s evolution
lungs to organism function, by natural selection is based on another fact that every
and involved in poor body species, in the absence of environmental checks tends
the synthesis of temperature to increase in geometrical manner. If a population of
hormones as well as regulation, a given species doubles in one year and if there are no
normal growth and depressed checks on its increase, it will increase four times the next
development. immune year and so on.
function, and Statement d is incorrect. Darwin defined evolution as
spoon like shape “descent with modification,” the idea that species change
of nails (9). over long time, give rise to new species, and share a
common ancestor. New species is not a result of sudden
Magnesium Involved in more Loss of appetite, change according to Darwin.
than fatigue,
weakness, Whereas, Hugo de Vries in his mutation theory of
300 enzyme evolution believed new species rise by sudden changes
nausea,
called mutations rather than by gradual processes.
According to him it was mutation and not natural
Nutrient Function Signs and selection which is the primary factor in evolution.
symptoms of
Deficiency 248. Solution: (d)
Magnesium Reactions, protein Vomiting, Exp) Option d is the correct answer.
(Continued) synthesis, muscle numbness, Genetic material is the hereditary substance in the
function, nerve tingling muscle cell known as DNA (deoxyribonucleic acid) or RNA
function, blood cramps, seizures, (ribonucleic acid) which carries all information specific
sugar control, and personality to an organism.
blood pressure changes, and Statement 1 is correct. Replication is an important
control abnormal heart criterion for a molecule to act as a genetic material eg DNA
rhythms (15). and RNA have the ability to direct their duplications. But
molecules such as protein fail to generate its replica thus,
Zince Involved in cell Stunted growth they cannot be categorised as genetic material.
metabolism, depressed Statement 2 is correct. A molecule to act as a genetic
enzyme activity, immune material should provide the scope for slow changes
immune functions, function, hair, eye (mutation) that are required for evolution. A mutation is
protein synthesis, and skin lesions, a change in a DNA sequence.
wound healing delayed wound Statement 3 is correct. Stability as one of the properties
DNA synthesis and healing, and taste of genetic material. The genetic material should be
cell division. alterations (21). chemically and structurally stable enough not to change
247. Solution: (d) with different stages of life cycle, age or with change in
physiology of the organism.
Exp) Option d is the correct answer.
Various theories about the mechanism of evolution 249. Solution: (d)
have been proposed which includes Lamarck’s theory of Exp) Option d is the correct answer.
“Inheritance of acquired characters”; De Vries’ theory of
‘mutation’ and Darwin’s theory of Natural selection. Blood has four main components: plasma, red blood
cells, white blood cells, and platelets.
Statement a is correct. Charles Darwin emphasizing on
gradual evolution of life forms concluded that existing Statement 1 is correct: Platelets are also called
living forms share similarities to varying degrees not thrombocytes. These are cell fragments produced from
only among themselves but also with life forms that special cells in the bone marrow. Platelets can release a
existed millions of years ago. Many such life forms variety of substances most of which are involved in the
do not exist anymore as there had been extinctions of coagulation or clotting of blood. A reduction in their
different life forms in the years gone by just as new forms number can lead to clotting disorders which will lead to
of life arose at different periods of history of earth. excessive loss of blood from the body.
Statement b is correct. According to Darwin, organisms Statement 2 is correct: Plasma also contains small
produce more offspring than can survive because amounts of minerals like Na+, Ca++, Mg++, HCO3-

Workbook 86
.
SCIENCE & TECHNOLOGY

, Cl-, etc. Glucose, Amino acids, Lipids, etc., are also or adaptive immune response, and it has developed
present in the plasma as they are always in transit in the and evolved to protect the host from the surrounding
body. Factors for coagulation or clotting of blood are also environment in which a variety of toxins and infectious
present in the plasma in an inactive form. agents including bacteria, fungi, viruses and parasites are
Statement 3 is correct: Plasma is a straw coloured, found.
viscous fluid constituting nearly 55 per cent of the Statement 1 is correct: Tears provides lubrication,
blood. 90-92 per cent of plasma is water and proteins prevents drying of the ocular surface epithelia, helps
contribute 6-8 per cent of it. provide a smooth surface for refracting light, supplies
oxygen. Antimicrobial compounds found in tears such as
250. Solution: (d) lysozyme. So it is an important component of the innate
Exp) Option d is the correct answer. defense system of the eye providing protection against a
DNA extraction is used to extract a useable DNA sample range of potential pathogen.
for further examination. Commercial extraction kits Statement 2 is correct: On tooth surfaces saliva plays an
are available and provide all components necessary for important role in acquired pellicle formation. Acquired
extraction. DNA can be extracted from different biological pellicle plays a major role in crystal growth homeostasis
samples, such as; of the teeth, and in physico-chemical defense of tooth
1. Fingernail material -can serve as an important surfaces. Acquired pellicle plays a major role in bacterial
source of DNA. Because of the special composition adhesion on tooth surfaces.
and structure of fingernails that embody DNA in Statement 3 is correct: Skin acts not only as a physical
keratinized cells, DNA extraction procedures are barrier but also as a chemical shield. Sebaceous glands
more complex than usual protocols applied for fresh associated with hair follicles produce large amounts of
somatic cells. fatty acids which create an acidic environment that is
2. Skin – DNA can be extracted from skin cells, skin hostile to microorganisms.
cells from the lower layers produce the best result.
3. Hair – Hair roots contain genomic DNA while hair 253. Solution: (d)
shafts contain mitochondrial DNA. Exp) Option d is the correct answer.
4. Blood – Blood is an excellent source of human DNA. Microorganisms are an organism that can be seen only
DNA is present in white blood cells of humans, but through a microscope. Microorganisms include bacteria,
not red blood cells which lack nuclei. protozoa, algae, and fungi. viruses are not considered
5. Saliva – DNA comes from white blood cells and living organisms but sometimes they are classified as
buccal epithelial cells (cheek cells) in the saliva. microorganisms.
251. Solution: (b) Statement 1 is correct: Coliform bacteria are organisms
that are present in the environment and in the feces of all
Exp) Option b is the correct answer.
warm-blooded animals and humans. Coliform bacteria
Pair 1 is correct: Kyasanur Forest disease (KFD) is caused will not likely cause illness. However, their presence
by Kyasanur Forest disease virus (KFDV), a member of in drinking water indicates that disease-causing
the virus family Flavivirida. Hard ticks are the reservoir of organisms (pathogens) could be in the water system. Most
the KFD virus and once infected, remain so for life. pathogens that can contaminate water supplies come from
Pair 2 is incorrect: Leishmaniasis is caused by protozo- the feces of humans or animals. Testing drinking water for
an parasites which are transmitted by the bite of infect- all possible pathogens is complex, time-consuming, and
ed female phlebotomine sandflies. expensive. It is relatively easy and inexpensive to test for
coliform bacteria. If coliform bacteria are found in a water
Pair 3 is incorrect: Brucellosis is a bacterial infection sample, water system operators work to find the source of
that spreads from animals to people. Most commonly, contamination and restore safe drinking water.
people are infected by eating raw or unpasteurized dairy Statement 2 is correct: We use microbes or products
products. Sometimes, the bacteria that cause brucellosis derived from them every day. A common example is the
can spread through the air or through direct contact with
production of curd from milk. Micro-organisms such as
infected animals.
Lactobacillus and others commonly called lactic acid
Pair 4 is correct: Typhoid fever is caused by Salmonella bacteria (LAB) grow in milk and convert it to curd.
typhi bacteria. Typhoid fever is rare in developed During growth, the LAB produces acids that coagulate
countries. It is still a serious health threat in the developing and partially digest the milk proteins. A small amount of
world, especially for children. curd added to the fresh milk as inoculum (An inoculum
is the population of microorganisms or cells that is
252. Solution: (d)
introduced in the fermentation medium or any other
Exp) Option d is the correct answer. suitable medium) or starter contain millions of LAB,
The innate immune response is the first mechanism for which at suitable temperatures multiply, thus converting
host defense found in all multicellular organisms. The milk to curd, which also improves its nutritional quality
innate immune system is more ancient than the acquired by increasing vitamin B12. In our stomach too, the LAB

87 Workbook
.
SCIENCE & TECHNOLOGY

play very beneficial role in checking disease causing 255. Solution: (c)
microbes. Exp) Option c is the correct answer.
Statement 3 is correct: Lactic Acid Bacteria originally Assertion is correct. Colour Blindness is a sex-linked
found on plants. Lactic acid bacteria can find their recessive disorder due to defect in either red or green
way to milk and constitute normal flora. Others are cone of eye resulting in failure to discriminate between
found in the intestine of humans and animals. This group red and green colour. This defect is due to mutation in
includes facultative anaerobes, non-spore forming, gram- certain genes present in the X chromosome. It occurs in
positive, catalase negative cocci and rods. These bacteria about 8 per cent of males and only about 0.4 per cent of
utilize lactose (milk sugar) as a source of carbon and females.
convert it into lactic acid (lactic acid fermentation). Lactic Reason is incorrect. Breakdown of protein is linked
acid fermentation may be homofermentative (exclusive with cataracts here. Cataract is a cloudy area in the lens
production of lactic acid) or Fundamentals of Dairy of your eye which happen because of normal changes in
heterofermentative (besides lactic acid, production of eyes as one gets older. Around age 40, the proteins in the
other compounds e.g., acetic acid, carbon dioxide, ethanol lens of your eye start to break down and clump together.
as end products). This clump makes a cloudy area on your lens or a cataract.
However, there is no evidence that breakdown of proteins
254. Solution: (c) in the lens of male’s eyes is faster than in female’s eyes.
Exp) Option c is the correct answer. Hence this reason statement is incorrect.
B-cells and T-cells are a type of lymphocyte (cell flowing Reason is not the right explanation of Assertion. Colour
in the lymphatic system of the body) which is responsible blindness is more common in men than in women
for the mechanism of immunity in the human body. Its because the genes responsible for the color blindness are
function is to recognize foreign matter and attack them. located on the X chromosome. Males have only one X
They are involved in the acquired or antigen-specific chromosome whereas females have two X chromosomes.
immune response and present a line of defence that uses In males, only one defective X chromosome is enough
past behaviours and interactions to learn to recognize to cause colour blindness. In females, two defective X
specific foreign threats and attack them when they chromosomes are required to cause colour blindness. This
reappear. type of inheritance pattern is called X-linked inheritance
Statement a is incorrect: Both B-cells and T-cells and it primarily affects males.
are present in the lymphatic system of the body.
The leukocytes (white blood cells) present in the
bloodstream do not provide adaptive immunity. They
are responsible for innate (non-specific) immunity.
Examples are neutrophils, basophils, etc.
Statement b is incorrect: Both B-cells and T-cells are
produced in the bone marrow.
It must however be noted that they mature in different
locations. While the B-cells remain in marrow to mature,
the T-cells migrate to the Thymus (a lymphatic gland
near the lungs) in order to mature.
Statement c is correct: While the function of B-cells
is specifically to create antibodies which help kill the 256. Solution: (d)
marked/ infected cells, the T-cells are of 2 types Exp) Option d is the correct answer.
• Helper T-cells: Stimulate B-cells to make antibodies. Statement 1 is correct: Frogs can also breathe through
• Killer T-cells their skin though they have lungs. They need to keep
Thus T-cells too have a role in creation of antibodies, their skin moist to be able to breathe through their skin,
even if not direct. so if their skin dries out they are not able to absorb oxygen.
They use their skin to absorb oxygen when underwater,
Statement d is incorrect: The second type of T-cell, the but if there is not enough oxygen in the water, they will
Killer T-cell is involved in the actual destruction of a drown.
body cell that is infected.
Statement 2 is correct: A cockroach has small openings
The B-cells produce antibodies which are Y-shaped on the sides of its body. Other insects also have similar
proteins that are specific to each pathogen and are able openings. These openings are called spiracles. Insects have
to lock onto the surface of an invading cell and mark it a network of air tubes called tracheae for gas exchange.
for destruction by other immune cells. So, the B-cells do Oxygen rich air rushes through spiracles into the tracheal
not actually kill the infected cells, they just make sure tubes, diffuses into the body tissue, and reaches every cell
that the infected cells are marked for destruction by the of the body. Similarly, carbon dioxide from the cells goes
antibodies that the B-cells produce. into the tracheal tubes and moves out through spiracles.

Workbook 88
.
SCIENCE & TECHNOLOGY

accomplished by providing different types of barriers to


the entry of the foreign agents into our body, e.g., skin on
our body is the main barrier which prevents entry of the
micro-organisms.
Statement 2 is correct: Acquired Immunity is pathogen
specific immunity which is acquired after encounter of
the pathogen. This means when our body encounters a
pathogen for the first time it produces a response called
primary response which is of low intensity. Subsequent
encounter with the same pathogen elicits a highly
intensified secondary or anamnestic response. Thus,
acquired immunity is characterized by memory. The
primary and secondary immune responses are carried out
with the help of two special types of lymphocytes present
Statements 3 is correct: Some prokaryotes—bacteria in our blood, i.e., B-lymphocytes and T-lymphocytes.
and archaea—that live in low-oxygen or no oxygen Statement 3 is correct: The principle of immunization
environments rely on anaerobic respiration i.e. or vaccination is based on the property of ‘memory’ of
respiration without the oxygen to break down fuels. the immune system. Under this, antigenic proteins of
For example, some archaea called methanogens can pathogen or inactivated/weakened pathogen (vaccine)
use carbon dioxide as a terminal electron acceptor, are introduced into the body. The antibodies produced
producing methane as a by-product. in the body against these antigens would neutralise the
pathogenic agents during actual infection.
257. Solution: (d)
Exp) Option d is correct. 258. Solution: (b)
Statement 1 is correct: Innate immunity is present at the Exp) Option b is correct.
time of birth; it is a non-specific type of defence. This is A fungus is any member of the group of eukaryotic

89 Workbook
.
SCIENCE & TECHNOLOGY

Workbook 90
.
SCIENCE & TECHNOLOGY

organisms. (Eukaryotic organisms are any cell or


organism that possesses a clearly defined nucleus). There
are any of about 144,000 known species of organisms
of the kingdom Fungi, which includes the yeasts, rusts,
smuts, mildews, Molds, and mushrooms
Statement 1 is incorrect: Yeast is a unicellular fungus
(and not Multicellular) producing moist to waxy colony.
It is about 5-10 times larger than bacteria and reproduces
asexually by budding or transverse division and sexually
through spore formation Morphologically, these are
usually spherical or ellipsoidal in shape.
Statement 2 is correct: Yeast can reproduce both
sexually and asexually. Asexual mode of reproduction
includes budding, binary fission and more commonly 260. Solution: (c)
through spore production. Exp) Option c is correct.
Statement 3 is correct: Fungi grow in moist Chromosomes are thread-like structures located inside
environment and are chemo-organoheterotrophs. the nucleus of animal and plant cells. Each chromosome is
The term chemotroph describes organism that generate made of protein and a single molecule of deoxyribonucleic
energy by oxidation of organic or inorganic compounds. acid (DNA).
Chemo organoheterotrophs use organic compound as a Option a is correct: The unique structure of chromosomes
source of energy, electron and carbon. keeps DNA tightly wrapped around spool-like proteins,
Most of the fungi grow on dead organic matter and are called histones. Without such packaging, DNA molecules
known as saprophytes. Few fungi are parasitic. Fungi would be too long to fit inside cells. For example, if all of
are usually aerobic though some are anaerobic e.g., as in the DNA molecules in a single human cell were unwound
rumen of cattle. Yeasts are mostly facultative anaerobes from their histones and placed end-to-end, they would
(i.e., they can survive either in the presence or absence stretch 6 feet.
of oxygen). Option b is correct: For an organism to grow and function
properly, cells must constantly divide to produce new cells
259. Solution: (a) to replace old, worn-out cells. During cell division, it is
Exp) Option a is the correct answer. essential that DNA remains intact and evenly distributed
A gene is the basic physical and functional unit of heredity. among cells. Chromosomes are a key part of the process
The purpose of genes is to store information. that ensures DNA is accurately copied and distributed in
the vast majority of cell divisions. Still, mistakes do occur
Option 1 is correct: The number of genes in an on rare occasions. Changes in the number or structure of
organism’s genome (the entire set of chromosomes) varies chromosomes in new cells may lead to serious problems.
significantly between species. For example, the human For example, in humans, one type of leukemia and some
genome contains an estimated 20,000 to 25,000 genes, other cancers are caused by defective chromosomes made
whereas the genome of the bacteria contains around 5,500 up of joined pieces of broken chromosomes.
genes.
Option c is incorrect: Chromosomes vary both in
Option 2 is incorrect: Genes are made up of DNA. number and shape among living things. Most bacteria
Some genes act as instructions to make molecules called have one or two circular chromosomes. Humans, along
proteins. However, many genes do not code for proteins with other animals and plants, have linear chromosomes
i.e., there are genes that don’t even make proteins. that are arranged in pairs within the nucleus of the cell.
Option 3 is incorrect: Genes are generally composed Option d is correct: The only human cells that do not
of deoxyribonucleic acid (DNA). However, in some contain pairs of chromosomes are reproductive cells, or
viruses, genes consist of a closely related compound gametes, which carry just one copy of each chromosome.
called ribonucleic acid (RNA) instead of DNA. DNA When two reproductive cells unite, they become a single
is responsible for storing and transferring genetic cell that contains two copies of each chromosome. This
information, while RNA directly codes for amino acids cell then divides and its successors divide numerous
and acts as a messenger between DNA and ribosomes to times, eventually producing a mature individual with a
make proteins. full set of paired chromosomes in virtually all of its cells.

91 Workbook
.
SCIENCE & TECHNOLOGY

SCIENCE & TECHNOLOGY


APPLIED SCIENCE
(a) Water from soil cannot rise to aerial parts.
2.1. Previous Years Questions (b) Roots are starved of energy.
1. What is the difference between Bluetooth and Wi- (c) Tree is infected by soil microbes.
Fi devices? [UPSC CSE Pre. 2011] (d) Roots do not receive oxygen for respiration.
(a) Bluetooth uses 2.4 GHz radio frequency band, 5. At present, scientists can determine the
whereas Wi-Fi can use 2.4 GHz or 5 GHz arrangement or relative positions of genes or
frequency band DNA sequences on a chromosome. How does this
(b) Bluetooth is used for Wireless Local Area knowledge benefit us? [UPSC CSE Pre. 2011]
Networks (WLAN) only, whereas Wi-Fi is used 1. It is possible to know the pedigree of livestock.
for Wireless Wide Area Networks (WWAN) only 2. It is possible to understand the causes of all
(c) When information is transmitted between two human diseases.
devices using Bluetooth technology, the devices 3. It is possible to develop disease-resistant animal
have to be in the line of sight of each other, but breeds.
when Wi-Fi technology is used the devices need Which of the statements given above is/are correct?
not be in the line of sight of each other
(d) The statements (a) and (b) given above are (a) 1 and 2 only
(b) 2 only
correct in this context
(c) 1 and 3 only
2. What is “Virtual Private Network”? (d) 1, 2 and 3
[UPSC CSE Pre. 2011] 6. A genetically engineered form of brinjal, known as the
(a) It is a private compute network of an organization Bt-brinjal, has been developed. The objective of this is
where he remote users can transmit encrypted [UPSC CSE Pre. 2011]
information through the server of the
(a) To make it pest-resistant
organization. (b) To improve its taste and nutritive qualities
(b) It is a computer network across a public (c) To make it drought-resistant
internet that provides users access to their (d) To make its shelf-life longer
organization’s network while maintaining the
security of the information transmitted. 7. A company marketing food product advertises
(c) It is a computer network in which users can that its items do not contain trans-fats. What does
access a shared pool of computing resources this campaign signify to the customers?
through a service provider [UPSC CSE Pre. 2011]
(d) None of the statements (a), (b) and (c) given 1. The food products are not made out of
above is a correct description of Virtual Private hydrogenated oils.
Network. 2. The food products are not made out of animal
fats/oils.
3. A new optical disc format known as the Blu-ray
Disc (BD) is becoming popular. In What way is it 3. The oils used are not likely to damage the
different form the traditional DVD? cardiovascular health of the consumers.
[UPSC CSE Pre. 2011] Which of the statements given above is/are correct?
1. DVD supports standard Definition video while (a) 1 only
BD supports High definition Video. (b) 2 and 3 only
2. Compared to a DVD, the BD format has several (c) 1 and 3 only
times more storage capacity. (d) 1, 2 and 3
3. Thickness of BD is 2.4 mm while that of DVD 8. Regular intake of fresh fruits and vegetables is
is 1.2 mm. recommended in the diet since they are a good
Which of the statements given above is/are correct? source of antioxidants. How do antioxidants help
(a) 1 only a person maintain health and promote longevity?
(b) 1 and 2 only [UPSC CSE Pre. 2011]
(c) 2 and 3 only (a) They activate the enzymes necessary for vitamin
(d) 1, 2 and 3 synthesis in the body and help prevent vitamin
deficiency
4. When the bark of a tree is removed in a circular (b) They prevent excessive oxidation of
fashion all around near its base, it gradually dries carbohydrates, fats and proteins in the body and
up and dies because? [UPSC CSE Pre. 2011] help avoid unnecessary wastage of energy

Workbook 92
.
SCIENCE & TECHNOLOGY

(c) They neutralize the free radicals produced in the 13. The function of heavy water in a nuclear reactor is
body during metabolism to [UPSC CSE Pre. 2011]
(d) They activate certain genes in the cells of the (a) Slow down the speed of neutrons
body and help delay the ageing process (b) Increase the speed of neutrons
(c) Cool down the reactor
9. Aspartame is an artificial sweetener sold in the
(d) Stop the nuclear reaction
market. It consists of amino acids and provides
calories like other amino acids. Yet, it is used as a 14. Microbial fuel cells are considered a source of
low-calorie sweetening agent in food items. What sustainable energy. Why? [UPSC CSE Pre. 2011]
is the basis of this use? [UPSC CSE Pre. 2011] 1. They use living organisms as catalysts to generate
(a) Aspartame is as sweet as table sugar, but unlike electricity from certain substrates.
table sugar, it is not readily oxidized in human 2. They use a variety of inorganic materials as
body due to lack of requisite enzymes. substrates.
(b) When aspartame is used in food processing, the 3. They can be installed in waste water treatment
sweet taste remains, but it becomes resistant to plants to cleanse water and produce electricity.
oxidation.
Which of the following statements given above is/
(c) Aspartame is as sweet as sugar, but after ingestion
are correct?
into the body, it is converted into metabolites
that yield no calories. (a) 1 only
(d) Aspartame is several times sweeter than table (b) 2 and 3 only
sugar, hence food items made with small (c) 1 and 3 only
(d) 1, 2 and 3
quantities of aspartame yield fewer calories on
oxidation. 15. Graphene is frequently in news recently. What is
its importance? [UPSC CSE Pre. 2012]
10. Satellites used for telecommunication relay are
kept in a geostationary orbit. A satellite is said to 1. It is a two-dimensional material and has good
be in such a orbit when: [UPSC CSE Pre.2011] electrical conductivity.
1. The orbit is geosynchronous. 2. It is one of the thinnest but strongest materials
tested so far.
2. The orbit is circular.
3. It is entirely made of silicon and has high optical
3. The orbit lies in the plane of the earth’s equator. transparency.
4. The orbit is at an altitude of 22,236. 4. It can be used as ‘conducting electrodes’ required
Select the correct answer using the codes given for touch screens, LCDs and organic LEDs.
below: Which of the statements given above are correct?
(a) 1, 2 and 3 only (a) 1 and 2 only
(b) 1, 3 and 4 only (b) 3 and 4 only
(c) 2 and 4 only (c) 1, 2 and 4 only
(d) 1, 2, 3 and 4 (d) 1, 2, 3 and 4
11. An artificial satellite orbiting around the Earth 16. What is the difference between a CFL and an LED
does not fall down. This is so because the attraction lamp? [UPSC CSE Pre. 2012]
of Earth [UPSC CSE Pre. 2011] 1. To produce light, a CFL uses mercury vapour
(a) does not exist at such distance and phosphor while an LED lamp uses semi-
(b) is neutralized by the attraction of the moon conductor material.
(c) provides the necessary speed for its steady 2. The average life span of a CFL is much longer
motion than that of an LED lamp
(d) provides the necessary acceleration for its
3. A CFL is less energy-efficient as compared to an
motion
LED lamp.
12. A layer in the Earth’s atmosphere called Ionosphere Which of the statements given above is/are correct?
facilitates radio communication. Why? (a) 1 only
[UPSC CSE Pre. 2011] (b) 2 and 3 only
1. The presence of ozone causes the reflection of (c) 1 and 3 only
radio waves to Earth. (d) 1, 2 and 3
2. Radio waves have a very long wavelength.
17. To meet its rapidly growing, energy demand,
Which of the statements given above is/are correct? some opine that India should pursue research
(a) 1 only and development on thorium as the future fuel
(b) 2 only of nuclear energy. In this context, what advantage
(c) Both 1 and 2 does thorium hold over uranium?
(d) Neither 1 nor 2 [UPSC CSE Pre. 2012]

93 Workbook
.
SCIENCE & TECHNOLOGY

1. Thorium is far more abundant in nature than 1. To enable them to withstand drought
uranium. 2. To increase the nutritive value of the produce
2. On the basis of per unit mass of mined mineral, 3. To enable them to grow and do photosynthesis
thorium can generate more energy compared to in spaceships and space stations
natural uranium. 4. To increase their shelf life
3. Thorium produces less harmful waste compared Select the correct answer using the codes given
to uranium. below:
Which of the statements given above is/are correct? (a) 1 and 2 only
(a) 1 only (b) 3 and 4 only
(b) 2 and 3 only (c) 1, 2 and 4 only
(c) 1 and 3 (d) 1, 2, 3 and 4
(d) 1, 2 and 3
22. What are the reasons for the people’s resistance to
18. Which of the following is/are cited by the scientists the introduction of Bt brinjal in India?
as evidence/evidences for the continued expansion [UPSC CSE Pre. 2012]
of universe? [UPSC CSE Pre. 2012] 1. Bt brinjal has been created by inserting a gene
1. Detection of microwaves in space from a soil fungus into its genome.
2. Observation of redshift phenomenon in space 2. The seeds of Bt brinjal are terminator seeds and
3. Movement of asteroids in space therefore, the farmers have to buy the seeds
4. Occurrence of supernova explosions in space before every season from the seed companies.
Select the correct answer using the codes given 3. There is an apprehension that the consumption
below: of Bt brinjal may have adverse impact on health.
(a) 1 and 2 only 4. There is some concern that the introduction
(b) 2 only of Bt brinjal may have adverse effect on the
(c) 1 ,3 and 4 biodiversity.
(d) None of the above can be cited as evidence. Select the correct answer using the codes given
below:
19. What is the role of ultraviolet (UV)
radiation in the water purification systems? (a) 1, 2 and 3 only
[UPSC CSE Pre. 2012] (b) 2 and 3 only
1. It inactivates /kills the harmful microorganisms (c) 3 and 4 only
in water. (d) 1, 2, 3 and 4
2. It removes all the undesirable odors from the 23. Recombinant DNA technology (Genetic
water. Engineering) allows genes to be transferred
3. It quickens the sedimentation of solid particles, [UPSC CSE Pre. 2013]
removes turbidity and improves the clarity of 1. across different species of plants
water.
2. from animals to plants
Which of the statements given above is/are correct?
3. from microorganisms to higher organisms
(a) 1 only
(b) 2 and 3 only Select the correct answer using the codes given
(c) 1 and 3 only below.
(d) 1, 2 and 3 (a) 1 only
(b) 2 and 3 only
20. With reference to ‘stem cells’, frequently in the (c) 1 and 3 only
news, which of the following statements is/are (d) 1, 2 and 3
correct? [UPSC CSE Pre. 2012]
1. Stem cells can be derived from mammals only. 24. Mycorrhizal biotechnology has been used in
2. Stem cells can be used for screening new drugs. rehabilitating degraded sites because mycorrhiza
3. Stem cells can be used for medical therapies. enables the plants to [UPSC CSE Pre. 2013]
Select the correct answer using the codes given 1. Resist drought and increase absorptive area
below: 2. Tolerate extremes of pH
(a) 1 and 2 only 3. Resist disease infestation
(b) 2 and 3 only Select the correct answer using the codes given
(c) 3 only below.
(d) 1, 2 and 3 (a) 1 only
21. Other than resistance to pests, what are the (b) 2 and 3 only
prospects for which genetically engineered plants (c) 1 and 3 only
have been created? [UPSC CSE Pre. 2012] (d) 1, 2 and 3

Workbook 94
.
SCIENCE & TECHNOLOGY

25. Which of the following statements is/are correct? Vitamin Deficiency disease
[UPSC CSE Pre. 2013]
1. Viruses lack enzymes necessary for the 1. Vitamin C Scurvy
generation of energy. 2. Vitamin D Rickets
2. Viruses can be cultured in any synthetic medium. 3. Vitamin E Night blindness
3. Viruses are transmitted from one organism to Which of the pairs given above is/ are correctly
another by biological vectors only. matched?
Select the correct answer using the codes given (a) 1 and 2 only
below. (b) 3 only
(a) 1 only (c) 1, 2 and 3
(b) 2 and 3 only (d) None
(c) 1 and 3 only 30. Consider the following diseases:
(d) 1, 2 and 3 [UPSC CSE Pre. 2014]
26. Which of the following diseases can be transmitted 1. Diphtheria
from one person to another through tattooing? 2. Chickenpox
[UPSC CSE Pre. 2013] 3. Smallpox
1. Chikungunya Which of the above diseases has/have been
2. Hepatitis B eradicated in India?
3. HIV-AIDS (a) 1 and 2 only
(b) 3 only
Select the correct answer using the codes given
(c) 1, 2 and 3
below. (d) None
(a) 1 only
(b) 2 and 3 only 31. Among the following organisms, which one does
(c) 1 and 3 only not belong to the class of other three?
(d) 1, 2 and 3 [UPSC CSE Pre. 2014]
(a) Crab
27. Consider the following phenomena: (b) Mite
[UPSC CSE Pre. 2013] (c) Scorpion
1. Size of the sun at dusk. (d) Spider
2. Color of the sun at dawn 32. In addition to fingerprint scanning, which
3. Moon being visible at dawn of the following can be used in the biometric
4. Twinkle of stars in the sky identification of a person? [UPS CSE Pre. 2014]
5. Polestar being visible in the sky 1. Iris scanning
Which of the above are optical illusions? 2. Retinal scanning
3. Voice recognition
(a) 1, 2 and 3
(b) 3, 4 and 5 Select the correct answer using the code given below.
(c) 1, 2 and 4 (a) 1 only
(d) 2, 3 and 5 (b) 2 and 3 only
(c) 1 and 3 only
28. With reference to Agni-IV Missile, which of the (d) 1, 2 and 3
following statements is/are correct?
33. Which of the following pairs is/are correctly
[UPSC CSE Pre. 2014]
matched? [UPSC CSE Pre. 2014]
1. It is surface-to surface missile.
2. It is fuelled by liquid propellant only. Spacecraft Purpose
3. It can deliver one-tonne nuclear warheads about 1. Cassini-Huygens Orbiting the Venus and
7500km away. transmitting data to the
Select the correct answer using the code given below. Earth
(a) 1 only 2. Messenger Mapping and
(b) 2 and 3 only investigating
(c) 1 and 3 only 3. Voyager 1 and 2 Exploring the outer
(d) 1, 2 and 3 solar system
29. Consider the following pairs: Select the correct answer using the code given below.
[UPSC CSE Pre. 2014] (a) 1 only

95 Workbook
.
SCIENCE & TECHNOLOGY

(b) 2 and 3 only Select the correct answer using the code given below.
(c) 1 and 3 only (a) 1 only
(d) 1, 2 and 3 (b) 2 only
(c) 1 and 2 Both
34. With reference to technology for solar power
(d) Neither 1 nor 2
production, consider the following statements :
[UPSC CSE Pre. 2014] 38. With reference to ‘Near Field Communication
1. ‘Photovoltaics’ is a technology that generates (NFC) Technology’, which of the following
electricity by direct conversion of light into statements is/are correct? [UPSC CSE 2015]
electricity, while ‘Solar Thermal’ is a technology 1. It is a contactless communication technology
that utilizes the Sun’s rays to generate heat which that uses electromagnetic radio fields.
is further used in electricity generation process. 2. NFC is designed for use by devices which can
2. Photovoltaics-generates Alternating Current be at a distance of even a metre from each other
(AC), while Solar Thermal generates Direct 3. NFC can use encryption when sending sensitive
Current (DC). information.
3. India has manufacturing base for Solar Thermal Select the correct answer using the code given below.
technology, but not for photovoltaics. (a) 1 and 2 only
Which of the statements given above is/are correct? (b) 3 only
(a) 1 only (c) 1 and 3 only
(b) 2 and 3 only (d) 1, 2 and 3
(c) 1, 2 and 3
39. With reference to bio-toilets used by the Indian
(d) None
Railways, consider the following statements:
35. Consider the following statements: [UPSC CSE Pre. 2015]
[UPSC CSE Pre. 2014] 1. The decomposition of human waste in the bio-
1. Maize can be used for the production of starch. toilets is initiated by a fungal inoculum.
2. Oil extracted from maize can be a feedstock for 2. Ammonia and water vapour are the only end
biodiesel. products in this decomposition which are
3. Alcoholic beverages can be produced by using released into the atmosphere.
maize. Which of the statements given above is/are correct?
Which of the statements given above is/are correct? (a) 1 only
(a) 1 only (b) 2 only
(b) 1 and 2 only (c) Both 1 and 2
(c) 2 and 3 only (d) Neither 1 nor 2
(d) 1, 2 and 3 40. With reference to ‘fly ash’ produced by the power
36. With reference to ‘fuel cells’ in which hydrogen- plants using coal as fuel, which of the following
rich fuel and oxygen are used to generate statements is/are correct? [UPSC CSE Pre. 2015]
electricity. Consider the following statements: 1. Fly ash can be used in the production of bricks
[UPSC CSE Pre. 2015] for building construction.
1. If pure hydrogen is used as a fuel, the fuel cell 2. Fly ash can be used as a replacement for some of
emits heat and water as by-products. the Portland cement concrete.
2. Fuel cells can be used for powering buildings 3. Fly ash is made up of silicon dioxide and calcium
and not for small devices like laptop computers. oxide only, and does not contain any toxic
elements.
3. Fuel cells produce electricity in the form of
Alternating Current (AC). Select the correct answer using the code given below.
Which of the statements given above is / are correct? (a) 1 and 2 only
(b) 2 only
(a) 1 only (c) 1 and 3
(b) 2 and 3 only (d) 3 only
(c) 1 and 3 only
(d) 1, 2 and 3 41. In which of the following activities are Indian
Remote Sensing (IRS) satellites used?
37. With reference to the use of nano-technology in
[UPSC CSE Pre. 2015]
health sector, which of the following statements is/
are correct? [UPSC CSE Pre. 2015] 1. Assessment of crop productivity
1. Targeted drug delivery is made possible by 2. Locating groundwater resources
nanotechnology. 3. Mineral exploration
2. Nanotechnology can largely contribute to gene 4. Telecommunications
therapy. 5. Traffic studies

Workbook 96
.
SCIENCE & TECHNOLOGY

Select the correct answer using the code given below. 3. made India the only country to be successful in
(a) 1, 2 and 3 only making its spacecraft orbit the Mars in its very
(b) 4 and 5 only first attempt
(c) 1 and 2 only Which of the statements given above is/are correct?
(d) 1, 2, 3, 4 and 5 (a) 1 only
42. The term ‘IndARC’, sometimes seen in the news, is (b) 2 and 3 only
the name of [UPSC CSE Pre. 2015] (c) 1 and 3 only
(a) an indigenously developed radar system (d) 1, 2 and 3
inducted into Indian Defence 47. Which of the following statements is/are correct?
(b) India’s satellite to provide services to the Viruses can infect[UPSC CSE Pre. 2016]
countries of Indian Ocean Rim 1. bacteria
(c) a scientific establishment set up by India in
Antarctic region 2. fungi
(d) India’s underwater observatory to scientifically 3. plants
study the Arctic region Select the correct answer using the code given below.
43. In the context of modern scientific research, (a) 1 and 2 only
consider the following statements about ‘IceCube’, (b) 3 only
a particle detector located at South Pole, which (c) 1 and 3 only
was recently in the news : [UPSC CSE Pre. 2015] (d) 1, 2 and 3
1. It is the world’s largest neutrino detector, 48. In the context of the developments in
encompassing a cubic kilometre of ice. Bioinformatics, the term ‘transcriptome’,
2. It is a powerful telescope to search for dark sometimes seen in the news, refers to
matter [UPSC CSE Pre. 2016]
3. It is buried deep in the ice. (a) a range of enzymes used in genome editing
Which of the statements given above is/are correct? (b) the full range of mRNA molecules expressed by
(a) 1 only an organism
(b) 2 and 3 only (c) the description of the mechanism of gene
(c) 1 and 3 only expression
(d) 1, 2 and 3 only (d) a mechanism of genetic mutations taking place
in cells
44. What is ‘Greased Lightning-10 (GL-10)’, recently
in the news? [UPSC CSE Pre. 2016] 49. With reference to ‘Li-Fi’, recently in the news,
(a) Electric plane tested by NASA which of the following statements is/are correct?
(b) Solar-powered two-seater aircraft designed by [UPSC CSE Pre. 2016]
Japan 1. It uses light as the medium for high-speed data
(c) Space observatory launched by China transmission.
(d) Reusable rocket designed by ISRO 2. It is a wireless technology and is several times
faster than ‘Wi-Fi’.
45. With reference to `Astrosat’, the astronomical
observatory launched by India, which of the Select the correct answer using the code given below.
following statements is/are correct? (a) 1 only
[UPSC CSE Pre. 2016] (b) 2 only
1. Other than USA and Russia, India is the only (c) Both 1 and 2
country to have launched a similar observatory (d) Neither 1 nor 2
into space. 50. India is an important member of the ‘International
2. Astrosat is a 2000 kg satellite placed in an orbit at Thermonuclear Experimental Reactor’. If this
1650 km above the surface of the Earth. experiment succeeds, what is the immediate
Select the correct answer using the code given below. advantage for India? [UPSC CSE Pre. 2016]
(a) 1 only (a) It can use thorium in place of uranium for power
(b) 2 only generation
(c) Both 1 and 2 (b) It can attain a global role in satellite navigation
(d) Neither 1 nor 2 (c) It can drastically improve the efficiency of its
fission reactors in power generation
46. Consider the following statements:
(d) It can build fusion reactors for power generation
[UPSC CSE Pre. 2016]
1. The Mangalyaan launched by ISRO is also called 51. With reference to agriculture in India, how can
the Mars Orbiter Mission the technique of ‘genome sequencing’, often seen
2. made India the second country to have a in the news, be used in the immediate future?
spacecraft orbit the Mars after USA [UPSC CSE Pre. 2017]

97 Workbook
.
SCIENCE & TECHNOLOGY

1. Genome sequencing can be used to identify 2. Sexual transmission of Zika virus disease is
genetic markers for disease resistance and possible.
drought tolerance in various crop plants. Which of the statements given above is/are correct?
2. This technique helps in reducing the time (a) 1 only
required to develop new varieties of crop plants. (b) 2 only
3. It can be used to decipher the host-pathogen (c) Both 1 and 2
relationships in crops. (d) Neither 1 nor 2
Select the correct answer using the code given below: 56. What is the purpose of ‘evolved Laser
(a) 1 only Interferometer Space Antenna (eLISA) project?
(b) 2 and 3 only [UPSC CSE Pre. 2017]
(c) 1 and 3 only
(a) To detect neutrinos
(d) 1, 2 and 3
(b) To detect gravitational waves
52. What is the application of Somatic Cell Nuclear (c) To detect the effectiveness of missile defense
Transfer (SCNT) Technology system
[UPSC CSE Pre. 2017] (d) To study the effect of solar flares on our
(a) Production of biolarvicides communication systems
(b) Manufacture of biodegradable plastics 57. The terms ‘Event Horizon’, ‘Singularity’, `String
(c) Reproductive cloning of animals Theory’ and ‘Standard Model’ are sometimes seen
(d) Production of organisms free of diseases in the news in the context of [UPSC CSE Pre. 2017]
53. Organic Light Emitting Diodes (OLEDs) are used (a) Observation and understanding of the Universe
to create digital display in many devices. What (b) Study of the solar and the lunar eclipses
are the advantages of OLED displays over Liquid (c) Placing satellites in the orbit of the Earth
Crystal displays? [UPSC CSE Pre. 2017] (d) Origin and evolution of living organisms on the
Earth
1. OLED displays can be fabricated on flexible
plastic substrates. 58. With reference to India’s satellite launch vehicles,
2. Roll-up displays embedded in clothing can be consider the following statements:
made using OLEDs. [UPSC CSE Pre. 2018]
3. Transparent displays are possible using OLEDs. 1. PSLVs launch the satellites useful for Earth
Select the correct answer using the code given below resources monitoring whereas GSLVs are
(a) 1 and 3 only designed mainly to launch communication
(b) 2 only satellites.
(c) 1, 2 and 3 2. Satellites launched by PSLV appear to remain
(d) None of the above statements is correct permanently fixed in the same position in the
sky, as viewed from a particular location on
54. Consider the following pairs: Earth.
[UPSC CSE Pre. 2017] 3. GSLV Mk III is a four-staged launch vehicle
Commonly Unwanted or controversial with the first and third stages using solid rocket
used/consumed chemicals likely to motors; and the second and fourth stages using
materials producefound in them liquid rocket engines.
Which of the statements given above is/are correct?
1. Lipstick Lead
(a) 1 only
2. Soft Drinks Brominated vegetable oils (b) 2 and 3 only
3. Chinese fast Monosodium glutamate (c) 1 and 2 only
food (d) 3 only
Which of the pairs given above is/are correctly 59. Consider the following statements:
matched? [UPSC CSE Pre. 2018]
(a) 1 only 1. The Earth’s magnetic field has reversed every few
(b) 2 and 3 only hundred thousand years.
(c) 1 and 3 only 2. When the Earth was created more than 4000
(d) 1, 2 and 3 million years ago, there was 54% oxygen and no
55. Consider the following statements: carbon dioxide.
[UPSC CSE Pre. 2017] 3. When living organisms originated, they
1. In tropical regions, Zika virus disease is modified the early atmosphere of the Earth.
transmitted by the same mosquito that transmits Which of the statements given above is/are correct?
dengue. (a) 1 only

Workbook 98
.
SCIENCE & TECHNOLOGY

(b) 2 and 3 only (c) 2 and 3 only


(c) 1 and 3 only (d) 1, 2 and 3
(d) 1, 2 and 3
64. “3D printing” has applications in which of the
60. With reference to the Genetically Modified following? [UPSC CSE Pre. 2018]
mustard (GM mustard) developed in India, 1. Preparation of confectionery items
consider the following statements: 2. Manufacture of bionic ears
[UPSC CSE Pre. 2018] 3. Automotive industry
1. GM mustard has the genes of a soil bacterium 4. Reconstructive surgeries
that give the plant the property of pest-resistance
to a wide variety of pests. 5. Data processing technologies
2. GM mustard has the genes that allow the plant Select the correct answer using the code given below.
cross-pollination and hybridization. (a) 1, 3 and 4 only
3. GM mustard has been developed jointly by the (b) 2, 3 and 5 only
IARI and Punjab Agricultural University. (c) 1 and 4 only
(d) 1, 2, 3, 4 and 5
Which of the statements given above is/are correct?
(a) 1 and 3 only 65. When the alarm of your smartphone rings in
(b) 2 only the morning, you wake up and tap it to stop the
(c) 2 and 3 only alarm which causes your geyser to be switched on
(d) 1, 2 and 3 automatically. The smart mirror in your bathroom
shows the day’s weather and also indicates the level
61. What is “Terminal High Altitude Area Defense
of water in your overhead tank. After you take
(THAAD)”, sometimes seen in the news?
some groceries from your refrigerator for making
[UPSC CSE Pre. 2018]
breakfast, it recognizes the shortage of stock in it
(a) An Israeli radar system and places an order for the supply of fresh grocery
(b) India’s indigenous anti-missile program items. When you step out of your house and lock
(c) An American anti-missile system the door, all lights, fans, geysers and AC machines
(d) A defense collaboration between Japan and
get switched off automatically. On your way to
South Korea.
office, your car warns you about traffic congestion
62. With reference to the Indian Regional Navigation ahead and suggests an alternative route, and if you
Satellite System (IRNSS), consider the following are late for a meeting, it sends a message to your
statements: [UPSC CSE Pre. 2018] office accordingly. [UPSC CSE Pre. 2018]
1. IRNSS has three satellites in geostationary and In the context of emerging communication
four satellites in geosynchronous orbits. technologies, which one of the following terms best
2. IRNSS covers entire India and about 5500 sq. km applies to the above scenario?
beyond its borders. (a) Border Gateway Protocol
3. India will have its own satellite navigation system (b) Internet of Things
with full global coverage by the middle of 2019. (c) Internet Protocol
Which of the statements given above is/are correct? (d) Virtual Private Network
(a) 1 only 66. In which of the following areas can GPS technology
(b) 1 and 2 only be used? [UPSC CSE Pre. 2018]
(c) 2 and 3 only 1. Mobile phone operations
(d) None
2. Banking operations
63. Consider the following pairs: 3. Controlling the power grids
[UPSC CSE Pre. 2018]
Select the correct answer using the code given below:
Term Sometimes in Context of Seen in (a) 1 only
the news (b) 2 and 3 only
(c) 1 and 3 only
1. Belle II experiment Artificial Intelligence
(d) 1, 2 and 3
2. Blockchain Digital/
technology Cryptocurrency 67. The terms ‘WannaCry, Petya and Eternal Blue’
sometimes mentioned in the news recently are
3. CRISPR – Cas9 Particle Physics related to [UPSC CSE Pre. 2018]
Which of the pairs given above is/are correctly (a) Exoplanets
matched? (b) Cryptocurrency
(a) 1 and 3 only (c) Cyber attacks
(b) 2 only (d) Mini satellites

99 Workbook
.
SCIENCE & TECHNOLOGY

68. The identity platform ‘Aadhaar’ provides open 72. With reference to communication technologies,
“Application Programming Interfaces (APIs)”. what is/are the difference/ differences between
What does it imply? [UPSC CSE Pre. 2018] LTE (Long-Term Evolution) and VoLTE (Voice
1. It can be integrated into any electronic device. over Long-Term Evolution)?
2. Online authentication using iris is possible. [UPSC CSE Pre. 2019]
Which of the statements given above is/are correct? 1. LTE is commonly marketed as 3G and VoLTE is
(a) 1 only commonly marketed as advanced 3G.
(b) 2 only 2. LTE is data-only technology and VoLTE is voice-
(c) Both 1 and 2 only technology.
(d) Neither 1 nor 2 Select the correct answer using the code given below.
69. In the context of digital technologies for (a) 1 only
entertainment, consider the following statements: (b) 2 only
[UPSC CSE Pre. 2019] (c) Both 1 and 2
1. In Augmented Reality (AR), a simulated (d) Neither 1 nor 2
environment is created and the physical world is
73. The word ‘Denisovan’ is sometimes mentioned in
completely shut out.
media in reference to [UPSC CSE Pre. 2019]
2. In Virtual Reality (VR), images generated from a
computer are projected onto real-life objects or (a) fossils of a kind of dinosaurs
surroundings. (b) an early human species
3. AR allows individuals to be present in the world (c) a cave system found in North-East India
and improves the experience using the camera of (d) a geological period in the history of Indian
smart-phone or PC. subcontinent
4. VR closes the world, and transposes an 74. With reference to the recent developments in
individual, providing complete immersion science, which one of the following statements is
experience. not correct? [UPSC CSE Pre. 2019]
Which of the statements given above is/are correct? (a) Functional chromosomes can be created by
(a) 1 and 2 only joining segments of DNA taken from cells of
(b) 3 and 4 different species.
(c) 1, 2 and 3 (b) Pieces of artificial functional DNA can be created
(d) 4 only in laboratories.
70. Consider the following statements about a digital (c) A piece of DNA taken out from an animal cell
signature: [UPSC CSE Pre. 2019] can be made to replicate outside a living cell in
1. An electronic record that identifies the certifying a laboratory.
authority issuing it (d) Cells taken out from plasma and animals can be
2. Used to serve as a proof of identity of an made to undergo cell division in laboratory petri
individual to access information or server on dishes.
Internet 75. What is Cas9 protein that is often mentioned in
3. An electronic method of signing an electronic news? [UPSC CSE Pre. 2019]
document and ensuring that the original content
(a) A molecular scissors used in targeted gene
is unchanged
editing
Which of the statements given above is/are correct? (b) A biosensor used in the accurate detection of
(a) 1 only pathogens in patients
(b) 2 and 3 only (c) A gene that makes plants pest-resistant
(c) 3 only (d) A herbicidal substance synthesized in genetically
(d) 1, 2 and 3 modified crops
71. In the context of wearable technology, which of the
76. Consider the following techniques/phenomena:
following tasks is/are accomplished by wearable
devices? [UPSC CSE Pre. 2019] 1. Budding and grafting in fruit plants
1. Location identification of a person 2. Cytoplasmic male sterility
2. Sleep monitoring of a person 3. Gene silencing
3. Assisting the hearing-impaired person Which of the above is/are used to create transgenic
Select the correct answer using the code given below: crops?
(a) 1 only (a) 1 only
(b) 2 and 3 only (b) 2 and 3 only
(c) 3 only (c) 1 and 3 only
(d) 1, 2 and 3 (d) None

Workbook 100
.
SCIENCE & TECHNOLOGY

77. ‘RNA interference (RNAi)’ technology has gained (b) New horizons
popularity in the last few years. Why? (c) Lisa Pathfinder
[UPSC CSE Pre. 2019] (d) Evolved LISA
1. It is used in developing gene silencing therapies. 82. With reference to solar water pumps, consider the
2. It can be used in developing therapies for the following statements: [UPSC CSE Pre. 2020]
treatment of cancer. 1. Solar power can be used for running surface
3. It can be used to develop hormone replacement pumps and not for submersible pumps.
therapies. 2. Solar power can be used for running centrifugal
4. It can be used to produce crop plants that are pumps and not the ones with piston.
resistant to viral pathogens. Which of the statements given above is/are correct?
Select the correct answer using the code given below. (a) 1 only
(a) 1, 2 and 4 (b) 2 only
(b) 2 and 3 (c) Both 1 and 2
(c) 1 and 3 (d) Neither 1 nor 2
(d) 1 and 4 only
83. Consider the following activities:
78. Which of the following are the reasons for the [UPSC CSE Pre. 2020]
occurrence of multi-drug resistance in microbial 1. Spraying pesticides on a crop field
pathogens in India? [UPSC CSE Pre. 2019] 2. Inspecting the craters of active volcanoes
1. Genetic predisposition of some people 3. Collecting breath samples from spouting whales
2. Taking incorrect doses of antibiotics to cure for DNA analysis
diseases At the present level of technology, which of the
3. Using antibiotics in livestock farming above activities can be successfully carried out by
4. Multiple chronic diseases in some people using drones?
Select the correct answer using the code given below. (a) 1 and 2 only
(a) 1 and 2 only (b) 2 and 3 only
(c) 1 and 3 only
(b) 2 and 3 only
(d) 1, 2 and 3
(c) 1, 3 and 4 only
(d) 2, 3 and 4 only 84. Goldsmiths blow the outermost zone of a flame
with a metallic blow-pipe for melting gold and
79. Which one of the following statements is not silver. For which one of the following reasons do
correct? [UPSC CSE Pre. 2019] they use the outermost zone of the flame?
(a) Hepatitis B virus is transmitted much like HIV. [UPSC CSE Pre. 2020]
(b) Hepatitis B unlike Hepatitis C, does not have a (a) It is the hottest part providing most heat.
vaccine. (b) This luminous zone doesn’t produce any ash.
(c) Globally, the number of people infected with (c) It is least-hottest part protecting the design of
Hepatitis B and C viruses arc several times more jewelry.
than those infected with HIV. (d) Middle zone vaporizes gold leading to loss of
(d) Some of those infected with Hepatitis B and C metal.
viruses do not show the symptoms for many
years. 85. With reference to carbon nanotubes, consider the
following statements: [UPSC CSE Pre. 2020]
80. Recently, Scientists observed the merger of giant 1. They can be used as carriers of drugs and
‘blackholes’ billions of light-years away from the antigens in the human body.
Earth. What is the significance of this observation?
2. They can be made into artificial blood capillaries
[UPSC CSE Pre. 2019] for an injured part of the human body.
(a) ‘Higgs boson particles’ were detected. 3. They can be used in biochemical sensors.
(b) ‘Gravitational waves’ were detected.
(c) Possibility of inter-galactic space travel through 4. Carbon nanotubes are biodegradable.
‘wormhole’ was confirmed. Which of the statements given above are correct?
(d) It enabled the scientists to understand (a) 1 and 2 only
‘singularity’. (b) 2, 3 and 4 only
(c) 1, 3 and 4 only
81. “The experiment will employ a trio of spacecraft
(d) 1, 2, 3 and 4
flying in formation in the shape of an equilateral
triangle that has sides one million kilometers 86. Consider the following statements:
long, with lasers shining between the craft.” The [UPSC CSE Pre. 2020]
experiment in question refers to 1. Genetic changes can be introduced in the cells
[UPSC CSE Pre. 2020] that produce eggs or sperms of a prospective
(a) Voyager-2 parent.

101 Workbook
.
SCIENCE & TECHNOLOGY

2. A person’s genome can be edited before birth at 91. With reference to Visible Light Communication
the early embryonic stage. (VLC) technology, which of the following
3. Human induced pluripotent stem cells can be statements are correct? [UPSC CSE 2020]
injected into the embryo of a pig. 1. VLC uses electromagnetic spectrum wavelengths
375 to 780 nm.
Which of the statements given above is/are correct?
2. VLC is known as long-range optical wireless
(a) 1 only communication.
(b) 2 and 3 only 3. VLC can transmit large amounts of data faster
(c) 2 only than Bluetooth.
(d) 1, 2 and 3 4. VLC has no electromagnetic interference.
87. What is the importance of using Pneumococcal Select the correct answer using the code given below:
Conjugate Vaccines in India? (a) 1, 2 and 3 only
[UPSC CSE Pre. 2020] (b) 1, 2 and 4 only
(c) 1, 3 and 4 only
1. These vaccines are effective against pneumonia (d) 2, 3 and 4 only
as well as meningitis and sepsis.
2. Dependence on antibiotics that are not effective 92. With reference to “Blockchain Technology”,
against drug-resistant bacteria can be reduced. consider the following statements:
[UPSC CSE Pre. 2020]
3. These vaccines have no side effects and cause no
1. It is a public ledger that everyone can inspect,
allergic reactions. but which no single user controls.
Select the correct answer using the code given below. 2. The structure and design of blockchain is such
(a) 1 only that all the data in it are about cryptocurrency
(b) 1 and 2 only only.
(c) 3 only 3. Applications that depend on basic features of
(d) 1, 2 and 3 blockchain can be developed without anybody’s
permission.
88. In the context of recent advances of human Which of the statements given above is/are correct?
reproductive technology, “Pronuclear Transfer” is (a) 1 only
used for [UPSC CSE Pre. 2020] (b) 1 and 2 only
(a) fertilization of egg in vitro by the donor sperm (c) 2 only
(b) genetic modification of sperm producing cells (d) 1 and 3 only
(c) development of stem cells into functional 93. Consider the following: [UPSC CSE Pre. 2022]
embryos 1. Aarogya Setu
(d) prevention of mitochondrial diseases in offspring 2. COWIN
89. In India, the term “Public Key Infrastructure” is 3. DigiLocker
used in the context of [UPSC CSE Pre. 2020] 4. DIKSHA
(a) Digital security infrastructure Which of the above are built on to open-source
(b) Food security infrastructure digital platforms?
(c) Health care and education infrastructure (a) 1 and 2 only
(d) Telecommunication and transportation (b) 2, 3 and 4 only
infrastructure (c) 1, 3 and 4 only
90. With the present state of development, Artificial (d) 1, 2, 3 and 4
Intelligence can effectively do which of the 94. With reference to Web 0, consider the following
following? [UPSC CSE Pre. 2020] statements: [UPSC CSE Pre. 2022]
1. Bring down electricity consumption in industrial 1. Web 3-0 technology enables people to control
units their own data.
2. Create meaningful short stories and songs 2. In Web 0 world, there can be blockchain based
3. Disease diagnosis social networks.
4. Text-to-Speech Conversion 3. Web 3-0 is operated by users collectively rather
5. Wireless transmission of electrical energy than a corporation.
Select the correct answer using the code given below: Which of the statements given above are correct?
(a) 1, 2, 3 and 5 only (a) 1 and 2 only
(b) 1, 3 and 4 only (b) 2 and 3 only
(c) 2, 4 and 5 only (c) 1 and 3 only
(d) 1, 2, 3, 4 and 5 (d) 1, 2 and 3

Workbook 102
.
SCIENCE & TECHNOLOGY

95. With reference to “Software as a Service (SaaS)”, (c) 1 and 3 only


consider the following statements: (d) 1, 2 and 3
[UPSC CSE Pre. 2022]
100. In the context of vaccines manufactured to prevent
1. SaaS buyers can customise the user interface and COVID-19 pandemic, consider the following
can change data fields. statements: [UPSC CSE Pre. 2022]
2. SaaS users can access their data through their 1. The Serum Institute of India produced
mobile devices. COVID-19 vaccine named Covishield using
3. Outlook, Hotmail and Yahoo! Mail are forms of mRNA platform.
SaaS. 2. Sputnik. V vaccine is manufactured using
Which of the statements given above are correct? vector-based platform.
(a) 1 and 2 only 3. COVAXIN is an inactivated pathogen-based
(b) 2 and 3 only vaccine.
(c) 1 and 3 only
Which of the statements given above are correct?
(d) 1, 2 and 3
(a) 1 and 2 only
96. Which one of the following statements best (b) 2 and 3 only
reflects the idea behind the “Fractional Orbital (c) 1 and 3 only
Bombardment System” often talked about in (d) 1, 2 and 3
media? [UPSC CSE Pre. 2022]
(a) A hypersonic missile is launched into space to 101. If a major solar storm (solar flare) reaches the
counter the asteroid approaching the Earth and Earth, which of the following are the possible
explode it in space. effects on the Earth? [UPSC CSE Pre. 2022]
(b) A spacecraft lands on another planet after 1. GPS and navigation systems could fail.
making several orbital motions. 2. Tsunamis could occur at equatorial regions.
(c) A missile is put into a stable orbit around the 3. Power grids could be damaged.
Earth and deorbits over a target on the Earth. 4. Intense auroras could occur over much of the
(d) A spacecraft moves along a comet with the same Earth.
speed and places a probe on its surface. 5. Forest fires could take place over much of the
97. Which one of the following is the context in which planet.
the term “qubit” is mentioned? 6. Orbits of the satellites could be disturbed.
[UPSC CSE Pre. 2022] 7. Shortwave radio communication of the aircraft
(a) Cloud Services flying over polar regions could be interrupted.
(b) Quantum Computing Select the correct answer using the code given below;
(c) Visible Light Communication Technologies (a) 1, 2, 4 and 5 only
(d) Wireless Communication Technologies (b) 2, 3, 5, 6 and 7 only
98. Consider the following technologies: (c) 1, 3, 4, 6 and 7 only
[UPSC CSE Pre. 2022] (d) 1, 2, 3, 4, 5, 6 and
1. Closed-circuit Television 102. With reference to Non-Fungible Tokens (NFTs),
2. Radio Frequency Identification consider the following statements:
3. Wireless Local Area Network [UPSC CSE Pre. 2022]
Which of the above are considered communication 1. They enable the digital representation of physical
Short-Range devices/technologies? assets.
(a) 1 and 2 only 2. They are unique cryptographic tokens that exist
(b) 2 and 3 only on a blockchain.
(c) 1 and 3 only 3. They can be traded or exchanged at equivalency
(d) 1, 2 and 3 and therefore can be used as a medium of
99. Consider the following statements: commercial transactions.
[UPSC CSE Pre. 2022] Which of the statements given above are correct?
1. Biofilms can form on medical implants within (a) 1 and 2 only
human tissues. (b) 2 and 3 only
2. Biofilms can form on food and food processing (c) 1 and 3 only
surfaces. (d) 1, 2 and 3
3. Biofilms can exhibit antibiotic resistance. 103. Consider the following statements:
Which of the statements given above are correct? [UPSC CSE Pre. 2022]
(a) 1 and 2 only 1. Other than those made by humans, nanoparticles
(b) 2 and 3 only do not exist in nature.

103 Workbook
.
SCIENCE & TECHNOLOGY

2. Nanoparticles of some metallic oxides are used 1. Used in analyzing seed purity
in the manufacture of some cosmetics. 2. Use in drug delivery system
3. Nanoparticles of some commercial products 3. Improve yield and profitability of crops
which enter the environment are unsafe for 4. Creation of transgenic animals
humans.
Select the correct answer using the code given below:
Which of the statements given above is/are correct? (a) 1 and 3 only
(a) 1 only (b) 1, 2 and 4 only
(b) 3 only (c) 2, 3 and 4 only
(c) 1 and 2 (d) 1, 2, 3 and 4
(d) 2 and 3
108. It is a scientific method to detect concealed
104. Consider the following statements: information stored in the brain by measuring
[UPSC CSE Pre. 2022] electroencephalographic (EEG) brain responses,
DNA Barcoding can be a tool to: or brainwaves, non-invasively by sensors placed
1. Assess the age of a plant or animal. on the scalp in response to words, phrases, or
2. Distinguish among species that look alike. pictures. It includes neither question answer
session nor blood pressure, pulse rate, respiration
3. Identify undesirable animal or plant materials in
and skin conductivity of accused. The method
processed foods.
mentioned here is?
Which of the statements given above is/are correct? (a) Narco-Analysis
(a) 1 only (b) Brain fingerprinting
(b) 3 only (c) Polygraph test
(c) 1and 2 (d) AVATAR
(d) 2 and 3
109. With reference to the “Genome India Project”,
Practice Question which of the following statements is incorrect?
(a) The initiative aims to identify genetic variations
2.2. Biotechnology for common diseases.
(b) It is an initiative of the Department of
105. Consider the following statements about Hand Biotechnology, Ministry of Science and
guided cloning technique:
technology.
1. In this technique nucleus from the egg is taken (c) It aims to provide information about the
by a hand-held fine knife. structure and organization of human genes in
2. In this technique the genetic material comes the Indian population
from single donor’s cell. (d) The information from the project will not be
3. This technique has been developed by National available in the public domain.
Dairy Research Institute (NDRI).
4. India was the first country to clone a buffalo 110. Indigen initiative sometimes mentioned in news,
using this technique. is:
(a) Genome sequencing of the population in the
Which of the statements given above are correct?
country.
(a) 2, 3 and 4 only (b) Captive breeding of endangered species.
(b) 1, 2 and 3 only (c) Creating high yielding varieties of indigenous
(c) 2 and 4 only seeds.
(d) 1, 2, 3 and 4 (d) Using traditional water harvesting techniques in
106. Recently, ‘novel Open reading frames’ has been dryland region.
in news. In this context, which of the following 111. Consider the following statement about Genetic
statements is correct about novel Open reading Engineering Appraisal Committee (GEAC):
frames? 1. It functions in the Ministry of Environment,
(a) It is an emerging technology which enhances the Forest and Climate Change (MoEF&CC).
visual abilities of a person. 2. It has been constituted under Environmental
(b) They are new genes emerging in the genome of Protection Act, 1986.
living organisms. 3. The Committee is chaired by the Minister of
(c) It is a virus with potential to create global Environment, Forests and Climate Change.
pandemic like Covid-19.
Which of the statements given above is/are correct?
(d) These are new virtual reality frames developed
by Google. (a) 1 and 3 only
(b) 1 and 2 only
107. With reference to application of DNA technology (c) 2 only
which of the following is/are correct? (d) 1, 2 and 3

Workbook 104
.
SCIENCE & TECHNOLOGY

112. Consider the following statements: Which of the following statements given above is/are
1. Biotechnology Industry Research Assistance correct?
Council (BIRAC) is a not-for-profit Public (a) 1 only
Sector Enterprise set up by Department of (b) 2 only
Biotechnology. (c) Both 1 and 2
2. E-Yuva scheme to promote a culture of applied (d) Neither 1 nor 2
research among young students is launched by 117. Consider the following statements about the
BIRAC. MANAV human atlas project:
Which of the statements given above is/are correct? 1. The project aims at creating database network of
(a) 1 only human tissues.
(b) 2 only 2. Participants who are involved in active scientific
(c) Both 1 and 2 research can only be a part of this network.
(d) Neither 1 nor 2 3. The data collected under the project will be
113. Consider the following statements with regard to useful for drug developers.
the National Genomic Grid: Which of the statements given above are correct?
1. It will study genomic data of patients suffering (a) 1 and 3 only
from a Genetically Inherited disease. (b) 1 and 2 only
2. The study will utilize the technique of genome (c) 2 and 3 only
sequencing. (d) 1, 2 and 3
Which of the statements given above is/are correct? 118. Which of the following statement(s) about Atal Jai
(a) 1 only Anusandhan Biotech mission is/are correct?
(b) 2 only 1. It aims to transform Health, Agriculture and
(c) Both 1 and 2 Energy sectors.
(d) Neither 1 nor 2
2. It is a joint mission of the Ministry of Health
114. Consider the following statements: and Family Welfare, Ministry of Agriculture and
1. Unlike Cloning, Genetic engineering allows Farmers’ Welfare and Ministry of Power.
parents to give their children genes that have 3. One of the components of the scheme is
been intentionally designed or modified in the the management and treatment of inherited
laboratory. disorders.
2. Both Genetic modification and Cloning can Select the correct answer using the code given below:
occur naturally. (a) 1 only
Which of the statements given above is/are correct? (b) 2 and 3 only
(a) 1 only (c) 2 only
(b) 2 only (d) 1 and 3 only
(c) Both 1 and 2 119. Which of the following technique(s) could be used
(d) Neither 1 nor 2 in the forensic analysis?
115. In relation to the developments in the field of 1. Mitochondrial DNA Testing.
genetics, which of the following statements is 2. Brain Electrical Oscillation Signature (BEOS)
incorrect? Profiling.
(a) DNA fragments, which result from the breakup 3. Real Time-Polymerase Chain Reaction Test.
of DNA strands, cannot be separated.
(b) Disease causing mitochondria carried by Select the correct answer using the code given below.
a woman can be replaced with a healthy (a) 1 and 3 only
mitochondrion. (b) 2 and 3 only
(c) We can create ‘recombinant’ molecules of DNA, (c) 2 only
which are composed of DNA from different (d) 1, 2 and 3
genomes.
(d) DNA sequence of an organism can be altered 120. Consider the following pairs:
through genome editing.
Term/Phrase Context/Topic
116. Consider the following statements: sometimes seen in
1. Gene Therapy Product (GTP) is defined as any news
biological entity, having the required gene, that
could introduce modifications in the genome for 1. Dickinsonia Artificial Intelligence
therapeutic benefit. 2. novel Open Genetic research
2. GTPs work by repairing, replacing or Reading Frames
deactivating dysfunctional disease-causing
genes aiming to restore normal function. 3. Little foot Digital currency

105 Workbook
.
SCIENCE & TECHNOLOGY

Which of the pairs given above is/are correctly 3. It ensures maximum safety as there is no
matched? emission of harmful chemicals in the process.
(a) 1 and 2 only Which of the statements given above is/are correct?
(b) 2 and 3 only (a) 1 only
(c) 2 only (b) 1 and 2 only
(d) 1 and 3 only (c) 2 and 3 only
121. Consider the following statements with respect (d) 1, 2 and 3
to the comparison between genetically modified 125. With reference to Transgenic Animals, consider
organisms and cloned organisms: the following statements:
1. While genetically modified organisms have 1. These animals contain foreign genes which are
modified DNA with desired traits, a cloned administered deliberately into them.
organism is the genetically identical copy of a 2. Transgenesis in animals is done through
biological entity. recombinant DNA technology.
2. Unlike cloning, genetic modification does not 3. Pigs can be modified through transgenesis to
disrupt the natural process of gene flow. make them more suitable as organ donors for
3. While genetic modification cannot be done human transplant patients.
naturally, cloning of some organisms occur Which of the statements given above is/are correct?
naturally.
(a) 1 only
Which of the statements given above is/are correct? (b) 1 and 2 only
(a) 1 and 2 only (c) 2 and 3 only
(b) 2 only (d) 1, 2 and 3
(c) 1 only
(d) 2 and 3 only 126. Regarding gene therapy, which of the following
statement is incorrect?
122. With reference to the stem cells, consider the (a) In this method, a mutated gene is revised,
following statements: removed, or replaced at the DNA level.
1. Stem cells are unspecialized cells capable of (b) This method results in destruction of the
dividing and renewing themselves. problem causing genes.
2. Unlike embryonic stem cells, adult stem cells can (c) It can be used to treat both inherited genetic
become any cell in the body. diseases and acquired disorders.
3. Stem cell therapy is a non-surgical procedure of (d) Viral vectors are used for the transfer of genetic
repairing damaged tissues. materials.
4. It has no side effects as it induces the body to 127. Which of the following is/are the applications of
recover through natural healing. Nanotechnology:
Which of the statements given above is/are correct? 1. Green Technology
(a) 2 and 3 only 2. Cosmetics
(b) 1, 3 and 4 only 3. Construction of spacecraft and satellites
(c) 2 and 4 only 4. New forms of carbon
(d) 1, 2, 3 and 4 5. To create artificial organs and implants
123. Regarding the Bt Cotton, which of the following Select the correct answer using the codes given
statements is incorrect? below:
(a) It is a genetically modified cotton containing (a) 1, 2, and 3 only
pesticide gene from bacteria Bacillus (b) 2, 3, 4 and 5 only
thuringiensis. (c) 1, 2, 3 and 5 only
(b) It is the only approved genetically modified crop (d) 1, 2, 3, 4 and 5
for commercialisation in India.
128. With reference to the biofortification technology,
(c) It provides effective protection to cotton plants
consider the following statements.
against the bollworm attack.
(d) It boosted India’s cotton productivity making it 1. It is the process of enriching the food crops
largest cotton exporter in the world. with vitamins and other minerals during the
processing.
124. Consider the following statements with respect to 2. Iron-biofortification of rice has not been
the 3D Bio printing: successful because the process is susceptible to
1. It is an additive manufacturing process that uses stress factors.
cells to print living structures layer by layer. 3. Golden rice developed by International Rice
2. It can mimic the actual micro and macro Research Institute (IRRI) aims to address
environment of human tissues. Vitamin-A Deficiency.

Workbook 106
.
SCIENCE & TECHNOLOGY

Which of the statements given above is/are correct? 1. The permissible limit for genetically modified
(a) 1 and 2 only organisms (GMO) in imported food crops is set
(b) 3 only by Food Safety and Standards Authority of India.
(c) 1 and 3 only 2. Cotton is the only GM crop allowed to be
(d) 1, 2 and 3 imported to India.
3. A transgenic organism is one that has been
129. With reference to the Single-Celled Proteins altered through recombinant DNA technology.
(CSP), consider the following statements.
4. Use of the unapproved GM variant is punishable
1. Single-celled proteins (SCP) are used as the under the Environmental Protection Act, 1986.
protein supplement in animal feeds.
Which of the statements given above is/are correct?
2. Production of CSP can help in reducing the
(a) 1 and 2 only
environmental pollution. (b) 2, 3 and 4 only
3. Blue green algae known as Spirulina is used for (c) 1, 3 and 4 only
production of CSP. (d) 1, 2, 3 and 4
Which of the statements given above is/are correct?
134. Which of the following methods can be used to
(a) 1 and 2 only improve the shelf life of a food product?
(b) 2 and 3 only 1. Waxing
(c) 1 and 3 only
2. Blanching
(d) 1, 2 and 3
3. Ohmic heating
130. With reference to recent developments regarding 4. Vacuum degassing
‘Recombinant Vector Vaccines’, consider the 5. Product irradiation
following statements:
Select the correct answer using the code given below:
1. Genetic engineering is applied in the (a) 1 and 2 only
development of these vaccines. (b) 1, 2 and 4 only
2. Bacteria and viruses are used as vectors. (c) 1, 2, 4 and 5 only
Which of the statements given above is/are correct? (d) 1, 2, 3, 4 and 5.
(a) 1 only 135. Which of the following can be the possible benefits
(b) 2 only of using nanotechnology in agriculture?
(c) Both 1 and 2 1. Reduced nutrient run-off into water
(d) Neither 1 nor 2 2. Enhanced irrigation efficiency
3. High seed and root metabolism
2.3. Science in Environment and Agriculture Select the correct answer using the code given below:
131. Which of the following statement is incorrect (a) 1 only
about Biotech-KISAN Scheme? (b) 1 and 3 only
(a) Biotech KISAN will connect farmers with (c) 2 and 3 only
science in different agro-climatic zones of the (d) 1, 2 and 3
country. 136. Which of the following statements is/are correct
(b) Biotech KISAN hubs created under the scheme regarding the ‘Aerial Seeding’?
will get financial assistance for first 2 years. 1. It is a weather modification technique that aims
(c) The scheme includes Biotech KISAN fellowship to cause rainfall by dispersing certain chemical
to landless labourers. substances into the air.
(d) The scheme is under administrative control of 2. This technique has never been deployed in India
Ministry of Science and Technology. due to associated environmental concerns.
132. Which of the following statement is correct about Select the correct answer using the code given below:
golden rice? (a) 1 only
(a) It was created as an intervention for vitamin C (b) 2 only
deficiency (c) Both 1 and 2
(b) The golden color of the rice is due to the use of a (d) Neither 1 nor 2
new type of fertilizer. 137. In the context of bio-pesticides, consider the
(c) To tackle vitamin deficiency, India has allowed following statements?
commercial cultivation of golden rice. 1. Fungal species cannot be used in the production
(d) Golden rice is completely not for profit project. of bio-pesticides.
133. With reference to the GMO (genetically modified 2. Compared to conventional pesticides, bio-
organism) crops, consider the following pesticides usually have lesser impact on non-
statements: target species.

107 Workbook
.
SCIENCE & TECHNOLOGY

Select the correct answer using the code given below: (c) 1 and 2 only
(a) 1 only (d) 2 and 3 only
(b) 2 only 142. Which of the following statements is/are correct
(c) Both 1 and 2 regarding hydroponics, a farming technique?
(d) Neither 1 nor 2
1. It allows the cultivation of plants without using
138. What are the possible benefits from the use of soil.
direct seeding of rice (DSR) technique in place of 2. Water usage in hydroponics is more than that of
traditional transplantation, for the cultivation of traditional farming.
paddy? 3. In hydroponics, chemical fertilizers cannot be
1. Reduce the practice of stubble burning used.
2. Less water will be needed for irrigation Select the correct answer using the code given below:
3. Shorter maturity period for crops (a) 1, 2 and 3
Select the correct answer using the code given below: (b) 2 and 3 only
(a) 1 and 2 only (c) 1 only
(d) 1 and 2 only
(b) 2 and 3 only
(c) 1 and 3 only 143. With reference to GM crops, consider the
(d) 1, 2 and 3 following statements:
139. With reference to the use of various emerging 1. A GM crop has a gene artificially inserted into
technologies in agriculture, consider the following it from another species to give it some desired
statements: properties like – pest resistance, herbicide
tolerance, drought resistance etc.
1. Blockchain technology can be used for
2. Currently, only GM crop permitted for
digitization of land records.
cultivation in India is Bt Cotton.
2. Internet of Things can be used to monitor soil
3. Bt cotton can tolerate Glyphosate, a herbicide
conditions.
without affecting the cotton crop.
3. Artificial Intelligence can be used to predict
weather, sowing time and irrigation schedule. Which of the following statements given above is/are
correct?
Which of the statements given above is/are correct? (a) 1 only
(a) 1 only (b) 1 and 2 only
(b) 1 and 2 only (c) 1 and 3 only
(c) 2 and 3 only (d) 1, 2 and 3
(d) 1, 2 and 3
144. With reference to the practice of tissue culture,
140. ‘PUSA bio-decomposer’ is sometimes seen in the consider the following statements.
news in context of stubble burning, consider the 1. Tissue culture allows the regeneration of plant
following statements regarding it. from different parts of a plant except the root.
1. It improves the fertility and productivity of the 2. Plants produced through this method are
soil. identical to the original plant.
2. Food grains produced using the Pusa 3. Tissue culture allows the recovery of healthy
Decomposer qualify as organic farming. plant from diseased plant.
Which of the statements given above is/are correct? Which of the statements given above is/are correct?
(a) 1 only (a) 1 and 2 only
(b) 2 only (b) 2 and 3 only
(c) Both 1 and 2 (c) 1 and 3 only
(d) Neither 1 nor 2 (d) 1, 2 and 3
141. In the context of Food irradiation, consider the 145. With reference to the agricultural practice of
following statements: apiculture, consider the following statements.
1. Gamma rays emitted from Cobalt 60 can be used 1. Harvesting honey is normally done every month
for food irradiation. throughout the year.
2. Perishable items like fruits and vegetables cannot 2. Bee-keeping practice is not bound by the
be preserved using food irradiation. locational factor.
3. Irradiation noticeably change the taste, texture, 3. Apis Indica is one of the most common bee
or appearance of food items. species reared in India.
Which of the statements given above is/are correct? Which of the statements given above is/are correct?
(a) 1 and 3 only (a) 1 and 2 only
(b) 1 only (b) 2 and 3 only

Workbook 108
.
SCIENCE & TECHNOLOGY

(c) 3 only 2. A new drug can obtain approval in India only


(d) 1 ,2 and 3 after the clinical trial has been completed.
146. With reference to the plant breeding technique, 3. The applications for conducting clinical trial
for drugs manufactured in India need to be
consider the following statements.
examined within 30 days.
1. Plant breeding is a technique to develop disease
resistant crops. Which of the statements given above is/are correct?
2. Plant breeding technique has been practiced (a) 1 and 2 only
10,000 years ago. (b) 2 only
(c) 1 and 3 only
3. Irradiation can be used to induce mutations in
(d) 2 and 3 only
plants.
Which of the statements given above is/are correct? 150. With reference to proton therapy, consider the
(a) 1 and 2 only following statements:
(b) 2 and 3 only 1. It is a type of radiation therapy which uses
(c) 1 and 3 only protons to treat cancer.
(d) 1, 2 and 3 2. In this therapy, protons do not exit the target
tumor and the healthy tissue stays unaffected.
147. Consider the following statements with respect to 3. The positive charge on a proton is equal in
cryopreservation and food irradiation: magnitude to the negative charge on an electron.
1. Cryopreservation is a technique of preserving
Which of the statements given above are correct?
cells and tissue at very low temperature.
(a) 1 and 2 only
2. Food irradiation is the process of passing the
(b) 2 and 3 only
food through high energy electrons to eliminate
(c) 1 and 3 only
microorganisms from it.
(d) 1, 2 and 3
3. While cryopreservation preserves food without
compromising its nutritional content, irradiation 151. In the context of recent advances in 3 D
compromises the nutritional quality of food. bioprinting, which of the above activities can be
Which of the statements given above is/are correct? successfully carried out?
(a) 1 and 2 only 1. Functional human skin
(b) 3 only 2. Load-bearing bones
(c) 2 and 3 only 3. Functional liver tissues
(d) 1, 2 and 3 4. Replace lengthy clinical trials.
Select the correct answer using the codes given
2.4. Health and Medicine below:
(a) 1, 2 and 3 only
148. Consider the following statements regarding
(b) 2 and 4 only
‘Somatic cell nuclear transfer’: (c) 1, 3 and 4 only
1. It involves transferring the nucleus of a somatic (d) 1, 2, 3 and 4
cell into an egg which has had its nucleus
removed. 152. With reference to Lab Grown/ cultured meat,
2. Somatic Cell Nuclear Transfer technique is consider the following statements:
currently the basis for cloning animals like sheep. 1. The cellular structure of meat is same as natural
3. This method can be used to generate tissues meat.
or even organs for transplant into the specific 2. It can be made by using animal’s stem cells.
patient. 3. Texture and nutrient profile of cultured meat is
4. Scientists have not yet managed to clone a low compared to natural meat.
primate. 4. Unlike natural meat, cultured meat doesn’t
Which of the following statements given above is/are require antibiotic treatment.
correct? Which of the statements given above are correct?
(a) 1, 2 and 3 only (a) 1, 2 and 3 only
(b) 2 and 3 only (b) 1, 2 and 4 only
(c) 2, 3 and 4 only (c) 1, 3 and 4 only
(d) 1, 2, 3 and 4 (d) 2, 3 and 4 only
149. Consider the following statements with regards to 153. A person who is born with a hereditary disease,
Drug and Cosmetics act, 1940: undergoes correction of a gene defect that is
1. The central drugs standard control organisation causing the disease. Here, genes are inserted
approves new drug in India. into a person’s cells and tissues to treat a disease.

109 Workbook
.
SCIENCE & TECHNOLOGY

Correction of a genetic defect involves delivery of 2. In vaccination, antigenic proteins of the


a normal gene into the individual or embryo to pathogen are introduced into the body.
take over the function of and compensate for the 3. Preformed antibodies can be injected into the
non-functional gene. body in case of a deadly microbial attack.
Which of the following terms best describes the Which of the statements given above are correct?
above-mentioned process? (a) 1 and 2 only
(a) Molecular Diagnosis (b) 2 and 3 only
(b) Gene Therapy (c) 1 and 3 only
(c) Active Immunity (d) 1, 2 and 3
(d) Crossbreeding 158. With reference to mRNA Vaccine, consider the
154. Consider the following statements: following statements:
1. Vaccines contain antibodies that triggers an 1. It uses the genetic sequence of a portion of the
immune response. virus rather than an actual virus.
2. Some vaccines require multiple doses for 2. They can be made in large quantities and in
production of long-lived antibodies and memory lesser time than traditional vaccines.
cells. Which of the statements given above is/are correct?
3. Herd immunity provides full protection to those (a) 1 only
who cannot be safely vaccinated. (b) 2 only
4. There is no threat of a disease being caused from (c) Both 1 and 2
live attenuated vaccine. (d) Neither 1 nor 2
Which of the statements given above is/are incorrect? 159. Consider the following statements:
(a) 1 and 2 only 1. In India, one can get their DNA checked with
(b) 2 and 3 only their parents to resolve parental dispute without
(c) 1, 3 and 4 only court order.
(d) 1, 2, 3 and 4 2. DNA profiling could be misused for caste-based
profiling.
155. With reference to ‘Polymerase Chain Reaction
Which of the statements given above is/are correct?
(PCR)’, consider the following statements:
(a) 1 only
1. It is a technique used for early diagnosis of
(b) 2 only
diseases. (c) Both 1 and 2
2. It is being used to detect mutations in genes in (d) Neither 1 nor 2
suspected cancer patients.
3. HIV in suspected AIDS patients cannot be 160. Which one of the following Statements about
detected using this technique. Saviour Sibling experiment is incorrect?
(a) Stem cells of saviour siblings can be used to treat
Which of the statements given above is/are correct? serious blood disorders like Thalassemia, Sickle
(a) 1, 2 and 3 cell anemia.
(b) 2 only (b) Saviour sibling experiment has been successfully
(c) 1 and 2 only conducted recently in India.
(d) 2 and 3 only (c) In vitro fertilization cannot be used to conceive
a saviour sibling.
156. Which of the following is/are the application of
(d) Embryo of a saviour sibling must have a
stem cells?
matching Human Leukocyte Antigen (HLA) to
1. Repairing a damaged tissue prevent any complications.
2. Skin replacement
161. With reference to Assisted Reproductive
3. Bone marrow transplantation
Technologies (ARTs), which one of the following
4. Treatment of Parkinson’s disease statements is not correct?
Select the correct answer using the code given below: (a) In-Vitro Fertilization involves fertilization of
(a) 1 and 4 only woman’s eggs and man’s sperms outside the body
(b) 1, 2 and 3 only in a laboratory dish.
(c) 2, 3 and 4 only (b) Intrauterine insemination requires injection of
(d) 1, 2, 3 and 4 semen into the uterus at a predetermined time
to increased possibility of fertilization.
157. With reference to vaccination, consider the (c) An intrafallopian transfer involves fertilization
following statements: of the egg inside the woman’s fallopian tube.
1. The main aim of vaccination is to teach the (d) Intracytoplasmic Sperm Injection involves the
immune system how to recognize and eliminate use of a special needle to inject the egg inside the
a pathogen. sperms.

Workbook 110
.
SCIENCE & TECHNOLOGY

162. The presence of which of the following bacterium Which of the statements given above is/are correct?
in the Aedes aegypti mosquitoes reduces the (a) 1 and 2 only
mosquitoes’ ability to transmit viruses like (b) 2 and 3 only
dengue, Zika, and chikungunya? (c) 1 and 3 only
(a) Mollicutes (d) 1, 2 and 3
(b) Spiro plasma
(c) Wolbachia 167. With reference to Vaccination which of the
(d) None of the above following statements is/are correct?
1. Passive immunization is a process in which
163. In the context of developmental biology, which of individuals could receive antibodies directly
the following statements regarding the ‘Organoid’
from another source.
is/are correct?
2. Preformed antibodies are derived from the blood
1. It is a three-dimensional miniature structure
that replicates a human organ. serum of previously infected people or animals.
2. It can be created using the stem cells. Select the correct answer using the code given below:
3. It can achieve the functional maturity similar to (a) 1 only
a human organ. (b) 2 only
Select the correct answer using the code given below: (c) Both 1 and 2
(d) Neither 1 nor 2
(a) 1 and 2 only
(b) 2 and 3 only 168. Which of the following are the modes through
(c) 1 and 3 only which an HIV infection may get transmitted to
(d) 3 only humans?
164. Which of the following would be the most likely 1. Blood/ Organ/ Tissue transplants.
consequence/consequences, if an edible oil 2. Sharing of needles, syringes and other injection
contains industrial trans-fat? equipment.
1. There would be an increased risk of damage to 3. From an HIV infected mother to foetus/ infant.
the cardiovascular health of the consumers. 4. Sharing dishes etc where they may be exchange
2. FSSAI will not allow the said edible oil to be sold of saliva.
in the market. Select the correct answer using the code given below:
3. The shelf life of the edible oil would drastically (a) 1, 2 and 3 only
reduce.
(b) 2, 3 and 4 only
Select the correct answer using the code given below: (c) 1, 3 and 4 only
(a) 1 and 3 only (d) 1, 2, 3 and 4
(b) 1 only
(c) 1 and 2 only 169. Which of the following are the drugs that can
(d) 2 and 3 only be used by sports persons to enhance their
performance?
165. Consider the following statements: 1. Melatonin
Assertion (A): Organ transplant fails if the donor 2. Anabolic Androgenic Steroids
organ does not match the recipient’s blood.
3. Erythropoietin
Reason (R): The immune system identifies foreign 4. Human Growth Hormone
cells and attack them.
Select the correct answer using the code given below:
Which of the options given below is correct based on
the above given statements? (a) 1, 2 and 3 only
(b) 2, 3 and 4 only
(a) Both A and R are true and R is the correct
(c) 1, 2 and 4 only
explanation of A.
(d) 1, 2, 3 and 4
(b) Both A and R are true but R is not the correct
explanation of A. 170. With reference to various types of vaccines,
(c) A is true but R is false. consider the following statements:
(d) A is false but R is true. 1. Unlike DNA vaccines, RNA vaccines instruct
166. Consider the following statements: cells in our body to produce a protein that
1. Innate immunity offers a lifelong protection with induces an immune response.
fast acting response from the immune system. 2. RNA vaccines can be synthesised chemically
2. Adaptive immunity is an organism’s acquired without the need for any bacteria or cells.
immunity to a specific pathogen. 3. Unlike DNA vaccines, RNA vaccines have zero
3. Helper T cells are the most important cells in risk of integrating with our own genetic material.
innate immunity. Which of the above statements is/are correct?

111 Workbook
.
SCIENCE & TECHNOLOGY

(a) 1 only 175. Consider the following statements about the


(b) 1 and 2 only various techniques of ‘Assisted Reproductive
(c) 3 only Technology’ (ART):
(d) 2 and 3 only 1. Invitro Fertilization cannot be performed if the
171. With reference to the cloning, which of the fallopian tubes are blocked or absent.
following statements is/are correct? 2. Gamete Intrafallopian Transfer involves
1. Cloning technique can be used to produce combining eggs and sperms outside the human
offspring that are genetically similar to their body.
donor parent. 3. In Zygote Intrafallopian Transfer, eggs are
2. Single-celled organisms like bacteria make exact fertilized with the male sperm outside the
copies of themselves each time they reproduce. human body.
3. Cloning technique can bring back extinct 4. Unlike Gamete Intrafallopian Transfer, Invitro
species. Fertilization does not result in “biologically
related” child.
Select the correct answer using the code given below:
(a) 1 only Which of the above statements is/are correct?
(b) 2 and 3 only (a) 1, 2 and 3 only
(c) 2 only (b) 1 and 4 only
(d) 1, 2 and 3 (c) 2, 3 and 4 only
(d) 2 and 3 only
172. Which of the following statements given below
correctly describes ‘Transcription’? 176. With reference to a birth control measure called as
(a) It is the process of copying genetic information contraceptive pills method, which of the following
from one strand of DNA to RNA. statement is incorrect?
(b) It is the process of breaking a segment of a (a) Emergency contraceptive pills do not induce an
chromosome and reattaching itself with another abortion.
chromosome with mutual exchange. (b) The copper-bearing IUD is the most commonly
(c) It is the process of copying genetic information used contraceptive pill.
from RNA to a single stranded DNA. (c) It prevents the mature eggs being released from
(d) It is the process of breaking a segment of a the ovary
chromosome and attaching with another (d) It does not protect the person from Sexually
chromosome without mutual exchange. Transmitted Diseases (STDs)
173. The chemical compound ‘N-Nitroso- 177. Which of the following can be the possible use of
dimethylamine’ recently seen in news, is related to Magnetism in medicine?
(a) Compounds used in fertilizers causing Ground 1. Imaging diagnosis of the whole-body system.
water contamination. 2. Mass spectrometers.
(b) Catalysts used in bioremediation process of soil
3. Treatment of Parkinson’s disease.
pollution.
(c) Explosives used in mines under controlled 4. Treatment of depression.
manner. Select the correct answer using the code given below.
(d) Are strong Carcinogenic compounds among (a) 1, 2 and 3 only
animals and most likely in humans. (b) 2, 3 and 4 only
174. Consider the following statements with reference (c) 1, 2, 3 and 4
to food irradiation: (d) 1, 3 and 4 only
1. Gamma irradiation is used for insect 178. “Stethoscope is a medical instrument. It is
disinfestation of cereals, pulses and grains. generally used by the doctors for listening to the
2. The Codex Alimentarius Commission and sounds produced within the human body mainly
Codex Standards recommends the complete ban in the heart and lungs. In stethoscopes, doctor
on irradiation of food items. listens to the sound of the patient’s heartbeat.”
3. Food Irradiation can delay ripening of fruits and Based on above paragraph, which physical principle
store foods for years without refrigeration. is used by stethoscopes to listens to the sound of the
Which of the statements given above is/are correct? patient’s heartbeat.”
(a) 1 and 3 only (a) Doppler effect
(b) 2 only (b) Tyndall effect
(c) 1 and 2 only (c) Multiple Reflection of sound
(d) 1, 2 and 3 (d) Principle of ultrasound

Workbook 112
.
SCIENCE & TECHNOLOGY

179. Which of the following are applications of 184. Consider the following statements about Radio
ultrasound? Frequency Identification (RFID) technology:
1. used in welding of thermoplastics. 1. Like Barcoding, RFID uses a pattern of black
2. to detect the presence of the flaw or defect in bars and white spaces in which the information
metal blocks. is coded.
3. used in humidifiers to spray the water. 2. Both the antenna and the tag are to be located
4. used by animals to detect their prey. very near to each other for communication to
5. in radar surveillance to determine the speed of take place.
automobiles. 3. This technology has been deployed in FASTag.
Select the correct answer using the code given below: Which of the statements given above is/are correct?
(a) 1, 2 and 5 only (a) 1 and 2 only
(b) 1, 2, 3 and 4 only (b) 3 only
(c) 3, 4 and 5 only (c) 2 and 3 only
(d) 2, 3 and 5 only (d) 1, 2 and 3

2.5. Information Technology 185. “A software company X has developed a health app
for smartphones that tracks diet, sleep patterns,
180. With reference to WiMAX technology consider heart rate or blood sugar levels. The app can
the following statements: alert to adverse situations in the user’s health and
1. The speed of WiMAX is higher than Wi-Fi. suggest behavioural modifications towards a more
2. It receives its signal directly from the positive outcome.” In the context of emerging
communication satellites. communication technologies, which one of the
Which of the statements given above is/are correct? following terms best applies to the above scenario?
(a) 1 only (a) Virtual Reality
(b) 2 only (b) Internet of Behaviour
(c) Both 1 and 2 (c) LIDAR
(d) Neither 1 nor 2 (d) Quantum Computing
181. In which of the following signature a physical 186. This configuration has become an essential
mark is made by the person on the object which component of the overall 5G architectural
is to be signed? landscape which allows operators to separate
(a) Click wrap signature the packet traffic layers to support multiple
(b) Electronic signature applications and services running in parallel for
(c) Digital signature a range of users who require different levels of
(d) Wet mark signature quality, latency, and bandwidth. The technology
182. Which of the following statement best describes mentioned here is:
Cognitive Computing? (a) Network Slicing
(a) Building smart machines capable of performing (b) Network virtualization
tasks that typically require human intelligence. (c) VPN
(b) Use of quantum phenomena such as (d) Tor
superposition and entanglement to perform
computation. 187. Which of the following statements is/are correct
(c) Developing implantable brain–machine about the technology used for tackling cyber
interfaces (BMIs). security threats?
(d) Use of computer to simulate human thought 1. Use of smart grid technologies have reduced the
processes. threat of cyber-attacks.
183. Consider the following statements about Bharat 2. Behavior analysis of the network helps detect
QR code: potential and real-time cyber threats.
1. It is a single unified QR code capable of accepting 3. Deep learning can play an important role in
payments from Visa, MasterCard, RuPay Cards. detecting cyber threats.
2. It requires an upfront investment in form of 4. Sinkholing is the latest technology to redirect
Point of Sale (PoS) machine. malicious traffic ensuring protection from
3. It has been developed by Reserve Bank of India. cyber-attacks.
Which of the statements given above is/are correct? Select the correct answer using the code given below:
(a) 1 only (a) 1 and 2 only
(b) 1 and 3 only (b) 1, 2 and 3 only
(c) 2 and 3 only (c) 2, 3 and 4 only
(d) 1, 2 and 3 (d) 1, 2, 3 and 4

113 Workbook
.
SCIENCE & TECHNOLOGY

188. Although the 4th industrial revolution has given Which of the statements given above is/are correct?
us a lot but it has also created problems like deep (a) 1 only
fakes. With respect to deep fakes which of the (b) 2 only
following statements is/are correct? (c) Both 1 and 2
1. Deep fake uses deep learning technology to (d) Neither 1 nor 2
make images of fake events.
194. Online data theft means an information is illegally
2. Deep fake can only alter pictorial data and copied or taken from a business or other individual
cannot affect audio files. through the internet. Which of the following is/
3. An unskilled person cannot create deep fakes, it are among data stealing methods used by hackers?
require technical specialists. 1. Malware
Select the correct answer using the code given below: 2. Smishing
(a) 1 only 3. Keylogger
(b) 1 and 2 only Select the correct answer using the code given below.
(c) 2 and 3 only
(d) 1, 2 and 3 (a) 1 and 2 only
(b) 3 only
189. ‘SolarWinds hack’, recently seen in news, relates to (c) 1 and 3 only
which one of the following? (d) 1, 2, and 3
(a) International standards for generation of 195. With respect to telecommunication technology
renewable energy. 4G and 5G, what advantages 5G has over 4G?
(b) A recently discovered cyber-attack.
1. Internet speed
(c) Global collaboration for conducting a solar
probe. 2. Lower Latency
(d) A portal to report on cyber security incidents. 3. Less interference
4. Longer range
190. Recently seen in news, the Quantum Key
5. Lower Initial rollout cost.
Distribution (QKD) is primarily a mechanism to:
(a) multiply specific sequences of RNA of a virus. Select the correct statement using the code given
(b) undertake highly-secure communication. below:
(c) explore the possibility of using block chain (a) 1, 2 and 3 only
technology. (b) 1, 3 and 4 only
(d) conduct gene-editing and gene sequencing. (c) 1, 2 and 4 only
(d) 1, 2, 3, 4 and 5
191. Which of the following statements regarding the
HAM Radio, seen in news recently, is correct? 196. In the context of communication technologies,
what is/are the difference/ differences between 4G
(a) Frequent communication hindrances are
and 5G?
encountered while communicating through it.
(b) A licence is required to operate it in India. 1. 5G has higher latency than that of the 4G.
(c) It was first used in India during the 1971 2. Telecom spectrum required for 5G is more than
Bangladesh liberation war. that of 4G.
(d) It cannot access satellite frequencies. 3. Unlike 4G, 5G does not allow ‘network slicing’.
192. Which one of the following statements is incorrect Select the correct answer using the code given below.
with reference to zero-click attack? (a) 1 and 2 only
(a) Spywares using zero click attacks are more (b) 2 only
dangerous than those using text links or (c) 2 and 3 only
messages. (d) 1, 2 and 3
(b) Zero click attacks provide access to target device 197. BSNL has launched the world’s first satellite-based
in real time. narrowband IoT (Internet of Things) network.
(c) Messaging and voice calling apps are highly What is/are the advantage/advantages of such a
vulnerable to such attacks. network?
(d) Detection of zero-click attack is easier in 1. PAN-India coverage including Indian seas.
encrypted environments. 2. Easy deployment of this network into existing
193. Consider the following statements regarding the cellular network architecture.
CollabCAD: 3. The network does not leave any dark patch
1. It is a computer enabled software system, within the boundary of India.
providing total engineering solutions. 4. Low power consumption and low component
2. It will provide a platform to the students of Atal cost.
Tinkering Labs to create and modify 3D designs. Select the correct answer using the code given below:

Workbook 114
.
SCIENCE & TECHNOLOGY

(a) 1 and 2 only to the location where it is needed, to improve


(b) 2, 3 and 4 only response times and save bandwidth.
(c) 1, 2 and 4 only
(d) 1, 2, 3 and 4 202. ‘Edge Computing’ technology has gained
popularity in the last few years. In which of the
198. Some digital coin miners illegally gain access following areas can the application of ‘Edge
to many computers. The miners stealthily drop Computing’ technology be used?
malware in an unsuspecting user’s computer. 1. 5th Generation network
These malware runs surreptitiously and turns 2. Autonomous vehicles
devices into cryptocurrency-mining botnets. 3. Smart Grid
Which of the following terms best applies to the 4. In-hospital patient monitoring
above scenario? 5. Traffic management
(a) Crypto-jacking
Select the correct answer using the codes given
(b) Blockchain
below:
(c) Artificial Intelligence
(d) Crypto-blocking (a) 1, 2, 3 and 5 only
(b) 2, 3, 4 and 5 only
199. Many supermarkets offer free loyalty cards to (c) 4 and 5 only
customers that give them access to reduced prices (d) 1, 2, 3, 4 and 5
not available to non-members. The cards make
it easy for stores to track who is buying what, 203. Consider the following statements with reference
when they are buying it and at what price. After to the Big Data:
analysing the data, stores can then use this data to 1. Big Data refers to a large amount of data which
offer customers coupons targeted to their buying is of structured, semi-structured or unstructured
habits and decide when to put items on sale or nature.
when to sell them at full price. 2. Big Data problems can be solved by Cloud
computing technology.
Which of the following terms describes this process
to obtain useful information? 3. NITI Aayog is planning to develop the ‘National
Data & Analytics Platform’, for evidence-based
(a) Big Data
policymaking guided by Big Data.
(b) Data mining
(c) Artificial intelligence Which of the statements given above is/are correct?
(d) Cloud Computing (a) 1 and 2 only
(b) 2 and 3 only
200. With reference to Quantum computing, consider (c) 1 and 3 only
the following statements: (d) 1, 2 and 3
1. Quantum computer is based on laws that govern
the behaviour of atoms and subatomic particle. 204. Consider the following systems:
2. Quantum bits can be either 0 or 1 at a time. 1. Cognitive computing can simulate human
thought.
3. Which of the statements given above is/are
correct? 2. The personal digital assistants we have on our
phones and computers are the highest example
(a) 1 only
of cognitive computing systems.
(b) 2 only
(c) Both 1 and 2 Which of the statements given above is/are correct?
(d) Neither 1 nor 2 (a) 1 only
(b) 2 only
201. Which of the following statements best describes (c) Both 1 and 2
the term ‘Edge computing’? (d) Neither 1 nor 2
(a) It is a framework for computation derived from
recurrent neural network theory that maps input 205. Consider the following statements with reference
signals into higher dimensional computational to the applications of deoxyribonucleic acid
spaces through the dynamics of a fixed, non- (DNA) computing:
linear system. 1. DNA based computing uses biological molecules
(b) It is a form of ubiquitous computing which uses instead of silicon chips for computing.
sensors to perceive its environment and react 2. DNA computing can be used in cryptography to
accordingly build an intrusion detection model.
(c) It is a concept in software engineering and 3. Unlike traditional binary coding, DNA
computer science where computing is made to computing uses chemical bases to hide the
appear anytime and everywhere. messages.
(d) It is a distributed computing paradigm that 4. DNA computing-based clustering involves using
brings computation and data storage closer strands to assign edges and vertices.

115 Workbook
.
SCIENCE & TECHNOLOGY

Which of the statements given above is/are correct? Which of the statement(s) given above is/are correct?
(a) 1, 2 and 3 only (a) 1 and 3 only
(b) 2, 3 and 4 only (b) 3 only
(c) 1, 2 and 4 only (c) 1 and 2 only
(d) 1, 2, 3 and 4 (d) 2 only
206. Which of the following is incorrect with reference 210. Which of the following describes the
to the QR (quick response) code and Bar code Chandrashekhar Limit?
functions: (a) It is the maximum mass a white dwarf star can
(a) QR code contains more information than a have before becoming unstable.
barcode. (b) It is the minimum distance after which an object
cannot return to Earth.
(b) QR code only contains information in a
(c) It is the limit below which electron degeneracy
horizontal direction.
pressure is not enough to prevent gravity.
(c) Unlike a normal barcode, QR code is two- (d) It is the gravity of Sun, which holds together all
dimensional. the planets in our solar system.
(d) QR code works even if the printed code is
damaged or soiled. 211. Which of the following is correct with reference to
Hyper spectral imaging technology:
207. With reference to ‘Cybercrimes’, consider the (a) Hyper spectral imaging is a technique that
following statements regarding the difference analyzes a wide spectrum of light instead of
between Zero Click Attacks and Spear Phishing: assigning primary colors to each pixel.
1. Zero-click attacks trick a victim into revealing (b) Hyper spectral imaging has been particularly
confidential information whereas Spear useful in satellite technology only.
Phishing gain control over a device without any (c) Unlike optical technologies which can scan full
human interaction. color spectrum, Hyper spectral imaging can
2. Zero-click attacks can be detected easily as only scan for a single color.
compare to Spear Phishing due to visibility on (d) The spectral range in hyper spectral data cannot
the data packets being sent or received. extend beyond the visible range.
Which of the statements given above is/are correct? 212. Consider the following statements with reference
(a) 1 only to the Laser and its applications:
(b) 2 only 1. Lasers can have application in processing of data.
(c) Both 1 and 2 2. Lasers are quite useful in the process of
(d) Neither 1 or 2 Microscopy.
3. Laser machining is dependent on the material
208. Consider the following statements with reference hardness.
to the Machine learning:
Which of the statements given above is/are correct?
1. Machine learning is a subset of deep learning
that relates the recurrent neural networks and (a) 2 only
artificial neural networks together. (b) 3 only
(c) 1 and 2 only
2. The primary aim of machine learning is to (d) 2 and 3 only
develop computer programs that access the
required data and utilize it for learning by 213. Which of the following statements about ‘light
themselves. amplification by the stimulated emission of
3. Machine learning plays a significant role in self- radiation’ (Laser) technology is incorrect?
driving cars. (a) Laser is a device that stimulates atoms or
molecules to emit light.
Which of the statements given above is/are correct?
(b) Helium-neon lasers were the first lasers with
(a) 1 and 2 only broad commercial applications.
(b) 2 and 3 only (c) Lasers do not occur in nature and have to be
(c) 1 and 3 only created artificially.
(d) 1, 2 and 3 (d) Lasers produce a narrow beam of light waves all
209. Consider the following statements: having very different wavelengths.
1. Sonar transmits electromagnetic waves to 214. With reference to communications technologies
analyse various attributes of the target. like 5G and 6G, consider the following statements:
2. LiDAR technology uses light pulses to determine 1. 5G has a new interface, which uses millimeter
the distance between the sensor and the object. wave spectrum.
3. RADAR technology uses sound waves to 2. Unlike 5G that works at high frequencies only,
determine the angle of inclination of objects on 6G is divided into three different bands- the low,
the surface. the mid and the high frequencies.

Workbook 116
.
SCIENCE & TECHNOLOGY

3. 6G seeks to utilize the terahertz band of 4. VLC has no electromagnetic interference.


frequency which is currently unutilized. Select the correct answer using the code given below:
Which of the above statements given above is/are (a) 1, 2 and 3 only
correct? (b) 1, 2 and 4 only
(a) 1 and 2 only (c) 1, 3 and 4 only
(b) 2 only
(d) 2, 3 and 4 only
(c) 1 and 3 only
(d) 3 only
2.6. Astronomy and Space
215. With reference to modern Communication
technologies, consider the following statements: 219. With reference to ISRO International cooperation,
1. Unlike a bar-code, Radio-frequency consider the following pairs:
identification (RFID) tags need not be in line of
sight to be read. Mission Name Participating Country
2. Unlike Near field communication (NFC) 1. SARAL France
technology, RFID devices can act not only as a 2. NISAR Japan
reader, but also as a tag.
3. Bluetooth Low Energy (BLE) devices remain in 3. Chandrayaan 1 USA
sleep mode unless a connection initiates. 4. TRISHNA Russia
Which of the above options/statements is/are Which of the following pairs given above are
correct? correctly matched?
(a) 1 and 2 only (a) 1 and 4 only
(b) 2 only (b) 1 and 3 only
(c) 1 and 3 only (c) 3 and 4 only
(d) 1, 2 and 3 (d) 2 and 4 only
216. With reference to communications technologies, 220. With reference to the Vikas engine of ISRO,
what is/are the differences between RADAR and
consider the following statements:
LiDAR?
1. It belongs to the family of liquid fueled rocket
1. Unlike LIDAR, RADAR can easily operate in
engines.
night and cloudy weather conditions.
2. To detect objects, LIDAR uses comparatively 2. It is used to power both Polar Satellite Launch
long wavelength than RADAR. Vehicle (PSLV) and Geosynchronous Launch
Vehicle (GSLV).
Which of the above statements is/are correct?
3. It has been indigenously designed and made
(a) 1 only with no foreign contribution.
(b) 2 only
(c) Both 1 and 2 Which of the statements given above is/are correct?
(d) Neither 1 nor 2 (a) 1 and 2 only
(b) 2 and 3 only
217. With reference to computed tomography (CT) scan (c) 1 and 3 only
and X-RAYS, consider the following statements: (d) 1, 2 and 3
1. Unlike chest X-ray, Chest CT scan delivers
3-dimension view of the chest. 221. It is a technology developed by Indian Institute of
2. CT scan uses more ionizing radiation than X-ray. Spices Research (IISR), where a combination of
selected beneficial microorganisms is compacted
Which of the above statements is/are correct? into a package to give effects of fertilizer. It is easy
(a) 1 only to storage, usage and transport. This technology is
(b) 2 only known as:
(c) Both 1 and 2 (a) Bio-synthesizer
(d) Neither 1 nor 2 (b) Pseudo-capsule
218. With reference to Visible Light Communication (c) Bio-capsules
(VLC) technology, which of the following (d) Bio-Scaffolder
statements are correct? [2020] 222. In the context of Opposition Event which was
1. VLC uses electromagnetic spectrum wavelengths recently in news, which of the following Celestial
375 to 780 nm. Event best describes it?
2. VLC is known as long-range optical wireless (a) Sun, Earth and an outer planet are lined up, with
communication. the Earth in the middle
3. VLC can transmit large amounts of data faster (b) New Moon comes between the Sun and Earth
than Bluetooth. and blocks out the Sun’s rays

117 Workbook
.
SCIENCE & TECHNOLOGY

(c) Earth comes between the Sun and the Full Moon (c) 1 and 3 only
and blocks the Sun’s direct rays from lighting up (d) 1, 2 and 3
the Moon
227. With reference to the New Space India Ltd (NSIL),
(d) The sun vertically shines above Equator.
consider the following statements:
223. These satellites are placed into orbit at a distance 1. It was incorporated under the Companies Act,
of around 35,800 km from the earth’s surface. 2013 as a wholly owned Government of India
They rotate in the same direction as the earth company.
and one revolution of such satellites is the same 2. Its primary responsibility includes enabling
as one day on earth. These satellites are used as Indian industries to take up high technology
communication satellites and for weather-based space related activities.
applications. Orbital path is circular. The satellite 3. It does not undertake any satellite or launch
type mentioned here is: vehicle building activities.
(a) Geosynchronous transfer Satellite Which of the statements given above are correct?
(b) Sun-synchronous Satellite (a) 1 and 2 only
(c) Geostationary Satellite (b) 2 and 3 only
(d) Polar Satellite (c) 1 and 3 only
224. Consider the following statements about India- (d) 1, 2 and 3
based Neutrino Observatory (INO): 228. With reference to the space stations, consider the
1. It aims to determine the mass of neutrinos. following statements:
2. It will be the first of its kind research facility in 1. Russia has decided to withdraw from the
India for the study of neutrinos. International Space Station by 2025.
3. The observatory has been recently shifted to 2. Tiangong is China’s own space station being
Ladakh. built in the low-Earth orbit.
Which of the statements given above is/are correct? 3. NASA has the final authority over the use of the
International Space Station.
(a) 1 only
(b) 1 and 2 only 4. India will be joining the International Space
(c) 2 and 3 only Station in 2024.
(d) 1 and 3 only Which of the statements given above is/are correct?
(a) 1 and 2 only
225. In the context of ‘Artificial Sun’, consider the (b) 1, 2 and 3 only
following statements: (c) 3 and 4 only
1. It is based on nuclear fission technology. (d) 2, 3 and 4 only
2. The device can potentially unlock a powerful
clean energy source. 229. Regarding the various space missions/satellites,
consider the following pairs:
3. The power is generated by applying powerful
magnetic field to a contained loop of hot plasma. Mission/ Initiative Purpose
Which of the statements given above is/are correct? 1. Arktika-M Exploration of the
(a) 1 only South Polar region of
(b) 1 and 2 only Moon
(c) 2 only
2. Sindhu Netra Identification of
(d) 2 and 3 only warships and merchant
226. Which of the following statements is/are correct ships in the Indian
about the PASIPHAE Project, recently seen in Ocean Region.
news? 3. Amazonia-1 Monitoring
1. It is a sky surveying project which aims to study deforestation in the
stars. Amazon region
2. It will use a novel polarimeter instrument Which of the pairs given above is/are correctly
known as WALOP (Wide Area Linear Optical matched?
Polarimeter). (a) 1 and 2 only
3. The project is solely implemented and funded by (b) 2 and 3 only
the European Space Agency. (c) 3 only
Select the correct answer using the code given below: (d) 1, 2 and 3
(a) 1 and 2 only 230. With reference to Sounding Rockets, consider the
(b) 2 and 3 only following statements:

Workbook 118
.
SCIENCE & TECHNOLOGY

1. It is a technology based on ultrasound where 235. Regarding the Lagrange points in space, consider
sound beams are used to hit the enemy area. the following statements:
2. Currently India does not have any operational 1. Lagrange Points are positions where the
Sounding Rocket. gravitational forces of a two-body system
Which of the statements given above is/are correct? produce enhanced regions of attraction and
(a) 1 only repulsion.
(b) 2 only 2. There are three Lagrange points in the Earth-
(c) Both 1 and 2 Sun system.
(d) Neither 1 nor 2 3. These points are used by spacecraft to reduce
fuel consumption needed to remain in position.
231. In the context of modern scientific research,
consider the following statements about the 4. L3 Lagrange point of the Earth-Sun system
‘Square Kilometre Array’: is ideal for solar observatories as it gets an
uninterrupted view of the sun.
1. It will be the world’s largest optical telescope
positioned in the Arctic region. Which of the statements given above is/are correct?
2. It will enable the scientists to study the origin (a) 1, 2 and 3 only
and evolution of galaxies. (b) 2 and 3 only
Which of the statements given above is/are correct? (c) 1 and 3 only
(d) 1, 2, 3 and 4
(a) 1 only
(b) 2 only 236. “This mission will observe far-infrared light, or
(c) Both 1 and 2 light with wavelengths much longer than what is
(d) Neither 1 nor 2 visible to the human eye at an altitude of about
232. There is a growing interest in the exploration 130,000 feet (40 km). The mission will try to find
of Moon’s South Polar Region. What is the answers about formation of giant stars in the
importance of this region of the Moon? galaxy. It will carry an instrument to measure the
motion and speed of gas around newly-formed
1. It is estimated to hold a large amount of water.
stars. During the flight, the mission will study four
2. It offers an opportunity to have a better main targets, including two star-forming regions
understanding of the solar system’s origin. in the Milky Way galaxy”.
3. Its regolith has traces of some of the essential
resources. The above paragraph refers to which of the following?
(a) ASTHROS
Select the correct answer using the code given below:
(b) Voyager
(a) 1 and 2 only (c) Dragonfly
(b) 2 and 3 only (d) LISA Pathfinder
(c) 3 only
(d) 1, 2 and 3 237. Which of the following statements correctly
describes the application of the Antarctic
233. “It is a Mars mission launched by the United Arab
Impulsive Transient Antenna (ANITA)?
Emirates. It is the first interplanetary mission for
the Arab World.” (a) To8 discover Earth-size planets in the Sun’s
habitable zone.
The above lines refer to which of the following? (b) To detect cosmic gravitational waves.
(a) Hope Probe (c) To detect ultra-high energy cosmic-ray
(b) Dragonfly neutrinos.
(c) Tianwen-1 (d) To measure the amount of ice in Antarctica’s ice
(d) Voyager sheets.
234. Consider the following statements regarding 238. Event Horizon, recently seen in news, refers to?
the Indian National Space Promotion and
(a) It is defined as the boundary of the black hole
Authorization Centre (IN-SPACe):
where the velocity needed to escape exceeds the
1. It will regulate the usage of ISRO facilities by the speed of light, which is the speed limit of the
private sector. cosmos.
2. It has been created under the direct control of (b) It is defined as the velocity of a spacecraft needed
the Prime Minister’s Office (PMO). to escape earth’s gravity.
Which of the statements given above is/are correct? (c) It is the outermost corona of the sun where
(a) 1 only periodic events of solar bursts occur.
(b) 2 only (d) It is the name of the recently launched NASA
(c) Both 1 and 2 spacecraft that will explore Jupiter’s moon called
(d) Neither 1 nor 2 Horizon.

119 Workbook
.
SCIENCE & TECHNOLOGY

239. It is a ripple in space-time caused by some of Satellite Application


the most violent and energetic processes in the
Universe. Albert Einstein predicted its existence 1. RISAT 2B Disaster Management
in 1916 in his general theory of relativity. It was, System
for the first time detected in the year 2017 during 2. CARTOSAT-3 Earth observation
the collision of two stars. 3. CMS-01 Planetary observation
What is being referred to in the above paragraph? Which of the pairs given above is/are correctly
(a) Dark energy matched?
(b) Dark matter (a) 1 and 2 only
(c) Gravitational waves (b) 2 only
(d) Event horizon (c) 1 and 3 only
(d) 1, 2 and 3
240. Consider the following statements, with reference
to different Earth orbits and its features: 244. With reference to the Indian Regional Navigational
1. Polar orbiting satellite senses only the polar Satellite System (IRNSS), consider the following
regions of Earth. statements:
2. Geostationary satellite captures the same view of 1. IRNSS will provide Standard Positioning Service
(SPS) only to the authorised government
Earth with each observation.
agencies.
3. Low Earth Orbit satellite always orbits along
2. All the satellites in the constellation will be
Earth’s equator. placed in a geostationary orbit.
Which of the statements given above is/are correct? 3. It is recognised by the International Maritime
(a) 1 only Organisation (IMO) as a part of the World Wide
(b) 2 only Radio Navigation System.
(c) 1 and 2 only Which of the statements given above is/are correct?
(d) 2 and 3 only (a) 1 and 2 only
241. With reference to the Hypersonic Technology (b) 2 only
(c) 3 only
Demonstrator Vehicle (HSTDV), consider the
(d) 1 and 3 only
following statements:
1. It uses indigenously developed scramjet 245. The Artemis program, sometimes seen in the
propulsion system. news, is:
2. It can be used for long-range cruise missiles as (a) A program for resource exploration in the Arctic
well as for launching satellites. region.
(b) A mission to land the first woman and next man
Which of the statements given above is/are correct? on the Moon.
(a) 1 only (c) A Multi-national initiative to reduce effects of
(b) 2 only climate change in the Antarctica region.
(c) Both 1 and 2 (d) A satellite internet constellation for providing
(d) Neither 1 nor 2 satellite Internet access.
242. Consider the following statements regarding 246. Match the following pairs of countries/union and
India’s Gaganyaan mission: their Global Navigation Satellite Systems:
1. It will put a manned spacecraft in a Medium Country/Union Navigation system
Earth Orbit (MEO) 2000 km above Earth’s
surface. 1. China A. Galileo
2. It will use GSLV Mk III as a launch vehicle. 2. Russia B. BeiDou
3. If successful India would become the 5th nation 3. Japan C. QZSS
in the world to have sent crews to space.
4. European Union D.GLONASS
Which of the statements given above is/are correct? Select the correct answer using the code given below:
(a) 1 and 2 only (a) 1-C, 2-D, 3-B, 4-A
(b) 2 only (b) 1-B, 2-A, 3-C, 4-D
(c) 2 and 3 only (c) 1-B, 2-D, 3-C, 4-A
(d) 1, 2 and 3 (d) 1-C, 2-A, 3-B, 4-D
243. Consider the following pairs of satellites and its 247. Consider the following statements, with reference
application: to India’s Shukrayaan mission:

Workbook 120
.
SCIENCE & TECHNOLOGY

1. The objective of the mission is to investigate the 2. It is useful to develop long range air-to-air
surface processes and the subsurface rock layers missiles.
of planet Venus. Which of the statements given above is/are correct?
2. The mission includes collaborative contributions (a) 1 only
from other countries. (b) 2 only
Which of the statements given above are correct? (c) Both 1 and 2
(d) Neither 1 nor 2
(a) 1 only
(b) 2 only 252. A ramjet is a variant of jet engine that are used in
(c) Both 1 and 2 space missions. What is its significance?
(d) Neither 1 nor 2 1. They work most efficiently at hypersonic speeds.
248. With reference to the Starlink Network Project, 2. They are mostly employed for movement in
sometimes seen in the news, which of the vacuum conditions.
statements given below is/are correct? 3. They can quickly move an aircraft from standstill
to very high acceleration.
1. It is a project by the European Space Agency to
provide space-based internet services. Select the correct answer using the code given below:
2. The project intends to put a constellation of (a) 1 and 3 only
thousands of satellites in Low Earth Orbit (LEO). (b) 2 and 3 only
(c) 1 only
3. The project aims to reduce the transmission (d) None of the Above
latency for faster internet services.
Select the correct answer using the code given below: 253. Which of the following are the areas of potential
application of air-breathing scramjet technology?
(a) 1 and 2 only
1. Hypersonic missiles
(b) 2 and 3 only
(c) 3 only 2. Air transportation
(d) 1 and 3 only 3. Satellite Launch
Select the correct answer using the codes given
249. With reference to Polar Satellite Launch Vehicle below.
(PSLV), which of the following statements is/are (a) 1 and 2 only
correct? (b) 1 and 3 only
1. It is a four stage Satellite Launch Vehicle. (c) 3 only
2. It can launch satellites to Sun-Synchronous Polar (d) 1, 2, and 3
Orbits only.
254. With reference to Gaganyaan Mission, consider
Select the correct answer using the codes given the following statements:
below. 1. Polar Satellite Launch Vehicle (PSLV) will be
(a) 1 only used to launch Gaganyaan.
(b) 2 only 2. The spacecraft will be placed in a low earth orbit
(c) Both 1 and 2 of 300-400 km.
(d) Neither 1 nor 2 3. ISRO will collaborate with French space agency
CNES in the field of space medicine.
250. With reference to Data Relay Satellite Systems
(DRSS), consider the following statements: Which of the statements given above is/are correct?
1. DRSS helps in tracking and communicating with (a) 1 and 3 only
other satellites. (b) 2 only
(c) 2 and 3 only
2. Indian DRSS satellites will be placed in (d) 1, 2 and 3
geostationary orbit.
3. India will be the first Asian country to have 255. ‘Amazonia-1’ recently seen in news, is related to:
DRSS. (a) It is a unique species of banana plant recently
discovered in the Amazon rainforests.
Which of the statements given above is/are correct?
(b) It is a distinct species of primitive human that
(a) 1 only lived during the Ediacaran period.
(b) 1 and 2 only (c) It is the first dedicated commercial mission of
(c) 2 and 3 only ISRO’s NewSpace India Limited.
(d) 1, 2 and 3 (d) It is the world’s first sovereign digital currency.
251. With reference to the Solid Fuel Ducted Ramjet 256. Which of the following statements is/are correct
(SFDR) Technology, consider the following: regarding to the Satellite Orbits?
1. It is a missile propulsion system based on solid 1. All Geostationary orbits are Geosynchronous,
fuelled air-breathing ramjet engine. while the vice versa is not true.

121 Workbook
.
SCIENCE & TECHNOLOGY

2. The Sun Synchronous Polar Orbit or the Polar 260. Consider the following statements:
orbit refers to a Low Earth Orbit (500- 2000km) Statement 1: Chandrayaan-2 has detected the
in the polar plane. unambiguous presence of hydroxyl and water
3. Satellites in the LEO have a lower orbital velocity molecules on the Moon.
than the ones in higher orbits.
Statement 2: The formation of water and hydroxyl
Select the correct answer using the code given below: occurs due to interaction of solar winds with the
(a) 1 and 2 only lunar surface.
(b) 1 and 3 only Which one of the following is correct in respect of
(c) 2 and 3 only the above statements?
(d) 1, 2 and 3
(a) Both Statement 1 and Statement 2 are correct
257. Which of the following statements in context of and Statement 2 is the correct explanation of
satellite launch vehicles in India is/are correct? Statement 1
1. Geosynchronous Satellite Launch Vehicle (b) Both Statement 1 and Statement 2 are correct
(GSLV) has the capability to put a heavier and Statement 2 is not the correct explanation
payload in the orbit as compared to the Polar of Statement
Satellite Launch Vehicle (PSLV). (c) Statement 1 is correct but Statement 2 is not
2. PSLVs are designed mainly to deliver earth correct
observation or remote sensing satellites. (d) Statement 1 is not correct but Statement 2 is
3. GSLVs are designed for launching mainly correct
communication satellites into higher Geo 261. Consider the following statements with reference
synchronous orbit. to Dark Matter, sometimes seen in news:
4. Both PSLV and GSLV are three-stage launch 1. Dark matter does not absorb, reflect or emit
vehicles. light.
Select the correct answer using the code given below: 2. Dark matter interacts with the rest of the
(a) 1, 2 and 3 only universe only through its gravity.
(b) 1 and 4 only 3. Dark matter is a category of antimatter.
(c) 2 and 3 only
(d) 1, 2, 3 and 4 Which of the statements given above is/are correct?
(a) 1 and 2 only
258. Consider the following statements: (b) 2 only
1. Dark matter does not interact with the (c) 1 and 3 only
electromagnetic forces including light. (d) 1, 2 and 3
2. Dark energy causes the universe to expand at an
accelerated rate. 2.7. Energy
3. Dark energy and Dark matter together constitute
less than 10% of the universe. 262. Which of the following statements is/are incorrect
about hydrogen energy?
Which of the statements given above is/are correct?
1. Hydrogen gas is extracted from water using the
(a) 1 only
(b) 1 and 2 only technique of electrolysis.
(c) 3 only 2. The efficiency of a hydrogen cell is less than that
(d) 1, 2 and 3 of an internal combustion engine.
3. Hydrogen energy does not emit any greenhouse
259. Which among the following are the reasons for gas into the environment.
satellites launches in India from the east coast?
4. Hydrogen fuel only produces electrical energy
1. The rotation of Earth from west to east gives which can be used as an energy source.
initial boost to rocket carrying satellite.
Select the answer using the code given below:
2. The low upper atmospheric pressure on the east
coast provides smooth launch to rocket. (a) 1 and 4 only
3. The lower orbital velocity of Earth on the east (b) 1, 2 and 3 only
coast requires low power from rockets. (c) 2, 3 and 4 only
(d) 1, 2, 3 and 4
4. In case of failure of the launch, the satellite does
not fall on the built-up hinterland. 263. With reference to Organic Photovoltaic (OPV)
Select the correct answer using the code given below: Cells, consider the following statements:
(a) 1 and 2 only 1. It is a type of solar cell where the absorbing layer
(b) 2 and 3 only is typically made of polymers.
(c) 3 and 4 only 2. Manufacturing cost is cheaper than the inorganic
(d) 1 and 4 only Photovoltaic cells.

Workbook 122
.
SCIENCE & TECHNOLOGY

3. Compared to inorganic photovoltaic cells, OPV 268. With reference to nuclear technology consider the
cell have low efficiency and low stability. following statements:
Which of the statements given above is/are correct? 1. Nuclear fuel like U-235 can be used directly to
(a) 1 and 2 only produce electricity.
(b) 2 and 3 only 2. Currently Nuclear power reactor produces
(c) 1 and 3 only energy through process of nuclear fusion only.
(d) 1, 2 and 3 3. Solid graphite can be used as a moderator in
264. With reference to Geothermal Energy consider the nuclear reactors.
following statements: 4. Nuclear technology can be used in medical
1. It is the energy that is generated from the heat diagnosis and treatment.
within the Earth. Which of the statements given above is/are correct?
2. Active Geothermal resources are usually (a) 1 and 2 only
concentrated along major plate boundaries. (b) 4 only
3. Puga and Surajkund are important sites for (c) 1 and 3 only
geothermal energy in India. (d) 3 and 4 only
Which of the statements given above are correct? 269. With reference to Lithium-Sulfur battery consider
(a) 1 and 2 only the following statements:
(b) 1 and 3 only 1. It is more energy efficient than lithium-ion
(c) 2 and 3 only battery.
(d) 1, 2 and 3
2. Sulfur electrode in Lithium-Sulfur battery break
265. ‘Global Energy demand is set to increase between after repeated charge cycles.
25-40% by 2040. It is believed world over there Which of the statements given above is/are correct?
is need to decarbonize the planet and need to be
(a) 1 only
driven by clean energies like Green Hydrogen.’
(b) 2 only
In the context of Green Hydrogen which of the (c) Both 1 and 2
statement given below is correct? (d) Neither 1 nor 2
(a) Hydrogen stored in fossil fuels.
(b) Hydrogen generated from carbon capture. 270. What are the advantages of floating solar energy
(c) Hydrogen generated from totally renewable plants?
resources. 1. Increased Efficiency of a Solar Panel
(d) Hydrogen that is produced biologically. 2. Lower cost of Installation
266. With reference to wind energy in India consider 3. Lower evaporation of water bodies.
the following statements: Select the correct statement using the code given
1. Wind is an intermittent and site-specific resource below:
of energy. (a) 1 and 2 only
2. Tamil Nadu has the highest wind energy (b) 2 and 3 only
potential in country. (c) 1 and 3 only
3. Offshore wind power plants can be built only till (d) 1, 2 and 3 only
the limit of 12 nautical miles from the shore. 271. Consider the following statements with reference
Which of the statements given above is/are correct? to the challenges in setting up floating solar power
(a) 1 only plants?
(b) 2 and 3 only 1. Construction costs for floating solar plants are
(c) 1 and 2 only usually higher than those of a ground-mounted
(d) 1, 2 and 3 solar farm.
267. It is a device that converts chemical energy to 2. Bodies of water exert a cooling effect, which
electrical energy by the action of microorganisms improves the performance of solar photovoltaic
and uses bacteria as the catalyst to oxidize organic panels of floating solar plants.
and inorganic matter to generate electric current. 3. Water contact may lead to degradation and
It has applications in various fields such as power corrosion of the plant which results in higher
generation systems, bio-recovery, and waste-water maintenance costs.
treatment. The device mentioned here is: Which of the statements given above are correct?
(a) Bio-electrification (a) 1 and 2 only
(b) Microbial Fuel cell (b) 2 and 3 only
(c) Bio-synthesis (c) 1 and 3 only
(d) Microbial- accumulation (d) 1, 2 and 3

123 Workbook
.
SCIENCE & TECHNOLOGY

272. Consider the following statements: combustion engines in manufacturing of electric


1. Lithium-Ion batteries are not rechargeable and vehicles.
have to be discarded after being exhausted. 3. Cost of manufacturing of these batteries is less
2. High voltages can sometimes cause the Lithium- than nickel based batteries.
ion batteries to burst and catch fire. Which of the above statements is/are incorrect?
Which of the statements given above is/ are correct? (a) 1 and 2 only
(a) 1 only (b) 2 only
(b) 2 only (c) 2 and 3 only
(c) Both 1 and 2 (d) 1, 2 and 3
(d) Neither 1 nor 2
277. To meet its rapidly growing energy demand,
273. What is/are the likely limitation(s) in promoting some opine that India should pursue research
Compressed Bio Gas (CBG) as a transport fuel in and development on thorium as the future fuel
India? of nuclear energy. In this context, what advantage
1. Unlike Compressed Natural Gas (CNG), CBG does thorium hold over uranium?
cannot be transported through pipelines. 1. Thorium is far more abundant in nature than
2. High up-front installation cost of CBG uranium.
production plants. 2. On the basis of per unit mass of mined mineral,
Select the correct answer using the code given below: thorium can generate more energy compared to
natural uranium.
(a) 1 only
(b) 2 only 3. Thorium produces less harmful waste compared
(c) Both 1 and 2 to uranium.
(d) Neither 1 nor 2 Which of the statements given above is/are correct?
274. Consider the following statements regarding (a) 1 only
Winter diesel: (b) 2 and 3 only
(c) 1 and 3 only
1. It contains additives to maintain lower viscosity. (d) 1, 2 and 3
2. It meets the Bureau of Indian Standards (BIS)
specifications for BS VI grade. 278. Which of the following are the methods used for
3. It has lower Sulphur content, which may lead to Waste to energy conversion?
better performance of engines. 1. Torrefaction
Which of the statements given above are correct? 2. Polycrack
(a) 1 and 2 only 3. Soil Liquefaction
(b) 2 and 3 only 4. Pulverization
(c) 1 and 3 only Select the correct answer using the code given below.
(d) 1, 2 and 3 (a) 1 and 2 only
275. Consider the following pairs regarding the (b) 2, 3 and 4 only
proposed/under construction nuclear power (c) 4 only
plants in India and their location: (d) 1 and 3 only

Nuclear power plant Location 279. Which of the following statements correctly
defines Chemosynthesis?
1. Chutka Madhya Pradesh (a) It is the process by which certain microbes create
2. Gorakhpur Haryana energy by mediating chemical reactions.
(b) It is defined as directional cell movement of cells
3. Kovvada Karnataka
towards concentration gradients of solubilized
Which of the above pairs is/are correctly matched? attractants.
(a) 1 only (c) It is defined as random cell movement in the
(b) 1 and 2 only absence of chemoattractant gradients.
(c) 2 and 3 only (d) It is a drug treatment that uses powerful
(d) 1 and 3 only chemicals to kill fast-growing cells in your body.
276. Consider the following statements with respect to 280. In recent times, hydrogen has been increasingly
Lithium –ion (Li-ion) batteries: mooted as an alternate source of energy. What is/
1. These are rechargeable batteries with very high are the advantage/advantages of using hydrogen as
energy density. fuel?
2. At their present stage of development, these 1. Hydrogen can be generated from renewable
batteries are a viable alternative to internal energy resources.

Workbook 124
.
SCIENCE & TECHNOLOGY

2. As a fuel, it is more efficient than petrol. 284. Which of the following statements is/are correct
3. The process of extracting hydrogen is highly about the compressed natural gas (CNG)?
energy efficient. 1. It is stored in a liquid state in high-pressure
4. Use of hydrogen can lead to zero vehicular storage cylinders.
emissions. 2. The burning of CNG produces gases containing
Select the correct answer using the code given below: carbon nanoparticles.
(a) 1, 2 and 4 only 3. It is used for manufacturing of chemicals and
(b) 1 and 4 only fertilisers.
(c) 2 and 4 only Select the correct answer using the code given below.
(d) 1, 2, 3 and 4 (a) 1 only
(b) 1 and 3 only
281. Consider the following statements with reference
(c) 2 and 3 only
to the Fast Breeser Reactors:
(d) 1, 2 and 3
1. Fast Breeder reactors produce more fissionable
material than they consume to generate energy.
2.8. Defence Technology
2. Electricity generated by Fast Breeder Reactors
can be a good source of green energy. 285. Which one of the following is the best description
3. Currently two fast breeder reactors are operating of ‘Muntra’, that was in news recently?
in India. (a) Unmanned tank developed by DRDO.
Which of the statements given above is/are correct? (b) Scheme for long term loans to Self Help Groups.
(c) Medicinal plant helpful in increasing immunity.
(a) 1 and 2 only
(d) Military exercise between India and UAE.
(b) 2 and 3 only
(c) 1 and 3 only 286. With reference to Microwave Weapons, consider
(d) 1, 2 and 3 the following statements:
282. With reference to the difference between the 1. They are direct energy weapons which aims a
Aluminium-Air batteries and Lithium-ion highly focused energy at a target.
batteries, consider the following statements: 2. It uses electromagnetic radiation to heat the
1. Aluminium-Air batteries have a higher energy water in a human target’s skin.
density than Lithium-ion batteries. Which of the statements given above is/are correct?
2. Aluminium-Air batteries cannot be recharged (a) 1 only
like Lithium-ion batteries. (b) 2 only
3. Aluminium-Air batteries may be cheaper in cost (c) Both 1 and 2
than Lithium-ion batteries. (d) Neither 1 nor 2
Which of the statements given above is/are correct? 287. Which of the following features of Tejas defense
(a) 1 and 3 only Aircraft is not true?
(b) 2 only (a) It is being manufactured by Hindustan
(c) 1 and 2 only Aeronautics limited.
(d) 1, 2 and 3 (b) It is a fully weaponized light fighter with a single
engine.
283. Consider the following statements with respect to (c) It is an ultrasonic aircraft being developed in
the difference between Pressurised Heavy Water India.
Reactors (PHWRs) and Fast Breeder Reactors (d) It has capabilities of air to air and air to ground
(FBRs): attacks.
1. While PHWR uses natural uranium as fuel, a
fast breeder reactor uses thorium to produce 288. Defence Research and Development Organisation
electricity. (DRDO) has developed an Advanced Chaff
Technology. Which one of the following is the
2. While PHWRs have a neutron moderator, it is
correct usage of this technology?
not present in FBR.
(a) It is used in detection of enemy’s camp at high
3. Unlike FBR, PHWRs produce less radioactive
terrain above 12000 ft.
waste as they use heavy water as coolant.
(b) It is used to protect naval ships from enemy’s
Which of the statements given above is/are correct? radar and Radio Frequency (RF) missile seekers.
(a) 1 and 3 only (c) It is used in cryogenic engine in order to boost
(b) 2 only the efficiency of the Launch Vehicle.
(c) 2 and 3 only (d) It is used in the development of cruise missiles to
(d) 1, 2 and 3 increase their accuracy and precision.

125 Workbook
.
SCIENCE & TECHNOLOGY

289. In the context of Indian defence, what is 294. With reference to the present version of
‘Rudram-I’? BRAHMOS Missile, which of the following
1. It is India’s first indigenous anti-radiation missile. statements is/are correct?
2. It is a supersonic surface-to-air missile with a 1. It is a medium range supersonic cruise missile
range of 30 km. 2. It has a flight range of 1000 Km
Which of the statements given above is/are correct? 3. Brahmos missile has sea and air-launched
(a) 1 only versions only
(b) 2 only Select the correct answer using the codes given
(c) Both 1 and 2 below.
(d) Neither 1 nor 2 (a) 1 only
290. In the context of Indian defence, what is (b) 1 and 3 only
‘SAHAYAK-NG’? (c) 3 only
(a) A subsonic cruise missile. (d) 1, 2, and 3
(b) An indigenously developed radar system. 295. With reference to Defense System of India,
(c) A GPS aided air droppable container. consider the following statements:
(d) An Offshore Patrol Vessel. 1. Ballistic missiles are powered initially by a rocket
291. Consider the following statements: or series of rockets in stages.
1. Project 17A involves manufacturing of 2. Unlike Cruise missiles, ballistic missiles can
indigenous fighter aircrafts for the Indian air carry nuclear warheads.
force. 3. S-400 is an anti-missile defence system which
2. All materials required under Project 17A are to can engage both cruise missiles and ballistic
be procured indigenously. missiles.
3. Project 75 includes building six scorpene-class Which of the statements above is/are correct?
attack submarines for the Indian Navy. (a) 1 and 2 only
Which of the statements given above is/are correct? (b) 2 and 3 only
(a) 1 and 2 only (c) 1 and 3 only
(b) 2 only (d) 1, 2 and 3
(c) 3 only 296. Hyperspectral imaging is being used regularly
(d) 1 and 3 only in modern day machines. Which among the
292. Consider the following pairs: following are applications of this technique?
1. Detecting freshness of meats and fruits
Ship/Submarine Function 2. Remote Sensing
1. Himgiri State-of-the-art guided 3. Tracking pollution levels
missile frigate 4. Early diagnose of cancer
2. Imphal A guided missile destroyer Select the correct answer using the code given below:
3. Khanderi Scorpene-class submarine (a) 1 and 4 only
under Project 75 (b) 2 and 3 only
(c) 1, 3 and 4 only
Which of the pairs given above is/are correctly
matched? (d) 1, 2, 3 and 4
(a) 1 only
(b) 3 only 2.9. New Technology and Miscellaneous
(c) 2 and 3 only
297. With reference to Torrefaction Technology
(d) 1, 2 and 3
consider the following statements:
293. In the context of Mission Shakti, which of the 1. It involves the heating of biomass in the absence
statements given below is/are correct? of oxygen.
1. It demonstrated India’s capability to interdict 2. Weight is lost and energy is gained in Torrefaction
and intercept a satellite in outer space. compared to original biomass.
2. India is only the fourth nation to achieve this 3. It is an indigenous technology developed by
feat after USA, Russia and China. ICAR.
Select the correct answer using the code given below: Which of the statements given above is/are correct?
(a) 1 only (a) 1 only
(b) 2 only (b) 2 and 3 only
(c) Both 1 and 2 (c) 1 and 3 only
(d) Neither 1 nor 2 (d) 1, 2 and 3

Workbook 126
.
SCIENCE & TECHNOLOGY

298. The term ‘India Stack’ is often seen in news, which 302. With reference to the Large Hadron Collider
of the statement given below best describes the beauty (LHCb) experiment, consider the following
term? statements:
(a) A set of APIs that allows different organization 1. It aims to investigate the differences between
to utilize the unique digital infrastructure to matter and antimatter.
solve India’s hard problems. 2. The experiment uses a single enclosed particle
detector instead of a series of subdetectors.
(b) It is a group of IT professionals dedicated to face
the challenges posed by different malware and Which of the statements given above is/are correct?
spywares to India’s critical Infrastructure. (a) 1 only
(c) It is a cloud-based platform where any individual (b) 2 only
(c) Both 1 and 2
can download open-source software for their
(d) Neither 1 nor 2
personal use.
(d) It is an initiative by Union Government to make 303. In the context of modern scientific research,
youth aware with new IT technologies and consider the following statements about
related threats. ‘Nanomicelles’, which was recently in the news:
1. Being extremely small, they are not stable at
299. What are the advantages of POLYCRACK room temperature.
technology over conventional approach of treating 2. They can be used as a carrier for drug delivery.
solid waste? 3. They have a potential application in cancer
1. Pre-segregation of waste is not required. treatment.
2. It is a zero-discharge process. Which of the statements given above is/are correct?
3. The foot print of the plant is small hence the area (a) 2 only
required for installing the plant is less. (b) 2 and 3 only
4. It has high tolerance to moisture hence drying of (c) 1 and 3 only
(d) 1, 2 and 3
waste is not required.
Select the correct statement using the code given 304. Nano technology has applications in which of the
below: following?
1. Treatment of cancer
(a) 1, 2 and 3 only
2. Detection of food borne pathogens
(b) 1, 3 and 4 only
3. Quantum dots
(c) 1, 2 and 4 only
(d) 1, 2, 3 and 4 only 4. Detection of humidity and soil nutrient levels
Select the correct answer using the code given below:
300. Which of the following is/are an application of (a) 1 and 2 only
Geospatial technology? (b) 2 and 3 only
(a) Help in management of logistics by tracking (c) 1, 3 and 4 only
goods and ensuring their safety, (d) 1, 2, 3 and 4
(b) Used for predicting plant yield and their nutritive 305. The term “Hyperloop” commonly seen in news
value. refers to
(c) In wildlife conservation projects by helping to (a) High speed buses running on Hydrogen gas.
prevent and address calamities. (b) High-speed transportation system where a pod-
(d) All of the above like vehicle is propelled through low- Pressure
tubes.
301. “Khadi Prakritik Paint” is an innovative new (c) Low pressure particle collider developed by
paint developed by Khadi and Village Industries CERN.
Commission. Which of the following statements (d) Elevated corridors built with rail lines where
is/are correct regarding this paint? freight trucks can be placed which would then
1. It is a first-of-its-kind product completely made move on rails at high speed.
from jute extracts. 306. Which one of the following emerging technology
2. It is free from heavy metals like lead and mercury. comprises of a network of tubes through which
3. It has antifungal and antibacterial properties. a pod may travel free of air resistance or friction
conveying people or objects from one point to
Select the correct answer using the code given below: another?
(a) 1 and 3 only (a) Driverless Autonomous Cars
(b) 2 and 3 only (b) Tracked Electric Vehicle Project
(c) 1 and 2 only (c) Hyperloop
(d) 1, 2 and 3 (d) Flying hotel pods

127 Workbook
.
SCIENCE & TECHNOLOGY

307. Which of the following are applications of 311. Consider the following pairs:
“Photonics”?
Terms recently seen Context/Topic
1. Disease Detection.
in news
2. Energy-saving lighting.
3. Identifying structural defects in buildings. 1. BlackRock Fossil discovery
4. Precision manufacturing. 2. Zolgensma Digital currency
Select the correct answer using the code given below: 3. Moushik Microprocessor
(a) 1, 2, 3, and 4 Which of the pairs given above is/are correctly
(b) 2, 3 and 4 only matched?
(c) 1 and 4 only (a) 1 and 2 only
(d) 1, 3 and 4 only (b) 2 only
(c) 3 only
308. Consider the following statements about Indian
(d) 1 and 3 only
Nanoelectronics User Program
1. Under the project, Centre of Excellence in 312. How is the Science and Engineering Research
Nanoelectronics’ has been created at all IIT’s. Board (SERB) different from the Technology
2. The major focus is research and training of in- Development Board (TDB)?
house researchers and student. 1. The SERB has been established by an act of
Parliament whereas the TDB has been created
3. The program is funded by Department of
by an executive order of the Union Government.
Science and Technology.
2. The SERB functions under the Ministry of
Which of the statements given above is/are incorrect? Science and Technology whereas the TDB
(a) 1 only functions under the Ministry of Electronics and
(b) 1 and 3 only Information Technology.
(c) 2 only 3. The SERB aims to promote basic research in
(d) 2 and 3 only science and engineering whereas the TDB aims
to promote commercialisation of indigenous
309. With reference to ‘Natural language Processing technology and adaptation of imported
(NLP)’, consider the following statements: technology.
1. It deals with the interaction between computers
Select the correct answer using the code given below:
and humans using the natural language.
(a) 1 only
2. It uses algorithms to identify and extract the (b) 3 only
natural language rules and convert into a form (c) 2 and 3 only
that computers can understand. (d) 1, 2 and 3
3. NLP cannot be used to analyze the sentiments
and moods of the customer. 313. Which of the following is/are the characteristic/
characteristics of the ‘4D printing’ technology?
Which of the following statements given above is/are 1. 4D printing uses the same techniques as 3D
correct? printing to create products.
(a) 1 only 2. Unlike 3D printed objects, 4D printed objects
(b) 1 and 2 only can change their shape over time.
(c) 2 and 3 only 3. 4D printed objects can adapt according to some
(d) 1, 2 and 3 external environmental factors.
310. Consider the following statements regarding Select the correct answer using the code given below:
Gelatin Sticks: (a) 1 and 3 only
1. Gelatin sticks cannot be used without a (b) 2 and 3 only
detonator. (c) 1 only
2. Its manufacture is regulated by Petroleum and (d) 1, 2 and 3
Explosives Safety Organization (PESO). 314. Which of the following statements is/are correct
3. Prior license is required to sell gelatin sticks but regarding to the Geo-fencing?
not to buy it. 1. It is a location-based service.
Which of the statements given above is/are correct? 2. It can be used for marine and terrestrial
(a) 1 only navigation.
(b) 1 and 2 only 3. It can be used to draw virtual zones around work
(c) 2 and 3 only areas.
(d) 1, 2 and 3 Select the correct answer using the code given below:

Workbook 128
.
SCIENCE & TECHNOLOGY

(a) 1 and 3 only 3. Manufacturing of perfume


(b) 2 and 3 only 4. Production of Reclaimed Rubber
(c) 1 only Select the correct answer using the code given below.
(d) 1, 2 and 3
(a) 1 and 4 only
315. Which of the following applications is/are based (b) 1, 2 and 3 only
on geofencing? (c) 2, 3 and 4 only
1. Quarantine alert (d) 1, 2, 3 and 4
2. Location tracking 317. Consider the following statements regarding
3. Assets monitoring various applications of isotopes.
4. Social media ads 1. It is used in food irradiation to kill the germs.
5. Security 2. It helps in determining the thickness of plastic
or metal.
Select the correct answer using the code given below:
3. It can be used to trace and discover any leaks
(a) 1 and 2 only
present in underground pipes.
(b) 1, 2 and 5 only
(c) 2, 3, 4 and 5 only 4. It can be used to generate electrical power in
spacecraft.
(d) 1, 2, 3, 4 and 5
Which of the statements given above is/are correct?
316. Which of the following are the possible (a) 1, 2 and 3 only
applications of Coal tar? (b) 2 and 4 only
1. Manufacturing of naphthalene balls (c) 1 and 4 only
2. Treatment of skin (d) 1, 2, 3 and 4

129 Workbook
.
SCIENCE & TECHNOLOGY

SOLUTIONS

6. Solution (a)
2.1. Previous Years Questions
Exp) Option a is correct.
1. Solution (a) Bt brinjal was developed by Maharashtra Hybrid Seed Co.
Exp) Option a is correct. The Bt brinjal has been developed to give resistance
Bluetooth uses short-wavelength radio waves of a against lepidopteron insects. It is banned in India.
frequency range between 2.4 and 2.485 GHz. Bluetooth
enables one to create a personal area network wherein 7. Solution (d)
multiple devices talk to each other wirelessly via Exp) Option d is correct.
Bluetooth—a typical usage is home control automation Trans fatty acids (TFAs) or Trans fats are the most harmful
systems. type of fats which can have much more adverse effects on
Wi-Fi can use 2.4 GHz or 5 GHz frequency band - A 2.4 our body than any other dietary constituent.
GHz connection travels farther at lower speeds, while 5 Artificial TFAs are formed when hydrogen is made to
GHz frequencies provide faster speeds at shorter range. react with the oil to produce fats resembling pure ghee/
butter. In our diet the major sources of artificial TFAs
2. Solution (b)
are the partially hydrogenated vegetable oils (PHVO)/
Exp) Option b is correct. vanaspati/ margarine while the natural TFAs are present
Virtual private network, or VPN, is an encrypted in meats and dairy products, though in small amounts
connection over the Internet from a device to a network.
The encrypted connection helps ensure that sensitive data 8. Solution (c)
is safely transmitted. It prevents unauthorized people Exp) Option c is correct.
from eavesdropping on the traffic and allows the user to Free radicals are atoms that contain an unpaired electron.
conduct work remotely. Due to this lack of a stable number of outer shell electrons,
they are in a constant search to bind with another electron
3. Solution (b)
to stabilize themselves—a process that can cause damage
Exp) Option b is correct. to DNA and other parts of human cells. Antioxidants
The disc thickness of BD is 1.2 mm which is same as neutralize the free radicals produced in the body during
DVDs and CDs. metabolism.

4. Solution (a) 9. Solution (d)


Exp) Option a is correct. Exp) Option d is correct.
When the bark of a tree is removed in a circular fashion Aspartame is several times sweeter than table sugar, hence
all around near its base, it gradually dries up and dies - the food items made with small quantities of aspartame yield
phloem layer of tissue just below the bark is responsible fewer calories on oxidation.
for carrying food produced in the leaves by photosynthesis 10. Solution (a)
to the roots. Without food, the roots ultimately die and
stop sending water and minerals to the leaves. There is Exp) Option a is correct.
no water supply to the leaves, hence no photosynthesis. Geostationary orbits is a circular orbit at approximately
Therefore plant has no energy and it dies. 36,000km from the Earth’s equator. A spacecraft in this
orbit appears to an observer on Earth to be stationary in
5. Solution (c) the sky. This particular orbit is used for meteorological
Exp) Option c is correct. and communications satellites. The geostationary orbit is
DNA sequencing is a laboratory technique used to deter- a special case of the geosynchronous orbit, which is any
orbit with a period equal to Earth’s rotation period.
mine the exact sequence of bases (A, C, G, and T) in a
DNA molecule. The DNA base sequence carries the in- 11. Solution (d)
formation a cell needs to assemble protein and RNA mol-
Exp) Option d is correct.
ecules. DNA sequence information is important to scien-
tists investigating the functions of genes. The technology Satellites don’t fall from the sky because they are orbiting
of DNA sequencing was made faster and less expensive as Earth. Even when satellites are thousands of miles away,
a part of the Human Genome Project. Earth’s gravity still attracts them. Gravity, combined
with the satellite’s momentum from its launch into space
It is not yet possible to understand the causes of all human - cause the satellite go into orbit above Earth, instead of
diseases. falling back down to the ground.

Workbook 130
.
SCIENCE & TECHNOLOGY

12. Solution (d) in the earth’s continental crust is generally estimated to be


Exp) Option d is correct. three to five times that of uranium
The ionized part of the Earth’s atmosphere is known as the It is estimated that one ton of thorium can produce as
ionosphere. Ultraviolet light from the sun collides with much energy as 35 tons of uranium in a liquid fluoride
thorium reactor.
atoms in this region knocking electrons loose.
All of the above are advantages of Thorium over Uranium.
In particular the ionosphere is widely known for affecting
signals on the short-wave radio bands where it “reflects” 18. Solution (a)
signals enabling these radio communications signals to be
Exp) Option a is correct.
heard over vast distances.
Movement of asteroids in space and occurrence of
13. Solution (a) supernova explosions in space code are not evidences of
Exp) Option a is correct. continued expansion of universe.
Moderator is required in a Nuclear reactor to slow down A redshift reveals how an object in space (star/planet/
the neutrons produced during the fission reaction so that galaxy) is moving compared to us. It lets astronomers
the chain reaction can be sustained. Heavy Water is an measure a distance for the most distant (and therefore
excellent moderator due to its high moderating ratio and oldest) objects in our universe. Studying light from
low absorption cross section for neutrons. galaxies throughout our universe, astronomers have
noticed something surprising: almost all of it is redshifted.
14. Solution (c) In fact, not only is it redshifted, galaxies that are farther
Exp) Option c is correct. away are more redshifted than closer ones. So it seems
Microbial fuel cell (MFC) is a bio-electrochemical system that not only are all the galaxies in the universe moving
that can convert chemical energy to electrical energy away from us, the farther ones are moving away from us
through microbial catalysis at an electrode. Pollutants in the fastest. This is an evidence of continued expansion of
the wastewater, containing carbon, nitrogen, phosphorus, universe.
or heavy metals, can be degraded/stabilized in the The cosmic microwave background (CMB) is thought to
chambers of MFC. be leftover radiation from the Big Bang, or the time when
the universe began. According to Big Bang theory, when
15. Solution (c) the universe was born it underwent a rapid inflation and
Exp) Option c is correct. expansion, and continues to expand even today.
Graphene is a one-atom-thick layer of carbon atoms (not 19. Solution (a)
silica) arranged in a hexagonal lattice. It is the building-
block of Graphite. Graphene is the thinnest compound Exp) Option a is correct.
known to man at one atom thick, the lightest material Ultraviolet water purification is the most effective method
known, the strongest compound discovered (between for disinfecting bacteria from the water. Ultraviolet (UV)
100-300 times stronger than steel). rays penetrate harmful pathogens in water and destroy
In the field of optics, transparency is the physical illness-causing microorganisms by attacking their genetic
property of allowing light to pass through the material core (DNA).
without appreciable scattering of light. 20. Solution (b)
Graphene has high optical transparency and can uniformly Exp) Option b is correct.
absorb light across the visible and near-infrared parts of
the spectrum and thus it is potentially suitabile for use in A stem cell is a cell with the unique ability to develop into
spin transport. specialised cell types in the body. This can range from
muscle cells to brain cells. In some cases, they can also fix
16. Solution (c) damaged tissues.
Exp) Option c is correct. Stem cells can be derived from all multicellular organisms
and not just mammals.
LEDs were first introduced to the market in 1962.
They work by the movement of electrons through a 21. Solution (c)
semiconductor material.
Exp) Option c is correct.
With a lifespan of approximately 25,000–35,000 hours, an Genetically modified organisms (GMOs) can be defined
LED bulb lasts 2 to 4 times longer than a CFL, and 25 to 35 as organisms (i.e. plants, animals or microorganisms) in
times longer than a standard incandescent bulb. which the genetic material (DNA) has been altered in a
17. Solution (d) way that does not occur naturally by mating and/or natural
recombination. The technology is often called “modern
Exp) Option d is correct. biotechnology” or “gene technology”, sometimes also
Thorium (Th) is radioactive chemical element of the “recombinant DNA technology” or “genetic engineering”.
actinoid series of the periodic table. Thorium abundance It allows selected individual genes to be transferred from

131 Workbook
.
SCIENCE & TECHNOLOGY

one organism into another, also between nonrelated There is strong evidence for the transmission of hepatitis B
species. Foods produced from or using GM organisms are virus (HBV) infection, hepatitis C virus (HCV) infection,
often referred to as GM foods. and syphilis by tattooing. Tattooing may also transmit the
So far, scientists have been unable to create GMO which human immunodeficiency virus (HIV).
can grow and do photosynthesis in spaceships and space
27. Solution (c)
stations.
Exp) Option c is correct.
22. Solution (c)
An optical illusion is a visual stimuli that is perceived by
Exp) Option c is correct. the eyes and then comprehended by the brain in a way
Bt brinjal has been created by inserting a crystal that is different from reality.
protein gene (Cry1Ac) from the soil bacterium Bacillus Most of the stars are shining with a steady light. The
thuringiensis into the genome of brinjal. movement of air (sometimes called turbulence) in the
Bt brinjal does not produce terminator seeds. atmosphere of Earth causes the starlight to get slightly
bent as it travels from the distant star through the
23. Solution (d)
atmosphere down to us on the ground. The twinkling of
Exp) Option d is correct. a star is due to atmospheric refraction of starlight. The
Genetic engineering (or recombinant DNA technology) starlight, on entering the earth’s atmosphere, undergoes
allows selected individual gene sequences to be transferred refraction continuously before it reaches the earth. The
from an organism into another and also between non- atmospheric refraction occurs in a medium of gradually
related species. changing refractive index.
The organisms derived from genetic engineering are This means that some of the light reaches us directly and
termed genetically modified organisms (GMOs). A some gets bent slightly away. To our eyes, this makes the
transgenic food is defined as a food that is derived from
star seem to twinkle.
or contains GMOs.
28. Solution (a)
24. Solution (d)
Exp) Option a is correct.
Exp) Option d is correct.
Agni -IV is a surface-to-surface missile that uses a two-
Mycorrhiza are symbiotic associations between plant
stage rocket engine powered by solid propellant with
roots and certain soil fungi which play a key role in
nutrient cycling in the ecosystem and also protect plants a range of 3500-4000 km.
against environmental and cultural stress.
29. Solution: (a)
Arbuscular Mycorrhizal Fungi biotechnology significantly
improve soil attributes, increase above and below ground Exp) Option a is correct.
biodiversity, significantly improve tree/shrub seedlings Night blindness is caused by Vitamin A deficiency.
survival, growth and establishment on moisture and Vitamin E deficiency can cause nerve and muscle
nutrient stressed soils. Arbuscular mycorrhizal fungi damage, vision problems and a weakened immune
(AMF) biotechnology have also been shown to drive plant system.
succession and may prevent invasion by alien species.
Scurvy is caused by not having enough vitamin C in your
25. Solution (a) diet for at least 3 months.
Exp) Option a is correct. The most common cause of rickets is a lack of vitamin D
Viruses lack metabolic enzymes and thus they cannot or calcium in a child’s diet.
generate their own energy. They cannot be cultured in 30. Solution (b)
any synthetic medium as they require a growth medium
Exp) Option b is correct.
containing living cell.
A mechanical vector carries a pathogen on its body from All of the above are infectious diseases, but only Smallpox
one host to another, not as an infection. A biological was eradicated from India, in 1979.
vector carries a pathogen from one host to another after 31. Solution (a)
becoming infected itself. Thus, Viruses are transmitted
from one organism to another not only by biological Exp) Option a is correct.
vectors but also by mechanical vectors. Arachnids are a class of joint-legged invertebrates, with
eight legs.
26. Solution (b)
Mite, Scorpion and Spider are Arachnids, whereas Crab
Exp) Option b is correct. is a Crustacean.
Several infectious diseases have been found to be
associated with tattooing, including some transfusion 32. Solution (d)
transmitted diseases (TTDs). Exp) Option d is correct.

Workbook 132
.
SCIENCE & TECHNOLOGY

Biometrics are physical or behavioral human 39. Solution (d)


characteristics to that can be used to digitally identify a Exp) Option d is correct.
person to grant access to systems, devices or data.
The biological degradation of human waste is done by
Facial recognition, retinal scans, palm vein recognition, Anaerobic Bacteria inoculums.
ear recognition, Voice, Signature and DNA among others
Bio-toilets disposes human waste in 100% eco-friendly
33. Solution (b) manner and generates colour
Exp) Option b is correct. less, odourless water and inflammable Methane-Gas for
Launched in 1997, Cassini-Huygens was a joint endeavour cooking.
of ESA, NASA and ASI, the Italian space agency.
40. Solution (a)
It was the first space probe to orbit Saturn.
Exp) Option a is correct.
34. Solution (a) Fly ash is one of the residues generated by coal combustion.
Exp) Option a is correct. In India, fly ash bricks are used for construction.
Photovoltaic cells generate direct current (DC) electricity. Fly ash has pozzolanic properties, which make it a suitable
Whereas, Solar Thermal Technology uses the Sun’s rays replacement for Portland cement used in concrete.
to generate heat which is then used to run a turbine and Fly ash contains traces of heavy metals and other metals
provide AC. that are known to be detrimental to human health.
India has manufacturing base for both Solar Thermal
technology and photovoltaic cells. 41. Solution (d)
Exp) Option d is correct.
35. Solution (d)
All the statements are correct.
Exp) Option d is correct.
Remote sensing data provides essential information that
All of the above are application of maize crop. helps in monitoring various applications such as image
36. Solution (a) fusion, change detection and land cover classification.
Exp) Option a is correct. Remote sensing is a key technique used to obtain
information related to the earth’s resources and
A fuel cell is a device that converts the chemical energy
from a fuel into electricity through a chemical reaction of environment.
positively charged hydrogen ions with oxygen or another Cartosat-1 & 2, Resourcesat-1 & 2, Oceansat-1 & 2,
oxidizing agent. When pure hydrogen is used as the fuel, Risat-1, Megha-Tropiques, SARAL, Scatsat and INSAT
the only by-products generated are pure water and heat. series are some of the Indian Remote Sensing Satellites
Fuel cells produce electricity in form of Direct Current (IRS).
(DC) and can power laptops, cell phones, cars etc.
42. Solution (d)
37. Solution (c) Exp) Option d is correct.
Exp) Option c is correct. IndARC is India’s first underwater moored observatory in
Nanotechnology provides for Targeted drug delivery the Kongsfjorden fjord, half way between Norway and the
applications through the use of designed nanomaterials North Pole in the Arctic region.
as well as forming delivery systems from nanoscale It is expected to help scientists understand the Arctic
molecules such as liposomes. climate process and its influence on the Indian monsoon
Polymer nanoparticles (PNPs) like all nanoparticles are system.
capable of regaining their size once inside the cell through
43. Solution (d)
the physiological change in pH. They are also great
candidates for gene therapy, because they are highly stable Exp) Option d is correct.
and they offer controlled release of active ingredients. IceCube is the world’s largest neutrino detector,
encompassing a cubic kilometer of ice. The IceCube
38. Solution (c) Neutrino Observatory is the first detector of its kind and
Exp) Option c is correct. is designed to observe the cosmos from deep within the
NFC is the set of protocols that enable electronic devices South Pole ice.
to establish radio communication with each other by The IceCube telescope is a powerful tool to search for dark
touching the devices together or bringing them into matter and could reveal the physical processes associated
proximity to a distance of typically 10 cm or less, an with the enigmatic origin of the highest energy particles
antenna can be used to extend the range to 20 cm. NFC in nature. In addition, exploring the background of
often establish a secure channel and use encryption when neutrinos produced in the atmosphere, IceCube studies
sending sensitive information. the neutrinos themselves.

133 Workbook
.
SCIENCE & TECHNOLOGY

44. Solution (a) Li-Fi, short for Light Fidelity, uses the frequencies
Exp) Option a is correct. generated by LED bulbs, which flicker on and off to beam
information through the air.
GL-10 is a battery-powered 10-engine remotely piloted
plane similar as Unmanned Aerial Vehicle (UAV). It is Li-Fi allows speeds that are 100 times faster than Wi-Fi
designed and developed by NASA in such a way that it which uses radio waves to transmit data.
can take off like a helicopter and fly like an airplane. 50. Solution (d)
45. Solution (d) Exp) Option d is correct.
Exp) Option d is correct. The aim of the International Thermonuclear Experimental
Astrosat is India’s first dedicated multi wavelength space Reactor (ITER) project, which began in 2006, is to
observatory. Astrosat with a lift-off mass of about 1513 kg determine whether nuclear fusion can be a source of
was launched into a 650 km orbit by PSLV-C30. energy and electricity in the future.
One of the unique features of ASTROSAT mission is that Fusion is the energy source of the Sun and stars. In the
enables the simultaneous multi-wavelength observations tremendous heat and gravity at the core of these stellar
of various astronomical objects with a single satellite. bodies, hydrogen nuclei collide, fuse into heavier helium
atoms and release tremendous amounts of energy in the
46. Solution (c) process.
Exp) Option c is correct.
51. Solution (d)
USA (1971), USSR (1971) and European Space Agency
Exp) Option d is correct.
(2003) had spacecrafts orbit the Mars before India did it
with its Mars Orbiter Mission (2014). All of the above given options are applications of genome
sequencing.
The United Arab Emirates (UAE) became the first Arab
country and the fifth country to reach Mars and the Genome sequencing is the process of determining the
second country after India to successfully enter Mars’ complete DNA sequence of an organism’s genome.
orbit on its first try. 52. Solution (c)
47. Solution (d) Exp) Option c is correct.
Exp) Option d is correct. SCNT is a laboratory strategy for creating a viable embryo
According to the type of the host they infect, viruses are from a body cell and an egg cell.
classified mainly into the following four types: 53. Solution (c)
• Plant viruses including algal viruses, Cauliflower Exp) Option c is correct.
mosaic virus.
All the statements are correct regarding the advantages of
• Animal viruses including human viruses like Polio OLEDs over LEDs.
virus, Retro virus etc.
• Mycoviruses are viruses that infect fungi. 54. Solution (d)
• Bacteriophage are viruses which infect bacteria. Exp) Option d is correct.
All the above are correctly matched.
48. Solution (b)
Exp) Option b is correct. 55. Solution (c)
A transcriptome is the full range of messenger RNA, or Exp) Option c is correct.
mRNA, molecules expressed by an organism. Zika virus is transmitted to humans through the of an
Transcriptome of a cell or a tissue is the collection of RNAs infected Aides aegypti and Aides albopictus mosquitoes.
transcribed in it. In contrast with the genome, which is These are the same mosquitoes that spread dengue
characterized by its stability, the transcriptome actively and chikungunya diseases. Zika Virus can be sexually
changes. Messenger RNA (mRNA) is a single-stranded transmitted between humans. A pregnant woman can
RNA molecule that is complementary to one of the DNA pass Zika virus to her foetus during pregnancy.
strands of a gene. Messenger RNA (mRNA) carries the 56. Solution (b)
genetic information copied from DNA in the form of a
series of three-base code “words,” each of which specifies Exp) Option b is correct.
a particular amino acid. Transcriptomics is an emerging The eLISA is a European Space Agency mission designed
and continually growing field in biomarker discovery to detect and accurately measure gravitational waves.
for use in assessing the safety of drugs or chemical risk eLISA is a constellation of 3 spacecraft arranged in an
assessment. equilateral triangle with one mother spacecraft and 2
daughter spacecrafts. Each arm of the imaginary triangle,
49. Solution (c) from the mother to each daughter spacecraft, will measure
Exp) Option c is correct. about a million km.

Workbook 134
.
SCIENCE & TECHNOLOGY

57. Solution (a) • Mustard is a self-pollinating plant because of its


Exp) Option a is correct. individual flowers containing both female and male
reproductive organs, thus it not ideal for cross-
Event Horizon is the horizon of the black hole. This is
pollination. Therefore, GM Mustard uses the Barnase/
where space time is most bent.
Barstar technique that allows for cross-pollination
Singularity is the actual point of ‘infinite’ density where and hybridization in mustard.
the mass of the black hole is concentrated.
String theory proposes that the fundamental constituents 61. Solution (c)
of the universe are one dimensional “strings” rather Exp) Option c is correct.
than point-like particles. It is also called the ‘Theory of THAAD is an anti-ballistic missile defence system,
Everything’. designed and manufactured by the US company Lockheed
Standard Model of particle physics is the theory Martin. It is designed to shoot down short, medium and
describing three of the four known fundamental forces in intermediate-range ballistic missiles in their terminal
the universe as well as classifying all known elementary phase (descent or reentry) by intercepting with a hit-to-
particles. kill approach.
58. Solution (a) 62. Solution (a)
Exp) Option a is correct. Exp) Option a is correct.
Polar Satellite Launch Vehicle (PSLV) and Geo- IRNSS, renamed NAVIC (Navigation with Indian
synchronous Launch Vehicle (GSLV) are satellite launch Constellation). It can provide accurate position up to 1500
vehicle developed by ISRO. km from India’s boundary.
PSLV: IRNSS will provide two types of services, namely, Standard
Launches mainly ‘earth-observation’ or ‘remote sensing’ Positioning Service (SPS) which is provided to all the
satellites into sun-synchronous circular polar orbit of 600- users and Restricted Service (RS), which is an encrypted
900 km altitude. service provided only to the authorised users. The IRNSS
can provide a position accuracy of better than 20 m.
Satellite weight up to 1750 kg.
Four staged launch vehicle; first and third stage use solid 63. Solution (b)
rocket motors, second and fourth stage use liquid rocket Exp) Option b is correct.
engines. The Belle II experiment is a particle physics experiment
GSLV: designed to study violations of the Standard Model of
Launches mainly ‘communication satellites’ to the highly particle physics.
elliptical Geosynchronous Transfer Orbit (GTO). Blockchain technology is widely used in maintaining a
“Geo-synchronous” Satellites appear permanently fixed to digitized, de-centralized public ledger of cryptocurrencies.
a place, thus avoiding the need of changing direction of CRISPR/Cas9 is a gene-editing technology.
antennas.
GSLV Mk III is a three-staged vehicle with first stage using 64. Solution (d)
solid-fuel motor, second stage using liquid fuel and the Exp) Option d is correct.
third stage called Cryogenic Upper Stage, using cryogenic All the above given options are proven applications of 3D
engine. printing.
GSLV Mk-III launched Chandrayaan-2 and will be used Three-dimensional printing (3-D printing), also known
to launch the Gaganyaan. as Additive manufacturing (AM) is a process of joining
materials to make objects from 3D model data, usually
59. Solution (c) layer upon layer. It is basically a process of making three
Exp) Option c is correct. dimensional solid objects from a digital file.
A Geomagnetic Reversal is a change in a planet’s magnetic
field such the positions of magnetic north and magnetic 65. Solution (b)
south are interchanged, while geographic north and Exp) Option b is correct.
geographic south remain unchanged. The Internet of Things (IoT) refers to a system of
When Earth was created the atmosphere had abundance interrelated, internet-connected objects that are able to
Hydrogen sulphide and Carbon dioxide. collect and transfer data over a wireless network without
human intervention.
60. Solution (b)
Exp) Option b is correct. 66. Solution (d)
• GM Mustard (DMH-11) is transgenic mustard Exp) Option d is correct.
developed by Centre for Genetic Manipulation of All of the above are areas where GPS technology is used.
Crop Plants at Delhi University. Global Positioning System (GPS) is a satellite based global

135 Workbook
.
SCIENCE & TECHNOLOGY

navigation systemdeveloped by the USA. It provides LTE is commonly marketed as 4G, it supports download
accurate location and time information anywhere on speeds of upto 100 mbps and upload speeds upto 50 mbps.
the Earth. Banking operations require precise local and VoLTE can simultaneously send data and voice over the
international timing to schedule money transfers, this network without reducing quality of voice call.
is provided by atomic clocks onboard the GPS satellites.
GPS technology aids in recording location of power grids 73. Solution (b)
and power grids rely on GPS to stay in sync. Exp) Option b is correct.
67. Solution (c) The Denisovan or Denisova hominins are an extinct
Exp) Option c is correct. species or subspecies of archaic humans in the genus.
They were discovered in 2010.
All the given above are types of cyberattacks. Wannacry
is a ransomware that spread across computer networks 74. Solution (a)
worldwide. Petya is a family of encrypting malware Exp) Option a is correct.
affecting Microsoft Windows-based systems. Eternal Blue
is a cyberattack exploit developed by the US National The joining of DNA segments from two cells of different
Security Agency. species is possible but making a Functional chromosome
out of it has not been discovered yet.
68. Solution (c)
75. Solution (a)
Exp) Option c is correct.
Exp) Option a is correct.
An API is a set of programming code that enables
data transmission between one software product and Cas9 protein has been in news for its property of being a
another. It also contains the terms of this data exchange. molecular scissors used in targeted gene editing. Its main
Authentication is required before this data exchange can function is to cut DNA and thus alter a cell’s genome. Cas9
be allowed. plays a vital role in the immunological defense of certain
bacteria against DNA viruses.
69. Solution (b) The gene editing technique using such DNA scissors is
Exp) Option b is correct. titled “CRIPSR-Cas9”
What is written in statement 1 is true about Virtual Reality
(VR) and statement 2 is correct description of Augmented 76. Solution (b)
Reality (AR). Exp) Option b is correct.
Augmented Reality (AR) is an interactive experience of A transgenic crop is a genetically modified organism
a real-world environment where designers enhance parts (GMO) which contains one or more genes that have been
of users’ physical world with computer-generated input. inserted artificially either from an unrelated plant or from
A Simple example is, during football games, broadcasters different species altogether.
use AR to draw lines on the field to illustrate and analyse Budding and grafting are methods of vegetative
plays. propagation, not transgenic crops.
Virtual Reality (VR) is a computer-generated environment
with scenes and objects that appear to be real, making the 77. Solution (a)
user feel they are immersed in their surroundings. Virtual Exp) Option a is correct.
Reality places the user inside an experience.
Hormone replacement therapy (HRT) is a treatment to
70. Solution (c) relieve symptoms of the menopause. It replaces hormones
(Oestrogen and Progesterone) that are at a lower level as
Exp) Option c is correct.
you approach the menopause.
The statements 1 and 2 are correct for “Digital Signature
Certificates” but not for “Digital Signatures”. 78. Solution (b)
The Central Government has appointed a Controller Exp) Option b is correct.
of Certifying Authorities who grants a license to the • Multi-drug resistance is not a hereditary problem.
Certifying Authorities to issue digital signature certificates
to the subscriber. • Overuse and misuse of antimicrobial agents is the
single most important cause of development of
71. Solution (d) resistance.
Exp) Option d is correct. • Anti-biotics are also used indiscriminately as growth
All the statements are correct regarding tasks that can be promoters in animals or used to prevent diseases in
accomplished by wearable devices. healthy animals. The antibiotics used in livestock
are ingested by humans when they consume food,
72. Solution (d) this transfers resistant bacteria to humans through
Exp) Option d is correct. consumption or environment.

Workbook 136
.
SCIENCE & TECHNOLOGY

• There is also no conclusive evidence to prove that 83. Solution (d)


presence of multiple chronic diseases is a reason for Exp) Option d is correct.
presence for occurrence of multi-drug resistance in
Drones is a term used in aviation to refer to Unmanned
India. Aerial Vehicle (UAV). Drones can perform all the
79. Solution (b) activities listed in the question. However, Government
has clarified that it is illegal to use drones for spraying
Exp) Option b is correct. insecticides/pesticides on crops under the provisions of
There is no vaccine against Hepatitis C as the hepatitis Insecticides Act, 1968. When Whales come to surface to
C virus is more variable than hepatitis A and hepatitis B breath is called spouting. Drones are being used to collect
viruses. breath samples from their blowholes when they spout.
These samples are being used for DNA analysis.
However, there are vaccines against Hepatitis A and B.
Hepatitis B vaccine is called first “anti-cancer” vaccine 84. Solution: (a)
because it prevents Hepatitis B, the leading cause of liver Exp) Option a is correct.
cancer worldwide.
The substances which vapourise during burning, give
80. Solution (b) flames. For example, kerosene oil and molten wax rise
through the wick and are vapourised during burning
Exp) Option b is correct.
and form flames. Charcoal, on the other hand, does not
Gravitational waves are distortions in the fabric of space vapourise and so does not produce a flame.
and time caused by the movement of massive objects, The outermost zone is the hottest part of the flame. This
like the ripples made on a pond’s surface when someone provides for maximum heat while burning ensuring quick
throws a rock in the water. These cosmic ripples would process. Also, complete combustion takes place in this
travel at the speed of light and, squeeze and stretch zone preventing any sort of blackening of metal. Hence,
anything in their path as they pass by. goldsmiths blow the outermost zone of a flame with a
Albert Einstein predicted the existence of gravitational metallic blow-pipe for melting gold and silver.
waves in 1916 in his general theory of relativity. Option b is incorrect. The outermost part is the non-
In 2015, LIGO physically sensed the undulations in luminous zone of the flame. It is blue in color. The
spacetime caused by gravitational waves generated by luminous zone is the middle portion. Luminous zone
collision of two black holes which happened 1.3 billion contains unburnt carbon particles which can get deposited
years ago. But, the ripples didn’t make it to Earth until on the metal if that is used for melting gold and silver.
2015. Option c is incorrect. The innermost zone is least hot
and also contains unburnt wax vapors. The outermost
81. Solution (d) zone is the hottest part of the flame. It provides ample heat
Exp) Option d is correct. energy required for melting of gold.
Laser Interferometer Space Antenna (LISA) is a European Option d is incorrect. Large amount of heat is required
Space Agency (ESA) mission designed to detect and for vaporization of gold which is not possible generally.
Also, middle portion is moderately hot and contains
accurately measure gravitational waves.
unburnt products due to partial combustion.
LISA pathfinder was a proof of concept mission with arm
size reduced from millions of kilometers long to 40 cm. 85. Solution (d)
Whereas, Evolved LISA or eLISA is a constellation of 3 Exp) Option d is correct.
spacecraft arranged in an equilateral triangle with one All the statements given above are correct regarding
mother spacecraft and 2 daughter spacecrafts. Each Carbon Nanotubes.
arm of the imaginary triangle, from the mother to each Carbon Nanotubes are allotropes of Carbon. They weigh
daughter spacecraft, will measure about a million km. almost 1/6th of the steel, with a mechanical tensile strength
of almost 400 times that of steel.
82. Solution (d)
Exp) Option d is correct. 86. Solution (d)
Solar power using solar photovoltaic cells can be used for Exp) Option d is correct.
surface as well as submersible water pumps. Genome editing can be employed to make changes to
Solar power pumps can be used for running centrifugal specific parts of a genome through Somatic cell therapy
(after birth) and germline genome editing (before birth).
pump as well as positive displacement pump which
The embryonic cells (blastomeres) are totipotent cells.
usually uses a piston to transfer water.
Pluripotent cells have the ability to undergo self-renewal
Additional info:- and to give rise to all cells of the tissues of the body.
PM-Kusum is an initiative of MNRE to promote usage of Inducing such human cells in a pig we can cultivate a
Solar pumps among farmers. human like heart, lung, liver, spleen etc.

137 Workbook
.
SCIENCE & TECHNOLOGY

87. Solution (d) contracts and banking. It has application in any field
Exp) Option d is correct. which requires storage of data.
Streptococcus pneumoniae (pneumococcus) is a leading 93. Solution: (d)
cause of bacterial pneumonia, meningitis and sepsis in Exp) Option d is the correct answer.
children. Otitis media, sinusitis and bronchitis are more
common, but less serious manifestations of this infection. “Closed source” software has source code that only the
person, team, or organization who created it and
PCV are used to protect infants against diseases caused by
maintains exclusive control over. But Open source
Streptococcus pneumoniae bacterium.
software is software with source code that anyone can
PCV can have side-effects like mild fever, redness, inspect, modify, and enhance.
headache, chills etc.
Some examples of public services being delivered through
Additional info:- systems that use FOSS building blocks, including Aadhaar,
India launched its first indigenously developed PCV in GSTN , and the DigiLocker.
2020. Many other solutions launched by the government
including Digilocker, Diksha, Aarogya Setu, the
88. Solution (d)
Covid-19 vaccination platform CoWIN have also been
Exp) Option d is correct. built on top of open-source digital platforms.
Mitochondrial Gene Therapy is a medical technique
in which defective mitochondria carried by a woman is 94. Solution: (d)
replaced with the healthy mitochondria of a donor. The Exp) Option d is the correct answer.
two most common techniques in mitochondrial gene Web 3.0 is the next version of the internet, where services
therapy are maternal spindle transfer and pronuclear will run on blockchain. It is a decentralised internet
transfer. that runs on a public blockchain, which is also used for
89. Solution (a) cryptocurrency transactions.
Exp) Option a is correct. Statement 1 is correct: Presently, only centralized
repositories are the ones that own user data and profit
Public Key Infrastructure (PKI) is a technology for from it. In Web 3.0, users can own and be properly
authenticating devices and users in the digital world. It
compensated for their time and data. In Web 3.0 people
combines encryption and authentication making online
will control their own data .
communication secure and thus trustworthy.
Statement 2 is correct: Web 3.0 will be permissionless
Nowadays most basic authentication involves a private
key (also known as a secret key), which only one person and democratic. For example Twitter will not be able to
should be able to use to decrypt those messages and a censor posts and Facebook will not be able to maintain a
public key, which anyone can use to encrypt a message. database of billions of users that can be potentially used to
PKI govern encryption keys through the issuance of digital influence elections. Thus people can maintain blockchain
certificates and their management therefore verifying the based social networks in Web 3.0.
owner of the private key. Statement 3 is correct: Web3 will mean that sharing
SSL certificates on websites are one of the most common photos, communicating with friends and buying things
examples of PKI. online will no longer by synonymous with Big Tech
companies but be done through a multitude of small
90. Solution (d) competing services on the blockchain. It means that all the
Exp) Option d is correct. value that’s created can be shared amongst more people,
All the statements are correct regarding the present state rather than just the owners, investors and employees.
of development of capabilities of AI. Thus it is operated by users collectively rather than
corporation.
91. Solution (c)
95. Solution: (d)
Exp) Option c is correct.
Exp) Option d is the correct answer.
Instead of radio frequency, VLC uses light to transfer light
to transfer data. Light cannot penetrate through obstacles Software-as-a-Service (SaaS) is a software licensing
and thus it can transmit signals over short distances only, model. It allows access to software on a subscription basis
mostly within a room. using external servers.
Statement 1 is correct: Today’s web-based software is
92. Solution (d) flexible enough to be modified for specific business uses
Exp) Option d is correct. but also individual users. Buyers can customize the user
Not all cryptocurrencies operate on a blockchain and not interface (UI) to change the look and feel of the program,
all blockchains utilize cryptocurrencies as part of their as well as modify specific areas, such as data fields, to
design. Blockchain Technology has many applications alter what data appears. Several business process features
other than Cryptocurrency such as in governance, smart can also be turned off and on at will.

Workbook 138
.
SCIENCE & TECHNOLOGY

Statement 2 is correct: SaaS allows each user to access Industrial control


programs via the Internet. The user need not install Local Area Networks
the software on his/her computer. Thus users can access
data through their mobile devices. SaaS allows users to Medical implants
access the software through a web browser from multiple Metering devices
locations. He or she can have remote desktop software and Remote control
can work from home Radio frequency identification (RFID)
Statement 3 is correct: Outlook, Hotmail or Yahoo! Road Transport Telematics
Mail are forms of SaaS. With these services, a user log
into their account over the Internet, often from a web Telemetry.
browser. The email software is located on the service 99. Solution: (d)
provider’s network and your messages are stored there as
well. You can access your email and stored messages from Exp) Option d is the correct answer.
a web browser on any computer or Internet-connected Biofilms are a collective of one or more types of
device microorganisms that can grow on many different surfaces.
Microorganisms that form biofilms include bacteria,
96. Solution: (c) fungi and protists.
Exp) Option c is the correct answer. Statement 1 is correct: Biofilms are ubiquitous and they
The “Fractional Orbital Bombardment System” is seen form on virtually all surfaces immersed in natural aqueous
in news recently because China deployed this technique environment, e.g., water pipes, living tissue, tooth surface,
in launching long March rocket. This technique allows implanted medical devices, dental implants, etc.
it to propel an extremely maneuverable, nuclear-capable Statement 2 is correct: Diverse microorganisms are
glider into orbit, allowing it to speed towards its target able to grow on food matrixes and along food industry
at hypersonic speed. The idea behind Fractional Orbital infrastructures. Thus biofilm can form on food and
Bombardment System (FOBS) is, a warhead is put into a food processing surfaces. For example escherichia
stable orbit and it deorbits over the target. If the target coli, Salmonella enterica etc. Salmonella enterica
and the launch position is lined up and the warhead keeps contaminates a food pipeline biofilm, may induce massive
going round, it will complete a circle outbreaks and even death in children and elderly.
97. Solution: (b) Statement 3 is correct: A biofilm is a structured
consortium of bacteria embedded in a self-produced
Exp) Option b is the correct answer. polymer matrix consisting of polysaccharide, protein
A qubit (or quantum bit) is the quantum mechanical and DNA. Bacterial biofilms show increased tolerance to
analogue of a classical bit. In classical computing the antibiotics and disinfectant chemicals as well as resisting
information is encoded in bits, where each bit can have phagocytosis and other components of the body’s defence
the value zero or one. system. Thus they cause chronic infections.
Just like a binary bit is the basic unit of information in 100. Solution: (b)
classical (or traditional) computing, a qubit (or quantum
bit) is the basic unit of information in quantum computing. Exp) Option b is the correct answer.
In quantum computing the information is encoded Statement 1 is incorrect: The Serum Institute of India
in qubits. A qubit is a two-level quantum system where produced COVID-19 vaccine named Covishield but
the two basis qubit states are usually written as 0 and 1. not using mRNA platform. It has been prepared using
A qubit can be in state 0 or 1 or (unlike a classical bit) the viral vector platform. In the vaccine, a chimpanzee
in a linear combination of both states. The name of this adenovirus – ChAdOx1 – has been modified to enable
phenomenon is superposition. it to carry the COVID-19 spike protein into the cells of
humans. This cold virus is basically incapable of infecting
98. Solution: (d) the receiver, but can very well teach the immune system to
Exp)Option d is the correct answer. prepare a mechanism against such viruses.
Short Range Devices (SRD) are radio devices that offer a The same technology was used to prepare vaccines for
low risk of interference with other radio services, usually viruses like Ebola.
because their transmitted power, and hence their range, is Statement 2 is correct: Sputnik V, the world’s first
low. The definition ‘Short Range Device’ may be applied registered vaccine, is based on the human adenoviral
to many different types of wireless equipment, including vector-based platform. The gene from adenovirus, which
various forms of: causes the infection, is removed while a gene with the
Access control (including door and gate openers) code of a protein from another virus spike is inserted.
Alarms and movement detectors Statement 3 is correct: Covaxin is an inactivated viral
vaccine. It is developed with Whole-Virion Inactivated
Closed-circuit television (CCTV) Vero Cell-derived technology. That is, it contains
Cordless audio devices, including wireless microphones inactivated viruses that cannot infect a person, but

139 Workbook
.
SCIENCE & TECHNOLOGY

still can teach the immune system to prepare a defence into a digital form can be an NFT. Everything from your
mechanism against the active virus. drawings, photos, videos, GIF, music, in-game items,
selfies, and even a tweet can be turned into an NFT, which
101. Solution: (c) can then be traded online using cryptocurrency.
Exp) Option c is the correct answer. It is a unique digital asset that represents ownership of
Solar flares are intense bursts of radiation from the sun real-world items like art, video clips, music, and more.
that can directly influence electronics on Earth. They are Statement 2 is correct. Non-fungible tokens (NFTs)
often associated with coronal mass ejections (CMEs), are cryptographic assets on a blockchain with unique
which are large clouds of gas that erupt from the sun’s identification codes and metadata that distinguish them
interior into its atmosphere. from each other.
Statement 1 and 3 are correct: The large solar flare Statement 3 is incorrect. Unlike cryptocurrencies, they
is projected to strike satellites in the Earth’s upper cannot be traded or exchanged at equivalency. This
atmosphere. Thus it affects GPS navigation, mobile differs from fungible tokens like cryptocurrencies,
phone transmissions, and satellite TV. The flares might which are identical to each other and, therefore, can
potentially disrupt power grids and electrical networks serve as a medium for commercial transactions.
in various regions of the planet.
Statement 2 is incorrect: Solar flares can temporarily alter 103. Solution: (d)
the upper atmosphere creating disruptions. But it does Exp) Option d is the correct answer.
not cause Tsunamis. It only affects the upper atmosphere. Statement 1 is incorrect: Naturally occurring
Statement 4 is correct: The high-energy particles emitted nanoparticles can be found in volcanic ash, ocean spray,
by the flare could also cause certain atoms in the Earth’s fine sand and dust, and even biological matter (e.g.
atmosphere to glow, leading to auroras in Central viruses). Synthetic nanoparticles are equally, if not more
Europe as well as North America. The phenomenon diverse than their naturally occurring counterparts.
could only be seen if the weather is clear.
Statement 5 is incorrect: There is no causal link between
the coronal mass ejections (CMEs) and the deadly forest
fires. There is a common misconception that solar storms
are associated with extreme geological and atmospheric
effects on Earth. The sun cannot produce solar flares or
CMEs that physically burn the landscape, if it did, I doubt
complex life would have had much of a chance at evolving
over the last few billion years if our planet was constantly
being sterilized.
Statement 6 is correct: The scientists found that solar
flares and storms heated and expanded the atmosphere
and that these effects could extend to the satellites’ orbits.
The density of atmospheric gases increases, slowing
the satellites and causing them to lose altitude by a few Statement 2 is correct: The field of nanotechnology is
kilometres per day. being greatly explored by cosmetic industries in order to
improve the efficacy of cosmetic products. The cosmetics
Statement 7 is correct: Solar flares allow interference
are formulated by using different types of metal and metal
with the short wave radios that are being used, emitting
oxide nanoparticles such as silver nanoparticles (AgNPs),
more rays allowing people to get more of a sun burn
gold nanoparticles (AuNPs) and titanium dioxide
during this time frame.
nanoparticles (TiO2 NPs), zinc oxide nanoparticles, (ZnO
A study by NASA of polar flights during a solar storm in NPs), iron oxide nanoparticles, (Fe2O3 NPs) and carbon-
2003 showed that passengers received about 12 percent based NPs. The applications of different metal oxides as
of the annual radiation limit recommended by the one of the active ingredients in cosmetics and sunscreens
International Committee on Radiological Protection. The have attracted enormous interest from cosmetologists
exposures were greater than on typical flights at lower with advanced knowledge of chemistry, toxicology,
latitudes, and confirmed concerns about commercial dermatology, rheology, and even marketing.
flights using polar routes.
Statement 3 is correct: According to the National
102. Solution: (a) Nanotechnology Initiative thousands of tons of silica,
Exp) Option a is the correct answer. alumina and ceria, in the form of ultrafine abrasive
particle mixtures including nanoparticles, are used each
Non-fungible tokens (NFTs) are cryptographic assets year in slurries for precision polishing of silicon wafers.
on a blockchain with unique identification codes and The exposed population to nano materials continues to
metadata that distinguish them from each other. increase as their application expands. Despite obvious
Statement 1 is correct. Anything that can be converted benefits of the power of small materials, there are open

Workbook 140
.
SCIENCE & TECHNOLOGY

questions about how the nanoparticles used for day-to- Exp) Option b is correct.
day life may affect the environment. One of the crucial The University of Cambridge has found out new genes
issues that have to be addressed in the near future, emerging in the genome of living organisms. They have
before massive fabrication of nano materials, is their called them novel Open Reading Frames or as nORFs.
toxicity to humans and impact on the environment.
Human Genome Project (HGP) was an international,
104. Solution: (d) collaborative research program conducted from 1990
Exp) Option d is the correct answer. to 2003. The main goal was to completely map and
understand all the genes of human beings.
DNA barcoding is a molecular technology that allows
the identification of any biological species by amplifying, But even after the research, the number of ‘known’
sequencing and querying the information from genic genes has remained constant for two decades. Hence, a
team from the University of Cambridge has set out to
and/or intergenic standardized target regions belonging
find whether new genes emerge in the genome of living
to the extranuclear genomes
organisms and if they do, how they do so.
Option 1 is incorrect: Geologists use the radiometric
Novel Regions: These novel regions were explored and
dating of uranium and the isotopes of other radioactive
cannot be defined by our current ‘definition’ of a gene.
elements, combined with observations of fossils and
Hence, these novel regions are called novel Open Reading
sediment layers, to chart the course of Earth’s natural
Frames or as nORFs. These regions are also broadly
history and the evolution of life. To measure the age of
involved in diseases. They were seen as dysregulated (poor
plant and animal remains from the more recent past,
ability to manage) in 22 cancer types.
scientists use a radioactive isotope of carbon, called
carbon-14, as their clock. As carbon-14 decays, with 107. Solution (d)
a half-life of about 5,730 years, it becomes nitrogen-14.
Exp) Option d is correct.
Using this clock, they have dated bones, campfires and
other objects as old as 60,000 years, and in some cases Option 1 is correct. Plant cultivor’s identification and
even older. seed purity test were mainly based on phenotypic traits.
Rapidly changing cropping pattern lead to confusion in
Option 2 is correct: DNA barcodes allow non-experts to the varietal names and their local names. DNA molecular
objectively identify species – even from small, damaged, marker has high specificity, selectivity, simplicity,
or industrially processed material. Just as the unique accuracy and genetic stability can reflects the differences
pattern of bars in a universal product code (UPC) in DNA level without environmental impacts, and thus
identifies each consumer product, a “DNA barcode” is has great advantages in seed purity identification.
a unique pattern of DNA sequence that can potentially
identify each living thing. Option 2 is correct. Drug delivery systems in medicine
that are based on bacterial or viral hosts could prove
Option 3 is correct. cpDNA and mtDNA barcoding hazardous. Use of the soil bacterium Agrobacterium
protocols are being used more and more in the food tumefaciens as a vehicle for gene transfer is very effective.
industry and food supply chains for food labeling, not DNA technology can significantly improve the medicine
only to support food safety but also to uncover food piracy delivery system.
in freshly commercialized and technologically processed
products. DNA barcoding helps to identify undesirable Option 3 is correct. DNA technology is used to develop
animal or plant materials in processed foods. transgenic crops which improves crop resistance to insects
and certain herbicides and delaying ripening. This helps
in improving yield and profitability.
2.2. Biotechnology Option 4 is correct. Transgenic animals model
105. Solution (d) advancements in DNA technology in their development.
Exp) Option d is correct. 108. Solution (b)
Statement 1 is correct: In the hand-guided cloning Exp) Option b is correct.
technique, the nucleus from the egg is taken by a hand- Option a is incorrect. Narco-Analysis is a controlled
held fine knife. administration of intravenous hypnotic medications called
Statement 2 is correct. In hand-guided cloning truth drugs on a suspect to procure vital information.
technique, the technique such as chromosomes etc. Option b is correct. Brain fingerprinting is an objective,
come from single donor’s cell, that is from one parent. scientific method to detect concealed information stored
Statement 3 is correct. The hand-guided cloning in the brain by measuring electroencephalographic (EEG)
technique developed at NDRI, is an advanced brain responses, or brainwaves, non-invasively by sensors
modification of the “Conventional Cloning Technique”. placed on the scalp. The technique involves presenting
Statement 4 is correct. In 2009, the NDRI had produced words, phrases, or pictures containing salient details
the first female buffalo calf at the institute. about a crime or investigated situation on a computer
screen, in a series with other, irrelevant stimuli. Brain
106. Solution (b) responses to the stimuli are measured and try to decipher

141 Workbook
.
SCIENCE & TECHNOLOGY

the information stored in brain.


Option c is incorrect. In a polygraph test also popularly
known as lie detector test, the accused person’s
physiological indicators are taken into account which
includes blood pressure, pulse rate, respiration and skin
conductivity.
Option d is incorrect. Automated Virtual agent
for Truth Assessment in Real time or AVATAR for
interrogating an individual via video conference. The
system uses Artificial Intelligence to assess changes in
person’s eyes, voices, gestures and posture that raise flag
about possible deception.

109. Solution (d)


Exp) Option d is correct.
Option a is correct. The initiative will pave the way for
identifying genes and genetic variations for common
110. Solution (a)
diseases, treating Mendelian disorders, enabling the Exp) Option a is correct.
transformation of the Precision Medicine landscape in Option a is correct. ‘IndiGen’ initiative is to map genome
India, and thus improving the healthcare of the general sequencing of a larger swathe of the population in the
country. Project aims to widen public understanding in
population in our country.
India about genomes and the information that genes hide
Option b is correct. It is an initiative of the Department about one’s susceptibility to disease.
of Biotechnology, Ministry of Science and Technology Under the ‘IndiGen’ initiative those who do get their genes
mapped will get a card and access to an app, which will
that will aim to sequence the human genome of the Indian allow them and doctors to access “clinically actionable
population for various beneficial purposes. The IISc’s information” on their genomes.
Centre for Brain Research, an autonomous institute, will 111. Solution (b)
serve as the nodal point of the project. Exp) Option b is correct.
Option c is correct. It aims to identify the locations of Statement 1 is correct. The Genetic Engineering
Appraisal Committee (GEAC) functions in the Ministry
many human genes that will ultimately make up the of Environment, Forest and Climate Change (MoEF&CC).
genome in the Indian population and provide information Statement 2 is correct. The Genetic Engineering Appraisal
about their structure and organization. The first stage of Committee (GEAC) is a statutory body constituted under
the ‘Rules for the Manufacture, Use /Import /Export
the project will look at samples of “10,000 persons from and Storage of Hazardous Microorganisms/ Genetically
all over the country” to form a “grid” that will enable the Engineering Organisms or Cells, 1989 notified under the
Environment (Protection) Act, 1986.
development of a “reference genome”.
Statement 3 is incorrect. GEAC is chaired by the Special
Option d is incorrect. The data generated would be Secretary/Additional Secretary of MoEF&CC and co-
accessible to researchers anywhere for analysis. As the chaired by a representative from the Department of
Biotechnology (DBT).
genetic landscape differs across the world, it is necessary
112. Solution (c)
that genetic data is shared in order to derive greater
Exp) Option c is correct.
knowledge from information and serve the purpose of
Statement 1 is correct. Biotechnology Industry
enabling better treatment outcomes. Research Assistance Council (BIRAC) is a not-for-

Workbook 142
.
SCIENCE & TECHNOLOGY

profit Public Sector Enterprise set up by Department of fragments can be separated by a technique known as
Biotechnology (DBT). BIRAC is an industry-academia gel electrophoresis. Since DNA fragments are negatively
interface and implements its mandate through a wide charged molecules, they can be separated by forcing them
range of impact initiatives. to move towards the anode under an electric field through
Statement 2 is correct. E-Yuva scheme is mandated to a medium/matrix.
promote a culture of applied research and need-oriented Restriction enzymes belong to a larger class of enzymes
(societal or industry) entrepreneurial innovation among called nucleases. These are of two kinds; exonucleases and
young students and researchers. endonucleases. Exonucleases remove nucleotides from
This scheme is launched by BIRAC. It is implemented the ends of the DNA whereas, endonucleases make cuts at
through dedicated hubs called E-Yuva Centers (EYCs) specific positions within the DNA.
housed within the University/Institute set up and Option b is correct. MRT or Mitochondrial donation is
mentored by a BIRAC BioNEST supported bio-incubator. a medical technique in which defective mitochondria
carried by a woman is replaced with the healthy
113. Solution (b)
mitochondria of a donor. Mitochondrial donation is an
Exp) Option b is correct. assisted reproductive technology which can help some
Statement 1 is incorrect: The National Genomic Grid parents to avoid transmitting mitochondrial DNA disease
aims to study genomic factors influencing cancer and to their biological children. The term collectively refers to
identifying the right treatment modalities for the a number of specific techniques aimed at ensuring only
Indian population. The National Genomic Grid will be healthy mitochondria is passed on to an embryo.
formed in line with the National Cancer Tissue Biobank Option c is correct. Restriction endonucleases are used
(NCTB). in genetic engineering to form ‘recombinant’ molecules
Statement 2 is correct. This study is carried out through of DNA, which are composed of DNA from different
the technique of Genome Sequencing that is the process sources/genomes. When cut by the same restriction
of determining the complete DNA sequence of an enzyme, the resultant DNA fragments have the same kind
organism’s genome at a single time. of ‘sticky-ends’ and, these can be joined together (end-to-
end) using DNA ligases.
114. Solution (c) Option d is correct. Genome editing, or genome
Exp) Option c is correct. engineering, or gene editing, is a type of genetic
Transgenic animals are those animals in which DNA is engineering in which DNA is inserted, deleted,
altered to produce new type of organisms by inserting modified or replaced in the genome of a living organism.
or deleting genes. This process is called genetic
116. Solution (c)
engineering.
Exp) Option c is correct.
Cloning is the creation of a perfect replica of a particular
organism i.e., creating similar populations of genetically Statement 1 is correct. Gene therapy is defined as
identical individuals. therapeutic strategy that transfers DNA to a patient’s cells
to correct a defective gene or a gene product in order to
Statement 1 is correct. Cloning allows would‐be parents treat diseases that are not curable with conventional drugs.
to give their children the same genes as a pre‐existing Gene therapy products are being studied to treat diseases
person, whereas genetic engineering allows them to give including cancer, genetic diseases, and infectious diseases.
their children genes that have been intentionally designed, There are a variety of types of gene therapy products,
modified or selected in the laboratory for some particular including: Plasmid DNA: Circular DNA molecules can
purpose. be genetically engineered to carry therapeutic genes into
Statement 2 is correct. Cloning can happen naturally human cells.
by asexual mode of reproduction. And also cloning can Statement 2 is correct.
happen artificially by molecular cloning and reproductive
cloning. GTPs include:
Genetically modified plants are created artificially. But Recombinant viral vectors: adenovirus, retrovirus
recent studies have shown that genetically modified plants Non-viral vectors: naked DNA transfection Microbial/
can come about by natural means as well. A research group bacterial vectors (Salmonella, E. coli): recombinant
in Sweden has now described the details of such an event bacteria derived vehicles
among higher plants. It is likely that the gene transfer was Modifications resulting from the use of CRISPR and
mediated by a parasite or a pathogen. other similar technologies ex vivo genetically modified
cells: gene modified/ augmented stem cells, iPS (induced
115. Solution (a) pluripotent stem) cells, CAR-T cells etc.
Exp) Option a is correct. Soluble/particulate/emulsion/Nano based
Option a is incorrect. The cutting of DNA by restriction interventions containing any form of genetic material/
endonucleases results in the fragments of DNA. These nucleic acid for the purpose of clinical gene therapy

143 Workbook
.
SCIENCE & TECHNOLOGY

DNA vaccines where the final product is nucleic acid and period. The fossils are known only in the form of imprints
is administered for vaccination/ therapy. and casts in sandstone beds.
117. Solution (a) Researchers have discovered three fossils of the earliest
known living animal — the 550-million-year-old
Exp) option a is correct. ‘Dickinsonia’ — on the roof of the Bhimbetka Rock
Statement 1 is correct. MANAV human atlas is a project Shelters, about 40 km from Bhopal. One can identify
funded by Department of Biotechnology which aims at the fossils from the white leaf-like patches with a central
creating database network of all tissues in the human vertebra (central midrib) and connecting veins. While one
body. fossil is 17 inches long, the other two are much smaller.
Statement 2 is incorrect: Students from the fields of Pair 2 is correctly matched. An open reading frame is a
biochemistry, biotechnology, microbiology, botany, portion of a DNA molecule that, when translated into
zoology, bioinformatics, health sciences, systems amino acids, contains no stop codons.
biologists, pharmacologists, and data sciences can A team from the University of Cambridge set out to
associate with this project. Participants having a science find whether new genes emerge in the genome of living
background but not necessarily involved in active organisms, and if they do, how they do so. In the last seven
scientific research can be also part of this network. years, the team extensively studied the human genome
Statement 3 is correct: The data collected under the and has now catalogued 1, 94, 000 novel regions.
project will be parallelly helpful to both researchers These ‘novel’ genomic regions cannot be defined by
and clinicians and drug developers, who handle human current ‘definition’ of a gene. Hence, these novel regions
bodies in diseased conditions. are called – novel Open Reading Frames or as nORFs.
118. Solution (a) Mutations in nORFs do have physiological consequences
and a majority of mutations that are often annotated as
Exp) Option a is correct benign have to be re-interpreted.
Statement 1 is correct: Atal Jai Anusandhan Biotech Pair 3 is incorrectly matched. “Little Foot” is the
Mission - Undertaking Nationally Relevant Technology nickname given to a nearly complete Australopithecus
Innovation (UNaTI), which is expected to transform fossil skeleton found in 1994–1998 in the cave system of
Health, Agriculture and Energy sectors during the next Sterkfontein, South Africa.
5 years.
Little Foot is the fossil of an early human forerunner that
Statement 2 is incorrect: This mission is undertaken by inhabited South Africa 3.67 million years ago. Little Foot’s
the Department of Biotechnology under Ministry of species blended ape-like and human-like traits and is
Science and Technology. considered a possible direct ancestor of humans.
Statement 3 is incorrect: Another initiative called the
UMMID initiative has been launched by Ministry of 121. Solution: (c)
Science and Technology. Under it, the focus is on the Exp) Option c is correct
management and treatment of inherited disorders. Genetically Modified Organisms (GMO) are those plants,
119. Solution (d) bacteria, fungi and animals whose genes have been altered
by manipulation.
Exp) Option d is correct.
Statement 1 is correct. A genetically modified organism
Option 1 is correct: Mitochondrial DNA Analysis - It (GMO) is an animal, plant, or microbe whose DNA has
is a method used by forensic scientists to match DNA been altered using genetic engineering techniques in the
from an unknown sample to a sample collected at a crime laboratory. It is done in order to favour the expression of
scene. desired physiological traits or the generation of desired
Option 2 is correct. Brain Electrical Oscillation biological products. In contrast, cloning describes a
Signature (BEOS) Profiling – It is applicable in the number of different processes that can be used to produce
forensic field for the detection of a person as suspect, genetically identical copies of a biological entity. The
accused, witness or complainant. copied material, which has the same genetic makeup as
Option 3 is correct. The polymerase chain reaction the original, is referred to as a clone. Researchers have
(PCR) revolutionized forensic science techniques. It cloned a wide range of biological materials, including
helped in forensic individualization of biological material genes, cells, tissues and even entire organisms, such as a
by forming the basis of the so-called “genetic fingerprint”. sheep.
It helps in comparing stains found at a crime scene and a Statement 2 is incorrect. The production of GMOs
suspect, or the identification of human remains. imposes high risks to the disruption of ecosystem and
biodiversity because the “better” traits produced from
120. Solution (c) engineering genes can result in the favouring of one
Exp) Option c is correct. organism. Hence, it can eventually disrupt the natural
Pair 1 is incorrectly matched. Dickinsonia is an extinct process of gene flow.
genus of a basal animal that lived during the late Ediacaran Statement 3 is incorrect. Though much of the controversy

Workbook 144
.
SCIENCE & TECHNOLOGY

around genetically modified organism is driven by the more than 80 disorders including neuromuscular and
belief that the process of moving genes from one species degenerative disorders. Eg. Bone-marrow transplant is
to another is “unnatural,” new research shows some 1 in used in Leukemia (blood cancer), sickle-cell anaemia, and
20 flowering plants are naturally transgenic. immunodeficiency disorders.
Dozens of plants, including bananas, peanuts, Surinam Statement 4 is correct. Unlike most of the medical
cherries, hops, cranberries and tea, contain the treatments, stem cell therapy comes with no side
Agrobacterium microbe. The research follows on the heels effects. It also prevents infections and reduces the risk
of the 2015 discovery that sweet potatoes are naturally of complications through natural healing. Once the stem
transgenic. cells are injected at the site of injury, they stimulate the
Cloning can also occur naturally. In nature, some plants growth of new cells, which repair and heal the damaged
and single-celled organisms, such as bacteria, produce tissue.
genetically identical offspring through a process called
123. Solution: (d)
asexual reproduction. Natural clones, also known as
identical twins, occur in humans and other mammals. Exp) Option d is correct
These twins are produced when a fertilized egg splits, Bt Cotton is a genetically modified cotton grown in India
creating two or more embryos that carry almost identical for almost 20 years.
DNA. Identical twins have nearly the same genetic Option a is correct. Bt Cotton is the genetically modified
makeup as each other, but they are genetically different (GM) cotton which contains the pesticide gene from the
from either parent. bacteria Bacillus thuringiensis (Bt). According to the
122. Solution: (b) Ministry of Agriculture, from 2005, adoption of Bt cotton
rose to 81% in 2007, and up to 93% in 2011.
Exp) Option b is correct
Option b is correct. As of date, the Genetic Engineering
Stem cells are the body’s raw materials — cells from which Approval Committee (GEAC) has approved the
all other cells with specialized functions are generated. environmental release of only Bt cotton in India for
They are basically those cells which may replicate commercial cultivation. It was approved in 2002. Bt
themselves many times, or proliferate. brinjal is still under moratorium while GM mustard is yet
Statement 1 is correct. Stem cells are undifferentiated to be approved for commercialisation.
biological cells that can differentiate into specialized cells Option c is correct. This pesticide gene in Bt cotton
and can divide to produce more stem cells. Stem cells protects the plant from bollworm, thereby increasing
are distinguished from other cell types by two important yields and reducing insecticide spraying on the cotton
characteristics. First, they are unspecialized cells capable plant.
of dividing and renewing themselves through cell
division, sometimes after long periods of inactivity. Second, Option d is incorrect. Bt Cotton transformed India’s
under certain physiologic or experimental conditions, cotton sector, as cotton productivity almost doubled in
they can be induced to become tissue- or organ-specific six years. India’s share in the global production of cotton
cells with special functions. In some organs, such as the increased from 12 per cent in 2002 to 25 per cent by 2014.
gut and bone marrow, stem cells regularly divide to repair From a net importer of cotton, India became the second-
and replace worn out or damaged tissues. largest exporter of cotton (and not largest).
Statement 2 is incorrect. Embryonic stem cells are 124. Solution: (b)
derived from embryos that develop from eggs that have Exp) Option b is correct
been fertilized in vitro. An adult stem cell is thought to
be an undifferentiated cell, found among differentiated Bioprinting is a process where biomaterials such as cells
cells in a tissue or organ. Embryonic are pluripotent and growth factors are combined to create tissue-like
stem cells i.e. they can become any type of cell in the structures that imitate natural tissues. The technique
body. This versatility allows embryonic stem cells to be is widely applicable to the fields of medicine and
used to regenerate or repair diseased tissue and organs. bioengineering.
Compared with embryonic stem cells, adult stem cells Statement 1 is correct. 3D Bioprinting is a form of additive
have a more limited ability to give rise to various cells manufacturing that uses cells and other biocompatible
of the body. Other types of stem cells are foetal stem cells materials as “inks”, also known as bioinks, to print living
(taken from aborted foetal tissue) and umbilical stem structures layer-by-layer which mimic the behaviour
cells (taken from umbilical cord). of natural living systems. In essence, bioprinting works
Statement 3 is correct. Stem cell therapy is a type of in a similar way to conventional 3D printing. A digital
treatment option that uses a patient’s own stem cells model becomes a physical 3D object layer-by-layer. In
to repair damaged tissue and repair injuries. It is a this instance, however, a living cell suspension is utilized
non-surgical procedure with no incisions of the body. instead of a thermoplastic or a resin.
Stem cells are usually taken from one of the two areas Statement 2 is correct. The greatest importance of
in the patient’s body: bone marrow or adipose (fat) bioprinting lies in the resulting tissue-like structures that
tissue in their upper thigh/abdomen. It is used to treat mimic the actual micro- and macro-environment of

145 Workbook
.
SCIENCE & TECHNOLOGY

human tissues and organs. This is critical in drug testing Option b is correct. Gene therapy is used in the
and clinical trials, with the potential, for example, to replacement of genes that cause medical ill-health. This
drastically reduce the need for animal trials. The process method generally destroys the problem causing genes.
could also eradicate the headaches associated with organ On the other hand, the goals of gene editing are to disrupt
donation and transplantation. harmful genes or to repair mutated genes.
Statement 3 is incorrect. There is an emission of Option c is correct. Both inherited genetic diseases and
harmful chemicals during the bioprinting process. acquired disorders can be treated with gene therapy.
Other challenges include ensuring the compatibility Examples of these disorders are primary immune
of materials and cells with the printer and printing deficiencies, where gene therapy has been able to fully
processes in printing vascular network. In addition, it may correct the presentation of patients, and/or cancer, where
take longer for vascular networks to develop and mature the gene therapy is still at the experimental stage.
throughout the tissue structure than for cells to survive. Option d is correct. Transfer of genetic material is
125. Solution: (d) done commonly by using viral vectors that use their
own biological capacities to enter the cell and deposit
Exp) Option d is correct the genetic material. Several types of viruses, including
A transgenic animal is one whose genome has been retrovirus, adenovirus, adeno-associated virus (AAV),
altered by the transfer of a gene or genes from another and herpes simplex virus, have been modified in the
species or breed. laboratory for use in gene therapy applications.
Statement 1 is correct. A transgenic animal is one that
carries a foreign gene that has been deliberately inserted 127. Solution: (d)
into its genome. Transgenesis can in principle be used to Exp) Option d is correct.
alter many phenotypic properties that may increase the Statement 1 is correct. Nanotechnology can be used
value of agriculturally important animals. These include to prevent, monitor and alleviate a wide range of
growth rate, fat composition, milk production, and hair environmental problems, while significantly reducing
texture. cost and improving performance. Current and future
Statement 2 is correct. The foreign gene in transgenic applications of nanotechnology will allow us to:
animals is constructed using recombinant DNA • Develop new “green” processing technologies that
methodology. This involves either combining of DNA minimize the amount of undesired byproducts;
from different genomes or the insertion of foreign DNA
• Detect and remove the finest contaminants from
into a genome. Example: GloFish are a type of transgenic
air, water, and soil, which would enhance the ability
zebrafish (Danio rerio) that have been modified through
of governments to respond to terrorist threats and
the insertion of a green fluorescent protein (gfp) gene.
ensure the safety of water supplies.
Statement 3 is correct. Xenotransplantation is one of the
Statement 2 is correct. Cosmetics and personal
most promising areas for the exploitation of the transgenic
products companies have been extremely active in using
technology, where transgenic pigs are engineered to
nanotechnology to improve their existing products and to
express immunological characteristics that would
develop new ones.
make their organs well tolerated after transplantation
into humans. Transgenic animals are also engineered Statement 3 is correct. In the field of military, improved
for increased production of dairy proteins for food, materials, lighter but with tough, heat resistant properties,
enhanced production of wool or enhanced disease are being used in the design and construction of
resistance. In biotechnology, the use of large transgenic spacecraft and satellites, and this process will gain
animals or ‘bioreactors’ has been attempted, e.g., for the from nanotechnology. There is also the possibility of
cost-effective production of high-value pharmaceuticals nanotechnology facilitating improvements in civilian
in the blood or milk of transgenic sheep or cows. security equipment.
Statement 4 is correct. The science of metals, ceramics,
126. Solution: (a)
colloids and polymers, has always concerned itself with
Exp) Option a is correct controlling the structure of materials on the Nano scale.
Gene therapy is a technique that modifies a person’s genes Some specific areas in which Nano science technology is
(introduction, removal or change in the content of an contributing to materials science now include: new forms
individual’s genetic material) to treat or cure disease. of carbon; Nano composites; quantum dots and wires;
Option a is incorrect. In gene editing, a mutated gene is and nanostructured materials produced by self-assembly.
revised, removed, or replaced at the DNA level. In gene Statement 5 is correct. The medical area of Nano science
therapy, the effect of a mutation is offset by inserting application is projected as one of the most potentially
a “healthy” version of the gene. Both approaches may valuable, with many projected benefits to humanity.
provide a durable benefit to patients, and both gene With the advent of new materials, and the synergy
therapy and gene editing, alone or in combination, may of nanotechnologies and biotechnologies, it could be
lend themselves to the development of transformative possible to create artificial organs and implants that
genomic medicines. are more akin to the original, through cell growth on

Workbook 146
.
SCIENCE & TECHNOLOGY

artificial scaffolds or biosynthetic coatings that increase animal manure and even sewage, to produce large
biocompatibility and reduce rejection. quantities of Single Celled Proteins (CSP).
128. Solution: (b) 130. Solution: (c)
Exp) Option b is correct Exp) Option c is correct.
Biofortification is the process by which the nutritional Statement 1 is correct. Genetic engineering is applied in
quality of food crops is improved through agronomic the development of Recombinant vector vaccines. Genetic
practices, conventional plant breeding, or modern engineering involves techniques to alter the chemistry of
biotechnology. genetic material (DNA and RNA), to introduce these into
Greater number of people suffer from micronutrient, host organisms and thus change the phenotype of the host
protein and vitamin deficiencies or ‘hidden hunger. In organism. The techniques of genetic engineering include
this context, Biofortification is a new strategy to combat creation of recombinant DNA, use of gene cloning and
hidden hinger. gene transfer.
Statement 1 is incorrect. Biofortification differs from Statement 2 is correct. Recombinant vaccines are made
conventional fortification in that biofortification aims of a viral or bacterial vector. They are used as channels for
to increase nutrient levels in crops during plant growth entry into host organisms.
rather than through manual means during processing of
the crops. 2.3. Science in Environment and Agriculture
Statement 2 is incorrect. Iron-biofortification of rice,
131. Solution (c)
beans, sweet potato, cassava and legumes has been
successful. Partially iron-biofortified rice has improved Exp) Option c is correct.
the iron stores of reproductive-age women in the Option a is correct. Biotech KISAN will connect farmers
Philippines. Iron-biofortified pearl millet has increased with science in the 15 agro-climatic zones of the country
the iron stores and reversed iron deficiency in school in a manner, which constantly links problems with
children in India available solutions. They will follow hub and spoke
Statement 3 is correct. Golden Rice is a biofortified model. In each of these 15 regions, a farmer organization
crop developed by International Rice Research Institute will be the hub connected to different science labs,
(IRRI) and its national research partners. It is aimed to Krishi Vigyan Kendra and State Agriculture Universities
complement existing interventions to address vitamin co-located in the region. The hub will reach out to the
A deficiency (VAD). VAD is a serious public health farmers in the region and connect them to scientists and
problem affecting millions of children and pregnant institutions.
women globally. Option b is correct. Biotech KISAN Hub will get financial
assistance for initially 2 years of Rs 60 lakhs per year and
129. Solution: (d) on the basis of a review for additional 3 years.
Exp) Option d is correct Option c is incorrect. The scheme includes the Mahila
Single cell protein (SCP) refers to dead, dry cells of Biotech- KISAN fellowships, for training and education
microorganisms, such as yeast, fungi, bacteria and algae. in farm practices, for women farmers. The Scheme also
Statement 1 is correct. Single-cell proteins are the aims to support the women farmers/ entrepreneur in their
dried cells of microorganism, which are used as the small enterprises, making her a grass root innovator.
protein supplement in human foods or animal feeds. Option d is correct. Biotech-Krishi Innovation Science
Microorganisms like algae, fungi, yeast and bacteria, Application Network (Biotech-KISAN) is a Department
utilize inexpensive feedstock and wastes are used as of Biotechnology, Ministry of Science and Technology
sources of carbon and energy for growth to produce initiative that empowers farmers, especially women
biomass, protein concentrate or amino acids. farmers. It aims to understand the problems of water,
Statement 2 is correct. Single Celled Protein (SCP) soil, seed and market faced by the farmers and provide
can be produced from wastes, thus lead to reduction simple solutions to them.
in environmental pollution. In this, the agricultural
132. Solution (d)
and some industrial wastes which may contribute to a
significant level of pollution in watercourses are used for Exp) Option d is correct.
SCP production. Thus, the utilization of such materials Option a is incorrect. Golden rice was created as an
in SCP processes serves two functions as reduction in additional intervention for vitamin A deficiency (and
pollution and creation of edible protein. not vitamin C). This dietary deficiency is the most
Statement 3 is correct. Microbes being used as a source significant cause of childhood blindness, and also the
of good protein for production of protein at industrial biggest killer of children under 5 years old, globally.
scale. For example, Spirulina which is a Blue-green Option b is incorrect. Golden Rice is any variety of rice
algae is grown on materials like waste water from potato which makes beta-carotene (and not fertilizer use),
processing plants (containing starch), straw, molasses, thus giving the rice a yellow (Golden) color. White rice

147 Workbook
.
SCIENCE & TECHNOLOGY

is naturally low in the pigment beta-carotene, which the Statement 2 is correct. Blanching is carried out by briefly
body uses to make Vitamin A. immersing the food in boiling water followed by an ice
Option c is incorrect. India has been cautious while bath to rapidly cool the food. When a food is blanched
allowing cultivation of GM food. So as of now only Bt properly, the flavor, colour, texture, and nutritional
cotton has been allowed for commercial cultivation in value is preserved. Blanching gently softens the outside
India. of the food while keeping the interior crisp, sweetens the
produce a little, and causes the vegetable to hold its colour
Option d is correct. The technology in Golden Rice for a longer period of time. Thus, helps in increasing the
varieties has been donated by its inventors Professors shelf life.
Ingo Potrykus and Peter Beyer. So, the technology in
Golden Rice varieties is owned by the public in countries Statement 3 is correct. Ohmic heating (or electro-
where rice is the staple crop. No one involved with the conductive heating) generates heat by the passage of
development or promotion of Golden Rice has any electrical current through food which resists the flow
financial interest in its use: the project is completely not of electricity. So, any food that has electric resistance can
for profit. generate heat on its own when current is passed through
it, which helps in preserving the food for longer durations
and thus increasing the shelf life of food product.
Statement 4 is correct. Vacuum degassing is also used
in the food industry to remove encapsulated air from
products such as beverages, minced meat, sausage meat,
cheese, pasta, vegetable puree, mustard or jam. It helps
in increasing the shelf life of the product. The process
normally takes place immediately before packaging the
product. It may be carried out in a filler, mixer or extruder
production line.
133. Solution (c) Statement 5 is correct. Irradiation exposes foodstuff
Exp) Option c is correct. to ionized radiation. It increases shelf life of food
products by eliminating the microorganisms. It is used in
Statement 1 is correct. The permissible limit for pasteurizing milk and canning fruits.
genetically modified organisms (GMO) in imported food
crops is set by Food Safety and Standards Authority of 135. Solution (d)
India. FSSAI issued an order in February 2021 setting Exp) Option d is correct.
the permissible limit for genetically modified organisms
(GMO) in imported food crops at 1%. Statement 1 is correct – Compared to bulk form of
chemical inputs in crops, use of nano-nutrients can
The task of regulating GMO levels in imported reduce nutrient run-off into ground and surface water
consumables was initially with the Genetic Engineering and can thus reduce environmental pollution.
Appraisal Committee (GEAC) under the Union
environment ministry. Their role was diluted with the Statement 2 is correct – With the use of nanotechnology,
enactment of the Food Safety and Standards Act, 2006 micro-irrigation can be made more feasible. It can
and FSSAI was asked to take over approvals of imported certainly enhance the irrigation efficiency as the
goods. absorption from micro-irrigation systems is close to 90%
while flood irrigation has efficiency only around 60%.
Statement 2 is incorrect. India allows the import of GM
soybean and canola oil. The only crops approved for Statement 3 is correct – The seed or root treatments
cultivation is (bt) cotton. involve soaking the seeds in an aqueous suspension of
nano pyrite. The core idea is to leverage the potential of
Statement 3 is correct. A transgenic organism is one that an energy molecule (nano iron pyrite) to boost the seed
has been altered through recombinant DNA technology, and root metabolism. If the metabolic activity of the seed
which involves either the combining of DNA from or roots is bolstered at the very onset of their journey in
different genomes or the insertion of foreign DNA into the soil, the plant can grow better and assimilate nutrients
a genome. optimally. It eventually leads to higher production
Statement 4 is correct. Use of the unapproved GM and provides an opportunity to cut down on fertiliser
variant can attract a jail term of 5 years and fine of Rs. requirements.
1 lakh under the Environmental Protection Act, 1986.
136. Solution (d)
134. Solution (d) Exp) Option d is correct.
Exp) Option d is correct. Statement 1 is incorrect. Aerial Seeding is a plantation
Statement 1 is correct. In waxing, the surface of fruit or technique wherein seed balls — seeds covered with a
vegetable is coated in a certain way to reduce moisture mixture of clay, compost, char and other components
loss. This helps in increasing the shelf life of food. Waxing — are sprayed using aerial devices, including planes,
also enhances the appearance of fruit or vegetable. helicopters or drones. The balls fall to the ground with

Workbook 148
.
SCIENCE & TECHNOLOGY

the help of the coating of clay, compost, char and other for monitoring of soil conditions like moisture and
material, that provides the required weight for seeds to temperature. The Internet of things uses a network of
drop on a predetermined location rather than disperse in sensors, electrical systems, wireless communication to
the wind. These balls/pellets sprout when there is enough track various parameters in real time of systems under
rain with nutrients present within them helping in initial observation. Data relating to these parameters is used in
growth. decision making.
Statement 2 is incorrect. Recently, the Haryana Forest Statement 3 is correct. Machine learning and Artificial
Department has deployed aerial seeding technique to Intelligence can be used for predictive agricultural
improve green cover in the Aravalli area of Faridabad. analytics, i.e., to predict on weather, sowing time,
irrigation schedule, etc. Microsoft has built an AI app for
137. Solution (b) projects in Andhra Pradesh which recommends farmers
Exp) Option b is correct. on sowing date, land preparation, seed treatment, manure
Statement 1 is incorrect. Recently, the institute of use, pest management etc.
Pesticide Formulation Technology (IPFT) under the
Department of Chemicals and Petrochemicals, Ministry 140. Solution (c)
of Chemicals and Fertilizers has successfully developed Exp) Option c is correct.
a new Bio-Pesticide Formulation for insect control in PUSA bio-decomposer is a decomposer capsule which
seed spice crops. This bio-pesticide is based on entomo- could be converted into a liquid solution and sprayed
pathogenic fungus Verticillium lecanii. Thus, fungal on fields with crop stubble. The solution when sprayed
species can be used to produce bio-pesticides. in the fields can decompose crop residue and turn it into
Statement 2 is correct. Bio pesticides are inherently manure. It has been developed by the Indian Agricultural
less toxic to humans and the environment, do not leave Research Institute (IARI), Pusa.
harmful residues, and are usually more specific to target ‘Pusa Decomposer’ is essentially a fungi-based liquid
pests. Often, they affect only the target pest and closely solution that can soften hard stubble to the extent that it
related organisms, substantially reducing the impact can be easily mixed with soil in the field to act as compost.
on non-target species. A further advantage of most This would then rule out the need to burn the stubble, and
microbial pesticides is that they replicate in their target also help in retaining the essential microbes and nutrients
hosts and persist in the environment due to horizontal in soil that are otherwise damaged when the residue is
and vertical transmission, which may cause long-term burned.
suppression of pest populations even without repeating
the application. Statement 1 is correct. The decomposer improves the
fertility and productivity of the soil as the stubble works
138. Solution (d) as manure and compost for the crops and lesser fertilizer
Exp) Option d is correct. consumption is required in the future. The soil loses its
richness due to stubble burning and it also destroys the
Statement 1 is correct. Farmers and agricultural experts
say that large scale use of direct seeding of rice (DSR) useful bacteria and fungi in the soil, apart from causing
technique to plant paddy could be a long-term solution harm to the environment. It is an efficient and effective,
to the issue of stubble burning. With the use of DSR, cheaper, doable and practical technique to stop stubble
there will be very little stubble left in the field after burning.
harvesting and it will be easier to manage the crop residue. Statement 2 is correct. Also, food grains produced using
Statement 2 is correct. Another benefit is that such paddy the Pusa Decomposer qualify as organic farming, since
crops require less water than the one cultivated through it involves no growth hormones, antibiotics, no genetically
traditional transplantation. modified organisms, and no leaching of surface water or
ground water.
Statement 3 is correct. Paddy cultivated using DSR
technique will mature around 10-11 days earlier than 141. Solution (b)
the one planted through traditional transplantation.
Exp) Option b is correct.
139. Solution (d) Food irradiation (the application of ionizing radiation
Exp) Option d is correct. to food) is a technology that improves the safety and
extends the shelf life of foods by reducing or eliminating
Statement 1 is correct. Blockchain technology allows
microorganisms and insects.
development of tamper proof ledger-based data
storage. In agriculture, it can be used for digitization of Statement 1 is correct. There are three sources of
land records and their management. States like Andhra radiation approved for use on foods-
Pradesh and Telangana have started using blockchain Gamma rays are emitted from radioactive forms of the
for this. Such tamper proof land records aid in contract element cobalt (Cobalt 60) or of the element cesium
farming. (Cesium 137). Gamma radiation is used routinely to
Statement 2 is correct. Internet of Things allow use sterilize medical, dental, and household products and is
of wireless sensors, drones and satellite imagery also used for the radiation treatment of cancer.

149 Workbook
.
SCIENCE & TECHNOLOGY

X-rays are produced by reflecting a high-energy stream of a joint venture between US-based Monsanto and the
electrons off a target substance (usually one of the heavy Maharashtra Hybrid Seeds Co Ltd (Mahyco) introduced
metals) into food. X-rays are also widely used in medicine Bt cotton to India.
and industry to produce images of internal structures. Statement 3 is incorrect. Bt cotton is not herbicide
Electron beam (or e-beam) is similar to X-rays and is resistant, Herbicide Tolerant Bt (Ht Bt) cotton, on the
a stream of high-energy electrons propelled from an other hand is derived with the insertion of an additional
electron accelerator into food. gene, from another soil bacterium, which allows the plant
Statement 2 is incorrect. Items like fruits, potatoes, to resist the common herbicide glyphosate. The use of
onions, spices, herbs and some ready-to-eat foods can HT Bt cotton is banned in India because the herbicide-
be preserved through food irradiation. tolerance can easily spread through pollination, and
eventually give rise to a variety of ‘super-weeds’ that are
Statement 3 is incorrect. Irradiation does not make
resistant to existing herbicide
foods radioactive, compromise nutritional quality,
or noticeably change the taste, texture, or appearance 144. Solution: (b)
of food. In fact, any changes made by irradiation are
Exp) Option b is correct
so minimal that it is not easy to tell if a food has been
irradiated. Tissue culture is the process of producing plants from
tissues of the desired plant in an artificial nutrient medium
142. Solution (c) under controlled environment.
Exp) Option c is correct. Statement 1 is incorrect. Explant is the part of a plant
Statement 1 is correct. Hydroponics is a method of which has got the regeneration potential and is capable to
growing plants without the use of soil. It is the practice give rise to the whole plant.
of growing plants in a nutrient-rich solution. With this Explants regenerate shoots and roots. There are various
technique, soil is replaced with a solution of water and types of explants frequently used for regeneration
nutrients for plants to grow. Since, in hydroponics plants purpose: nodal segments, apical meristems, roots,
are not grown on soil, so it can help to reduce the nutrient cotyledons, embryo, leaf disc, leaf blade, pedicle, petiole,
loss from soil on account of farming. anther, ovary etc.
Statement 2 is incorrect. Hydroponics uses much less Statement 2 is correct. Plants produced through this
water than traditional farming. Hydroponics uses up method are genetically identical to original plant.
to 90% less water than traditional farming. Hydroponics The application of methods of tissue culture leads to
addresses several issues within the food system—water the propagation of a large number of plants in very
scarcity and land shortages predominantly. short durations. Each of these plants will be genetically
Statement 3 is incorrect. The nutrients used in hydroponic identical to the original plant from which they were
systems can come from many different sources, including grown, i.e., they are somaclones.
fish excrement, duck manure, purchased chemical Statement 3 is correct. With the application of the tissue
fertilizers, or artificial nutrient solutions. culture method, the healthy plants can also be recovered
from the diseased plants. For example, even if the plant
143. Solution (b) is infected with a virus, the meristem (apical and axillary)
Exp) option b is correct is free of virus. Hence, one can remove the meristem and
Relevance) GM crops and biotechnology: Recently, grow it in vitro to obtain virus-free plants.
Shetkari Sanghatana, a Maharashtra based farmers’ union
145. Solution: (c)
body has announced agitation for use of unapproved
genetically modified seeds of cotton, maize, rice, mustard, Exp) Option c is correct
soybean and brinjal. Apiculture is the process of beekeeping, involving all
Statement 1 is correct. The term genetically modified aspects of the sector: knowledge of bees, bee products,
(GM), as it is commonly used, refers to the transfer of genes their uses and markets, trade and equipment fabrication
between organisms using a series of laboratory techniques Honey and wax are the two economically important
for cloning genes, splicing DNA segments together, and products of bee keeping.
inserting genes into cells. Collectively, these techniques Statement 1 is incorrect.
are known as recombinant DNA technology. Other
terms used for GM plants or foods derived from them Harvesting the honey is done by smoking the bees off
are genetically modified organism (GMO), genetically the parts which needs to be harvested and cut the combs
engineered (GE), bioengineered, and transgenic. carefully.
Statement 2 is correct. Bt cotton is the first and only Harvests are normally possible during and shortly
Genetically Modified (GM) crop that is allowed in after the two main flowering seasons, namely October/
India. It has alien genes from the soil bacterium Bacillus November and February-June.
thuringiensis (Bt) that allows the crop to develop a It takes four to six months for a new hive to produce
protein toxic to the common pest pink bollworm. In 2002, enough honey to harvest. Many expert beekeepers

Workbook 150
.
SCIENCE & TECHNOLOGY

recommend not to harvest any honey from a new hive irradiation is a process that uses radiation to control pests
until it has successfully made it through one winter in food and prevent spoilage.
Statement 2 is incorrect. Selection of suitable location Statement 1 is correct. Cryopreservation is a process
for keeping the beehives is very important point to that preserves organelles, cells, tissues, or any other
remembered in bee-keeping. Bee-keeping can be practiced biological constructs by cooling the samples to very low
in any area where there are sufficient bee pastures of some temperatures to maintain their viability. The word cryo
wild shrubs, fruit orchards and cultivated crops. comes from the Greek word “kayos” meaning “frost”. It
Statement 3 is correct. Apis Indica is also referred to means preservation in a “frozen state”. Cryopreservation
as the Indian bee. It can be easily domesticated and is uses liquid nitrogen to deep-freeze, and thus preserve,
most commonly used for honey production in India. biological materials.
The annual yield of honey is 2 to 5 kg per colony. Statement 2 is correct. Food irradiation (the application
of ionizing radiation to food) is a technology that
146. Solution: (d) improves the safety and extends the shelf life of foods
Exp) optional d is correct by reducing or eliminating microorganisms and insects.
Plant breeding, application of genetic principles to During irradiation, gamma rays, x-rays, or high-energy
produce plants that are more useful to humans. This is electrons pass through the food, destroying or inactivating
accomplished by selecting plants found to be economically bacteria and viruses that cause foodborne illness.
or aesthetically desirable, first by controlling the mating Statement 3 is incorrect. Preservation of food through
of selected individuals, and then by selecting certain cryopreservation delays spoilage and keeps foods safe
individuals among the progeny. by preventing microorganisms from growing and by
Statement 1 is correct. Plant breeding is the purposeful slowing down the enzyme activity that causes food to
manipulation of plant species in order to create desired spoil. As the water in the food freezes into ice crystals, it
plant types that are better suited for cultivation, give becomes unavailable to those microorganisms that need
better yields and are disease resistant. It involves it for growth. In contrast, food irradiation does not make
application of genetic principles to produce plants that foods radioactive, compromise nutritional quality,
are more useful to humans. It is done by selecting plants or noticeably change the taste, texture, or appearance
which are found to be economically desirable, then by of food. In fact, any changes made by irradiation are
controlling the mating of selected individuals, and then so minimal that it is not easy to tell if a food has been
by selecting certain individuals among the progeny. irradiated. Irradiated food is considered safe by the World
Health Organization (WHO)

2.4. Health and Medicine


148. Solution (a)
Exp) Option a is correct.
Statement 1 is correct. Somatic cell nuclear transfer
involves transferring the nucleus of a cell, which includes
its DNA, into an egg which has had its nucleus removed.
Process:
A somatic cell is isolated and extracted from an adult
female. Then the nucleus and all of its DNA from an egg
Statement 2 is correct. Conventional plant breeding has cell is removed. After that the nucleus from the somatic
been in practice for around thousands of years, since the cell is transferred to the egg cell. After being inserted into
beginning of human civilisation. The recorded evidence the egg, the somatic cell nucleus is reprogrammed by the
of plant breeding can be date backed to 9,000-11,000 host cell and is stimulated with a shock. The egg cell, with
years ago. In fact, many present-day crops are the result its new nucleus, will behave just like a freshly fertilised
of domestication in ancient times. egg. It developed into an embryo, which is implanted into
Statement 3 is correct. Irradiation can be used to induce a surrogate mother.
mutations in plants with the goal to produce varieties Statement 2 is correct. Somatic Cell Nuclear Transfer
that display improved product quality, have higher yields technique is currently the basis for cloning animals
and yield stability, greater resilience to climate change and (such as the famous Dolly the sheep), and has been
tolerance to environmental stresses theoretically proposed as a possible way to clone
humans. But there are moral and ethical objections
147. Solution: (a) against reproductive cloning.
Exp) Option a is correct Statement 3 is correct. The application of Somatic
Cryopreservation is a technique in which low temperature Cell Nuclear Transfer research to generate tissues or
is used to preserve the living cells and tissue while food even organs for transplant into the specific patient.

151 Workbook
.
SCIENCE & TECHNOLOGY

The resulting cells would be genetically identical to the the number of protons in a nucleus equals the number
somatic cell donor, thus avoiding any complications from of electrons orbiting the nucleus, the atom is electrically
immune system rejection. neutral.
Statement 4 is incorrect. In 2018, Chinese Scientists have 151. Solution (d)
managed to clone two identical long-tailed macaques,
the first primates to have undergone cloning. This Exp) Option d is correct.
brings human cloning one step closer. 3D Bioprinting is a form of additive manufacturing that
uses cells and other biocompatible materials as “inks”, also
149. Solution (c) known as bioinks, to print living structures layer-by-layer
Exp) Option c is correct. which mimic the behavior of natural living systems.
Statement 1 is correct. According to the Drugs and Option 1 is correct – Scientists have developed a new 3D
Cosmetics Act, 1940, The central drugs standard control bioprinter that can create totally functional human skin,
organisation (CDSCO) is responsible for approval of which can be used for transplants as well as testing of
drugs, approving the conduct of clinical trials, laying cosmetic and pharmaceutical products. It replicates the
down the standards for drugs, control over the quality natural structure of the skin, with a first external layer,
of imported drugs in the country. It also coordinates the the epidermis with its stratum corneum, which acts as
activities of state drug control organisations by providing protection against the external environment, together
expert advice with a view of bring about the uniformity in with another thicker, deeper layer, the dermis. This last
the enforcement of the Drugs and Cosmetics Act. layer consists of fibroblasts that produce collagen, the
protein that gives elasticity and mechanical strength to
Statement 2 is incorrect. Clinical trials of new drugs and the skin.
vaccines, and their approvals, are governed by the New
Option 2 is correct – Researchers from Indian Institute
Drugs and Clinical Trials Rules, 2019. The rules provide
of Technology (IIT) Delhi and IIT Kanpur have used a
for ‘accelerated approval process’ in several situations like
different approach to mimic the development biology
the current pandemic.
pathway by which adult load-bearing, long bones
In such situations, there is a provision for granting are formed. The bone construct was fabricated by
approval to a drug that is still in clinical trials, provided combining tissue engineering and 3D bioprinting.
there is a prima facie case of the product being of The cartilage was first 3D bioprinted using bioink and
meaningful therapeutic benefit. cartilage characteristics were thoroughly characterised.
Statement 3 is correct. As per the new Drugs and Clinical The researchers then added a thyroid hormone
Trial Rules, 2019, the time for approving applications (Triiodothyronine or T3) to the cartilage to facilitate the
for clinical trial has been reduced to 30 days for drugs differentiation of cartilage into bone-like cells.
manufactured in India and 90 days for the drugs Option 3 is correct – The Sree Chitra Tirunal Institute for
developed outside India. Medical Sciences and Technology (SCTIMST) has already
developed bioinks to print functional liver tissues from
150. Solution (d) hepatocytes. These artificial tissues can be used in the
Exp) Option d is correct. pharma industry for drug screening to test liver toxicity.
Statement 1 is correct. Proton therapy, also called Option 4 is correct - 3D bioprinting could help
proton beam therapy, is a type of radiation therapy. It uses eliminate the need for testing new drugs on animals
protons rather than x-rays to treat cancer. A proton is a thus replacing lengthy and expensive clinical trials. 3D
positively charged particle. At high energy, protons can printed tissue prove to be an effective means of testing
destroy cancer cells. Doctors may use this therapy alone. new pharmaceuticals. Thus, the drug could be thoroughly
They may also combine it with x-ray radiation therapy, assessed and brought to market more quickly.
surgery, chemotherapy, and/or immunotherapy.
152. Solution (b)
Statement 2 is correct. In regular radiation therapy, the
Exp) Option b is correct.
beam of energy goes into the body, through the tumor,
and out the other side. This “exit dose” of radiation might Statement 1 is correct. In terms of cellular structure,
affect healthy tissue beyond the tumor. After delivering cultured or cultivated meat is the same as conventional
the energy to the tumor, the protons stop: They do meat except that cultured meat does not come directly
not exit the tumor and go into healthy tissue on the from animals.
other side. In this way, proton therapy reduces radiation Statement 2 is correct. Scientists will use the animal’s
exposure and potential damage to healthy tissue, stem cells to create meat. Stem cells are the building
especially in sensitive areas such as the brain, eyes, spinal blocks of cells and tissues. By feeding the stem cells with
cord, heart, major blood vessels and nerves. amino acids and carbohydrates, the muscle cells will be
Statement 3 is correct. Proton, a stable subatomic multiplied and grown in the lab.
particle that has a positive charge equal in magnitude to a Statement 3 is incorrect. Once artificially meat is
unit of electron charge and a rest mass of 1.67262 × 10−27 created, it resembles actual meat in terms of appearance,
kg, which is 1,836 times the mass of an electron. When texture, and nutrient profile.

Workbook 152
.
SCIENCE & TECHNOLOGY

Statement 4 is correct. It does not require antibiotics, full protection to those who cannot safely be vaccinated.
unlike animals raised for meat. Thus, it reduces the threat But with herd immunity, these people will have
of antibiotic resistance posed to public health. substantial protection, thanks to those around them
being vaccinated.
153. Solution (b)
Exp) Option b is correct.
Option a is incorrect. Molecular Diagnosis: It is the use of
molecular biology for early detection of diseases (at a time
when the symptoms of the disease are not yet visible).
Here, very low concentration of a bacterial or viral
disease-causing agent can be detected by amplification of
their nucleic acid using techniques like Polymerase Chain
Reaction (PCR) and Enzyme Linked Immuno-Sorbent
Assay (ELISA).
Option b is correct. Gene Therapy: It is the insertion of
genes into an individual’s cells and tissues to treat diseases
especially hereditary diseases. It does so by replacing
a defective mutant allele with a functional one or gene
targeting which involves gene amplification.
Option c is incorrect. Active immunity: A type of acquired
Statement 4 is incorrect. In some rare cases the antigen
immunity whereby resistance to a disease is built up by
either having the disease or receiving a vaccine against it. added by live attenuated vaccine changes over time and
starts to behave like a wild or naturally occurring virus.
Option d is incorrect. Crossbreeding: Interbreeding to For instance vaccine-associated paralytic poliomyelitis
breed (animals or plants) using parents of different races, (VAPP) is an adverse event following exposure to Oral
varieties, breeds, etc.
Polio Vaccine (OPV). OPV is made with live attenuated
154. Solution (c) (weakened) polioviruses that can result in a case of VAPP.
Exp) Option c is correct. 155. Solution (c)
Statement 1 is incorrect. Vaccines contain weakened or Exp) Option (c) is the correct answer.
inactive parts of a particular organism (antigen and not
antibody) that triggers an immune response within the Polymerase chain reaction (PCR) is a common laboratory
body. Newer vaccines contain the blueprint for producing technique used to make many copies (millions or
antigens rather than the antigen itself. billions!) of a particular region of DNA. This DNA region
can be anything the experimenter is interested in. For
example, it might be a gene whose function a researcher
wants to understand, or a genetic marker used by forensic
scientists to match crime scene DNA with suspects.
Typically, the goal of PCR is to make enough of the target
DNA region that it can be analyzed or used in some other
way. For instance, DNA amplified by PCR may be sent for
sequencing, visualized by gel electrophoresis, or cloned
into a plasmid for further experiments.
PCR is used in many areas of biology and medicine,
including molecular biology research, medical diagnostics,
and even some branches of ecology.
Statement 1 is correct- Using conventional methods of
Statement 2 is correct. Some vaccines require multiple diagnosis (serum and urine analysis, etc.) early detection
doses, given weeks or months apart. This is sometimes of the disease is not possible. ‘Recombinant DNA
needed to allow for the production of long-lived technology’, ‘Polymerase Chain Reaction (PCR)’ and
antibodies and development of ‘Enzyme Linked Immuno-sorbent Assay (ELISA)’ are
memory cells. In this way, the body is trained to fight the some of the techniques that serve the purpose of early
specific disease-causing organism, building up memory of diagnosis.
the pathogen so as to rapidly fight it if and when exposed Statement 2 is correct- PCR is being used to detect
in the future. mutations in genes in suspected cancer patients.
Statement 3 is incorrect. No single vaccine provides Statement 3 is incorrect- PCR is now routinely used to
100% protection, and herd immunity does not provide detect HIV in suspected AIDS patients.

153 Workbook
.
SCIENCE & TECHNOLOGY

156. Solution (d) 159. Solution (b)


Exp) Option d is correct. Exp) Option b is correct.
Stem cells are undifferentiated biological cells that can Statement 1 is incorrect. In India to resolve parental
differentiate into specialized cells and can divide to disputes through DNA you need a court order and it
produce more stem cells. Stem cell therapy promotes the will be done in the supervision of authorities from police
reparative treatment of diseased, dead or injured tissue. and court.
Possible applications of stem cell therapy are:
Statement 2 is correct. DNA profiles can reveal extremely
Repair of a damaged tissue that can’t heal on its own. sensitive information of an individual & hence could be
Transplantation of stem cells into the body misused for caste/community-based profiling. There are
Bone marrow transplantation criticisms that the DNA profiling is a violation of human
rights as it could also compromise with the privacy of the
Skin replacement
individuals.
Treatment of Parkinson’s disease and Alzheimer’s
disease by replenishing the damaged tissue. 160. Solution (c)
Tissue regeneration Exp) Option c is correct.
Adult stem cells are usually limited to forming specialized Option a is correct: Saviour siblings refer to babies
cells. These stem cells could be used to regenerate many, created to serve an older sibling as a donor of organs,
but not all, tissues in the body. bone marrow, umbilical cord or stem cells to treat
diseases like thalassemia, sickle cell anemia.
157. Solution (d)
Statement b is correct: Doctors successfully conducted
Exp) Option d is correct.
India’s first saviour sibling experiment where baby’s
Statement 1 is correct. The main aim of vaccination bone marrow was used to save the child.
is to teach the immune system how to recognize and
eliminate a pathogen. It stimulates the body to make Option c is incorrect: IVF is a fertilization process in
antibodies against antigens of pathogens. It also primes which egg is combined with the sperm outside the body in
immune cells to remember the types of antigens that a glass. IVF is a commonly used method to create saviour
cause infection. That allows for a faster response to the babies. They are created with In vitro fertilisation (IVF) so
disease in the future. that they undergo pre-implantation genetic diagnosis (or
testing) to get rid of any genetic disorders and also see
Statement 2 is correct. In vaccination antigenic proteins
bone marrow compatibility.
of the pathogen are introduced into the body. The
antibodies produced in the body against these antigens Option d is correct: Human Leukocyte Antigen (HLA)
would neutralise the pathogenic agents during actual complex is a group of related proteins that are encoded
infection. The vaccines also generate memory – B and by the major histocompatibility complex (MHC) gene
T-cells that recognise the pathogen quickly on subsequent complex in humans. These cell surface proteins are
exposure and overwhelm the invaders with a massive responsible for the regulation of the immune system.
production of antibodies. A close match between a donor’s and a patient’s HLA
Statement 3 is correct. Preformed antibodies can be markers is essential for a successful transplant outcome.
injected into the body in case of a deadly microbial HLA matching promotes the growth and development of
attack. If a person is infected with some deadly microbes to new healthy blood cells (called engraftment) and reduces
which a quick immune response is required as in tetanus, the risk of a post-transplant complication called graft-
we need to directly inject the preformed antibodies or versus-host (GVHD) disease. Therefore, embryo of a
antitoxin (a preparation containing antibodies to the saviour sibling must have a matching Human Leukocyte
toxin). Antigen (HLA) to prevent any complications.
158. Solution (c) 161. Solution (d)
Exp) Option c is correct. Exp) Option d is correct.
Statement 1 is correct. An m-RNA or simply “RNA Option d is incorrect. Intracytoplasmic Sperm Injection
vaccine” uses the genetic sequence of a portion of is a special technique of assisted reproduction that is
the virus (rather than an actual virus that is used in most useful for male factor infertility where the sperm
traditional vaccine) that can be injected into the body. count or quality is very poor. This mainly involves the
Once taken in by the body’s cells it makes an antigen to same initial steps as for IVF, except that for the process
which the immunity machinery manufactures antibodies. of fertilisation, a special needle is used to inject the
In this way, the adverse reactions that can result from the sperms inside the egg. Therefore, eliminating the need
traditional approach of introducing an actual virus can be for millions of sperms to fertilise the egg and pregnancy
avoided. can be achieved even with very low sperm count.
Statement 2 is correct. One of the advantages of m-RNA Option a is correct. In-Vitro Fertilization (IVF) is a
vaccines is that they can be made in large quantities and method of assisted reproduction in which the woman’s
in a lesser time than traditional vaccines. eggs and man’s sperms are fertilised outside the body in

Workbook 154
.
SCIENCE & TECHNOLOGY

a laboratory dish, which is why it is also called a ‘test tube 164. Solution (b)
baby’. One or more of these fertilised eggs (embryos) are Exp) Option b is correct.
then transferred in the woman’s womb, so that they can
Statement 1 is correct. Trans-fats are associated with
stick in the uterine lining and grow. It is one of the most
increased risk of heart attacks and death from coronary
commonly used ART procedures and is used to overcome heart disease. As per the World Health Organisation
infertility due to many causes like blocked fallopian tubes, (WHO), approximately 5.4 lakh deaths take place each
endometriosis and even when the cause of a couple’s year globally because of intake of industrially produced
infertility is not known. trans fatty acids. The WHO has also called for global
Option b is correct. Intrauterine insemination (IUI) elimination of trans-fats by 2023.
is aimed at addressing fertility problems arising out of Statement 2 is incorrect. The Food Safety and Standards
low sperm count or poor sperm motility. In IUI, semen Authority of India (FSSAI) has capped the amount of
is injected into the uterus at a predetermined time to trans fatty acids (TFA) in oils and fats to 3% for 2021
increased odds of fertilization. and 2% by 2022 from the current permissible limit of 5%
Statement c is correct. In Intrafallopian transfer, the man’s through an amendment to the Food Safety and Standards
sperm and a woman’s egg are first combined in a lab. Then (Prohibition and Restriction on Sales) Regulations.
the eggs are implanted into the fallopian tubes and the The revised regulation applies to edible refined oils,
fertilization occurs inside a woman’s body. Intrafallopian vanaspati (partially hydrogenated oils), margarine,
transfers are good options for couples with unexplained bakery shortenings, and other mediums of cooking such
infertility, with sperm mobility issues, or when the woman as vegetable fat spreads and mixed fat spreads.
has an issue with her fallopian tube, such as a blocked Statement 3 is incorrect. Industrial trans-fats are
tube. produced by adding hydrogen to liquid vegetable oils
to make them solid, which increases their stability at
162. Solution (c) room temperature and extends shelf life. Trans-fats are
Exp) Option c is correct. unsaturated fats which have been partially saturated with
hydrogen to extend the shelf life.
When Aedes aegypti mosquitoes carry natural bacteria
called Wolbachia, they reduce the mosquitoes’ ability 165. Solution: (a)
to transmit viruses like dengue, Zika, chikungunya and
Exp) Option a is the correct answer.
yellow fever. A study has suggested that the “Wolbachia
method” could be used to significantly reduce the spread Assertion is correct: Organ transplant fails if the
of dengue. The method involves introducing Wolbachia donor organ does not match the recipient’s blood and
into populations of Aedes aegypti. Wolbachia are safe, tissue. Human organs cannot be transplanted from any
natural bacteria present in up to 60% of insect species given donor. Tissue matching, blood group matching is
including some mosquitoes. They are safe for humans, essential before undertaking any transplant, otherwise
animals and the environment. the grafts would be rejected sooner or later.
Reason is correct and also explains Assertion: Without
163. Solution (a) matching of tissues and blood for transplanting the
Exp) Option a is correct. immune system will distinguish between self and non-
self-cells. In this way immune system is able to identify
Statement 1 is correct. Organoids are a group of
cells belonging to the body and foreign cells that need to
cells grown in laboratories into three-dimensional,
be attacked.
miniature structures that mimic the cell arrangement of
a fully-grown organ. They are 3D organ-like-structures 166. Solution: (a)
that aim to replicate human organs. Exp) Option a is correct.
Statement 2 is correct. Organoids are grown in the lab The immune system fights germs and foreign substances
using stem cells that can become any of the specialised on the skin, in the tissues of the body and in bodily
cells seen in the human body, or stem cells taken from fluids such as blood. The immune system is made up of
the organ or adults cells that have been induced to behave two parts: the innate, (general) immune system and the
like stem cells, scientifically called induced pluripotent adaptive (specialized) immune system. These two systems
stem cells (iPSC). Stem cells are provided with nutrients work closely together and take on different tasks.
and other specific molecules to grow and become cells Statement 1 is correct. Innate immunity is also known
resembling a specific organ. as genetic or natural immunity. Innate immunity is the
Statement 3 is incorrect. Although Organoids are tiny immunity that one is born with. This type of immunity is
(typically the size of a pea) structures that replicate written in one’s genes, offering lifelong protection. The
human organs but they cannot achieve all the functional innate immune response is fast acting and non-specific,
maturity of human organs and often resemble the early meaning it does not respond differently based on the
stages of a developing tissue. Most organoids contain specific virus or bacteria that it detects.
only a subset of all the cells seen in a real organ, but lack Statement 2 is correct. Adaptive immunity is an
blood vessels to make them fully functional. organism’s acquired immunity to a specific pathogen.

155 Workbook
.
SCIENCE & TECHNOLOGY

It is also referred to as acquired immunity. Adaptive nutrients and gases between a mother and the foetus.
immunity is not immediate, nor does it always last So, if a mother is infected with HIV while she’s pregnant,
throughout an organism’s entire lifespan, although it the infection gets transmitted to her foetus too through
can. The adaptive immune response is marked by clonal the placenta. Similarly, if a mother gets infected with
expansion of T and B lymphocytes, releasing many HIV while she is still breastfeeding, the infection gets
antibody copies to neutralize or destroy their target transmitted to the infant as well as breast milk too is a
antigen. body fluid.
Statement 3 is incorrect. Helper T cells are the most Option 4 is incorrect: It has been established that HIV
important cells in adaptive immunity, as they are does not transmit through saliva (despite it being a body
required for almost all adaptive immune responses. They fluid).
not only help activate B cells to secrete antibodies and
macrophages to destroy ingested microbes, but they also 169. Solution: (b)
help activate cytotoxic T cells to kill infected target cells. Exp) Option b is the correct answer.
As dramatically demonstrated in AIDS patients, without Performance Enhancing Drugs (PED) are substances that
helper T cells we cannot defend ourselves even against are often taken by athletes in order to boost their physical
many microbes that are normally harmless. Helper T cells performance in their sport by heightening the abilities
can only function when activated to become effector cells. of their bodies. Not only is this an unfair practice, but
also some of these substances pose a serious threat to the
167. Solution: (c)
health and well-being of athletes. Common PEDs include
Exp) Option c is the correct answer. anabolic-androgenic steroids, human growth hormone,
Vaccines typically need time (weeks or months) to creatine, erythropoietin and blood doping, amphetamines
produce protective immunity in an individual and may and stimulants, and beta - hydroxy - beta- methylbutyrate.
require several doses over a certain period of time to Option 1 is incorrect: Melatonin is a hormone which
achieve optimum protection. Passive immunization, controls the circadian rhythm (sleep cycle) in organisms.
however, has an advantage in that it is quick acting, It is secreted by the pineal gland. Normally it is produced
producing an immune response within hours or days, by the brain in response to darkness, aligning sleep with
faster than a vaccine. darkness.
Statement 1 is correct: Passive immunization, or passive Option 2 is correct: Anabolic Androgenic Steroids like
immunotherapy, is a process in which individuals receive Testosterone are primarily used to treat delayed puberty
antibodies from another source rather than producing and muscle-wasting diseases like HIV. However, when
those antibodies on their own. Passive immunization used by a healthy individual in unnatural quantities these
also works quickly, providing protection within hours or build increased muscle tissue and increase body mass by
days. acting like the body’s natural male hormone, testosterone.
Statement 2 is correct: Preformed antibodies, are derived However, steroids cannot improve an athlete’s agility or
from the blood serum of previously infected people skill.
or animals, are often administered in an antiserum to Its abuse causes much damage to the body in the long run
another person in order to provide immediate, passive like cardiac issues, liver damage, poor bone health, etc
immunization against fast-acting toxins or microbes, Option 3 is correct: Erythropoietin (EPO) a hormone
such as those in snakebites or tetanus infections. secreted by the kidneys that increases the rate of
production of red blood cells in response to falling levels
168. Solution: (a)
of oxygen in the tissues. It is a type of peptide hormone.
Exp) Option a is the correct answer.
It is used as a PED (blood doping) because it boosts
Human Immunodeficiency Virus (HIV) is a virus that exercise performance through increase in oxygen
causes AIDS and sometimes even death amongst human transport capacity following regular administration of
beings by weakening the immune system and its ability to EPO.
protect the body from diseases. In such a case even, small Its abuse can thicken the blood, leading to clotting,
infections can prove deadly. heart attack and stroke. Repeated doses of EPO can also
Option 1 is correct: Although rare, blood, organs or stimulate the development of antibodies directed against
tissues from an infected person if transplanted into a EPO, which can result in anaemia.
healthy person leads to transmission of HIV. Option 4 is correct: Human Growth Hormone (HGH) is
Option 2 is correct: HIV is primarily transmitted a hormone naturally produced in the human body by the
through infected body fluids. So, sharing needles (tip pituitary gland in the brain. It is also a peptide hormone.
pierces skin and touches blood), syringes (contains the It spurs growth in children and adolescents. It also helps
blood which may remain stuck to the walls) and other to regulate body composition, body fluids, muscle and
injection equipment creates a possibility of transfer of bone growth, sugar and fat metabolism, etc.
infected blood from one to another. It is used as a PED because Athletes and bodybuilders
Option 3 is correct: The placenta in the uterus of a claim that hGH increases lean body mass and decreases
pregnant mother is the site of diffusion of blood, the fat mass and has “anti-aging” properties.

Workbook 156
.
SCIENCE & TECHNOLOGY

It causes harmful side effects like joint pain, carpal 172. Solution: (a)
tunnel syndrome, soft tissue swelling, enlarged breasts Exp) Option a is the correct answer.
in men, and an increased risk of diabetes and high blood
In every organism there is unidirectional flow of
pressure.
information from DNA to proteins which involves two
170. Solution: (d) steps, i.e., transcription and translation which is called
central dogma.
Exp) Option d is correct.
Option a is correct. Transcription is the process in which
Vaccines train your immune system to create antibodies,
the genetic information is transferred from DNA to RNA.
just as it does when it’s exposed to a disease. Vaccination
The process involves synthesis of complementary RNA
is a simple, safe, and effective way of protecting you
polynucleotide chain on DNA.
against harmful diseases, before you come into contact
with them. It uses your body’s natural defenses to build Option b is incorrect. When a segment of a chromosome
resistance to specific infections and makes your immune breaks away and reattaches itself with another chromosome
system stronger. is called translocation. When there is a mutual exchange
of segments between two chromosomes, it is called as
Statement 1 is incorrect. DNA and RNA vaccines both reciprocal translocation.
instruct cells in our body to produce a protein that induces
an immune response. Option c is incorrect. The process in which the
information present in genetic RNA is transferred to
Once inserted into host cells, the genetic material is read a single stranded complementary DNA strand which
by the cell’s own protein-making machinery and used to is then converted into double stranded DNA is called
manufacture antigens, which then trigger an immune reverse transcription.
response.
Option d is incorrect. If a segment of a chromosome
Both DNA and RNA vaccines are relatively easy to breaks away and attaches with another chromosome,
produce, but the manufacturing process differs slightly without mutual exchange, it is called Robertsonian
between them. translocation, which may result in decrease of
Statement 2 is correct. Unlike DNA vaccines, RNA chromosome number of the cell.
vaccines can be synthesised chemically, without the need
for any bacteria or cells. 173. Solution: (d)
RNA vaccines are easier to synthesize because this can Exp) Option d is correct.
be done chemically, from a template in the lab, without Research has found that many drugs produced by global
the need for any bacteria or cells. In both cases, vaccines pharmaceutical industry have the presence of N-Nitroso-
for different antigens could be manufactured within the dimethylamine or NDMA. These contaminated drugs
same facilities, further reducing costs. This is not possible include ranitidine, tuberculosis drugs (rifampin and
for most conventional vaccines. rifapentine), diabetes drugs (metformin and pioglitazone)
Statement 3 is correct. While mRNA vaccines cannot and sartan group of blood pressure drugs.
affect our genes, this is a potential risk with DNA vaccines. N-Nitroso-dimethylamine and many of its related
RNA vaccines encode the antigen of interest in messenger compounds together called nitrosamines are strong
RNA (mRNA) or self-amplifying RNA (saRNA). Because carcinogens among animals, and likely have the
of its transitory nature, there is zero risk of it integrating same effect in humans. Nitrosamines are often formed
with our own genetic material. whenever so-called nitrosating agents (compounds
containing oxidised nitrogen, for example) react with
171. Solution: (d) compounds called amines in an acidic environment.
Exp) Option d is the correct answer. Risks associated with NDMA: Through research, it was
Statement 1 is correct. Cloning techniques are the lab found that nitrosamines trigger cancers in multiple
methods used to produce offspring that are genetically species. Epidemiological studies hinted strongly that
same to the donor parent. Clones of adult animals are nitrosamines were carcinogenic in humans too. One of
made by a method called somatic cell atomic transfer. the most challenging aspects of controlling nitrosamines
Statement 2 is correct. Some clones already exist in in drugs is that they are toxic even at tiny levels.
nature. Single-celled organisms like bacteria make
174. Solution: (a)
exact copies of themselves each time they reproduce. In
humans, identical twins are similar to clones. They share Exp) Option a is the correct answer.
almost the exact same genes. Identical twins are created Food irradiation is a technology in which food products
when a fertilized egg splits in two. are subjected to a low dosage of radiation to treat them
Statement 3 is correct. Cloning eventually could bring for germs and insects, increasing their longevity and shelf
back extinct species such as the passenger pigeon. For life. It is the application of ionizing radiation to food
now, the technique holds promise for helping endangered in which energy is transmitted without direct contact
species including a Mongolian wild horse that was cloned in the targeted food. The radiation can be emitted by a
and born at a Texas facility. radioactive substance or generated electrically.

157 Workbook
.
SCIENCE & TECHNOLOGY

Statement 1 is correct. Gamma Irradiation is the process Assisted Reproductive Technology (ART) includes In
of exposing various products to ionizing gamma rays vitro fertilization-embryo transfer (IVF-ET), gamete
(similar to UV and X-rays) in a highly controlled manner. intrafallopian transfer (GIFT), zygote intrafallopian
It is done for a specific goal, such as elimination of the transfer (ZIFT), and frozen embryo transfer (FET).
virus, bacteria, pathogens, insects and other undesirable
organisms etc. It is used for inhibition of sprouting in
bulbs and tubers, insect disinfestation of cereals, pulses
and grains, microbial decontamination of dry spices
etc. for preservation/shelf life extension by applying pre-
determined radiation doses.
The isotope Cobalt-60 is the most common source of
gamma rays for irradiation processing for a variety of
applications, including sterilization, decontamination and
materials modification.
Statement 2 is incorrect. The Codex Alimentarius
Commission, the highest international body on food
standards, allows irradiation when “necessary to achieve
a legitimate technological purpose.”
The Joint Expert Committee on Food Irradiation (JECFI) Statement 1 is incorrect. In Invitro Fertilization, eggs
convened by the Food and Agricultural Organization are surgically removed from the ovary and fertilised with
(FAO), the International Atomic Energy Agency sperm outside the body in a Petri dish. The fertilized eggs
(IAEA), and the World Health Organization (WHO) (embryos) are then placed in the women’s uterus, thus
are of particular importance in the development of food bypassing the fallopian tubes. This technique is specially
irradiation in the international arena. beneficial where women have blocked or absent
Their declarations resulted in the development and fallopian tubes, or where men have low sperm counts.
eventual adoption of the Codex General Standard In the female reproductive tract, there is one ovary and
for Irradiated Foods stipulating both the general and one fallopian tube on each side of the uterus. Fertilization
technological requirements for the process, and its of eggs take place here.
associated Recommended Code of Practice for the Statement 2 is correct.
Operation of Radiation Facilities.
Gamete Intrafallopian Transfer is a three-step procedure
Codex has also developed a set of guidelines for irradiated that involves removing the eggs, combining them with
food labelling. The Codex General Standard for the sperm, and immediately placing them in the fallopian
Labelling of Pre-Packaged Foods contains provisions tubes, where the egg is fertilized. GIFT is an option only
for labelling of irradiated foods that a food which has if the woman has a healthy uterus and fallopian tubes. As
been treated with ionizing radiation shall carry a written there is no IVF procedure, a person does not have to
statement indicating such treatment in close proximity to choose which embryo to transfer.
the name of the food.
A specific advantage of the GIFT procedure is that the
Statement 3 is correct. Irradiation can serve purposes embryos reaching the uterine cavity at a potentially more
such as: appropriate (i.e., later) stage of development than with
• Delay of Sprouting and Ripening – to inhibit IVF-ET.
sprouting (e.g., potatoes) and delay ripening of fruit Statement 3 is correct.
to increase longevity.
In Zygote Intrafallopian Transfer (ZIFT) eggs are
• Sterilization – irradiation can be used to sterilize
removed and fertilized with the male partner’s sperm
foods, which can then be stored for years without
in vitro. Then, zygotes (fertilized eggs) are inserted into
refrigeration. Sterilized foods are useful in hospitals
the fallopian tubes. Success rates for ZIFT and GIFT are
for patients with severely impaired immune
about the same as for IVF.
systems, such as patients with AIDS or undergoing
chemotherapy. Foods that are sterilized by irradiation A frozen embryo transfer is the process of using a frozen
are exposed to substantially higher levels of treatment embryo from a previous IVF cycle. It’s gaining popularity
than those approved for general use. due to its safety and pregnancy success rates. During
this procedure, a woman undergoes the standard IVF
175. Solution: (d) procedure, but instead of transferring the embryos back
Exp) Option d is correct. into the uterus in the same cycle, they are frozen for use
Assisted reproductive technology (ART) refers to fertility in a future cycle.
treatments and procedures that can help with difficulties Statement 4 is incorrect. All the above techniques result
or an inability to conceive children. in a “biologically related” child.

Workbook 158
.
SCIENCE & TECHNOLOGY

176. Solution: (b) worse over time. As the disease progresses, people may
Exp) Option b is correct have difficulty walking and talking.
Birth Control is a way to prevent pregnancy. The Option 4 is correct The magnetic pulse field is being
contraceptive pill is a type of contraception that contains looked at for treating depression.
hormones that prevent pregnancy. Transcranial magnetic stimulation (TMS) is a non-
Option 1 is correct. Contraceptive pills are small doses invasive, drug free alternative treatment for major
of either progestogens or progestogen– Oestrogen depressive disorder. Studies have shown that as many
combinations which are orally administered. Emergency as 60% of people with treatment-resistant depression
contraceptive pills prevent pregnancy by preventing or experience an improvement of their symptoms with TMS
delaying ovulation and they do not induce an abortion. therapy.
Option 2 is incorrect. The copper-bearing intrauterine 178. Solution: (c)
device (IUD) is a small, flexible plastic frame with copper
sleeves or wire around it. Exp) Option c is correct.
It is different from the contraceptive pill. When the sound is reflected more than once and the
repeated reflection that results in the persistence of sound
A specifically trained health care provider inserts it into a is called multiple reflection. Stethoscope is a medical
woman’s uterus through her vagina and cervix.
instrument used for listening to sounds produced within
Option 3 is correct. Pills usually inhibit ovulation, which the body, mainly in the heart or lungs. In stethoscopes the
is a process of release of eggs from ovary, and implantation sound of the patient’s heartbeat reaches the doctor’s
as well as alter the quality of cervical mucus to prevent/ ears by multiple reflection of sound.
retard entry of sperms. Pills are very effective with lesser
Option a, b and d are incorrect
side effects and are well accepted by the females.
Doppler effect means the apparent change in the
Option 4 correct. Contraceptive pill doesn’t protect
the person from being infected by STDs, including frequency of the light observed by the observer due
HIV. In contrary, the latex condoms which is another to relative motion between the source of light and the
contraceptive method is the best protection from most observer.
STDs. Tyndall effect means the scattering of light by very small
particles suspended in a gas or liquid.
177. Solution: (c)
Principle of ultrasound -Ultrasound is an imaging
Exp) Option c is correct technique where high-frequency sound waves (2-15MHz)
Weak ion currents that travel along the nerve cells in our are used to generate an image.
body produce magnetic fields because an electric current
always produces a magnetic field. 179. Solution: (b)
Option 1 is correct The magnetic field inside the body Exp) Option b is correct.
forms the basis of obtaining the images of different body Ultrasounds are high frequency waves. Ultrasounds are
parts. able to travel along well defined paths even in the presence
This is done using a technique called Magnetic Resonance of obstacles.
Imaging (MRI). Analysis of these images helps in medical Statement 1 is correct. Ultrasonic welding uses
diagnosis. ultrasonic energy at high frequencies (20–40 kHz) to
Option 2 is correct. Mass spectrometry, also called mass produce low amplitude (1–25 μm) mechanical vibrations.
spectroscopy, analytic technique by which chemical These vibrations generate heat at the joint interface of
substances are identified by the sorting of gaseous ions in the parts being welded which resulted in melting of the
electric and magnetic fields according to their mass-to- thermoplastic materials and weld formation after cooling.
charge ratios. Statement 2 is correct. Ultrasonic waves are allowed
Clinical mass spectrometry uses the mass spectrometry to pass through the metal block and detectors are used
technology for diagnostic purposes. It is used to to detect the transmitted waves. If there is even a small
diagnose metabolism deficiencies, to determine whether defect, the ultrasound gets reflected back indicating the
biomarkers or enzymes are present, and for toxicology presence of the flaw or defect.
testing. Statement 3 is correct. An ultrasonic humidifier is
Option 3 is correct The Magnetic pulse fields is used to a device that uses a metal or ceramic “diaphragm” to
treat Parkinson’s disease. produce mist. The diaphragm vibrates against the water,
Pulsed Electromagnetic Field (PEMF) therapy have creating droplets that are blown out into your room as a
demonstrated success for Parkinsons Disease and other fine mist
degenerative neurologic conditions. Parkinson’s disease Statement 4 is correct. The process in which sound
is a brain disorder that leads to shaking, stiffness, and waves and echoes are used to determine objects in space
difficulty with walking, balance, and coordination. is called echolocation. Bats emit sound waves that lie in
Parkinson’s symptoms usually begin gradually and get the ultrasound frequency range from their mouth and

159 Workbook
.
SCIENCE & TECHNOLOGY

nose. These sound waves hit the objects around them its writing your name in cursive on a piece of paper,
and reflect, producing echoes. The bats then detect these document or contract, often with initials on each page
echoes; it helps them judge the object’s size, shape, and indicating the extent of what is being signed.
distance. These days ultrasonic echolocation is being
widely used in traffic management applications and for 182. Solution (d)
sorting and counting objects in an assembly line Exp) Option d is correct.
Statement 5 is incorrect. The Doppler effect (and not Option a is incorrect. Artificial intelligence (AI) is
ultrasound) can be used to calculate the speed of any wide-ranging branch of computer science concerned
automobile. Calculations of change of radio waves with building smart machines capable of performing
from rotating satellites, for example, are used in the tasks that typically require human intelligence. AI is an
navigation systems of marine and can also be used in interdisciplinary science with multiple approaches, but
radar surveillance to determine the speed of automobiles. advancements in machine learning and deep learning are
The Doppler Effect is also called the Doppler shift. It is creating a paradigm shift in virtually every sector of the
a change in the wavelength or frequency of a wave for a tech industry.
viewer who is walking relative to the wave source. Option b is incorrect. Quantum computing is the
use of quantum phenomena such as superposition and
2.5. Information Technology entanglement to perform computation. Computers that
perform quantum computations are known as quantum
180. Solution (a) computers.
Exp) Option a is correct. Option c is incorrect. Elon Musk Neuralink aims at
Statement 1 is correct. WiMAX would operate similar developing implantable brain–machine interfaces (BMIs).
to WiFi but at higher speeds. WiMAX has the ability to Option d is correct. Cognitive computing is the use of
provide service even in areas that are difficult for wired computerized models to simulate the human thought
infrastructure to reach and the ability to overcome the process in complex situations where the answers may be
physical limitations of traditional wired infrastructure. ambiguous and uncertain.
Statement 2 is incorrect. WiMAX does not receive its
signal from satellite in space. It receives its signal from 183. Solution (a)
internet provider and transmits it through towers. Exp) Option a is correct
Statement 1 is correct. Bharat QR code is an interoperable
payment acceptance solution that supports Visa,
MasterCard. Amex and RuPay cards & BHIM-UPI for
wider acceptance.
Statement 2 is incorrect. Bharat QR code does not require
any upfront expenditure as it uses a printed QR code and
customer uses mobile phone to make the payment.
Statement 3 is incorrect. Bharat QR code has been
developed by National Payments Corporation of India,
Master card and Visa. It has been devised based on the
181. Solution (d) directions set up by Reserve Bank of India.
Exp) Option d is correct 184. Solution (b)
Option a is incorrect. Click Wrap Signatures is used for Exp) Option b is correct.
mostly online purchases, where a simple tick in a box Statement 1 is incorrect. RFID tag uses a small electronic
is an acceptance of the terms and conditions contained; chip which is surrounded by antenna unlike barcoding
referred to somewhere on the website. A user is required which uses a pattern of black bars and white spaces, in
to click the box before the services can be consumed. which the information is coded.
Option b is incorrect. Electronic Signature (E-Signature) Statement 2 is incorrect. A tag can be read from up to
is a digital version of a wet signature, where many several feet away and does not, need to be within direct
countries have defined the legality and description of line-of-sight of the reader to be tracked.
uses for e-signatures. These are the most common form
of signatures used by organizations globally. Statement 3 is correct. Radio Frequency Identification
(RFID) technology is at the core of FASTag, the flagship
Option c is incorrect. Digital Signature is an initiative of the Ministry of Road Transport and Highways
authentication mechanism that enables a code to be (MoRTH) for implementing electronic toll collection
attached as a signature. It is required for certain specific across the national highways in India.
agreements, and issued through a certification agency.
Option d is correct. The wet signature is any physical 185. Solution (b)
mark on documents created by a person. In most cases Exp) Option b is correct.

Workbook 160
.
SCIENCE & TECHNOLOGY

What is Internet of Behaviour? Option d is incorrect. Tor is free and open-source


Data collection (BI, Big Data, CDPs, etc.) provides software for enabling anonymous communication.
valuable information about customer behaviours, It directs internet traffic through a free, worldwide,
interests and preferences, and this has been referred to volunteer overlay network consisting of more than seven
as the Internet of Behaviour (IoB). The IoB attempts to thousand relays in order to conceal a user’s location and
understand the data collected from users’ online activity usage from anyone conducting network surveillance or
from a behavioural psychology perspective. It seeks to traffic analysis.
address the question of how to understand the data, and 187. Solution (c)
how to apply that understanding to create and market new
products, all from a human psychology perspective. Exp) Option c is correct.
Statement 1 is incorrect. Smart Grid Technologies
Software company BMC has developed a health app for
like smart meters and field devices have left critical
smartphones that tracks diet, sleep patterns, heart rate or
infrastructures vulnerable to attack (and not increased
blood sugar levels. The app can alert to adverse situations
their security). Each one of these technology advances
in the user’s health and suggest behavioral modifications
creates a weak point in digital security. It’s no secret that
towards a more positive outcome. Health Passport (with
cyber attackers would love to take down the infrastructures
apps such as Aarogya Setu in India, and The Health that supply the nation’s electricity, oil or gas.
Code in China) and Social Distancing Technologies are
partners in this emerging health technology. Statement 2 is correct. Use of data mining for behavior
analysis is popular and is widely used to target social
Option a is incorrect. Virtual reality (VR) refers to media and online advertisements to the right set of
a computer-generated simulation in which a person audience. Interestingly, behavior analytics is being
can interact within an artificial three-dimensional increasingly explored to develop advanced cyber security
environment using electronic devices, such as special technologies. Behavioral analytics helps determine
goggles with a screen or gloves fitted with sensors. patterns on a system and network activities to detect
Option c is incorrect. Lidar stands for Light Detection potential and real-time cyber threats.
and Ranging. It is a remote sensing method that uses light Statement 3 is correct. Deep learning is being used to
in the form of a pulsed laser to measure ranges (variable analyze data such as logs, transaction and real-time
distances) to the Earth. These light pulses—combined communications to detect threats or unwarranted
with other data recorded by the airborne system — activities. It is a feature of artificial intelligence which is
generate precise, three-dimensional information about now being used in cyber security. To create an additional
the shape of the Earth and its surface characteristics. layer of information and authentication, AI comes into
Option d is incorrect. Quantum computing is the the picture.
use of quantum phenomena such as superposition and Statement 4 is correct. Sinkholing is a technique used to
entanglement to perform computation. Computers that redirect malicious traffic from its original destination to
perform quantum computations are known as quantum a server under the control of a defender, thus protecting
computers. your network from being disrupted by DDoS attacks or
botnets. The server which acts as the C&C (Command &
186. Solution (a) Control) of this traffic is called a sinkhole.
Exp) Option a is correct.
188. Solution (a)
Option a is correct. Network slicing allows operators
to separate the packet traffic layer from the control Exp) Option a is correct
layer, supporting multiple applications and services Statement 1 is correct. Deep fakes use a form of artificial
running in parallel for a range of users who require intelligence called deep learning to make images of fake
different levels of quality, latency, and bandwidth. This events, hence the name deep fake.
configuration has become an essential component Statement 2 is incorrect. Not just pictorial data but
of the overall 5G architectural landscape. This means audio can also be deep faked too, to create “voice skins”
that 5G systems will have many logical network slices, or “voice clones” of public figures.
or “fast-track lanes,” to support specific applications and
Statement 3 is incorrect. New techniques allow unskilled
customers. people (and not just technical specialists) to make deep
Option b is incorrect. Network virtualization is the fakes with a handful of photos; fake videos are likely to
process of combining hardware and software network spread. This can create all sorts of problems in the society
resources and network functionality into a single, at regional and national level.
software-based administrative entity.
189. Solution (b)
Option c is incorrect. A virtual private network (VPN)
extends a private network across a public network and Exp) Option b is correct.
enables users to send and receive data across shared The ‘SolarWinds hack’ is a cyber-attack recently
or public networks as if their computing devices were discovered in the United States. It has emerged as one of
directly connected to the private network. the biggest ever targeted attack against the US government,

161 Workbook
.
SCIENCE & TECHNOLOGY

its agencies and several other private companies. In fact, it foreign country can access the apps in a device located in
is likely a global cyber-attack. Instead of directly attacking India.
the federal government or a private organisation’s Option c is correct: Messaging and voice calling apps
network, the hackers target a third-party vendor which receive and parse data from multiple sources at various
supplies software to them. times and are hence highly vulnerable to such attacks.
190. Solution (b) Option d is incorrect: In an encrypted environment
Exp) Option b is correct. detection of zero click attack becomes harder because
the data packets sent or received are not visible.
Quantum Key Distribution (QKD) is primarily
a mechanism to undertake highly-secure 193. Solution (c)
communication. Quantum key distribution (QKD) is
Exp) Option c is correct.
a secure communication method which implements a
cryptographic protocol involving components of quantum Statement 1 is correct. CollabCAD is a collaborative
mechanics. The technology enables two communicating network, computer enabled software system, providing
sides to come up with random secret keys shared by both a total engineering solution from 2D drafting and
of them and known exclusively to them so only they can detailing to 3D product design.
use it to encrypt and decrypt messages, thus achieving Statement 2 is correct. The aim of this initiative is to
highly-secure communication. Defence Research and provide a great platform to students of Atal Tinkering
Development Organisation (DRDO) has successfully Labs (ATLs) across country to create and modify 3D
demonstrated communication between its two labs using designs with free flow of creativity and imagination. This
Quantum Key Distribution (QKD) technology. software would also enable students to create data across
191. Solution (b) the network and concurrently access the same design data
for storage and visualization.
Exp) Option b is correct.
Option a is incorrect: HAM Radio is a simple radio- 194. Solution (d)
antenna set that is used by amateurs to communicate Exp) Option d is correct.
using radio frequencies. The benefit of the amateur radio Malware, Smishing and Keylogger are examples of data
network is that we do not encounter communication stealing methods used by hackers.
hindrances. Since it works independent of conventional
systems, there is very less congestion and it is better 1. Malware: Malware is a malicious software that is
suited for use in communication during emergencies and written with the intent of compromising a system
national disasters. and stealing the data available on the system. These
programmes can perform a variety of functions some
Option b is correct: Used since pre independence era of which include stealing or deleting sensitive data,
(since 1921), a licence is required to operate HAM Radio modifying system’s core functionalities, and secretly
in India. Upon registration a callsign starting with ‘VU2’ tracking the victim’s activities.
is assigned to the operator. The licence comes in 2 grades
- Grade I and Grade II. 2. Smishing: Smishing is a form of phishing in which
someone tries to trick you into giving them your
Option c is incorrect: HAM Radio was first used in 1921 private information via a phone call or SMS message.
(one way) in India. Two way communication using HAM Smishing is becoming an emerging and growing
was started in 1922. Amateur Radio Licencing in India threat in the world of online security.
was closed upon the start of World War II in 1939, and
was later opened on a temporary basis at the end of the Phishing is a fake email masquerading as legitimate.
war (1946). Hackers create phishing emails through which they
intend to steal your confidential information like
Option d is incorrect: HAM Radio is extremely useful passwords and bank account details.
as it can access satellite frequencies across the globe
as well. This helps in using HAMs as a second line of 3. Keylogger: Keylogger is a specific form of spyware
communication during disasters. that simply records the keys you type and where you
type them. These logs are then sent to the attacker
192. Solution (d) who can analyse them to find your passwords, chats,
Exp) Option d is correct. credit card numbers and much more.
Option a is correct: Spywares like Pegasus has evolved 195. Solution (a)
from its earlier spear-phishing methods using text links Exp) Option a is correct
or messages to ‘zero-click’ attacks which do not require
any action from the phone’s user. This had made what was Option 1,2 and 3 is correct. Due to the new technologies,
without a doubt the most powerful spyware out there, spectrum and frequencies it
more potent and almost impossible to detect or stop. uses, 5G has several benefits over 4G;
Option b is correct: Zero click cyber-attacks are fully higher speeds, less latency, capacity for a larger number
remote-controlled attacks and provide access to the of connected devices, less interference and better
target in real time, for example an attacker located in a efficiency.

Workbook 162
.
SCIENCE & TECHNOLOGY

Option 4 is incorrect. The shorter wavelength means architecture. It has excellent extended range in buildings
5G can carry a lot of data much faster than 4G but it also and underground, and has enhanced network security &
means a much shorter range compared to 4G. The range reliability. The coverage of the network will be so vast
of 5G connectivity is not great as the frequency waves are
that it will not leave any dark patch within the boundary
only able to travel a short distance. Added to this, setback
is the fact that 5G frequency is interrupted by physical of India, from Kashmir &Ladakh to Kanyakumari, and
obstructions such as trees, towers, walls and buildings. from Gujarat to the North East, including the Indian seas.
Option 5 is incorrect. The costs related to the Statement 4 is correct. The network has very low power
development of 5G infrastructure or adaptations to consumption and low component cost.
existing cellular infrastructure will be high. This amount
will be further compounded by the ongoing maintenance 198. Solution (a)
costs needed to ensure the high-speed connectivity.
Exp) Option a is correct.
196. Solution (b)
Cryptocurrencies are created through a process called
Exp) Option b is correct.
mining. To mine digital coins, miners need to use high-
Statement 1 is incorrect. Fifth Generation cellular
end processors that will consume a lot of electricity.
technology (5G) has faster speed and lower latency than
that of fourth generation technology (4G). Latency is Crypto-jacking scripts do not use the victim’s data. But
the amount of time data takes to travel between its source they drain the CPU’s resources, which slows down the
and destination. system, increases electricity usage, and causes irreparable
Statement 2 is correct. The bandwidth frequency damage to the hardware.
required for 5th generation technology is higher than that
of 4th generation technology. 5G will support significantly 199. Solution (b)
greater data speed and ultra-low latency. Therefore, Exp) Option b is correct.
larger chunks of spectrum will be required for 5G as
compared to that is required for 4G. As analyzed by Data mining is a process used by companies to turn raw
Ericson, “deploying the 3.5 GHz and 26 GHz band on data into useful information. By using software to look
existing macro sites can provide a capacity improvement for patterns in large batches of data, businesses can learn
of approximately 10 times compared with the LTE systems
more about their customers to develop more effective
in low and mid bands”. Spectrum requirement for 5G
network lies around three key frequencies ranges: Sub-1 marketing strategies, increase sales and decrease costs.
GHz, 1-6 GHz and above 6 GHz. The choice of spectrum
among these ranges will depend on the capacity and 200. Solution (a)
coverage requirement of the region. Exp) Option a is correct
Statement 3 is incorrect. A 5G network can even function Statement 1 is correct: A quantum computer seeks to
as many separate networks – all at the same time. This
exploit the laws that govern the behaviour of atoms and
unique 5G technology is called network slicing. Slices of
the network can be tailored for a specific purpose and act subatomic particles. At that tiny scale, many laws of
as its own independent network. Each slice can optimize classical physics cease to apply, and the unique laws of
the characteristics that are needed for a specific service quantum physics come into play.
without wasting resources on things it doesn’t need. It’s
the smart 5G Core Network that makes slicing possible, Statement 2 is incorrect. In a quantum computer,
which also guarantees the connection and performance information is stored in quantum bits, or qubits. And a
that each slice was set up to deliver. qubit can be both 0 and 1 at the same time. In classical
computers, bits of information are stored as either 0 or 1.
197. Solution (d)
Exp) Option d is correct. 201. Solution (d)
Statement 1 is correct. BSNL has launched the world’s Exp) Option d is correct.
first satellite-based narrowband IoT (Internet of Things)
network in partnership with Skylotec India. Narrowband Option d is correct. Edge computing is a distributed
Internet of Things (IoT) is a standards-based low power computing paradigm that brings computation and
wide area (LPWA) technology developed to enable a wide data storage closer to the location where it is needed,
range of new IoT devices and services. This new ‘Made
to improve response times and save bandwidth. Edge
in India’ Solution, which is indigenously developed, will
connect with BSNLs satellite- ground infrastructure and computing enables data to be analyzed, processed, and
provide PAN-India coverage, including Indian seas. transferred at the edge of a network. It means that the data
Statements 2 and 3 are correct. One of its advantage is analyzed locally, closer to where it is stored, in real-time
is its easy deployment into existing cellular network without latency.

163 Workbook
.
SCIENCE & TECHNOLOGY

managing the opening and closing of extra lanes, and, in


future, managing autonomous car flows.
In-hospital patient monitoring: An edge on the hospital
site could process data locally to maintain data privacy.
Edge also enables right-time notifications to practitioners
of unusual patient trends or behaviors (through analytics/
AI), and creation of 360-degree view patient dashboards
for full visibility.
Smart grid: Sensors and IoT devices connected to an edge
platform in factories, plants and offices are being used to
monitor energy use and analyse their consumption in
real-time. With real-time visibility, enterprises and energy
companies can strike new deals, for example where high-
powered machinery is run during off-peak times for
electricity demand. This can increase the amount of green
Option a is incorrect. Reservoir computing is a energy (like wind power) an enterprise consumes.
framework for computation derived from recurrent
neural network theory that maps input signals into higher 203. Solution: (d)
dimensional computational spaces through the dynamics Exp) Option d is correct.
of a fixed, non-linear system called a reservoir. Big data is a term applied to data sets whose size or type
Option b is incorrect. Sentient computing is a form of is beyond the ability of traditional relational databases to
ubiquitous computing which uses sensors to perceive its capture, manage and process the data with low latency.
environment and react accordingly. A common use of Big data has one or more of the following characteristics:
the sensors is to construct a world model which allows high volume, high velocity or high variety. Artificial
location-aware or context-aware applications to be intelligence (AI), mobile, social and the Internet of Things
constructed. (IoT) are driving data complexity through new forms and
Option c is incorrect. Ubiquitous computing (or sources of data.
“ubicomp”) is a concept in software engineering and Statement 1 is correct. Big Data simply refers to a large
computer science where computing is made to appear amount of data which is of structured, semi-structured
anytime and everywhere. In contrast to desktop or unstructured nature. The data pool is so voluminous
computing, ubiquitous computing can occur using any that it becomes difficult for an organization to manage
device, in any location, and in any format. and process it using traditional databases and software
202. Solution (d) techniques. Therefore, big data not only implies the
enormous amount of available data but it also refers to
Exp) Option d is correct. the entire process of gathering, storing, and analyzing that
Edge computing enables data to be analysed, processed data.
and transferred at the edge of a network. The idea is to
Statement 2 is correct. The Internet of Things,
analyse the data locally closer to where it is stored, in
the Internet, and the rapid development of mobile
real-time without latency rather than send it far away to a
communication networks have spawned big data
centralised data centre.
problems and have created problems of speed, structure,
Application of Edge Computing can be realised in the volume, cost, value, security privacy, and interoperability.
following areas: Traditional IT processing methods are impotent when
Autonomous vehicles: With edge computing, it will faced with big data problems, because of their lack of
be possible to remove the need for drivers in all trucks scalability and efficiency. Big Data problems need to be
except the front one, because the trucks will be able to solved by Cloud computing technology, while big data
communicate with each other with ultra-low latency. can also promote the practical use and implementation of
Virtualised radio networks and 5G (vRAN): Operators Cloud computing technology. There is a complementary
are increasingly looking to virtualise parts of their mobile relationship between them.
networks (vRAN). This has both cost and flexibility Statement 3 is correct. With a population of 1.2 billion, the
benefits. The new virtualised RAN hardware needs to do relevance of Big Data becomes all the more pronounced
complex processing with a low latency. Operators will for India. Recently, NITI Aayog also echoed the idea of
therefore need edge servers to support virtualising their evidence-based policymaking guided by Big Data. NITI
RAN close to the cell tower. Aayog is currently working on a plan in collaboration
Traffic management: Edge computing can enable more with the private players to develop the ‘National Data &
effective city traffic management. Examples of this include Analytics Platform’, which will act as a single source of
optimising bus frequency given fluctuations in demand, sectorial data for citizens, policymakers, and researchers.

Workbook 164
.
SCIENCE & TECHNOLOGY

204. Solution: (a) quality. This method is of particular interest when dealing
Exp) Option a is correct. with large heterogeneous data with an unknown number
of clusters. It helps in reducing the time complexity by
Emerging technologies are science-based innovations
with the potential to create a new industry or transform an high parallelism features of DNA.
existing one which will substantially alter the business and 206. Solution: (b)
social environment along with emerging new cognitive
computing models. Exp) Option b is correct.
Cognitive computing, a subfield of artificial intelligence, A QR code (Quick Response code) is a type of matrix
simulates human thought processes in machines using barcode (or two-dimensional barcode invented in 1994
self-learning algorithms through data mining, pattern by the Japanese automotive company Denso Wave. A
recognition, and natural language processing. barcode is a machine-readable optical label that can
contain information about the item to which it is attached.
Statement 1 is correct. The goal of cognitive computing
is to simulate human thought processes in a Option a is correct. QR code contains information
computerized model. Using self-learning algorithms that in both a horizontal and a vertical direction, hence
use data mining, pattern recognition and natural language the name “2-dimensional code.” Due to this structural
processing, the computer can mimic the way the human difference, a QR code contains a hundred times more
brain works. information than a barcode and has a greater potential to
store more information in a smaller space than a barcode.
Statement 2 is incorrect. The personal digital assistants
we have on our phones and computers now (Siri and Option b is incorrect. One of the biggest differences
Google among others) are not true cognitive systems; comes in terms of the differences between their designs or
they have a pre-programmed set of responses and can simply how they look. This also applies to the design of the
only respond to a preset number of requests. two codes. A barcode (and not QR code) only contains
information in a horizontal direction.
205. Solution: (d) Option c is correct. QR code, is similar to a barcode. Like
Exp) Option d is correct. barcode, it also contains machine-readable information
DNA computing is a branch of biomolecular computing about the object it is attached to. Unlike a normal barcode,
concerned with the use of DNA as a carrier of information however, a QR code is two-dimensional, i.e., it contains
to make arithmetic and logic operations. information in both the vertical and horizontal directions.
Statement 1 is correct. DNA based computing Option d is correct. Adaptation and error corrections
uses biological molecule instead of silicon chip for are less in the QR codes. This is by far the most desirable
computing. So (A)(T)(G)(C) will be used Instead of 0 and advantage of QR codes over conventional barcodes. QR
1used in computer processing. In the field of DNA-based codes have an error rate of 7-30%. Simply put, even if the
computing, the DNA contains the information, but the packaging of the product in question or the printed
molecules are floating around in solution. code is damaged or soiled, the QR code works. This
feature is smartly used by companies and businesses.
Statement 2 is correct. DNA computing can be used
Because of the error-correction feature, they can (and do)
in cryptography. Deploying DNA algorithms in
put a small logo or a picture within the code to make it
cryptography to build an intrusion detection model is
more associable to the business in question.
the most recent development of DNA computing model.
The ability to store 108 terabytes of data in 1 gram of DNA
has led to the potential holding a huge one-time pad.
Statement 3 is correct. Another application of DNA
computing is in DNA steganography, in which a novel
method was used to hide the messages in a microdot.
Instead of the traditional binary encoding, each letter
was denoted by three chemical bases i.e., the letter A
was encoded by CGA. These messages are then encoded
into DNA sequences and concealed by mixing it in a tube
with a large amount of sonicated random human DNA.
This led to the formation of microdots, which was then
decoded by the receiver with appropriate primers (short
sequence with complementary bases).
Statement 4 is correct. Clustering deals with deriving QR code and Bar code
highly meaningful relationships in a complex collection
of data by creating a structure using various concepts 207. Solution: (d)
and algorithms. DNA- based clustering involves using Exp) Option d is correct.
strands to assign edges and vertices. Iterative calculations
Pegasus spyware has evolved from its earlier spear-
are performed for every produced cluster to improve
phishing methods using text links or messages to ‘zero-

165 Workbook
.
SCIENCE & TECHNOLOGY

click’ attacks which do not require any action from the


phone’s user. It is the worrying aspect of spyware.
Statement 1 is incorrect. A Zero-Click attack helps
spyware like Pegasus gain control over a device without
human interaction or human error. Spear Phishing
is a social engineering attack where a hacker sends a
fraudulent message which is designed to trick a victim
into revealing confidential information or infect their
device with malicious software.
Statement 2 is incorrect. Zero-click attacks are hard
to detect given their nature and hence even harder to
prevent. Detection becomes even harder in encrypted
environments, where there is no visibility on the data
packets being sent or received.
Zero-click attacks occur only when an attacker is able to
take over a device remotely after successfully exploiting
vulnerabilities in the software and hardware of the phone
209. Solution: (d)
208. Solution: (b)
Exp) Option d is the correct answer.
Exp) Option b is correct.
Statement 1 is incorrect. Sonar stands for Sound
Machine learning (ML) is a type of artificial intelligence Navigation and ranging. It transmits sound waves that are
(AI) that allows software applications to become more then returned in form of echoes which are used to analyse
accurate at predicting outcomes without being explicitly various qualities or attributes of the target or object.
programmed to do so. Machine learning algorithms use Statement 2 is correct. LiDAR technology uses light
historical data as input to predict new output values. pulses or laser beams to determine the distance between
Statement 1 is incorrect. Machine learning is an the sensor and the object. The laser travels to the object
application and subset of AI (Artificial Intelligence) and is reflected back to the source and the time taken for
the laser to be reflected back is then used to calculate the
that provides a system with the ability to learn from its
distance.
experiences and improve accordingly without someone
physically programming those changes into it. Machine Statement 3 is incorrect. RADAR technology uses
electromagnetic waves or radio signals to determine the
learning lets systems and devices improve without being
distance and angle of inclination of objects on the surface.
programmed to that particular level. It makes use of data
for training so that it can find some accurate results. 210. Solution: (a)
Deep learning is basically a subset of machine learning Exp) Option a is the correct answer.
that relates the recurrent neural networks and artificial
The Chandrasekhar Limit of 1.4 solar masses, is the
neural networks together. Its algorithms are exactly like theoretical maximum mass a white dwarf star can
machine learning. have and still remain a white dwarf (though this limit
Statement 2 is correct. The primary aim of machine does vary slightly depending on the metallicity). Above
learning is to develop computer programs that this mass, electron degeneracy pressure is not enough
access the required data and utilize it for learning to prevent gravity from collapsing the star further into
by themselves. The process of learning begins with a neutron star or black hole.The limit is named after the
observations or data, such as examples, direct experience, Nobel laureate Subrahmanyan Chandrasekhar, who first
or instruction, in order to look for patterns in data and proposed the idea in 1931.
make better decisions in the future based on the examples 211. Solution: (a)
that we provide. Exp) Option a is correct.
Statement 3 is correct. There are number of applications Hyperspectral imaging, like other spectral imaging,
of the machine learning. One of the most exciting collects and processes information from across the
applications of machine learning is self-driving cars. electromagnetic spectrum. The goal of hyperspectral
Machine learning plays a significant role in self-driving imaging is to obtain the spectrum for each pixel in the
cars. Tesla, the most popular car manufacturing company image of a scene, with the purpose of finding objects,
is working on self-driving car. It is using unsupervised identifying materials, or detecting processes.
learning method to train the car models to detect people Statement a is correct. Hyper spectral imaging (HSI) is a
and objects while driving. technique that analyses a wide spectrum of light instead

Workbook 166
.
SCIENCE & TECHNOLOGY

of just assigning primary colours (red, green, blue) to not get a chance to melt under such ultrafast time scale
each pixel. The light striking each pixel is broken down interactions.
into many different spectral bands in order to provide
more information on what is imaged. 213. Solution: (d)
Statement b is incorrect. The algorithms and the image Exp) Option d is correct.
processing methodologies associated with HSI are a Laser is an acronym for “light amplification by the
product of military research, and were primarily used stimulated emission of radiation.”
to identify targets and other objects against background Statement a is correct. Laser is a device that stimulates
clutter. In the past, HSI has seen civil applications, and atoms or molecules to emit light at particular
has particularly been useful in satellite technology. It wavelengths. It amplifies that light, typically producing
might become an inexpensive, promising, and quick a very narrow beam of radiation. The emission generally
tool for the assessment of tissue conditions at diagnosis covers an extremely limited range of visible, infrared, or
and during surgery. The medical applications include ultraviolet wavelengths.
forensics, detection of colorectal and gastric cancer.
Statement b is correct. Helium-neon lasers were the first
Statement c is incorrect. In HSI, the unique colour lasers with broad commercial applications. Because they
signature of an individual object can be detected. Unlike could be adjusted to generate a visible red beam instead
other optical technologies that can only scan for a of an infrared beam, they found immediate use projecting
single colour, HSI is able to distinguish the full colour straight lines for alignment, surveying, construction, and
spectrum in each pixel. Therefore, it provides spectral irrigation. Soon eye surgeons were using pulses from
information in addition to 2D spatial images. ruby lasers to weld detached retinas back in place without
Statement d is incorrect. Hyper spectral data cubes can cutting into the eye.
contain absorption, reflectance, or fluorescence spectrum Statement c is correct. Lasers do not occur in nature.
data for each image pixel. It is assumed that HSI data is However, we have figured ways to artificially create this
spectrally sampled at more than 20 equally distributed special type of light.
wavelengths. The spectral range in hyper spectral data
can extend beyond the visible range (ultraviolet, Statement d is incorrect. Lasers produce a narrow beam
infrared). of light in which all of the light waves have similar
wavelengths. This is why laser beams are very narrow,
212. Solution: (c) very bright, and can be focused into a very tiny spot.
Exp) Option c is the correct answer. 214. Solution: (c)
Ever since the advent of the first LASER (Light Exp) Option c is correct.
Amplification by Stimulation Emission of Radiation) in
1960, there has been a steady increase in the application Each generation of wireless broadband is defined as a
of lasers. Applications have kept on becoming more set of telephone network standards that describe the
and more diverse as the capabilities of the lasers have technological implementation of the system.
increased. Statement 1 is correct. 5G has a new interface, which uses
Statement 1 is correct: The ability to focus laser beams millimeter wave spectrum (30-300 GHz), which enables
onto very small spots and to switch them on and off more devices to be used within the same geographic
billions of times per second makes lasers important area; 4G can support about 4,000 devices per square
tools in telecommunications and information kilometre, whereas 5G will support around one million.
processing. For example: In laser supermarket scanners, This means more Netflix streaming, voice calls and You
a rotating mirror scans a red beam while clerks move Tube carried, without interruption, over the limited air
packages across the beam. space.
Statement 2 is correct: Lasers are useful in the process Statement 2 is incorrect. There are three broad categories
of Microscopy. Laser microscopy is a class of techniques of 5G bands– the low, the mid and the high and all three
for generating microscopic images. Scanning may be have their own list of pros and cons. The 6G spectrum
achieved by moving either the laser beam or the sample. will land somewhere around 100GHz to 3THz. This high
Laser scanning microscopy is used in biological research frequency will be capable of delivering a wireless internet
to obtain high-resolution, high-contrast imagery of a of 5gpbs to 10gbps, it’s a massive jump from a 5G network
sample. Laser microscopes can scan samples point by which is expected to deliver 500mbps to 1gbps speed.
point, resulting in optical sectioning that can even be Statement 3 is correct. 6G (sixth-generation wireless) is
used to construct precise 3D imagery. the successor to 5G cellular technology.
Statement 3 is incorrect: Laser machining is not It seeks to utilize the terahertz band of
dependent on the material hardness but on the optical frequency which is currently unutilized.
properties of the laser and the optical and thermo- Terahertz waves fall between infrared waves and
physical properties of the material. Several recent pieces microwaves on the electromagnetic spectrum. 6G speed
of research have shown that laser cutting is best achieved is expected to be 100 times faster than 5G with enhanced
with ultrafast lasers, as the material only ablates and does reliability and wider network coverage. 6G internet will

167 Workbook
.
SCIENCE & TECHNOLOGY

be to support one microsecond-latency communication significantly less energy than classic Bluetooth—hence
(delay of one-microsecond in communication). This is the name Low Energy. The hidden difference is that
1,000 times faster - or 1/1000th the latency - than one Bluetooth Low Energy remains in sleep mode unless
millisecond throughput in compare to 5G. a connection initiates. The actual connection times
only last a few milliseconds, unlike Bluetooth, which
215. Solution: (c) connects for a few seconds or a few hours at a time.
Exp) Option c is correct. These short connections are necessary because data rates
Near field communication (NFC), Radio-frequency are significantly higher (1 Mb per second). It is radio
identification (RFID) and Bluetooth Low Energy (BLE) technology that allows devices to be networked within a
are modern communication technologies that allow one distance of about 10 meters.
to automatically identify objects or people and due to their BLE is not voice compatible.
different operating characteristics, complete each other.
Statement 1 is correct. RFID stands for ‘Radio-frequency
identification’ where digital data is encoded in RFID tags
or smart labels which is captured by a reader. It is similar
to barcoding where data from a tag is captured by a device.
The disadvantage with methods like a bar-code is that it
needs to be in the line of sight to be read whereas RFID
tags need not be in line of sight.

216. Solution: (a)


Exp) Option a is correct.
Statement 1 is correct. RADAR can detect objects at
long distance and through fog or clouds. But its lateral
resolution is limited by the size of the antenna. Unlike
RADAR technology, LiDAR pulses are adversely affected
Statement 2 is incorrect. Near field communication by atmospheric weather conditions such as dense fogs,
smoke, and even rain. The light pulses will be distorted
(NFC) is the set of protocols that enables electronic
during flight and this will affect the accuracy of the data
devices to establish radio communication with each other
collected.
by touching the devices together or bringing them into
proximity to a distance of typically 10cm or less. It works Statement 2 is incorrect. The difference between LiDAR
by the principle of induction. and RADAR is their wavelength. LIDAR uses short
wavelength as it is based on light to detect small objects.
NFC is also based on the RFID protocols. The main While Radar uses radio waves of longer wavelength
difference to RFID is that an NFC device can act not only and hence does not allow the detection of small objects.
as a reader, but also as a tag (not the other way round). RADAR cannot provide the user with the precise image
In peer-to-peer mode, it is also possible to transfer of an object because of the longer wavelength. The
information between two NFC devices. wavelength of RADAR is between 30 cm and 3 mm,
while LiDAR has a micrometer range wavelength.
217. Solution: (c)
Exp) Option c is correct.
A computerized tomography (CT) scan combines a series
of X-ray images taken from different angles around the
body. It uses computer processing to create cross-sectional
images (slices) of the bones, blood vessels and soft tissues
inside your body. CT scan images provide more-detailed
information than plain X-rays do.
Statement 1 is correct. Chest CT scan is a more detailed
type of chest X-ray that compounds the power of X-rays
and computers to deliver a 3D view of your chest. A CT
scan makes a number of detailed slices (pictures), which
then are merged into one projection. This creates a clear
Statement 3 is correct. Bluetooth Low Energy is for view that shows the position, shape, and size of the organs
specific use cases with limited data transfer and requires in your chest.

Workbook 168
.
SCIENCE & TECHNOLOGY

A CT scan produces detailed images of organs, bones, Pair 3 is correctly matched. Chandrayaan-1: ISRO’s
soft tissue and blood vessels and can be used to more maiden mission to Moon, the Chandrayaan-1, has been
easily diagnose cancer, heart disease, appendicitis, an exemplary example of international cooperation with
musculoskeletal disorders, trauma and infectious diseases. its international payloads. It has also earned several
national and international laurels and was instrumental
A chest X-ray helps to indicate abnormal formations or a
in the ISRO-NASA joint discovery of water molecules
large variety of chest diseases such as pneumonia, cystic
on the moon surface, unattained by any of the previous
fibrosis, emphysema, cancer, etc. In addition, a chest missions of such nature.
X-ray is often used for an emergency diagnosis due to its
fast and easy usage. It provides a 2-dimensional view of Pair 2 is incorrectly matched. NISAR: ISRO and NASA
the chest. are realizing a joint satellite mission called NISAR (NASA
ISRO Synthetic Aperture Radar) for earth science
Statement 2 is correct. Computed tomography (CT studies. The mission will observe Earth and measure its
scan) contains 5 to 10 times more ionizing radiation changing ecosystem and masses globally. It is the world’s
than X-ray. CT scans are an important component of most expensive imaging-satellite and the two space
Covid diagnosis as well as determining the severity of agencies intend to launch the satellite by 2022.
the disease. CT scan is far more sensitive than saturation Pair 4 is incorrectly matched. TRISHNA: ISRO and the
monitoring by pulse oximetry in detecting lung damage, French space agency CNES have partnered in developing
and can help timely administration of steroids. advanced upgradation satellites like TRISHNA to monitor
the water cycle to help in finding out proper ways to
218. Solution: (c) utilize it.
Exp) Option c is correct.
220. Solution (a)
Statement 1 is correct: Visible Light Communication
(VLC) systems employ visible light for communication Exp) Option a is correct.
that occupy the spectrum from 380 nm to 750 nm Statement 1 is correct. Vikas belongs to family of liquid
corresponding to a frequency spectrum of 430 THz to 790 fuelled rocket engines conceptualized and designed by
THz. ISRO’s Liquid Propulsion Systems Centre in the 1970s.
Statement 2 is incorrect: VLC is a communication Statement 2 is correct. It is the workhorse liquid rocket
technology in which the visible spectrum is modulated to engine powering second stage of India’s Polar Satellite
transmit data. Due to the limited propagation distance Launch Vehicle (PSLV), second stage and four strap-on
of the light emitting diodes (LEDs), VLC is a short- stages of Geosynchronous Launch Vehicle (GSLV) and is
range (and not long-range) wireless communication part of first stage i.e., twin engine core liquid stage (L110)
technology. of GSLV Mk-III.
Statement 3 is incorrect. The early production VIKAS
Statement 3 is correct: Aside from the massive size of
engines used imported French components which were
the visible light spectrum, in comparison to entire radio
later replaced by domestically produced equivalents.
spectrum, light travels 300,000 Km per second, which ISRO and the French Space agency signed a deal to co-
is way faster than the 344 meters per second traveled develop a 60-ton liquid-fuel engine. While the engine was
by radio waves in air. This means that communication not an all-new model, it was an improvised version of a
using light is virtually instantaneous, which also makes smaller French engine in the Viking series. The French
VLC the fastest means of communication among those had named their engine Viking, but the Indian-side, led
commercially available in the market. Thus the speed and by Nambi Narayanan, that negotiated the contract had
size of data transfer is faster than radio frequency based decided on an Indian name - Vikas.
Bluetooth technology.
221. Solution (c)
Statement 4 is correct: Unlike radio frequency (RF)
communication systems, VLC systems operate at optical Exp) Option c is correct.
frequencies and emit no electromagnetic interference. Option a is incorrect. Bio-synthesizer is a new
Therefore, they can be safely deployed in hospitals and instrumentation concept for robots for complex
healthcare facilities without any concerns on possible laboratory automation on low volume scale.
interference to medical instruments. Option b is incorrect. Pseudo-capsule is a structure
similar to a capsule, that surround some carcinomas.
2.6. Astronomy and Space Option c is correct. Bio-capsules is a technology
developed by Indian Institute of Spices Research
219. Solution (b)
(IISR), where a combination of selected beneficial
Exp) Option b is correct. microorganisms such as Trichoderma, Pseudomonas
Pair 1 is correctly matched. SARAL: The Indo-French and Bacillus is compacted into a package to give effects
joint mission, named SARAL (Satellite for ALTIKA of fertilizer. It is easy to storage, usage and transport.
and ARGOS) for studying the ocean from space using Option d is incorrect. Bio-Scaffolder is a kind of
altimetry was successfully launched in 2013. customizable 3D bioprinter.

169 Workbook
.
SCIENCE & TECHNOLOGY

222. Solution (a) only a theory.


Exp) Option a is correct Statement 3 is incorrect: India-based Neutrino
Option a is correct: Opposition is the event when Observatory (INO) is established at Pottipuram in
the sun, Earth and an outer planet are lined up, with Theni District of Tamil Nadu. Recently, it has received
the Earth in the middle. The time of opposition is the environmental clearances.
point when the outer planet is typically also at its closest
225. Solution (d)
distance to the Earth for a given year, and because it is
close, the planet appears brighter in the sky. Exp) Option d is correct.
Option b is incorrect: Solar eclipses happen when the Statement 1 is incorrect. China successfully powered up
New Moon comes between the Sun and Earth and blocks its “artificial sun” nuclear fusion reactor for the first time,
out the Sun›s rays. state media reported on Friday, marking a great advance
Option c is incorrect: A total lunar eclipse takes place in the country’s nuclear power research capabilities.
when the Earth comes between the Sun and the Full Statement 2 is correct. The HL-2M Tokamak reactor
Moon and blocks the Sun’s direct rays from lighting up is China’s largest and most advanced nuclear fusion
the Moon. experimental research device, and scientists hope that
Option d is incorrect: Equinox occurs when the sun is the device can potentially unlock a powerful clean
vertically above the equator. At this time, the sun equally energy source.
illuminates the Southern and Northern hemisphere. Statement 3 is correct. The power in the reactor is
At this time the world experiences equal day and night. generated by applying powerful magnetic field to a
Equinox occur twice a year. contained loop of hot plasma. The plasma can reach
temperature of more than 150-million-degree Celsius.
223. Solution (c)
Exp) Option c is correct. 226. Solution (a)
Option a is incorrect. A geosynchronous transfer orbit Exp) Option a is correct.
is a Hohmann transfer orbit — an elliptical orbit used to Statement 1 is correct. Polar-Areas Stellar-Imaging in
transfer between two orbits in the same plane — used to Polarisation High-Accuracy Experiment (PASIPHAE) is
reach geosynchronous or geostationary orbit. an international collaborative sky surveying project.
Option b is incorrect. Sun-synchronous orbit is a near Scientists aim to study the polarisation in the light coming
polar orbit in which the satellite passes over any given from millions of stars.
point of the planet’s surface at the same local mean solar The PASIPHAE survey will measure starlight polarisation
time. When a satellite has a sun-synchronous orbit, it over large areas of the sky.
means that the satellite has a constant sun illumination. Statement 2 is correct. PASIPHAE Project will use a
Option c is correct. Geostationary satellites are placed into novel polarimeter instrument known as WALOP (Wide
orbit at a distance of around 35,800 km from the earth’s Area Linear Optical Polarimeter). It is a polarimeter
surface. They rotate in the same direction as the earth mounted on two small optical telescopes. It will be used
and one revolution of such satellites is the same as one to detect polarised light signals emerging from the stars
day on earth. These satellites are used as communication along high galactic latitudes.
satellites and for weather-based applications. Orbital path Statement 3 is incorrect. Scientists from the University of
is circular. Crete, Greece, Caltech, USA, Inter-University Centre for
Option d is incorrect. Polar satellites revolve around the Astronomy and Astrophysics (IUCAA), India, the South
earth in a north-south direction around the earth. They African Astronomical Observatory and the University of
are very useful in applications where the field vision of the Oslo, Norway, are involved in this project, steered by the
entire earth is required in a single day. Institute of Astrophysics, Greece.
224. Solution (a) The project has been funded by the world’s leading
institutions. The Infosys Foundation, India, Stavros
Exp) Option a is correct. Niarchos Foundation, Greece and USA’s National Science
Statement 1 is correct: The scientists at INO would Foundation have each provided a grant of $1 million,
be keen to determine the mass of the neutrinos and combined with contributions from the European Research
ordering of the mass of 3 different types of neutrinos. Council and the National Research Foundation in South
(electron, muon and tau) Africa.
Statement 2 is incorrect: An underground laboratory 227. Solution (a)
for the study of neutrinos was created in 1951 in the
gold mines of Kolar in Karnataka, but it shut down when Exp) Option a is correct.
the mines closed in 1992. In fact, this laboratory was the Statement 1 is correct. NewSpace India Limited (NSIL),
first one to establish that the interaction of cosmic rays incorporated on 6 March 2019 (under the Companies Act,
with the atmosphere also generated neutrinos. Such 2013) is a wholly owned Government of India company,
neutrinos were detected in this laboratory in 1965 and under the administrative control of Department of Space
it was the first concrete proof of what was until then (DOS).

Workbook 170
.
SCIENCE & TECHNOLOGY

Statement 2 is correct. NSIL is the commercial arm of 229. Solution (b)


Indian Space Research Organisation (ISRO) with the Exp) Option b is correct.
primary responsibility of enabling Indian industries to
take up high technology space related activities. It is also Pair 1 is incorrectly matched. Arktika-M is a space satellite
launched to monitor the climate and environment in
responsible for promotion and commercial exploitation
the Arctic.
of the products and services emanating from the Indian
space programme. The Arktika-M will have a highly elliptical orbit that
passes high over northern latitudes allowing it to monitor
Statement 3 is incorrect. Building of Satellites (both
northern regions for lengthy periods before it loops back
Communication and Earth Observation) as per user
down under Earth.
requirements is also a mandate of NSIL.
The satellite will also be able to retransmit distress signals
The major business areas of NSIL include: from ships, aircraft or people in remote areas as part of
Production of Polar Satellite Launch Vehicle (PSLV) and the International Cospas-Sarsat satellite-based search and
Small Satellite Launch Vehicle (SSLV) through industry; rescue programme. Arktika-M is launched by Russia.
Production and marketing of space-based services, Pair 2 is correctly matched. Indian Space Research
including launch services and space-based applications Organisation (ISRO) launches the ‘’Sindhu Netra Satellite”.
like transponder leasing, remote sensing and mission It was a part of ISRO’s PSLV-C51 mission.
support services; The satellite is capable of automatically identifying
Transfer of technology developed by ISRO centres/ units the warships and merchant ships in the Indian Ocean
and constituent institutions of Dept. of Space; Region (IOR). It will be useful to identify suspicious
Marketing spin off technologies and products/ services movements of sea-faring vessels close to India’s maritime
emanating out of ISRO activities borders.
Consultancy services Pair 3 is correctly matched. Amazonia-1 is the optical
earth observation satellite of National Institute for Space
228. Solution (a) Research (INPE). This satellite will provide remote
Exp) Option a is correct. sensing data to users for monitoring deforestation
in the Amazon region and analysis of diversified
A space station is essentially a large spacecraft that agriculture across the Brazilian territory.
remains in low-earth orbit for extended periods of time.
It is like a large laboratory in space, and allows astronauts The sun-synchronous Amazonia-1 is the first satellite
to come aboard and stay for weeks or months to carry out to be fully designed, integrated, tested, and operated by
experiments in microgravity. Brazil’s National Institute for Space Research.
Statement 1 is correct. Russia has announced that it would India’s Polar Satellite Launch Vehicle PSLV-C51
be withdrawing from the International Space Station by successfully launched Amazonia-1 along with 18 co-
2025, and build and manage its own floating laboratory passenger satellites today (February 28, 2021) from Satish
that will be launched into orbit by 2030. Earlier, the Mir Dhawan Space Centre SHAR, Sriharikota
space station, functional from 1986 to 2001, was operated 230. Solution (d)
by Russia after the collapse of the former Soviet Union.
Exp) Option d is correct.
Statement 2 is correct. Tiangong is China’s new multi-
Statement 1 is incorrect. Sounding rockets are one or
module space station, which is set to be operational for at
two stage solid propellant rockets used for probing the
least 10 years. The space station will operate in low-Earth
upper atmospheric regions and for space research. They
orbit at an altitude of 340-450 km above Earth’s surface.
also serve as easily affordable platforms to test or prove
The T-shape station has one core module at the centre and prototypes of new components or subsystems intended
a lab capsule on each side. for use in launch vehicles and satellites.
Statement 3 is incorrect. NASA does not possess Statement 2 is incorrect. Currently there are operational
authority over the use of the International Space Station. sounding rockets in India. It includes three versions
The ISS is a collaboration of the five participating space namely RH-200, RH-300-Mk-II and RH-560-Mk-III.
agencies that have been running it: NASA (United These cover a payload range of 8 to 100 kg and an apogee
States), Roscosmos (Russia), JAXA (Japan), ESA range of 80 to 475 km.
(Europe), and CSA (Canada). The ownership and use of
the space station are established by intergovernmental 231. Solution (b)
treaties and agreements. Exp) Option b is correct.
Statement 4 is incorrect. It was announced in 2019, that Statement 1 is incorrect. A new intergovernmental
India will not join the International Space Station and organisation called Square Kilometre Array Observatory
will instead build a space station on its own by 2030. In (SKAO) Council has approved the building of the world’s
doing so, it will be joining the league of US, Russia, and largest radio telescope, Square Kilometre Array (SKA).
China to an elite space club. The Indian space station will The telescope will consist of a vast array of radio
be much smaller than the International Space Station and receivers and will be positioned across South Africa
will be used for carrying out microgravity experiments. and Australia.

171 Workbook
.
SCIENCE & TECHNOLOGY

India is participating in SKAO through the Department of Option d is incorrect. The Voyager program is an
Atomic Energy (DAE) and the Department of Science and ongoing American scientific program that employs two
Technology (DST).
robotic Interstellar probes, Voyager 1 and Voyager 2. They
Statement 2 is correct. SKA will allow astronomers to
look deeper into the universe and unravel secrets about were launched in 1977 to take advantage of a favorable
its evolution. Among its goals are: studying the universe alignment of Jupiter and Saturn, to fly near them while
and its evolution, the origin and evolution of cosmic collecting data for transmission back to Earth. After
magnetism, and dark energy and evolution of galaxies. It
will address some of the biggest questions about the nature launch the decision was taken to additionally send
of gravity, dark energy, and how galaxies formed and Voyager 2 near Uranus and Neptune to collect data for
subsequently evolved. Scientists are also optimistic that transmission back to Earth. As of 2021, the two Voyagers
the SKA will be able to detect very weak extra-terrestrial
signals and search for molecules that support life are still in operation past the outer boundary of the
heliosphere in interstellar space. They both continue to
232. Solution (d)
collect and transmit useful data to Earth.
Exp) Option d is correct.
Statement 1 is correct. The permanently shadowed 234. Solution (a)
craters of the South Polar Region of Moon are estimated
to hold nearly 100 million tons of water. Exp) Option a is correct.
Statement 2 is correct. Its craters have been untouched by Statement 1 is correct. The Union Government has
sunlight for billions of years — offering an undisturbed created the Indian National Space Promotion and
record of the solar system’s origins.
Authorization Centre (IN-SPACe) to provide a level
Its elemental and positional advantages make it a suitable
pit stop for future space exploration. playing field for private companies to use Indian space
Statement 3 is correct. Its regolith has traces of infrastructure.
hydrogen, ammonia, methane, sodium, mercury, and IN-SPACe will hand-hold, promote and guide the private
silver — making it an untapped source of essential
resources. industries in space activities through encouraging policies
and a friendly regulatory environment. It is set up to not
233. Solution (a)
only to oversee the space activity of the private sector,
Exp) Option a is correct.
but also to handhold and share ISRO facilities. It acts
Hope Probe or the Emirates Mars Mission is the first
Arab mission to Mars. It has been launched to provide a as a single window nodal agency for enabling and
complete picture of the Martian atmosphere and its layers. regulating space activities and usage of ISRO facilities
It is expected to reach the red planet’s orbit in 2021. It is by private entities. The decision of the body is binding on
an orbiter which will orbit the Mars for a full martial year
of 687 days. The spacecraft will orbit Mars to study the ISRO as well.
Martian atmosphere and its interaction with outer space Statement 2 is incorrect. It has been created under the
and solar winds. Hope will collect data on Martian climate
Department of Space and it functions autonomously to
dynamics, which should help scientists understand why
Mars’ atmosphere is decaying into space. Hope probe is ISRO. (not under Prime Minister’s Office)
the first interplanetary mission launched by any Arab
country. It is a mission of the United Arab Emirates 235. Solution (c)
(UAE). It is the first interplanetary mission for the Arab
Exp) Option c is correct.
World.
Option b is incorrect. Dragonfly is a planned spacecraft Statement 1 is correct. Lagrange Points are positions
and NASA mission, which will send a robotic rotorcraft to in space where the gravitational forces of a two-body
the surface of Titan, the largest moon of Saturn. It would system like the Sun and the Earth produce enhanced
be the first aircraft on Titan and is intended to make the
first powered and fully controlled atmospheric flight, regions of attraction and repulsion.
from vertical takeoffs to landing, on any planet’s moon, Lagrange points is a position where the gravitational pull
with the intention of studying prebiotic chemistry and
extraterrestrial habitability. of two large masses precisely equals the centripetal force
Option c is incorrect. Tianwen-1 is an interplanetary required for a small object to move with them.
mission by the China National Space Administration Statement 2 is incorrect. There are five Lagrange points
(CNSA) to send a robotic spacecraft to Mars, consisting
of an orbiter, deployable camera, lander and the Zhurong in the Earth- Sun system where a small mass can orbit in
rover. a constant pattern.

Workbook 172
.
SCIENCE & TECHNOLOGY

ASTHROS will make the first detailed 3D maps of the


density, speed, and motion of gas in these regions to gain
insight into how stellar feedback works. Stellar feedback
provides information about the formation of a star.
237. Solution (c)
Exp) Option c is correct.
Antarctic Impulsive Transient Antenna (ANITA) is
a radio telescope which is used to detect ultra-high
energy cosmic-ray neutrinos from a scientific balloon
flying over the continent of Antarctica. ANITA is the
first NASA observatory for neutrinos of any kind. It
includes an array of radio antennas attached to a helium
balloon which flies over the Antarctic ice sheet. Neutrinos
are high energy subatomic particles with no electric
charge and a very small mass. They are considered to
Statement 3 is correct. Lagrange Points are positions in be the second most abundant particle in the universe
space where the gravitational forces of a two-body system after photons or light particles. The ANITA instrument
like the Sun and the Earth produce enhanced regions of detects these ultra-high energy neutrinos by use of the
attraction and repulsion. These can be used by spacecraft Askaryan effect. (Askaryan effect predicts the production
to reduce fuel consumption needed to remain in of coherent radio emission from the cascade of particles
position. produced in a high energy particle interaction.)
Statement 4 is incorrect. The L1 point of the Earth-Sun
system affords an uninterrupted view of the sun and is 238. Solution (a)
currently home to the Solar and Heliospheric Observatory Exp) Option a is correct
Satellite. The third Lagrange point, L3, remains hidden
The event horizon is the threshold around the black hole
behind the Sun at all times, opposite Earth’s orbit. For
now, science has not found a use for this spot. where the escape velocity surpasses the speed of light.
The event horizon of a black hole is linked to the object’s
236. Solution (a) escape velocity — the speed that one would need to
Exp) Option a is correct. exceed to escape the black hole’s gravitational pull. The
Astrophysics Stratospheric Telescope for High Spectral closer someone came to a black hole, the greater the speed
Resolution Observations at Submillimeter-wavelengths they would need to escape that massive gravity.
(ASTHROS) is a new mission of NASA to send a Option B refers to what is called Escape Velocity.
telescope, on a football stadium-sized balloon, high into
the stratosphere to observe wavelengths of light invisible 239. Solution (c)
from the Earth. Exp) Option c is correct
Various aspects of this mission are- Gravitational waves are ripples in the fabric of spacetime
The mission will try to find answers about formation of produced by the violent phenomena like exploding
giant stars in the galaxy. stars and collisions between ultradense neutron stars or
The telescope mission called is likely to be launched in merging black holes. Gravitational waves are washing
December 2023 from Antarctica. It will spend about three over Earth all the time, but our instruments have not been
weeks drifting on air currents above the icy southern sensitive enough to detect them until very recently in
continent, said the American space agency on Wednesday. 2017 by the LIGO observatory.
Managed by NASA’s Jet Propulsion Laboratory, ASTHROS The visible universe—including Earth, the sun, other
will observe far-infrared light, or light with wavelengths stars, and galaxies, etc is made up of what we call “ordinary
much longer than what is visible to the human eye at an matter”. This makes up less than 5 percent of the mass of
altitude of about 130,000 feet (40 km) roughly four times the universe.
higher than commercial airliners fly. The rest of the universe appears to be made of a mysterious,
ASTHROS will carry an instrument to measure the invisible substance called dark matter (25 percent) and
motion and speed of gas around newly-formed stars. a force that repels gravity known as dark energy (70
During the flight, the mission will study four main percent).
targets, including two star-forming regions in the Unlike normal matter, dark matter does not interact with
Milky Way galaxy. the electromagnetic force. This means it does not absorb,
It will also for the first time detect and map the presence reflect or emit light, making it extremely hard to spot. In
of two specific types of nitrogen ions which can reveal fact, researchers have been able to infer the existence of
places where winds from massive stars and supernova dark matter only from the gravitational effect it seems to
explosions have reshaped the gas clouds. have on visible matter.

173 Workbook
.
SCIENCE & TECHNOLOGY

Usually astronomers believe that the expansion of 242. Solution (b)


the universe would slow down due to the combined Exp) Option b is correct.
gravitational pull of all the cosmos’ stars and galaxies.
Statement 1 is incorrect. India’s first manned space flight
However, in the 1990s, it was discovered that the
‘Gaganyaan’ is expected to send three persons into the
universe is still not only expanding but also the process of
space for seven days. The spacecraft will be placed in a
expansion has become swifter. Scientists now believe this
Low Earth Orbit (LEO) of 300-400 km above Earth’s
is happening due to some kind of force which they have
surface.
termed as dark energy.
Statement 2 is correct. ISRO’s heavy lift launcher, GSLV
240. Solution (b) Mk III will be used as a launch vehicle for the Gaganyaan
Exp) Option b is correct. mission. The high thrust solid propellant strap-on
boosters S200 play an important role in the human-rated
Statement 1 is incorrect. Polar orbiting satellites are in GSLV Mk III.
an orbital plane that is inclined at nearly 90 degrees to
the equatorial plane. This inclination allows the satellite Statement 3 is incorrect. The mission will make India the
to sense the entire globe, including the Polar Regions, fourth nation in the world to launch a Human Spaceflight
providing observations of locations that are difficult to Mission. The US, Russia and China are the only three
reach via the ground. Many polar-orbiting satellites are countries to have sent crews to space. Thus, it would
considered sun-synchronous, meaning that the satellite make India 4th and not 5th country to have sent crew to
passes over the same location at the same solar time each space if successful.
cycle. Almost all satellites used for remoted sensing and 243. Solution (a)
Geographic Information System (GIS) work are polar
orbiters. Exp) Option a is correct.
Statement 2 is correct. Geostationary (geosynchronous) Pair 1 is correctly matched. RISAT-2B is a radar imaging
satellites orbit 36,000 km over the equator. Geostationary earth observation satellite developed by ISRO. It was
satellites follow Earth’s rotation and travel at the same launched by PSLV-C46. Its main applications are disaster
rate of the rotation; because of this, the satellites appear management system and Earth observation.
to an observer on Earth to be fixed in one location. These Pair 2 is correctly matched. Cartosat-3 satellite is a 3rd
satellites capture the same view of Earth with each generation agile advanced satellite having high resolution
observation and so provide almost continuous coverage imaging capability for Earth observation. It was launched
of one area. They are used for weather forecasting, by PSLV-C47. Cartosat-3 will address the increased user’s
satellite TV, and communications. demands for large scale urban planning, rural resource
Statement 3 is incorrect. Low Earth Orbit (LEO) and infrastructure development, coastal land use and land
satellites do not have to always orbit along Earth’s cover etc.
equator. Satellites in Geostationary orbit have to always Pair 3 is incorrectly matched. CMS-01 (2020), launched
orbit along Earth’s equator. Low Earth Orbiting satellites by PSLV-C50, is a communication satellite envisaged for
do not always have to follow a particular path around providing services in Extended-C Band of the frequency
Earth in the same way – their plane can be tilted. This spectrum. The Extended-C Band coverage will include
means there are more available routes for satellites in LEO, Indian mainland, Andaman-Nicobar & Lakshadweep
which is one of the reasons why LEO is a very commonly Islands.
used orbit. It is the orbit most commonly used for satellite
imaging, as being near the surface allows it to take images 244. Solution (c)
of higher resolution. Exp) Option c is correct.
241. Solution (c) Statement 1 is incorrect. IRNSS will provide two types
Exp) Option c is correct. of services, namely, Standard Positioning Service (SPS)
which is provided to all the users and Restricted Service
Statement 1 is correct. The Hypersonic Technology (RS), which is an encrypted service provided only to
Demonstrator Vehicle (HSTDV) uses indigenously the authorised users. The IRNSS System is expected to
developed scramjet propulsion system. The scramjets
provide a position accuracy of better than 20 m in the
are a variant of a category of jet engines called the air
primary service area.
breathing engines. The ability of engines to handle
airflows of speeds in multiples of speed of sound, gives it a Statement 2 is incorrect. The IRNSS system consists of a
capability of operating at those speeds. Hypersonic speeds constellation of eight satellites and a supporting ground
are those which are five times or more than the speed of segment. Three of the satellites in the constellation
sound. will be placed in a geostationary orbit (GEO) and the
Statement 2 is correct. The indigenous development remaining five in a geosynchronous inclined orbit
of HSTDV technology will boost the development of (GSO).
the systems built with hypersonic vehicles at its core, Statement 3 is correct. On November 11, 2020, India
including both offensive and defensive hypersonic cruise became the fourth country in the world to have its
missile systems and also in the space sector. independent regional navigation satellite system (IRNSS)

Workbook 174
.
SCIENCE & TECHNOLOGY

recognised by the International Maritime Organisation 248. Solution (b)


(IMO) as a part of the World Wide Radio Navigation Exp) Option b is correct.
System (WWRNS).
Statement 1 is incorrect. The Starlink Network Project
245. Solution (b) is being developed by SpaceX, the world’s leading private
Exp) Option b is correct. company in space technology. It aims at providing low-
cost and reliable space-based internet services to the
The Artemis program of National Aeronautics and Space world.
Administration (NASA) will land the first woman and
next man on the Moon by 2024. Orion is a class of Statement 2 is correct. The Starlink satellites — the
partially reusable space capsules to be used in Artemis 12,000 for which SpaceX has permission, as well as the
program. It will use innovative technologies to explore other 30,000 that it wants to launch — will be deployed
more of the lunar surface than ever before. NASA will in the Low Earth Orbit (LEO) between 350 km to 1,200
collaborate with commercial and international partners km.
and establish sustainable exploration by the end of the Statement 3 is correct. The project aims to reduce the
decade. ARTEMIS stands for Acceleration, Reconnection, transmission latency (the time lag) for faster internet
Turbulence and Electrodynamics of the Moon’s Interaction services. A satellite in geostationary orbit has a latency of
with the Sun. Its one of the objectives is to measure what about 600 milliseconds. A satellite in the Low Earth Orbit,
happens when sun’s radiation hits our rocky moon, 200-2,000 km from the Earth’s surface, can bring the lag
when there is no magnetic field to protect it. down to 20-30 milliseconds, roughly the time it takes for
terrestrial systems to transfer data.
246. Solution (c)
Exp) Option c is correct. 249. Solution (a)
Correctly matched pairs are – Pair 1-B is correct. BeiDou, Exp) Option a is correct
or BDS, is a regional GNSS owned and operated by the Statement 1 is correct: PSLV is 4 stage Satellite Launch
People’s Republic of China. China is currently expanding Vehicle.
the system to provide global coverage with 35 satellites by Statement 2 is incorrect: It can take up to 1,750 kg of
2020. BDS was previously called Compass. payload to Sun-Synchronous Polar Orbits of 600 km
Pair 2-D is correct. GLONASS (Globalnaya altitude. PSLV has also been used to launch various
Navigazionnaya Sputnikovaya Sistema, or Global satellites into Geosynchronous and Geostationary orbits,
Navigation Satellite System) is a global GNSS owned and like satellites from the IRNSS constellation.
operated by the Russian Federation. The fully operational
system consists of 24+ satellites. 250. Solution (b)
Pair 3-C is correct. QZSS is a regional GNSS owned by Exp) Option b is correct
the Government of Japan and operated by QZS System Statement 1 is correct: India plans to ring in its own
Service Inc. (QSS). QZSS complements GPS to improve era of space-to-space tracking and communication of its
coverage in East Asia and Oceania. Japan plans to have space assets this year by putting up a new satellite series
an operational constellation of 4 satellites by 2018 and called the Indian Data Relay Satellite System.
expand it to 7 satellites for autonomous capability by 2023. Statement 2 is correct: IDRSS satellites of the 2,000
Pair 4-A is correct. Galileo is a global GNSS owned kg class would be launched on the GSLV launcher to
and operated by the European Union. The EU declared geostationary orbits around 36,000 km away
the start of Galileo Initial Services in 2016 and plans to Statement 3 is incorrect: The U.S. is putting up its third-
complete the system of 24+ satellites by 2020. generation advanced fleet of TDRS (Tracking & Data Relay
247. Solution (c) Satellites), Russia has its Satellite Data Relay Network and
Europe is building its own European Data Relay System.
Exp) Option c is correct. China is into its second generation Tianlian II series.
Statement 1 is correct. The scientific objectives of ISRO’s
Venus mission Shukrayaan are – (1) investigation of the 251. Solution (c)
surface processes and shallow subsurface stratigraphy Exp) Option c is correct.
(the study of rock layers); (2) solar wind interaction Statement 1 is correct. Solid Fuel Ducted Ramjet (SFDR)
with Venusian Ionosphere; (3) studying the structure, Technology: SFDR technology is a missile propulsion
composition and dynamics of the atmosphere. system based on the concept of Ramjet Engine principle.
Statement 2 is correct. The Indian Space Research The system utilizes a solid fuelled air-breathing ramjet
Organisation (ISRO) has short-listed 20 space-based engine. Unlike solid-propellant rockets, the Ramjet takes
experiment proposals for the proposed mission. These up oxygen from the atmosphere during flight. Thus, it is
include collaborative contributions from Russia, France, light in weight and can carry more fuel.
Sweden and Germany. The one already selected is France’s Statement 2 is correct. Successful demonstration of
VIRAL instrument (Venus Infrared Atmospheric Gas SFDR technology will enable India to develop indigenous
Linker). long range air-to-air missiles. Air-to-air missiles which

175 Workbook
.
SCIENCE & TECHNOLOGY

use SFDR technology can achieve longer ranges as they do 255. Solution (c)
not require oxidizers (that take oxygen from atmosphere).
Exp) Option c is correct.
The missile based on SFDR fly at supersonic speeds and
high maneuverability ensures the target aircraft cannot Amazonia-1 is the optical earth observation satellite
get away. of Brazil. PSLV-C51/Amazonia-1 is the first dedicated
252. Solution (d) commercial mission of NewSpace India Limited (NSIL),
Exp) Option d is correct. a Government of India company under Department of
Space. NSIL is the commercial arm of the Indian Space
Statement 1 is incorrect. A ramjet is a form of air-
breathing jet engine that uses the vehicle’s forward motion Research Organisation (ISRO).
to compress incoming air for combustion without a This satellite will provide remote sensing data to users
rotating compressor. for monitoring deforestation in the Amazon region and
Ramjets work most efficiently at supersonic speeds analysis of diversified agriculture across the Brazilian
around Mach 3 (three times the speed of sound) and can territory.
operate up to speeds of Mach 6. However, the ramjet
efficiency starts to drop when the vehicle reaches 256. Solution (a)
hypersonic speeds.
A scramjet engine is an improvement over the ramjet Exp) Option a is correct.
engine as it efficiently operates at hypersonic speeds Statement 1 is correct.
and allows supersonic combustion. Thus, it is known as
The Geosynchronous Orbit and Geostationary Orbit
Supersonic Combustion Ramjet, or Scramjet.
are defined on the basis of following conditions:
Statement 2 is incorrect. Ramjets cannot work in vacuum
as it is a form of air-breathing jet engine that uses the Condition 1: If the satellite is placed at an altitude of
vehicle’s forward motion to compress incoming air 35786 km above the Earth, where its velocity is such that
for combustion without a rotating compressor. Fuel is it takes exactly 23 hrs 56 minutes 4 sec to revolve around
injected in the combustion chamber where it mixes with
the Earth.
the hot compressed air and ignites.
Statement 3 is incorrect. Ramjets cannot produce thrust Condition 2: If the Orbital plane and the Equatorial plane
at zero airspeed; they cannot move an aircraft from a lies in the same plane throughout.
standstill. A ramjet-powered vehicle, therefore, requires If both the above conditions are satisfied, the satellite is
an assisted take-off like a rocket assist to accelerate it to a
speed where it begins to produce thrust. said to be placed in Geostationary orbit. If only condition
1 is satisfied, the orbit is called a Geosynchronous orbit.
253. Solution (d) Thus, all Geostationary orbits are Geosynchronous as
Exp) Option d is correct. well, while the vice versa is not true.
Nearly 70% of the propellant (fuel-oxidiser combination) The Geostationary satellites remain permanently fixed
carried by today’s launch vehicles consists of oxidiser. In
an air-breathing scramjet engine, air from the atmosphere with respect to a point on earth while the Geosynchronous
is rammed into the engine’s combustion chamber at a satellites have a varying position with respect to a point on
supersonic speed of more than Mach two. earth and wider coverage.
Mastering the air-breathing scramjet technology will Geosynchronous Transfer Orbit refers to an elliptical
lead to the development of hypersonic missiles, faster
Geosynchronous orbit from where the satellite is
civilian air transportation and facilities for putting
satellites into orbit at a low cost. transferred to a higher orbit or to Geostationary orbit
through orbit maneuvering.
254. Solution (c)
Statement 2 is correct. The Sun Synchronous Polar
Exp) Option c is correct.
Orbit or the Polar orbit refers to a Low Earth Orbit
Statement 1 is incorrect: ISRO’s Geosynchronous Satellite
Launch Vehicle GSLV Mk III, the three-stage heavy-lift (500-2000km) in the polar plane. From this orbit, when
launch vehicle, will be used to launch Gaganyaan as it has the satellite observes a given ground location, the sun is
the necessary payload capability. always at the same location in the sky.
Statement 2 is correct: The spacecraft will be placed in a Statement 3 is incorrect. The satellites in the LEO travel
low earth orbit of 300-400 km. It will consist of a service much faster (i.e., they have a higher orbital velocity) than
module and a crew module, collectively known as the
Orbital Module. the ones in higher orbits because the gravitational pull of
Statement 3 is correct: Russia will help with the selection Earth is highest in the LEO, satellites in the LEO are more
of astronauts, while France will help with its expertise in strongly pulled by the planet than those in the MEO or
space medicine. geostationary orbit.

Workbook 176
.
SCIENCE & TECHNOLOGY

making it extremely hard to spot. In fact, researchers have


been able to infer the existence of dark matter only from
the gravitational effect it seems to have on visible matter.
Statement 2 is correct. The dark energy is evenly
distributed throughout the universe. It does not have
any local gravitational effects, but rather a global effect
on the universe as a whole. This leads to a repulsive
force, which tends to accelerate the expansion of the
universe. Thus, Dark energy is the name given to the
mysterious force that’s causing the rate of expansion of
our universe to accelerate over time, rather than to slow
down. The rate of expansion and its acceleration can be
measured by observations based on the Hubble law. These
measurements, together with other scientific data, have
confirmed the existence of dark energy and provide an
estimate of just how much of this mysterious substance
exists.
257. Solution: (a) Statement 3 is incorrect. It turns out that roughly 68%
Exp) Option a is correct. of the universe is dark energy. Dark matter makes up
Geosynchronous Satellite Launch Vehicle (GSLV) is an about 27%. The rest - everything on Earth, everything
expendable space launch vehicle designed, developed, ever observed with all of our instruments, all normal
and operated by the Indian Space Research Organisation matter - adds up to less than 5% of the universe.
(ISRO) to launch satellites and other space objects into
259. Solution: (d)
Geosynchronous Transfer Orbits.
Exp) Option d is the correct answer.
Polar Satellite Launch Vehicle (PSLV) is the third
generation launch vehicle of India. It is the first Indian Statement 1 is correct: As the earth rotates anti-
launch vehicle to be equipped with liquid stages. clockwise i.e. from west to east. A satellite launched from
the sites near the equator towards the east direction will
Statement 1 is correct. GSLV has the capability to put get an initial boost equal to the velocity of Earth surface.
a heavier payload in the orbit than the Polar Satellite This initial boost aids in reducing cost of rockets used to
Launch Vehicle (PSLV). launch satellites.
PSLV can carry satellites up to a total weight of 2000 kg Statement 2 is incorrect: Atmospheric pressure is
into space and reach up to an altitude of 600-900 km. dependent on altitude and it has nothing to do with the
GSLV can carry weight up to 5,000 kg and reach up to direction. Furthermore, atmospheric pressure has little
36,000 km. to do with satellites being launched from the east coast.
Statement 2 and 3 is correct. PSLV is designed mainly to Statement 3 is incorrect: Orbital velocity is defined as
deliver earth observation or remote sensing satellites. the minimum velocity a body must maintain to stay in
Whereas, GSLV has been designed for launching orbit. Due to the inertia of the moving body, the body has
communication satellites. GSLV delivers satellites into a tendency to move on in a straight line. Orbital Velocity
a higher elliptical orbit, Geosynchronous Transfer Orbit has no role in satellites being launched from east coast.
(GTO) and Geosynchronous Earth Orbit (GEO).
Statement 4 is correct: The launching stations are
Statement 4 is incorrect. PSLV is the third-generation generally located near eastern coastline so that, just in
launch vehicle of India with 4 stages while GSLV is a case of failure of the launch, the satellite and its debris
three-stage launcher with strap-on motors. etc. does not fall on built-up hinterland.
258. Solution: (b) 260. Solution: (a)
Exp) Option b is correct. Exp) Option a is correct.
Dark matter is a hypothetical form of matter thought Statement 1 is correct. Chandrayaan-2 has detected the
to account for approximately 85% of the matter in the unambiguous presence of hydroxyl and water molecules
universe. Dark energy is a hypothetical form of energy on the Moon. The findings were confirmed by a paper
whose negative pressure counteracts gravity and is published in the latest issue of the fortnightly journal
assumed to be responsible for the universe expanding at Current Science. The paper was authored by scientists
an accelerating rate. from the Indian Institute of Remote Sensing (IIRS) in
Statement 1 is correct. The mass of the universe is made Dehradun, SAC in Ahmedabad, UR Rao Satellite Centre
up of material that scientists cannot directly observe in Bengaluru, and ISRO.
which is known as dark matter. Unlike normal matter, Statement 2 is also correct. Presence of water is due to
dark matter does not interact with the electromagnetic the phenomenon of ‘Space Weathering’. It is a process
force. This means it does not absorb, reflect or emit light, of formation of water and hydroxyl occurs due to

177 Workbook
.
SCIENCE & TECHNOLOGY

interaction of solar winds with the lunar surface. Space


weathering along with the impact from small meteorites 2.7. Energy
often lead to chemical changes on the surface of the 262. Solution (c)
moon. This ultimately leads to formation of either the
Exp) Option c is correct.
reactive hydroxyl molecules or the more stable form of
water molecules. Statement 1 is correct. Hydrogen gas is extracted
from water by a technique known as electrolysis, which
261. Solution: (a) involves running a high electric current through water
Exp) Option a is the correct answer. to separate hydrogen and oxygen atoms. The electrolysis
process is pretty expensive since it involves high energy
The visible universe including Earth, the sun, other stars,
expenditure.
and galaxies is made of protons, neutrons, and electrons
bundled together into atoms. They make up only 5% of Statement 2 is incorrect. Hydrogen fuel cells are more
the universe. energy-efficient than internal combustion engines.
However, lithium-ion batteries are still the most energy
The astronomers theorize that the faster expansion rate efficient and the highest performing energy source.
of the Universe is due to a mysterious, dark force or
energy that is pulling galaxies apart. The term ‘dark’ is Statement 3 is incorrect. Hydrogen energy is renewable
used to denote the unknown. and has a minimal environmental impact, but its
separation from oxygen requires other non-renewable
sources such as coal, oil and natural gas. Fossil fuels are
still needed to produce hydrogen fuel which leads to
emission of greenhouse gases.
Statement 4 is incorrect. The hydrogen gas can also just
be burned (mechanical energy and not just electrical
energy) to power vehicle engines. The by-products of
this chemical reaction are water and carbon that is used
to produce methane and coal.

Statement 1 is correct. Dark matter is a mysterious


substance that composes about 27% of the makeup of the
universe. Unlike normal matter, dark matter does not
interact with the electromagnetic force. This means
it does not absorb, reflect or emit light, making it
extremely hard to detect. In fact, researchers have been
able to infer the existence of dark matter only from the
gravitational effect it seems to have on visible matter.
Such fast rotation is possible only when there is more
mass, and that extra mass is believed to come from dark
matter.
Statement 2 is correct. Dark matter interacts with the
rest of the universe only through its gravity (that’s how 263. Solution (d)
we know it exists). Many large lab experiments have tried Exp) Option d is correct.
to detect elementary particles that could be candidates for Statement 1 is correct. An Organic Photovoltaic (OPV)
dark matter. However, such dark matter particles have not cell is a type of solar cell where the absorbing layer is
been detected until now. based on organic semiconductors (OSC) – typically
Statement 3 is incorrect. Dark matter is not antimatter, either polymers or small molecules.
because the unique gamma rays that are produced when Statement 2 is correct. Manufacturing cost is cheaper
antimatter annihilates with matter are not observed. than the inorganic Photovoltaic Cells due to large
Anti-matter has some properties opposite with respect to availability of raw material. Soluble organic molecules
the usual matter. enable roll-to-roll processing techniques and allow for
low-cost manufacturing. The wide abundance of building-
For example, the electron has as its antiparticle the block materials reduces supply and price constraints.
antielectron. The electron and the antielectron have
exactly the same masses, but they have exactly opposite Statement 3 is correct. The main disadvantages
electrical charges. associated with organic photovoltaic cells are low
efficiency, low stability and low strength. This is in
Dark matter is not antimatter. Antimatter is composed comparison to inorganic photovoltaic cells such as
of antiparticles, while dark matter is composed of particles silicon solar cells.
that don’t react with electromagnetic spectra. Antimatter
annihilates matter on contact, producing gamma rays. 264. Solution (d)

Workbook 178
.
SCIENCE & TECHNOLOGY

Exp) Option d is correct. 266. Solution (a)


Statement 1 is correct. Geothermal energy is the thermal Exp) Option a is correct
energy generated and stored inside the Earth’s crust. Statement 1 is correct. Wind is an intermittent and
When underground water comes in contact with the hot site-specific resource of energy. Intermittent means
spot, steam is generated. that which cannot be dispatched when there is demand.
Statement 2 is correct. The most active geothermal It gives variable power, which is consistent from year to
resources are usually found along major plate year but varies greatly over shorter time scales. Therefore,
boundaries where earthquakes and volcanoes are it must be used together with other power sources to give
concentrated. Most of the geothermal activity in the a reliable supply.
world occurs in an area known as the “Ring of Fire.” Statement 2 is incorrect. Gujarat has highest potential
Statement 3 is correct. Puga in Jammu & Kashmir and of wind energy with 84431.33 MW. Tamil Nadu has total
Surajkund in Jharkhand are important sites for geothermal potential of 33799.65MW and ranks 4 in overall potential.
energy in India. Other important sites are However Tamil Nadu ranks first in installed wind power
Tattapani in Chhattisgarh with 9300 MW capacity.
Cambay Graben in Gujarat Statement 3 is incorrect. In India there are two main
maritime areas in which structures such as offshore
Manikaran in Himachal Pradesh
wind farms can be built:
Chhumathang in Jammu & Kashmir
Indian territorial waters, which generally extend up to 12
nautical miles (nm) from the baseline; a
Exclusive Economic Zone (EEZ), beyond the 12 nm
limit and up to 200 nm, where under international law,
India has right construct structures such as wind farm
installations.
267. Solution (b)
Exp) Option b is correct.
Option a is incorrect. Bioelectricity is the electrical
currents and electrical potentials generated by or
occurring within living cells, tissues, and organisms.
Option b is correct. Microbial Fuel cell is a device
that converts chemical energy to electrical energy by
the action of microorganisms and uses bacteria as the
catalyst to oxidize organic and inorganic matter to
generate electric current. It has applications in various
fields such as power generation systems, bio-recovery, and
waste-water treatment.
Option c is incorrect. The creation of an organic
compound in a living organism is referred to as
biosynthesis. Biosynthesis refers to the production of a
complex chemical compound from simpler precursors
265. Solution (c)
in a living organism. It is usually involving enzymes that
Exp) Option c is correct. will catalyze the reaction) and energy source (e.g. ATP).
Option a is incorrect. Hydrogen produced from fossil Examples of biosynthesis include photosynthesis.
fuels is called grey hydrogen; this constitutes the bulk of
the hydrogen produced today. 268. Solution (d)
Option b is incorrect. Hydrogen generated from fossil Exp) Option d is correct.
fuels with carbon capture and storage options is called Statement 1 is incorrect. U-235 cannot be used directly
blue hydrogen. to produce electricity they have to undergo enrichment
Option c is correct. Hydrogen generated entirely from process before being used to generate electricity.
renewable power sources is called green hydrogen. In the Statement 2 is incorrect. Nuclear fusion-based power
process, electricity generated from renewable energy is plant is presently not in operation for meeting the
used to split water into hydrogen and oxygen. energy needs. However, some nuclear fusion-based
Option d is incorrect. Biohydrogen is the hydrogen that power plants are under construction around the world.
is produced biologically. One of the ways is through use of More common ones are the nuclear fission plants.
microorganisms like production of Hydrogen by algae as Statement 3 is correct. Water, solid graphite and heavy
algae produce hydrogen under certain conditions. water are used as a moderator in nuclear reactors.

179 Workbook
.
SCIENCE & TECHNOLOGY

Statement 4 is correct. Nuclear technology can be used solar photovoltaic panels by 5–10 percent. Over time, this
in medical diagnosis and treatment. It can also be used translates into significant cost savings. So, the statement
in different industries for different purposes such as in the talks about the benefit of floating solar plants, but
irradiation of food, sterilization of disposable products the context of the question talks about the challenges
etc. associated with floating solar power plants. Thus,
the statement in itself is correct but in the context of
269. Solution (c) the question it is incorrect. Other potential benefits
Exp) Option c is correct. include reduced shading, reduced civil works, reduced
Statement 1 is correct. Lithium-Sulfur battery is grid interconnection costs, reduced water evaporation,
considered to be the successors of the Lithium-ion (Li- improved water quality, and reduced algal blooming.
ion) batteries because of their lower cost of production, Statement 3 is correct. A floating solar plant has moving
energy efficiency and improved safety. parts that are subject to constant friction and mechanical
Statement 2 is correct. Lithium-Sulfur battery had an stress. Systems that are poorly designed and maintained
intrinsic problem with the sulfur electrode, which could suffer from catastrophic failures. The installation
would break after repeated charge cycles making its is at risk of degradation and corrosion due to moisture,
superior capacity redundant. especially in more aggressive coastal environments. This
causes degradation and corrosion of the plant leading to
270. Solution (c) higher maintenance costs.
Exp) Option c is correct.
272. Solution (b)
Option 1 is correct. Solar Panels are durable and can
perform under high temperatures. But same as other Exp) Option b is correct.
electronics, with higher temperatures, come with Statement 1 is incorrect: Lithium-ion batteries
decreased power outputs. The efficiency of a solar panel convert chemical energy into electrical energy using
tends to decrease as the temperatures rise, which can be an electrochemical reaction (using cathode, anode
concerning for property owners who are looking to get and electrolyte.) These reactions are reversible and
a solar panel installation in a hot and sunny climate. The so a lithium battery once depleted can be recharged
bodies of water that host the floating solar panels will again. Therefore, it is used in portable electronics like
help the solar-powered systems cool down, which means smartphones, laptops, etc.
that the floating solar panel installation will increase the Statement 2 is correct: They tend to overheat and can be
efficiency of a solar panel in the hot climates than they damaged at high voltages. In some cases, this can lead to
otherwise would. combustion. This can cause transportation restrictions
Option 2 is incorrect. A floating solar panel installation on large quantities of Li-ion batteries. It is one of the few
requires additional costs than more traditional solar panel limitations of Lithium-ion batteries.
installations. Because this is a relatively new solar power
technology it requires specialized solar power equipment 273. Solution (b)
and more niche solar panel installation knowledge, it Exp) Option b is correct.
will typically require a higher price tag than installing Statements 1 is incorrect. Chemically, CBG is the same as
a similar-sized solar power farm on a rooftop, or solid CNG — both are compressed methane — and in gaseous
ground. form. The difference is that while CNG is a by-product of
Option 3 is correct. The floating solar panel installation petroleum, CBG can be produced from any biomass, be it
provides shade to the body of water and reduces the crop residue, cattle dung, sugarcane press mud, municipal
evaporation from these ponds, reservoirs, and the lakes. wet waste or effluents from a sewage treatment plant.
This is a particularly useful environmental benefit of solar This makes CBG a commercially viable option as it can
energy in areas that are more susceptible to droughts, as be directly used to replace CNG in transportation fuel.
water loss due to evaporation and can add up over time Just like CNG, CBG too can be transported through
and contribute to a shortage. cylinders or pipelines to retail outlets.
271. Solution (c) Statement 2 is correct. Financial barriers like high
capital cost, unavailability of long-term financing
Exp) Option c is correct.
options, high interest rate and high-risk perception
Statement 1 is correct. Floating systems present specific by financial institutions are identified as the most
challenges related to anchoring and mooring them in prominent barriers to biogas dissemination. The high
place, accounting for possible water level variations, the capital cost and low revenue accrual act as entry barriers
reservoir’s bed type and depth, and extreme weather for small private players/developers. Lack of access to
situations such as high winds and waves. Accordingly, long-term financing and high interest rate (12–14%)
engineering and construction costs are usually higher affects the economic viability of the biogas projects.
than those of a ground-mounted solar farm.
Statement 2 is incorrect. A benefit is that bodies of water 274. Solution (d)
exert a cooling effect, which improves the performance of Exp) Option d is correct.

Workbook 180
.
SCIENCE & TECHNOLOGY

Statement 1 is correct. Winter diesel is a specialised fuel they still lack the range that would make them a viable
that was introduced by IOCL last year specifically for alternative to internal combustion engines.
high altitude regions and low-temperature regions such Statement 3 is incorrect: Lithium ion batteries are
as Ladakh, where ordinary diesel can become unusable. expensive to manufacture. They cost about 40 percent
Winter diesel contains additives to maintain lower higher than nickel-cadmium batteries.
viscosity. It can be used in temperatures as low as -30°C.
Statement 2 is correct. The winter diesel meets the 277. Solution (d)
Bureau of Indian Standards (BIS) specifications for BS VI Exp) Option d is correct.
grade diesel. Statement 1 is correct. Thorium is 3 times as abundant
Statement 3 is correct. It has lower sulphur content, as uranium, almost matching the amount of lead and
which would lead to lower deposits in engines and better gallium in the Earth’s crust. Thorium composes 0.0006%
performance. of the earth’s crust whereas Uranium composes 0.00018%
of earth’s crust, where a substantial amount of Uranium is
275. Solution (b) found in dissolved sea water.
Exp) Option b is correct. Statement 2 is correct. On the basis of per unit mass
Pair 1 is correctly matched. Chutka nuclear power plant of mined mineral, thorium can generate more energy
is a proposed power plant- to be built in Madhya Pradesh. compared to natural uranium. It is estimated that
It is designated for two indigenous 700 MWe PHWR one ton of thorium can produce as much energy as 35
units. NPCIL has initiated pre-project activities. tons of uranium in a liquid fluoride thorium reactor.
Conventional reactors utilizes less than one percent of
Pair 2 is correctly matched. Gorakhpur nuclear power
uranium, whereas a well working reprocessing reactor can
plant is an under-construction power plant located in
utilize 99% of its thorium fuel. However, when comparing
Fatehabad district of Haryana.
breeder reactors for Uranium and Thorium, the energy
Pair 3 is incorrectly matched. Kovvada nuclear power per kilogram would be similar.
plant is a new site approved in Andhra Pradesh’s
northern coastal Srikakulam. It is designated for six 1208 Statement 3 is correct. Thorium produces less harmful
MW capacity units. waste compared to uranium. There is up to two orders of
magnitude less of nuclear waste in the liquid fluoride
thorium reactor, eliminating the need for large scale
and long-term storage for the waste. This is because the
Thorium-Uranium fuel cycle does not irradiate U-238,
so it does not produce atoms bigger than uranium.
Furthermore, it takes a couple hundred of years for the
radioactivity of the waste to drop to safe levels, whereas it
takes tens of thousands of years for current nuclear waste
to drop to safe level.
Also, Thorium is safer and more efficient to mine than
uranium, thus making it more environmentally friendly.
Thorium mines has an open pit which does not require
ventilation, whereas uranium mines is closed off where
the radon level reach potentially dangerous levels.
278. Solution: (a)
Exp) Option a is correct.
Energy can be generated in form of electricity and/or heat
from the primary treatment of waste. It is done through
Waste-to-energy process.
Waste-to-Energy (WtE) or Energy-from-Waste (EfW) is
a form of energy recovery and the process of generating
276. Solution (d)
energy in the form of electricity and/or heat by processing
Exp) Option d is correct of waste into a fuel source. There are various kind of Waste
Statement 1 is incorrect: Lithium ion batteries are to energy processes as follows:
rechargeable batteries with a weak energy density. The Statement 1 is correct. The torrefaction technology
battery consists of electrolyte, which allows for ionic involves heating straw, grass, sawmill residue, and wood
movement, and the two electrodes are the constituent biomass to 250 degrees Celsius – 350 degrees Celsius.
components of a lithium-ion battery cell. This changes the elements of the biomass into ‘coal-like’
Statement 2 is incorrect: Lithium-ion battery technology pellets. These pellets can be used for combustion along
has made it to being one of the most preferred power with coal for industrial applications like steel and cement
source for electric and hybrid electric vehicles; however, production.

181 Workbook
.
SCIENCE & TECHNOLOGY

gradients of solubilized attractants. Chemotaxis is a


fundamental form of cell behaviour that involves a
complex response of a cell to an external stimulus
Option c is incorrect. Chemokinesis is defined as
random cell movement in the absence of chemoattractant
gradients, chemotactic agents that induce a cell to migrate
toward them.
Option d is incorrect. Chemotherapy is a drug treatment
Statement 2 is correct. Polycrack technology is the that uses powerful chemicals to kill fast-growing cells in
your body. Chemotherapy is most often used to treat
world’s first patented heterogeneous catalytic process
cancer, since cancer cells grow and multiply much more
that converts multiple feedstocks into hydrocarbon liquid
fuels, gas, carbon, and water. quickly than most cells in the body.
The process is a closed-loop system and does not emit any 280. Solution: (a)
hazardous pollutants into the atmosphere. Exp) Option a is correct.
The combustible, non-condensed gases are re-used for The finance minister in the Union budget for 2020-21
providing energy to the entire system and thus, the only formally announced the National Hydrogen Mission
emission comes from the combustion of gaseous fuels. which aims for generation of hydrogen from green
This process will produce energy in the form of light diesel power resources.
oil which is used to light furnaces. Statement 1 is correct.
Indian Railways has commissioned country’s first Depending on the nature of the method of its extraction,
governmental Waste to Energy Plant based on polycrack hydrogen is categorised into three categories, namely,
technology, at Bhubaneswar in East Coast Railway. grey, blue and green. Hydrogen produced from fossil
Statement 3 is incorrect. Liquefaction is a phenomenon fuels is called grey hydrogen; this constitutes the bulk of
in which the strength and stiffness of a soil is reduced by the hydrogen produced today. Hydrogen generated from
earthquake shaking or other rapid loading. Liquefaction fossil fuels with carbon capture and storage options is
and related phenomena have been responsible for called blue hydrogen; hydrogen generated entirely from
tremendous amounts of damage in historical earthquakes renewable power sources is called green hydrogen. In
around the world. It is not a waste to energy conversion the last process, electricity generated from renewable
technique. energy is used to split water into hydrogen and oxygen.
Statement 4 is incorrect. Pulverization (comminution, Statement 2 is correct. Hydrogen is two-three times
crushing, grinding) is the process of applying an external more efficient than burning petrol. The energy in 1
force to a (solid) material of a certain size to destroy kilogram of hydrogen gas is about the same as the energy
it and reduce it into pieces that are smaller than the in 2.8 kilograms of petrol.
original size. In this method, collected solid waste is Hydrogen has a high energy density as compared to
powdered by grinding machines, thereby changing other fuels, thus it produces more energy in lesser weight
its volume and physical characteristics. This pulverized due to which it can prove to be a viable option for heavy
solid waste is further disposed-off by land filling. This is vehicles covering long routes in the future. The refuelling
costly technique. It is not a waste to energy conversion time required for hydrogen is also lesser when compared
technique. to electric vehicles and vehicles running on conventional
fuels.
279. Solution: (a)
Statement 3 is incorrect. Although hydrogen is a
Exp) Option a is correct. clean molecule, the process of extracting it is energy-
Most life on Earth is dependent upon photosynthesis, intensive. For instance, to extract hydrogen from water
the process by which plants make energy from sunlight. electrolysis is to be performed which itself uses electricity.
However, at hydrothermal vents in the deep ocean a unique Electrolysis is the process of using electricity to split water
ecosystem has evolved in the absence of sunlight, and its into hydrogen and oxygen.
source of energy is completely different: chemosynthesis. Statement 4 is correct. Hydrogen is one of the cleanest
Option a is correct. Chemosynthesis is the process fuels, which on being burnt in air produces only water as
by which certain microbes create energy by mediating a by-product and no carbon-based emissions are released,
chemical reactions. The chemosynthetic microbes thereby leading to zero vehicular emissions.
provide the foundation for biological colonization of
vents. Chemosynthetic microbes live on or below the 281. Solution: (a)
seafloor, and even within the bodies of other vent animals Exp) Option a is correct.
as symbionts. The importance of nuclear energy, as a sustainable
Option b is incorrect. Chemotaxis is defined as energy resource for our country, was recognised at the
directional cell movement of cells towards concentration very inception of our atomic energy programme more

Workbook 182
.
SCIENCE & TECHNOLOGY

than four decades ago. A three-stage nuclear power Hence, there is no need of large quantity of fuel materials
programme, based on a closed nuclear fuel cycle, was then for the annual external feed and thus eliminates the need
chalked out. The three stages are: for large capacity waste storage spaces with complex
• Natural uranium fuelled Pressurised Heavy Water construction features.
Reactors (PHWRs), • Green energy often comes from renewable energy
• Fast Breeder Reactors (FBRs) utilising plutonium- technologies such as solar energy, wind power,
based fuel, and, geothermal energy, biomass and hydroelectric
• Advanced nuclear power systems for utilisation of power. Each of these technologies works in different
thorium. ways, whether that is by taking power from the sun, as
There are presently 22 reactors with a total capacity of with solar panels, or using wind turbines or the flow
6780 MW in operation. of water to generate energy.
Statement 1 is correct: Fast Breeder Reactor (FBR) Statement 3 is incorrect: A breeder reactor is designed
produces more fissionable material than it consumes to produce more fissionable plutonium than the fissile
to generate energy. This special type of reactor is uranium isotope U 235 and this is achieved by converting
designed to extend the nuclear fuel supply for electric the unproductive non-fissionable U 238 into fissile
power generation. A breeder reactor employs either
plutonium isotope. Currently two fast breeder reactors
uranium-238 or thorium, of which sizable quantities are
available. Uranium-238, for example, accounts for more are operating in Russia. In India, a prototype 500 MW
than 99 percent of all naturally occurring uranium. fast breeder reactor (FBR) is under construction at the
Madras Atomic Power Station in Kalpakkam. The Indira
Statement 2 is correct: Electricity generated by Fast
Breeder Reactor would be a source of green energy Gandhi Centre for Atomic Research is responsible for the
as the waste from the first stage nuclear programme is design of this reactor. The Kalpakkam FBR uses U 238
reprocessed and used as fuel in FBR. The spent fuel from and not thorium to breed fissile plutonium and is yet to
this reactor can be fed back into the reactor core several become operational. China and Japan are also reportedly
times, till the spent fuel contains only short-lived fission operating FBRs. These reactors are expected to be safe,
products. This is the concept of FBR with closed fuel cycle. more efficient and pose fewer environmental hazards.

Fig. How a Breeder reactor works

282. Solution: (d) all metal–air batteries, Al–air batteries hold great promise
Exp) Option d is correct. for future large-scale energy applications due to their
An aluminium-air cell consists of a pair of negative and lowest cost and high theoretical specific capacity. It has
positive electrodes and an electrolyte. In this case, the one of the highest energy densities for a battery.
anode is formed by aluminium, while the ‘air-breathing’ Statement 2 is correct. One of the key downsides
cathode delivers oxygen from the ambient air. The of aluminium-air batteries is that they cannot be
electrolyte is water-based and is essentially a solution of recharged like lithium-ion batteries. The battery pack is
potassium hydroxide or sodium hydroxide. much lighter as compared to lithium-ion battery. It thus
Statement 1 is correct. Aluminium-Air batteries have the requires battery swapping, not re-charging, so it faces
higher energy density than Lithium-ion batteries. Among infrastructure and logistical challenges. Large scale use of

183 Workbook
.
SCIENCE & TECHNOLOGY

aluminium-air battery-based vehicles would require the Statement 1 is incorrect. CNG is produced by
wide availability of battery swapping stations. compressing natural gas to less than 1% of its volume
Statement 3 is correct. Aluminium-air based batteries at standard atmospheric pressure. To provide adequate
driving range, CNG is stored in a gaseous state in high-
are expected to be significantly cheaper than lithium-
pressure storage cylinders.
ion batteries. Thereby reducing the cost of electric
vehicle usage and boosting electric vehicle adoption in the Statement 2 is correct. The recent research by Council
country. Aluminium-air battery-based electric vehicles of Scientific and Industrial Research (CSIR) pinpoints
are expected to offer much greater range of 400 km or that gases produced while burning CNG contain carbon
nanoparticles that are suspected to cause cancer.
more per battery compared to lithium-ion batteries
This is a result of combustion which causes carbon
which currently offer a range of 150-200 kilometres per disintegration. Although it has great potential for being
full charge. a cleaner technology but it does have negative aspect to
283. Solution: (b) nanoparticles.
Statement 3 is correct. Natural gas is also used as a
Exp) Option b is correct
starting material for the manufacture of a number of
Nuclear reactors generate energy through fission, the chemicals and fertilisers. When natural gas is heated
process by which an atomic nucleus splits into two or strongly, the methane present in it decomposes to form
more smaller nuclei. During fission, a small amount of carbon and hydrogen. This hydrogen is then used for
mass is converted into energy, which can be used to power manufacturing
a generator to create electricity.
Statement 1 is incorrect. The Pressurised Heavy Water 2.8. Defence Technology
Reactors (PHWRs) uses natural Uranium as fuel.
PHWRs not only produce energy from natural uranium 285. Solution (a)
but also produce fissile plutonium (Pu)-239. The Fast Exp) Option a is correct
Breeder Reactors (FBRs) use plutonium-239 (and not Muntra is an unmanned tank has been created by the
thorium) for generating electricity. Once a sufficient Defence Research and Development Organisation
amount of plutonium-239 is built up, thorium will be used (DRDO) which can be operated remotely. It is developed
in the reactor, to produce Uranium-233. in the Chennai lab of DRDO.
Statement 2 is correct. PHWR uses heavy water The Muntra-S variant will India’s first step towards a
(deuterium oxide D2O) as its coolant and neutron tracked unmanned ground vehicle (tank) and has great
moderator. The heavy water coolant is kept under applications in unmanned surveillance missions.
pressure, allowing it to be heated to higher temperatures
without boiling. The heavy water creates greatly enhanced The Muntra-M and Muntra-N variant will help for
neutron economy, allowing the reactor to operate without operation in areas where there is a field of land mines
fuel-enrichment facilities (offsetting the additional or nuclear radiations or biological weapon risk without
expense of the heavy water) and enhancing the ability of sending any personnel.
the reactor to make use of alternate fuel cycles. FBRs do 286. Solution (c)
not have a neutron moderator, and use less-moderating
Exp) Option c is correct.
coolants such as liquid sodium, so its neutrons remain
high-energy. Breeder reactors use a small core, which is Statement 1 is correct. Microwave weapons aim highly
important to sustain chain reactions. focused energy in the form of sonic, laser, or microwaves
directly at a target.
Statement 3 is incorrect. The increased rate of fuel
movement through the PHWR results in higher volumes Statement 2 is correct. Microwave weapons use a focused
of spent fuel. However, fast reactors have the potential beam of high-frequency electromagnetic radiation to
to produce less radioactive waste because fuel is highly heat the water in a human target’s skin, causing pain
enriched in fissile material. There is no need of large and discomfort. The process is similar to microwave oven
quantity of fuel materials for the annual external feed where electromagnetic waves are used to agitate the water
in FBR and thus eliminates the need for large capacity molecules in the food and the vibration produces heat
waste storage spaces with complex construction features. which cooks the food.

284. Solution: (c) 287. Solution (c)


Exp) Option c is correct. Exp) Option c is correct.
Compressed natural gas or CNG is a primary clean The Tejas is an Indian single-engine, delta wing, multirole
light fighter designed by the Aeronautical Development
energy source of natural gas mainly comprised of Agency (ADA) and Hindustan Aeronautics Limited
methane that is stored under high pressures while (HAL) for the Indian Air Force and Indian Navy.
remaining in its gaseous form mainly to transport Option a is correct. Tejas is being manufactured
it, or as storage for later use as vehicle fuel. It is clear, by Hindustan Aeronautics Limited and designed by
odorless, and non-corrosive. Aeronautical Development Agency.

Workbook 184
.
SCIENCE & TECHNOLOGY

Option b is correct. Tejas is a fully weaponised single by the Government of India in 2015. These ships are
engine light fighter. being built with enhanced stealth features, advanced
Option c is incorrect. The Tejas is an indigenous light indigenous weapon and sensor fit along with several
weight, multi role supersonic aircraft (not ultrasonic). other improvements.
Option d is correct. The Tejas is designed to carry a Statement 2 is incorrect. Under Project 17A, 80% of the
veritable plethora of air-to-air, air-to- surface, precision material/ equipment required (not completely) for the
guided and standoff weaponry. project are sourced from indigenous vendors and Indian
firms and MSMEs within the country.
288. Solution (b) Statement 3 is correct. Project 75 is the programme
Exp) Option b is correct. by the Indian Navy to build six Scorpene-Class attack
Defence Research and Development Organisation submarines, which will also feature advanced air-
(DRDO) has developed an Advanced Chaff Technology. independent propulsion systems. This is for enabling them
Chaff is a passive expendable electronic countermeasure to stay submerged for longer duration and substantially
technology. It is used worldwide to protect naval ships increase their operational range. The six submarines are
from enemy’s radar and Radio Frequency (RF) missile - INS Kalvari, INS Khanderi, INS Karanj, INS) Vagir ,
seekers. INS Vela and INS Vagsheer.
Significance: Under this technology, a very less quantity 292. Solution (d)
of chaff material deployed in the air acts as a decoy. It Exp) Option d is correct.
will deflect the enemy’s missiles for safety of the ships.
Pair 1 is correctly matched. INS Himgiri is first
289. Solution (a) Project 17A stealth frigate built by the Garden Reach
Exp) Option a is correct. Shipbuilders and Engineers Limited (GRSE), Kolkata.
P17A ships will be the most advanced state-of-the-
Statement 1 is correct. The Defence Research and art Guided Missile Frigates equipped with a powerful
Development Organisation (DRDO) has successfully weapon and sensor package capable of neutralising threats
flight tested indigenously developed Anti-Radiation in all three dimensions, air, surface and subsurface.
Missile, Rudram-1. It is the first indigenous anti-
Pair 2 is correctly matched. INS Imphal is the third ship
radiation missile.
under Project 15B, built by Mazagon Dock Shipbuilders,
Statement 2 is incorrect. It is an air-to-surface missile Mumbai. These ships have been designed indigenously
with a strike range of around 100 to 150 km. It has been by the Indian Navy’s Directorate of Naval Design.
developed indigenously by DRDO. Enhanced stealth features have been achieved through
shaping of hull and use of radar transparent deck fittings
290. Solution (c)
which make these ships difficult to detect. Project 15B
Exp) Option c is correct. ships will be equipped to carry and operate two multiple
Defence Research and Development Organisation role helicopters.
(DRDO) along with Indian Navy conducted the Pair 3 is correctly matched. INS Khanderi is the second
successful maiden test trial of ‘SAHAYAK-NG’, India’s of the six Scorpene-class submarines that are being
first indigenously designed and developed Air Dropped built under Project 75 at Mazagon Dock Shipbuilders,
Container from IL 38SD aircraft (Indian Navy) off the Mumbai. The project is being carried out in collaboration
coast of Goa. It is capable of carrying critical stores for with the French submarine builder Naval Group
ships in distress at sea from Indian Navy’s Fixed Wing (formerly DCNS).
Aircraft. SAHAYAK-NG is an advanced version of
SAHAYAK Mk I. The newly developed GPS aided air 293. Solution (c)
dropped container is having the capability to carry Exp) Option c is correct.
a payload that weighs upto 50 kg and can be dropped Statement 1 is correct. On March 27, 2019 India
from heavy aircraft. conducted Mission Shakti, an anti-satellite missile test,
The trial was conducted by Indian Navy to enhance its from the Dr. A P J Abdul Kalam Island launch complex.
operational logistics capabilities and provide critical The test successfully demonstrated India’s capability to
engineering stores to ships which are deployed more interdict and intercept a satellite in outer space based on
than 2000 km from the coast. It reduces the requirement complete indigenous technology. It was a technological
of ships to come close to the coast to collect spares and mission carried out by the Defence Research and
stores. Development Organisation (DRDO).
291. Solution (c) Statement 2 is correct. After this test, India joined an
exclusive group of space faring nations consisting of
Exp) Option c is correct USA, Russia and China. The test was done in the lower
Statement 1 is incorrect. Under the Project 17A, a atmosphere to ensure that there is no space debris. The
total of seven stealth frigates at an estimated cost of DRDO’s Ballistic Missile Defence interceptor was used
Rs 50,000 crore will be built. The project was cleared for the test.

185 Workbook
.
SCIENCE & TECHNOLOGY

294. Solution (a) fish, citrus fruit inspection, distribution of sugar in


Exp) Option a is correct melons, and sorting of potatoes. For example apple
bruise is not visible on the early stage and it takes few
Statement 1 is correct: It is medium range supersonic days to show dark color mark. In this type of scenario
cruise missile. It is developed jointly by India and Russia. hyperspectral imaging techniques can be used to
Statement 2 is incorrect: The versions of the BrahMos track the early stage of bruise for the quality control.
that are being tested have an extended range of around (Hence, option 1 is correct)
400 kilometers, as compared to its initial range of 290 2. Remote Sensing: In remote sensing technology it is
kilometers, with more versions of higher ranges currently very important to distinguish earth surface features,
under development. each features have different spectrum band. (Hence,
Statement 3 is incorrect: The missile has land, sea, and option 2 is correct)
air-launched versions. 3. Environmental Monitoring: Hyperspectral imaging
is becoming widely popular for tracking changes in
295. Solution: (c)
the environment. It is commonly used to understand
Exp) Option c is correct. surface CO2 emissions, map hydrological formations,
The Integrated Guided Missile Development Programme tracking pollution levels, and more. (Hence, option 3
(IGMDP) conceived by renowned scientist Dr. A P J is correct)
Abdul Kalam to enable India attain self-sufficiency was 4. Seed Viability Study: By using the hyperspectral
started in 1983 completed in March 2012.It was an image and plotting the reflectance spectrum one
Indian Ministry of Defence programme for the research can conclude that whether those seed are viable or
and development of the comprehensive range of missiles. not viable. Seed might be looking same through
India now moved on with the new Indian Ballistic naked eyes but its viability will be trace down by the
Missile Defence Program as an initiative to develop and hyperspectral image.
deploy a multi-layered ballistic missile defence system to 5. Biotechnology: Hyperspectral technology has
protect India from ballistic missile attacks. become popular in the biological and medical
Statement 1 is correct. Ballistic missiles are powered applications. It is easy and quick to acquire the data
by rockets initially but then they follow an unpowered, that can be used in the laboratory. Mostly they are
free-falling trajectory towards their target. While cruise used in the study of the wound analysis, fluorescence
missiles are self-propelled systems till the end of its flight. microscopy, and cell biology.
Statement 2 is incorrect. Both cruise missiles and 6. Pharmaceuticals: Hyperspectral imaging technique
Ballistic missiles can carry either nuclear or conventional is widely used to enhance the quality control. It is
warheads. used widely to control the counterfeit or illegal drugs,
Statement 3 is correct. The S-400 Triumph air defence managing the packaging of medicine and mixing of
system developed by Almaz Central Design Bureau of the powder.
Russia integrates a multifunction radar, autonomous 7. Medical Diagnose: Early disease detection and
detection and targeting systems, anti-aircraft missile disease prevention are very important for the healthy
systems, launchers, and command and control centre. body. Hyperspectral imaging technology can be used
The S-400 missile defence system is equipped with four to detect the early of various types of cancer or retinal
different missiles which can engage enemy aircraft, disease. (Hence, option 4 is correct)
ballistic missiles, cruise missiles and AWACS planes
at 400 km, 250 km, the medium-range 120 km and the 2.9. New Technology and Miscellaneous
short-range 40 km.
297. Solution (a)
296. Solution: (d) Exp) Option a is correct
Exp) Option d is the correct answer Statement 1 is correct. Torrefaction involves the heating
Hyperspectral imaging makes use of electromagnetic of biomass in the absence of oxygen to a temperature
radiation at wavelengths both inside and outside of the of typically 200 to 400°C. The structure of the biomass
visible spectrum to characterize materials and objects. changes that makes the material brittle, and more
The information collected by a hyperspectral imaging hydrophobic.
system, a hyperspectral cube, is actually three- Statement 2 is incorrect. In Torrefaction technology,
dimensional. In addition to width and height, there is weight loss is about 30% and the energy loss is about 10%.
a depth component associated with the wavelength at So, both weight and energy are lost in Torrefaction
which each photograph was taken, meaning that the when compared to original biomass.
hyperspectral cube is simply a stack of several images. Statement 3 is incorrect. Torrefaction is a Swedish
Several applications of this technique are as under: technology that can convert stubble into ‘bio-coal’.
1. Food: Hyperspectral imaging is widely used in the
food sector. It is used in different discipline of food 298. Solution (a)
industry, bruise detection in apples, freshness of the Exp) Option a is correct

Workbook 186
.
SCIENCE & TECHNOLOGY

Option a is correct. India Stack is a set of APIs that allows Option c is correct. Earlier the major issue with wildlife
governments, businesses, startups and developers to conservation projects was locating the particular species.
utilize a unique digital Infrastructure to solve India’s hard With the help of geospatial technologies it has now
problems towards presence-less, paperless, and cashless become easy tracing species populations in certain
service delivery. The Open API team at iSPIRT has been areas, preventing and addressing calamities.
a pro-bono partner in the development, evolution, and
301. Solution (b)
evangelization of these APIs and systems.
Exp) Option b is correct.
299. Solution (d) Statement 1 is incorrect. The “Khadi Prakritik Paint” is a
Exp) Option d is correct. first-of-its-kind product based on cow dung as its main
Polycrack has the following advantages over the ingredient. With cow dung as the main ingredient behind
conventional approach of treating solid waste:- this paint, cows will become a source of income even if
they are non-milching cows. This technology will increase
Pre-segregation of waste is not required to reform the consumption of cow dung as a raw material for eco-
the waste. Waste as collected can be directly fed into friendly products and will generate additional revenue to
Polycrack.-- Statement 1 is correct. farmers and gaushalas. Also, it is expected to boost local
It has high tolerance to moisture hence drying of waste is manufacturing and create sustainable local employment
not required. -- Statement 4 is correct. through technology transfer.
Waste is processed and reformed within 24 hours. Statement 2 is correct. The Khadi Prakritik Paint is a non-
Excellent air quality surrounding the plant. toxic paint free from heavy metals like lead, mercury,
chromium, arsenic, cadmium and others.
Biological decomposition is not allowed as the Waste is
treated as it is received. Statement 3 is correct. The paint has antifungal and
antibacterial properties. It is cost-effective as well as
The foot print of the plant is small hence the area odourless and has been certified by the Bureau of Indian
required for installing the plant is less when compared Standards.
with conventional method of processing. -- Statement 3
is correct. 302. Solution (a)
All constituents are converted into valuable energy Exp) Option a is correct.
thereby making it Zero Discharge Process. -- Statement Statement 1 is correct. The Large Hadron Collider beauty
2 is correct (LHCb) experiment specializes in investigating the slight
Gas generated in the process is re-used to provide energy differences between matter and antimatter by studying
to the system thereby making it self-reliant and also bring a type of particle called the “beauty quark”, or “b quark”.
down the operating cost. The LHCb experiment will shed light on why we live in a
universe that appears to be composed almost entirely of
There is no atmospheric emission during the process matter, but no antimatter.
unlike other conventional methods except for combustion
gases which have pollutants less than the prescribed Statement 2 is incorrect. Instead of surrounding the
norms in the world over. entire collision point with an enclosed detector, the
LHCb experiment uses a series of subdetectors to detect
Operates around 450 degrees, making it a low temperature mainly forward particles – those thrown forwards by the
process when compared with other options. collision in one direction. To catch the b quarks, LHCb
Safe and efficient system with built-in safety features has developed sophisticated movable tracking detectors
enables even an unskilled user to operate the machine close to the path of the beams circling in the LHC.
with ease.
303. Solution (b)
Low capital cost and low operating cost.
Exp) Option b is correct.
Fully automated system requires minimum man power.
Statement 1 is incorrect. The nanomicelles are less than
300. Solution (d) 100nm in size and are stable at room temperature.
Once injected intravenously these nanomicelles can easily
Exp) Option d is correct.
escape the circulation and enter the solid tumours where
Option a is correct. With the use of Global Positioning the blood vessels are found to be leaky. These leaky blood
Satellite and remote sensing technologies it had been vessels are absent in the healthy organs.
more efficient to manage the logistics as now we can
Statement 2 is correct. Similar to nano shells and
track the on a real-time basis and ensure their safety. nanovesicles, nanomicelles are extremely small structures
Option b is correct. Light detection and ranging and have been noted as an emerging platform in targeted
technologies (LiDARs) are being used to accurately therapy. Nanomicelles are globe-like structures with a
predict plant yield and nutritive value. Hand-held hydrophilic outer shell and a hydrophobic interior. This
devices are also being developed that can produce imagery dual property makes them a perfect carrier for delivering
to assess real-time forage quality. drug molecules.

187 Workbook
.
SCIENCE & TECHNOLOGY

Statement 3 is correct. A multi-disciplinary, multi- first proposed by Elon Musk, CEO of the aerospace firm
institutional team has recently created a nanomicelle that SpaceX.
can be used to deliver a drug named docetaxel, which How does Hyperloop work?
is commonly used to treat various cancers including
Hyperloop technology comprises of a network of tubes
breast, colon and lung cancer.
through which a pod may travel free of air resistance or
304. Solution (d) friction conveying people or objects from one point to
another. The system is fully autonomous and sealed, so no
Exp) Option d is correct.
driver-related error is anticipated. In a sealed environment
Nanotechnology refers to the fabrication, study and use with almost no air resistance, the pods are expected to
of materials, structures, devices and systems in the scale reach very high speeds of around 700 mph or 1,125 km/h.
of 1 to 100 nm.
Applications of Nano technology-
Nano technology can provide rapid and sensitive
detection of cancer-related molecules, enabling scientists
to detect molecular changes even when they occur only in
a small percentage of cells.
By using nanotechnology-based methods, the
pathogenic bacteria present in complex food products
can be detected with high sensitivity and specificity as
compared to the conventional methods.
Quantum dots (QDs) are man-made nanoscale crystals
that that can transport electrons. When UV light hits
these semiconducting nanoparticles, they can emit In 2018, Maharashtra Government had approved the
light of various colours. These artificial semiconductor Mumbai-Pune Hyperloop project.
nanoparticles have found applications in composites, Option a is incorrect. A self-driving car, also known as an
solar cells and fluorescent biological labels. autonomous vehicle (AV or auto), driverless car, or robo-
Nano sensors in agriculture are used to detect the car is a vehicle that is capable of sensing its environment
humidity of soil, pesticide residue, nutrient requirement and moving safely with little or no human input.
and crop pest identification. Option b is incorrect. The TEV (Tracked Electric
305. Solution (b) Vehicle) Project is an initiative developing technologically
advanced highways capable of handling the transportation
Exp) Option b is correct. of people and light freight more efficiently. TEV’s design
Hyperloop- is a high-speed transportation system where is a compact, electrified infrastructure that would
a pod-like vehicle is propelled through low-pressure continually power conventional, rubber tire electric
tubes. In a hyperloop, the travel pods are suspended vehicles such as cars, taxis, vans and minibuses and allow
in the air through magnetic levitation. This is done to them to operate under software control.
overcome the resistance due to friction and achieve very Option d is incorrect. Flying hotel pods: A fleet of drones
high speeds. The magnetic pods levitating inside the tube that are portable hotels are being designed by a company
can achieve speed as high as 1,000 kilometers per hour. in Canada. The hospitality concept, Driftscape, is a
Thus, the pods can travel faster than aircrafts. mobile, self-sustaining hotel that uses drone technology.
Metrino Pods- These are small, fully automatic, driverless Driftscape allows guests to travel while sleeping, roam or
vehicles that travel independently suspended under an touch down in diverse locations. It is made up of several
overhead network of light guideways, 5 to 10 m above modular units that include food and beverage elements.
street level.
307. Solution (a)
Pod taxis- These are automated driverless vehicles Exp) Option a is correct.
carrying small group of people that will operate on
elevated/underground shuttle networks or traditional Photonics is the science and technology of light. It
roadways using personal rapid transit systems. encompasses generating, guiding, manipulating,
amplifying and detecting light. Lasers, optical fibres, the
Freight Rail road- These are elevated corridors built with cameras and screens in our phones, optical tweezers,
rail lines where freight trucks can be placed and then lighting in our cars, homes, computer screens and TVs
would move on rails at high speed. are just a few examples of photonics
306. Solution (c) Application of Photonics:
Exp) Option c is correct. Option 1 is correct. Health
Hyperloop is a proposed mode of transportation that Photonics has the potential to revolutionise healthcare
moves freight and people quickly, safely, on-demand because of the ability of light to detect and measure
and direct from origin to destination. The concept was diseases in a fast, sensitive and accurate way.

Workbook 188
.
SCIENCE & TECHNOLOGY

Option 2 is correct. Lighting and energy savings techniques rely on machine learning to derive meaning
Photonics is used also in advanced lighting technology, from human languages.
such as Solid-State Lighting (SSL) for general lighting Statement 2 is correct. NLP entails applying algorithms
applications. SSL is based on light-emitting diodes (LEDs) to identify and extract the natural language rules such that
and organic LED (OLED) technologies. the unstructured language data is converted into a form
Option 3 is correct. Safety and security that computers can understand.
Photonics technologies have several practical applications Statement 3 is incorrect. NLP can be used in sentiment
in safety and security. Fibre sensors are used to detect analysis (understand mood of the customer), topic
structural defects in the building sector, prevent modelling (discovering abstract topics that occur in a
environmental pollution and develop driver assistance collection of documents), text categorisation (categorising
systems. text into organised groups).
Option 4 is correct. High-Quality Manufacturing Natural Language Processing is the driving force behind
Personal assistant applications such as OK Google, Siri,
Lasers have become a versatile tool. Laser processing has Cortana, and Alexa.
become essential for high-volume, low-cost and precision
manufacturing. 310. Solution (b)
308. Solution (b) Exp) Option b is correct.
Exp) Option b is correct. Statement 1 is correct. Gelatin sticks are cheap explosive
materials used by industries for the purpose of mining
Statement 1 is incorrect. With the support of the project and construction related work, like building structures,
“Centers of Excellence in Nanoelectronics” by Ministry roads, rails and tunnels etc. They cannot be used without
of Electronics and Information Technology (MeitY) at a detonator.
Indian Institute of Science, Bangalore (IISc) and the
Indian Institute of Technology, Bombay (IITB) (and Statement 2 is correct. The manufacture is regulated by
not at all IIT’s) has enabled the process of creating a Petroleum and Explosives Safety Organization (PESO),
significant infrastructure, which will promote research formerly known as Department of Explosives. The PESO
and training in the area of nanoelectronics in India. comes under the Ministry of Commerce and Industry,
and carries out administration work of the Explosive Act
Statement 2 is correct. The major focus of this project 1884 and Explosives Rules 2008 etc.
is research and training of in-house researchers
and students at these Institutes, the facilities created Statement 3 is incorrect. In India, prior license is required
under such a national initiative must also be available not just to make, but also to sell and buy explosives. This
to researchers and students from other institutions, includes gelatin sticks also.
laboratories and industry. 311. Solution (c)
Statement 3 is incorrect. The program is funded by Exp) Option c is correct.
the Department of Information Technology (DeitY)
(and not by DST), the Ministry of Communications Pair 1 is incorrectly matched. BlackRock is an android
and Information Technology (MCIT), and was malware, which can steal information like passwords and
launched in 2008.Knowledge Base: The goals of Indian credit card information from smartphone applications,
Nanoelectronics User Program are: including Amazon, Facebook, Gmail and Tinder.
BlackRock isn’t exactly a new malware. It is based on the
To promote research in the field of Nanoelectronics leaked source code of the Xeres malware, itself derived
To provide hands-on-training in Nanoelectronics research from malware called LokiBot. The only big difference
to academic investigators from around India between BlackRock and other Android banking trojans is
To provide access to sophisticated facilities for carrying that it can target more apps than previous malwares.
out research projects of current interest Pair 2 is incorrectly matched. Zolgensma is a gene
To provide a platform for researchers to benefit from replacement therapy indicated for the treatment of
complementary expertise spinal muscular atrophy (SMA) in paediatric patients.
To motivate academics to establish/enhance research The Zolgensma one-time gene replacement therapy is
programs in their home institutions commercially available in United States and is billed as a
ray of hope for treating children below with SMA type 1.
309. Solution (b) Zolgensma gene therapy is considered as world’s most
Exp) Option b is correct. expensive medicine, which nearly costs 2.1 million US
Statement 1 is correct. Natural language processing dollars (Rs 16 crore) in United States.
(NLP) is a branch of artificial intelligence that deals with SMA is a disease caused by loss of nerve cells, which carry
the interaction between computers and humans using electrical signals from the brain to the muscles.
the natural language. The ultimate objective of NLP is Pair 3 is correctly matched. Indian Institute of Technology
to read, decipher, understand, and make sense of the Madras researchers have booted up a microprocessor
human languages in a manner that is valuable. Most NLP called ‘Moushik’. Moushik is a processor-cum-system on

189 Workbook
.
SCIENCE & TECHNOLOGY

a chip that can cater to the rapidly-growing IoT devices. It move or change once triggered by a stimulus.
aims to reduce dependence on other countries and as it is This preprogramming step enables the creation of
open source the design can be picked up by start-ups and smart, responsive objects that can adapt to specific
customised to their needs. environmental factors.
312. Solution (b) 314. Solution (a)
Exp) Option b is correct. Exp) Option a is correct.
Statement 1 is incorrect. Science and Engineering Statement 1 is correct. Geo-fencing is a location-based
Research Board (SERB) is a statutory body established service in which an app or other software uses GPS, RFID,
through an Act (the Science and Engineering Research Wi-Fi or cellular data to trigger a pre-programmed action
Board Act, 2008) of Parliament in 2008. TDB is also a when a mobile device or RFID tag enters or exits a virtual
statutory body established in 1996 under the Technology boundary set up around a geographical location, known
Development Board Act, 1995. as a geofence.
Statement 2 is incorrect. SERB functions under the Statement 2 is incorrect. Geo-fencing can’t be utilised
Ministry of Science and Technology. It is chaired by the for navigational purpose. It is used to trigger actions
Secretary to the Department of Science and Technology when a person or device encounters a pre-set virtual
and has other senior government officials and eminent boundary. Depending on how a geofence is configured
scientists as members. TDB also functions under the it can prompt mobile push notifications, trigger text
Ministry of Science and Technology. TDB is also chaired messages or alerts, send targeted advertisements on
by the Secretary, Department of Science & Technology. social media, allow tracking on vehicle fleets, disable
Statement 3 is correct. SERB aims to promote certain technology or deliver location-based marketing
basic research in science and engineering and to data. Thus, cannot be used for marine and terrestrial
provide financial assistance to scientists, academic navigation.
institutions and other agencies for such research. Statement 3 is correct. A geofence is most commonly
Technology Development Board aims at accelerating defined within the code of a mobile application, especially
the development and commercialisation of indigenous since users need to opt-in to location services for the
technology or adapting imported technology to wider geofence to work. Geofencing can be helpful with
domestic application. The board provides financial telematics, allowing companies to draw virtual zones
assistance in the form of Equity, Soft loans, or Grants. TDB around sites, work areas and secure areas. They can
provides equity capital or loans to industrial concerns be triggered by a vehicle or a person and send alerts or
and financial assistance to research and development warnings to the operator.
institutions.
315. Solution (d)
313. Solution (d) Exp) Option d is correct
Exp) Option d is correct. Option d is correct. Geofencing is a location-based
Statement 1 is correct. 3D printing is a rapid prototyping service in which an app or other software uses GPS, RFID,
and manufacturing technology that deposits material Wi-Fi or cellular data to trigger a pre-programmed
layer by layer to create three-dimensional objects. action when a mobile device or RFID tag enters or exits
Fundamentally, 4D printing uses the same techniques as a virtual boundary set up around a geographical location,
3D printing to create parts. The key difference is that 4D known as a geofence.
printed objects change their shape over time once printed, It can prompt mobile push notifications, trigger text
while 3D printed object maintain the same, fixed form. messages or alerts, send targeted advertisements on
Statements 2 and 3 are correct. 4D printing is the social media, allow tracking on vehicle fleets, help in
method of 3D printing where there is an addition of the assets monitoring, disable certain technology or deliver
dimension i.e. time. Generally, the input in a 4D printer is location-based marketing data depending on how a
a “smart material” that can be a hydrogel, a shape memory geofence is configured. Recently, it has been used by the
polymer or a printed active composites (PAC’s). These government in COVID-19 Quarantine Alert System
smart materials due to their material properties can be (CQAS) to track if a person has jumped quarantine or
configured to specific shape when subjected to external escaped from isolation.
stimuli like water, heat, pressure, temperature etc. The 316. Solution: (d)
most popular technology being shape memory alloy,
where a change of temperature triggers a shape change. Exp) Option d is correct.
Other successful approaches use electro-active polymers, Coal tar is a by-product of bituminous coal (45-85%
pressurized fluids or gasses, chemical stimulus and even Carbon) or black coal, which contains a tar-like substance
in response to light. These objects are in contrast to 3D called bitumen, or asphalt. It is in form of viscous liquid
printed objects that are rigid. mixture containing hydrocarbon compounds, derived
During the 4D process, a geometric code is added along with coke, from the destructive distillation of coal
in coking ovens. The possible applications of coal tar are
that contains “instructions” on how a shape will
as follow -

Workbook 190
.
SCIENCE & TECHNOLOGY

Statement 1 is correct. Naphthalene also called white tar Isotopes are those elements which have the same atomic
and tar camphor is a white solid that evaporates easily. It number but different mass numbers. For example, take
is present in fuels such as petroleum and coal and has the case of hydrogen atom, it has three atomic species,
been used in mothballs and moth flakes. Naphthalene namely protium , deuterium and tritium. The atomic
balls are obtained from coal tar. number of each one is 1, but the mass number is 1, 2 and
Statement 2 is correct. Coal tar belongs to a class of drugs 3, respectively.
known as keratoplastics. Typical coal tar products work Statement 1 is correct. Through food radiation, food can
by causing the skin to shed dead cells from its top layer
be purified, and cleansed of any disease-causing bacteria.
and slow down the growth of skin cells. Dermatologists
may use tar and extracts of crude coal tar in skin Disease-causing bacteria are killed when the food is
treatment like the treatment of chronic eczema (Eczema, exposed to certain levels of gamma rays. The emissions
or dermatitis, is a skin condition that causes patches of of these gamma rays are caused by radioactive isotopes.
itchiness, inflammation, swelling, and cracked skin.). Statement 2 is correct. Different types of isotopes are
Statement 3 is correct. Perfume uses several natural used to measure thickness of metal or plastic sheets.
ingredients and substances for its manufacturing. Coal The strength of the radiations penetrating the subject
tar is a substance which is used as fixatives along with materials indicates the precise thickness of these sheets.
mosses, resins, or synthetic chemicals enable perfume to Statement 3 is correct..Isotopes make effective tracers as
evaporate slowly and emit odours longer.
detection of isotope radioactivity is a very simple process.
Statement 4 is correct. Coal Tar enhances the Tensile For instance, the isotope tritium 3H can be used to trace
Strength of Reclaim Rubber to a great extent and used and discover any leaks present in underground pipes.
as a softening solvent in the manufacturing of Reclaim
rubber. Hence it is a must ingredient for manufacturing Statement 4 is correct. Isotopes can be used to generate
High Tensile Reclaim Rubber. electricity for example, the isotope plutonium-238 is used
to generate electrical power in spacecraft. This power is
317. Solution: (d) generated when the isotope decays, producing heat which
Exp) Option d is correct. is then converted by circuit devices into electricity.

191 Workbook
.
ENVIRONMENT

ENVIRONMENT
ENVIRONMENT AND ECOLOGY
(c) 3 only
3.1. Previous Years Questions (d) 1, 2 and 3
1. In the context of ecosystem productivity, marine 5. Consider the following kinds of organisms:
upwelling zones are important as they increase the
[ UPSC CSE Pre. 2012]
marine productivity by bringing the
[ UPSC CSE Pre. 2011] 1. Bacteria
1. Decomposer microorganism to the surface. 2. Fungi
2. Nutrients to the surface. 3. Flowering plants
3. Bottom-dwelling organisms to the surface. Some species of which of the above kinds of
Which of the statements given above is/are correct? organisms are employed biopesticides?
(a) 1 and 2 (a) 1 only
(b) 2 only (b) 2 and 3 only
(c) 2 and 3 (c) 1 and 3 only
(d) 3 only (d) 1, 2 and 3
2. If a tropical rain forest is removed, it does not 6. Consider the following kinds of organisms:
regenerate quickly as compared to a tropical [UPSC CSE Pre. 2012]
deciduous forest. This is because 1. Bat
[UPSC CSE Pre. 2011] 2. Bee
(a) The soil of rain forest is deficient in nutrients 3. Bird
(b) Propagules of the trees in a rain forest have poor
viability Which of the above is/are pollinating agent/agents?
(c) The rain forest species are slow growing (a) 1 and 2 only
(d) Exotic species invade the fertile soil of rain forest (b) 2 only
(c) 1 and 3 only
3. Consider the following: (d) 1, 2 and 3
[UPSC CSE Pre. 2011]
1. Photosynthesis 7. The Millennium Ecosystem Assessment describes
2. Respiration the following major categories of ecosystem
services—provisioning, supporting, regulating,
3. Decay of organic matter
preserving and cultural. Which one of the
4. Volcanic action
following is supporting service?
Which of the above add carbon dioxide to the carbon [UPSC CSE Pre. 2012]
cycle on Earth?
(a) Production of food and water
(a) 1 and 4 only (b) Control of climate and disease
(b) 2 and 3 only
(c) Nutrient cycling and crop pollination
(c) 2, 3 and 4 only
(d) Maintenance of diversity
(d) 1, 2, 3 and 4
8. With reference to the food chains in ecosystems,
4. What would happen if phytoplankton of an ocean
is completely destroyed for some reason? which of the following kinds of organism is/are
known as decomposer organism/organisms?
[UPSC CSE Pre. 2012]
[UPSC CSE Pre. 2013]
1. The ocean as a carbon sink would be adversely
affected. 1. Virus
2. The food chains in the ocean would be adversely 2. Fungi
affected. 3. Bacteria
3. The density of ocean water would drastically Select the correct answer using the codes given
decrease. below:
Select the correct answer using the codes given (a) 1 only
below: (b) 2 and 3 only
(a) 1 and 2 only (c) 1 and 3 only
(b) 2 only (d) 1, 2 and 3

Workbook 192
.
ENVIRONMENT

9. In the grasslands, trees do not replace the grasses but also its functional role in the community of
as a part of an ecological succession because of: organisms? [UPSC CSE Pre. 2013]
[UPSC CSE Pre. 2013] (a) Ecotone
(a) Insects and fungi (b) Ecological niche
(b) Limited sunlight and paucity of nutrients (c) Habitat
(c) Water limits and fire (d) Home range
(d) None of the abovess 15. Which one of the following is the best description
10. On the planet earth, most of the freshwater exists of the term ‘ecosystem’? [UPSC CSE Pre. 2015]
as ice caps and glaciers. Out of the remaining (a) A community of organisms interacting with one
freshwater, the largest proportion another
[UPSC CSE Pre. 2013] (b) That part of the Earth which is inhabited by
(a) is found in atmosphere as moisture and clouds living organisms
(b) is found in freshwater lakes and rivers (c) A community of organisms together with the
(c) exists as groundwater environment in which they live
(d) exists as soil moisture (d) The flora and fauna of a geographical area

11. With reference to food chains in ecosystems, 16. Biological Oxygen Demand (BOD) is a standard
consider the following statements: criterion for [UPSC CSE Pre. 2017]
UPSC CSE Pre. 2013] (a) Measuring oxygen levels in blood
(b) Computing oxygen levels in forest ecosystems
1. A food chains illustrates the order in which a
(c) Pollution assay in aquatic ecosystems
chain of organisms feed upon each other.
(d) Assessing oxygen levels in high altitude regions
2. Food chains are found within the populations of
a species. 17. Due to some reasons, if there is a huge fall in the
3. A food chain illustrates the numbers of each population of species of butterflies, what could be
organism which are eaten by others. its likely consequence/consequences?
Which of the statements given above is/are correct? [ UPSC CSE Pre. 2017]
(a) 1 only 1. Pollination of some plants could be adversely
(b) 1 and 2 only affected.
(c) 1, 2 and 3 2. There could be a drastic increase in the fungal
(d) None infections of some cultivated plants.
3. It could lead to a fall in the population of some
12. Consider the following organisms: species of wasps, spiders and birds.
[UPSC CSE Pre. 2013] Select the correct using the code given below:
1. Agaricus (a) 1 only
2. Nostoc (b) 2 and 3 only
3. Spirogyra (c) 1 and 3 only
Which of the above is/are used as biofertilizer/ (d) 1, 2 and 3
biofertilizers? 18. Consider the following statements:
(a) 1 and 2 [UPSC CSE Pre. 2018]
(b) 2 only
1. Most of the world’s coral reefs are in tropical
(c) 2 and 3
waters.
(d) 3 only
2. More than one-third of the world’s coral reefs are
13. Which of the following adds/add nitrogen to the located in the territories of Australia, Indonesia
soil? [UPSC CSE Pre. 2013] and Philippines.
1. Excretion of urea by animals 3. Coral reefs host far more number of animal
2. Burning of coal by man phyla than those hosted by tropical rainforests.
3. Death of vegetation Which of the statements given above is/are correct?
Select the correct answer using the codes given (a) 1 and 2 only
below: (b) 3 only
(a) 1 only (c) 1 and 3 only
(b) 2 and 3 only (d) 1, 2 and 3
(c) 1 and 3 only 19. Which of the following leaf modifications occur(s)
(d) 1, 2 and 3 in the desert areas to inhibit water loss?
14. Which one of the following terms describes not [UPSC CSE Pre. 2018]
only the physical space occupied by an organism, 1. Hard and waxy leaves

193 Workbook
.
ENVIRONMENT

2. Tiny leaves 23. Which of the following are nitrogen-fixing plants?


3. Thorns instead of leaves [UPSC CSE Pre. 2022]
Select the correct answer using the codes given 1. Alfalfa
below: 2. Amaranth
(a) 2 and 3 only 3. Chickpea
(b) 2 only 4. Clover
(c) 3 only 5. Purslane (Kulf
(d) 1, 2 and 3 6. Spinach
20. Which of the following statements best describes Select the correct answer using the code given below:
“carbon fertilization”? [UPSC CSE Pre. 2018] (a) 1, 3 and 4 only
(a) Increased plant growth due to increased (b) 1, 3, 5 and 6 only
concentration of carbon dioxide in the (c) 2, 4, 5 and 6 only
atmosphere. (d) 1, 2, 4, 5 and 6
(b) Increased temperature of Earth due to increased 24. “Biorock technology” is talked about in which one
concentration of carbon dioxide in the of the following situations? [UPSC CSE Pre. 2022]
atmosphere. (a) Restoration of damaged coral reefs
(c) Increased acidity of oceans as a result of (b) Development of building materials using plant
increased concentration of carbon dioxide in the residues
atmosphere. (c) Identification of areas for exploration/extraction
(d) Adaptation of all living beings on Earth to the of shale gas
climate change brought about by the increased (d) Providing salt licks for wild animals in forests/
concentration of carbon dioxide in the protected areas
atmosphere
Practice Questions
21. “System of Rice Intensification” of cultivation, in
which alternate wetting and drying of rice fields is 3.2. Ecology and Ecosystem
practised, results in: [UPSC CSE Pre. 2022]
1. Reduced seed requirement 25. Which one of the following is the correct
2. Reduced methane production sequence of ecosystems in the order of decreasing
productivity?
3. Reduced electricity consumption
(a) Oceans, lakes, grasslands, mangroves
Select the correct answer using the code given below: (b) Mangroves, oceans, grasslands, lakes
(a) 1 and 2 only (c) Mangroves, grasslands, lakes, oceans
(b) 2 and 3 only (d) Oceans, mangroves, lakes, grasslands
(c) 1 and 3 only
26. On the planet earth, most of the freshwater exists
(d) 1, 2 and 3
as ice caps and glaciers. Out of the remaining
22. With reference to polyethylene terephthalate, the freshwater, the largest proportion
use of which is so widespread in our daily lives, (a) is found in atmosphere as moisture and clouds
consider the following statements: (b) is found in freshwater lakes and rivers
[UPSC CSE Pre. 2022] (c) exists as groundwater
1. Its fibres can be blended with wool and cotton (d) exists as soil moisture
fibres to reinforce their properties. 27. Consider the following:
2. Containers made of it can be used to store any 1. Photosynthesis
alcoholic beverage. 2. Respiration
3. Bottles made of it can be recycled into other 3. Decay of organic matter
products.
4. Volcanic action
4. Articles made of it can be easily disposed of by
Which of the above add carbon dioxide to the carbon
incineration without causing greenhouse gas
cycle on Earth?
emissions.
(a) 1 and 4 only
Which of the statements given above are correct? (b) 2 and 3 only
(a) 1 and 3 (c) 2, 3 and 4 only
(b) 2 and 4
(d) 1, 2, 3 and 4
(c) 1 and 4
(d) 2 and 3 28. Consider the following pair:

Workbook 194
.
ENVIRONMENT

Term Meaning Select the correct answer using the code given below:
(a) 1 only
1. Ecotone A transitional area of (b) 1 and 2 only
vegetation between (c) 2 only
two different plant (d) 3 only
communities.
32. Which of the following ecological pyramid can be
2. Ecotype A population that both upright and inverted?
is adapted to local
environmental 1. Pyramid of Biomass
conditions. 2. Pyramid of Numbers
3. Pyramid of Energy
3. Ecocline A gradation from one
ecosystem to another Select the correct answer using the code given below:
when there is no sharp (a) 1 and 2 only
boundary between the (b) 1 only
two. (c) 2 and 3 only
Which of the above given pair/s is/are correctly (d) 1, 2 and 3
matched? 33. Consider the following types of biotic interaction
(a) 1 only and its examples.
(b) 2 only
(c) 2 and 3 only Biotic interaction example
(d) 1, 2 and 3
1. Commensalism A. Biological method of
29. Consider the following biomes: pest control
1. Estuary 2. Predation B. Introduction of goats
2. Tundra in Galapagos islands
3. Oceans consisting of tortoise
4. Temperate grassland 3. Competition C. Association between
Arrange the above biomes in ascending order based the cattle egret and
on the Net Primary Productivity. grazing cattle
(a) 2-3-4-1 Select the correct answer using the code given below:
(b) 2-3-1-4 (a) 1-C, 2-B, 3-A
(c) 3-2-4-1 (b) 1-C, 2-A, 3-B
(d) 3-2-1-4 (c) 1-A, 2-C, 3-B
30. Which of the following statement is/are incorrect (d) 1-B, 2-A, 3-C
about biotic interactions in an ecosystem? 34. Which of the following statement is the correct
1. Lichens on a tree bark shows the symbiotic sequence of food chain?
relation of parasitism. (a) Plankton - Snail - Shark - Mackerel
2. In competition fitness of one species is (b) Phytoplankton - Copepod - Bluefish - Swordfish
significantly lowered in presence of another (c) Zooplankton - Phytoplankton - Anchovy - Tuna
species. (d) Algae- Zooplankton – Shark - Tuna
3. Phytophagous’ has air breathing roots and it is
an example of commensalism with respect to its 35. Consider the following statements:
host plant. 1. Long life
Select the correct answer using the code given below: 2. Insoluble in fats
(a) 1 only 3. Biologically inactive
(b) 1 and 3 only Which of the statements given above is/are the
(c) 2 and 3 only propertiesrequiredforapollutantforbiomagnification
(d) 3 only to occur?
31. With respect to succession in ecology which of the (a) 1 only
following species is/are correct? (b) 1 and 3 only
(c) 2 and 3 only
1. Seres is the entire sequence of succession that (d) 1,2 and 3
leads to the climax community.
2. Process of succession occur faster in the middle 36. Which of the following factors are among the
of the large continents. abiotic components of environment?
3. Under primary succession a well-developed soil 1. Symbionts
is colonized by pioneer species at the first. 2. Soil

195 Workbook
.
ENVIRONMENT

3. Decomposers 1. Dominant species modify the growth of other


4. Water species in a community.
5. Non-green plants 2. Community with no single dominant species is
Select the correct answer using the code given below: characterized by low species diversity.
(a) 1, 2 and 3 only 3. Organisms in a community develop vertical
(b) 1, 2 and 4 only stratification to avoid conflict with other species.
(c) 2 and 4 only Which of the statements given above is/are correct?
(d) 2, 4 and 5 only (a) 1 and 2 only
37. Consider the following statements: (b) 2 and 3 only
1. Some bacteria and algae are primary producers. (c) 1 and 3 only
2. Saprotrophs are decomposers of dead organic (d) 1, 2 and 3
matter. 42. Which of the following statements correctly
3. Human beings are omnivores organisms. explains the meaning of the ‘Carrying Capacity’ of
Which of the statements given above are correct? an ecosystem?
(a) 1 and 2 only (a) It refers to the number of resources each
(b) 2 and 3 only individual within the community is consuming.
(c) 1 and 3 only (b) It is the maximum population size that an
(d) 1, 2 and 3 ecosystem can support without degrading itself.
38. In the context of Ecotones, consider the following (c) It refers to the total size of renewable resources
statements: available in the ecosystem.
1. It is a zone of junction between two or more diverse (d) It is the sum of all the physical and chemical
ecosystems. factors that a species needs to survive in an
2. Mangrove forests and grasslands are examples of ecosystem.
ecotones. 43. Consider the following statements:
Which of the statements given above is/are correct? 1. Eco-sensitivity is characterized by low levels of
(a) 1 only resilience that cannot be restored to its original
(b) 2 only form easily.
(c) Both 1 and 2 2. Acclimation is the compensatory adjustment
(d) Neither 1 nor 2
of organism to change in the environment in
39. Consider the following statements: natural condition.
1. Niche describes unique functional role of a species 3. Adjustments within a single organism that
in an ecosystem. improve survival in response to environmental
2. No two species can have identical niches in the change is known as Acclimatization.
same habitat. Which of the statements given above is/are correct?
Which of the statements given above is/are correct? (a) 1 only
(a) 1 only (b) 2 and 3 only
(b) 2 only (c) 3 only
(c) Both 1 and 2 (d) 1 and 3 only
(d) Neither 1 nor 2
44. Consider the following pairs:
40. Consider the following pairs:
Terms Description
Type of Niche Feature
1. Environment Area composed of
1. Food niche Competition with biotic and abiotic
species for food components where
2. Physical niche Reproduction organisms live.
3. Habitat niche Land shape 2. Biome Narrow zone on the
Which of the pairs given above is/are correctly surface of the earth
matched? where soil, water, and
(a) 1 only air combine to sustain
(b) 1 and 2 only life.
(c) 2 and 3 only 3. Ecosystem A system formed by the
(d) 1, 2 and 3 interaction of abiotic
41. Consider the following statements with respect to and biotic components
the: of the environment.

Workbook 196
.
ENVIRONMENT

Which of the pair/s given above is/are correctly 1. Secondary Succession is faster than Primary
matched? Succession.
(a) 1 and 2 only 2. Species diversity is more in Primary Succession
(b) 3 only than in Secondary Succession.
(c) 1 and 3 only 3. Secondary succession can occur after a wildfire,
(d) 1,2 and 3 while primary succession occurs when a new
land is formed.
45. Which of the following statement best defines the
Net Primary Productivity of an ecosystem? 4. Mosses are pioneer community and oak forest is
a climax community.
(a) It is the difference between the energy that is
fixed by the autotrophs and their own respiration Which of the statements given above is/ are correct?
losses. (a) 1, 2 and 3 only
(b) Amount of biomass produced per unit area over (b) 2, 3 and 4 only
a time period by plants during photosynthesis. (c) 1, 3 and 4 only
(c) The rate of biomass production by the autotrophs (d) 1, 2, 3 and 4
during photosynthesis. 50. Which of the following statements regarding the
(d) The rate of formation of new organic matter by term ‘Ecological Niche’ is incorrect?
consumers.
(a) It refers to functional role of a species within an
46. With reference to the ecological succession in a ecosystem.
habitat, consider the following statements. (b) No two species within an ecosystem share the
1. Autogenic Succession refers to the changes same niche.
introduced by living inhabitants of the (c) Ecological niches are not well developed in
community itself. Primary Successions.
2. Allogenic Successions refers to the changes (d) It refers to interaction only with the biotic
brought about by the outside forces which are components.
abiotic in nature. 51. With reference to ecosystem, which of the
Which of the statements given above is/are correct? following statement correctly explains the process
(a) 1 only of catabolism?
(b) 2 only (a) Creation of bigger structures by bonding of
(c) Both 1 and 2 smaller units like cells, or amino acids.
(d) Neither 1 nor 2 (b) Conversion of dead matter into humus.
(c) Downward movement of water soluble inorganic
47. In ecological studies, biological interaction known nutrients into the soil.
as amensalism refers to which of the following? (d) Breakdown of detritus into smaller particles by
(a) A large tree having shade upon a small plant. chemical process.
(b) Two species competing to eat the same food.
(c) Parasites getting nourishment from the host. 52. Consider the following statements with reference
(d) Beetles living upon the cow dung. to the ecological community:
1. The species constituting the majority of
48. Consider the following statements: the biomass are the dominant species in an
1. The pioneer species are those species which ecosystem.
initiate the development of an ecological 2. There may be more than one dominant species
community in an area with currently no life in some ecological communities.
form’s existence. 3. Plants and animals of each layer differ in size,
2. Pioneer species can tolerate and trade through behaviour and adaptation in order to avoid
any type of prevailing harsh environmental competition.
conditions. Which of the statements given above is/are correct?
3. In pioneer community, the plants and animals (a) 2 only
are in balance with each other and their (b) 3 only
environment. (c) 1 and 3 only
Which of the statements given above is/are correct? (d) 1, 2 and 3
(a) 1 and 2 only
53. Which among the following is a reason for ecotone
(b) 2 only
to have a greater diversity of species than the
(c) 1 and 3 only
neighbouring ecosystems?
(d) 1, 2 and 3
(a) Lack of well-defined boundaries for the species
49. With reference to the concepts related to (b) Higher availability of physical space
“Ecological Succession”, consider the following (c) Lack of dominant species in the ecosystem
statements: (d) Lack of predator species in the ecotone

197 Workbook
.
ENVIRONMENT

54. Which of the following statements correctly


Biotic interaction example
describe the term ‘Ecological footprint’?
(a) The amount of carbon-dioxide that any activity 1. Commensalism A. Biological method
by any human or machine produces. of pest control
(b) The amount of oxygen required to replenish the 2. Predation B. Introduction of
environment in a particular area. goats in Galapagos
(c) The total greenhouse gas (GHG) emissions islands consisting
caused by an individual, organization or place of tortoise
expressed as carbon dioxide equivalent. 3. Competition C. Association between
(d) The area needed to provide the renewable the cattle egret and
resources that a population consumes and to grazing cattle
absorb its waste.
Select the correct answer using the code given below:
55. Consider the following statements with reference (a) 1-C, 2-B, 3-A
to the biogeochemical cycles: (b) 1-C, 2-A, 3-B
1. A perfect nutrient cycle is one in which the (c) 1-A, 2-C, 3-B
nutrients are replaced as fast as they are used up. (d) 1-B, 2-A, 3-C
2. Sedimentary cycles are considered to be a perfect
biogeochemical cycle. 3.3. Functions of ecosystem
3. The nutrients are released from lithosphere 59. Consider the following statements regarding types
through weathering of rocks in a sedimentary of food chains:
cycle. 1. In the grazing food chain the primary source
Which of the statements given above is/are correct? of energy is living plant biomass while in the
(a) 1 only detritus food chain the source of energy is dead
(b) 3 only organic matter or detritus.
(c) 1 and 3 only 2. Detritus food chain is totally independent of
(d) 1 and 2 only grazing food chain.
56. With reference to the ecological pyramids, Which of the statements given above is/are correct?
consider the following statements (a) 1 only
(b) 2 only
1. It does not include saprophytes.
(c) Both 1 and 2
2. It do not consider the same species belonging to (d) Neither 1 nor 2
two or more trophic levels.
3. It do not accommodate food web. 60. Consider the following statements:
1. In aquatic food chain, zooplanktons are the
Which of the statements given above are correct? primary producers.
(a) 1 and 2 only 2. In grazing food chain, the Sun is the primary
(b) 2 and 3 only source of energy.
(c) 1 and 3 only
(d) 1, 2 and 3 Which of the statements given above is/are correct?
(a) 1 only
57. With reference to Nitrogen Fixation, consider the (b) 2 only
following statements: (c) Both 1 and 2
1. Atmospheric phenomenon such as thunder and (d) Neither 1 nor 2
lightning contribute to nitrogen fixation. 61. With reference to detritus food chain, consider the
2. Microorganisms like Azotobacter and following:
Beijemickia are capable of fixing atmospheric 1. Bacteria and fungi obtain energy by decomposing
nitrogen. dead organic substances.
3. Nitrogen fixation is the process of conversion of 2. It helps in recycling inorganic nutrients.
nitrates into elemental nitrogen. Which of the statements given above is/are correct?
Which of the statements given above is/are correct? (a) 1 only
(a) 1 and 2 only (b) 2 only
(b) 1 only (c) Both 1 and 2
(c) 2 and 3 only (d) Neither 1 nor 2
(d) 1, 2 and 3
62. Consider the following statements regarding
58. Consider the following types of biotic interaction “Ecological Pyramids”:
and its examples? 1. The Pyramid of Numbers is always inverted.

Workbook 198
.
ENVIRONMENT

2. In Aquatic ecosystems, the Pyramid of Biomass 67. With reference to the Nitrogen Cycle, consider the
can be inverted. following statements:
Which of the statements given above is/are incorrect? 1. Nitrogen fixation is the process of conversion
(a) 1 only of elemental nitrogen to ammonia, nitrites or
(b) 2 only nitrates.
(c) Both 1 and 2 2. Atmospheric phenomenon such as thunder and
(d) Neither 1 nor 2 lightning contribute to nitrogen fixation.
3. Microorganisms like Azotobacter are capable
63. Consider the following statements: of fixing atmospheric nitrogen into ammonium
1. Only about 10 per cent of the energy is ions.
transferred to each trophic level from the lower
trophic level. Which of the statement(s) given above is/are correct?
2. Pyramid of energy is always upright. (a) 1 only
(b) 1 and 2 only
Which of the statements given above is/are correct? (c) 1 and 3 only
(a) 1 only (d) 1, 2 and 3
(b) 2 only
(c) Both 1 and 2 68. With reference to nutrient cycling, consider the
(d) Neither 1 nor 2 following:
1. The nutrient cycling in an entire ecosystem is
64. Consider the following statements: known as biogeochemical cycle.
1. Bioaccumulation is the process by which a 2. Phosphorous circulates mainly by the means of
pollutant from the environment enters a food a gaseous cycle.
chain.
3. Hydrologic cycle is driven by solar energy.
2. In Biomagnification, there is an increase in
concentration of pollutants from one link in a Which of the statements given above is/are correct?
food chain to another. (a) 1 and 2 only
3. In order for biomagnification to occur, the (b) 3 only
pollutant must be biologically inactive. (c) 1 and 3 only
(d) 2 and 3 only
Which of the statements given above is/are correct?
(a) 1 only 69. Which of the following statements is/are correct
(b) 1 and 2 only about Sulphur cycle?
(c) 2 and 3 only 1. It is a sedimentary cycle as it does not involve
(d) 1, 2 and 3 gaseous component.
2. Sulphur is taken by plants in the form of
65. With reference to the Phosphorus Cycle, consider
sulphates.
the following statements:
1. Phosphorus is released in the ecosystem by Select the correct answer using the code given below:
weathering of rocks, erosion and respiration of (a) 1 only
organisms. (b) 2 only
2. Phosphorus occurs most abundantly in nature in (c) Both 1 and 2
the form of phosphate ion. (d) Neither 1 nor 2
3. The Phosphorus cycle are unidirectional moving 70. With reference to ecological succession, consider
from soil to plants and to animals. the following:
Which of the statement(s) given above is/are correct? 1. The entire sequence of communities that
(a) 1 only successively change in a given area are called
(b) 2 only transitional communities.
(c) 1 and 2 only 2. A community that is in near equilibrium with
(d) 1 and 3 only the environment is called a climax community.
66. Consider the following statements: Which of the statements given above is/are correct?
1. Maximum concentration of global carbon is (a) 1 only
found in the atmosphere followed by the oceans. (b) 2 only
2. Carbon component of the atmosphere decreases (c) Both 1 and 2
during photosynthesis. (d) Neither 1 nor 2
Which of the statement(s) given above is/are correct? 71. Consider the following statements:
(a) 1 only 1. Secondary succession occurs after complete or
(b) 2 only partial destruction of existing community.
(c) Both 1 and 2 2. Secondary succession is relatively slower as
(d) Neither 1 nor 2 compared to primary succession.

199 Workbook
.
ENVIRONMENT

3. When succession is brought about by living (b) 1, 2 and 4 only


inhabitants of that community itself, the process (c) 2 and 3 only
is called Autotrophic succession. (d) 1 and 3 only
Which of the statements given above is/are correct? 75. With reference to the Carbon and Phosphorus
(a) 1 only Nutrient Cycles, consider the following statements:
(b) 1 and 3 only 1. Phosphorous cycle completes faster than the
(c) 2 and 3 only Carbon cycle.
(d) 1, 2 and 3
2. Unlike Carbon, Phosphorus does not cycle
72. Consider the following statements: through the atmosphere.
1. Net primary productivity of an ecosystem is (a) 1 only
the rate of production of organic matter during (b) 2 only
photosynthesis. (c) Both 1 and 2
2. Primary productivity depends on the plant (d) Neither 1 nor 2
species inhabiting a particular area.
76. With reference to the features of a Sedimentary
3. Ecological succession is characterized by Biogeochemical Cycle, consider the following
increased productivity. statements:
Which of the statements given above is/are correct? 1. These cycles include movement of nutrients
(a) 1 and 2 only through rocks as well as aquatic bodies.
(b) 3 only 2. These have a longer time span compared to
(c) 2 and 3 only Gaseous Biogeochemical Cycle.
(d) 1, 2 and 3
3. It is an example of a perfect Biogeochemical
73. Consider the following pairs: cycle.
Which of the statements given above are correct?
Environmental Description
(a) 1 and 2 only
Process (b) 2 and 3 only
1. Fragmentation Detritivores break (c) 1 and 3 only
down detritus into (d) 1, 2 and 3
smaller particles. 77. In the context of ecology, the term ‘Seres’ refers to
which of the following?
2. Humification Degradation of the
(a) A zone of junction between two or more diverse
humus and release of ecosystems.
inorganic nutrients. (b) A unique functional role or place of a species in
3. Mineralisation Accumulation of an ecosystem.
organic matter into a (c) The terrestrial part of the biosphere.
(d) The stages leading to the complex community.
substance
Which of the pair given above is/are correctly 3.4. Terrestrial Ecosystem
matched?
(a) 1 only 78. Consider the following statements:
(b) 2 only 1. Tundra is found in the higher latitudes near the
(c) 1 and 3 only polar regions only.
(d) 2 and 3 only 2. Mammals in tundra region have small tail and
74. Consider the following statements: small ear.
1. The pyramid of numbers is inverted in the Which of the statements given above is/are correct?
detritus food chain. (a) 1 only
2. Pyramid of Biomass takes into account the (b) 2 only
amount of biomass produced by each trophic (c) Both 1 and 2
level. (d) Neither 1 nor 2
3. Pyramid of Biomass is always upright as the 79. “The annual rainfall exceeds 200 cm and is
number of organisms decline in higher trophic generally distributed throughout the year. The
levels. flora is highly diversified. The high rate of leaching
4. Pyramid of energy is inverted in aquatic makes these soils virtually useless for agricultural
ecosystem. purposes.”
Which of the statements given above is/are correct? The above information best describes which of the
(a) 1 and 2 only following region?

Workbook 200
.
ENVIRONMENT

(a) African Savannah 1. Location of a place in the rain shadow area of a


(b) Temperate deciduous forests mountain range.
(c) Tropical Rain forests 2. Location of a place in the interiors of a continent.
(d) Temperate Grassland Select the correct answer using the code given below.
80. In India, Tropical Semi-evergreen forests are (a) 1 only
found in which of the following regions? (b) 2 only
1. Western Ghats (c) Both 1 and 2
2. Andaman and Nicobar Islands (d) Neither 1 nor 2
3. Eastern Himalayas 85. Which of the following are the adaptations by the
Select the correct answer using the codes given desert plants to survive hot and dry conditions?
below. 1. Leaves are absent or reduced in size.
(a) 1 only 2. Leaves and stem are succulent.
(b) 1 and 2 only 3. The roots of desert plants are small.
(c) 2 and 3 only
Select the correct answer using the codes given
(d) 1, 2 and 3
below.
81. Which of the following are the causes of forest (a) 1 only
fires in India? (b) 1 and 2 only
1. Global warming (c) 2 and 3 only
2. Lack of soil moisture (d) 1, 2 and 3
3. Lightening 86. What is Miyawaki Method, recently seen in news?
4. Anthropogenic activities (a) It is a conventional slash and burn agriculture
Select the correct answer using the code given below: practiced in East Asia
(a) 1 and 2 only (b) It is a technique to build dense, native forests to
(b) 2, 3 and 4 only ensure that plant grow faster.
(c) 3 and 4 only (c) It is a is a traditional system of medicine
(d) 1, 2, 3 and 4 practiced in Tibet.
(d) It is a system designed to harvest surface runoff
82. Which of the following statements is/are incorrect water for agriculture.
with regard to deforestations?
1. Agriculture activities is the leading cause of 87. With respect to the various types of living
deforestation. organisms found in ecosystems, consider the
2. Deforestations make oceans acidic. following pairs:
3. Deforestation is responsible for the increasing Type of Organism Example
rate of evapotranspiration.
Select the correct answer using the code given below: 1. Autotrophs Elephant Grass
(a) 1 only 2. Phagotrophs Snake
(b) 1 and 3 only 3. Osmotrophs Bears
(c) 2 and 3 only
(d) 3 only Which of the pairs given above is/ are correct?
(a) 1 and 2 only
83. Among the following which could be the possible (b) 2 and 3 only
impact(s) of heavy grazing on grasslands? (c) 1 only
1. The grassland becomes vulnerable to water and
(d) 1, 2 and 3
wind erosion.
2. The grassland becomes prone to invasion of
xerophytic plants and burrowing animals. 3.5. Aquatic Ecosystem
3. Breakdown of the biogeochemical cycles of 88. With reference to cold-water corals, consider the
water and nitrogen can take place. following statements:
Select the correct answer using the code given below. 1. There are more cold-water coral reefs than
(a) 2 only tropical reefs in the oceans.
(b) 1 and 3 only 2. Cold-water corals requires zooxanthellae to
(c) 2 and 3 only survive.
(d) 1, 2 and 3 3. Lophelia reefs are coldwater corals which can
84. Which of the following factor(s) can lead to the host species of fish and invertebrates.
formation of desert ecosystems? Which of the statements given above is/are correct?

201 Workbook
.
ENVIRONMENT

(a) 1 and 2 only 2. Periphytons Organisms which


(b) 2 only remain attached to
(c) 1 and 3 only stems and leaves of
(d) 2 and 3 only rooted
plants or substances
89. With reference to aquatic ecosystems, consider the
which emerge above
following statements: the bottom mud.
1. The aquatic organisms have narrow temperature
3. Benthos They are the organisms
tolerance limit. which live in the
2. Suspended particulate matters like clay and silt bottom of the water
reduce the productivity of aquatic habitat. mass.
3. Increase in temperature increases the rate of Which of the pairs given above are correctly
matched?
oxygen depletion.
(a) 1 and 2 only
Select the correct answer using the codes given (b) 2 and 3 only
below. (c) 1 and 3 only
(a) 1 only (d) 1, 2 and 3
(b) 1 and 3 only 93. With respect to productivity in aquatic ecosystems,
(c) 2 and 3 only which of the following statement(s) is/are correct?
(d) 1, 2 and 3 1. Active photosynthesis is confined only to the
90. Which among the following is/are the applications photic zone.
of seaweeds? 2. The rate of photosynthesis is affected by the
transparency of water.
1. It acts as a bioindicator.
3. Respiration is restricted to the aphotic zone.
2. Helps in removal of heavy metals from marine
Select the correct answer using the code given below:
ecosystem.
(a) 1 and 2 only
3. Production of bioethanol (b) 2 and 3 only
4. Preparing toothpaste and cosmetics. (c) 3 only
Which of the statements given above is/are correct? (d) 1, 2 and 3
(a) 1 only 94. It is the process of natural aging of a lake by
(b) 2 and 3 only nutrient enrichment of its water. This process
(c) 1, 2 and 3 only happens naturally in environment but due to
increase in pollutants from human activities
(d) 1, 2, 3 and 4
like effluents from the industries, nutrients
91. Consider the following statements regarding from agriculture run-off, this process has been
aquatic ecosystems: significantly speeded up. Which of the following
processes has been defined here?
1. Lentic water system includes freshwater streams,
(a) Biomagnification
springs and rivers.
(b) Winterkill
2. Estuaries are more productive as compared to (c) Chemical Oxygen Demand
the adjacent sea or river. (d) Eutrophication
Which of the statements given above is/are correct? 95. Consider the following statements regarding
(a) 1 only Phytoplankton:
(b) 2 only 1. Their population increases with the depth in the
(c) Both 1 and 2 ocean water.
(d) Neither 1 nor 2 2. Their total biomass is greater than the total
plants on land.
92. Consider the following pairs: 3. They are found in marine as well as freshwater
environments.
Aquatic Organisms Characteristics
Which of the statements given above is/are correct?
1. Neustons They are active
swimmers and are (a) 1 only
relatively large and (b) 1 and 3 only
powerful. (c) 2 and 3 only
(d) 1, 2 and 3

Workbook 202
.
ENVIRONMENT

96. With reference to ‘Algal Blooms’, consider the 2. Bhitarkanika Andhra Pradesh
following statements: mangroves
1. Harmful Algal Blooms increases the level of
dissolved oxygen in the water. 3. Ratnagiri Maharashtra
mangroves
2. Presence of unusually warm water is conducive
for algal blooms. Which of the pairs given above is/are correctly
matched?
Which of the statement(s) given above is/are correct?
(a) 1 and 2 only
(a) 1 only (b) 3 only
(b) 2 only (c) 2 only
(c) Both 1 and 2 (d) 1, 2 and 3
(d) Neither 1 nor 2
101. Which of the following is/are the causes of coral
97. Which of the following adaptations are found in bleaching?
the Mangrove vegetation? 1. El Nino
1. Pneumatophores or arching prop roots that go 2. Increased Ocean temperatures
down into the water from the trunk of tree
3. Increased level of carbon dioxide in the oceans
2. Thick leaves with salt secreting glands
Select the correct answer using the code given below:
3. Viviparity mode of reproduction
(a) 1 and 2 only
4. Adventitious roots that come out from the mud (b) 2 only
to help plant respire in anaerobic soil conditions (c) 3 only
Select the correct answer using the codes given (d) 1, 2 and 3
below:
102. With reference to a pond ecosystem, consider the
(a) 1 and 2 only following statements:
(b) 1, 3 and 4 only
1. There is cyclic movement of energy in pond
(c) 2 and 3 only
ecosystem.
(d) 1, 2, 3 and 4
2. The conversion of inorganic into organic
98. Consider the following statements: material by algae takes place with the help of
1. Gujarat has the highest percentage of India’s total sunlight.
Mangrove cover. 3. In pond ecosystem, limnetic zone is devoid of
2. Mangroves exhibit Viviparity mode of fish population.
reproduction. Which of the statements given above is/are incorrect?
3. It protects coastal lands from tsunami, hurricanes (a) 1 only
and floods. (b) 2 only
Which of the statements given above is/are correct? (c) 1 and 3 only
(a) 1 and 2 only (d) 2 and 3 only
(b) 2 and 3 only 103. Consider the following statements
(c) 3 only 1. The population of decomposers is greater than
(d) 1, 2 and 3 that of any other organism class in an ecosystem.
99. With reference to Sunderbans, consider the 2. Zooplanktons constitutes the second trophic
following statements: level in a lake ecosystem.
1. These are the only mangrove forest which is Which of the statements given above is/ are correct?
inhabited by tigers in the world. (a) 1 only
2. It is a cuspate type of delta which is formed by (b) 2 only
Ganga and Brahmaputra. (c) Both 1 and 2
(d) Neither 1 nor 2
Which of the statements given above is/are correct?
(a) 1 only 104. With reference to the characteristics of the aquatic
(b) 2 only ecosystem, consider the following statements.
(c) Both 1 and 2 1. Brackish water ecosystem is more saline than
(d) Neither 1 nor 2 freshwater ecosystem and less saline than marine
water ecosystem.
100. Consider the following pairs:
2. Pangong Tso located in India is a freshwater
Mangroves Location ecosystem.
3. Dolphin species are found only in brackish water
1. Coringa mangroves Odisha ecosystem.

203 Workbook
.
ENVIRONMENT

Which of the statements given above is/are correct? 3. Diversity in benthic zone is very less.
(a) 1 only Select the correct answer using the code given below:
(b) 2 and 3 only (a) 1 and 2 only
(c) 1 and 3 only (b) 3 only
(d) 1, 2 and 3 (c) 2 and 3 only
105. With reference to Wetland Ecosystems, which of (d) 1 and 3 only
the following statements is/are correct? 108. With regard to characteristics of Oligotrophic
1. These are the transitional zones between lake, which of the following statements is/are
terrestrial and aquatic ecosystem. correct?
2. Wetlands have much lower ecological 1. Low primary productivity
productivity as compared to lakes. 2. Poor drinking water quality.
3. State Governments alone can give permission to 3. Found in hot and humid areas.
carry out any prohibited activity in the notified
4. High turbidity
wetlands.
4. There are a greater number of coastal wetlands Select the correct answer using the code given below:
in India than Inland wetlands. (a) 1 only
(b) 2 and 3 only
Select the correct answer from the codes given
(c) 1, 3 and 4 only
below:
(d) 1, 2 and 4 only
(a) 1 only
(b) 1 and 3 only 109. Which of the following impacts is/are associated
(c) 2 and 4 only with Ocean Disappearance?
(d) 1, 3 and 4 only 1. Hinderance in planetary cycling of nitrogen and
106. With reference to Dead Zones or Hypoxia, which phosphorous elements.
of the following statements is/are correct? 2. Decline in agricultural and industrial resources.
1. These are low oxygen areas in the oceans and 3. Disrupt the distribution of pressure in the upper
lakes where aquatic life hardly survives. atmosphere.
2. It is created when a water body is not able to Select the correct answer using the code given below.
receive any nutrient over a long period. (a) 1 and 3 only
3. Global warming aids in the formation of hypoxia (b) 2 and 3 only
as oxygen is less soluble in warm water. (c) 1 and 2 only
Select the correct answer from the codes given (d) 1, 2 and 3
below: 110. Consider the following statements:
(a) 1 only 1. The top few meters of the ocean, store as much
(b) 2 and 3 only heat as Earth’s entire atmosphere.
(c) 1 and 3 only 2. The oceans have absorbed 90% of the heat that
(d) 1, 2 and 3 has occurred in recent decades due to increased
107. With regard to different zones of lake, which of the greenhouse gases.
following statements is/are correct? Which of the statements given above is/are correct?
1. Rooted aquatic plants and algae grow in littoral (a) 1 only
zone. (b) 2 only
2. Limnetic zone is the area in a deep lake where (c) Both 1 and 2
no light reaches. (d) Neither 1 nor 2

Workbook 204
.
ENVIRONMENT

SOLUTIONS

the density of ocean water would not be decreased


3.1. Previous Years Questions drastically if the phytoplankton of the ocean is completely
1. Solution (b) destroyed.
Exp) Option b is correct 5. Solution (d)
When the wind blows parallel to a coastline, surface waters Exp) Option d is correct
are pushed offshore and water is drawn from below to All of the above are employed as biopesticides.
replace the water that has been pushed away. The upward
movement of this deep, colder water is called upwelling. Biopesticides include naturally occurring substances that
control pests (biochemical pesticides), microorganisms
The deeper water that rises to the surface during upwelling that control pests (microbial pesticides), and pesticidal
is rich in nutrients. These nutrients “fertilize” surface substances produced by plants containing added genetic
waters, encouraging the growth of plant life, including
material (plant-incorporated protectants) or PIPs.
phytoplankton. These phytoplankton serve as the ultimate
energy base in the ocean for large animal populations Bacteria and Fungi are more targeted in their activity than
higher in the food chain, providing food for fish, marine conventional chemicals. For example, a certain fungus
mammals, seabirds, and other critters. might control certain weeds, and another fungus might
control certain insects. The most common microbial
2. Solution (a) biopesticide is Bacillus thuringiensis.
Exp) Option a is correct
6. Solution (d)
Despite the amount of vegetation in the tropical rain
forest, the soil contains less organic matter than that of Exp) Option d is correct
temperate forests, because the warm humid conditions All of the above are pollinating agents.
encourage faster decay and recycling of nutrients back
into living forest. 7. Solution (c)
Exp) Option c is correct
3. Solution (c)
The Millennium Ecosystem Assessment (MA), a major
Exp) Option c is correct UN-sponsored effort to analyze the impact of human
Photosynthesis takes up carbon dioxide from the actions on ecosystems and human well-being, identified
atmosphere and gives out oxygen as a product. four major categories of ecosystem services: provisioning,
regulating, cultural and supporting services.
4. Solution (a)
Supporting Services include photosynthesis, nutrient
Exp) Option a is correct. cycling, the creation of soils, and the water cycle.
Statement 1 is correct. Phytoplankton are responsible These processes allow the Earth to sustain basic life
for most of the transfer of carbon dioxide from the forms, let alone whole ecosystems and people. Without
atmosphere to the ocean. Carbon dioxide is consumed supporting services, provisional, regulating, and cultural
during photosynthesis, and the carbon is incorporated in
services wouldn’t exist.
the phytoplankton, just as carbon is stored in the wood
and leaves of a tree. Thus, the complete destruction of the 8. Solution (b)
phytoplankton of an ocean would release this sequestered
Exp) Option b is correct
carbon dioxide to the atmosphere and adversely affect the
ocean as a carbon sink. A decomposer is an organism that decomposes, or breaks
Statement 2 is correct. Phytoplankton and algae form down, organic material such as the remains of dead
the basis of aquatic food webs. They are eaten by primary organisms. Viruses invade other organisms, but they’re
consumers like zooplankton, small fish, and crustaceans. not decomposers.
Primary consumers are in turn eaten by fish, small sharks, Decomposers include bacteria and fungi.
corals, and baleen whales. Thus, the complete destruction Decomposition is an important process because it allows
of the phytoplankton of an ocean would adversely affect organic material to be recycled in an ecosystem.
the food chains in the ocean.
Statement 3 is incorrect. Phytoplankton are 9. Solution (c)
photosynthesizing microscopic biotic organisms that Exp) Option c is correct.
inhabit the upper sunlit layer of almost all oceans and In the grasslands, trees do not replace the grasses as a
bodies of fresh water on Earth. The density of the ocean part of an ecological succession because of water limits
water depends on heat content and salinity. Thus, and fires. Grass ecosystems are climax ecosystems in

205 Workbook
.
ENVIRONMENT

grassland regions, where there is not enough rainfall to “Ecosystem” term was first coined by A.G. Tansely, in the
support a forest. The grasslands are found where rainfall year 1935.
is about 25-75 cm per year, not enough to support a An ecosystem is a community of living organisms in
forest, but more than that of a true desert. conjunction with the non-living components of their
Secondary succession is the sequential development environment, interacting as a system. These biotic and
of biotic communities after the complete or partial abiotic components are linked together through nutrient
destruction of the existing community. Succession cycles and energy flows. In simple terms, it is a community
is a progressive series of changes which leads to the of living and non-living things that work together.
establishment of a relatively stable climax community.
16. Solution (c)
10. Solution (c) Exp) Option c is correct
Exp) Option c is correct Biological oxygen demand (BOD) is the amount of
The distribution of water on the Earth’s surface is extremely dissolved oxygen needed (i. e., demanded) by aerobic
uneven. Only 3% of water on the surface is fresh; the micro-organisms to break down organic material present
remaining 97% resides in the ocean. Of freshwater, 69% in a given water sample. BOD is directly proportional to
resides in glaciers, 30% exists as groundwater, and less the level of pollution in the water body.
than 1% is located in lakes, rivers, and swamps.
It is similar to Chemical Oxygen Demand, both measure
11. Solution (a) the amount of organic compounds in water.
Exp) Option a is correct 17. Solution (c)
In ecology, a food chain is a series of organisms that eat Exp) Option c is correct
one another so that energy and nutrients flow from one
Butterflies play a crucial role in pollination.
to the next.
Some species of wasps, spiders and birds are natural
Food chain is the sequence of organisms through which
predators of butterflies. A fall in population of butterflies
the energy flows. If the species starts eating itself, then
could hamper their population.
flow of energy will not proceed.
A food chain shows how ‘energy’ is transferred from one 18. Solution (d)
living organism to another via food, it does not show the Exp) Option d is correct.
numbers. Rather it only shows who eats whom.
Statement 1 is correct. Most Coral reefs are located in
12. Solution (b) the tropical waters i.e. between the Tropics of Cancer
Exp) Option b is correct and Capricorn. It is because of strict environmental
conditions which they require to flourish.
Nostoc fixes atmospheric nitrogen into ammonia, which
may then be used or converted to a form suitable for plant They grow optimally in water temperatures between
growth. 23°–29°Celsius. Most reef-building corals also require
very saline (salty) water ranging from 32 to 42 parts per
Agaricus and Spirogya are not used as biofertilizer. thousand.
13. Solution (c) Coral reefs are found throughout the oceans, from
Exp) Option c is correct deep, cold waters to shallow, tropical waters. Temperate
and tropical reefs however are formed only in a zone
Nitrogen is the main constituent of Urea. extending at most from 30°N to 30°S of the equator; the
When plants die, microorganisms break down, or reef-building corals prefering to grow at depths shallower
decompose, dead plant cells. During the decomposition than 30 m (100 ft), or where the temperature range is
about plant matter, organic nitrogen is once again between 16-32and light levels are high.
converted to inorganic ammonium and released into the Statement 2 is correct. Indonesia, Australia and the
soil. Philippines are the largest reef nations with almost 44%
Burning of coal does not add nitrogen to soil, but it of world’s coral reefs. In Indonesia 17.95% of World’s
releases oxides of nitrogen into the atmosphere. corals is located, in Australia 17.22% of world’s corals
is located and in Philippines 8.81% of world’s corals is
14. Solution (b) located.
Exp) Option b is correct Coral reefs are large underwater structures composed
Ecological niche is a term for the position of a species of the skeletons of coral, which are marine invertebrate
within an ecosystem, describing both the range of animals. The coral species that build reefs are known
conditions necessary for persistence of the species, and its as hermatypic, or “hard,” corals because they extract
ecological role in the ecosystem. calcium carbonate from seawater to create a hard, durable
exoskeleton that protects their soft, sac-like bodies. Other
15. Solution (c)
species of corals that are not involved in reef building are
Exp) Option c is correct known as “soft” corals.

Workbook 206
.
ENVIRONMENT

Statement 3 is correct. Coral reefs include a wide range 21. Solution: (d)
of diversity with 32 of the 34 animal phyla present, Exp) Option d is the correct answer.
in contrast to only 9 phyla represented as free living
The System of Rice Intensification involves cultivating
species in tropical rainforest. This is because of high
rice with as much organic manure as possible. It employs
productivity of coral reefs.
young seedlings planted singly at wider spacing in a
Coral reefs are one of the most productive and complex square pattern; and with intermittent irrigation that keeps
coastal ecosystems with high biological diversity. The the soil moist but not inundated, and frequent inter
high productivity is owing to the combination of its own cultivation with weeder that actively aerates the soil.
primary production and support from its surrounding
System of Rice Intensification (SRI) includes a method
habitat. Because of the diversity of life found in the habitats
called Alternate Wetting and Drying (AWD) which is a
created by corals, reefs are often called the “rainforests of
form of controlled or intermittent irrigation of the rice
the sea”.
crops.
19. Solution (d) SRI methods have the following benefits and impacts,
Exp) Option d is correct. in general, compared to conventional methods of paddy
cultivation:
Statement 1 is correct. Desert plants are adapted to their
arid environment in many different ways. To increase paddy yields usually by 20-50% and sometimes
up to 100%
Some plants have hard and waxy leaves to reduce the
loss of water through transpiration. The waxy covering It reduces required seeds for transplanting by 60-80%(
on plant leaves, young stems, and fruit is called the Hence Option 1 is correct);
“cuticle”. It is composed of cutin, a wax-like material The electricity consumption will be less as it enables
produced by the plant that is chemically a hydroxy fatty reduce use of chemical fertilizers and agrichemicals
acid. The purpose of this covering is to help the plant and reduced use of irrigation water by 25-50% ( Hence
retain water. In arid regions, that is very important. Option 3 is correct);
Statement 2 is correct. Small leaves on desert plants also Option 2 is correct. By stopping flooding and relying
help reduce moisture loss during transpiration. Small mostly on organic fertilization, methane emissions are
leaves mean less evaporative surface per leaf. In addition, greatly reduced without offsetting increases in nitrous
a small leaf in the sun doesn’t reach as high a temperature oxide. Also, less carbon dioxide is generated due to
as a large leaf in the sun. Small leaves means less surface reduced reliance on and transport of manufactured
area for transpiration thus reduction in water loss. inputs. Oxford researchers in India found 30% reduction
Statement 3 is correct. In arid regions, some plants have in net greenhouse gas emissions with SRI, compared
thorns, instead of leaves to avoid loss of water through with current practice, and more than 50% reduction in
leaf transpiration. Desert plants have few or no leaves, greenhouse gas emissions per kilogram of rice produced.
and the foliage is either waxy, leathery, hairy or needle 22. Solution: (a)
shaped to reduce water loss through transpiration. Some
of them are entirely leafless, with pricks or thrones. Exp) Option a is correct.
Some desert plants grow leaves during the rainy season Polyethylene terephthalate (PET or PETE) is a strong, stiff
and then shed them when it becomes dry again. Some synthetic fibre and resin and a member of the polyester
cut down on water loss even further by temporarily family of polymers. PET is spun into fibres for permanent-
shutting down photosynthesis. press fabrics and blow-molded into disposable beverage
bottles.
20. Solution (a) Statement 1 is correct: PET fibres make them highly
Exp) Option a is correct. resistant to deformation, so they impart excellent
Carbon fertilization is the increased growth of plants resistance to wrinkling in fabrics. They are often used in
due to increased rate of photosynthesis in plants that durable-press blends with other fibres such as rayon,
results from increased levels of carbon dioxide in the wool, and cotton, reinforcing the inherent properties of
atmosphere. those fibres while contributing to the ability of the fabric
to recover from wrinkling.
The effect varies depending on the plant species, the
temperature, and the availability of water and nutrients. Statement 2 is incorrect: Maharashtra government has
announced the ban on sale of alcohol in PET bottles
It is also the artificial enrichment of the atmosphere from April 1. Plastic packaging is dangerous to human
of greenhouses with carbon dioxide used to improve health and the country liquor and country made foreign
production levels. liquor cannot be sold in such bottles. PET has become
Elevated CO2 concentration increases the plant growth widely used in carbonated-beverage bottles and in jars
and leaf photosynthesis, even under water stress for food processed at low temperatures. The low softening
conditions. Also, elevated CO2 concentration enhances temperature of PET—approximately 70 °C (160 °F)—
water use efficiency under water stresses. prevents it from being used as a container for hot foods.

207 Workbook
.
ENVIRONMENT

Statement 3 is correct: Polyethylene terephthalate is the


most widely recycled plastic. For instance, PET bottles 3.2. Ecology and Ecosystem
and containers are commonly melted down and spun into 25. Solution: (c)
fibres for fibrefill or carpets.
Exp) Option c is correct
Statement 4 is incorrect: Polyethylene, like other
Ecosystems in the order of decreasing productivity:
plastics, is not inert and is known to release additives
and other degradation products into the environment Mangroves > Grasslands > Lakes > Oceans.
throughout its lifetime. For example, the additive Ecological succession is the steady and gradual change
bisphenol-A used in the manufacture of many plastic in a species of a given area with respect to the changing
products is leached as plastics age, and hydrocarbon gases environment.
are produced during high-temperature decomposition Forest fires change the composition of ground which
(>202°C). hinders growth of tress.
23. Solution: (a) Grasslands occur where rainfall is usually low and/or the
soil depth and quality is poor. Low rainfall prevents the
Exp) Option a is correct. growth of a large number of trees and shrubs in abundance
Nitrogen fixation is a process that implies the as most of the water is first intercepted by grasses before it
transformation of the relatively non-reactive atmospheric reaches the shrubs.
N2 into its more reactive compounds (nitrates, nitrites, or
ammonia). 26. Solution: (c)
Nitrogen-Fixing Plants: An N-fixing crop is a natural way Exp) Option c is correct.
to provide plant-adjusted N without any industrial harm The distribution of water on the Earth’s surface is extremely
to nature. Nitrogen fixing plants are: uneven. Only 3% of water on the surface is fresh; the
Clovers, vetches, and peas are nitrogen-fixing plants remaining 97% resides in the ocean. Of freshwater, 69%
used by farmers worldwide and in the US Southern resides in glaciers, 30% exists as groundwater, and less
Great Plains in particular. than 1% is located in lakes, rivers, and swamps.
Peas or beans can be used as a summer nitrogen-fixing 27. Solution: (c)
cover crop or harvested for food. Both ways, they enrich Exp) Option c is correct.
the soil with plant-suitable N. Southern peas prefer warm
Photosynthesis takes up carbon dioxide from the
seasons and are sown when the soil temperature reaches
atmosphere and gives out oxygen as a product.
60F.
Respiration is the Biochemical process in which the cells
Beans: fava (aka faba, broad), alfalfa, green (aka French), of an organism obtain energy by combining Oxygen and
runner, field, sweet, peanuts (aka groundnuts), soybeans, glucose, resulting in the release of Carbon dioxide.
cream, black-eyed, or purple-hulled beans, lupins, lentils,
cowpeas, chickpeas. When Organisms die, they are decomposed by bacteria.
Carbon dioxide is released in the atmosphere during the
24. Solution: (a) decomposition process.
Exp) Option a is correct. Water vapour, carbon dioxide and sulphur dioxide are the
The Zoological Survey of India (ZSI), with help from most common volcanic gases.
Gujarat’s forest department, is attempting for the first 28. Solution (d)
time a process to restore coral reefs using bio rock or
Exp) Option d is correct.
mineral accretion technology. A bio rock structure was
installed one nautical mile off the Mithapur coast in the Pair 1 is correctly matched. Ecotone is a transitional
Gulf of Kachchh on January 19. area of vegetation between two different plant
communities. It has some of the characteristics of each
Bio rock is the name given to the substance formed by bordering biological community (called edge effect)
electro accumulation of minerals dissolved in seawater on and often contains species not found in the overlapping
steel structures that are lowered onto the sea bed and are communities. Ecotone is related with variation in species
connected to a power source, in this case solar panels that prevalence.
float on the surface.
Estuary - ecotone between fresh water and salt water.
The technology works by passing a small amount of
electrical current through electrodes in the water. When a Mangrove forests - ecotone between marine and terrestrial
positively charged anode and negatively charged cathode ecosystem.
are placed on the sea floor, with an electric current flowing Grassland - ecotone between forest and desert
between them, calcium ions combine with carbonate ions Pair 2 is correctly matched. An ecotype is a population
and adhere to the structure (cathode). This results in (restricted to one habitat) that is adapted to local
calcium carbonate formation. Coral larvae adhere to the environmental conditions. This species is genetically
CaCO3 and grow quickly. modified to survive in the particular habitat.

Workbook 208
.
ENVIRONMENT

For example, a population or ecotype of species found Statement 1 is correct: The final stage of a succession
at the foot of a mountain may differ in size, color, or is called climax community. All the stages which lead
physiology from a different ecotype living at higher up to climax community are called successional stages or
altitudes, thus reflecting a sharp change in local selective seres.
pressures. Statement 2 is correct: Succession occurs faster in areas
Pair 3 is correctly matched. Ecocline (ecological existing in the middle of the large continents. As here
gradient) is a gradation from one ecosystem to seeds of plants belonging to different stages would reach
another when there is no sharp boundary between much faster, establish and ultimately result in climax
the two. Ecocline is a variation of the physicochemical community.
environment between two biomes. Ecocline is related Statement 3 is incorrect: In a primary succession
to the variation of physicochemical factors like salinity, pioneer species inhabit over a hard substrate and over a
humidity, or temperature. few generations alter the habitat conditions. Whereas in
29. Solution (c) a secondary succession a well-developed soil is already
formed at the site. This makes it much faster as compared
Exp) Option c is correct. to primary succession which generally takes hundreds of
Net Primary Productivity (NPP) is the rate at which an years.
ecosystem accumulates energy or biomass, excluding the
energy it uses for the process of respiration. 32. Solution (a)
The correct sequence is Estuary > Temperate grassland > Exp) Option a is correct
Tundra > Oceans as clear from the chart. Statement 1 is correct: Pyramid of Biomass-It is the
pyramid in which individuals at each trophic level are
weighted. It is both upright (for most ecosystems on
land) and downward (many aquatic ecosystems).
Statement 2 is correct: Pyramid of Number-It is the
representation of the total number of individuals of
different species belonging to each trophic level. It is
mostly upright for instance grassland ecosystem. But in
some cases, it might be inverted e.g., detritus food chain.
Statement 3 is incorrect: Pyramid of Energy is always
upright, for it is governed by the laws of thermodynamics.
Only 10percent of energy is passed from one trophic level
to another
33. Solution (b)
Exp) Option b is correct.
Commensalism- One species benefits and the other is
30. Solution (b) unaffected. The egrets always forage close to where the
Exp) Option b is correct cattle are grazing. Cattle egrets eat the flies and bugs that
tender to bother the cattle. The movement of livestock
Statement 1 is incorrect: Lichens are a type of epiphytes,
also dislodges various insects on which cattle egrets feed
which grow on the surface of tree bark but takes its
on. So, this is called commensalism in which the egrets
nutrient and moisture from the air. They can perform
benefits and this has no effect on Cattle.
photosynthesis on their own. Thus, the relation between
lichen and host plant is of commensalism. Predation- One species benefits and the other is harmed.
Biological control methods adopted in agricultural pest
Statement 2 is correct: In competition two species
control are based on the ability of the predator to regulate
fight for a common resource. Thus, feeding efficiency of
prey population.
one species is reduced due to interfering and inhibitory
presence of another species. It simply means that the Competition- competition is defined as a process in
fitness of one species is lowered significantly due to the which the fitness of one species is significantly lower in
presence of other species. the presence of another species. The Abingdon tortoise
in Galapagos Islands became extinct within a decade
Statement 3 is incorrect: Phytophagous is the term after goats were introduced on the island, apparently due
used to define those insects which feed on plant sap to the greater browsing efficiency of the goats.
and other plant part. This relation is of predation in
nature. Epiphytes have air breathing roots and forms 34. Solution (b)
commensalism relation with plant. Exp) Option b is correct.
31. Solution (b) Phytoplankton - Copepod - Bluefish - Swordfish
Exp) Option b is correct Phytoplankton contains chlorophyll and produces their

209 Workbook
.
ENVIRONMENT

own food. Copepod is a zooplankton that feed directly Statement 1 is correct. In terrestrial ecosystem, producers
on phytoplankton. Bluefish is a pelagic fish that feeds on are basically herbaceous and woody plants, whereas
copepods. Swordfish is a predatory fish and is at the apex in aquatic ecosystem producers are various species of
of the food pyramid. microscopic algae. Some bacteria that live in the deep
Option a is incorrect. Plankton - Snail – Mackerel – ocean, near hydrothermal vents, also produce food
Shark is the correct sequence. through chemosynthesis.
Plankton is the foundation of the aquatic food web. Shark Statement 2 is correct. Saprotrophs are micro-
is predatory fish and is at the apex of the food pyramid. consumers, like bacteria and fungi, which obtain energy
Option c is incorrect. Phytoplankton - Zooplankton - and nutrients by decomposing dead organic substances
Anchovy – Tuna is the correct sequence. (detritus) of plant and animal origin.
Phytoplanktons contain chlorophyll and produce Statement 3 is correct. Omnivores are macroconsumers.
their own food. Zooplankton may be herbivores (eat These are organisms which consume both plants and
phytoplankton), carnivores (eat other zooplankton) animals. e.g. human beings, monkey.
or omnivores. Anchovy is a small fish that feeds on
38. Solution (c)
zooplankton. Tuna are predatory fish.
Exp) Option c is correct.
Option d is incorrect. Algae- Zooplankton - Tuna –
Shark is the correct sequence. Statement 1 is correct. Ecotone is a zone of junction
between two or more diverse ecosystems. It may be
Tuna is also a predatory fish but Shark is at the apex of narrow or quite wide. It has the conditions intermediate
food pyramid. to the adjacent eco-systems. Hence it is a zone of
35. Solution (a) tension. It is linear as it shows progressive increase in
species composition of one in coming community and a
Exp) Option a is correct. simultaneous decrease in species of the other outgoing ad-
Statement 1 is correct. Biomagnification (or joining community.
bioaccumulation) refers to the ability of living organisms Statement 2 is correct. The mangrove forests represent
to accumulate certain chemicals to a concentration an ecotone between marine and terrestrial ecosystem.
larger than that occurring in their inorganic, non-living It has the conditions intermediate to the adjacent
environment. ecosystems. Other examples of ecotones are – grassland,
If a pollutant is long-lived, then it is possible that estuary and river bank
concentration may take place. Otherwise if a pollutant is
short-lived; it will be broken down before it can become 39. Solution (c)
dangerous. Exp) Option c is correct.
Statement 2 is incorrect. The pollutant must be soluble Statement 1 is correct: Niche is the unique functional role
in fats (otherwise it will be soluble in water and it will be of a species in an ecosystem. The functional characteristics
excreted by the organism). Pollutants that dissolve in fats of a species in its habitat is referred to as “niche” in that
are retained for a long time. common habitat. Niche describes biological, chemical
Statement 3 is incorrect. The pollutant must be and physical factors needed by a species for survival,
biologically active because it probably won’t cause any health and reproduction.
problems. Since biomagnifications involves accumulation Statement 2 is correct: Niche is unique for a species. No
of toxins, so it must be biologically active. two species can have exact identical niches. Therefore, the
knowledge of niche requirement of a species is essential to
36. Solution (c) conserve that species.
Exp) Option c is correct.
40. Solution (a)
Only 2 and 4 are Abiotic factors.
Exp) Option a is correct.
The relationship and interaction between different
Pair 1 is correctly matched. Food niche includes what
organisms and our environment are very complex. It
the species eats or decomposes and what other species it
includes both living, called biotic, and non-living, called
competes with for food.
abiotic, components.
Pair 2 is incorrectly matched. Physical and chemical
Abiotic Components – Energy, Radiation, Temperature
niche includes temperature, land shape, land slope,
& heat flow, Water, Atmospheric gases and wind, Fire, humidity and other requirements of the species.
Gravity, Topography, Soil, Geologic substratum.
Pair 3 is incorrectly matched. Habitat niche includes
Biotic Components - Green plants, Non-green plants, where the species lives. Whereas, reproductive niche
Decomposers, Parasites, Symbionts, Animals, Man. includes how and when the species reproduces.
37. Solution (d) 41. Solution: (c)
Exp) Option d is correct. Exp) Option c is the correct answer.

Workbook 210
.
ENVIRONMENT

Statement 1 is correct: In each community, a few over 42. Solution: (b)


topping species (dominant species) are present in greater Exp) Option b is the correct answer.
bulk. By their greater number or biomass (living weight)
In ecological terms, the carrying capacity of an ecosystem
the dominant species modify the habitat characteristic
is the size of the population that can be supported
and influence the growth of other species in the indefinitely upon the available resources and services
community. In most communities only a single species, of that ecosystem. In other words, it is defined as the
being particularly conspicuous, is dominant. In such maximum population size that an ecosystem can
case, the community is named after the dominant species. sustainably support without degrading itself.
Example: spruce forest community. In some communities,
The limits of an ecosystem depends on three factors:
however, there may be more than one dominant species,
as in oak-fir forest in the west Himalayas. • the amount of resources available in the ecosystem;
Statement 2 is incorrect: Species diversity and dominance • the size of the population or community; and
are interrelated. Communities with one or a few dominant • the amount of resources each individual within the
species are characterised by low species diversity whereas community is consuming.
communities, where no single species is truly dominant
and individuals are equally distributed among all species, 43. Solution: (a)
are characterised by high species diversity. Exp) Option a is the correct answer.
Statement 3 is correct: Different organisms in a community Statement 1 is correct: Eco-sensitivity is expected to
develop a characteristic pattern of stratification to have low levels of resilience, and hence is difficult to be
minimise competition and conflict among the members recovered or restored if disturbed by external influences.
of the community. Stratification is a practical strategy The ultimate purpose of identifying ecologically sensitivity
to minimise interspecific competition i.e. competition or eco-sensitivity is to promote environment-friendly
between different species. For example, consider a tall management regimes and conservation of the ecological
growing tree which outgrows a potentially short one wealth of the sites.
under the same conditions. The former occupies a higher Statement 2 is incorrect: Acclimation involves
level. The tall trees with widespread canopy dominate the physiological, anatomical, or morphological adjustments
area and influence light and moisture conditions. As a within a single organism that improve performance or
result the tall trees control the characteristic in a forest survival in response to environmental change. The
community. Only those species of organisms can survive extent of this acclimation is constrained by the genome
which can withstand the environmental conditions of the individual. It is the process by which an organism
created by the dominant species. adjusts to a change in its environment. It generally
refers to the ability of living things to adjust to changes in
Tropical forests represent good example of vertical
climate, and usually occurs in a short time of the change.
stratification as shown in the figure below:

211 Workbook
.
ENVIRONMENT

Statement 3 is incorrect: Acclimatization is the


compensatory adjustment of organism to change in the
environment, in nature or natural condition is known
as acclimatization. Adjustment to seasonal changes in
temperature comes under acclimatization. Scientists
distinguish between acclimation and acclimatization
because the latter adjustment is made under natural
conditions when the organism is subject to the full range
of changing environmental factors. Acclimation, however,
refers to a change in only one environmental factor under
laboratory conditions.
44. Solution: (c)
Exp) Option c is correct.
Environment is derived from a French word - Environer/ 46. Solution: (c)
Environner meaning “neighbourhood”. Exp) Option c is correct
Pair 1 is correct. Environment refers to the area where Ecological Succession is a process in which a series of
organisms live. Environment is the natural component communities replaces one another due to large scale
in which biotic (living) and abiotic (non-living) factors destruction done either due to natural or manmade factor.
interact among themselves and with each other. These This process continues until establishment of a relatively
interactions shape the habitat and ecosystem of an stable climax community. The succession of vegetation
organism. community in any habitat of an ecosystem, on the basis of
Pair 2 is incorrect. The biosphere (not biome) is a narrow changes in the environmental conditions of that habitat,
zone on the surface of the earth where soil, water, and air can be classified into two types of succession which are
combine to sustain life. The biosphere includes all the mentioned as below:
places on earth where life is found and includes all of the
Statement 1 is correct. Autogenic succession is one
different biomes.
which takes place when the changes in environmental
A biome is a collection of different ecosystems that share conditions of a given habitat are affected by the biota
similar climate conditions. itself, for example introduction of changes by the
Pair 3 is correct. Ecosystem is a system formed by the vegetation inhabiting the habitat.
interaction of all living organisms (biotic component) Statement 2 is correct. Allogenic succession is one which
with each other and with the physical and chemical takes place in a particular habitat wherein the changes in
factors (abiotic component) of the environment in which the environmental conditions of that habitat are affected
they live, all linked by transfer of energy and material.
due to variations in the abiotic or physical components
Abiotic components are inorganic materials- air, water of the environment or the habitat. For example, global
and soil, whereas biotic components are producers, climatic changes, rapid rate of siltation of lakes, ponds
consumers and decomposers.. etc., accelerated rate of erosion of the region concerned,
45. Solution: (a) upliftment or subsidence of ground surface etc.
Exp) Option a is correct. 47. Solution: (a)
Net primary productivity is defined as the difference of Exp) Option a is correct
the energy that’s fixed by the autotrophs and their own Amensalism refers to a form of biological interactions
respiration losses. Gross primary productivity minus
in which one species is harmed, whereas the other is
respiration losses is the net primary productivity (NPP).
unaffected.
Net primary productivity is the available biomass for
the consumption to heterotrophs (herbiviores and Statement a is correct. A large tree shades a small plant,
decomposers). retarding the growth of the small plant, but on opposite
Option b is incorrect. Primary production is defined side the small plant has no effect on the large tree. This is
as the amount of biomass or organic matter produced an example of amensalism.
per unit area over a time period by plants during Statement b is incorrect. Competition refers to a form of
photosynthesis. The rate of biomass production is called biological interaction in which both species are harmed
productivity. by the interaction. For example, if two species eat the
Option c is incorrect. Gross primary productivity of same food, and there isn’t enough for both, both may have
an ecosystem is the rate of production of organic matter access to less food than they would if alone. They both
during photosynthesis. suffer a shortage of food.
Option d is incorrect. Secondary productivity is Statement c is incorrect. Predation and parasitism
defined as the rate of formation of new organic matter by is another form of biological interactions. In this, one
consumers. species is benefitted, whereas the other is harmed. For

Workbook 212
.
ENVIRONMENT

example, tick gains benefit by sucking blood whereas the Statement 2 is incorrect: Species Diversity is much less
host is harmed by losing blood. (not more) in Primary Succession than in Secondary
Statement d is incorrect. Commensalism refers to a Succession. This is because as Primary Succession occurs
biological interaction in which one species benefit, the in bare landscapes which do not have soils, or diverse
other is unaffected. Example: cow dung provides food biota or mobilised nutrients, so such ecosystems are
and shelter to dung beetles. The beetles have no effect on unable to support a variety of organisms. However, with
the cows. time, as soils develop, nutrients enter circulation, new
species emerge which interact among themselves and
48. Solution: (a) with the abiotic components to create a greater diversity
Exp) Option a is correct. of species. More food and more energy become available
for use thus increasing species diversity in Secondary
Succession is when one community of plants and animals
Succession compared to Primary Succession.
replaces another in an ecosystem. During succession
some species colonise an area and their population Statement 3 is correct: In secondary succession, a
become more numerous whereas populations of other previously occupied area is re-colonized following a
species decline and even disappear. The entire sequence disturbance that kills much of its community.
of communities that successively change in a given area A classic example of secondary succession occurs in
are called sere(s). oak forests cleared by wildfire. Wildfires will burn most
Statement 1 is correct. The pioneer species are those vegetation and kill animals unable to flee the area. Their
species which initiate the development of an ecological nutrients, however, are returned to the ground in the form
community in an area with currently no life form’s of ash.
existence. In primary succession on rocks these are Primary succession occurs when new land is formed
usually lichens which are able to secrete acids to dissolve or bare rock is exposed, providing a habitat that can be
rock, helping in weathering and soil formation. colonized for the first time.
Statement 2 is correct. Pioneer species can tolerate and Statement 4 is correct: The first plant to colonize an area
trade through any type of prevailing harsh environmental is called the pioneer community.
conditions.
The final stage of succession is called the climax
Statement 3 is incorrect. In a climax community (not in community.
pioneer species), the plants and animals are in balance
In primary succession on a terrestrial site, the new site is
with each other and their environment. A climax
first colonised by a few hardy pioneer species that are often
community refers to a stable ecosystem in its final stage
microbes, lichens and mosses. Forests form the climax
of ecological succession. The climax community remains
community in most of the ecosystems. For example, oak
stable as long as the environment remains unchanged.
forest would be a climax community.
49. Solution: (c)
50. Solution: (d)
Exp) Option c is correct.
Exp) Option d is correct.
The composition and structure of any community in any
ecosystem keeps changing in response to each other as well An ecological niche refers to the interrelationship of a
as the changes in the environment. This gradual change species with all the biotic and abiotic factors affecting it.
over time in the species composition of a community is An ecosystem is the structural and functional unit of the
known as Ecological Succession. There are primarily two environment around us. Earth has a variety of ecosystems,
stages of ecological succession - Primary Succession and which can be grouped into certain categories on the
Secondary Succession, both with distinct features and basis of the abiotic factors (light & water availability.
characteristics, a comparison of which has been discussed Temperature, etc) and biotic components (types of plants
below. and animals inhabiting them).
Statement 1 is correct: Secondary Succession occurs Statement a is correct: The term Niche, in Ecosystem
faster than Primary Succession. This is because Primary studies is defined as the sum total of all the interactions
Succession occurs in a place where there is no existing of a species in an ecosystem. It refers to the function/
community of plant or animals and the nutrients have role played by a species in an ecosystem. It describes both
not begun to be mobilised and cycled, for example rocks the range of conditions necessary for persistence of the
left behind retreating glaciers, scorched ground left after species, and its ecological role in the ecosystem.
lava flow, etc. Starting this process is hard and takes a lot Statement b is correct: Each species has a unique
of time to show effect. Niche within that ecosystem. No two species can have
Secondary Succession takes place on the site where the the exact same niche, otherwise they would be in direct
climax community of Primary succession has been competition for resources with one another. If this
partially or completely destroyed. Thus, it has a lot of occurs, then one species will outcompete the other. If
soil, mobilised nutrients and fertile dead and decaying the losing species then does not adapt, it would lead to
organic matter to start, so it takes place much faster. its extinction.

213 Workbook
.
ENVIRONMENT

Statement c is correct: During Primary Successions, community (due to their large size, population,
the communities of organisms are not well established. productivity, or other factors) or make up a greater
The system is in a constant state of flux/ change. There proportion of the biomass. The dominant species is a
is not a great diversity of organisms and other helping species identifiable from the perspective of ecological
factors like nutrients, so niches are not well developed. dominance. A dominant species is a species that prevails
With time, as communities become more diversified and in an ecosystem, especially when they are the most
complex, there are more interactions leading to more well numerous and have a high ecological impact. This means
established niches. they constitute the majority of the biomass.
Statement d is incorrect: A Niche is the sum total of Statement 2 is correct: In most communities only a single
all the interactions of a species within the ecosystem it species, being particularly conspicuous, is dominant and
inhabits. This includes biotic interactions like predation, in such cases, the community is named after the dominant
competitions, etc, as well as its interactions with the species, as for example Spruce Forest community. In
abiotic components in its environment. Abiotic factors some communities, however, there may be more than
like light duration, temperature, etc also have great one dominant species, as in oak-fir forest in the West
bearing on what ultimately will be the role, i.e, niche of Himalayas.
a species. Statement 3 is correct: Different organisms in a
community develop a characteristic pattern of
51. Solution: (d)
stratification to minimize competition and conflict
Exp) Option d is correct. among the members of the community. Plants and
Decomposers break down complex organic matter into animals of each layer differ in size, behaviour and
inorganic substances like carbon dioxide, water and adaptation from those of other layers. Tropical forests
nutrients and the process is called decomposition. The represent good example of vertical stratification.
important steps in the process of decomposition are
fragmentation, leaching, catabolism, humification and 53. Solution: (a)
mineralisation. Exp) Option a is the correct answer.
Option a is incorrect. Anabolism is the mechanism that Ecotones are areas of steep transition between
takes smaller units like nutrients, cells, or amino acids and ecological communities, ecosystems, and/or ecological
bonds them together to create bigger structures. regions along an environmental or other gradients.
Examples of ecotones include marshlands (between
Option b is incorrect. Humification is the process of
dry and wet ecosystems), mangrove forests (between
conversion of dead matter into humus. Under this process
terrestrial and marine ecosystems), grasslands (between
accumulation of a dark coloured amorphous substance
desert and forest), and estuaries (between saltwater and
called humus that is highly resistant to microbial action
freshwater). These ecosystems often are separated, from
and undergoes decomposition at an extremely slow
adjacent ecosystems by a transition zone or a diffused
rate. Being colloidal in nature it serves as a reservoir of
boundary zone called ecotone. As there is a lack of well-
nutrients.
defined boundaries, organisms of adjacent ecosystems
Option c is incorrect. Leaching is the process of intermingle in the ecotone zone; consequently, they may
downward movement of water-soluble inorganic have a greater diversity of species than the neighbouring
nutrients into the soil that get precipitated as unavailable ecosystems.
salts.
Option d is correct. Bacterial and fungal enzymes 54. Solution: (d)
degrade detritus into simpler inorganic substances Exp) Option d is the correct answer.
through secrete digestive enzymes over the detritus. This Ecological Footprint accounting measures the demand
process is called as catabolism. It is the set of metabolic on and supply of nature. The ecological footprint (EF)
pathways that breaks down molecules into smaller units estimates the biologically productive land and sea
that are either oxidized to release energy or used in area needed to provide the renewable resources that
other anabolic reactions. Catabolism breaks down large a population consumes and to absorb the wastes it
molecules into smaller units. generates—using prevailing technology and resource-
management practices—rather than trying to determine
52. Solution: (d) how many people a given land area or the entire planet
Exp) Option d is the correct answer. can support.
An aggregation of populations of different species The amount of carbon-dioxide that any activity by any
(plant and/or animals) in an area, living together with human or machine produces is called carbon foot-print
mutual tolerance and beneficial interactions amongst not ecological footprint.
themselves and with their environment, form a biotic
community. 55. Solution: (c)
Statement 1 is correct: Ecological dominance is the Exp) Option c is the correct answer.
degree to which one or more species have a significant The mineral nutrients move from the non-living to the
influence over the other species in their ecological living and then back to the non-living components of

Workbook 214
.
ENVIRONMENT

the ecosystem in a more or less circular manner. This is Atmospheric fixation: Lightening, combustion and
known as biogeochemical cycling. volcanic activity help in the fixation of nitrogen. The
Statement 1 is correct: When we describe biogeochemical periodic thunderstorms convert the gaseous nitrogen in
cycle; we often say that a cycle is perfect or imperfect. A the atmosphere to ammonia and nitrates.
perfect nutrient cycle is one in which the nutrients Industrial fixation: At high temperature and high
are replaced as fast as they are used up. There are two pressure, molecular nitrogen is broken into atomic
basic types of biogeochemical cycles, gaseous and nitrogen which then combines with hydrogen to form
sedimentary. In the gaseous type of biogeochemical cycle ammonia.
there is a prominent gaseous phase. Cycling of carbon and Bacterial fixation: There are two types of bacteria which
nitrogen represents gaseous biogeochemical cycles. In can function nitrogen fixation:
sedimentary cycles the main reservoir is the lithosphere
from which the nutrients are released largely by Symbiotic bacteria e.g. Rhizobium in the root nodules of
weathering of rocks. leguminous plants.
Statement 2 is incorrect: Most gaseous cycles are Free-living or symbiotic e.g. 1. Nostoc 2. Azotobacter 3.
generally considered perfect. Sedimentary cycles are Cyanobacteria 4. Beijemickia
considered relatively imperfect, as some nutrients are Statement 3 is incorrect. Nitrogen fixation is the process
lost from the cycle into the soil and sediments and that involves the conversion of gaseous nitrogen into
become unavailable for immediate cycling. Ammonia. In denitrification, special denitrifying
Statement 3 is correct: The nutrients are released from bacteria like Pseudomonas and Thiobacillus convert
lithosphere (main reservoir) via weathering of rocks. nitrites/nitrates to elemental nitrogen.
The main pool of elements is rocks, soil and minerals.
Sedimentary cycles get disrupted by local disturbances as 58. Solution: (b)
the material remains immobilized in the earth’s crust. Exp) Option b is correct.
Commensalism- One species benefits and the other is
56. Solution: (d)
unaffected. The egrets always forage close to where the
Exp) Option d is correct cattle are grazing. Cattle egrets eat the flies and bugs that
Ecological Pyramids are the pyramidical representation tender to bother the cattle. The movement of livestock
of trophic levels of different organisms based on their also dislodges various insects on which cattle egrets feed
ecological position (producer to final consumer). on. So, this is called commensalism in which the egrets
Statement 1 is correct. In ecological pyramid, Saprophytes benefits and this has no effect on Cattle.
are not given any place in ecological pyramids despite Predation- One species benefits and the other is harmed.
playing a very important role in ecosystem. These are Biological control methods adopted in agricultural pest
organisms that feeds on non-living organic matter known control are based on the ability of the predator to regulate
as detritus at a microscopic level. prey population.
Statement 2 is correct. The ecological pyramid never Competition- competition is defined as a process in
takes into account the same species belonging to two or which the fitness of one species is significantly lower in
more trophic levels. For example, omnivores are those the presence of another species. The Abingdon tortoise
animals who have wide food preferences, as they can eat in Galapagos Islands became extinct within a decade
both plant and animal matter. Thus, they can be placed after goats were introduced on the island, apparently due
into two or more trophic levels. to the greater browsing efficiency of the goats.
Statement 3 is correct. Ecological pyramids do not
accommodate a food web. Ecological Pyramids assume 3.3. Functions of ecosystem
a simple food chain, something that almost never exists
in nature, whereas in reality, a food web is mostly found. 59. Solution (a)
Exp) Option a is correct.
57. Solution: (a)
Statement 1 is correct. In nature, two main types of food
Exp) Option a is correct. chains have been distinguished: a. Grazing Food Chain
Nitrogen Cycle is a biogeochemical process through and b. Detritus Food chain.
which nitrogen is converted into many forms. Thus, 1. Grazing food chain - The consumers which start the
consecutively passing from the atmosphere to the soil food chain, utilising the plant or plant part as their
to organism and back into the atmosphere. It involves food, constitute the grazing food chain. This food
several processes such as nitrogen fixation, nitrification, chain begins from green plants at the base and the
denitrification, decay and putrefaction. primary consumer is herbivore.
Statements 1 and 2 are correct. Nitrogen fixation is the 2. Detritus Food Chain - It starts from dead organic
process that involves the conversion of gaseous nitrogen matter of decaying animals and plant bodies
into Ammonia, a form in which it can be used by plants. consumed by the micro-organisms and then to
Atmospheric nitrogen can be fixed by the following three detritus feeding organism called detrivores or
methods: decom-poser and to other predators.

215 Workbook
.
ENVIRONMENT

The distinction between these two food chains is the about ten percent of the transferred energy is stored as
source of energy for the first level consumers. In the flesh. The remaining is lost during transfer, broken down
grazing food chain the primary source of energy is living in respiration, or lost to incomplete digestion by higher
plant biomass while in the detritus food chain the source trophic level.
of energy is dead organic matter or detritus. Statement 2 is correct. An energy pyramid, reflects
Statement 2 is incorrect. The grazing and detritus food the laws of thermodynamics, with conversion of
chains are linked. The initial energy source for detritus solar energy to chemical energy and heat energy at
food chain is the waste materials and dead organic each trophic level. There is also loss of energy which is
matter from the grazing food chain. depicted at each transfer to another trophic level. Hence
the pyramid is always upright, with a large energy base
60. Solution (b) at the bottom.
Exp) Option b is correct.
64. Solution (b)
Statement 1 is incorrect. In Aquatic ecosystem,
phytoplanktons are the primary producers. Zoo Exp) Option b is correct.
planktons feed on the phytoplanktons, and hence they Statement 1 is correct. Bioaccumulation refers to how
are the primary consumers. Zoo planktons are eaten by pollutants enter a food chain. In bioaccumulation there
fishes and so on. is an increase in concentration of a pollutant from the
Statement 2 is correct. The consumers which start the environment to the first organism in a food chain.
food chain with plants as their food, constitute the grazing Statement 2 is correct. Biomagnification refers to the
food chain. This food chain begins from green plants at tendency of pollutants to concentrate as they move from
the base, which has the Sun as its primary source of one trophic level to the next. Hence, there is an increase
energy. The primary consumer is herbivores in this chain. in concentration of a pollutant from one link in a food
chain to another.
61. Solution (c) Statement 3 is incorrect. In order for biomagnification
Exp) Option c is correct. to occur, the pollutant must be: long-lived, mobile,
Statement 1 is correct: Bacteria and fungi obtain energy soluble in fats, biologically active. But even if a
and nutrients by decomposing dead organic substances pollutant is not active biologically, it may biomagnify,
(detritus) of plant and animal origin. but we really don’t worry about it much, since it probably
won’t cause any problems. Ex, DDT.
Statement 2 is correct: The products of decomposition
such as inorganic nutrients which are released in the 65. Solution (b)
ecosystem are reused by producers and thus recycled.
Exp) Option b is correct.
62. Solution (a) Statement 1 is incorrect. On land phosphorus is usually
Exp) Option a is correct. found in the form of phosphates. By the process of
weathering and erosion phosphates enter rivers and
Statement 1 is incorrect. The Pyramid of Numbers
streams that transport them to oceans. Unlike carbon
deals with the total number of individuals of different cycle, there is no respiratory release of phosphorus into
species, belonging to each trophic level in an ecosystem. atmosphere.
Depending upon the number of species, the pyramid of
numbers may be upright or inverted. E.g., in grassland Statement 2 is correct. Phosphorus occurs most
ecosystem pyramid, the number of individuals is abundantly in nature in the form of phosphate ion. The
decreased from lower level to higher trophic level natural reservoir of phosphorus is rocks, which contains
(upright pyramid). A pyramid of numbers does not take phosphorus in the form of phosphates.
into account the fact that the size of organisms being Statement 3 is incorrect.
counted in each trophic level can vary.
Statement 2 is correct. Pyramid of biomass is usually
determined by collecting all organisms occupying each
trophic level separately and measuring their dry weight.
This overcomes the size difference problem because all
kinds of organisms at a trophic level are weighed. In many
aquatic ecosystems, the pyramid of biomass can assume
an inverted form.
63. Solution (c)
Exp) Option c is correct. From the above figure, we can say that the phosphorus
Statement 1 is correct. According to Raymond cycle is made up of a series of smaller processes. They
Lindeman, during the transfer of organic food energy are not unidirectional i.e moving in one direction from
from one trophic level to the next higher level, only soil to plants and to animals.

Workbook 216
.
ENVIRONMENT

66. Solution (b) Hydrogen Sulphide and Sulphur dioxide. Organisms


Exp) Option b is correct like marshes, tidal flats release Sulphur in the form of
Hydrogen Sulphide (a gaseous component) by anaerobic
Statement 1 is incorrect. Maximum concentration of
respiration. Sources like volcanic eruption, fossil fuels
global carbon is found in the Oceans (71%), while only
release large amount of Sulphur dioxide (a gaseous
1% is found in the atmosphere.
component) in the atmosphere.
Statement 2 is correct. Carbon component of the
Statement 2 is correct. Whatever the source, Sulphur is
atmosphere decreases during photosynthesis while it is
absorbed by the plant in the form of sulphate. It is then
replenished by Respiratory activities of the producers and
incorporated through a series of metabolic processes into
consumers
sulphur bearing amino acid which is incorporated in the
• Decomposition of organic wastes and dead organic proteins of autotroph tissues. It then passes through the
matter by decomposers grazing food chain.
• Burning of wood, and fossil fuels
70. Solution (b)
• Combustion of organic matter
Exp) Option b is correct.
• Weathering of carbonate containing rocks
Ecological succession is the process by which the
• Volcanic eruptions and hot springs structure of a biological community evolves over time.
• Forest fires, deforestation Statement 1 is incorrect: The entire sequence of
67. Solution (d) communities that successively change in a
Exp) Option d is correct. given area are called sere(s). The individual transitional
communities are termed seral stages or seral communities.
Statement 1 is correct. Nitrogen fixation is the process of
conversion of elemental nitrogen to ammonia, nitrites or Statement 2 is correct: The changes finally lead to
nitrates. Nitrogen needs to be fixed, before it can be taken a community that is in near equilibrium with the
up by plants. environment and that is called a climax community.
Statement 2 is correct. It can be achieved in 3 different 71. Solution (a)
ways. One of the ways is by thunder and lightning. The Exp) Option a is correct.
periodic thunderstorms convert the gaseous nitrogen
in the atmosphere to ammonia and nitrates. Statement 1 is correct: Secondary succession is the
sequential development of biotic communities after
Statement 3 is correct. Certain microorganisms like
the complete or partial destruction of the existing
Azotobacter, anaerobic Clostridium are capable of fixing
community. A mature or intermediate community may
atmospheric nitrogen into ammonium ions.
be destroyed by natural events such as floods, droughts,
68. Solution (c) fires, or storms or by human interventions such as
deforestation, agriculture, overgrazing, etc
Exp) Option c is correct.
Statement 2 is incorrect: Secondary succession is
Statement 1 is correct: Elements or mineral nutrients are
relatively faster as compared to primary succession.
always in circulation moving from non-living to living and
This is because the secondary succession starts on a well-
then back to the non-living components of the ecosystem
in a more or less circular fashion. This circulation in an developed soil already formed at the site.
ecosystem is known as biogeochemical cycle. Statement 3 is incorrect: When succession is brought
Statement 2 is incorrect: Phosphorus, calcium and about by living inhabitants of that community itself,
magnesium circulate by means of the sedimentary the process is called autogenic succession, while change
cycle. The element involved in the sedimentary cycle brought about by outside forces is known as allogenic
normally follows a basic pattern of flow through erosion, succession. Succession in which, initially the green plants
sedimentation, mountain building, volcanic activity and are much greater is quantity is known as autotrophic
biological transport through the excreta of marine birds. succession.
Statement 3 is correct: The hydrologic cycle is the 72. Solution (c)
continuous circulation of water in the Earth-atmosphere
Exp) Option c is correct.
system which is driven by solar energy. Water moves from
one reservoir to another by the processes of evaporation, Statement 1 is incorrect: Gross primary productivity of
transpiration, condensation, precipitation, deposition, an ecosystem is the rate of production of organic matter
runoff, infiltration, and groundwater flow. during photosynthesis. Gross primary productivity minus
respiration losses (R), is the net primary productivity
69. Solution (b) (NPP).
Exp) Option b is correct. Statement 2 is correct: Primary productivity depends
Statement 1 is incorrect. Although Sulphur cycle is on the plant species inhabiting a particular area. It also
mostly sedimentary, it involves gaseous components like depends on a variety of environmental factors, availability

217 Workbook
.
ENVIRONMENT

of nutrients and photosynthetic capacity of plants. Statement 3 is incorrect. The pyramid of biomass is
Therefore, it varies in different types of ecosystems. normally upright except for that observed in oceans
Statement 3 is correct: A succession is usually where large numbers of zooplanktons depend on a
characterized by the following: increased productivity, the relatively smaller number of phytoplanktons.
shift of nutrients from the reservoirs, increased diversity
of organisms, and a gradual increase in the complexity
of food webs. Thus, productivity increases following
ecological succession.
73. Solution: (a)
Exp) Option a is correct.
Decomposers break down complex organic matter into
inorganic substances like carbon dioxide, water and
nutrients and the process is called decomposition. The Statement 4 is incorrect. Pyramid of energy is the only
important steps in the process of decomposition are type of ecological pyramid that is always upright as the
fragmentation, leaching, catabolism, humification and energy flow in a food chain is always unidirectional. Also,
with every increasing trophic level, some energy is lost
mineralisation.
into the environment.
Pair 1 is correct. Fragmentation is the process of breaking
down of detritus into smaller particles by the detritivores.
Pair 2 is incorrect. The process of degradation of humus
by some microbes and release of inorganic nutrients
occurs by the process known as mineralisation. On the
other hand, Humification is the process of accumulation
of a dark coloured amorphous substance called humus 75. Solution: (b)
that is highly resistant to microbial activity. Humification
Exp) Option b is correct.
and mineralisation occur during decomposition in the
soil. A Nutrient cycle is a pathway by which a chemical
substance (nutrients like nitrogen, carbon, water,
Pair 3 is incorrect. Accumulation of organic matter into a phosphorus, etc) moves through the biotic and abiotic
substance is process of humification (not mineralization) compartments of the earth.
where humus is generated.
Statement 1 is incorrect: Phosphorous cycle completes
The humus is degraded by some microbes and release slower (not faster) than the Carbon cycle. Carbon Cycle
of inorganic nutrients occurs by the process known as is fast because it involves processes like respiration,
mineralisation. Bacterial and fungal enzymes degrade photosynthesis, combustion, absorption and
detritus into simpler inorganic substances. This process is evaporation for mobilising the nutrient. These are all
called as catabolism. quick physical and chemical reactions.
74. Solution: (a)
Exp) Option a is correct.
An ecological pyramid is a graphical representation
designed to show the biomass or bio productivity at each
trophic level in a given ecosystem. Ecological pyramids
begin with producers on the bottom (such as plants)
and proceed through the various trophic levels (such as
herbivores that eat plants, then carnivores that eat flesh,
then omnivores that eat both plants and flesh, and so on).
The highest level is the top of the food chain.
On the other hand, The Phosphorus Cycle involves
Statement 1 is correct. The pyramid of numbers is processes like weathering and erosion for mobilising the
inverted in the detritus food chain. Pyramid of numbers nutrient. These processes are slow and time taking.
takes into account the number of organisms in each
trophic level. The pyramid of numbers is usually upright Statement 2 is correct: The biggest point of difference
between the Carbon and Phosphorus nutrient cycles is
except for some situations like that of the detritus food
that Carbon cycle involves the atmosphere, whereas the
chain, where many organisms feed on one dead plant or
Phosphorus cycle does not involve atmosphere, rather
animal. it uses lithosphere and hydrosphere for its circulations.
Statement 2 is correct. Pyramid of Biomass takes into This is also the reason why Carbon is known as a Gaseous
account the amount of biomass produced by each trophic Nutrient Cycle, whereas Phosphorus is known as a
level. Sedimentary Nutrient cycle.

Workbook 218
.
ENVIRONMENT

Statement 3 is incorrect: A Biogeochemical cycle is


termed as Perfect when all the nutrients that entered
circulation get completely recycled and become available
for use again. A Gaseous nutrient cycle is perfect. This
is because processes like evaporation and condensation
or ammonification and denitrification are the types of
reversible physical and chemical reactions driving the
circulation.
However, a Sedimentary nutrient cycle is an example of
an Imperfect Biogeochemical Cycle, as many sediments
containing the nutrients that are in circulation get
locked away in immobile reservoirs like rocks and get
lost for a very long time, becoming unavailable for use.
77. Solution: (d)
Exp) Option d is correct
Option a is incorrect. Ecotone refers to a zone of
76. Solution: (a) junction between two or more diverse ecosystems. For
Exp) Option a is correct. instance, the mangrove forests are ecotone. It is because
A Biogeochemical cycle is a pathway by which a chemical they are located in a zone of junction between marine and
substance (nutrients like nitrogen, carbon, water, terrestrial ecosystems. Whereas, grasslands represent an
phosphorus, etc) moves through the biotic and abiotic ecotone between forest and desert.
compartments of the earth. These are called so because Option b is incorrect. Niche refers to the unique
this entire circulation pathway involves transfer of functional role or place of a species in an ecosystem.
nutrients through living organisms (bio) and components A niche is unique for a species and no two species in a
of earth (geo - like lithosphere, hydrosphere, atmosphere) habitat can have the same niche. Niche is important for
of nutrients (chemicals). There are broadly 2 categories of the conservation of organisms. If we need to conserve a
these cycles - Gaseous and Sedimentary biogeochemical species in its native habitat, we need to know about the
cycles. niche requirements of the species and ensure that all
requirements of its niche are fulfilled.
Sedimentary biogeochemical cycles are those in which the
main reservoir of the nutrient is lithosphere (sediments of Option c is incorrect. Biome is the terrestrial part of the
rock). For example: Sulphur cycle, Phosphorus cycle, etc. biosphere. They are characterized by climate, vegetation,
animal life and general soil type.
Statement 1 is correct: All sedimentary nutrient cycles
generally have two phases in the circulation of their Option d is correct. Seres or successional stages refers to
respective nutrients. One is the Rock Phase, in which the the stages leading to the climax community. It is part of
nutrient is locked as sediment in the rocks on earth. the Ecological succession which is the process of change
The other is the Solution Phase, in which the nutrient in the species structure of an ecological community over
contained in the sediments is loosened by weathering time. Plants and animal species in an area are replaced or
and transported to rivers or seas by erosion. Then it changed into another over a period of time. In this, one
dissolves in the water to create a mobile solution which community replaces another until a stable and mature
becomes absorbable by living organisms. So, nutrients climax community develops.
in a sedimentary biogeochemical cycle circulate through
rocks as well as aquatic bodies. 3.4. Terrestrial Ecosystem
Statement 2 is correct: Sedimentary nutrient cycles have 78. Solution (b)
a longer time span than Gaseous nutrient cycles. Here
Time Span means the total time it takes for a particular Exp) Option b is correct.
piece of nutrient containing sediment to enter the cycle, Tundra ecosystem is the ecosystem that is devoid of trees
go through the entire circulation and be locked into its and covered with snow for most of the year. They are
original form again. found in cold climates and in the regions with limited or
This is because sedimentary cycles involve processes like scarce rainfall.
weathering and erosion to mobilise the nutrients, which Statement 1 is incorrect: There are two types of tundra-
are slow and time taking processes. On the other hand, arctic and alpine. Arctic tundra occurs below the polar ice
Gaseous nutrient cycles are dependent on processes like cap. It occupies the northern fringe of Canada, Alaska,
evaporation, condensation, ammonification, etc. to European Russia, Siberia and island group of Arctic
mobilise their respective nutrients, which are very quick Ocean. Alpine tundra occurs at high mountains at all
chemical and physical reactions. latitudes.

219 Workbook
.
ENVIRONMENT

Statement 2 is correct: Organisms in Tundra have Statement 2 is correct. Due to deforestation, carbon
evolved to withstand extreme environment. Mammals of dioxide level in the atmosphere increases as due to
the tundra region have large body size, small tail and small deforestation, the stored carbon is released in the
ear to avoid the loss of heat from the surface. atmosphere contributing to global warming. More CO2
gets dissolved in the sea, thereby decreasing the Ocean’s
79. Solution (c) pH and making the oceans more acidic.
Exp) Option c is correct. Statement 3 is incorrect. Evapotranspiration is the
Tropical rain forests are among the most diverse and rich process of evaporating water from leaves through plant
communities on the earth. Both temperature and humidity transpiration during photosynthesis. Deforestation
remain high and more or less uniform. The annual rainfall reduces evapotranspiration, taking away this cooling
exceeds 200 cm and is generally distributed throughout function and causes local temperature to rise.
the year. The flora is highly diversified. The high rate of 83. Solution (d)
leaching makes these soils virtually useless for agricultural
purposes. They are mainly located between the latitudes Exp) Option d is correct.
of 23.5°N (the Tropic of Cancer) and 23.5°S (the Tropic of Statement 1 is correct. Heavy grazing reduces the
Capricorn)—the tropics. Tropical rainforests are found in mulch cover over the soil, so does the humus cover. This
Central and South America, western and central Africa, produces puddling of the surface layer. In turn it reduces
western India, Southeast Asia, the island of New Guinea, the infiltration of water into the soil and accelerates
and Australia. run off, resulting in soil erosion. Lack of humus cover
makes soil more vulnerable to wind erosion.
80. Solution (d) Statement 2 is correct. Due to heavy grazing, the quality
Exp) Option d is correct. of grasslands deteriorates rapidly, the mulch cover of the
Tropical Semi-evergreen forests have a mixture of the soil reduces, and microclimate becomes dry. Due to
wet evergreen trees and the moist deciduous trees. It is reduction of mulch cover, the soil and grasslands becomes
characterized by the dense forests and is filled with a large readily prone to invasions by xerophytic plants and
variety of trees of both types. Semi-evergreen forests are borrowing animals.
found in the Western Ghats, Andaman and Nicobar Statement 3 is correct. Heavy grazing changes and
Islands, and the Eastern Himalayas. Therefore, options disrupts the stratification and periodicity of the
1,2 and 3 are correct. primary producers. It thus disrupts the energy flow in
the ecosystem, which results in a breakdown of the
81. Solution (d) biogeochemical cycles of water, carbon and nitrogen.
Exp) Option d is correct.
84. Solution (c)
Many major fires in India are triggered mainly by human
activities. Sometimes even a small spark from a cigarette Exp) Option c is correct.
butt, or a carelessly discarded lit matchstick can set the Statement 1 is correct. Deserts are formed in regions
fire going. with less than 25 cm of annual rainfall, or sometimes in
Emerging studies link climate change (global warming) hot regions where there is more rainfall, but unevenly
to rising instances of fires globally, especially the massive distributed in the annual cycle. Deserts in temperate
fires of the Amazon forests in Brazil and in Australia in regions often lie in “rain shadows”, that is, where high
the last two years. Fires of longer duration, increasing mountains block off moisture from the seas. In these
intensity, higher frequency and highly inflammable nature rain shadow areas, the rainfall is low which can lead to
are all being linked to climate change. the formation of deserts.
In recent Uttarakhand forest fires, the lack of soil Statement 2 is correct. In the interior of the continents,
moisture too is being seen as a key factor. In two the temperatures are extreme due to continentality. In
consecutive monsoon seasons (2019 and 2020), rainfall these areas, the moisture laden winds from the oceans
has been deficient by 18% and 20% of the seasonal could not reach or reach with very little or no moisture.
average, respectively. This results in very little rainfall in the interiors leading to
Sometimes forest fires are also caused by natural causes formations of deserts.
like lightening. 85. Solution (b)
82. Solution (d) Exp) Option b is correct.
Exp) Option d is correct. Deserts are formed in regions with less than 25 cm of
Statement 1 is correct. Around 80% of global deforestation annual rainfall, or sometimes in hot regions, where there
is a result of agricultural production, which is also the is more rainfall, but unevenly distributed in the annual
leading cause of habitat destruction. Animal agriculture cycle. The plants in desert conserve water by various
— livestock and animal feed is a significant driver of adaptations.
deforestation, and is also responsible for approximately Statement 1 is correct: Leaves are absent or reduced in
60% of direct global greenhouse gas (GHG) emissions. size to reduce water loss.

Workbook 220
.
ENVIRONMENT

Statement 2 is correct: Leaves and stem are succulent and not an example of osmotrophs. Examples of osmotrophs
water storing. are earthworms, bacteria, fungi, etc.
Statement 3 is incorrect: Root system is well developed Bears are an example of Omnivores - a subcategory of
and spread over large area to absorb as much water as secondary consumers (i.e. type of Phagotrophs) that
possible. consume both plants and animals. Other examples of
this category are cockroach, man, etc
86. Solution (b)
Exp) Option b is correct.
3.5. Aquatic Ecosystem
This method of growing forests in urban areas was first
practised by Japanese botanist Akira Miyawaki in 1980s, 88. Solution (c)
after whom it is named. Exp) Option c is correct.
Forests regenerated by this method are claimed by some Statement 1 is correct: According to the United Nations
studies to grow 10 times faster and become 30 times Environment Programme, there are more cold-water
denser and 100 times more biodiverse than those coral reefs worldwide than tropical reefs. The largest cold-
planted through conventional methods. water coral reef is the Rost Reef off Norway.
This happens because a large quantity of seeds of Statement 2 is incorrect: Like tropical corals, they
different species are planted in a small space, thus provide habitat to other species, but deep-water corals do
making them grow denser and more diverse. not require zooxanthellae to survive.
This method encourages usage of a wide variety of Statement 3 is correct: Deep-sea corals together with
seeds and involves growing all levels of forest layers - other habitat-forming organisms host a rich fauna of
undergrowth, shrubs, trees and canopies. associated organisms. Lophelia reefs can host up to 1,300
The special feature of this method that makes it suitable species of fish and invertebrates. Various fish aggregate on
for urban forestry is that it prefers small plots of land and deep sea reefs. Deep sea corals, sponges and other habitat-
does not require big contiguous spaces to show results. forming animals provide protection from currents and
predators, nurseries for young fish, and feeding, breeding
87. Solution: (a) and spawning areas for numerous fish and shellfish
Exp) Option a is correct. species.
All living organisms need food in order to derive energy 89. Solution (c)
for life functions like growth, reproduction, movement, Exp) Option c is correct.
etc. The manner in which different organisms obtain food
is not uniform, rather it is varied. On the basis of how Statement 1 is incorrect: Since water temperatures are
they obtain food, organisms are classified into 3 groups, less subject to change, the aquatic organisms have narrow
discussed below. temperature tolerance limit. As a result, even small
changes in water temperature are a great threat to the
Pair 1 is correct: Autotrophs are organisms that are survival of aquatic organisms.
capable of manufacturing their own food. These
generally include green plants that have chlorophyll Statement 2 is correct: Transparency affects the extent of
and can use sunlight and nutrients to manufacture food. light penetration. Suspended particulate matters such as
So, Elephant grass, a type of major tropical grass that clay, silt etc make the water turbid. Consequently, it limits
grows to great heights in tropical areas, is an example of the extent of light penetration and the photosynthetic
plants, i.e. Autotrophs. This category also includes the activity in a significant way.
small green floating plants in aquatic ecosystems called Statement 3 is correct: Oxygen is less soluble in warm
phytoplanktons. water. Warm water also enhances decomposer activity.
Pair 2 is correct: Phagotrophs are organisms that are Therefore, increasing the temperature of a waterbody
not capable of producing their own food. They have increases the rate at which oxygen is depleted from
to kill and eat other organisms as food in order to water.
derive energy. They are also known as Heterotrophs. 90. Solution (d)
This category includes both Primary Consumers
(i.e. herbivores that consume plants) and Secondary Exp) option d is correct.
Consumers (i.e. carnivores/ omnivores that consume Statement 1 and 2 are correct: Seaweeds absorbs the
other animals). Snake is an example of Secondary nutrients including the heavy metals from the marine
Consumer. It eats small animals like birds, rats. etc, as it ecosystem. So, if there is imbalanced amount of nutrients
cannot manufacture its own food. So, it is an example of in the water, it also gets reflected in the seaweeds.
Phagotrophs. Therefore, acting as an indicator of the health of the
Pair 3 is incorrect: Osmotrophs are organisms that feed marine ecosystem.
on dead and decaying organic matter and break it down The green seaweed Ulva rigida can act as a bioindicator
into constituent nutrients while feeding on it to derive of metals (Zn, Cu, Pb and Cd) in a low-salinity coastal
energy. They are also known as Saprotrophs. Bears are environment

221 Workbook
.
ENVIRONMENT

Statement 3 is correct: Seaweed is highly suitable for 94. Solution (d)


bioethanol. Between 85 and 90% of seaweed is water, Exp) Option d is correct.
which means seaweed is very suitable for biofuel-making
methods like anaerobic digestion to make biogas and Option d is correct: Eutrophication-It is the process
fermentation to make ethanol. In addition, many seaweed of natural aging of a lake by nutrient enrichment
species, like sugar kelp, have high carbohydrate and low of its water. It is a natural process but there has been a
lignin content that is perfect for making bioethanol. radical acceleration in this process due to human
activities (heavy use of fertilizers, untreated sewage,
Statement 4 is correct: Seaweed is also used as an etc.). This phenomenon is called cultural or accelerated
ingredient in preparing toothpaste, cosmetics and paints. eutrophication.
91. Solution (b) Option a is incorrect: Biomagnification- It refers to
Exp) Option b is correct. increase in concentration of pollutants at successive
trophic level. This happens as toxic substance is
Statement 1 is incorrect. Lentic water systems are still or accumulated by an organism and cannot be metabolized
stagnant water bodies. They include pools, ponds, some or excreted, thus passed on to next higher trophic level.
swamps, bogs and lakes. Whereas, lotic water systems are
moving water bodies. They include freshwater streams, Option b is incorrect: Winterkill-In winter the oxygen
springs, rivulets, creeks, brooks, and rivers. in frozen waterbodies, which are covered with ice, gets
depleted and as a result fishes die. Due to the cutting off
Statement 2 is correct. In estuaries, fresh water from
light, Photosynthesis stops but respiration continues.
rivers meets ocean water and the two are mixed by action
of tides. Estuaries are highly productive as compared to Option c is incorrect: Chemical Oxygen Demand- the
the adjacent river or sea. Coastal bays, river mouths and amount of oxygen required to oxidize the total organic
tidal marshes form the estuaries. matter (biodegradable and non-biodegradable) present in
the water.
92. Solution (b)
Exp) Option b is correct.
Pair 1 is incorrectly matched: Neustons are those
unattached organisms which live at the air-water
interface. They are of two types with some living on top
of the air-water interface other spent their time beneath
the air-water interface. Ex- beetles. ‘Nektons’ are active
swimmers and are relatively large and powerful.
Pair 2 is correctly matched: Periphytons are organisms
which remain attached to stem and leaves of plants or
substances that emerge above the bottom mud.
Pair 3 is correctly matched: Benthos are those organisms
which are found living in the bottom of the water
mass. Practically every aquatic ecosystem contains well
developed benthos. 95. Solution (c)
Exp) Option c is correct.
93. Solution (a)
Statement 1 is incorrect. Phytoplankton are limited
Exp) Option a is correct.
to the uppermost layers of the ocean where light
Statement 1 is correct: Sunlight plays a major role in intensity is sufficient for photosynthesis to take place.
controlling the productivity of aquatic ecosystem. In the The photosynthetic rate varies with light intensity. These
photic zone there is good penetration of light and as micro-algae are present throughout the lighted regions of
result photosynthesis activity is confined only till this all the seas and oceans including the Polar Regions.
zone. Statement 2 is correct. Their total biomass is many
Statement 2 is correct: Transparency affects the extent times greater than that of the total plants on land
of light penetration and as result it affects photosynthesis and they serve as the “pasture grounds” in the aquatic
activity. Suspended particulate matters such as clay, environment.
silt, phytoplankton, etc. make water turbid and restrict Statement 3 is correct. Phytoplankton are microscopic
photosynthesis. plant organisms that live in aquatic environments, both
Statement 3 is incorrect: Respiration is the process salty and fresh.
of consumption of oxygen by the living things. It is
applicable throughout the water column, both in photic 96. Solution (b)
and aphotic zones. Exp) Option b is correct

Workbook 222
.
ENVIRONMENT

Statement 1 is incorrect: Due to Harmful Algal Blooms, mangrove plants. However, they do not come out from
oxygen is depleted in water and leads to low dissolved the mud. In fact, they emerge from the main trunk
oxygen levels. This happens due to decaying and
above the ground level.
decomposition of dead algae, which consumes oxygen.
Statement 2 is correct: Two common causes of algal Stilt roots or adventitious aerial roots help in providing
blooms are nutrient enrichment and warm waters. mechanical support to the plant.
Nutrient enrichment of waters in form of phosphate and
nitrate can cause algal blooms. Unusually warm waters
have been found conducive for blooms.

Adventitious roots or stilt roots

98. Solution (b)


97. Solution (c) Exp) Option b is correct.
Exp) Option c is correct.
Statement 1 is incorrect: West Bengal has the highest
Statement 1 is incorrect. Pneumatophores or blind
roots or air roots are one of the adaptations found in mangrove cover followed by Gujarat. West Bengal has
the Mangrove plants, however, they do not go down 42.45% of the country’s mangrove cover, while Gujarat
into the water. Rather they come out of the mud vertically has 23.66%.
and help the plant overcome respiration problem in the
anaerobic soil conditions. Statement 2 is correct: Mangroves exhibit Viviparity
mode of reproduction. i.e. seeds germinate in the tree
itself (before falling to the ground). This is an adaptative
mechanism to overcome the problem of germination in
saline water.
Statement 3 is correct: Mangrove plants have (additional)
special roots which help to impede water flow and thereby
enhance the deposition of sediment in areas (where it is
already occurring), stabilize the coastal shores, provide
breeding ground for fishes. It prevents coastal soil
Pneumatophores of Mangrove plants
erosion while also protecting coastal lands from tsunami,
Statement 2 is correct. Thick leaves containing salt
secreting glands are one of the adaptations found in the hurricanes and floods.
mangrove plants. They help the plant in dealing with the
99. Solution (a)
salinity stress. These leaves secrete excess salt, which can
be seen in the form of crystals of salt on the back of the Exp) Option a is correct
leaves.
Statement 1 is correct. The Sunderbans Delta is the
For dealing with salt stress, some plants block absorption
of salt at the roots itself. only mangrove forest in the world inhabited by tigers.
Statement 3 is correct. Mangroves exhibits Viviparity It constitutes over 60% of the country’s total mangrove
mode of reproduction i.e., seeds germinate in the tree forest area.
itself (before falling to the ground). This is an adaptive
Statement 2 is incorrect. Sunderbans forms the largest
mechanism to overcome the problem of germination in
saline water. river delta in the world. This is an arcuate (and not
Statement 4 is incorrect. Adventitious roots also cuspate) type of delta which features many active, short
called Stilt roots are one of the adaptations found in the distributaries pushing heavy sediment into their mouths.

223 Workbook
.
ENVIRONMENT

Statement 1 is correct. El Nino elevates the sea


temperature and destroys coral reefs. It is a climate
pattern that starts with a band of warm ocean water in
the central and east-central Pacific around the equator
and affects global weather.
Statement 2 is correct. Due to global warming or some
local phenomena, the average temperature of ocean
water increases. Most coral species live in waters close
to the warmest temperature they can tolerate i.e., a slight
increase in ocean temperature can harm corals.
Statement 3 is correct. Due to a rise in carbon dioxide
levels, there is an increase in the acidity of ocean water
which inhibits the corals’ ability to create calcareous
skeletons, which is essential for their survival.
100. Solution (b)
102. Solution: (c)
Exp) Option b is correct.
Exp) Option c is correct.
Mangroves are salt-tolerant variety of plants that
A pond is a shallow water body in which four
grow in warm tropical and subtropical tidal areas like
basic components of an ecosystem-- Productivity;
estuaries and marine coastlines. According to latest forest
Decomposition; Energy flow; and Nutrient cycling are
survey report, the total area in India under mangroves
well exhibited.
is 4,921 sq. km. Coringa mangrove forests in Andhra
Pradesh are located in the estuaries of the Godavari and Statement 1 is incorrect. There is unidirectional
the Krishna rivers. These mangroves are part of Coringa (not cyclic) movement of energy towards the higher
Wildlife Sanctuary. Bhitarkanika mangroves are located trophic levels and its dissipation and loss as heat to the
on the Odisha coast. It is also an important Ramsar environment. The energy enters the plants (from the sun)
wetland site famous for saltwater crocodiles and the through photosynthesis during the making of food. This
largest-known nesting ground for Olive Ridley Sea turtles. energy is then passed on from one organism to another in
Ratnagiri mangroves are located in Maharashtra along a food chain.
the Konkan coast. Other than these sites, mangroves The nutrient movement is a cyclic movement where the
are also present in Andaman and Nicobar Island, gulf of nutrients revolve round with an ecosystem.
Kutch and Sundarbans etc. Statement 2 is correct. The conversion of inorganic
into organic material by algae takes place with the
help of sunlight. The autotrophic components of the
pond ecosystem include phytoplankton, algae and the
submerged and marginal plants found at the edges.
Statement 3 is incorrect. The Limnetic Zone is generally
classified as the open water area of the lake or pond. This is
the typical area of dense fish populations because oxygen
levels are typically higher due to contact with the air.
103. Solution: (c)
Exp) Option c is correct.
Population refers to the number of individuals of a
particular species present within a geographical area at a
particular given time.
A lake is an example of an Aquatic Ecosystem (an
ecosystem consisting of primarily water). It is a freshwater
type aquatic ecosystem (low salt content).
Food Chain is the relationship of eating and being eaten
between various species in an ecosystem. This relationship
is often times depicted graphically through a pyramid
showing the various trophic levels. The size of each
trophic level in the pyramid diagram denotes the strength
Mangrove sites in India of the particular community in relation to the others.
Statement 1 is correct: The number of decomposers
101. Solution (d) is greater than that of any other organism class in the
Exp) Option d is correct. ecosystem. All the biomass in the food chain, including

Workbook 224
.
ENVIRONMENT

the plants and the animals, both big and small, when deep-water habitats. It therefore supports plants and
they die, are acted upon by millions of microorganisms animals specifically adapted to such shallow flooding or
like bacteria and fungi. They decompose the dead organic waterlogging E.g., Mangroves, lake littorals, floodplains
stuff to release nutrients back into the environment. These and other marshy or swampy areas.
are known as Detrivores and Decomposers. Statement 2 is incorrect: Lakes are less productive when
Statement 2 is correct: Zooplanktons are small, free compared to wetland from the viewpoint of ecosystem
floating animals like crustaceans, protozoans, etc that and biodiversity. Wetlands are usually rich in nutrients
are found near the surface of the water body like a lake. (derived from surroundings and their sediments) and
These animals feed on the phytoplanktons. Therefore, have abundant growth of aquatic Macrophytes. They
they occupy the next, i.e. second trophic level in an support high densities and diverse fauna, particularly
aquatic ecosystem like a lake. birds, fish and macroinvertebrates, and therefore, have
Phytoplanktons are small, often microscopic plants high value for biodiversity conservation.
like algae, etc that keep floating freely on the surface of Statement 3 is incorrect: The Ministry of Environment
the water. They are the primary producers of the lake has notified the Guidelines for implementing Wetlands
ecosystem and occupy the first trophic level. (Conservation and Management) Rules, 2017. Permission
Fishes belong to a group known as Nektons - a group of for carrying out any prohibited activity within a
animals that are swimmers. These feed on small animals notified wetland can only be given by the Ministry
like zooplanktons. of Environment upon a specific request made by State
Government. State govt alone cannot permit to use for
104. Solution: (a) any prohibited activity.
Exp) Option a is correct Statement 4 is incorrect: India has totally 27, 403
On the basis of the habitats, the world ecosystems can be wetlands, of which 23,444 are inland wetlands and
divided into terrestrial ecosystem and aquatic ecosystem. 3,959 are coastal wetlands. The coastal wetlands occupy
an estimated 6,750 sq km and are largely dominated by
Aquatic ecosystems are water-based habitats and they
mangrove vegetation. Most of the wetlands in India are
are not called as biomes. Further, aquatic systems can be
directly or indirectly linked with major river systems such
subdivided into following ecosystems
as the Ganges, Cauvery, Krishna, Godavari and Tapti.
Statement 1 is correct. Freshwater ecosystems are those
where range of salinity is less than 5 ppt. Brackish water 106. Solution: (c)
ecosystems are those where range of salinity is between 5 Exp) Option c is correct
to 35 ppt. Marine ecosystems are those where range of
According to some estimates, around 10% or more oceans
salinity is 35 ppt or more. Thus, Brackish water ecosystem
around the world are now dead zones. This year’s Gulf of
is more saline than freshwater ecosystem and less saline
Mexico dead zone was the largest ever recorded. The dead
than marine water ecosystem.
zone is spread over 16,400 square kilometres. Following is
Statement 2 is incorrect. Pangong Tso is a brackish the explanation about dead zones:
lake (not freshwater lake) spanning the Indian/ Chinese Statement 1 is correct. Dead Zones are low-oxygen areas
border, in a valley in the upper drainage basin of the Indus in the oceans and lakes across the world where aquatic
River. Due to its extreme salinity, Pangong Tso does not life cannot survive. Only a few organisms can survive in
have any vegetation in the deeper parts, but at the margins hypoxic conditions, and other needs oxygen to survive,
and marshy areas, typical, steppe vegetation is seen. hence, these areas are called dead zones.
Statement 3 is incorrect. Dolphins are animals of Statement 2 is incorrect. Dead zones are created due
aquatic ecosystems. They are found in the sea, brackish to increase in nutrients and not less nutrients. When a
water and fresh water ecosystems. For example, A total water body receives too many nutrients such as phosphorus
of 15 species of marine and harbour dolphins have been and nitrogen, it causes a change in the productivity of the
reported from Indian waters and coasts. Besides, the ecosystem. It is called eutrophication, a process due to
Irrawaddy Dolphins are found in brackish waters of which dead zones occur. When the level of these nutrients
Odisha. Amongst the fresh water dolphins, the Gangetic is normal, it helps in the growth of cyanobacteria or
Dolphin is found in the Ganges River and its tributaries. blue-green algae. But when the levels of these nutrients
Apart from this, Indus River Dolphins have also been are more than the required amount, cyanobacteria grow
reported in Indus River in Punjab. out of control, which can be dangerous. They deplete the
105. Solution: (a) water of oxygen before decomposing, thereby suffocating
the species living in that area.
Exp) Option a is correct
Statement 3 is correct. Global warming increases
A wetland is an area of land that is saturated with water, the temperature of the water. Warm water holds less
either seasonally or permanently. dissolved oxygen than cold water because the molecules
Statement 1 is correct - Wetlands are transition zones are moving faster than in cold water and thereby allow
(ecotone) between terrestrial and aquatic ecosystems. oxygen to escape from the water. Also, Freshwater can hold
These habitats experience periodic flooding from adjacent more dissolved oxygen than saltwater because saltwater

225 Workbook
.
ENVIRONMENT

has less space for oxygen molecules due to the sodium Option 3 is incorrect. Oligotrophic lakes are usually
and chloride ions it contains. Therefore, the warmer and found in the cold regions of the world where mixing of
saltier the water, the less dissolved oxygen it will contain. nutrients is rare and slow due to the low temperatures of
the lake waters.
107. Solution: (d)
Option 4 is incorrect: The waters of these lakes are usually
Exp) Option d is correct. quite clear due to the limited growth of algae in the lake.
A typical lake has three distinct zones (limnetic, littoral So turbidity is low.
and the benthic zone) of biological communities linked
to its physical structure. With regard to light penetration, 109. Solution: (d)
a lake can have five distinct zones- Euphotic, Littoral, Exp) Option d is correct.
Limnetic, Profoundal and Benthic .
Oceans are single, large, continuous body of water
encircling all the landmass of the earth. They account for
four- fifth of the Southern Hemisphere and three fifth of
the Northern Hemisphere containing 97.2 percent of the
world’s total water.
Statement 1 is correct. Phytoplankton, tiny plant-like
organisms that live in the sea and oceans are responsible
for at least 50% of the oxygen on Earth. The disappearance
of oceans will result in disappearance of majority
phytoplankton’s resulting in decline in oxygen levels. The
loss of oxygen is significant enough to affect the planetary
cycling of elements such as nitrogen and phosphorous
Statement 1 is correct: Littoral zone refers to the which are essential for plants and overall life on Earth.
nearshore area that is the shallow and warmest part Statement 2 is correct. The oceans have always been
of the lake where sunlight penetrates all the way to the a great source of food and other products of value to
bottom sediments and allows rooted aquatic plants, or
man. The animals and plants of the sea constitute a vast
macrophytes, and attached algae to grow. This zone is
resource from which man can derive food, fertilizers for
most affected by changes at the shoreline.
agriculture and raw material for industry.
Statement 2 is incorrect: Profundal zone refers to the
Statement 3 is correct. The ocean’s surface has six or more
area in a deep lake where no light reaches. Limnetic zone
refers to the area of open water where the lake is too deep permanent centres of high pressure. These high-pressure
to allow rooted plant growth. Instead, this zone contains a areas give birth to the planetary wind system over the
variety of free-floating organisms such as phytoplankton earth. These planetary winds determine the amount of
and zooplankton. rainfall and its distribution over the earth’s surface. Hence
their disappearance will disrupt the overall atmospheric
Statement 3 is correct: Benthic Zone refers to bottom pressure system across globe.
sediment where a diverse variety of activities take place.
The lake bottom may be made up of sand, mud, marl and/ 110. Solution: (c)
or a mixture of any or all of these materials. Changes that
Exp) Option c is correct.
occur in the benthic zone of the littoral zone are sensitive
to shoreline development and can result in reduced food The total amount of heat stored by the oceans is called
sources, spawning and safety areas. So there is reduced “ocean heat content,” and measurements of water
diversity in this zone which results in a reduction in fish temperature reflect the amount of heat in the water at a
populations. time and location.
Statement 1 is correct. When sunlight reaches the Earth’s
108. Solution: (a)
surface, the world’s oceans absorb some of this energy and
Exp) Option a is correct. store it as heat. Water has a much higher heat capacity
Oligotrophic lakes are those that are unproductive: net than air and holds four times more heat than the air,
primary production is low. nutrients are in poor supply, meaning the oceans can absorb larger amounts of heat
and secondary production is depressed. Eutrophic lakes, energy with only a slight increase in temperature. Hence,
on the other hand, are productive. the top few meters of the ocean store as much heat as
Option 1 is correct: An oligotrophic lake or water body is Earth’s entire atmosphere.
one which has a relatively low primary productivity due to Statement 2 is correct. Covering more than 70% of
the low nutrient content in the lake. Earth’s surface, our global ocean has a very high heat
Option 2 is incorrect: The waters of these lakes are usually capacity. It has absorbed 90% of the global warming
quite clear due to the limited growth of algae in the lake. that has occurred in recent decades due to increasing
The waters of such lakes are of high-drinking quality. greenhouse gases.

Workbook 226
.
ENVIRONMENT

ENVIRONMENT
ENVIRONMENTAL POLLUTION
5. Vultures which used to be very common in
4.1. Previous Years Questions Indian countryside some years ago are rarely seen
1. There is a concern over the increase in harmful nowadays. This is attributed to
algal blooms in the seawaters of India. What could [UPSC CSE Pre. 2012]
be the causative factors for this phenomenon? (a) The destruction of their nesting sites by new
[UPSC CSE Pre. 2011] invasive species.
1. Discharge of nutrients from the estuaries. (b) A drug used by cattle owners for treating their
2. Run-off from the land during the monsoon. diseased cattle.
3. Upwelling in the seas. (c) Scarcity of food available to them.
(d) A widespread, persistent and fatal disease among
Select the correct answer from the codes given them.
below:
(a) 1 only 6. Biomass gasification is considered to be one of the
(b) 1 and 2 only sustainable solutions to the power crisis in India.
(c) 2 and 3 only In this context, which of the following statements
(d) 1, 2 and 3 is/are correct? [UPSC CSE Pre. 2012]
1. Coconut shells, groundnut shells and rice husk
2. Consider the following: [UPSC CSE Pre. 2011] can be used in biomass gasification.
1. Carbon dioxide 2. The combustible gases generated consist of
2. Oxides of Nitrogen hydrogen and carbon dioxide only.
3. Oxides of Sulphur 3. The combustible gases generated from biomass
Which of the above is/are the emission/emissions gasification can be used for direct heat generation
from coal combustion at thermal power plants? but not in internal combustion engines.
(a) 1 only Select the correct answer using the codes given
(b) 2 and 3 only below:
(c) 1 and 3 only (a) 1 only
(d) 1, 2 and 3 (b) 2 and 3 only
3. Recently, “oilzapper” was in the news. What is it? (c) 1 and 3 only
[UPSC CSE Pre. 2011] (d) 1, 2 and 3
(a) It is an eco-friendly technology for the 7. With reference to the usefulness of the by-
remediation of oily sludge and oil spills products of sugar industry, which of the following
(b) It is the latest technology developed for under- statements is/are correct? [UPSC CSE Pre. 2013]
sea oil exploration
(c) It is a genetically engineered high biofuel- 1. Bagasse can be used as biomass fuel for the
yielding maize variety generation of energy.
(d) It is the latest technology to control the 2. Molasses can be used as one of the feedstocks for
accidentally caused flames from oil wells. the production of synthetic chemical fertilizers.
3. Molasses can be used for the production of
4. Lead, ingested or inhaled, is a health hazard. After ethanol.
the addition of lead to petrol has been banned,
what still are the sources of lead poisoning? Select the correct answer using the codes given
[UPSC CSE Pre. 2012] below:
1. Smelting units (a) 1 only
2. Pens and pencils (b) 2 and 3 only
(c) 1 and 3 only
3. Paints
(d) 1, 2 and 3
4. Hair oils and cosmetics
Select the correct answer using the codes given 8. Which of the following can be found as pollutants
below: in the drinking water in some parts of India?
[UPSC CSE Pre. 2013]
(a) 1, 2 and 3 only
(b) 1 and 3 only 1. Arsenic
(c) 2 and 4 only 2. Sorbitol
(d) 1, 2, 3 and 4 3. Fluoride

227 Workbook
.
ENVIRONMENT

4. Formaldehyde 2. Neem seeds are used in the manufacture of


5. Uranium biofuels and hospital detergents.
Select the correct answer using the codes given 3. Neem oil has applications in pharmaceutical
below. industry.
(a) 1 and 3 only Which of the statements given above is/are correct?
(b) 2, 4 and 5 only (a) 1 and 2 only
(c) 1, 3 and 5 only (b) 3 only
(d) 1, 2, 3, 4 and 5 (c) 1 and 3 only
(d) 1, 2 and 3
9. Due to improper/indiscriminate disposal of hold
and used computers or their parts, which of the 13. It is possible to produce algae based biofuels, but
following are released into the environment as what is/are the likely limitation(s) of developing
e-waste? [UPSC CSE Pre. 2013] countries in promoting this industry?
1. Beryllium [UPSC CSE Pre. 2017]
2. Cadmium 1. Production of algae based biofuels is possible in
3. Chromium seas only and not on continents.
4. Heptachlor 2. Setting up and engineering the algae based
5. Mercury biofuels production requires high level of
expertise/ technology until the construction is
6. Lead
complete.
7. Plutonium
3. Economically viable production necessitates the
Select the correct answer using the codes given setting up of large scale facilities which may raise
below: ecological and social concerns.
(a) 1, 3, 4, 6 and 7 only Select the correct answer using the code given below:
(b) 1, 2, 3, 5 and 6 only
(a) 1 and 2 only
(c) 2, 4, 5 and 7 only
(b) 2 and 3 only
(d) 1, 2, 3, 4, 5, 6 and 7
(c) 3 only
10. There is some concern regarding the nanoparticles (d) 1, 2 and 3
of some chemical elements that are used by the
14. In the context of solving pollution problems, what
industry in the manufacture of various products.
is/are the advantage/advantages of bioremediation
Why? [UPSC CSE Pre. 2014]
technique? [UPSC CSE Pre. 2017]
1. They can accumulate in the environment and
1. It is a technique for cleaning up pollution by
contaminate water and soil.
enhancing the same biodegradation process that
2. They can enter the food chains. occurs in nature.
3. They can trigger the production of free radicals. 2. Any contaminant with heavy metals such
Select the correct answer using the code given below. as cadmium and lead can be readily and
(a) 1 and 2 only completely treated by bioremediation using
(b) 3 only microorganisms.
(c) 1 and 3 only 3. Genetic engineering can be used to create
(d) 1, 2 and 3 microorganisms specifically designed for
bioremediation.
11. Brominated flame retardants are used in
many household products like mattresses and Select the correct answer using the code given below:
upholstery. Why is there some concern about their (a) 1 only
use? [UPSC CSE Pre. 2014] (b) 2 and 3 only
1. They are highly resistant to degradation in the (c) 1 and 3 only
environment. (d) 1, 2 and 3
2. They are able to accumulate in humans and 15. Which of the following is/are the possible
animals. consequence/s of heavy sand mining in river
Select the correct answer using the code given below. beds? [UPSC CSE Pre. 2018]
(a) 1 only 1. Decreased salinity in the river
(b) 2 only 2. Pollution of groundwater
(c) Both 1 and 2 3. Lowering of the water table
(d) Neither 1 nor 2
Select the correct answer using the code given below:
12. With reference to Neem tree, consider the (a) 1 only
following statements: [UPSC CSE Pre. 2014] (b) 2 and 3 only
1. Neem oil can be used as a pesticide to control the (c) 1 and 3 only
proliferation of some species of insects and mites. (d) 1, 2 and 3

Workbook 228
.
ENVIRONMENT

16. Why is there a great concern about the ‘microbeads’ Select the correct answer using the code given below.
that are released into environment? (a) 1 and 2 only
[UPSC CSE Pre. 2019] (b) 2 and 3 only
(a) They are considered harmful to marine (c) 1 and 3 only
ecosystems. (d) 1, 2 and 3
(b) They are considered to cause skin cancer in
children. 21. According to India’s National Policy on Biofuels,
(c) They are small enough to be absorbed by crop which of the following can be used as raw materials
plants in irrigated fields. for the production of biofuels?
(d) They are often found to be used as food [UPSC CSE Pre. 2020]
adulterants. 1. Cassava
2. Damaged wheat grains
17. Consider the following: [UPSC CSE Pre. 2019]
3. Groundnut seeds
1. Carbon monoxide
4. Horse grams
2. Methane
5. Rotten potatoes
3. Ozone
6. Sugar beet
4. Sulphur dioxide
Select the correct answer using the code given below:
Which of the above are released into atmosphere due
(a) 1, 2, 5 and 6 only
to the burning of crop/biomass residue?
(b) 1, 3, 4 and 6 only
(a) 1 and 2 only (c) 2, 3, 4 and 5 only
(b) 2, 3 and 4 only (d) 1, 2, 3, 4, 5 and 6
(c) 1 and 4 only
(d) 1, 2, 3 and 4 22. Consider the following statements:
[UPSC CSE Pre. 2020]
18. In the context of which one of the following are
the terms ‘pyrolysis and plasma gasification’ 1. Coal ash contains arsenic, lead and mercury.
mentioned? [UPSC CSE Pre. 2019] 2. Coal-fired power plants release sulphur dioxide
(a) Extraction of rare earth elements and oxides of nitrogen into the environment.
(b) Natural gas extraction technologies 3. High ash content is observed in Indian coal.
(c) Hydrogen fuel-based automobiles Which of the statements given above is/are correct?
(d) Waste-to-energy technologies (a) 1 only
19. In the context of proposals to the use of hydrogen - (b) 2 and 3 only
enriched CNG (H -CNG) as fuel for buses in public (c) 3 only
transport, consider the following statements: (d) 1, 2 and 3
[UPSC CSE Pre. 2019] 23. Which of the following are the reasons/factors for
1. The main advantage of the use of H -CNG is the exposure to benzene pollution?
elimination of carbon monoxide emissions. [UPSC CSE Pre. 2020]
2. H -CNG as fuel reduces carbon dioxide and 1. Automobile exhaust
hydrocarbon emissions. 2. Tobacco smoke
3. Hydrogen up to one -fifth by volume can be 3. Wood burning
blended with CNG as fuel for buses.
4. Using varnished wooden furniture
4. H -CNG makes the fuel less expensive than
5. Using products made of polyurethane
CNG.
Select the correct answer using the code given below:
Which of the statements given above is/are correct?
(a) 1, 2 and 3 only
(a) 1 only
(b) 2 and 4 only
(b) 2 and 3 only
(c) 1, 3 and 4 only
(c) 4 only
(d) 1, 2, 3, 4 and 5
(d) 1, 2, 3 and 4
20. Which of the following statements are correct 24. Among the following crops, which one is the most
about the deposits of ‘methane hydrate? important anthropogenic source of both methane
[UPSC CSE Pre. 2019] and nitrous oxide? [UPSC CSE Pre. 2022]
1. Global warming might trigger the release of (a) Cotton
methane gas from these deposits. (b) Rice
(c) Sugarcane
2. Large deposits of ‘methane hydrate’ are found in (d) Wheat
Arctic Tundra and under the seafloor.
3. Methane in atmosphere oxidizes to carbon 25. Consider the following: [UPSC CSE Pre. 2022]
dioxide after decade or two. 1. Carbon monoxide

229 Workbook
.
ENVIRONMENT

2. Nitrogen oxide Select the correct answer using the code given below.
3. Ozone (a) 1 and 2 only
4. Sulphur dioxide (b) 2 and 3 only
Excess of which of the above in the environment is/ (c) 1 only
are cause(s) of acid rain? (d) 1 and 3 only
(a) 1, 2 and 3 30. Consider the following statements regarding
(b) 2 and 4 only Tropospheric Ozone:
(c) 4 only 1. It is beneficial as it protects the Earth from Sun’s
(d) 1, 3 and 4 ultraviolet rays.
2. It is formed by photochemical reactions
Practice Question between oxides of nitrogen and volatile organic
compounds.
4.2. Pollution and Major pollutants 3. Its concentration in the troposphere is higher
26. Vultures which used to be very common in during the summer months as compared to the
Indian countryside some years ago are rarely seen winter months.
nowadays. This is attributed to Which of the statements given above is/are correct?
(a) The destruction of their nesting sites by new (a) 1 and 3 only
invasive species. (b) 2 only
(b) A drug used by cattle owners for treating their (c) 2 and 3 only
diseased cattle. (d) 1, 2 and 3
(c) Scarcity of food available to them.
(d) A widespread, persistent and fatal disease among 31. In India, the use of carbofuran, methyl
them. parathion, phorate and triazophos is viewed with
apprehension. These chemicals are used as
27. There is a concern over the increase in (a) pesticides in agriculture
harmful algal blooms in the seawaters of (b) preservatives in processed foods
India. What could be the causative factors for (c) fruit-ripening agents
this phenomenon? (d) moisturising agents in cosmetics
1. Discharge of nutrients from the estuaries. 32. Which of the following are the potential
2. Run-off from the land during the monsoon. applications/uses of fly ash?
3. Upwelling in the seas. 1. Portland cement concrete (PCC)
Select the correct answer from the codes given 2. Road construction
below: 3. Carbon dioxide capture and storage
(a) 1 only 4. Geopolymer
(b) 1 and 2 only 5. Ceramic tiles
(c) 2 and 3 only
(d) 1, 2 and 3 Select the correct answer using the code given below:
(a) 1 and 2 only
28. Why have the chemicals Chlordecone, (b) 3, 4 and 5 only
Hexachlorobutadiene and Pentachlorobenzene (c) 1, 2 and 5 only
been banned by the government in India? (d) 1, 2, 3, 4 and 5
(a) They were responsible for the deaths of vultures
in the past in India. 33. Consider the following pairs of pollutants and
(b) These are toxic chemicals used in the production diseases caused by them:
of plastic.
(c) They are mentioned under Stockholm Pollutant Disease
Convention on Persistent Organic Pollutants. 1. Lead A. stomach cramps and
(d) India has recently banned them as per London nausea
Convention on Marine Pollution.
2. Zinc B. Itai-itai disease
29. With reference to Sulphur dioxide emissions, 3. Arsenic C. damage to peripheral
which of the following statements is/are correct? and central nervous
1. India is among the top five anthropogenic system
sulphur dioxide emitters in the world.
2. Volcanic eruptions are one of the major causes of 4. Cadmium D. skin cancer
SO2 emissions in India. Select the correct answer using the code given below:
3. Electrostatic precipitator can be used to remove (a) 1-B, 2-D, 3-A, 4-C
industrial Sulphur dioxide emissions. (b) 1-C, 2-A, 3-B, 4-D

Workbook 230
.
ENVIRONMENT

(c) 1-C, 2-A, 3-D, 4-B Select the correct answer using the code given below
(d) 1-B, 2-A, 3-D, 4-C (a) 1, 2, 3 and 4 only
34. Which of the following can be the sources of (b) 2, 4 and 5 only
Nitrogen oxide emissions? (c) 2, 3 and 4 only
1. Lightning (d) 1, 2, 3, 4 and 5
2. Microbial processes in soils 39. Which of the following methods can be employed
3. Unvented gas stoves to tackle the menace of oil spills?
4. Hydro power plant 1. Bregoli
5. Gardening equipment 2. Containment booms
Select the correct option using the code given below: 3. Skimmers
(a) 1, 2 and 4 only Select the correct answer using the code given below:
(b) 2 and 3 only (a) 2 only
(c) 1, 2, 3 and 5 only
(b) 2 and 3 only
(d) 1, 2, 3, 4 and 5
(c) 1 and 2 only
35. Which of the following correctly describes the (d) 1, 2 and 3
Zero Liquid Discharge concept?
40. With reference to environment pollutants,
(a) Segregation of domestic biodegradable sewage
waste at the source. consider the following:
(b) Sealing and dumping of all radioactive waste 1. Primary pollutants persist in the form in which
from nuclear power plants in underground they are added to the environment.
containers. 2. Secondary pollutants are formed by the
(c) Collection and reuse of all fertiliser interaction among the primary pollutants.
rich agricultural solid waste to prevent 3. Ozone is a secondary pollutant
eutrophication.
(d) Recycling and chemical treatment of wastewater Which of the statements given above are correct?
in water intensive industries which is reduced to (a) 1 and 2 only
solid waste. (b) 2 and 3 only
(c) 1 and 3 only
36. Consider the following statements:
(d) 1, 2 and 3
1. Incineration.
2. Torrefaction. 41. With reference to ozone as a pollutant, consider
3. Hydrothermal carbonization. the following:
4. Bio methanation. 1. It is produced by burning fossil fuels like coal
Which of the following given above can be employed and petroleum.
for waste to energy generation? 2. It can lead to the formation of photochemical
(a) 1 and 4 only smog in the atmosphere.
(b) 3 and 4 only 3. Only ground level ozone is considered a
(c) 2, 3 and 4 only pollutant.
(d) 1, 2, 3 and 4
Which of the statements given above is/are correct?
37. With respect to bioremediation, the process of (a) 1 and 2 only
injecting air under pressure below the water table (b) 2 and 3 only
to increase ground water oxygen concentration (c) 1 and 3 only
which enhances the process of bio degradation is (d) 1, 2 and 3
known as:
(a) Bioreactor 42. With reference to PM2.5 pollutant, consider the
(b) Biosparging following statements:
(c) Bioaugmentation 1. Unlike PM10, PM2.5 is limited to outdoors
(d) Bioventing without causing indoor pollution.
38. Which of the following could lead to creation of 2. They only occur due to anthropogenic activities
e-waste? and not naturally.
1. Electric cookers Which of the statements given above is/are correct?
2. Circuit boards (a) 1 only
3. Solar panels (b) 2 only
4. Diabetic testing equipment (c) Both 1 and 2
5. Defibrillator (d) Neither 1 nor 2

231 Workbook
.
ENVIRONMENT

43. With reference to smog, consider the following 2. They are a source of air pollution.
statements: 3. India has not banned the microplastic yet.
1. It refers to a condition of fog that has some soot 4. Cosmetic manufacturers use microplastic to
or smoke in it. fight against ageing.
2. Photochemical smog is produced when sunlight Select the correct answer using the code given below:
reacts with nitrogen oxides and volatile organic
(a) 1 and 3 only
compounds (VOC).
(b) 1, 2 and 4 only
3. Fast blowing winds are one of the conditions (c) 2 and 3 only
necessary for the formation of smog. (d) 1, 2 and 3 only
Which of the statements given above is/are correct?
48. Which of the following statements is/are correct
(a) 1 and 2 only
regarding the radioactive pollution?
(b) 2 and 3 only
(c) 1 and 3 only 1. Most of wastes produced by nuclear power
(d) 1, 2 and 3 industry is high level waste.
2. Ionizing radiations have less penetration power
44. Consider the following statements about ‘Fly Ash’: as compared to Non-ionizing radiations.
1. It contains Silicon dioxide, Aluminium silicate 3. Non-ionizing radiations have longer wavelength
and oxides of calcium. as compared to Ionizing radiations
2. It can be mixed with cement to reduce the cost
Select the correct answer using the code given below:
of construction.
(a) 1 only
Which of the statements given above is/are incorrect? (b) 2 and 3 only
(a) 1 only (c) 3 only
(b) 2 only (d) 1 and 3 only
(c) Both 1 and 2
(d) Neither 1 nor 2 49. With reference to stubble burning, consider the
following statements:
45. With reference to the Fly Ash Utilization Policy in
India, consider the following: 1. It is a traditional practice in Punjab and Haryana
1. The use of fly ash has been extended to a distance to clean off the rice chaff to prepare the fields for
of 300 km from the power plant from the earlier winter sowing.
100 km distance. 2. If stubble is left in the field, pests like termites
2. ASHTRACK Mobile App has been launched by may attack the upcoming crop.
the Ministry of Road Transport and Highways. 3. Burning stubble on ground destroys the
3. At present, around 10% of the fly ash is being nutrients in the soil, making it less fertile.
utilized in India. Which of the statements given above is/are correct?
Which of the statements given above is/are correct? (a) 1 and 2 only
(a) 1 only (b) 2 and 3 only
(b) 2 only (c) 1 and 3 only
(c) 3 only (d) 1, 2 and 3
(d) 1, 2 and 3
50. Torrefaction technology, recently seen in news, is
46. Which among the following are the concerns
related to what?
regarding the industrial usage of nanoparticles?
1. They can enter the food chains and cause (a) It can be used to tackle the issue of stubble
depletion of biodiversity. burning.
2. They can accumulate in the environment and (b) It is used to convert coal to coking coal for use in
contaminate water and soil. steel industries.
3. They can trigger the production of free radicals (c) It is related to restoration of corals and stop the
that leads to destruction of ozone layer. phenomenon of coral bleaching.
(d) It is a technology to convert saline water to
Select the correct answer using the code given below:
potable fresh water for drinking.
(a) 1 and 2 only
(b) 3 only 51. With reference to the PUSA Decomposer, consider
(c) 1 and 3 only the following statements:
(d) 1, 2 and 3 1. It is essentially a fungi-based liquid solution that
47. Which of the following statements is/are correct can soften hard stubble and hence prevent the
regarding microplastics? need to burn stubble.
1. They are non-biodegradable. 2. It has been developed by NITI AAYOG.

Workbook 232
.
ENVIRONMENT

Which of the statements given above is/are correct? 55. With reference to radioactive pollution consider
(a) 1 only the following statements:
(b) 2 only 1. The effects of radioactive pollutant depends on
(c) Both 1 and 2 its half-life and energy releasing capacity.
(d) Neither 1 nor 2 2. Due to presence of higher salt concentrations in
seawater, radioactivity in sea is lesser than rivers.
52. Consider the following statements with respect to
‘Control of Air Pollution’: 3. Gamma ray is the strongest type of radiation
which can easily penetrate human skin.
1. Arresters are used to separate particulate matter
from the contaminated air. Which of the statements given above is/are correct?
2. The gaseous pollutants can be controlled through (a) 1 only
the techniques of Combustion and Adsorption. (b) 1 and 3 only
(c) 2 and 3 only
3. Adding more lead to petrol will help in reducing
(d) 1, 2 and 3
air pollution.
Which of the statement(s) given above is/are correct? 56. With reference to the Green Skill Development
(a) 1 and 2 only Programme, consider the following statements:
(b) 2 only 1. The programme utilises the network of
(c) 3 only Environmental Information System (ENVIS)
(d) 1, 2 and 3 hubs for skill development.
2. Under the programme, an annual Green General
53. With respect to techniques of bioremediation of Skill index evaluates the progress of states
contaminants, which of the following pairs is/are and union territories on environmental skill
correctly matched? parameters.
Technique Description Which of the above statements is/are correct?
(a) 1 only
1. Bioventing Injection of air under (b) 2 only
pressure below the (c) Both 1 and 2
water table to increase (d) Neither 1 nor 2
oxygen concentration in
groundwater 57. Due to improper/indiscriminate disposal
2. Rhizo-filtration A water remediation of hold and used computers or their parts,
technique which involves which of the following are released into the
the uptake of contaminants environment as e-waste?
by plant roots 1. Beryllium
3. Landfarming Contaminated soil is 2. Cadmium
excavated and periodically 3. Chromium
tilled until pollutants are 4. Heptachlor
degraded. 5. Mercury
Select the correct answer using the code given below: 6. Lead
(a) 1 and 2 only 7. Plutonium
(b) 2 and 3 only Select the correct answer using the codes given
(c) 3 only below:
(d) 1, 2 and 3 (a) 1, 3, 4, 6 and 7 only
54. Consider the following sentences about ‘Green (b) 1, 2, 3, 5 and 6 only
Crackers’: (c) 2, 4, 5 and 7 only
(d) 1, 2, 3, 4, 5, 6 and 7
1. Green crackers do not contain banned chemicals
such as lithium and arsenic. 58. Which of the following can be found as
2. They release water vapour and do not allow the pollutants in the drinking water in some parts of
dust particles to rise. India?
3. They are developed by the Council for Scientific 1. Arsenic
and Industrial Research (CSIR). 2. Sorbitol
Which of the statements given above is/are correct? 3. Fluoride
(a) 1 and 2 only 4. Formaldehyde
(b) 2 only 5. Uranium
(c) 2 and 3 only Select the correct answer using the codes given
(d) 1, 2 and 3 below.

233 Workbook
.
ENVIRONMENT

(a) 1 and 3 only 62. Consider the following statements with reference
(b) 2, 4 and 5 only to the Bio indicators of pollution:
(c) 1, 3 and 5 only 1. Bio indicators are living organisms which are
(d) 1, 2, 3, 4 and 5 utilized to screen the health of the natural
59. With reference to light pollution, consider the ecosystem in the environment.
following statements: 2. Frogs are Bio indicators of quality of environment
1. Light pollution is the excessive or inappropriate and changes in environment.
use of outdoor artificial light. 3. Decreased rate of planktons are the indication of
2. Light pollution impacts animal behaviours and high centralizations of phosphorus and nitrogen
their migration patterns. in water.
3. The dark-sky movement is a worldwide effort
aimed at minimizing the impact of artificial light Which of the statements given above is/are correct?
at night. (a) 1 and 2 only
Which of the statements given above is/are correct? (b) 2 and 3 only
(a) 1 and 2 only (c) 1 and 3 only
(b) 2 and 3 only (d) 1, 2 and 3
(c) 1 and 3 only
(d) 1, 2 and 3 63. Which of following can be found in Hospital
waste?
60. Consider the following statements with reference 1. Infectious waste
to Plastic pollution:
2. Sharps waste
1. Biodegradable plastic degrades significantly
faster in the seawater compared to freshwater. 3. Cytotoxic waste
2. Polyhydroxyalkanoates are biodegradable 4. Radioactive waste
plastics synthesized by different species of 5. Heavy metals
Bacteria and Archaea.
Select the correct answer using the codes given
3. Polylactic acid (PLA) is a biodegradable below:
polyester widely known for its tensile strength
and transparency. (a) 1, 2, 3 and 4 only
(b) 2, 3, 4 and 5 only
Which of the statements given above is/are correct?
(c) 1, 3, and 5 only
(a) 1 and 2 only
(b) 2 and 3 only (d) 1, 2, 3, 4 and 5
(c) 1 and 3 only 64. Consider the following statements:
(d) 1, 2 and 3
1. Vermicompost is artificial rearing of worms to
61. Which of the following pairs are correctly decompose organic food wastes into a nutrient-
matched? rich material.
List I List II 2. Pyrolysis is the process of combustion in the
presence of oxygen for disposal of mostly
Air pollutants Sources inorganic wastes.
1. Sulphur oxides Swamps and springs Which of the statements given above is/are correct?
(SOX) (a) 1 only
2. Carbon monoxide Incomplete (b) 2 only
(CO) combustion of fuel (c) Both 1 and 2
such as natural gas, (d) Neither 1 nor 2
coal or wood
65. Which among the following methods/techniques
3. Nitrogen oxides Livestock and poultry are used to manage nuclear waste?
(NOx) farm waste
1. Underground concrete vaults
4. Particulate Matter It originates from 2. Incinerated and disposed near surface facilities.
(PM) volcanoes, dust storms,
forest and grassland 3. Burial in deep land mines after incorporating the
fires nuclear waste into glass.
Select the correct answer using the code given below: Select the correct answer using the code given below:
(a) 1 and 2 only (a) 1 and 3 only
(b) 2 and 3 only (b) 2 and 3 only
(c) 2 and 4 only (c) 3 only
(d) 1, 2, 3 and 4 (d) 1, 2 and 3

Workbook 234
.
ENVIRONMENT

66. With reference to plastic and its disposal, consider


the following statements: 4.3. Environmental Issues
1. Plastic disposal poses environmental problem as 70. Which of the following statements is/are correct
they are chemically inert. about impact of methane in the atmosphere?
2. Oxo-degradable plastics does not fragment into 1. With rise in methane concentration, there
microplastics. is an increase in tropospheric ozone in the
3. Bioplastics can be manufactured from lactic atmosphere.
acid. 2. Methane has a shorter lifetime but a stronger
warming potential than carbon dioxide.
Which of the statements given above are correct?
Select the correct answer using the code given below:
(a) 1 and 2 only
(a) 1 only
(b) 2 and 3 only
(b) 2 only
(c) 1 and 3 only (c) Both 1 and 2
(d) 1, 2 and 3 (d) Neither 1 nor 2
67. Lead, ingested or inhaled, is a health hazard. 71. Which of the following reason(s) attribute to
After the addition of lead to petrol has been the predominance of ozone depletion over the
banned, what still are the sources of lead Antarctica?
poisoning? 1. Low temperature present for extended time
1. Smelting units periods
2. Pens and pencils 2. Reaction of gases with Polar Stratospheric
Clouds (PSCs)
3. Paints
3. Presence of Polar Vortex at the South pole
4. Hair oils and cosmetics
Select the correct answer using the code given below:
Select the correct answer using the codes given (a) 1 and 3 only
below: (b) 2 and 3 only
(a) 1, 2 and 3 only (c) 3 only
(b) 1 and 3 only (d) 1, 2 and 3
(c) 2 and 4 only
72. Which of the following statement is/are incorrect
(d) 1, 2, 3 and 4 about acid rain?
68. Consider the following statements with reference 1. Its impact on food crops is minimized by the use
to the Air pollution: of fertilizers.
1. Majority of deaths in India due to air pollution 2. Most of the acid particles in the atmosphere falls
occurred from ambient particulate matter back to earth as wet deposition.
pollution and household air pollution. 3. They make some conifers less cold tolerant and
2. Though Aerosols are a part of air pollution they even lead to their death.
are not dangerous to human health. 4. Indian soils are heavily impacted by acid rain
due to its tropical location.
Which of the statements given above is/are correct?
Select the correct answer using the code given below:
(a) 1 only
(a) 1 and 2 only
(b) 2 only
(b) 1 and 3 only
(c) Both 1 and 2 (c) 2 and 3 only
(d) Neither 1 nor 2 (d) 2 and 4 only
69. It is a colorless gas with a pungent odor. It is a 73. With reference to the Chlorofluorocarbons
liquid when under pressure, and it dissolves in (CFCs), consider the following statements
water very easily. It comes into the air mainly 1. CFCs are a class of highly reactive and flammable
from activities such as the burning of coal and gases.
oil at power plants or from copper smelting. In 2. They are mainly used as refrigerants and
nature, it can be released to the air from volcanic propellants in aerosol sprays.
eruptions. It forms acid rain when released into 3. The Montreal Protocol regulates the production
the atmosphere with nitrogen oxides. and consumption of CFCs due to their role in
The above passage describes the explanation of ozone depletion.
which of the following gas? Which of the statements given above is/are correct?
(a) Carbon monoxide (a) 1 only
(b) Nitrogen dioxide (b) 2 and 3 only
(c) sulfur dioxide (c) 1 and 2 only
(d) carbon dioxide (d) 1, 2 and 3

235 Workbook
.
ENVIRONMENT

74. Consider the following pairs Which of the statement(s) given above is/are
incorrect?
Type of Carbon Description (a) 1 only
1. Black carbon Emitted due to (b) 1 and 2 only
incomplete combustion (c) 2 and 3 only
of fossil fuels (d) 1, 2 and 3
2. Brown carbon Carbon released due to 78. With reference to the various ‘environmental
combustion of organic clean-up techniques’, consider the following pairs:
matter
Technique Application
3. Blue carbon Carbon captured by
oceans and other 1. Rhizofiltration Plants are used to
coastal ecosystems absorb, concentrate,
and precipitate
Which of the pairs given above are correct?
contaminants from
(a) 1 and 2 only polluted aqueous
(b) 2 and 3 only sources in their roots.
(c) 1 and 3 only
(d) 1, 2 and 3 2. Phytodegradation Contaminants are
taken up into the plant
75. Which of the following can be the environmental tissues where they
impacts of extensive palm oil production in the are metabolized, or
world? biotransformed.
1. Loss of orangutan habitats in Indonesia
3. Mycofiltration A form of
2. Loss of biodiversity bioremediation in
3. Encroachment on peat lands which a combination
4. Increase in greenhouse gas emissions of bacteria is used to
Select the correct answer using the code given below: decontaminate the
(a) 1 and 2 only area.
(b) 2 and 3 only Which of the pairs given above is/are correctly
(c) 2, 3 and 4 only matched?
(d) 1, 2, 3 and 4 (a) 2 only
(b) 3 only
4.4. EIA (c) 1 and 2 only
(d) 1 and 3 only
76. With reference to the Environmental Impact
Assessment (EIA), consider the following: 79. Which among the following are consequences or
effects of the acid rain?
1. It is a process of evaluating the likely
environmental impacts of a proposed project 1. It alters the composition of the soil.
or development, taking into both beneficial and 2. It causes respiratory issues in animals and
adverse impacts. humans.
2. It is statutorily backed by the Environment 3. It causes damage to buildings and monuments.
Protection Act, 1986. 4. It is responsible for declining fish populations in
3. The first step in the EIA process is scoping. some areas.
Which of the statements given above is/are correct? Select the correct answer using the code given below:
(a) 1 and 2 only (a) 1 and 3 only
(b) 2 and 3 only (b) 1, 2 and 3 only
(c) 1 and 3 only (c) 3 only
(d) 1, 2 and 3 (d) 1, 2, 3 and 4

77. With reference to Environmental Impact 80. Which of the followings are the impacts of
Assessment (EIA) Notification, 2006, consider the pollution on plants?
following statements: 1. Reduced yield of crops due to air pollution.
1. It was promulgated under the provisions of the 2. Affect their leaf metabolism.
Environment (Protection) Act, 1986. 3. Ozone damages plants by obstructing stomata.
2. Public hearing is mandatory for the preparation 4. Light pollution will impact pollination and
of EIA Reports for all development projects. subsequently food production
3. State Forest Departments prepare the Select the correct answer using the codes given
Environment Impact Assessment Reports. below:

Workbook 236
.
ENVIRONMENT

(a) 1, 2 and 3 only Select the correct answer using the code given below:
(b) 1, 2 and 4 only (a) 1 only
(c) 1, 3 and 4 only (b) 1 and 2 only
(d) 1, 2, 3 and 4
(c) 2 and 3 only
81. Consider the following statements: (d) 1, 2 and 3
1. Bioaccumulation is the process by which a
pollutant from the environment enters a food 85. With reference to the Ocean Gyres and garbage,
chain. consider the following statements:
2. For biomagnification to occur, the pollutant 1. The Great Pacific Garbage patch is a collection of
must be short-lived and insoluble in fats. marine debris mainly found in the North Pacific
3. Triclosan, an anti-fungal agent is a Ocean.
bioaccumulant. 2. The area in the centre of a gyre tends to be very
Which of the statements given above is/are correct? calm which traps the debris from surrounding.
(a) 1 only Which of the statements given above is/are correct?
(b) 2 and 3 only
(a) 1 only
(c) 1 and 3 only
(d) 1, 2 and 3 (b) 2 only
(c) Both 1 and 2
82. Consider the following statements with reference (d) Neither 1 nor 2
to the secondary pollutants:
1. Intense sunlight helps primary pollutants create 86. Consider the following statements with reference
secondary pollutants. to the Pet coke:
2. Ozone is a reactive oxidant gas produced 1. Pet coke contains more sulfur than coal.
naturally in Stratosphere. 2. India is the largest consumer of the pet coke in
Which of the statements given above is/are correct? the world.
(a) 1 only 3. In India, import of Pet coke is allowed for
(b) 2 only only cement, lime kiln, calcium carbide and
(c) Both 1 and 2 gasification industries.
(d) Neither 1 nor 2
Which of the statements given above is/are correct?
83. Which of the following pairs is/are correctly
(a) 1 and 2 only
matched?
(b) 2 and 3 only
List I List II (c) 1 and 3 only
Pollutant Sources of Indoor (d) 1, 2 and 3
pollution
87. Consider the following pairs:
1. Asbestos Building
construction List I List II
materials Technology Description

2. Carbon monoxide Unvented kerosene and 1. Land farming It is ex-situ


gas space heaters bioremediation
technology.
3. Radon Paint, gasoline and
water pipes 2. Bioreactors It is in-situ method
Select the correct answer using the code given below: which uses indigenous
microorganisms to
(a) 1 and 2 only
biodegrade.
(b) 2 and 3 only
(c) 1 and 3 only 3. Biosparging It can be used to reduce
(d) 1, 2 and 3 concentrations of
petroleum constituents
84. If all the trees in the world disappear, then which that are dissolved in
of the following is least likely to happen? groundwater
1. Formerly forested areas would become drier and
become more prone to extreme droughts. Which of the pairs given above is/are correct?
2. Most of the places would immediately become (a) 1 and 2 only
warmer. (b) 1 only
3. Previously forested ecosystems would become a (c) 3 only
source of carbon emission rather than a sink. (d) 1 and 3 only

237 Workbook
.
ENVIRONMENT

1. They can accumulate in the environment and


4.5. Health impact of pollution contaminate water and soil.
88. Consider the following pairs: 2. They can enter the food chains.
3. They can trigger the production of free radicals.
Pollutant Health Impact
Select the correct answer using the code given below.
1. Cadmium Heart diseases (a) 1 and 2 only
2. Lead Methemoglobinemia (b) 3 only
3. Mercury Minamata disease (c) 1 and 3 only
(d) 1, 2 and 3
Which of the pairs given above is/are correctly
matched? 93. In the questions given below, there are two
(a) 1 and 2 only statements marked as Assertion (A) and Reason
(b) 1 and 3 only (R). Read the statements and choose the correct
(c) 3 only option.
(d) 1, 2 and 3 Assertion (A): Landfill ‘capping’ involves covering
the waste in the landfill with soil.
89. With reference to health effects of various
pollutants, consider the following pairs: Reason (R): Landfill ‘capping’ has been mandated by
the Solid Waste Management Rules, 2016.
Pollutants Health Effects (a) Both (A) and (R) are true and (R) is the correct
1. Sulphur oxides eye and throat irritation explanation of (A).
(b) Both (A) and (R) are true and (R) is not the
2. Carbon monoxide difficulty in breathing
correct explanation of (A).
3. Lead damage to brain and (c) (A) is correct but (R) is wrong.
central nervous system (d) (A) is wrong but (R) is correct.
Select the correct answer using the code given below:
(a) 1 and 2 only 4.6. Renewable energy and energy efficiency
(b) 2 and 3 only
94. Which of the following statements is correct with
(c) 1 and 3 only
respect to green energy initiative in India?
(d) 1, 2 and 3
(a) India has set a target of 40% for bioethanol
90. Which of the following is not an effect of water blending of petrol by 2025.
pollution on ecosystems and human health? (b) 3rd Generation bio-fuel is produced by utilizing
(a) Polluted water increases the dissolved carbon surplus biomass and agriculture waste products.
dioxide content which leads to the elimination (c) India has already met its 2022 renewable energy
of sensitive organisms like planktons. target for biomass-based energy.
(b) Polluted water is a source of water borne diseases (d) Government has set a target of 60 GW by
like cholera and jaundice. Rooftop Solar Plants to be achieved by 2022.
(c) One of the causes behind the Minamata disease
95. Which of the following processes can be used to
was contaminated water.
exploit/ extract biomass-based energy?
(d) It leads to eutrophication of water bodies.
1. Bagasse based cogeneration in sugar mills
91. Which of the following statement is incorrect 2. Anaerobic Digestion
about uranium pollution in ground water? 3. Gasification
(a) Primary source of uranium in ground water is 4. Pyrolysis
geogenic.
(b) Excessive ground water extraction leads to Select the correct answer using the code given below.
uranium enrichment in aquifers. (a) 1 and 2 only
(c) World Health Organization has set a limit (b) 2 and 4 only
for safe drinking water at 100 micrograms of (c) 1, 3 and 4 only
uranium per liter. (d) 1, 2, 3 and 4
(d) Uranium contamination causes chronic kidney 96. Consider the following statements about the
diseases. compressed biogas:
92. There is some concern regarding the nanoparticles 1. It is a purified form of biogas that can be
of some chemical elements that are used by the produced from municipal solid waste.
industry in the manufacture of various products. 2. It is compressed methane and has the same
Why? calorific value as CNG.

Workbook 238
.
ENVIRONMENT

3. It lacks commercial viability as it is difficult to (d) A distributed energy system such as a rooftop
transport it through pipelines to retail outlets. solar plant does not qualify as a microgrid.
4. SATAT initiative was launched by the
government to promote the compressed biogas 98. With respect to the policy of net metering which of
technology. the following statements is/are correct?
Which of the statements given above is/are correct? 1. This is a billing mechanism used for crediting
(a) 1 and 3 only surplus solar electricity generated by user back
(b) 2 and 4 only to the grid.
(c) 1, 2 and 4 only 2. This policy is not applicable for an off-grid
(d) 1, 2, 3 and 4 system.
97. With respect to microgrid which of the following 3. There is no need for battery storage and backup
statement is incorrect? generators.
(a) It is a self-sufficient energy system that serves a Select the correct answer using the code given below:
discrete geographical area.
(b) Due to lower servicing area, it has high aggregate (a) 1 and 2 only
technical and commercial losses (AT&C). (b) 1 and 3 only
(c) It can be disconnected from central grid and still (c) 2 and 3 only
operate autonomously. (d) 1, 2 and 3

239 Workbook
.
ENVIRONMENT

SOLUTIONS

Producer gas can be used to run internal combustion


4.1. Previous Years Questions engines (both compression and spark ignition), can be
1. Solution (d) used as substitute for furnace oil in direct heat applications
and can be used to produce, in an economically viable
Exp) Option d is correct way, methanol—an extremely attractive chemical which
An algal bloom or marine bloom or water bloom is a rapid is useful both as fuel for heat engines as well as chemical
increase in the population of algae in an aquatic system. feedstock for industries
Algal blooms may occur in freshwater as well as marine 7. Solution (c)
environments.
Exp) Option c is correct.
Usually algal blooms are the result of an excess of nutrients
Molasses is the dark, sweet, syrupy bi-product made
(particularly phosphorus and nitrogen) into waters and
during the extraction of sugars from sugarcane and sugar
higher concentrations of these nutrients in water cause beets. It is a common sweetener and flavoring in many
increased growth of algae and green plants. baked goods and sauces. It is not used for production of
2. Solution (d) synthetic chemical fertilizers.
Exp) Option d is correct 8. Solution (c)
The main emissions from coal combustion at thermal Exp) Option c is correct
power plants are carbon dioxide (CO2), nitrogen oxides, Studies by the Central Water Commission show that
sulfur oxides, chlorofluorocarbons (CFCs), and airborne rivers (surface drinking water sources) in India are loaded
inorganic particles such as fly ash and soot. with toxic heavy metals such as lead, arsenic, copper,
cadmium, mercury and nickel. In India, drinking water
3. Solution (a) especially in wells is highly contaminated with Uranium.
Exp) Option a is correct Sobitol and Formaldehyde are not drinking water
Oilzapper feeds on hydrocarbon compounds present in pollutants.
crude oil and the hazardous hydrocarbon waste generated Sorbitol is a type of carbohydrate called a sugar alcohol.
by oil refineries, known as Oil Sludge and converts them It contains about one-third fewer calories than sugar and
into harmless CO2 and water. is 60 percent as sweet and occurs naturally in a variety of
4. Solution (b) berries and fruits.
Exp) Option b is correct Formaldehyde is a simple chemical compound made of
hydrogen, oxygen and carbon. All life forms – bacteria,
Important sources of lead poisoning include mining, plants, fish, animals and humans – naturally produce
smelting, manufacturing and recycling activities, and, in formaldehyde as part of cell metabolism.
some countries, the continued use of leaded paint, leaded
gasoline, and leaded aviation fuel. 9. Solution (b)
5. Solution (b) Exp) Option b is correct
Exp) Option b is correct Plutonium is a radioactive chemical element which is not
used in manufacturing of Computers. Heptachlor is an
Vultures are on the verge of extinction in India because a organochlorine compound that is used as an insecticide,
banned drug being used illegally to treat suffering cattle.
not computers.
Diclofenac, an anti-inflammatory drug used by farmers to
ease pain in cattle, is deadly to vultures. 10. Solution (d)
The endangered birds eat the remains of the drugged Exp) Option d is correct
animals and suffer kidney failure and visceral gout, which
is usually fatal. Nanomaterials are able to cross biological membranes and
access cells, tissues and organs that larger-sized particles
6. Solution (a) normally cannot. All of the above are concerns regarding
nanoparticles.
Exp) Option a is correct
Biomass gasification is a mature technology pathway 11. Solution (c)
that uses a controlled process involving heat, steam, and Exp) Option c is correct
oxygen to convert biomass to produce carbon monoxide Brominated flame retardants (BFRs) are mixtures of
(CO), hydrogen (H2) and traces of methane (CH4), man-made chemicals that are added to a wide variety
without combustion. This mixture is called producer gas. of products to make them less flammable. They are used

Workbook 240
.
ENVIRONMENT

commonly in plastics, textiles and electrical/electronic Burning of these residues emit gases like sulphur dioxide
equipment. (SO2), oxides of nitrogen (NOx), carbon dioxide (CO2),
Some BFRs are highly resistant to degradation in the carbon monoxide (CO), black carbon (BC), organic
environment and can bioaccumulate. carbon (OC), methane (CH4), volatile organic compounds
(VOC), non-methane hydrocarbons (NMHCs), ozone
12. Solution (d) (O3), and aerosols etc.
Exp) Option d is correct
18. Solution (d)
It prevents insects to eat plants and affects their brain and
hormonal system to work efficiently, leading to death. Exp) Option d is correct
The kernels of neem owing to its antibacterial properties Pyrolysis is the thermal decomposition of materials at
is used in pharmaceutical industry. Neem seeds have not high temperatures in absence of oxygen. It is commonly
proven as a source to manufacture biofuels. used in treatment of organic materials. Char is usually end
product of this process.
13. Solution (b)
Plasma gasification is an extreme thermal process using
Exp) Option b is correct plasma which converts organic matter into a syngas
Production of algae based biofuel is possible both in seas (synthesis gas) which is primarily made up of hydrogen
and on continent. and carbon monoxide. It is used for various gasification
Lack of technology and financial resources are the major processes.
limitations of developing countries in promoting biofuels.
19. Solution (b)
14. Solution (c) Exp) Option b is correct
Exp) Option c is correct H-CNG reduces emission of carbon monoxide emission
Bioremediation is a process that uses mainly upto 70%, but it does not eliminate carbon monoxide
microorganisms, plants, or microbial or plant enzymes to emissions.
detoxify contaminants in the soil and other environments. H-CNG is more expensive than traditional CNG.
Heavy metals such as cadmium and lead are not readily
absorbed or captured by microorganisms. 20. Solution (d)
Some highly chlorinated contaminants and high Exp) Option d is correct
molecular weight Polycyclic aromatic hydrocarbons Methane hydrate is an “ice” that only occurs naturally
(PAHs) present in coal, crude oil and gasoline are also in subsurface deposits where temperature and pressure
not readily amenable to microbial degradation. There are conditions are favourable for its formation. Owing to
also increasing concerns that the bioremediation products melting of ice, global warming might trigger the release of
may be more persistent or hazardous than the parent methane gas from these deposits.
compounds.
21. Solution (a)
15. Solution (b) Exp) Option a is correct
Exp) Option b is correct According to India’s National Policy on Biofuels the
There is no direct link between salinity and heavy sand following can be used as raw materials for the production
mining. of biofuels:
However, depletion of sand in the streambed causes the sugar cane, sugar beet, sweet sorghum, corn, cassava,
deepening of rivers and estuaries, and the enlargement of algae, bagasse, wood waste, agricultural and forestry
river mouths and coastal inlets. This can lead to saline- residues, edible and non-edible vegetable oils, and animal
water intrusion into the river, thus increasing its salinity. fat of diesel quality.
This threat is imminent in coastal areas.
22. Solution (d)
16. Solution (a)
Exp) Option d is correct
Exp) Option a is correct
Statement 1 is correct. Typically, coal ash consists of
Microbeads are plastic particles of less than one millimetre arsenic, lead, mercury, selenium, hexavalent chromium
in their largest dimension. Microbeads typically serve as among other carcinogens and neurotoxins.
cleansers and exfoliants in personal care products, such as
soaps, facial scrubs and toothpastes. They are considered Coal ash, also referred to as coal combustion residuals or
harmful to marine life. CCRs, is produced primarily from the burning of coal in
coal-fired power plants.
17. Solution (d) Statement 2 is correct. Coal-fired power plants also
Exp) Option d is correct produce large amounts of nitrogen oxides and sulfur
Burning of crop residue is a serious problem, especially dioxide—the pollutants that cause acid rain.
in the northern part of India causing widespread air Statement 3 is correct. India’s domestic coal reserves
pollution in the National Capital Region. have a high ash content—up to 40 to 45 percent.

241 Workbook
.
ENVIRONMENT

23. Solution (a) Diclofenac, an anti-inflammatory drug used by farmers to


Exp) Option a is correct. ease pain in cattle, is deadly to vultures.
Benzene is a colourless liquid with a sweet Odour. It The endangered birds eat the remains of the drugged
evaporates into the air very quickly and dissolves slightly animals and suffer kidney failure and visceral gout, which
in water. It is highly flammable. Human exposure to is usually fatal.
benzene has been associated with a range of acute and 27. Solution: (d)
long-term adverse health effects and diseases, including
cancer and aplastic anaemia. Exp) Option d is correct
Statement 1 is correct. Automobile exhaust accounts for An algal bloom or marine bloom or water bloom is a rapid
the largest source of benzene in the general environment. increase in the population of algae in an aquatic system.
Statement 2 is correct. Active and passive exposure to Algal blooms may occur in freshwater as well as marine
tobacco smoke is also a significant source of exposure. environments.
Benzene is highly volatile, and exposure occurs mostly Usually algal blooms are the result of an excess of nutrients
through inhalation. (particularly phosphorus and nitrogen) into waters and
Statement 3 is correct. Benzene is one of the main higher concentrations of these nutrients in water cause
organic compounds emitted by wood-burning, that has increased growth of algae and green plants.
the potential to cause cancer. 28. Solution (c)
Statement 4 and 5 are incorrect. Exp) Option c is correct.
24. Solution: (b) Option a is incorrect. Diclofenac is the drug responsible
Exp) Option b is the correct answer. for the massive deaths of vultures in India in the past.
The drug was once commonly used as a livestock anti-
Methane and nitrous oxide are important greenhouse inflammatory drug. The vultures would die due to feeding
gases. They contribute to global warming. Important on the dead cattle carcasses. Veterinary usage of diclofenac
anthropogenic sources of biogenic methane are wet rice has been banned in India since 2006.
fields, cattle, animal waste, landfills and biomass burning.
Option b is incorrect. They are toxic chemicals but
Flooded rice cultivation has been identified as one of not used in the production of plastics but used as
the leading global agricultural sources of anthropogenic insecticides and pesticides. Hence, this is not the correct
methane (CH4) emissions. Furthermore, it has been reason for their ban.
estimated that global rice production is responsible for
11% of total anthropogenic CH4 emissions. Option c is correct. They are mentioned under Stockholm
Convention on Persistent Organic Pollutants. Since
Paddies are a potential source of anthropogenic nitrous India is a signatory to the above mentioned convention,
oxide (N2O) emission as well. In paddies, both the soil it has to ratify any amendments to the convention and
and the rice plants emit N2O into the atmosphere. The hence ban the chemicals which are banned under the
rice plant in the paddy is considered to act as a channel convention.
between the soil and the atmosphere for N2O emission.
Option d is incorrect. These chemicals have nothing to
25. Solution: (b) do with marine pollution and hence this statement is
Exp) Option b is the correct answer. incorrect.
The pollutants, Sulphur dioxide, nitrogen oxide which 29. Solution (d)
are released by burning of coal and petroleum products
Exp) Option d is correct.
combine with moisture in ‘the air and rain water and
produce Sulphuric acid and nitric acid respectively and Statement 1 is correct. According to Greenpeace India
fall along with the rain called as acid rain. The acid rain is the leading emitter of anthropogenic sulphur dioxide
pollute soil, water and reduces the growth of crops and in 2019 followed by Russia and China.
fishes in river, streams, canals and ponds. It also stops Statement 2 is incorrect. The major causes of SO2
the growth of plants and destroys: the buildings made of emissions in India are from fossil fuel combustion at
marble and stones and statues made up of metals, Acid power plants. Other sources are fuel combustion in mobile
rain can cause respiratory problems such as asthma, sources such as locomotives, ships, and other equipment.
bronchitis and emphysema. Statement 3 is correct. An electrostatic precipitator is
used for removing sulfur dioxide and other polluting
4.2. Pollution and Major pollutants particles, through a wet liquid removal area or
compartment downstream and in the last section of plates
26. Solution: (b)
of the electrostatic precipitator. Sulfur dioxide in flue gas
Exp) Option b is correct from fossil-fuel power plants can be controlled by means
Vultures are on the verge of extinction in India because a of an absorption process called Flue Gas Desulfurization
banned drug being used illegally to treat suffering cattle. (FGD) too.

Workbook 242
.
ENVIRONMENT

30. Solution (c) Further with the latest technology, fly ash can be used for
Exp) Option c is correct. making geopolymer which acts as a cement-like product.
NTPC has successfully developed Fly Ash based Geo-
Statement 1 is incorrect. Ozone is a gas found in different Polymer Road Construction technique.
parts of the atmosphere. Ozone in the upper atmosphere,
or stratosphere, called Stratospheric Ozone helps protect 33. Solution (c)
the Earth from the sun’s harmful ultraviolet rays. Exp) Option c is correct.
At the lowest level of the atmosphere, the troposphere, Lead causes damages to the central and peripheral
exposure to ozone (tropospheric ozone) can be nervous system, blood systems, kidney and reproductive
harmful to both human health and some plants. It systems in humans. It also affects the endocrine systems
is a highly reactive gas; even short-term exposure to and impedes the brain development among children.
which is dangerous for those with respiratory conditions Zinc -Mass poisoning has been reported following
and asthma. Ozone in the troposphere is considered a the drinking of acidic beverages kept in galvanized
greenhouse gas, and may contribute to global warming. containers; fever, nausea, vomiting, stomach cramps,
Statement 2 is correct. Tropospheric Ozone i.e. Ozone and diarrhoea occurred 3–12 h after ingestion. Food
poisoning attributable to the use of galvanized zinc
in the lower atmosphere is formed by photochemical
containers in food preparation has also been reported.
reactions between oxides of nitrogen (NOx) and other
volatile organic compounds (VOCs) and gases in the air Arsenic- Arsenic can enter the water supply from natural
under the influence of sunlight and heat. deposits in the earth or from industrial and agricultural
pollution. It is widely believed that naturally occurring
Statement 3 is correct. Ozone in the lower atmosphere arsenic dissolves out of certain rock formations when
(troposphere) is primarily a “sunny weather problem”. ground water levels drop significantly. Accumulation
Ozone levels are typically highest during the afternoon of arsenic in the body parts like blood, nails and hairs
hours of the summer months, when the influence of causing skin lesions, rough skin, dry and thickening of
direct sunlight is the greatest. These high levels occur skin and ultimately skin cancer.
during what is known as the “ozone season,” which
Cadmium- Cadmium poisoning causes Itai – Itai
includes the summer months.
disease i.e. severe pain felt in the spine and joints. It
31. Solution (a) can also cause bones to become weaker. Large amounts of
Exp) Option a is correct. cadmium can damage the kidney, liver and heart and in
severe cases may cause death.
Carbofuran, Phorate and Triazophos are pesticides
used in agriculture. Carbofuran is a pesticide that is 34. Solution (c)
widely used to control insects and nematodes on a variety Exp) option c is correct.
of agricultural crops. Phorate is an organophosphate used Nitrogen Oxides are a family of poisonous, highly reactive
as an insecticide, pesticide and acaricide. Triazophos gases.
belongs to the class of organophosphates pesticides and Nitrogen oxides are produced in combustion processes,
are widely used as insecticides and acaricides. partly from nitrogen compounds in the fuel, but mostly
32. Solution (d) by direct combination of atmospheric oxygen and
nitrogen in flames. Nitrogen oxides are produced
Exp) Option d is correct. naturally by lightning, and also, to a small extent, by
All the given statements are correct: Fly Ash is a microbial processes in soils.
byproduct of coal burning in a thermal power plant. Fly The primary sources nitrogen dioxide in indoors are
ash contains aluminous and siliceous material that forms combustion processes, such as:
cement in the presence of water. When it is mixed with unvented combustion appliances, e.g. gas stoves
lime and water, it forms a compound similar to Portland
vented appliances with defective installations
cement. This makes fly ash suitable as a prime material
in blended cement, ceramic tiles, bricks, hollow blocks, welding
road construction, filling of mine voids etc. tobacco smoke
Fly ash are used for engineering applications including kerosene heaters
portland cement concrete (PCC), soil and road base
stabilization, flowable fills, grouts, structural fill and Thermal power plants, Cars, trucks, stoves, gardening
asphalt filler. equipment etc. uses fuel which emits the NO2.
Fly ash has a great potential for capture, utilization, and Hydro power plants do not use fuel as a source, so they
storage of CO2 (CCUS) applications. do not emit NO2.
Fly ash-based adsorbents have comparable performances
with commercial adsorbents when applied for CO2 35. Solution (d)
capture. Exp) Option d is correct.

243 Workbook
.
ENVIRONMENT

Zero liquid discharge (ZLD) is an engineering approach Exp) Option d is correct.


to water treatment where all water is recovered and E-Waste is a term used to cover items of all types of
contaminants are reduced to solid waste. electrical and electronic equipment (EEE) and its parts
The treatment of wastewater from water-intensive that have been discarded by the owner as waste without
industries so as to recover water and minerals for reuse the intention of re-use.
followed by treatment of non-recoverable waste to make Electronic waste is considered the fastest growing waste
it chemically inert before being filled in landfills is called stream in the developed world. The waste is the result of
Zero Liquid Discharge policy. advancing technology that leaves behind old computers,
36. Solution (d) laptops, televisions, and other electronic devices that
require specialized methods of recycling because of
Exp) Option d is correct. toxic by-products. A major concern regarding end-of-life
Statement 1 is correct. Incineration is a controlled products (EOL) is environmental contamination and
combustion process with the primary objective of volume seepage into the food chain.
reduction and energy recovery from the waste stream. List of some common e-waste items:
In this process, heat produced from combustion can be
recovered and converted to electric power. Thus, could be 1. Electric cookers
used for waste to energy generation. 2. Heaters Laptops
Statement 2 is correct. Torrefaction is a thermochemical 3. Circuit boards
process typically at 200-350 °C in the absence of oxygen, 4. Hard Drives
at atmospheric pressure. The process causes biomass to 5. Solar Panels
partly decompose, creating torrefied biomass or char, also 6. Power Distribution Systems (PDU’s)
referred to as ‘biocoal’. Biocoal has high calorific value. 7. Autoclave
Thus, could be used for waste to energy generation. 8. Defibrillator
Statement 3 is correct. Hydrothermal Carbonisation 9. Smart Watches
is a new waste to energy technology that accelerates 10. Heart Monitors
the process of geothermal conversion of wet waste with
an acid catalyst at high pressure and heat. It results in 11. Diabetic Testing Equipment
production of hydro-char that has properties similar 39. Solution (d)
to fossil fuels. Thus, could be used for waste to energy
Exp) Option d is correct.
generation.
Option 1 is correct. Oil spills in water can be cleaned
Statement 4 is correct. Biomethanation is a process by
with the help of bregoli – a by-product of paper industry
which organic material is microbiologically converted to
resembling saw dust, oil zapper, micro-organisms.
biogas under anaerobic conditions. Biogas is then used
for energy generation. Thus, could be used for waste to Oil spills from tankers at sea or leaks from underground
energy generation. storage tanks on land are very difficult to control as oil
tends to spread very fast, affecting a large area in a very
37. Solution (b) short time. It is one of the most dangerous of all water
Exp) Option b is correct pollutants.
Option a is incorrect: Bioreactor: It is an ex-situ Option 2 is correct. Floating barriers, called
technique of bioremediation which involves the containment Booms, are helpful in restricting the spread
of oil and to allow for its recovery, removal, or dispersal.
processing of contaminated material through an
engineered containment system. Option 3 is correct. Skimmers are devices used for
physically separating spilled oil from the water’s
Option b is correct: Biosparging: An in-situ technique surface.
of bioremediation in which air is injected under pressure
below the water table to increase ground water oxygen and 40. Solution (d)
enhance the rate of natural degradation of contaminants Exp) Option d is correct.
by naturally occurring bacteria.
Statement 1 is correct. A primary pollutant is an air
Option c is incorrect: pollutant emitted directly from a source. Example – DDT,
Bioaugmentation: Microorganisms are imported to a plastic, etc.
contaminated site to enhance degradation process. Statement 2 is correct. A secondary pollutant is not
Option d is incorrect: Bioventing: Under this air and directly emitted as such, but forms when other pollutants
nutrients are supplied through well of contaminated soil (primary pollutants) react in the atmosphere. Examples
to stimulate the growth of indigenous bacteria. It can be of a secondary pollutant include NO2, which is formed as
used where contamination is deep under the surface. NO combines with oxygen in the air; acid rain, which is
formed when sulfur dioxide or nitrogen oxides react with
38. Solution (d) water, etc.

Workbook 244
.
ENVIRONMENT

Statement 3 is correct. Ozone is an example of secondary Statement 3 is incorrect. Low wind speed is necessary
pollutant. It is formed when hydrocarbons (HC) and for the formation of smog. Smog is mostly formed
nitrogen oxides (NOx) combine in the presence of during winters because the wind speed is low which
sunlight. leads to stagnation of fog and smoke near the ground.
Cities located in basins surrounded by mountains may
41. Solution (b) have smog problems because the smog is trapped in the
Exp) Option b is correct. valley and cannot be carried away by wind. Los Angeles,
Statement 1 is incorrect. Ground-level ozone (O3) is California, and Mexico City, Mexico, both have high smog
not emitted directly from anthropogenic sources. It levels partly because of this kind of landscape.
is a “secondary” pollutant formed by a complicated 44. Solution (d)
series of chemical reactions in the presence of sunlight.
Photochemical reactions of NOx and VOCs (originating Exp) Option d is correct.
from largely from combustion processes) govern the Fly ash or flue ash is a coal /solid material combustion
concentration of ground-level O3 in the atmosphere. product that is composed of the particulates (fine
Statement 2 is correct. Ozone is the main ingredient in particles of burned fuel) that are driven out of coal-fired
photochemical smog. Photochemical smog is a brownish- boilers together with the flue gases.
grey haze caused by the action of solar ultraviolet radiation Statement 1 is correct. Depending upon the source and
on atmosphere polluted with hydrocarbons and oxides composition of the coal being burned, the components of
of nitrogen. It contains anthropogenic air pollutants, fly ash vary considerably, but all fly ash includes substantial
mainly ozone, nitric acid, and organic compounds, amounts of silicon dioxide (SiO2), aluminium silicate,
which are trapped near the ground by temperature calcium oxide (CaO) and certain heavy metals like lead,
inversion. copper.
Statement 3 is correct. Ozone at ground level is a harmful Statement 2 is correct. Cement can be replaced by fly
air pollutant, because of its effects on people and the ash up to 35%, thus reducing the cost of construction,
environment. But only ground level ozone is considered making roads etc.
a pollutant. Atmospheric ozone, also called good ozone
occurs naturally and forms the part of the ozone layer is 45. Solution (a)
not considered a pollutant. Exp) Option a is correct.
Statement 1 is correct. Use of fly ash earlier allowed
42. Solution (d)
within 100 kms radius of power plant, now has
Exp) Option d is correct. been extended to 300 kms. The policy will create new
Statement 1 is incorrect. PM2.5 is not just produced employment opportunities in the power plant areas and
outside but is also produced by some common indoor also make available raw material for construction at low
activities. Some indoor sources of fine particles are tobacco cost to help ‘Housing for All’ projects.
smoke, cooking (e.g., frying, sautéing, and broiling), Statement 2 is incorrect. To facilitate 100% ash
burning candles or oil lamps, and operating fireplaces and utilization by all coal based thermal power plants, a web
fuel-burning space heaters (e.g., kerosene heaters) portal for monitoring of fly ash generation and utilization
Statement 2 is incorrect. PM2.5 occurs both naturally data of Thermal Power Plants and a mobile based
and due to man-made activities. Anthropogenic application titled “ASHTRACK” has been launched by
activities releasing PM2.5 include construction sites, the Ministry of Power that will help to establish a link
unpaved roads, fields, smokestacks or fires. Naturally they between fly ash users and power plants executives for
are released during volcanic eruptions and forest fires. obtaining fly ash for its use in various areas.
Statement 3 is incorrect. At present, 63% of the fly ash is
43. Solution (a) being utilized in India. Target is for 100% utilisation of
Exp) Option a is correct. the fly ash.
Statement 1 is correct. Smog is short for Smoke Fog. It 46. Solution (d)
is caused by the burning of large amounts of fossil fuels
like coal, vehicular emission and industrial fumes. Smog Exp) Option d is correct.
contains soot particulates like smoke, sulphur dioxide, Statement 1 is correct. They can enter food chain and
nitrogen dioxide and other components. remain there as PoP (Persistent organic pollutants) or
Statement 2 is correct. Photochemical smog is produced inorganic pollutants and contaminate the food chain due
when sunlight reacts with nitrogen oxides and at least one to bioaccumulation.
volatile organic compound (VOC) in the atmosphere. Statement 2 is correct. After releasing in the environment,
Nitrogen oxides come from car exhaust, coal power plants, nanoparticles will accumulate in various environmental
and factory emissions. VOCs are released from gasoline, matrices such as air, water, soil and sediments including
paints, and many cleaning solvents. When sunlight hits wastewater sludge.
these chemicals, they form airborne particles and ground- Statement 3 is correct. Nanoparticles can result in
level ozone—or smog. increased production of reactive oxygen species (ROS),

245 Workbook
.
ENVIRONMENT

including free radicals like Cl–. Radicals like Cl– destroy field to sow the next crop. Open stubble burning emits
ozone. large amounts of toxic pollutants in the atmosphere
which contain harmful gases like methane (CH4), Carbon
47. Solution (d) Monoxide (CO), Volatile organic compound (VOC) and
Exp) Option d is correct. carcinogenic polycyclic aromatic hydrocarbons.
Statement 1 is correct. Microplastics are not biodegradable. Statement 2 is correct. Pests feed on the remaining
Thus, once in the environment, microplastics accumulate stubble and hence these pests may persist there even after
and persist, it is found in variety of environment including the sowing of new crops and hence may cause harm to
ocean and fresh water ecosystems. the crops.
Statement 2 is correct. Microplastics also are a source Statement 3 is correct. Stubble burning has several ill
of air pollution, occurring in dust and airborne fibrous effects. Besides causing pollution, burning stubble reduces
particles fertility of soil. It leads to loss of moisture within the soil.
It also destroys microbes in the soil.
Statement 3 is correct. Several countries banned the
products using microplastic, India has not banned the 50. Solution (a)
microplastic yet. On the order given by National Green
Exp) Option a is correct.
Tribunal (NGT) for analysis of microbeads in cosmetics,
the Bureau of Indian Standards did a study and classified Torrefaction is a Swedish technology and thermal
microbead products as ‘not fit for use’ in May 2017. process to convert biomass into a coal-like material,
“However, after that there has been no notification or which has better fuel characteristics than the original
instruction from the government banning the use of biomass. It can be used to convert rice stubble into ‘bio-
mircrobeads in cosmetics in India. coal’ and hence fight the menace of stubble burning in
north India.
Statement 4 is incorrect. Microplastics act as bulking
agent for the products, helping in increasing volume of 51. Solution (a)
the product. The microbeads used in face wash can cause
Exp) Option a is correct.
tiny rips in the skin letting in the bacteria contributing
to skin ageing and dark spots. Statement 1 is correct. It is essentially a fungi-based
liquid solution that can soften hard stubble to the extent
48. Solution (c) that it can be easily mixed with soil in the field to act as
Exp) Option c is correct compost. This would then rule out the need to burn the
Statement 1 is incorrect. The amount of waste produced stubble, and also help in retaining the essential microbes
by the nuclear power industry is small relative to other and nutrients in soil that are otherwise damaged when the
industrial activities. 97% of the waste produced is residue is burned.
classified as low- or intermediate-level waste (LLW or Statement 2 is incorrect. It has been developed by Indian
ILW) Agriculture Research Institute (IARI) in the form of
Statement 2 is incorrect. To produce ions in matter at capsules.
molecular level, Ionizing radiations have to maintain 52. Solution (a)
high energy. This means It has high frequency and low
wavelength. Thus due to high frequency, it has high Exp) Option a is correct.
penetration power and cause breakage of macromolecules Statement 1 is correct. Arresters are used to separate
whereas Non-ionizing radiations have low frequency and particulate matters from contaminated air. Scrubbers
have longer wavelength. They affect only those components are used to clean air for both dust and gases by passing it
which absorb them and have low penetrability. though a dry or wet packing materials.
Statement 3 is correct. Non-ionizing radiations are Statement 2 is correct. The gaseous pollutants can be
constituted by the electromagnetic waves at the longer controlled through the techniques of Combustion,
wavelength of the spectrum ranging from near infra- absorption and adsorption. These techniques can be
red rays to radio waves (includes higher wavelength employed singly or in combination. They are effective
ultraviolet rays, microwaves), whereas Ionizing radiations against the major greenhouse gases as well.
are constituted by short wavelength ultraviolet radiations
(UV), X-rays and gamma rays and energetic particles Statement 3 is incorrect. Using lead free petrol will help
produced in nuclear processes, electrically charged in reducing the air pollution.
particles like alpha and beta particles produced in
53. Solution (b)
radioactive decay and neutrons produced in nuclear
fission Exp) Option b is correct
Pair 1 is incorrectly matched. Bioventing is an in-situ
49. Solution (d)
technique of bioremediation which involves supply of
Exp) Option d is correct. air and nutrients to contaminated soil to stimulate the
Statement 1 is correct. Stubble (parali) burning is the growth of indigenous bacteria. Whereas the technique
act of setting fire to crop residue to remove them from the of injecting air under pressure below the water table

Workbook 246
.
ENVIRONMENT

to increase groundwater oxygen concentrations is called Statement 1 is correct. The Green Skill Development
bio-sparging. Programme (GSDP) is utilising the vast network
Pair 2 is correctly matched. Rhizo-filtration is a and expertise of Environmental Information System
technique used in wetlands and natural estuaries. It is (ENVIS) hubs and Resource Partners (RPs). ENVIS
a water remediation technique which involves uptake is a decentralized network of 66 centres of which 31
of contaminants by plant roots. It is a sub type of Centres dealing with ‘”State of the Environment and
Related Issues” are hosted by State Government /UT
phytoremediation.
Administrations called ENVIS Hubs and remaining 35
Pair 3 is correctly matched. Landfarming is an ex-situ Centres are hosted by environment-related governmental
technique of bioremediation which involves extraction and non-governmental organisations.
of contaminated soil and which is spread on a prepared Statement 2 is incorrect. It is the SDG Index that
bed and periodically tilled until the pollutants are evaluates the progress of states and union territories
degraded. on environmental skill parameters. Green skills are the
54. Solution (d) knowledge, abilities, values and attitudes needed to live in,
develop and support a sustainable and resource-efficient
Exp) Option d is correct. society. The Green General Skill index identifies four
Statement 1 is correct. Green crackers do not contain groups of work tasks that are especially important for
banned chemicals such as lithium, arsenic, barium green occupations:
and lead. They are called Safe Water Releaser (SWAS), Engineering and technical skills
Safe Thermite Cracker (STAR) and Safe Minimal
Science skills
Aluminium (SAFAL) crackers.
Operation management skills
Statement 2 is correct. Green crackers, Safe Water
Releasers (SWAS) , emit water vapour while exploding Monitoring skills
and thereby dilutes the dust particles. 57. Solution: (b)
Statement 3 is correct. ‘Green’ crackers were researched Exp) Option b is correct
and developed by scientists at CSIR-NEERI as per the
Supreme Court’s directions. They are designed to have E-waste broadly covers waste from all electronic and
electrical appliances and comprises of items such as
30% less particulate matter pollution compared to their
computers, mobile phones, digital music recorders/
conventional counterparts.
players, refrigerators, washing machines, televisions (TVs)
55. Solution: (b) and many other household consumer items. Computers
and associated parts contain several of harmful chemicals.
Exp) Option b is correct
Option 1 is correct: Beryllium is steel-gray, strong,
Statement 1 is correct. The effects of radioactive pollutant
lightweight and brittle metal. It offers the combination
depend on:
of high flexural rigidity, thermal stability, thermal
Half-life conductivity and low density. It is widely used in
Energy releasing capacity development of semiconductors that are used in
Rate of diffusion and deposition of the pollutant. computers. Beryllium is also used in Dental alloys,
aerospace materials, telescope, communication
Environmental factors like wind, temperature and rainfall equipment among other things. Hence with their
also influence their effects. improper disposal Beryllium becomes an e-waste hazard.
Statement 2 is incorrect. The most important natural Option 2 is correct: Cadmium is a soft, malleable,
radioactive isotope in the hydrosphere is potassium-40, ductile, metal. Till 2009, more than 80% batteries used
which is mostly found dissolved in water. Because of the Cadmium. Cadmium is still widely used in several
lower salt concentration, the radioactivity of rivers is applications including batteries, electroplating, nuclear
much less than that of seawater (and not vice-versa). fission, televisions, and in some semi-conductors. Hence
Statement 3 is correct. Gamma ray is the strongest of they become a e-waste with poor waste management.
the three radiations and can easily penetrate human Option 3 is correct: Chromium is a steely-grey, lustrous,
skin. But it can only be blocked by a very thick, strong, hard, and brittle metal. It is still used to make steel-
massive piece of concrete. tools, to add pigment in various applications, tanning
of leather. It is found as corrosion inhibitor on circuit
56. Solution: (a) boards of computers.
Exp) Option a is correct. Option 4 is incorrect: Heptachlor is an organochlorine
The Green Skill Development Programme (GSDP) of the compound that was used as an insecticide. It is not found
Ministry of Environment, Forest and Climate Change in computers and its parts. However, it is a persistent
(MoEF&CC) is an initiative for skill development in the organic pollutant (POP). It is listed under Stockholm
environment and forest sector to enable India’s youth to Convention which aims to reduce or eliminate the use of
get gainful employment and/or self-employment. POPs through the active measures of member countries.

247 Workbook
.
ENVIRONMENT

Option 5 is correct: Mercury is used in batteries view a starry sky due to excessive, misdirected or obtrusive
(laptops) and printed circuit boards in addition to artificial light. And in the United States, unnecessary
a variety of electrical equipment. It is known to cause lighting at night wastes an estimated $2.2 billion annually.
brain damage. Hence their poor handling makes them an
e-waste hazard. 60. Solution: (b)
Option 6 is correct: Lead makes up about 6% of the total Exp) Option b is correct.
weight of a computer and is used in the CRT of monitors Plastic pollution is the accumulation of plastic objects and
and to solder (a method of connecting) components on particles (e.g., plastic bottles, bags and microbeads) in the
printed circuit boards. About 40% of the lead found in Earth’s environment that adversely affects wildlife, wildlife
landfills comes from electronics. habitat, and humans. Plastics that act as pollutants are
Option 7 is incorrect: Plutonium is a radioactive categorized by size into micro-, meso-, or macro debris.
chemical element which is not used in manufacturing of Statement 1 is incorrect. Biodegradability is a convenient
Computers. It is used in nuclear explosives. term to describe materials that degrade naturally. In
58. Solution: (c) reality, biodegradable plastics need to be processed in a
composting facility - in temperatures ranging between
Exp) Option c is correct
50 and 60°C - in a concentrated microbial environment.
Studies by the Central Water Commission show that These are not conditions occurring naturally, much less
rivers (surface drinking water sources) in India are loaded at sea. In seawater, biodegradable plastics degrade at
with toxic heavy metals such as lead, arsenic, copper, significantly slower speed as compared to freshwater.
cadmium, mercury and nickel. In India, drinking water
especially in wells is highly contaminated with Uranium. Statement 2 is correct. Polyhydroxyalkanoates (PHAs)
are biodegradable plastics synthesized by many species
Sobitol and Formaldehyde are not drinking water of Bacteria and Archaea. These plastics are biodegradable
pollutants.
and are used in the production of bioplastics. They can
Sorbitol is a type of carbohydrate called a sugar alcohol. be either thermoplastic or elastomeric materials, with
It contains about one-third fewer calories than sugar and melting points ranging from 40 to 180 °C.
is 60 percent as sweet and occurs naturally in a variety of
berries and fruits. Statement 3 is correct. Biodegradable polyesters such
as polylactic acid (PLA) are widely used because of
Formaldehyde is a simple chemical compound made of their tensile strength, modulus, barrier properties and
hydrogen, oxygen and carbon. All life forms – bacteria, transparency. Biodegradable polymers are easily broken
plants, fish, animals and humans – naturally produce down by water and micro-organisms. They degrade into
formaldehyde as part of cell metabolism.
carbon dioxide and water when placed in compost.
59. Solution: (d)
61. Solution: (d)
Exp) Option d is correct.
Exp) Option d is the correct answer.
Light pollution is a global issue which became glaringly
obvious when the World Atlas of Night Sky Brightness, Pollutants are released from both stationary sources
a computer-generated map based on thousands of satellite and non-stationary sources. Stationary sources includes
photos, was published in 2016. commercial and industrial facilities, for example, thermal
power plants, chemical plants, paper factories and the
Statement 1 is correct. Light pollution is the excessive
or inappropriate use of outdoor artificial light, is municipal waste areas whereas non-stationary or mobile
affecting human health, wildlife behaviour, and our ability sources comprise of ships, aero planes and automobiles.
to observe stars and other celestial objects. Air can be polluted through natural processes or by
human-made activities.
Statement 2 is correct. Studies show that light pollution
is also impacting animal behaviours, such as migration Some of the known natural sources
patterns, wake-sleep habits, and habitat formation. of air pollution are:
Because of light pollution, sea turtles and birds guided by Volcanic eruptions
moonlight during migration get confused, lose their way,
and often die. Large numbers of insects, a primary food Forest fires
source for birds and other animals, are drawn to artificial Biological decay
lights and are instantly killed upon contact with light Radioactive materials
sources. Birds are also affected by this, and many cities
have adopted a “Lights Out” program to turn off building Pollen grains etc.
lights during bird migration. Pair 1 is correct: Sulphur dioxide is produced by
Statement 3 is correct. The dark-sky movement is a volcanoes, bogs, swamps, springs naturally and also
collection of worldwide efforts aimed at minimizing the through various industrial processes. It is passed
impact of artificial light at night. No single organization into the atmosphere when coal and oil burns. Flue-gas
heads it. According to the association, two-thirds of the desulfurization (FGD) plants, furnace oil of coal power
world’s population lives where they cannot adequately plant, paper mills, iron and steel industries,

Workbook 248
.
ENVIRONMENT

Petroleum refineries, sewage treatment plants, vehicles ecosystem. Changes in the population density may
engines, crackers smoke are some of the important indicate negative impacts to the ecosystem. Changes in
sources of sulphur compounds. populations may be a result of the relationship between
Acid rains are another form of major pollution threat that populations and food sources. Animal indicators also
is caused by sulphur dioxide and nitrogen oxides released help in detecting the amount of toxins present in the
into the air in high concentrations. Acid rain occurs when tissues of animals.
these substances mix and react with water, oxygen and Statement 3 is incorrect: Planktons are profoundly
other chemicals in the atmosphere forming more acidic sensitive to natural change they are best markers of
pollutants and fall down as rain. water quality and particularly lake conditions. One of
the reasons planktons are being considered in lakes is
Pair 2 is correct: Carbon monoxide (CO) is a colourless,
to monitor the water quality of the lake when there are
odourless, non-irritating but very poisonous gas. It is high centralizations of phosphorus and nitrogen; these
a product by incomplete combustion of fuel such as centralizations may be indicated by certain planktons
natural gas, coal or wood. Vehicular exhaust from cars, reproducing at an increased rate. (And not at a
trucks and other vehicles is a major source of carbon decreased rate) This is evidence of poor water quality that
monoxide. Kerosene and gas stoves, gas heaters, chimney may influence other organisms living in the water body.
and Thus, the plankton is key to the aquatic organisms, as both
Furnaces or boilers, generators, water heaters, and so on an indicator of water quality and as the main food source
release a significant amount of carbon monoxide. for many fish.
Pair 3 is correct: The nitrogen oxide is emitted from 63. Solution: (d)
high temperature combustion. Nitrogen dioxide is
responsible for photochemical smog, acid rain etc. Urea is Exp) Option d is the correct answer.
one of the main sources of nitrogen in fertilizers. Source Health-care activities protect and restore health and save
of nitrogen compounds are jet engines, fertilizers using lives. The health care activities also generate the waste
Ammonia, fossil fuel combustion, livestock, poultry and by-products. Of the total amount of waste generated
farm waste, vegetation, biomass burning and energy by health-care activities, about 85% is general, non-
production, etc. hazardous waste comparable to domestic waste. The
remaining 15% is considered hazardous material that
Pair 4 is correct: Particulate matter or fine particles are may be infectious, chemical or radioactive.
tiny particles of solid or liquid suspended in a gas or air.
In contrast, aerosol refers to particles and the gas together. Waste and by-products cover a diverse range of
Some particulates occur naturally, originating from materials, as the following list illustrates:
volcanoes, dust storms, forest and grassland fires, etc. Infectious waste: waste contaminated with blood and
The burning of fossil fuels in vehicles, power plants and other bodily fluids (e.g., from discarded diagnostic
various industrial processes contribute a large amount of samples), cultures and stocks of infectious agents from
aerosols in the atmosphere by human activities. laboratory work (e.g., waste from autopsies and infected
animals from laboratories), or waste from patients with
62. Solution: (a) infections (e.g., swabs, bandages and disposable medical
Exp) Option a is the correct answer. devices);
Bio indicators are used for assessing environmental Sharps waste: syringes, needles, disposable scalpels and
health and biogeographic changes taking place in the blades, etc.;
environment. Chemical waste: for example, solvents and reagents used
Statement 1 is correct: Bio indicators are living for laboratory preparations, disinfectants, sterilant and
organisms such as plants, planktons, animals, and heavy metals contained in medical devices (e.g. mercury
microbes, which are utilized to screen the health of in broken thermometers) and batteries;
the natural ecosystem in the environment. Each Cytotoxic waste: waste containing substances with
organic entity inside a biological system provides an genotoxic properties (i.e. highly hazardous substances
indication regarding the health of its surroundings. that are, mutagenic, teratogenic or carcinogenic), such
Planktons responds rapidly to changes taking place in the as cytotoxic drugs used in cancer treatment and their
surrounding environment and serving as an important metabolites;
biomarker for assessing the quality of water as well as an Radioactive waste: such as products contaminated by
indicator of water pollution. radionuclides including radioactive diagnostic material
Statement 2 is correct: Frogs are Bio indicators of or radiotherapeutic materials;
quality of environment and changes in environment.
Frogs are basically influenced by changes that take place 64. Solution: (a)
in their freshwater and terrestrial habitats. Exp) Option a is the correct answer.
Variations in the populations of animals may indicate Organic waste can be managed these days using
harmful changes caused due to pollution into the techniques such as Vermicomposting and Pyrolysis. The

249 Workbook
.
ENVIRONMENT

residue, thus obtained is beneficial for the soil health and specially engineered containers for safe storage and
improves crop productivity. disposal. This also allows for easier handling and
Statement 1 is correct: Vermiculture mainly refers to transport. (Refer Picture Below)
the scientific process of cultivating worm or artificial Option 1 is correct: Low level waste’s radioactivity
rearing of worms to decompose organic food wastes is usually compacted into steel canisters and stored
into a nutrient-rich material. The output of vermiculture
in concrete vaults underground. When full, vaults
is called vermicompost. It is formed by the process in
which earthworms consume the roughages in addition are sealed, covered and left. They ensure no significant
to the wastes from farms. Vermicompost is the excreta of radiation reaches the surface.
earthworm, which is rich in humus. Option 2 is correct: Low-level nuclear waste (LLW) has
Statement 2 is incorrect: Pyrolysis is a thermochemical a radioactive content not exceeding four giga-becquerels
treatment, which can be applied to any organic (carbon- per tonne. LLW does not require shielding during
based) product. Pyrolysis, which is also the first step handling and transport, and is suitable for disposal in
in gasification and combustion, occurs in the absence near surface facilities.
or near absence of oxygen. In this treatment, material
is exposed to high temperature, and in the absence of LLW is generated from hospitals and industry, as well
oxygen. Pyrolysis products produce charcoal, biochar and as the nuclear fuel cycle. It comprises paper, rags, tools,
non-condensable gases (hydrogen, methane, CO2). clothing, filters, etc., which contain small amounts of
mostly short-lived radioactivity. To reduce its volume,
65. Solution: (d) LLW is often compacted or incinerated before disposal.
Exp) Option d is the correct answer. LLW comprises some 90% of the volume but only 1% of
Radioactive waste is produced by industries such as the radioactivity of all radioactive waste.
mining, nuclear power generation, defense, medicine, Option 3 is correct: Intermediate and high-level waste
and certain types of scientific research. Radioactive waste
generate heat and greater levels of radioactivity. Most
is hazardous because it emits radioactive particles, which
if not properly managed can be a risk to human health and countries plan to use deep geological
the environment. disposal. The rock and soil acts as a barrier to the
Examples of treatment include decontaminating, radiation. Before this, high level waste is incorporated
shredding, compacting, drying and solidifying the into glass and stored for up to fifty years to allow heat to
waste. Most radioactive waste requires packaging in dissipate.

Workbook 250
.
ENVIRONMENT

66. Solution: (c) Option 4 is incorrect: Hair oils generally contain natural
Exp) Option c is the correct answer. and/or mineral oils. Similarly, due to strict regulations,
now almost no cosmetic products use lead in their
Plastics are large carbon-containing compounds called
compositions.
polymers, which are composed of repeating units of
shorter carbon-containing compounds called monomers. 68. Solution: (a)
Statement 1 is correct: Most plastic is chemically inert Exp) Option a is correct.
and will not react chemically with other substances. Air pollution is a major cause of premature death and
Because plastic doesn’t react chemically with most disease, and is the largest environmental health threat
other substances, it doesn’t decay. Therefore, plastic globally. The high burden of death and disease due to air
disposal poses a difficult and significant environmental pollution and its associated substantial adverse economic
problem. impact from loss of output could impede India’s aspiration
Statement 2 is incorrect: There have been significant efforts to be a $5 trillion economy by 2024. 1·67 million (95%
in recent decades toward developing and industrializing uncertainty interval 1·42–1·92) deaths were attributable
so-called “biodegradable” plastics that might have shorter to air pollution in India in 2019, accounting for 17·8%
residence times in the environment. Oxo-degradable (15·8–19·5) of the total deaths in the country.
plastics are one class of plastic materials that are Statement 1 is correct. The majority of these deaths were
commonly promoted as biodegradable. In reality, they from ambient particulate matter pollution (0·98 million
are conventional plastics (e.g., PE, PP, PET) containing [0·77–1·19]) and household air pollution (0·61 million
additives that accelerate the oxidation process. The [0·39–0·86]). The death rate due to household air pollution
major issue with oxo-degradable plastics is that they decreased by 64·2% (52·2–74·2) from 1990 to 2019, while
rapidly fragment into huge quantities of microplastics that due to ambient particulate matter pollution increased
when exposed to a combination of sunlight and oxygen. by 115·3% (28·3–344·4) and that due to ambient ozone
Statement 3 is correct: Polylactic acid (PLA) is a pollution increased by 139·2% (96·5–195·8).
biodegradable as well as recyclable polyester made from Statement 2 is incorrect. An aerosol is a tiny particle
renewable feedstock. Lactic acid as the raw material is (solid or liquid) in the atmosphere. Aerosols are a part
produced by fermentation of glucose or sucrose and is of air pollution and are dangerous to human health.
refined to a high purity. There are other polymers besides Some aerosols are so small that they are only made of a
plastics. Naturally occurring polymers, such as starches, few molecules and are invisible; some are visible but still
cellulose, soy protein, vegetable oil, triglycerides and very small. The smaller and lighter a particle is, the longer
bacterial polyesters, can be extracted from crops and is stays in the air. Some aerosols are a natural part of the
bacteria. atmosphere - coming from erupting volcanoes, sea salt,
and wildfires. However, humans add lots of aerosols to
67. Solution: (b) the atmosphere by burning fossil fuels such as coal, oil,
Exp) Option b is correct and gas. When we breathe in these tiny particles, they can
Lead (Pb) is a naturally occurring toxic metal found damage lung tissue and lead to lung diseases. Aerosols
in the Earth’s crust. Its widespread use has resulted in can also limit visibility, causing haze in many parts of the
extensive environmental contamination, human exposure world.
and significant public health problems in many parts 69. Solution: (c)
of the world. Important sources of lead poisoning
include mining, smelting, manufacturing and recycling Exp) Option c is correct.
activities, and, in some countries, the continued use of Option a is incorrect. Carbon monoxide is a colourless,
leaded paint, leaded gasoline, and leaded aviation fuel. odourless gas. Prolonged exposure to carbon monoxide
Option 1 is correct: Smelting is a process of applying heat rich atmospheres may be fatal. It is easily ignited. It
to ore in order to extract a base metal. Lead is extracted is just lighter than air and a flame can flash back to the
through the process of smelting. It is still widely used for source of leak very easily. Under prolonged exposure to
fire or intense heat the containers may violently rupture
car batteries, pigments, ammunition, cable sheathing,
and rocket. Carbon Monoxide is an odourless, tasteless,
weights for lifting, weight belts for diving, lead crystal
poisonous gas, CO that results from the incomplete
glass, radiation protection and in some solders.
combustion of carbon. Inhalation causes central nervous
Option 2 is incorrect: Pens and Pencils do not contain system damage and asphyxiation.
lead (Pb). Contrary to popular assumption, pencils’ lead Option b is incorrect. Nitrogen Dioxide (NO2) is one
is actually made from Graphite. of a group of highly reactive gases known as oxides of
Option 3 is correct: Lead is added to paint to nitrogen or nitrogen oxides (NOx). Other nitrogen oxides
accelerate drying, increase durability, maintain a fresh include nitrous acid and nitric acid. NO2 is used as the
appearance, and resist moisture that causes corrosion. indicator for the larger group of nitrogen oxides. NO2
It is one of the main health and environmental hazards primarily gets in the air from the burning of fuel. NO2
associated with paints. Due to environmental and health forms from emissions from cars, trucks and buses, power
hazards, lead-free paints are also available in the market. plants, and off-road equipment.

251 Workbook
.
ENVIRONMENT

Option c is correct. Sulfur dioxide is a colourless gas motion of air into or out of the polar stratosphere. Once
with a pungent odour. It is a liquid when under pressure, chemical changes occur in polar regions from reactions
and it dissolves in water very easily. Sulfur dioxide in the on PSCs, the isolation preserves those changes for many
air comes mainly from activities such as the burning of weeks to months.
coal and oil at power plants or from copper smelting.
In nature, sulfur dioxide can be released to the air from 72. Solution (d)
volcanic eruptions. Acid rain is caused by a chemical Exp) Option d is correct
reaction that begins when compounds like sulfur dioxide Statement 1 is correct: Acid rain damages plants as
and nitrogen oxides are released into the air. it leaches away nutrients with it. But in case of food
Option d is incorrect. Carbon dioxide is a colourless and crops, there is application of lime and fertilizers which
non-flammable gas at normal temperature and pressure. minimizes the impact of acid rain by supplementing
Although much less abundant than nitrogen and oxygen extra nutrients.
in Earth’s atmosphere, carbon dioxide is an important Statement 2 is incorrect: Wet deposition commonly
constituent of our planet’s air. A molecule of carbon occurs in wet weather where acidity comes down in form
dioxide (CO2) is made up of one carbon atom and two of rain, snow, fog, etc. But dry depositions accounts for
oxygen atoms. half the acidity which falls back on earth. It usually
occurs in dry weather and while falling down sticks to the
4.3. Environmental Issues ground, buildings, vegetation, car, etc.
70. Solution (c) Statement 3 is correct: Acid rain leads to washing away
nutrients. As upland and mountainous areas are more
Exp) Option c is correct. susceptible to acid rain, its effect on conifers is also
Statement 1 is correct. Methane is a key precursor gas of harmful. When calcium is leached from the needles of
the harmful air pollutant, tropospheric ozone. Globally, red spruce, these trees become less cold tolerant and
increased methane emissions are responsible for half of exhibit winter injury and even death.
the observed rise in tropospheric ozone levels. While Statement 4 is incorrect: Indian soils are mostly alkaline
methane does not cause direct harm to human health or and has a good buffering ability, which makes them less
crop production, ozone is responsible for about 1 million susceptible to harmful impacts of acid rain
premature respiratory deaths globally.
Statement 2 is correct. Methane is a short-lived climate 73. Solution (b)
pollutant with an atmospheric lifetime of around 12 Exp) Option b is correct.
years. While its lifetime in the atmosphere is much Statement 1 is incorrect: Chlorofluorocarbons are a
shorter than carbon dioxide (CO2), it is much more family of inert, non-toxic and non-flammable gases.
efficient at trapping radiation. In terms of per unit of
Statement 2 is correct: Chlorofluorocarbons are used
mass, the impact of methane on climate change over 20 as refrigerants, propellants in aerosol sprays, foaming
years is 86 times greater than CO2; over a 100-year period agents in plastic manufacturing. They are also used as fire
it is 28 times greater. extinguishing agents, solvents for cleaning electronic and
71. Solution (d) metallic components, for freezing foods etc.
Exp) Option d is correct. Statement 3 is correct: CFCs are potential destroyers
of Ozone layer in the upper atmosphere or stratosphere.
Statement 1 is correct: The severe ozone destruction Under the Montreal Protocol (1987), production and
represented by the ozone hole requires that low emissions of chlorofluorocarbons have been curbed
temperatures be present over a range of stratospheric successfully.
altitudes, over large geographical regions, and for
extended time periods. Low temperatures are important 74. Solution (d)
because they allow liquid and solid Polar Stratospheric Exp) Option d is correct.
Clouds (PSCs) to form. Pair 1 is correctly matched. Black carbon refers to the
Statement 2 is correct: Reactions on the surfaces of carbon emitted during incomplete combustion of fossil
liquid and solid Polar Stratospheric Clouds (PSCs) can fuels in coal-fired power plants, cars and other equipment.
substantially increase the relative abundances of the Black carbon warms the Earth by absorbing heat in the
most reactive chlorine gases. These reactions convert the atmosphere and by reducing albedo, (the ability to reflect
reservoir forms of reactive chlorine gases, chlorine nitrate sunlight) when deposited on snow and ice.
and hydrogen chloride (HCl), to the most reactive form. Pair 2 is correctly matched. Brown carbon refers to the
Statement 3 is correct: Stratospheric air in the polar carbon released due to combustion of organic matter.
regions is relatively isolated from other stratospheric Biomass burning (possibly domestic wood burning) is
regions for long periods in the winter months. The isolation shown to be a major source of brown carbon.
comes about because of strong winds that encircle the Pair 3 is correctly matched. Blue carbon is the term
poles, forming a polar vortex, which prevents substantial for carbon captured by the world’s ocean and coastal

Workbook 252
.
ENVIRONMENT

ecosystems. Blue carbon is the carbon stored in coastal It was in 1994, that the government notified the
and marine ecosystems. Environment Impact Assessment norms for the first
time through the EIA notification of 1994. This was later
75. Solution (d) replaced by the 2006 EIA notification.
Exp) Option d is correct. Statement 2 is incorrect. Public hearing during the
Statement 1 is correct. There has been large scale preparation of EIA Report is mandatory for only
deforestation for palm plantations. This has been quite certain kind of development projects.
substantial in countries like Indonesia and Malaysia. Public hearing during the preparation of EIA report is
This has threatened the orangutan habitats in these mandatory only in the case of Category A and Category
countries. Today, fewer than 50,000 orangutans exist in B1 projects.
the wild, split into smaller groups. B2 category projects like modernization of irrigation,
Statement 2 is correct. The large-scale deforestation expansion of roads and highway are exempted from
for palm plantations has resulted in loss of biodiversity. public hearing.
Substantial tracts of tropical forests are cleared to make Statement 3 is incorrect. The EIA report is prepared by
room for large plantations to service an ever-increasing the project proponent, not by the State Forest Department.
demand for palm oil. The project proponent submits the EIA report to the
Statement 3 is correct. As land on mineral soil has State Pollution Control Board (SPCB) and State Forest
become less available for plantations, the expansion of Department (if the project covers the forest lands).
oil palm is increasing on peat lands. As these areas are
drained, the peat is exposed to oxidation, resulting in 78. Solution: (c)
significant CO2 release over an extended period. Exp) Option c is the correct answer.
Statement 4 is correct. There has been an increase in The pollution of environment, particularly that caused
greenhouses gases emissions due to extensive palm by various industrial activities, have been responsible for
oil production in the world. It is due to - deforestation, the organic and inorganic matter in the ecosphere. The
forest degradation, peat land emissions - done for the utilization of organisms, primarily microbes, to clean
cultivation of palm oil. There is also the use of fires for up contaminated soils, aquifers, sludges, residues, and
land clearing and the emissions of methane from the air known as ‘Bioremediation’, is a rapidly changing and
effluent treatment ponds of palm oil mills. expanding area of environmental cleanup technique than
conventional physio-chemical methods.
4.4. EIA Some of the techniques of bioremediation are:
Pair 1 is matched correctly: Rhizofiltration is defined
76. Solution (a) as the use of plants, both terrestrial and aquatic; to
Exp) Option a is correct. absorb, concentrate, and precipitate contaminants
Statement 1 is correct. Environment Impact Assessment from polluted aqueous sources with low contaminant
or EIA can be defined as the study to predict the effect concentration in their roots. Rhizofiltration can partially
of a proposed activity/project on the environment. A treat industrial discharge, agricultural runoff etc. The
decision-making tool, EIA compares various alternatives advantages of rhizofiltration include it ability to be used
as in-situ or ex-situ applications.
for a project and seeks to identify the one which represents
the best combination of economic and environmental Pair 2 is matched correctly: In Phytodegradation,
costs and benefits. contaminants are taken up into the plant tissues where
they are metabolized, or biotransformed. Where the
It helps to identify possible environmental effects of the
transformation takes place depends on the type of plant,
proposed project, proposes measures to mitigate adverse
and can occur in roots, stem or leaves.
effects and predicts whether there will be significant
adverse environmental effects, even after the mitigation is Pair 3 is matched incorrectly: Mycoremediation is
implemented. a form of bioremediation in which fungi are used to
decontaminate the area. The term mycoremediation
Statement 2 is correct. EIA process legislation was notified refers specifically to the use of fungal mycelia in
in 2006 and is statutorily backed by the Environment bioremediation. The key to mycoremediation is
Protection Act, 1986. determining the right fungal species to target a specific
Statement 3 is incorrect. Screening is the first step in EIA pollutant. Mycofiltration is a similar process, using
process. fungal mycelia to filter toxic waste and microorganisms
from water in soil.
77. Solution (c)
Exp) Option c is correct. 79. Solution: (d)
Statement 1 is correct. The Environment Impact Exp) Option d is the correct answer.
Assessment Notification of 2006 was promulgated When atmospheric pollutants like oxides of nitrogen and
under the provisions of Environment Protection Act of sulphur react with rainwater and come down with the
1986. rain, then this results in Acid Rain. It includes any form of

253 Workbook
.
ENVIRONMENT

precipitation with acidic components in wet or dry forms- Pollutants, especially nondegradable ones, move through
rain, snow, fog, hail or even dust that is acidic. the various trophic levels in an ecosystem. Movement of
Effects of Acid Rain these pollutants involves two main processes:
Acid rain is very harmful to agriculture, plants, and i) Bioaccumulation
animals. It washes away all nutrients which are required ii) Biomagnification.
for the growth and survival of plants. Acid rain affects Statement 1 is correct. Bioaccumulation refers to how
agriculture by the way it alters the composition of the pollutants enter a food chain. In bioaccumulation there
soil. (Hence, option 1 is correct). is an increase in concentration of a pollutant from the
It causes respiratory issues in animals and humans. environment to the first organism in a food chain.
(Hence, option 2 is correct).
Acid rain also causes the corrosion of water pipes, which
further results in leaching of heavy metals such as iron,
lead and copper into drinking water.
It damages the buildings and monuments made up of
stones and metals. (Hence, option 3 is correct).
When acid rain falls down and flows into the rivers
and ponds it affects the aquatic ecosystem. It alters the
chemical composition of the water as it contains nitrogen.
Nitrogen pollution in our coastal waters is partially
responsible for declining fish and shellfish populations in
some areas (Hence, option 4 is correct).
80. Solution: (d) Statement 2 is incorrect.: Biomagnification is the
increasing concentration of a substance in the tissues of
Exp) Option d is the correct answer. tolerant organisms at successively higher levels in a food
Statement 1 is correct. The effects of pollution on plants chain. In order for biomagnification to occur, the pollutant
include “burning” at leaf tips or margins, twig dieback, must be: long-lived, mobile, soluble in fats, biologically
stunted growth, premature leaf drop, delayed maturity, active. But even if a pollutant is not active biologically, it
abortion or early drop of blossoms, and reduced yield may biomagnify, but we really don’t worry about it much,
or quality. Pollutants accounted for an estimated 5% since it probably won’t cause any problems. Ex, DDT.
loss in corn and soybean production in USA, based on Statement 3 is correct. Triclosan, an anti-fungal agent
research. is a bioaccumulant. It is an antibacterial and anti-
Statement 2 is correct. The effects of air pollution on fungal agent that slows or stops the growth of germs. It
plants can be either direct or indirect. Direct effect is is added to some antibacterial soaps and body washes,
when toxins harm plants by depositing on them directly toothpaste, and some cosmetics. Therefore, Triclosan
from the air and affecting their leaf metabolism and is most likely present in toiletries. Triclosan is a broad-
uptake of carbon, which they need to build their body spectrum antimicrobial agent that is frequently used in
and get energy to live. The chemicals responsible for pharmaceuticals and personal care products.
direct pollution include ozone and nitrogen oxides.
Statement 3 is correct. The ground level ozone is formed 82. Solution: (c)
when volatile organic compounds react with nitrogen Exp) Option c is correct.
oxides in the presence of sunlight. Ozone damages plants Secondary pollutants are pollutants which form in the
by preventing photosynthesis and obstructing stomata, atmosphere. These pollutants are not emitted directly
restricting respiration and stunting plant growth. When
from a source (like vehicles or power plants).
plants are exposed to this ozone throughout the season,
they first exhibit signs of damage on their leaves in the Statement 1 is correct. Intense sunlight helps primary
form of tiny light and dark spots, later followed by pollutants create secondary pollutants. Secondary
bronzing and reddening. Later on, leaves turn pale due pollutants are pollutants which form in the atmosphere.
to the lack of photosynthetic activity (chlorosis) and may These pollutants are not emitted directly from a source
die out. (like vehicles or power plants). Instead, they form as
Statement 4 is correct. About a third of all human food a result of the pollutants emitted from these sources
requires a pollinator. These pollinators go for the strongly reacting with molecules in the atmosphere to form a new
scented plants and those plants with bright flowering pollutant. Pollutants that are emitted into the environment
and are mainly pollinated at night. Increasing light from a source are called primary pollutants.
conditions will slow down all these processes and this Different types of secondary pollutants include:
will ultimately disrupt our food sources. Ozone (O3)
81. Solution: (c) Sulphuric acid and nitric acid (component of acid rain)
Exp) Option c is correct. Particulate matter

Workbook 254
.
ENVIRONMENT

Nitrogen dioxide (NO2) automobile exhaust from attached garages


Peroxyacyl nitrates (PANs) tobacco smoke
Statement 2 is correct. Ozone is the secondary pollutant Pair 3 is incorrect. Lead (not Radon) is used in paint,
that is formed due to reaction of intense sunlight and the gasoline, water pipes, and many other products. Radon
chemicals present in the atmosphere. Ozone is a reactive is a colorless, odorless gas, the primary source of indoor
oxidant gas produced naturally in trace amounts in air pollution. Radon sinks in air because it has a high
Earth’s atmosphere (Stratosphere). In the stratosphere density; it is therefore often found in the basements of
(the atmospheric layer above the troposphere) it is vital in homes, particularly in areas where with a lot of shale and
protecting the Earth from harmful ultraviolet radiation. boulders in the soil. Radon results from the radioactive
decay of radium in the soil, and it further decays to
produce radioactive daughters including polonium and
lead. Radon gas, along with decay products that can attach
to dust and airborne particles, enters the lungs and decays,
producing alpha and beta radiation that damages DNA
and causes lung cancer.
84. Solution: (d)
Exp) Option d is correct.
Since the onset of the industrial era, forests have declined
by 32%. Especially in the tropics, many of the world’s
remaining three trillion trees are falling fast, with about
83. Solution: (a) 15 billion cut each year, the Nature study states. In many
Exp) Option a is correct. places, tree loss is accelerating. In recent times there is an
Pair 1 is correct. Asbestos is a mineral fiber that occurs increased case of deforestation.
in rock and soil. Because of its fiber strength and heat Statement 1 is correct. Without trees, formerly forested
resistance it has been used in a variety of building areas would become drier and more prone to extreme
construction materials for insulation and as a fire- droughts. Trees mediate the water cycle by acting as
retardant. Asbestos has been used in a wide range of biological pumps: they suck water from the soil and
manufactured goods, mostly in: deposit it into the atmosphere by transforming it from
Building materials liquid to vapour. By doing this, forests contribute to cloud
formation and precipitation. Trees also prevent flooding
Roofing shingles by trapping water rather than letting it rush into lakes and
Ceiling and floor tiles rivers, and by buffering coastal communities from storm
Paper products surges. They keep soil in place that would otherwise wash
away in rain, and their root structures help microbial
Asbestos cement products
communities thrive.
Elevated concentrations of airborne asbestos can occur
Statement 2 is correct. Trees have a localized
after asbestos-containing materials are disturbed by
cutting, sanding or other remodeling activities. Improper cooling effect. They provide shade that maintains soil
attempts to remove these materials can release asbestos temperatures and they absorb heat rather than reflect it.
fibers into the air in homes, increasing asbestos levels and In the process of evapotranspiration, they also channel
endangering people living in those homes. energy from solar radiation into converting liquid water
into vapour. With all of those cooling services lost, most
Pair 2 is correct. Carbon monoxide is an odorless, places where trees formerly stood would immediately
colorless and toxic gas. Because it is impossible to see, become warmer.
taste or smell the toxic fumes, CO can kill you before you
are aware it is in your home. The effects of CO exposure Statement 3 is correct. With all trees on the planet wiped
can vary greatly from person to person depending on out, previously forested ecosystems “would become
age, overall health and the concentration and length of only a source of emission of carbon dioxide into the
exposure. atmosphere, rather than a sink. On a global scale, trees
combat warming caused by climate change by storing
Sources of CO include: carbon in their trunks and removing carbon dioxide from
unvented kerosene and gas space heaters the atmosphere. Deforestation already accounts for 13%
leaking chimneys and furnaces of total global carbon emissions, according to an IPCC
back-drafting from furnaces, gas water heaters, wood report published in August, while land use change in
stoves and fireplaces general accounts for 23% of emissions.
gas stoves 85. Solution: (c)
generators and other gasoline powered equipment Exp) Option c is correct.

255 Workbook
.
ENVIRONMENT

An ocean gyre is a large system of circular ocean currents by discharged chemicals. The bioremediation process
formed by global wind patterns and forces created by stimulates the growth of specific microbes that use the
Earth’s rotation. The movement of the world’s major ocean discharged chemical contaminants as a source of food
gyres helps drive the “ocean conveyor belt.” The ocean and energy.
conveyor belt circulates ocean water around the entire Pair 1 is correct. Land Farming is a bioremediation
planet. technology. Landfarming is an ex-situ waste treatment
Statement 1 is correct. The Great Pacific Garbage Patch is process that is performed in the upper soil zone or in
a collection of marine debris in the North Pacific Ocean. biotreatment cells. Contaminated soils are mixed with soil
Marine debris is litter that ends up in oceans, seas, and amendments such as soil bulking agents and nutrients,
other large bodies of water. The Great Pacific Garbage and then they are tilled into the earth.
Patch, also known as the Pacific trash vortex, spans Pair 2 is incorrect. A bioreactor refers to any
waters from the West Coast of North America to Japan. manufactured device or system that supports a
The patch is actually comprised of the Western Garbage biologically active environment. A bioreactor is a vessel
Patch, located near Japan, and the Eastern Garbage Patch, in which a chemical process is carried out which involves
located between the U.S. states of Hawaii and California. organisms or biochemically active substances derived
Statement 2 is correct. The area in the center of a gyre from such organisms. This process can either be aerobic
tends to be very calm and stable. The circular motion or anaerobic. In all cases, the bioreactor must provide the
of the gyre draws debris into this stable center, where environmental conditions necessary for the culture. It is
it becomes trapped. A plastic water bottle discarded off ex-situ bioremediation technique.
the coast of California, for instance, takes the California Pair 3 is correct. Biosparging is an in-situ remediation
Current south toward Mexico. Finally, the bottle travels technology that uses indigenous microorganisms to
eastward on the North Pacific Current. The gently rolling biodegrade organic constituents in the saturated zone. In
vortexes of the Eastern and Western Garbage Patches biosparging, air (or oxygen) and nutrients (if needed) are
gradually draw in the bottle. injected into the saturated zone to increase the biological
activity of the indigenous microorganisms. Biosparging
86. Solution: (d) can be used to reduce concentrations of petroleum
Exp) Option d is correct. constituents that are dissolved in groundwater, adsorbed
Petroleum coke (petcoke) is a byproduct of the oil refining to soil below the water table, and within the capillary
process. As refineries worldwide seek to operate more fringe. Although constituents adsorbed to soils in the
efficiently and extract more gasoline and other high value unsaturated zone can also be treated by biosparging,
fuels from each barrel of crude oil, a solid carbon material bioventing is typically more effective for this situation.
known as petcoke is produced.
Statement 1 is correct. Petroleum coke, the bottom-of- 4.5. Health impact of pollution
the-barrel leftover from refining crude and other heavy
88. Solution (b)
oils, is cheaper and burns hotter than coal. But it also
contains more carbon and sulphur than normal coal. A Exp) Option b is correct.
high sulfur content in petcoke reduces its market value, Pair 1 is correctly matched. Cadmium affects the heart,
and may preclude its use as fuel due to restrictions on accumulating in the human body, especially the kidneys.
sulfur oxides emissions for environmental reasons. Acute inhalation exposure to cadmium can damage the
Statement 2 is correct. India is the largest lungs. Chronic exposure can result in kidney, bone and
consumer of the pet coke in the world. As the lung disease.
world’s largest consumer of pet coke, India imports Pair 2 is incorrectly matched. Methemoglobinemia or
over half its annual pet coke consumption of about blue baby syndrome is caused by the presence of excess
27 million tonnes, mainly from the United States. nitrate in drinking water. Excess nitrate in drinking
Statement 3 is correct. India in 2018 banned the water reacts with hemoglobin to form non-functional
import of pet coke for use as fuel, but shipments for methemoglobin, and impairs oxygen transport. It is a
use as feedstock in some industries was allowed. Usage condition where a baby’s skin turns blue. This happens
of pet coke, a dirtier alternative to coal, in the energy- when there is not enough oxygen in the blood.
hungry country has come under scrutiny due to rising Lead causes damage to the central and peripheral nervous
pollution levels in major cities. Import of Pet coke is systems, blood systems, kidney and reproductive system
allowed for only cement, lime kiln, calcium carbide and in humans. It also effects the endocrine system, and
gasification industries, when used as the feedstock or in impedes brain development among children.
the manufacturing process on actual user condition. Pair 3 is correctly matched. Minamata disease,
87. Solution: (d) sometimes referred to as Chisso-Minamata disease, is
a neurological syndrome caused by severe mercury
Exp) Option d is correct. poisoning. Symptoms include ataxia, numbness in the
Bioremediation is the use of microbial species to clean hands and feet, general muscle weakness, narrowing of
up soil and groundwater that has been contaminated the field of vision and damage to hearing and speech

Workbook 256
.
ENVIRONMENT

89. Solution (d) Option d is correct: Exposure to uranium in drinking


Exp) Option d is correct. water is linked to chronic kidney diseases. It is has
become a major issue as groundwater depletion and
Pair 1 is correctly matched. Sulphur dioxide affects the nitrate pollution may be aggravating the already
respiratory system, particularly lung function, and can present natural uranium contamination to dangerous
irritate the eyes. It causes coughing, mucus secretion levels.
and aggravates conditions such as asthma and chronic
bronchitis. 92. Solution: (d)
Pair 2 is correctly matched. Carbon monoxide is Exp) Option d is correct
harmful when breathed because it displaces oxygen in Nano-particles are tiny materials having size ranges from
the blood and deprives the heart, brain and other vital 1 to 100 nm. They can be classified into different classes
organs of oxygen. Large amounts of CO can overcome based on their properties, shapes or sizes.
one in minutes without warning — causing one to lose Statement 1 is correct: Nanomaterials are able to cross
consciousness and suffocate. biological membranes and access cells, tissues and
Pair 3 is correctly matched. Exposure to high levels of organs that larger-sized particles normally cannot.
lead may cause anaemia, weakness, and kidney and brain Although numerous benefits, the unsafe discharge
damage. Very high lead exposure can cause death. Lead of nanoparticles in the environment is an issue. The
can also cross the placental barrier which can damage a unregulated exposure to the soil and water is expected to
developing baby›s nervous system. impair plant and fish growth and accumulate into edible
tissues.
90. Solution (a)
Statement 2 is correct: The accumulation in tissues of
Exp) Option a is correct animals allow nano-particles to enter the food chains as
Option a is incorrect. Polluted water or also the hot well.
water discharged from industries decreases the dissolved Statement 3 is correct: Free radicals are atoms or
oxygen content (and not necessarily increase carbon molecules that contain one or more unpaired electrons
dioxide content) which leads to the elimination of and are, in this sense, “free”. Various in-vitro and in-vivo
sensitive organisms like planktons. studies show that free radical formation can be triggered
Option b is correct. Polluted water is a source of by nanoparticles (fullerenes, carbon nanotubes,
various water borne diseases and is also responsible for quantum dots, emission particles) The chronic release
malnutrition in children and adults alike. of such reactive molecules in the body can lead to tissue
degeneration.
Option c is correct. Minamata disease was caused due
to human consumption of fishes living in the mercury 93. Solution: (c)
contaminated waters of Japan. Exp) Option c is correct.
Option d is correct. Agricultural runoff of fertilizers Many countries practice open dumping as a final disposal
of nitrogen and phosphorus increases the nutrient level method for Municipal Solid Waste (MSW), which not
of water in the lakes and ponds which lead to increased only causes surface water pollution, but the leachate (dark
growth of algae, called Eutrophication. liquid) discharge from the dumpsite also pollutes the
groundwater irreversibly.
91. Solution (c)
Methane is generated in the dumpsites which is estimated
Exp) Option c is correct. to have a global warming potential that is 84 times greater
Option a is correct: Uranium is a radioactive mineral than carbon dioxide.
whose primary source is geogenic (naturally occurring). To make this worse, methane in the landfills is particularly
Many of India’s aquifers are composed of clay, silt and dangerous often auto-ignites, causing fires in dumpsites
gravel carried down from Himalayan weathering by and generating smoke and emissions, thereby causing
streams of uranium-rich granitic rocks. severe air pollution.
Option b is correct: Anthropogenic factors such as Governments around the world have recognised the
ground water table decline and nitrate pollution may problem and over the years landfill mining or bio-mining
further enhance uranium mobilization. When over- has become a popular method of excavating waste from
pumping of these aquifers’ ground water occurs and their active or closed landfills and segregating the aggregates
water levels decline, it induces oxidation conditions generated with an ultimate purpose to clear them and
that, in turn, enhance uranium enrichment in the shallow reclaim the land.
ground water that remains. Statement (A) is correct but (R) is wrong. Bio-mining
Option c is incorrect: World Health Organisation has was mandated by the Solid Waste Management Rules,
set a provisional safe drinking water standard of 30 2016, wherein it was a preferred methodology over
micrograms of uranium (and not 100 micrograms) per ‘capping’ which simply meant covering the waste with
litre for India. soil.

257 Workbook
.
ENVIRONMENT

From 2014 onwards, the Swachh Bharat Mission has been 1) Bagasse based cogeneration in sugar mills -
emphasizing on reclamation of landfill sites to adhere to Cogeneration in Sugar Mills
the Solid Waste Management (SWM) Rules, 2016, and the Sugar industry has been traditionally practicing
guidelines of the Hon National Green Tribunal, with an cogeneration by using bagasse as a fuel. With the
aim to recover over an estimated 10,000 hectares of urban advancement in the technology for generation and
land that is locked in these dumpsites in India. utilization of steam at high temperature and pressure,
sugar industry can produce electricity and steam for
4.6. Renewable energy and energy efficiency their own requirements. It can also produce significant
surplus electricity for sale to the grid using same quantity
94. Solution (c)
of bagasse. For example, if steam generation temperature/
Exp) Option c is correct. pressure is raised from 400oC/33 bar to 485oC/66 bar, more
Option a is incorrect. As per the Ethanol Blending than 80 KWh of additional electricity can be produced
Programme, government has set a target of 10% of for each ton of cane crushed. The sale of surplus power
bioethanol blending of petrol by 2022. Earlier this generated through optimum cogeneration would help a
target is to be raised up to 20% by 2030. But, Prime sugar mill to improve its viability, apart from adding to
minister Narendra Modi announced June 5, 2021 the the power generation capacity of the country. Hence,
target of 20 per cent ethanol blended petrol has been Option 1 is correct.
advanced by five years to 2025. 2) Biogas is derived from the “Anaerobic Digestion” of
Option b is incorrect. It is 2nd Generation bio-fuel biomass. Biogas is produced when bio-degradable organic
which is based on utilizing surplus biomass and materials/wastes such as cattle-dung, biomass from farms,
agriculture waste products to produce bio-fuel. Whereas gardens, kitchens, industry, poultry droppings, night
in 3G bio-fuel are produced from specially engineered soil and municipals wastes are subjected to a scientific
microorganisms like algae. process, called Anaerobic Digestion (A.D.) in a Biogas
Plants. Hence, Option 2 is correct.
Option c is correct. As part of its renewable target for
2022, India had set a goal of 10000 MW from bio-power. Biogas Plant designs depend upon several factors and the
A total capacity of 10145 MW has been installed in feed stock to be processed is of paramount importance.
Biomass Power and Cogeneration Sector. This goal has Biogas is the mixture of gases (primarily methane
been already achieved. (CH4) and Carbon di-oxide (CO2) and traces of
Option d is incorrect. India has set a target of 100GW by Hydrogen Sulfide (H2S), Moisture) produced by the
solar energy by 2022. Of this target a sub-target has also decomposition/breakdown of bio-degradable organic
been at 40GW for Rooftop Solar Plants. matter in the absence of oxygen from raw materials such
as agricultural waste, cattle dung, poultry droppings,
municipal waste, plant material, sewage, green waste or
food/kitchen waste.
Biogas has a calorific value of about 5000 kcal per m3.
The digested slurry produced from Biogas Plants as a by-
product is a better source of nutrient enriched organic
manure for use in Agriculture. It not only helps in
improving the crop yield but also maintains soil health.
There is ample potential of setting up biogas plants
considering the livestock population of 512.06 million,
which includes about 300 million (299.98 million)
total population of bovines (comprising cattle, buffalo,
mithun and yak). The livestock sector contributes about
significantly to India’s GDP and will continue to increase.
The dissemination of biogas technology is a boon for
Indian farmers with its direct and collateral benefits.
3) Combustion, Gasification and Pyrolysis - The thermo
chemical processes for conversion of biomass to useful
95. Solution (d) products involve combustion, gasification or pyrolysis.
Exp) Option d is correct. The most commonly used route is combustion. The
advantage is that the technology used is similar to that of
Biomass energy is energy generated or produced by living a thermal plant based on coal, except for the boiler. The
or once-living organisms. The most common biomass cycle used is the conventional rankine cycle with biomass
materials used for energy are plants, such as corn and soy. being burnt in a high-pressure boiler to generate steam
The energy from these organisms can be burned to create and operating a turbine with the generated steam. The
heat or converted into electricity. exhaust of the steam turbine can either be fully condensed
Biomass based energy can be extracted through - to produce power, or used partly or fully for another useful

Workbook 258
.
ENVIRONMENT

heating activity. The latter mode is called cogeneration. In inefficient as a result electricity dissipates in transit. A
India, the cogeneration route finds application mainly microgrid overcomes this inefficiency by generating
in industries. Hence, Options 3 and 4 are correct. power close to those it serves as a result it decreases the
AT&C losses.
96. Solution (c)
Option c is correct. A microgrid can be disconnected
Exp) Option c is correct. from the central grid and operate independently. This
Statement 1 is correct. Compressed Biogas (CBG) is islanding capability allows them to supply power to their
a purified form of biogas which can be produced from customers when a storm or other calamity causes an
waste including municipal solid waste, sludge from outage on the power grid.
wastewater treatment plants, market residues, agricultural
residues, cattle dung, sugarcane press mud among others. Option d is correct. A key difference from a distributed
energy system is that a microgrid will keep the power
Statement 2 is correct. Chemically, CBG is the same as flowing when the central grid fails; a solar panel
Compressed Natural Gas (CNG) — both are compressed alone will not. Simple back-up generators also are
methane — and have the same calorific value. However, not microgrids. Such systems are only employed in
while CNG is a by-product of petroleum, CBG can be
emergencies on the other hand microgrids operate
produced from any biomass.
continuously managing and supplying energy.
Statement 3 is incorrect. CBG is a commercially
viable option as it can be directly used to replace CNG 98. Solution (d)
in transportation fuel. Just like CNG, CBG too can be Exp) Option d is correct.
transported through cylinders or pipelines to retail
outlets. Statement 1 is correct. Net metering is a billing
mechanism which allows domestic or commercial users
Statement 4 is correct. Sustainable Alternative to who generate their own electricity using solar panels or
Affordable Transport (SATAT) initiative was launched
photovoltaic systems to export their surplus energy back
in 2018 to promote the Compressed Biogas (CBG)
to the grid. If the home is net-metered, the electricity
technology. The scheme targeted production of 15 million
meter will run backwards to provide a credit against
metric tonnes (MMT) of CBG by 2023.
what electricity is consumed at night or other periods
97. Solution (b) when the home’s electricity use exceeds the system’s
Exp) Option b is correct output.
Statement 2 is correct. An off-grid system is generally a
stand-alone system, while an on-grid system is connected
to the main utility grid and incorporates the policy of net
metering.
Statement 3 is correct. Unlike off-grid PV systems,
owners don’t require battery storage systems while
using net metering since the extra power is stored by
the utility grid itself. It also eliminates the need for
exorbitant generators since the electricity grid also acts
as a backup generator.

Option a is correct. A microgrid is a self-sufficient


energy system that serves a discrete geographic footprint,
such as a college campus, hospital complex, business
centre, or neighbourhood. There are one or more kinds of
distributed energy sources (solar panels, wind turbines,
combined heat & power, generators) that produce its
power.
Option b is incorrect. Central grids push electricity
from power plants over long distances via transmission
and distribution lines. Delivering power from afar is

259 Workbook
.
ENVIRONMENT

ENVIRONMENT
BIODIVERSITY
(a) 1, 2 and 6
5.1. Previous Years Questions (b) 2, 4 and 6
1. Consider the following statements: (c) 1, 3 and 5
[UPSC CSE Pre. 2011] (d) 3, 4 and 6
1. Biodiversity is normally greater in the lower 5. The Himalayan Range is very rich in species
latitudes as compared to the higher latitudes. diversity. Which one among the following is the
2. Along the mountain gradients, biodiversity most appropriate reason for this phenomenon?
is normally greater in the lower altitudes as [UPSC CSE Pre. 2011]
compared to the higher altitudes. (a) It has high rainfall that supports luxuriant
Which of the statements given above is/are correct? vegetative growth
(a) 1 only (b) It is a confluence of different bio-geographical
zones
(b) 2 only
(c) Exotic and invasive species have not been
(c) Both 1 and 2
introduced in this region
(d) Neither 1 nor 2
(d) It has less human interference
2. Biodiversity forms the basis for human existence
6. Which one of the following groups of animals
in the following ways: [UPSC CSE Pre. 2011]
belongs to the category of endangered species?
1. Soil formation [UPSC CSE Pre. 2012]
2. Prevention of soil erosion (a) Great Indian Bustard, Musk Deer, Red Panda
3. Recycling of waste and Asiatic Wild Ass
4. Pollination of crops (b) Kashmir Stag, Cheetal, Blue Bull and Great
Select the correct answer using the codes given Indian Bustard
below: (c) Snow Leopard, Swamp Deer, Rhesus Monkey
and Saras (Crane)
(a) 1, 2 and 3 only
(d) Lion-tailed Macaque, Blue Bull, Hanuman
(b) 2, 3 and 4 only
Langur and Cheetal
(c) 1 and 4 only
(d) 1, 2 3 and 4 7. What is the difference between the antelopes Oryx
and Chiru?
3. A sandy and saline area is the natural habitat of an [UPSC CSE Pre. 2012]
Indian animal species. The animal has no predators
in that area but its existence is threatened due to (a) Oryx is adapted to live in hot and arid areas
the destruction of its habitat. Which one of the whereas Chiru is adapted to live in steppes and
semi-desert areas of cold high mountains.
following could be that animal?
(b) Oryx is poached for its antlers whereas Chiru is
[UPSC CSE Pre. 2011]
poached for its musk.
(a) Indian wild buffalo (c) Oryx exists in western India only whereas Chiru
(b) Indian wild ass exists in north-east India only.
(c) Indian wild boar (d) None of the statements (a), (b) and (c) given
(d) Indian gazelle above is correct.
4. Three of the following criteria have contributed to 8. Which of the following can be threats to the
the recognition of Western Ghats-Sri Lanka and biodiversity of a geographical area?
Indo-Burma regions as hotspots of biodiversity: [UPSC CSE Pre. 2012]
[UPSC CSE Pre. 2011] 1. Global warming
1. Species richness 2. Fragmentation of habitat
2. Vegetation density 3. Invasion of alien species
3. Endemism 4. Promotion of vegetarianism
4. Ethno-botanical importance
Select the correct answer using the codes given
5. Threat perception below:
6. Adaptation of flora and fauna to warm and (a) 1, 2 and 3 only
humid conditions (b) 2 and 3 only
Which three of the above are correct criteria in this (c) 1 and 4 only
context? (d) 1, 2, 3 and 4

Workbook 260
.
ENVIRONMENT

9. Consider the following: [UPSC CSE Pre. 2012] Which of the above is/are endangered?
1. Black-necked crane (a) 1 and 2 only
2. Cheetah (b) 3 only
3. Flying squirrel (c) 1, 2 and 3
4. Snow leopard (d) None
Which of the above are naturally found in India? 14. Consider the following: [UPSC CSE Pre. 2013]
(a) 1, 2 and 3 only 1. Star tortoise
(b) 1, 3 and 4 only 2. Monitor lizard
(c) 2 and 4 only 3. Pygmy hog
(d) 1, 2, 3 and 4 4. Spider monkey
10. What would happen if phytoplankton of an ocean Which of the above are naturally found in India?
is completely destroyed for some reason? (a) 1, 2 and 3 only
[UPSC CSE Pre. 2012] (b) 2 and 3 only
1. The ocean as a carbon sink would be adversely (c) 1 and 4 only
affected. (d) 1, 2, 3 and 4
2. The food chains in the ocean would be adversely
affected. 15. Consider the following animals:
[UPSC CSE Pre. 2013]
3. The density of ocean water would drastically
decrease. 1. Sea cow
2. Sea horse
Select the correct answer using the codes given
below: 3. Sea lion
(a) 1 and 2 only Which of the above is/are mammal/mammals?
(b) 2 only (a) 1 only
(c) 3 only (b) 1 and 3 only
(d) 1, 2 and 3 (c) 2 and 3 only
(d) 1, 2 and 3
11. Which of the following can be threats to the
biodiversity of a geographical area? 16. In which of the following States is lion-tailed
[UPSC CSE Pre. 2012] macaque found in its natural habitat?
[UPSC CSE Pre. 2013]
1. Global warming
1. Tamil Nadu
2. Fragmentation of habitat
2. Kerala
3. Invasion of alien species
3. Karnataka
4. Promotion of vegetarianism
4. Andhra Pradesh
Select the correct answer using the codes given
below: Select the correct answer using the codes given
below:
(a) 1, 2 and 3 only
(b) 2 and 3 only (a) 1, 2 and 3 only
(c) 1 and 4 only (b) 2 only
(d) 1, 2, 3 and 4 (c) 1, 3 and 4 only
(d) 1, 2, 3 and 4
12. Consider the following kinds of organisms:
17. Other than poaching, what are the possible reasons
[UPSC CSE Pre. 2012] for the decline in the population of Ganges River
1. Bat Dolphins? [UPSC CSE Pre. 2014]
2. Bee 1. Construction of dams and barrages on rivers
3. Bird 2. Increase in the population of crocodiles in rivers.
Which of the above is/are pollinating agent/agents? 3. Getting trapped in fishing nets accidentally
(a) 1 and 2 only 4. Use of synthetic fertilizers and other agricultural
(b) 2 only chemicals in crop-fields in the vicinity of rivers.
(c) 1 and 3 only Select the correct answer using the code given below.
(d) 1, 2 and 3
(a) 1 and 2 only
13. Consider the following fauna of India: (b) 2 and 3 only
[UPSC CSE Pre. 2013] (c) 1, 3 and 4 only
1. Gharial (d) 1, 2, 3 and 4
2. Leatherback turtle 18. If you walk through countryside, you are likely
3. Swamp deer to see some birds stalking alongside the cattle to

261 Workbook
.
ENVIRONMENT

seize the insects disturbed by their movement 24. With reference to ‘Red Sanders’, sometimes seen in
through grasses. Which one of the following is/are the news, consider the following statements:
such bird/birds? [UPSC CSE Pre. 2014] [UPSC CSE Pre. 2016]
1. Painted Stork 1. It is a tree species found in a part of South India.
2. Common Myna 2. It is one of the most important trees in the
3. Black-necked Crane tropical rain forest areas of South India.
Select the correct answer using the code given below. Which of the statements given above is/are correct?
(a) 1 and 2 only (a) 1 only
(b) 2 only (b) 2 only
(c) 2 and 3 only (c) Both 1 and 2
(d) 3 only (d) Neither 1 nor 2
19. With reference to ‘dugong’, a mammal found in 25. If you want to see gharials in their natural habitat,
India, which of the following statements is/are which one of the following is the best place to
correct? [UPSC CSE Pre. 2015] visit? [UPSC CSE Pre. 2017]
1. It is a herbivorous marine animal. (a) Bhitarkanika Mangroves
2. It is found along the entire coast of India. (b) Chambal River
3. It is given legal protection under Schedule I of (c) Pulicat Lake
the Wildlife (Protection) Act, 1972. (d) Deepor Beel
Select the correct answer using the codes given
below.
26. According to the Wildlife (Protection) Act,
(a) 1 and 2 only
1972, which of the following animals cannot
(b) 2 only be hunted by any person except under some
(c) 1 and 3 only provisions provided by law?
(d) 3 only [UPSC CSE Pre. 2017]
20. Which one of the following is the national aquatic 1. Gharial
animal of India? [UPSC CSE Pre. 2015] 2. Indian wild ass
(a) Saltwater crocodile 3. Wild buffalo
(b) Olive ridley turtle Select the correct answer using the code given below:
(c) Gangetic dolphin (a) 1 only
(d) Gharial (b) 2 and 3 only
21. Which one of the following regions of India has a (c) 1 and 3 only
combination of mangrove forest, evergreen forest (d) 1, 2 and 3
and deciduous forest? [UPSC CSE Pre. 2015]
27. In the context of solving pollution problems,
(a) Coastal Andhra Pradesh
(b) South-West Bengal what is/are the advantage/advantages of
(c) Southern Saurashtra bioremediation technique?
(d) Andaman and Nicobar Islands [UPSC CSE Pre. 2017]
1. It is a technique for cleaning up pollution by
22. In which of the following regions of India are
enhancing the same biodegradation process that
you most likely to come across the ‘Great Indian
occurs in nature.
Hornbill’ in its natural habitat?
[UPSC CSE Pre. 2016] 2. Any contaminant with heavy metals such
(a) Sand deserts of north-west India as cadmium and lead can be readily and
(b) Higher Himalayas of Jammu and Kashmir completely treated by bioremediation using
(c) Salt marshes of western Gujarat microorganisms.
(d) Western Ghats 3. Genetic engineering can be used to create
microorganisms specifically designed for
23. What is/are unique about ‘Kharai camel’, a breed bioremediation.
found in India? [UPSC CSE Pre. 2016]
Select the correct answer using the code given below:
1. It is capable of swimming up to three kilometres
in seawater. (a) 1 only
(b) 2 and 3 only
2. It survives by grazing on mangroves.
(c) 1 and 3 only
3. It lives in the wild and cannot be domesticated. (d) 1, 2 and 3
Select the correct answer using the code given below.
(a) 1 and 2 only 28. Consider the following statements:
(b) 3 only [UPSC CSE Pre. 2018]
(c) 1 and 3 only 1. The definition of “Critical Wildlife Habitat” is
(d) 1, 2 and 3 incorporated in the Forest Rights Act, 2006.

Workbook 262
.
ENVIRONMENT

2. For the first time in India, Baigas have been With reference to the States mentioned above, in
given Habitat Rights. terms of percentage of forest cover to the total area
3. Union Ministry of Environment, Forest and of State, which one of the following is the correct
Climate Change officially decides and declares ascending order?
Habitat Rights for Primitive and Vulnerable (a) 2-3-1-4
Tribal Groups in any part of India. (b) 2-3-4-1
(c) 3-2-4-1
Which of the statements given above is/are correct?
(d) 3-2-1-4
(a) 1 and 2 only
(b) 2 and 3 only 33. As per the Solid Waste Management Rules,
(c) 3 only 2016 in India, which one of the following
(d) 1, 2 and 3 statements is correct? [UPSC CSE Pre. 2019]
(a) Waste generator has to segregate waste five
29. Why is a plant called Prosopis juliflora often categories.
mentioned in news? [UPSC CSE Pre. 2018] (b) The Rules are applicable to notified urban
(a) Its extract is widely used in cosmetics. local bodies, notified towns and all industrial
(b) It tends to reduce the biodiversity in the area in townships only.
which it grows. (c) The Rules provide for exact and elaborate criteria
(c) Its extract is used in the synthesis of pesticides. for the identification of sites for landfills and
(d) None of these waste processing facilities.
(d) It is mandatory the part of waste generator that
30. Consider the following statements: the waste generated in one district cannot be
[UPSC CSE Pre. 2019] moved to another district.
1. Asiatic lion is naturally found in India only.
34. Consider the following statements:
2. Double-humped camel is naturally found in [UPSC CSE Pre. 2019]
India only.
The Environment Protection Act, 1986 empowers
3. One-horned rhinoceros is naturally found in the Government of India to
India only. 1. state the requirement of public participation in
Which of the statements given above is/are correct? the process of environmental protection, and the
(a) 1 only procedure and manner in which it is sought
(b) 2 only 2. lay down the standards for emission or discharge
(c) 1 and 3 only of environmental pollutants from various
sources
(d) 1, 2 and 3
Which of the statements given above is/are correct?
31. Consider the following pairs: (a) 1 only
[UPSC CSE Pre. 2019] (b) 2 only
(c) Both 1 and 2
Wildlife Naturally found in (d) Neither 1 nor 2
1. Blue-finned Cauvery River
Mahseer 35. Consider the following statements:
[UPSC CSE Pre. 2019]
2. Irrawaddy Dolphin Chambal River 1. As per recent amendment to the Indian Forest
3. Rusty-spotted Cat Eastern Ghats Act, 1927, forest dwellers have the right to fell
Which of the pairs given above are correctly the bamboos grown on forest areas.
matched? 2. As per the Scheduled Tribes and Other
(a) 1 and 2 only Traditional Forest Dwellers (Recognition of
Forest Rights) Act, 2006, bamboo is a minor
(b) 2 and 3 only
forest produce.
(c) 1 and 3 only
3. The Scheduled Tribes and Other Traditional
(d) 1, 2 and 3 Forest Dwellers (Recognition of Forests Rights)
32. Consider the following states: Act, 2006 allows ownership of minor forest
[UPSC CSE Pre. 2019] produce to forest dwellers.
1. Chhattisgarh Which of the statements given above is/are correct?
(a) 1 and 2 only
2. Madhya Pradesh
(b) 2 and 3 only
3. Maharashtra (c) 3 only
4. Odisha (d) 1, 2 and 3

263 Workbook
.
ENVIRONMENT

36. Consider the following statements: (d) Such a plant is invasive and harmful to the
[UPSC CSE Pre. 2019] ecosystem.
1. As per law, the Compensatory Afforestation
41. With reference to India’s biodiversity, Ceylon
Fund Management and Planning Authority
frogmouth, Coppersmith barbet, Gray -chinned
exists at both National and State levels.
minivet and White -throated redstart are
2. People’s participation is mandatory in the [UPSC CSE Pre. 2020]
compensatory afforestation programmes carried
out under the Compensatory Afforestation Fund (a) Birds
Act, 2016. (b) Primates
(c) Reptiles
Which of the statements given above is/are correct? (d) Amphibians
(a) 1 only
(b) 2 only 42. “If rainforests and tropical forests are the lungs
(c) Both 1 and 2 of the Earth, then surely wetlands function as its
(d) Neither 1 nor 2 kidneys.” Which one of the following functions of
wetlands best reflects the above statement?
37. In India, ‘extended producer responsibility’ [UPSC CSE Pre. 2022]
was introduced as an important feature in (a) The water cycle in wetlands involves surface
which of the following? [UPSC CSE Pre. 2019] runoff, subsoil percolation and evaporation.
(a) The Bio -medical Waste (Management and (b) Algae form the nutrient base upon which
Handling) Rules, 1998 fish, crustaceans, molluscs, birds, reptiles and
(b) The Recycled Plastic (Manufacturing and Usage) mammals thrive.
Rules, 1999 (c) Wetlands play a vital role in maintaining
(c) The e -Waste (Management and Handling) sedimentation balance and soil stabilization.
Rules, 2011 (d) Aquatic plants absorb heavy metals and excess
(d) The Food Safety and Standard Regulations, 2011 nutrients.
38. With reference to Indian elephants, consider the 43. With reference to “Gucchi” sometimes mentioned
following statements: [UPSC CSE Pre. 2020] in the news, consider the following statements:
1. The leader of an elephant group is a female. [UPSC CSE Pre. 2022]
2. The maximum gestation period can be 22 1. It is a fungus.
months. 2. It grows in some Himalayan Forest areas.
3. An elephant can normally go on calving till the 3. It is commercially cultivated in the Himalayan
age of 40 years only. foothills of north-eastern India.
4. Among the States in India, the highest elephant Which of the statements given above is/are correct?
population is in Kerala. (a) 1 only
Which of the statements given above is/are correct? (b) 3 only
(a) 1 and 2 only (c) 1 and 2
(b) 2 and 4 only (d) 2 and 3
(c) 3 only 44. Which of the following is not a bird?
(d) 1, 3 and 4 only [UPSC CSE Pre. 2022]
39. Which one of the following protected areas is well- (a) Golden Mahseer
known for the conservation of a sub-species of the (b) Indian Nightjar
Indian swamp deer (Barasinga) that thrives well in (c) Spoonbill
hard ground and is exclusively graminivorous? (d) White Ibis
[UPSC CSE Pre. 2020] 45. The “Miyawaki method” is well known for the:
(a) Kanha National Park [UPSC CSE Pre. 2022]
(b) Manas National Park (a) Promotion of commercial farming in arid and
(c) Mudumalai Wildlife Sanctuary semi-arid areas
(d) Tal Chhapar Wildlife Sanctuary (b) Development of gardens Using genetically
40. If a particular plant species is placed under modified flora
Schedule VI of The Wildlife Protection Act, 1972, (c) Creation of mini forests in urban areas
what is the implication? [UPSC CSE Pre. 2020] (d) Harvesting wind energy on coastal areas and on
sea surfaces
(a) A licence is required to cultivate that plant.
(b) Such a plant cannot be cultivated under any 46. Certain species of which one of the following
circumstances. organisms are well known as cultivators of fungi?
(c) It is a Genetically Modified crop plant. [UPSC CSE Pre. 2022]

Workbook 264
.
ENVIRONMENT

(a) Ant (a) Alpha diversity


(b) Cockroach (b) Beta diversity
(c) Crab (c) Gamma diversity
(d) Spider (d) Species evenness
Practice Question 52. Which of the following are the traits that increase
a species vulnerability to extinction?
5.2. Biodiversity in India 1. Poor Dispersal Ability
2. Ecological Specialization
47. If you want to see Seabuckthorn species in their
natural habitat, which of the following areas you 3. High Trophic Status
will have to visit? 4. Temperature Dependent Sex Determination
(a) Areas above the tree line in the Himalayan Select the correct answer using the code given below:
region, like the cold deserts of Ladakh and Spiti. (a) 2 and 3 only
(b) In the tropical evergreen forests of the Andaman (b) 2, 3 and 4 only
and Nicobar Islands and the Western Ghats. (c) 1, 2 and 3 only
(c) Banni grasslands in the salt marsh of the Rann (d) 1, 2, 3 and 4
od Kutch in Gujarat.
(d) Arid and semi-arid regions of western Rajasthan 53. Consider the following statements in reference to
and Thar desert. keystone species:
48. Mycorrhiza is a symbiotic association between: 1. Extinction of a keystone species results in the
degradation of whole ecosystem.
(a) Plants and fungi.
(b) Fungi and bacteria. 2. Top predators like tiger are considered keystone
(c) Legumes and rhizobia. species.
(d) Algae and bacteria. 3. Herbivores cannot be keystone species.
49. Consider the following statements with respect to Which of the statements given above is/are correct?
the biomes of India: (a) 1 and 2 only
1. The Terai–Duar savanna is a narrow lowland (b) 1 and 3 only
ecoregion at the base of the Himalayas. (c) 2 and 3 only
2. Thar desert is an example of hot and dry desert. (d) 1, 2 and 3
3. The coral reef biome of India includes only soft 54. Consider the following statements
corals. 1. A poikilotherm is an animal which is unable to
4. The Tropical Dry Evergreen Forest is majorly regulate its body temperature in accordance to
found in Tamil Nadu and Andhra Pradesh. the temperature of their surroundings.
Which of the statements given above is/are correct? 2. Although vertebrates are small in portion yet
(a) 1 and 2 only they dominate their environment due to their
(b) 1, 2 and 4 only size and mobility.
(c) 1, 3 and 4 only 3. Monotremes is a sub-division of mammals
(d) 1, 2, 3 and 4 which lay eggs rather than giving birth.
50. Consider the following sentences about Species Which of the statements given above is/are correct?
diversity: (a) 1 and 2 only
1. It refers to the variety of living organisms on (b) 1 and 3 only
earth. (c) 2 and 3 only
2. It is the ratio of one species population over the (d) 1, 2 and 3
total number of organisms across all species in
the given biome. 55. Which of the following statements is/are correct
Which of the statements given above is/are correct? about Marsupials?
(a) 1 only 1. Placenta is absent in Marsupial mammals.
(b) 2 only 2. Unlike placental mammals their gestation
(c) Both 1 and 2 period is very small.
(d) Neither 1 nor 2 Select the correct answer using the code given below:
51. It is a comparison of diversity between ecosystems, (a) 1 only
usually measured as the change in number of (b) 2 only
species between the ecosystems. This diversity is (c) Both 1 and 2
known as (d) Neither 1 nor 2

265 Workbook
.
ENVIRONMENT

56. Consider the following statements: 60. Consider the following statements regarding
1. A lichen is a combination of a bacteria and a millets:
fungus. 1. Millet cultivation can help in carbon
2. Lichens are capable of initiating primary sequestration.
succession. 2. They are generally not acid-forming foods
3. Lichens are indicators of air quality and are making them easy to digest.
sensitive to sulphur-dioxide pollution. 3. Millets are thermophilic crops.
Which of the statements given above is/are correct? Which of the statements given above are correct?
(a) 1 and 2 only (a) 1 and 2 only
(b) 1 and 3 only (b) 1 and 3 only
(c) 2 and 3 only (c) 2 and 3 only
(d) 1, 2 and 3 (d) 1, 2 and 3
57. Consider the following pairs: 61. With reference to the invasive species in India,
consider the following:
Type of Niche Feature
1. An invasive alien species is a problematic species
1. Food niche Competition with introduced outside its natural past or present
species for food distribution.
2. Physical niche Reproduction 2. Water Hyacinth is an invasive species native to
Amazon basin that populates ponds in India.
3. Habitat niche Land shape
3. Senna spectabilis has become a major threat to
Which of the pairs given above is/are correctly the Shola grasslands ecosystem in the Nilgiris.
matched?
4. Cartagena Protocol on Biosafety focuses to
(a) 1 only
address the issue of invasive species.
(b) 1 and 2 only
(c) 2 and 3 only Which of the statements given above is/are correct?
(d) 1, 2 and 3 (a) 1 and 2 only
(b) 2, 3 and 4 only
58. A biogeographic realm is a large spatial region (c) 1 and 3 only
within which the ecosystem shares a broadly (d) 1, 2, 3 and 4
similar biota. With respect to India, which of the
following realms is/are present? 62. Which of the following factors is/are responsible
1. Nearctic Realm for greater biological diversity in tropical regions
2. Palearctic Realm as compared to temperate regions?
3. Neotropical Realm 1. Tropical latitudes have remained undisturbed
4. Malayan Realm for millions of years in comparison to temperate
regions.
Select the correct answer using the code given below:
2. Availability of more solar energy in tropical
(a) 1 and 3 only regions.
(b) 1 and 4 only
3. Frequent and major seasonal variations in
(c) 2 and 4 only
tropical regions.
(d) 3 and 4 only
Select the correct answer using the code given below:
59. With reference to rules for import of exotic species (a) 1 and 2 only
in India, which of the following statements is/are (b) 1 and 3 only
incorrect? (c) 2 and 3 only
1. Exotic species are defined under the schedules of (d) 1, 2 and 3
Wildlife Protection Act, 1972.
2. A person who wants to import a live exotic 63. Consider the following:
animal will have to submit an application for 1. Bio-Diversity Conservation
license to National Biodiversity Authority. 2. Ganga Gram Yojana
3. The importer will have to attach a no objection 3. Industrial Effluent Monitoring
certificate of chief wildlife warden of the 4. River-Front Development
concerned state with the application. Which of the above are initiatives under the Namami
Select the correct answer using the code given below: Gange Programme?
(a) 1 only (a) 2 and 3 only
(b) 1 and 2 only (b) 1 and 3 only
(c) 2 and 3 only (c) 1, 2 and 4 only
(d) 1, 2 and 3 (d) 1, 2, 3 and 4

Workbook 266
.
ENVIRONMENT

64. With reference to the Vavilovian Centres, consider 68. With reference to the differences between the
the following statements: Tundra and Taiga biomes, consider the following
1. These are geographical areas where a group statements:
of organisms is said to have first developed its 1. Tundra is found in higher latitudes compared to
distinctive traits. Taiga biomes.
2. India is one of the main centres of Vavilovian 2. Most of the animals of the Tundra biomes have
Diversity. shorter lifespan than those in Taiga.
3. The productivity of Taiga biomes is more than
Which of the statements given above is/ are correct?
that of Tundra.
(a) 1 only 4. While Tundra lacks trees, Taiga biome supports
(b) 2 only large belt of softwood trees.
(c) Both 1 and 2
Which of the statements given above are correct?
(d) Neither 1 nor 2
(a) 1, 2 and 3 only
65. Consider the following statements: (b) 2, 3 and 4 only
Statement 1: Tropics account for a high biological (c) 1, 3 and 4 only
diversity. (d) 1, 2, 3 and 4
Statement 2: Tropical areas have undergone frequent 69. Consider the following statements:
glaciation in the past. 1. Biome can be exclusively differentiated on the
Which one of the following is correct in respect of basis of the area in which specific population or
the above statements? species resides.
2. The amount of insolation and precipitation are
(a) Both the Statement 1 and Statement 2 are correct
the main factors governing the distribution of
and Statement 2 is the correct explanation for biome on our planet.
statement 1
(b) Both statement 1 and Statement 2 are correct but Which of the statements given above is/are correct?
Statement 2 is not the correct explanation for (a) 1 only
Statement 1 (b) 2 only
(c) Statement 1 is correct But Statement 2 is not (c) Both 1 and 2
correct (d) Neither 1 nor 2
(d) Statement 1 is not correct but Statement 2 is 70. Consider the following pairs:
correct
Major World Biomes Dominant Vegetations
66. With reference to the mass extinctions, consider found in them
the following statements:
1. Coniferous Forests Moss and lichens
1. More than 99 percent of all organisms that have and Taiga
ever lived on Earth are extinct.
2. Climate change has been the single biggest 2. Temperate Teak and Sal
driver of mass extinctions. deciduous forest
3. The dinosaurs went extinct in the most recent 3. Tropical rain forest Epiphytes and liana
mass extinction caused by the asteroid impact. Which of the pairs given above is/are correctly
Which of the statements given above is/are correct? matched?
(a) 1 and 3 only (a) 1 only
(b) 2 only (b) 1 and 3 only
(c) 2 and 3 only (c) 2 and 3 only
(d) 1, 2 and 3 (d) 2 only
71. How is Steppes Grassland different from Savanna
67. Consider the following statements:
Grassland?
1. Primary productivity depends on the nature of
1. Steppes Grassland lie closer to the equator than
the plant species, availability of nutrients and Savanna Grassland.
photosynthetic capacity of plants.
2. Unlike Savanna Grassland, Steppe’s regions are
2. The primary productivity in oceans is much treeless with short grasses.
more than primary productivity of land.
Select the correct answer using the codes given
Which of the statements given above is/are correct? below:
(a) 1 only (a) 1 only
(b) 2 only (b) 2 only
(c) Both 1 and 2 (c) Both 1 and 2
(d) Neither 1 or 2 (d) Neither 1 nor 2

267 Workbook
.
ENVIRONMENT

72. Which of the following can be attributed as (a) 1 and 2 only


primarily the natural causes of biodiversity loss? (b) 2 only
1. Lack of pollination (c) 2 and 3 only
2. Rivalry among species (d) 1 and 3 only
3. Habitat destruction 76. Which of the following factors is/are responsible
4. Extension of agriculture for decreasing amount of Dissolved Oxygen (DO)
Select the correct answer using the code given below: in aquatic water bodies?
(a) 1, 3 and 4 only 1. Decreasing salinity of the water body
(b) 1 and 2 only 2. Warmer surface water
(c) 2, 3 and 4 only 3. Increasing atmospheric pressure
(d) 1, 2, 3 and 4
Select the correct answer using the codes given
73. Four major categories of ecosystem services are below:
provisioning, regulating, cultural and supporting (a) 1 only
services. Which of the following is/are the (b) 2 and 3 only
regulating services provided by the ecosystem? (c) 3 only
1. Trees provide shade to humans and other (d) 1 and 3 only
species.
77. Consider the following pairs:
2. Ecosystems provide living spaces for plants and
animals.
3. Prevention of soil erosion and maintaining of Lakes Description
soil fertility. 1. Pulicat lake freshwater lake
4. Pollination service provided by insect and bats. 2. Kanwar Lake oxbow lake
5. Maintaining a genetic diversity of plants and
animals. 3. Vembanad lake Lagoon lake
Select the correct answer using the code given below: Which of the pairs given above is/are correctly
matched?
(a) 1, 2 and 4 only
(b) 2, 3 and 5 only (a) 1 and 2 only
(c) 1, 3 and 4 only (b) 2 and 3 only
(d) 1, 2, 3, 4 and 5 (c) 3 only
(d) 1 and 3 only
74. Consider the following statements regarding
‘Desert Ecosystem’: 78. With reference to the Banni Grasslands, consider
1. Mammals as a group are poorly adapted for the following statements:
survival in Desert region across the world. 1. It is the largest natural grassland in the Indian
2. Some desert insects acquire water through subcontinent.
condensation on specialized hairs or body parts. 2. Mankadia tribe is found to inhabit these
3. Desert plants have adapted with spines and fine grasslands.
hairs to reflect heat and reduce the air flow over 3. The grasslands are under threat from Prosopis
the plant’s surface. juliflora, a non-native invasive tree.
Which of the statements given above is/are correct? Which of the statement given above is/are correct?
(a) 1 and 2 only (a) 1 only
(b) 2 only (b) 2 and 3 only
(c) 2 and 3 only (c) 1 and 3 only
(d) 1 and 3 only (d) 2 only
75. Consider the following statements: 79. Which of the following statements is the most
1. Both phytoplanktons and zooplanktons are appropriate definition of the term ‘endemic
capable of moving freely and actively with water species’?
currents. (a) They are those species which are found
2. Phytoplanktons are found on the surface of water exclusively in a particular area.
bodies, while Zooplanktons remain around the (b) They are species whose numbers are diminishing
dark and deeper areas of water. to a level that they might face extinction.
3. Phytoplanktons are photosynthetic, whereas (c) These species are not the natural inhabitants of a
Zooplanktons only take in oxygen and do not region but are introduced from outside.
produce it. (d) These species are likely to become extinct if the
Which of the statements given above is/are correct? factors threatening to their extinction continue.

Workbook 268
.
ENVIRONMENT

80. With reference to the Genetic Diversity, consider 84. This species is generally found at high altitude
the following statements: areas above 3000 meters in India. They are
1. It ensures that the species are able to adapt to facing a threat of habitat loss and poaching. To
changing environmental factors. conserve their habitat Secure Himalaya project
2. Low genetic diversity provides ability to the was launched with the support of UNDP. They are
species to resist emerging diseases. known to be found in Great Himalayan National
Park.
3. Different breeds of dogs is an example of genetic
diversity. Which of the following species has been described in
above paragraph?
Which of the statements given above is/are correct?
(a) Red Panda
(a) 1 and 2 only (b) Hangul
(b) 2 and 3 only (c) Snow Leopard
(c) 1 and 3 only (d) Kondana Rat
(d) 1, 2 and 3
85. With reference to the Himalayan Brown Bear,
5.3. Animal diversity in india consider the following:
1. It is a critically endangered species of bear found
81. With reference to ‘Black Browed Babbler’ species, only in India.
recently seen in news, consider the following 2. Its habitat include high altitude open valleys and
statements: pastures.
1. It is a songbird species mainly found in 3. It is the only beer species that hibernates in the
Indonesia. summers and not the winters.
2. It is categorised as critically endangered by the Which of the statements given above is/are correct?
International Union for Conservation of Nature. (a) 1 only
3. In India, it can be sighted in the state of (b) 2 only
Arunachal Pradesh. (c) 1 and 2 only
Which of the statements given above is/are correct? (d) 3 only
(a) 1 only 86. Consider the following statements with reference
(b) 1 and 2 only to Great Indian Bustard:
(c) 2 and 3 only 1. Its habitat is dry grasslands and scrublands.
(d) 1, 2 and 3
2. Its IUCN status is Critically Endangered.
82. Consider the following statements with reference 3. It is endemic to Indian subcontinent.
to Monotremes: Select the correct answer using the code given below:
1. They are the only egg-laying mammals. (a) 1 and 2 only
2. They are indigenous to Australia and New (b) 2 and 3 only
Guinea. (c) 1, 2 and 3
3. They have a slightly lower body temperature (d) 1 and 3 only
compared to ordinary mammals.
87. Consider the following pairs:
Which of the statements given above is/are correct?
(a) 1 only Animal Conservation Status
(b) 1 and 2 only 1. Ganges Dolphin Endangered
(c) 3 only
(d) 1, 2 and 3 2. Malabar Civet Critically Endangered

83. Consider the following pairs: 3. Snow Leopard Endangered


4. Red Panda Vulnerable
Vulture species IUCN Status
Which of the pairs given above is/are correctly
1. Long-billed Vulture Critically-endangered matched?
2. Egyptian Vulture Near threatened (a) 1 only
(b) 1 and 2 only
3. Slender-billed Endangered (c) 2, 3 and 4 only
Vulture (d) 1, 2 and 3 only
Which of the pairs given above is/are correctly
matched? 88. Consider the following statements:
(a) 1 only 1. Pygmy hog is an indicator species inhabiting
(b) 2 and 3 only grassland habitats.
(c) 2 only 2. Namdapha flying squirrel is restricted only to
(d) 1, 2 and 3 Barak valley in Manipur.

269 Workbook
.
ENVIRONMENT

3. Throughout its ecological history, India only 4. Leatherback


had one-horned rhinoceros out of five species of Select the correct answer using the given code below:
rhinos. (a) 1 and 4 only
Which of the statements given above is/are incorrect? (b) 2 and 3 only
(a) 1 only (c) 1, 3 and 4 only
(b) 1 and 2 only (d) 1, 2, 3 and 4
(c) 2 only 93. In which of the following regions of India are
(d) 2 and 3 only you most likely to come across the `Great Indian
89. Consider the following statements regarding a Hornbill’ in its natural habitat?
species. (a) Sand deserts of northwest India
(b) Higher Himalayas of Jammu and Kashmir
1. It is listed under schedule 1 of the wildlife
(c) Salt marshes of western Gujarat
protection act, 1972.
(d) Western Ghats
2. It is found only in India.
3. It is listed as endangered under IUCN Red list. 94. Consider the following statements:
4. India uses Block counting method to estimate its 1. It is a nocturnal bird found only in the northern
numbers. part of Andhra Pradesh in peninsular India.
2. The species was considered to be extinct until it
Select the correct species from the option given was rediscovered in 1986.
below:
3. It is naturally found in Sri Lankamalleswara
(a) Golden langur. Wildlife Sanctuary.
(b) Forest owlet
Which of the following species is described above?
(c) Bengal Tiger
(d) Asiatic lion (a) Sociable Lapwing
(b) White-bellied Heron
90. Which of the following statements is/are incorrect (c) Jerdon’s Courser
about crocodiles found in India? (d) Spoon Billed Sandpiper
1. Gharial is an indicator species of the pollution 95. With reference to the Indian Pangolin, consider
in the river. the following statements:
2. Gharial is a critically endangered species as per 1. They are distributed across high Himalayas and
the red book of IUCN. North-East India.
3. Gharial and salt water crocodile are the only 2. They are protected under Schedule I of the
species of crocodile found in India. Wildlife Protection Act, 1972.
Select the correct answer using the code given below: 3. They are hunted for consumption as a protein
(a) 1 and 2 only source.
(b) 2 and 3 only Which of the statements given above is/are correct?
(c) 3 only (a) 1 and 2 only
(d) 1, 2 and 3 (b) 2 and 3 only
(c) 3 only
91. With reference to Asian elephants which of the (d) 1, 2 and 3
following statements is/are incorrect?
1. Asian elephants has been recently given global 96. Small sized animals are rarely found in polar
protection under Convention of migratory regions because?
species (CMS). (a) They have a smaller surface area relative to their
2. MIKE Programme has been launched by CMS volume.
as an effort to conserve elephant population in (b) They have a larger volume relative to their
India. surface area.
(c) They have a larger surface area relative to their
Select the correct answer using the code given below: volume.
(a) 1 only (d) They have low metabolic rate.
(b) 2 only
(c) Both 1 and 2 97. Consider the following statements:
(d) Neither 1 nor 2 Statement 1: Koalas are examples of specialist
92. Which of the following species of turtle is/are species.
found in India? Statement 2: They can eat variety of food and thrive
1. Hawksbill in a varied range of habitats.
2. Green turtle Which one of the following is correct in respect of
3. Loggerhead the above statements?

Workbook 270
.
ENVIRONMENT

(a) Both the Statement 1 and Statement 2 are correct 102. Which of the following statement correctly
and Statement 2 is the correct explanation for explains the 10 per cent law with reference to food
statement 1 chain?
(b) Both statement 1 and Statement 2 are correct but (a) In a food chain only 10 per cent of the energy is
Statement 2 is not the correct explanation for transferred from the Sun to the lowest trophic
Statement 1 level.
(c) Statement 1 is correct but Statement 2 is not (b) In detritus food chain, only 10 per cent of the
correct energy is transferred to each trophic level from
(d) Statement 1 is not correct but Statement 2 is the lower trophic level.
correct (c) In grazing food chain, only 10 per cent of the
energy is transferred to each trophic level from
98. Which of the following is/are migratory birds that the lower trophic level.
come to India during winters? (d) In a food chain, minimum 10 per cent of the
1. Greater Flamingo energy is transferred back to the environment.
2. Lesser Florican
103. With reference to the Tropical Rainforests and
3. Western European Roller
the Temperate Deciduous Forests, consider the
4. Amur Falcon following statements:
Select the correct answer using the code given below: 1. Tropical rainforests have hardwood trees while
(a) 1, 2 and 3 only Temperate Deciduous forests have largely
(b) 1, 2 and 4 only softwood trees.
(c) 1, 3 and 4 only 2. The upper layers of soil of both biomes are
(d) 2, 3 and 4 only deficient in nutrients.
99. With reference to the adaptations of animals 3. Tropical rainforests are found in Central and
in the desert ecosystem, consider the following South America while Temperate Deciduous
statements. forests are found in north-eastern Asia.
1. Desert animals are more active in night time Which of the statements given above is/ are correct?
than day time. (a) 1 and 2 only
2. Desert animals are adapted to have long legs. (b) 2 and 3 only
(c) 1 and 3 only
3. Desert animals release their excreta in
(d) 1, 2 and 3
concentrated form.
4. Camel store water in their hump for temperature 104. With reference to the geographical spread of the
regulation. temperate grasslands across the world, consider
Which of the statements given above is/are correct? the following pairs.
(a) 1, 2 and 3 only Biome Region
(b) 2 and 3 only
(c) 1, 3 and 4 only 1. Steppes Eurasia.
(d) 1. 2, 3 and 4 2. Pampas South America.
100. If a tropical rain forest is removed, it does not 3. Veld Australia
regenerate quickly as compared to a tropical 4. Down South Africa
deciduous forest. This is because Which of the pairs given above are incorrectly
(a) The soil of rain forest is deficient in nutrients matched?
(b) Propagules of the trees in a rain forest have poor (a) 1 and 3 only
viability (b) 1, 3 and 4 only
(c) The rain forest species are slow growing (c) 2 and 4 only
(d) Exotic species invade the fertile soil of rain forest (d) 3 and 4 only
101. Which one of the following statements best 105. With reference to the Nilambur Teak, sometimes
describes the “Net Present Value of the Forest”? seen in the news, consider the following
(a) It is a tradeable certificate representing the right statements:
to emit one tonne of Carbon equivalent. 1. It is the first forest produce to get a GI tag in
(b) It is the rate of the accumulation of energy in the India.
form of biomass.
2. Nilambur teak is famous for its large size and
(c) It is a payment that a user has to make for
diverting forestland for non-forest use. durability.
(d) It is the fund used for the treatment of catchment 3. It exhibits high resistance to fungal decay.
areas of the forest. Which of the above statements is/are correct?

271 Workbook
.
ENVIRONMENT

(a) 1 only (c) 3 and 4 only


(b) 2 and 3 only (d) 1 and 4 only
(c) 1 and 3 only
110. Which of the following is not classified as a forest
(d) 1,2 and 3
produce as per Indian legislations/laws?
106. “These are the large brown algae. They are the 1. Fruits harvested outside forests
underwater forests that grow in relatively shallow 2. Silk cocoons reared outside forests
waters close to the shore. They are also considered 3. Tendu leaves collected outside forests
as keystone species and help reduce coastal 4. Timber found outside forests
erosion. They are recognized as one of the most
productive and dynamic ecosystems on Earth.” Select the correct option using the code given below:
(a) 1 and 2 only
Which of the following is the most likely referred in (b) 1 and 3 only
the above passage? (c) 1, 2 and 3 only
(a) Mangrove (d) 1, 2, 3 and 4
(b) Kelp
(c) Vallisneria 111. With respect to various medicinal plants and
(d) Sargassum the diseases they are used to treat, consider the
following pairs:
107. With reference to Mangrove Forests, which of the
following statements is/are correct? Medicinal Plants Disease Treated
1. Mangrove trees can germinate, grow and survive 1. Indian Snakeroot Anti-hypertensive
in salt water.
2. Cinchona Treating malaria
2. The dense roots of Mangroves help in protecting
shorelines from storms and cyclones. 3. Himalayan Yew Anti-cancer
3. According to India State of Forest Report 2019, 4. Indian Pitcher Plant Night blindness
area under mangroves has increased compared
Which of the pairs given above is/ are correct?
to 2017.
(a) 1, 2 and 3 only
4. The productivity of mangrove species generally (b) 2, 3 and 4 only
declines with an increase in latitude. (c) 1 and 3 only
Select the correct answers using the codes given (d) 1, 2, 3 and 4
below:
112. With respect to the effects of deforestation,
(a) 1 and 3 only consider the following statements:
(b) 1, 3 and 4 only
1. Carbon stored in tree trunks gets released
(c) 2 and 4 only
immediately upon being cut.
(d) 1, 2, 3 and 4
2. Deforestation causes a reduction in rainfall in a
108. With reference to rainforests, consider the given area.
following statements: 3. Deforestation can increase instances of flooding.
1. More than half of the world’s animals live in the Which of the statements given above is/ are correct?
rainforest areas. (a) 1 and 2 only
2. Rainforests are absent in temperate regions. (b) 2 and 3 only
3. Canopy blocks winds, rainfall, and sunlight, (c) 1 and 3 only
creating a humid and dark environment below. (d) 1, 2 and 3
Which of the statements given above is/are correct? 113. With respect to the concept of Agroforestry,
(a) 1 and 3 only consider the following statements:
(b) 2 only 1. Agroforestry refers to a practice of land
(c) 2 and 3 only management, wherein woody trees are grown
(d) 3 only alongside agricultural crops and livestock.
109. Which of the following is/ are the Artificial 2. Under agroforestry system trees and crops can
Methods of Forest Regeneration? be grown and arranged both temporally and
spatially.
1. Clear Felling System of seedling growth
3. India is the first country in the world to have
2. Manual selection and sowing of seeds officially adopted an Agroforestry Policy.
3. Manual selection and planting tree saplings
Which of the statements given above is/ are correct?
4. Coppice generation (a) 1 and 2 only
Select the correct option using the code given below: (b) 2 and 3 only
(a) 1 and 2 only (c) 1 and 3 only
(b) 2 and 3 only (d) 1, 2 and 3

Workbook 272
.
ENVIRONMENT

114. With reference to the relationship between forests 118. Consider the following pairs:
and climate change, consider the following
statements: Forest Type Type of Flora
1. All the tropical rainforests of the world have 1. Tidal Forests Sundari Tree
cumulatively stored 25% of the world’s carbon. 2. Tropical Deciduous Juniper
2. Tropical rainforests are the biggest source of Forest
oxygen in the biosphere.
3. Montane Forests khejri
3. Coastal ecosystems sequester more amount of
carbon per unit area compared to terrestrial 4. Tropical Thorn Babool
forests. Forest
Which of the statements given above is/ are correct? Which of the pairs given above is/ are correct?
(a) 1 and 2 only (a) 1 and 2 only
(b) 2 and 3 only (b) 2 and 3 only
(c) 1 and 3 only (c) 3 and 4 only
(d) 1, 2 and 3 (d) 1 and 4 only

115. Consider the following statements with reference


to Forest areas in World:
5.4. Plant diversity of India
1. India has the tenth largest forest area in the 119. Which of the following statements is incorrect
world. regarding ‘Ashwagandha’?
2. Russia has the largest forest area in the world. (a) Its distribution is limited to the Indian
Which of the above statements is/are correct? subcontinent.
(b) It grows in habitats up to an altitude of 1000 m.
(a) 1 only (c) It is used as a tranquilliser in India.
(b) 2 only (d) It has proven therapeutic effect against
(c) Both 1 and 2 COVID-19 infection.
(d) Neither 1 nor 2
120. With respect to the floral diversity in India which
116. Consider the following statements with reference of the following statement is/are correct?
to Forest cover in India according to Economic
1. The maximum diversity of fungi in India is in
survey 2020-21:
Eastern Ghats.
1. A forest cover is a land with a tree canopy density 2. Gymnosperm is a plant family which does not
of a minimum of 60 per cent. bear fruits naturally.
2. Orchards and palm plantations are not 3. Algae are non-differentiated plants which does
considered as Forest cover. not possess chlorophyll.
3. Madhya Pradesh had the largest forest cover in
Select the correct answer using the code given below
India.
(a) 1 only
4. Mizoram has the highest percentage of forest
(b) 1 and 2 only
cover with respect to total geographical area of
(c) 2 only
the state. (d) 2 and 3 only
Which of the above statements is/are correct?
121. A new species named Ischaemum janarthanamii,
(a) 2, 3 and 4 only
was recently discovered in Western Ghats. It
(b) 1 and 2 only
is known for their ecological and economic
(c) 3 and 4 only
importance, such as fodder. The species has
(d) 1, 3 and 4 only
adapted to survive harsh conditions, low nutrient
117. Which of the following species of exotic plants availability, and blossoms every monsoon. Which
were introduced by the Britishers during the species is described here?
colonial period in the regions mentioned below? (a) Giant Water Lily
1. Prosopis Juliflora in North India (b) Monkey Brush Vines
2. Eucalyptus in Nilgiris (c) Muraingrass
(d) Kapok Tree
3. Oaks in Western Himalayas
Select the correct answer using the code given below: 122. Which of the following statements is/are correct
(a) 1 and 2 only regarding Sea grass?
(b) 2 and 3 only 1. They carry out the process of photosynthesis.
(c) 1 and 3 only 2. They generally grow in deep waters.
(d) 1, 2 and 3 3. High salinity is a larger threat for sea grass beds.

273 Workbook
.
ENVIRONMENT

4. They inhabit all types of sub-stratas from mud 128. Consider the following sentences about types of
to rock roots:
Select the correct answer using the code given below: 1. Stilt roots are produced from the branches of the
(a) 1, 2 and 3 only tree which remain suspended in the air.
(b) 1 and 3 only 2. Prop roots are emerged from the butt of a tree
(c) 2 and 3 only above ground level.
(d) 1 and 4 only Which of the statements given above is/are correct?
123. With respect to insectivorous plants, which of the (a) 1 only
following statements is/are incorrect? (b) 2 only
(c) Both 1 and 2
1. These plants completely rely on insects for (d) Neither 1 nor 2
food as they are unable to produce food by
photosynthesis. 129. Consider the following pairs with reference to
2. Pitcher plants in India are confined to high different types of roots in plants:
rainfall equatorial regions of Andaman and
Root Type Feature
Nicobar Islands.
Select the correct answer using the code given below: 1. Prop-Root Produced from the
branches of the tree
(a) 1 only
(b) 2 only 2. Tap Root Spread laterally to
(c) Both 1 and 2 support the tree
(d) Neither 1 nor 2 3. Haustorial Root Roots of parasitic plants
124. With respect to invasive alien species, which of the Which of the pairs given above is/are correctly
following statements is incorrect? matched?
(a) A non-native species becomes invasive when it (a) 1 only
threatens local biodiversity. (b) 1 and 3 only
(b) They can lead to both increase and decrease in (c) 2 only
species richness in the host region. (d) 2 and 3 only
(c) In India Myna is an invasive alien species.
130. With reference to the process of ‘die back’ in plants,
(d) Nearly 50% of species in Indian flora are invasive
which of the following statements is incorrect?
alien species.
(a) It is the process of progressive dying from tip of
125. Which one of the following statements is not any portion of plant.
correct about the ‘Sea Buckthorn’ plantation? (b) It can be caused by a dense overhead canopy.
(a) It is a shrub that produces an orange-yellow (c) Due to it, both root and shoot dies within few
coloured edible berry. days.
(b) It is found in the shola forests of western ghats in (d) This is triggered as an adaptive mechanism to
Kerala and Tamil Nadu. avoid adverse conditions.
(c) It is rich in carotenoids and omega fatty acids
which can help people in high-altitude regions. 5.5. Marine organism
(d) It is used in manufacturing of cosmetics and
anti-ageing products. 131. Which of the following are the benefits of
Mangroves?
126. ‘Black wattle’ is an example of 1. They help in stabilizing the coastal shores
(a) A pest 2. They provide breeding ground for fishes
(b) Invasive alien plant 3. They protect coastal lands from tsunami and
(c) An exoplanet floods
(d) By-product from coal gasification 4. They enhance natural recycling of nutrients
127. Consider the following statements regarding the Select the correct answer using the code given below:
Himalayan trillium: (a) 1 and 2 only
1. It is a Himalayan herb, commonly used in (b) 2, 3 and 4 only
traditional medicines. (c) 1, 3 and 4 only
2. It is categorized as Vulnerable by the IUCN. (d) 1, 2, 3 and 4
Which of the statements given above is/are correct? 132. With reference to the phytoplankton, consider the
(a) 1 only following statements:
(b) 2 only 1. Phytoplankton are microscopic plant organisms
(c) Both 1 and 2 that live only in freshwater environments.
(d) Neither 1 nor 2 2. All phytoplankton are single-celled plants.

Workbook 274
.
ENVIRONMENT

3. They are limited to the uppermost layers of the Identify the National Park with the help of above
ocean. given information and select the correct answer:
Which of the statement given above is/are correct? (a) Keibul Lamjao National Park
(a) 1 only (b) Bandhavgarh National Park
(b) 1 and 2 only (c) Hemis National Park
(c) 3 only (d) Bandipur National Park
(d) 1, 2 and 3
137. With reference to National Parks in India, consider
133. Consider the following statements regarding the following statements:
Phytoplankton: 1. Guru Ghasidas National Park is home to the
1. Their population increases with the depth in the species of Golden langurs in India.
ocean water. 2. Bandipur National Park is located at the tri-
2. Their total biomass is greater than the total junction area of the states of Karnataka, Tamil
plants on land. Nadu and Kerala.
3. They are found in marine as well as freshwater 3. Pakke Tiger Reserve is famous for its hornbill
environments. conservation programme.
Which of the statements given above is/are correct? 4. Wayanad Wildlife Sanctuary has been
(a) 1 only endangered due to spread of the invasive African
(b) 1 and 3 only Mushi fish.
(c) 2 and 3 only
Which of the statements given above is/are correct?
(d) 1, 2 and 3
(a) 1 and 2 only
134. Seaweeds are a non-flowering marine algae which (b) 1, 2 and 3 only
play a major role in marine ecology. What is the (c) 2 and 4 only
importance of Seaweed? (d) 2, 3 and 4 only
1. It can act as a bio-indicator.
2. It removes toxic heavy metals found in marine 138. Consider the following statements regarding the
ecosystems. Project Dolphin:
3. It provides habitat for some marine life forms. 1. The proposed project is aimed at saving Gangetic
river dolphins only.
4. Its presence is essential for the survival of coral
reefs. 2. Protecting the dolphins and its habitat will
ensure conservation of aquatic lives of the river.
Select the correct answer using the code given below:
(a) 2 and 3 only Which of the statements given above is/are correct?
(b) 1, 3 and 4 only (a) 1 only
(c) 1, 2 and 3 only (b) 2 only
(d) 1, 2, 3 and 4 (c) Both 1 and 2
(d) Neither 1 nor 2
5.6. Protected Area network 139. Which of the following Indian Biosphere Reserves
135. With reference to the process for creation of a tiger is a part of UNESCO’s Man and Biosphere
reserve, consider the following: Network?
1. Tiger Reserves are notified by state governments 1. Nilgiri Biosphere Reserve
as per the Wildlife Protection Act, 1972. 2. Panna Biosphere Reserve
2. The Tiger Reserves cannot be de-notified by 3. Nokrek Biosphere Reserve
state governments 4. Manas Biosphere Reserve
3. Recently, Ramgarh Vishdhari wildlife sanctuary Select the correct option using the code given below:
has been approved as tiger reserve in Telangana.
(a) 1, 2 and 3 only
Which of the statements given above is/are correct? (b) 1, 3 and 4 only
(a) 1 only (c) 2, 3 and 4 only
(b) 2 and 3 only (d) 1, 2, 3 and 4
(c) 1 and 3 only
(d) 1, 2 and 3 140. Consider the following statements:
1. Diclofenac poisoning is one of the main reasons
136. “This National Park resides on the northern for decline in vulture population in recent times.
edges of the Satpura mountain ranges. Due to the
tropical monsoon climatic zone, the park has been 2. Vulture Action Plan (2020-25) has been jointly
characterized by well-defined winters summers launched by Central Zoo Authority and Bombay
and rains. The density of the tiger population here Natural History Society.
is one of the highest known in India.” Which of the statements given above is/are correct?

275 Workbook
.
ENVIRONMENT

(a) 1 only 2. The climate of the National Park varies from


(b) 2 only sub-tropical to temperate and Alpine.
(c) Both 1 and 2 3. The National Park is home to Van Gujjar
(d) Neither 1 nor 2 Nomadic Tribes.
141. Consider the following statements regarding Which of the statements given above is/are correct?
Deepor Beel: (a) 1 only
1. It is a saltwater lake located in the Indian state (b) 2 and 3 only
of Meghalaya. (c) 1 and 2 only
(d) 1 and 3 only
2. It has been declared as an Important Bird Area
by the BirdLife International. 146. With reference to Recovery Programme for
3. It is a region of dispute between India and Critically Endangered Species, which of the
Bangladesh. following statements is/are correct?
Which of the above statement(s) is/are incorrect? 1. Central Zoo Authority is the final body
(a) 1 and 2 only which decides to include a species under this
(b) 2 and 3 only programme.
(c) 1 and 3 only 2. It is part of a centrally sponsored scheme-
(d) 1, 2 and 3 Integrated Development of Wildlife Habitat.
Select the correct answer using the code given below:
142. With respect to Mukurthi National Park consider
(a) 1 only
the following statements (b) 2 only
1. It is a part of UNESCO World Heritage Site. (c) Both 1 and 2
2. This park is established for the protection of (d) Neither 1 nor 2
Nilgiri Tahr.
147. Recently the data of 4th All India Tiger Estimation
Which of the statements given above is/are incorrect? was made public. It showed an incredible raise in
(a) 1 only the population of tigers in India. With respect to
(b) 2 only the techniques used in tiger census which of the
(c) Both 1 and 2 following are correct?
(d) Neither 1 nor 2 1. DNA Finger-Printing
143. Consider the following pairs: 2. Camera Traps
3. Analyzing Stripe Pattern
Festivals State 4. Pugmark Analysis
1. Amur Falcon Assam Select the correct answer using the code given below:
festival (a) 1 and 4 only
2. Nuakhai Juhar Manipur (b) 1, 2 and 4 only
(c) 2, 3 and 4 only
3. Hornbill festival Nagaland (d) 1, 2, 3 and 4
Which of the pairs given above is/are correctly
148. With reference to the population of Leopard in
matched?
India, consider the following statements:
(a) 1 and 2 only
1. The population of leopard has consistently
(b) 2 and 3 only decreased during the last decade.
(c) 3 only
(d) 1, 2 and 3 2. The population of leopard is mostly concentrated
in the Brahmaputra plains and North-east India.
144. With regard to Soor Sarovar Lake, recently in 3. The population of leopard has decreased in all
news, consider the following statements: States during the last five years.
1. It is also known Keetham lake. Which of the statements given above is/are incorrect?
2. It was declared as a Wetland of International (a) 1 and 2 only
importance under Ramsar Convention. (b) 2 and 3 only
Which of the statements given above is/are correct? (c) 1 and 3 only
(a) 1 only (d) 1, 2 and 3
(b) 2 only 149. Which if the following statements regarding
(c) Both 1 and 2 Deemed Forests is/ are correct?
(d) Neither 1 nor 2 1. It has been clearly defined in the Forest
145. With reference to the Govind Pashu Vihar Conservation Act of 1980.
National Park, consider the following statements: 2. It comprises 1% of total forest land.
1. It is located in the state of Uttar Pradesh. Select the correct option using the code given below:

Workbook 276
.
ENVIRONMENT

(a) 1 only 154. Which of the following pairs of Protected Areas


(b) 2 only and their unique feature is/ are correctly matched?
(c) Both 1 and 2
(d) Neither 1 nor 2 Protected Areas Unique Feature
1. Pobitora Wildlife Highest One-Horned
150. This is a protected area located on the eastern
Sanctuary Rhinoceros population
coast of India and is home to the second largest
density in the world
mangrove ecosystem in the country. The protected
area also has a Ramsar site. Recently the state 2. Pakke Tiger Reserve Largest Tiger
government has launched a two-year conservation Population in Eastern
project for fishing cats in this protected area. Ghats
This protected area is also a home to salt water 3. Kaziranga National Indian Golden Tiger
crocodiles in India. In the above paragraph which Park was sighted here.
of the following protected area has been defined? Select the correct answer using the code given below:
(a) Sunderbans National Park (a) 1 and 2 only
(b) Sajnekhali Wildlife Sanctuary (b) 2 and 3 only
(c) Bhitarkanika National Park (c) 1 and 3 only
(d) Point Calimere Wildlife Sanctuary (d) 1, 2 and 3
151. Consider the following pairs: 155. The Chief Wildlife Warden can grant permit to a
Tiger reserve State person to enter or reside in a sanctuary or national
park for which of the following purpose(s)?
1. Pilbhit Uttar Pradesh 1. Study of Wildlife
2. Pench Maharashtra 2. Tourism
3. Ranthambor Rajasthan 3. Transacting lawful business with a person
Which of the pairs given above is/are correctly residing inside protected area
matched? Select the correct answer using the code given below:
(a) 1 and 2 only (a) 1 and 2 only
(b) 3 only (b) 2 and 3 only
(c) 1 and 3 only (c) 3 only
(d) 1, 2 and 3 (d) 1, 2 and 3
152. Which one of the following agencies are entrusted 156. Which of the following pairs is/are correctly
with the task of preparing ‘People’s Biodiversity matched?
Register’ in consultation with the local people in
India? Protected Area State
(a) National Biodiversity Authority (NBA) 1. Flamingo Sanctuary Gujarat
(b) Gram Sabha in coordination with Zilla Parishad
2. Coringa Wildlife Andhra Pradesh
(c) Biodiversity management committees (BMC)
Sanctuary
(d) District Collectorate
3. Jaladapara National Park West Bengal
153. With reference to the various protected areas
in Arunachal Pradesh, consider the following Select the correct answer using the code given below
statements: (a) 1 and 2 only
1. Papum Reserve Forest is an Important Bird and (b) 1 and 3 only
Biodiversity Area located in Arunachal Pradesh. (c) 2 and 3 only
(d) 1, 2 and 3
2. The Dongria Kondh tribe resides in the Pakke
Wildlife Sanctuary of Arunachal Paradesh. 157. With respect to Rajaji National Park, which of the
3. Hoolock Gibbons which is the only ape species following statements is/are incorrect?
in India is found in Namdapha National Park. 1. The Park is dissected into two parts by river
4. One can find a red panda in the Eaglenest Ganga.
Wildlife Sanctuary in Arunachal Pradesh. 2. It is home to Maldhari tribe in winters.
Which of the statements given above are correct? Select the correct answer using the code given below:
(a) 1 and 2 only (a) 1 only
(b) 2 and 3 only (b) 2 only
(c) 1, 3 and 4 only (c) Both 1 and 2
(d) 1, 2, 3 and 4 (d) Neither 1 nor 2

277 Workbook
.
ENVIRONMENT

158. Which of the following locations are identified for Select the correct answer using the code given below
possible lion relocation under Project Lion? (a) 1 only
1. Madhav National Park (b) 2 only
2. Sitamata Wildlife Sanctuary (c) Both 1 and 2
3. Nauradehi Wildlife Sanctuary (d) Neither 1 nor 2
4. Nagarhole National Park 163. Consider the following statements regarding
Select the correct answer using the code given below: Sacred Groves:
(a) 1 and 2 only 1. All Sacred groves are equally protected.
(b) 2, 3 and 4 only 2. Sacred groves usually have a significant religious
(c) 1, 3 and 4 only connotation for the protecting community.
(d) 1, 2, 3 and 4 3. Sacred groves help in conservation of biodiversity,
recharge of aquifers and soil conservation.
159. With respect to tiger reserves in India, which of
Which of the statements given above is/are correct?
the following statements is correct?
(a) 1 only
(a) These are created for the protection of tigers and (b) 1 and 3 only
also their prey base. (c) 2 and 3 only
(b) National Tiger Conservation Authority (NTCA) (d) 1, 2 and 3
is the final authority which creates a tiger reserve.
(c) State government on its own can alter the 164. Match the following pairs of Ramsar sites and
boundaries of a tiger reserve. their respective characteristics:
(d) By creation of a tiger reserve, the rights of forest
Ramsar sites Characteristics
dwellers are no longer recognized in the core
area. A. Tso Kar wetland 1. First Ramsar site in
Complex Bihar
160. With reference to a tiger reserve, which of the
following statements is/are correct? B. Kabartal Wetland 2. Important breeding
1. A tiger reserve is notified by a state government. area in India for
Black necked Crane
2. The concerned Gram Sabha is consulted for
determining the limits of buffer zone of a tiger C. Sur Sarovar 3. Human-made
reserve. reservoir
3. Tiger reserves are not treated as protected areas Select the correct answer using the code given below:
under Wildlife Protection Act 1972. (a) A-2, B-3, C-1
Select the correct answer using the code given below: (b) A-1, B-2, C-3
(c) A-2, B-1, C-3
(a) 1 only
(d) A-1, B-3, C-2
(b) 1 and 2 only
(c) 2 only 165. With reference to Panna Biosphere Reserve (PBR),
(d) 1, 2 and 3 consider the following statements:
1. PBR is the second Biosphere reserve in Madhya
161. Which of the following statement is incorrect
Pradesh to be included in UNESCO’s World
about Reserved and Protected Forests in India?
Network of biosphere reserves.
(a) The activities like hunting and grazing are 2. It is situated in the Satpura mountain range in
banned in reserve forest unless specifically the southern part of Madhya Pradesh.
permitted.
(b) In protected forest there is limited exploitation Which of the statements given above is/are correct?
of resources. (a) 1 only
(c) Both these protected areas are declared by (b) 2 only
respective state governments. (c) Both 1 and 2
(d) These terms were introduced for the first time (d) Neither 1 nor 2
under Forest Conservation Act, 1980. 166. With respect to biodiversity hotspots, which of the
following statements is incorrect?
162. With respect to conservation reserve and
community reserve which of the following (a) It must have lost at least 70% of its original
habitat.
statements is/are incorrect?
(b) There are 3 biodiversity hotspots present in
1. Initially these protected areas were not included India.
in the Wildlife Protection Act 1972. (c) Nicobar island is part of the biodiversity hotspot.
2. Both these protected areas are managed by State (d) It must contain at least 1500 species of vascular
Board for Wildlife. plants as endemics.

Workbook 278
.
ENVIRONMENT

167. This biosphere reserve lies in the eastern end of 172. With reference to the Biosphere Reserves (BRs),
the eastern ghat. It derives its name from a silk consider the following statements:
cotton tree. It is also a national park and a tiger 1. Nilgiri Biosphere Reserve was established as the
reserve. The forest fire in this biosphere reserve is first Biosphere Reserve in India.
a phenomenon that has been in play for decades. 2. Rann of Kutch is the largest Biosphere Reserve
The biosphere reserve in the above reference is: in India.
(a) Seshachalam Biosphere Reserve
3. Core zone of a BR is protected under the Wildlife
(b) Dibru-Saikhowa Biosphere Reserve
(Protection) Act, 1972.
(c) Panna Biosphere Reserve
(d) Similipal Biosphere Reserve 4. No human activity is allowed in the Buffer zone
of a BR.
168. With reference to the National Park, consider the Which of the statements given above is/are correct?
following statements:
(a) 1 and 2 only
1. The national parks in India constitute about 3%
(b) 2 and 4 only
of the geographical area of the country.
(c) 1 and 3 only
2. They are declared on the basis of UNESCO’s (d) 3 and 4 only
Man and Biosphere (MAB) programme.
3. Activities like grazing, hunting, forestry or 173. Kameng-Sonitpur Landscape, Brahmagiri-
cultivation etc. are strictly prohibited in national Nilgiri-Eastern Ghat Landscape and Anamalai-
parks. Nellianpathy-High Range Landscape, all three are
Which of the statements given above is/are correct? well known:
(a) 1 and 2 only (a) Tiger Reserves
(b) 3 only (b) Eco-sensitive Zones
(c) 2 and 3 only (c) Elephant Reserves
(d) 1, 2 and 3 (d) National Parks
169. Which one among the following can permit for 174. Consider the following pairs:
the removal or exploitation of wildlife or forest
produce in India? Tiger Reserve State
(a) Chief Minister of the State 1. Parambikulam Tamil Nadu
(b) Union Ministry of Environment, Forest and 2. Dandeli-Anshi Karnataka
Climate Change
(c) Gram Panchayat 3. Satkosia Odisha
(d) Chief Wild Life Warden 4. Kamlang Assam
170. Which of the following are part of the Biodiversity Which of the pairs given above is/are correctly
Heritage Sites? matched?
1. Dialong Village (a) 1 and 2 only
2. Ameenpur lake (b) 1 and 3 only
3. Majuli Island (c) 2 and 3 only
4. Ambaraguda (d) 3 and 4 only
Select the correct answer using the code given below: 175. With reference to Conservation Breeding
(a) 1, 2 and 3 only Programme in India, consider the following
(b) 1, 3 and 4 only statements:
(c) 2, 3 and 4 only 1. It has been started under the supervision of the
(d) 1, 2, 3 and 4 Central Zoo Authority.
171. With reference to the Important Coastal and 2. The Central Zoo Authority would provide 100%
Marine Biodiversity Areas (ICMBAs), consider financial assistance to the participating zoos.
the following statements: 3. The Laboratory for Conservation of Endangered
1. They were established to achieve the goals under Species (LaCONES) is the only institute in
the Aichi Biodiversity Target. the country working towards conservation
2. All the ICMBAs are upgraded as Protected Areas of endangered wildlife using modern
under the category of Communities Reserve. biotechnology.
Which of the statements given above is/are correct? Which of the statements given above is/are correct?
(a) 1 only (a) 1 and 2 only
(b) 2 only (b) 1 and 3 only
(c) Both 1 and 2 (c) 2 and 3 only
(d) Neither 1 or 2 (d) 1, 2 and 3

279 Workbook
.
ENVIRONMENT

176. Which one of the following statements correctly human made wetland. It is formed on a floodplain
describes the term ‘Biodiversity Coldspots’? of the River Ganga on the Madhya Ganga Barrage.
(a) Areas that have relatively low biological diversity Critically endangered gharial and the endangered
but are experiencing a high rate of habitat loss. hog deer are found here. It serves as a breeding
(b) Areas identified by the Conservation site for the near-threatened Indian grassbird. It
International that harbor the majority of Earth’s
is located within the boundaries of Hastinapur
species.
Wildlife Sanctuary.”
(c) Areas situated in polar areas that are experiencing
a high rate of habitat loss. The wetland mentioned here is
(d) Areas that have high species endemism and high (a) Sarsai Nawar
degree of threat. (b) Gogabeel
177. “ Recently this wetland has been recognised under (c) Nawabganj bird sanctuay
the 1971 Ramsar Convention on Wetlands. It is a (d) Haiderpur

Workbook 280
.
ENVIRONMENT

SOLUTIONS

• It must have 30% or less of its original natural


5.1. Previous Years Questions vegetation. In other words, it must be threatened.
1. Solution (c) 5. Solution (b)
Exp) Option c is correct Exp) Option b is correct.
There is an increase in biodiversity from the poles to the Himalayan forests nurture staggering diversity of life
tropics. This is often referred to as latitudinal diversity forms richness across longitudinal and altitudinal
gradient. gradients, and are therefore classified as one of the 36
Elevational diversity gradient (EDG) is an ecological global biodiversity hotspots. Nearly 50 per cent of India’s
pattern where biodiversity changes with elevation. The total flowering plants grow in the Himalayas, of which 30
EDG states that species richness tends to increase as per cent are endemic to the region.
elevation increases, up to a certain point, creating a Biogeographic classification of India is the division
“diversity bulge” at middle elevations. Therefore, in of India according to biogeographic characteristics.
mountains biodiversity is normally greater in the lower Biogeography is the study of the distribution of species
altitudes as compared to the higher altitudes. (biology), organisms, and ecosystems in geographic space
2. Solution (d) and through geological time.
Exp) Option d is correct There are ten biogeographic zones in India.
Bio diversity helps in the formation and maintenance of • Trans-Himalayan zone.
soil structure and the retention of moisture and nutrient • Himalayan zone · Desert zone.
levels. Tree and other vegetation help against soil erosion. • Semiarid zone.
Microbial activity assists in degradation of waste. • Western ghat zone.
Ecosystems perform the vital function of recycling • Deccan plateau zone.
nutrients. These nutrients include the elements of the
• Gangetic plain zone.
atmosphere as well as those found in the soil, which are
necessary for the maintenance of life. Biological diversity • North east zone.
is essential in this process. Plants are able to take up • Coastal zone
nutrients from the soil as well as from the air, and these • Islands
nutrients can then form the basis of food chains, to be
used by a wide range of other life forms. The soil’s nutrient The Himalayas is a confluence point for different bio-
status, in turn, is replenished by dead or waste matter geographical zones. In addition, it is at confluence of the
which is transformed by microorganisms; this may then two of the world’s main bio-geographical regions, the
feed other species such as earthworms which also mix and palearctic and oriental realms.
aerate the soil and make nutrients more readily available. 6. Solution (a)
Butterflies help in the pollination of crops. Exp) Option a is correct.
3. Solution (b) According to the IUCN, an endangered species is one
Exp) Option b is correct. that meets any one of the following criteria: a 50–70%
population decrease over 10 years, a total geographic area
Indian Wild Ass is only found in the Little Rann of Kutch less than 5,000 km2 (or local population area less than 500
in Gujarat. km2), a population size less than 2,500 adults, a restricted
It is protected under Schedule I of the Wildlife (Protection) population of 250 adults, or a statistical prediction that it
Act 1972, India and included on Appendix I of the will go extinct within the next 20 years.
Convention on International Trade in Endangered Species Great Indian Bustard, Musk Deer, Red Panda and Asiatic
(CITES), making international trade in this species illegal. Wild Ass are endangered species.
4. Solution (c) Note: The current IUCN status of Great Indian Bustard is
Exp) Option c is correct. critcally endangered. Based upon that none of the options
would be correct.
To qualify as a biodiversity hotspot, a region must meet
two strict criteria: 7. Solution (a)
• It must have at least 1,500 vascular plants as endemics Exp) Option a is correct.
— which is to say, it must have a high percentage of Oryx is one of four large antelope species of the genus
plant life found nowhere else on the planet. A hotspot, Oryx. The Tibetan antelope or Chiru is a medium sized
in other words, is irreplaceable. bovid native to the Tibetan plateau.

281 Workbook
.
ENVIRONMENT

Oryx is adapted to live in hot and arid areas whereas 16. Solution (a)
Chiru is adapted to live in steppes and semi-desert areas Exp) Option a is correct.
of cold high mountains.
Lion-tailed macaque is not found naturally in Andhra
8. Solution (a) Pradesh. It is endemic to Western Ghats and it’s habitat is
Exp) Option a is correct. mostly restricted to Evergreen Forests of the Ghats.
Promotion of vegetarianism is not a threat to the 17. Solution (c)
biodiversity of a geographical area.
Exp) Option c is correct.
9. Solution (b) The habitat of the Ganges river dolphin is within one of
Exp) Option b is correct. the most densely populated areas of the world. Ganges
Cheetah inhabits most of Africa and parts of Middle river dolphins and people both favour areas of the river
where fish are plentiful and the water current is slower.
East. Due to large scale hunting, the cheetah was declared
This has led to fewer fish for people and more dolphins
extinct from India in 1952.
dying as a result of accidentally being caught in fishing
10. Solution (a) nets, also known as bycatch.
Exp) Option a is correct. The Ganges river dolphin is still hunted for meat and oil,
Complete elimination of phytoplanktons from ocean which are both used medicinally. The oil is also used to
would not have a drastic effect on the density of the ocean attract catfish in net fishery.
water. Each year, 9,000 tons of pesticides and 6 million tons of
The two main factors that affect density of ocean water are fertilizers are used in the vicinity of the river - High levels
the temperature of the water and the salinity of the water. of pollution can directly kill prey species and dolphins.
The construction of dams and barrages has adversely
11. Solution: (a) affected the population of Ganges River Dolphin since
Exp) Option a is correct. early 20th Century.
Oryx is one of four large antelope species of the genus
18. Solution (b)
Oryx. The Tibetan antelope or Chiru is a medium sized
bovid native to the Tibetan plateau. Exp) Option b is correct.
Oryx is adapted to live in hot and arid areas whereas Painted Stork is a fish eating bird found near water bodies
Chiru is adapted to live in steppes and semi-desert areas such as wetlands, marshes, and flooded agricultural fields.
of cold high mountains. Whereas the Common Myna is an omnivore. In 2000, it
12. Solution: (d) was declared as one of the world’s most invasive species
by the IUCN.
Exp) Option d is correct.
All of the above (Bat, Bee and Bird) are pollinating agents. 19. Solution (c)
About three-fourths of all native plants in the world Exp) Option c is correct.
require pollination by an animal, most often an insect, The dugong is a herbivorous medium-sized marine
and most often a native bee. mammal. It is a herbivorous marine animal. The IUCN
Loss of pollinators threatens agricultural production, Red List lists the dugong as vulnerable, and the Convention
the maintenance of natural plant communities, and the on International Trade in Endangered Species of Wild
important services provided by those ecosystems, such as Fauna and Flora regulates and in some areas has banned
carbon cycling, flood and erosion control, and recreation. international trade. In India, it is given legal protection
under Schedule I of the Wildlife (Protection) Act, 1972.
13. Solution (c)
In India they occur in Gulf of Mannar, PalkBay, Gulf of
Exp) Option c is correct. Kutch and Andaman and Nicobar islands. Once abundant
All of the above are endangered in India. in Indian waters, Dugong population has now reduced to
about 200 individuals and is believed to be continuously
14. Solution (a) declining in its number and range.
Exp) Option a s correct.
20. Solution (c)
Spider monkeys are New World monkeys. They are found
in tropical forests of Central and South America, from Exp) Option c is correct.
southern Mexico to Brazil. Gangetic river dolphin is the National Aquatic Animal of
India.
15. Solution (b)
It is a critically endangered species in India and therefore,
Exp) Option b is correct. has been included in the Schedule I for the Wildlife
Sea cow and Sea lion are mammals who give birth to live (Protection) Act, 1972. The main reasons for decline
ones. in population of the species are poaching and habitat

Workbook 282
.
ENVIRONMENT

degradation due to declining flow, heavy siltation, contiguous patches in Tamil Nadu and Karnataka.
construction of barrages causing physical barrier for this However, it is not found tropical rain forests of South
migratory species. The Ganges river dolphin is primarily India.
found in the Ganges and Brahmaputra rivers and their Red Sanders is a protected species under the Convention
tributaries in India, Bangladesh, and Nepal. on International Trade in Endangered Species (CITES) of
Wild Fauna and Flora.
21. Solution (d)
Exp) Option d is correct. 25. Solution (b)
The Middle Andamans harbours mostly moist deciduous Exp) Option b is correct.
forests. North Andamans is characterised by the wet The National Chambal Gharial Wildlife Sanctuary
evergreen type, with plenty of woody climbers. The north is a protected area for critically endangered Gharial
Nicobar Islands (including Car Nicobar and Battimalv) Crocodiles, located at Palighat Village of Sawai Madhopur,
are marked by the complete absence of evergreen forests, which is adjacent to the banks of the Chambal River.
while such forests form the dominant vegetation in The course of the Chambal was through forests or ravines
the central and southern islands of the Nicobar group. or scrubland, away from human settlement. Hence,
Grasslands occur only in the Nicobars, and while wildlife flourished in the river and on its banks. Gharials
deciduous forests are common in the Andamans, they are were found in abundance but their number alarmingly
almost absent in the Nicobars. reduced to 200 in 1975.
In terms of diversity, density and growth, mangroves of In 1975 an area of land along the course of the river was
Andaman and Nicobar Islands are best in the country. declared as a sanctuary.
This atypical forest coverage is made-up of twelve 26. Solution: (d)
types namely (1) Giant evergreen forest (2) Andamans Exp) Option d is correct.
tropical evergreen forest (3) Southern hilltop tropical
evergreen forest (4) Cane brakes (5) Wet bamboo brakes All the above mention animals are listed under Schedule
(6) Andamans semi-evergreen forest (7) Andamans I of Wildlife (Protection) Act, 1972 and thus they are
accorded maximum protection under law.
moist deciduous forest (8) Andamans secondary moist
deciduous forest (9) Littoral forest (10) Mangrove forest 27. Solution: (c)
(11) Brackish water mixed forest (12) Submontane hill Exp) Option c is correct
valley swamp forest. The present forest coverage is claimed
to be 86.2% of the total land area. Bioremediation is a process that uses mainly
microorganisms, plants, or microbial or plant enzymes to
22. Solution (d) detoxify contaminants in the soil and other environments.
Exp) Option d is correct. Heavy metals such as cadmium and lead are not readily
Great Indian Hornbill is predominantly fruit-eating, but absorbed or captured by microorganisms.
is an opportunist and preys on small mammals, reptiles Some highly chlorinated contaminants and high
and birds. A large majority of their population is found in molecular weight Polycyclic aromatic hydrocarbons
India with a significant proportion in the Western Ghats (PAHs) present in coal, crude oil and gasoline are also
and the Nilgiris. not readily amenable to microbial degradation. There are
also increasing concerns that the bioremediation products
The nesting grounds of the birds in the Nilgiris North may be more persistent or hazardous than the parent
Eastern Range are also believed to support some of their compounds.
highest densities.
28. Solution: (a)
23. Solution (d)
Exp) Option a is correct.
Exp) Option d is correct.
The Baigas were discriminated against and often evicted
Kharai camels are known for their mangrove diet and from forest areas by government agencies. They became
swimming for long hours. Gujarat is the only State in India India’s first community to get habitat rights. This tribe is
which is home to Kharai Camels. It grazes on mangrove found in Madhya Pradesh and Chhattisgarh.
trees and can swim up to three kilometres into the sea in
The Ministry of Tribal Affairs is implementing the Scheme
search of their primary food mangroves.
for Development of Particularly Vulnerable Tribal Groups
These animals live in wild and cannot be domesticated, (PVTGs)
though they live around a few nomadic tribes. Under the Scheduled Tribes and Other Traditional Forest
24. Solution (a) Dwellers (Recognition of Forest Rights) Act, 2006 the
District Level Committee shall ensure that all Particularly
Exp) Option a is correct. Vulnerable Tribal Groups receive habitat rights, in
Red Sanders is a non-fragrant variety of sandalwood consultation with the concerned traditional institutions of
that mostly grows in rocky, hilly regions. Mostly grows Particularly Vulnerable Tribal Groups and their claims for
in deciduous forests of Andhra Pradesh and in some habitat rights are filed before the concerned Gram Sabhas.

283 Workbook
.
ENVIRONMENT

29. Solution (b) As per the Scheduled Tribes and Other Traditional Forest
Exp) Option b is correct. Dwellers (Recognition of Forest Rights) Act, 2006, allows
ownership of minor forest produce to forest dwellers and
Prosopis juliflora (P juliflora), an exotic tree, is one of
bamboo is a minor forest produce.
the top invasive species in India. A native of South and
Central America, it was introduced in India to meet the 36. Solution: (a)
fuel wood requirement of the rural poor and to restore Exp) Option a is correct.
degraded lands.
There is no provision mandating people participation in
30. Solution (a) the compensatory afforestation programmes carried out
Exp) Option a is correct. under the Compensatory Afforestation Fund Act, 2016.
The act encourages people’s participation but does not
Double-humped camel is native to steppes region of make it mandatory.
Central Asia, in India it is found in Nubra Valley in
Ladakh. 37. Solution: (c)
One-horned rhino is naturally found in areas of Assam Exp) Option c is correct.
such as Kaziranga National Park, India and also in The Extended Producer Responsibility (EPR) was an
Chitwan National Park, Nepal. important feature of the e -Waste (Management and
Handling) Rules, 201. EPR is the responsibility of
31. Solution (c) the producer of electrical or electronic equipment of
Exp) Option c is correct. managing the product in a environment-friendly manner
Irrawaddy Dolphin is not found in Chambal River. It is after the consumer discards them.
usually found near sea coasts and in estuaries and rivers in 38. Solution (a)
parts of the Bay of Bengal and Southeast Asia. It is found
in Chilika Lake (Odisha). Exp) Option a is correct.
The elephant herd is led by the oldest and largest female
32. Solution (c) known as the matriarch.
Exp) Option c is correct. After 22 months growing inside its mother’s womb, female
According to the State of Forest Report, 2017 released by calves weigh 90-100 kg. Males are heavier and weigh up to
Forest Survey of India, the percentage of forest cover to 120 kg.
the total area of the state are: The process of giving birth to young ones is called calving.
Maharashtra (16.74%) > Madhya Pradesh (25.11%) > Elephants can calve at least till the age of 65. Karnataka
Odisha (32.98%) > Chhattisgarh (41.09) has the highest population of Elephants in India.
33. Solution: (c) 39. Solution (a)
Exp) Option c is correct. Exp) Option a is correct.
Solid Waste Management Rules, 2016 provide for exact Hard ground Indianswamp deer (Barasingha or Rucervus
and elaborate criteria for the identification of sites for duvaucelii), the state animal of Madhya Pradesh, has seen
landfills and waste processing facilities. a revival in the Kanha National Park and Tiger Reserve
Source) UPSC 2019 (KNPTR) after having been perilously close to extinction
for a long time.
34. Solution: (b)
KNPTR is on the Maikal range of the Satpura hills, and
Exp) Option b is correct. is spread over an area of 940 square kilometres between
Statement 1 is incorrect. The Environment Protection Mandla and Balaghat districts of Madhya Pradesh. The
Act, 1986 doesn’t contain any provision related to public swamp deer is endemic to KNPTR.
participation. In 1967, the numbers of swamp deer decreased to 66 due
Statement 2 is correct: As per Section 3 of the to rampant hunting, habitat loss and diseases. The number
Environment Protection Act, the Government of India is was estimated at around 551 in 1953.
empowered to make rules in the laying down standards The deer was brought back from the verge of extinction
for emission or discharge of environmental pollutants with the help of a successful breeding programme
from various sources. and conservation practices at the KNPTR. Various
35. Solution: (b) conservation methods were used, including habitat
improvement and captive breeding that led to an increase
Exp) Option b is correct. in the population to around 450 in 2015 and 800 in 2020.
As per recent amendment to the Indian Forest Act, 1927,
forest dwellers have the right to fell the bamboos grown 40. Solution (a)
on non-forest areas. Exp) Option a is correct.

Workbook 284
.
ENVIRONMENT

If a particular plant species is placed under Schedule VI cyprinid and known to be the toughest among the fresh
of The Wildlife Protection Act, 1972, then No person is water sport fish. Golden Mahseer lives in fast-moving
allowed to cultivate the specified plant, except with a waters, inhabiting hill streams with a rocky and stony
licence granted by the Chief Wild Life Warden. The substrate. They can be found in temperatures between 5°C
specified endemic plants in Schedule VI are prohibited and 25°C. The Golden Mahseer inhabits the Himalayan
from cultivation and plantin, except with a granted license. foothills, the Indus, Ganga and Brahmaputra basins and
The plants mentioned in the sixth schedule are can also be found down south in the Balamore, Cauvery,
Tambraparini, and Kosi Rivers.
• Beddomes’cycad
The Indian nightjar species are nocturnal birds with long
• Blue Vanda pointed wings and short legs.
• Kuth Spoonbill is any member of six species of long-legged
• Ladies slipper orchids wading birds that constitute the subfamily Plataleinae
• Pitcher plant of the family Threskiornithidae (order Ciconiiformes),
which also includes the ibises. Spoonbills are found in
• Red Vanda estuaries, saltwater bayous, and lakes.
41. Solution (a) White Ibises are large wading birds with football-shaped
Exp) Option a is correct. bodies. They are found in freshwater marshes, coastal
estuaries, mangroves, flooded pastures, mudflats, and
All the above are names of bird varieties found in India. swamps.
42. Solution: (d) 45. Solution: (c)
Exp) Option d is correct. Exp) Option c is the correct answer.
Just as forests are called as the ‘lungs of the earth’, Pair 1 is incorrect: Āryadeva (3rd century), a
wetlands are the ‘kidneys’ that regulate water and filter disciple of Nāgārjuna, is a central figure in the
waste from the landscape. development of early Indian Madhyamaka philosophy.
Wetlands include mangroves, peat lands and marshes, TheMadhyamakaschool of Buddhism, the followers
rivers and lakes, deltas, floodplains and flooded forests, of which are called Mādhyamikas, was one of the two
rice-fields, and even coral reefs. Aquatic plants of wetlands principal schools of Mahāyāna Buddhism
absorb heavy metals and excessive nutrients thereby Pair 2 is correct: Dignāga, (born c. 480 CE—died
purifying water. For instance, Mangrove trees have the c. 540), was a Buddhist logician and author of the
ability of storing metals, transferring these elements from Pramānasamuccaya (“Compendium of the Means of
the sediment and concentrating them in their tissues True Knowledge”), a work that laid the foundations of
They can serve as a means for the immobilization and Buddhist logic.
removal of pollutants. Pair 3 is correct: Nathamuni founded aSanskrit-Tamil
43. Solution: (c) school at Srirangam (Tamil Nadu state), which continues
to be a great Vaishnavite centre in South India.
Exp) Option c is correct.
Statement 1 is correct: Guchhi mushroom is a species of 46. Solution: (a)
fungus in the family Morchellaceae of the Ascomycota. Exp) Option a is the correct answer
They are pale yellow in colour with large pits and ridges The fungus-growing ants are a New World group of less
on the surface of the cap, raised on a large white stem. than 200 species, all obligate symbionts with a fungus
Statement 2 is correct: It is grown in the foothills of they use for food. The These ants are found across the
Himalayas in Himachal Pradesh, Uttaranchal, and Jammu American continents and the West Indies. These ants
and Kashmir. are known as fungus growers because they maintain an
Statement 3 is incorrect: The mushrooms cannot be obligate mutualism with fungi cultured inside their
cultivated commercially and grow in conifer forests nests, and which is the only food source for the larvae and
across temperature regions. It takes months for villagers an important resource for the adult ants as well 
to collect enough of these mushrooms, dry them and bring
them to the market. These mushrooms are also replete 5.2. Biodiversity in India
with health benefits. They are rich in potassium, vitamins
and copper. They are also a rich source of vitamin D apart 47. Solution (a)
from several B-vitamins Exp) Option a is correct.
Seabuckthorn is a shrub which produces an orange-
44. Solution: (a)
yellow coloured edible berry. In India, it is found above
Exp) Option a is correct. the tree line in the Himalayan region, generally in dry
Mahseer roughly translates as mahi fish and sher areas such as the cold deserts of Ladakh and Spiti. A
tiger; it is also referred as tiger among fish. It is a large major part is covered by this plant in Himachal Pradesh,

285 Workbook
.
ENVIRONMENT

Ladakh, Uttarakhand, Sikkim and Arunachal Pradesh. September, favors development and sustenance of the
The Himachal Pradesh government has recently decided Tropical Dry Evergreen forest in this narrow strip.
to start planting seabuckthorn in the cold desert areas of 50. Solution (c)
the state.
Exp) Option c is correct.
The Seabuckthorn Plantation has many Ecological,
Medicinal and Economical benefits. Statement 1 is correct. Species diversity refers to the
variety of living organisms on earth. It is defined as the
1. Ecological Benefits: prevents soil-erosion, checks
number of species and abundance of each species that live
siltation in rivers and helps preserve floral biodiversity.
in a particular location. The number of species that live in
2. Medicinal Benefits: as a folk medicine, seabuckthorn a certain location is called species richness.
has been widely used for treating stomach, heart and
skin problems. Its fruit and leaves are rich in vitamins, Statement 2 is correct. It is the ratio of one species
carotenoids and omega fatty acids and it can help population over the total number of organisms across
troops in acclimatising to high-altitude. all species in the given biome. ‘Zero’ would be infinite
3. Economic Benefits: used in making juices, jams, diversity, and ‘one’ represents only one species present.
nutritional capsules etc. It is an important source of 51. Solution (b)
fuel wood and fodder.
Exp) Option b is correct.
48. Solution (a) Option a is incorrect. Alpha diversity refers to the
Exp) Option a is correct. diversity within a particular area or ecosystem, and is
A mycorrhiza is a symbiotic association between plants usually expressed by the number of species (i.e., species
and fungi. richness) in that ecosystem.
In mycorrhiza association, the fungus invades the root to Option b is correct. Beta diversity is a comparison of
absorb nutrients and form a type of mutualistic parasitism. diversity between ecosystems, usually measured as the
Plants rely on mycorrhizal fungi for mineral nutrients (i.e., change in number of species between the ecosystems.
phosphorus), and in return the fungus receives nutrients Option c is incorrect. Gamma diversity is a measure of
formed by the plant. Mycorrhizae play important roles in the overall diversity for the different ecosystems within a
plant nutrition, soil biology, and soil chemistry. region.
49. Solution (b) Option d is incorrect. Species evenness measures the
proportion of species at a given site, e.g., low evenness
Exp) Option b is correct.
indicates that a few species dominate the site
Statement 1 is correct. The Terai–Duar savanna and
grasslands is a narrow lowland ecoregion at the base of 52. Solution (d)
the Himalayas having Sal trees. It harbours the world’s Exp) Option d is correct.
tallest grasslands, which are the most threatened and
rare worldwide. This tropical and subtropical grasslands, Option 1 is correct. Due to poor dispersal ability in
savannas, and shrublands biome stretch from western some species there is large chance of them going extinct
Bhutan to southern Nepal’s Terai, to India’s Bhabar and due to any localized event. This poor dispersal ability is
Doon Valley. Each end crosses the border into India’s the region why India is relocating its rhinos and lions
states of Uttarakhand, Uttar Pradesh and Bihar. It is a over a wider geographical region.
mosaic of tall riverside grasslands, savannas and evergreen Option 2 is correct. When a species become ecologically
and deciduous forests, depending on soil quality and the specialized its range of survival decreases. It can only
amount of rain each area receives. survive in particular conditions and with any externality
Statement 2 is correct. Thar desert is an example of hot if there is a change in these conditions, chance of
and dry desert which have thorny bushes to reduce loss extinction increases.
of water. It is spread over India and Pakistan and within Option 3 is correct. When species are at a higher tophic
India, states of Rajasthan and Gujarat. level in the food chain they usually have lower population
Statement 3 is incorrect. The coral reef biome of India than those at the lower levels e.g. carnivores are fewer in
includes both the soft corals that need sunlight to survive number than
as well as the hard corals, although majority are soft herbivores. Recent extinctions of Tasmanian tiger, Sicilian
corals. wolf, Japanese sea lion, etc substantiate this trait.
Statement 4 is correct. The Tropical Dry Evergreen Option 4 is correct. Rising temperatures can lead to
Forest is a unique forest found only in Tamil Nadu and trouble for animals that use temperature dependent
Andhra Pradesh. It is distribution is restricted to the sex determination (TSD). In this gender of the animal is
narrow coastal strip from Vishakapattinam in Andhra controlled by the temperature that an animal is exposed
Pradesh in the north to Ramanathapuram in Tamil Nadu to while it is developing as an embryo or larvae. Extreme
in the south. The dissymmetric climatic condition of heat due to climate change can lead to an imbalance in
this region, where rainfall occurs both during summer sexes and could stifle reproduction, which could lead to
and winter and extended the dry season from March to population decline and even extinction.

Workbook 286
.
ENVIRONMENT

53. Solution (a) to colonize bare rock. Therefore, it is one of the pioneer
Exp) Option a is correct species.
Statement 1 is correct: Keystone species are those which Statement 3 is correct: Lichens are sensitive to sulphur
have a disproportionately large impact on a particular dioxide pollution. Thus, lichen serve as good bio-
ecosystem relative to its population. If a keystone species indicators for air pollution.
is lost, it will result in the degradation.
57. Solution (a)
of whole ecosystem. For example, certain plant species
(ebony tree, Indian-laurel) exclusively depends upon bats Exp) Option a is correct.
for its pollination. If the bat population is reduced, then Pair 1 is correctly matched. Food niche includes what
regeneration of particular plants becomes more difficult. the species eats or decomposes and what other species it
Statement 2 is correct: All top predators (Tiger, Lion, competes with for food.
Crocodile) are considered as keystone species because Pair 2 is incorrectly matched. Physical and chemical
they regulate all other animals’ population indirectly. niche includes temperature, land shape, land slope,
Statement 3 is incorrect: Herbivores can also be keystone humidity and other requirements of the species.
species, ex- Elephants. Their consumption of plants Pair 3 is incorrectly matched. Habitat niche includes
helps control the physical and biological aspects of an where the species lives. Whereas, reproductive niche
ecosystem. includes how and when the species reproduces.
54. Solution (d) 58. Solution (c)
Exp) Option d is correct. Exp) Option c is correct.
Statement 1 is correct. Poikilotherms are generally Option 1 is incorrect. Nearctic realm is the 2nd largest
known as cold-blooded animals. They are not capable of all realms. It comprises US, Canada, Greenland and
of regulating their body’s temperature according to the highland regions of Mexico.
temperature of the surrounding. These include reptiles,
fishes, amphibians, insects and other invertebrates. Option 2 is correct. Palaearctic realm is represented in
India by the Himalayan region. This is the largest realm
Statement 2 is correct. Vertabrates are the most which is made up of the area north of Sahara Desert, area
advanced animals on the earth and have backbones north of Himalayan mountains in Asia, northern region
and spinal columns. It gives them advantage of size of the Arabian peninsula and the entire area of Europe.
and mobility over other animals and often allows them
to dominate their environment even when they only Option 3 is incorrect. Neotropical realm is made up
represent a very small percentage of all animals. of tropical regions of Americas and it includes South
America, Central America, the Caribbean, etc. India does
Statement 3 is correct. Monotremes are mammal species
which lay eggs instead of giving birth to their young. not have any of its part in neotropical realm.
There are only 5 living monotremes: duck-billed platypus Option 4 is correct. Malayan realm is also known as
and 4 species of spiny anteaters. Indo-Malayan or Oriental realm. Except the Himalayan
region the entire Indian subcontinent is part of this
55. Solution (b) realm.
Exp) Option b is correct.
Statement 1 is incorrect. Marsupials are also known as
pouched mammals. They do have placenta but it is very
short-lived and does not make as much of a contribution
to fetal nourishment.
Statement 2 is correct. Marsupial does not have long
gestation period because of yolk-type placenta. They
give birth very early and embryo climbs from the mother’s
birth canal to the nipples.
56. Solution (c)
Exp) Option c is correct.
Statement 1 is incorrect: A lichen is a peculiar
combination of an alga and a fungus. It is an example of
symbiotic relationship. It cannot make its own food and
depends on photosynthesis happening within the algae or
cyanobacterium (green alga) and the fungus allows algae 59. Solution (b)
or cyanobacterium to grow in a drier environment than it Exp) Option b is correct.
could otherwise tolerate. Statement 1 is incorrect. The advisory has defined
Statement 2 is correct: Lichens are capable of initiating exotic animals as those that are mentioned under the
primary succession. They are typically, the first organisms Appendices of the Convention on International Trade

287 Workbook
.
ENVIRONMENT

in Endangered Species of Wild Fauna and Flora Statement 3 is correct. Senna spectabilis is a deciduous
(CITES), but not under the schedules of the Wild Life tree native to tropical areas of America. The thick foliage
(Protection) Act, 1972. of the tree arrests the growth of other indigenous tree and
Statement 2 is incorrect. A person trying to import a live grass species. Thus, it causes food shortage for the wildlife
exotic animal will have to submit an application for grant population and also adversely affect the germination and
of a license to the DGFT (Director General of Foreign growth of the native species. The Kerala Government
Trade) under the Ministry of Commerce and Industry is planning to arrest the rampant growth of invasive
(and not National Biodiversity Authority). plants, especially Senna spectabilis, in the forest areas
of the Nilgiri Biosphere Reserve (NBR), including the
Statement 3 is correct. The importer will also have to
Wayanad Wildlife Sanctuary.
attach a no objection certificate (NOC) of the chief
wildlife warden of the state concerned along with the Statement 4 is correct. The Cartagena Protocol on
application to DGFT. For those people who have already Biosafety to the Convention on Biological Diversity
imported exotic animals, a declaration will have to be (CBD) is an international agreement which aims to
made within six months. ensure the safe handling, transport and use of living
modified organisms (LMOs). It recognizes that there is an
60. Solution (d) urgent need to address the impact of invasive species. It
Exp) Option d is correct. states that “Each Contracting Party shall, as far as possible
Statement 1 is correct. Millet crops have a good ability and as appropriate, prevent the introduction of, control or
to sequester carbon and so help climate adaptation, eradicate those alien species which threaten ecosystems,
considering the water needs and methane emission of rice habitats or species”.
fields. 62. Solution (a)
Statement 2 is correct. Millets are nutritious, non- Exp) Option a is correct.
glutinous (non-sticky) and are not acid-forming foods,
thus making them very easy to digest. “Millets are low The tropical region accounts for greater biological
in carbohydrate and take longer to digest. That’s why diversity than the temperate regions.
breakdown of glucose is slower. It takes longer for glucose Statement 1 is correct. Speciation is generally a function
to enter bloodstream and hence blood sugar levels are of time, unlike temperate regions subjected to frequent
stable. glaciations in the past, tropical latitudes have remained
Statement 3 is correct. Millets are generally thermophilic relatively undisturbed for millions of years and thus,
(thriving at relatively higher temperatures) and xerophilic had a long evolutionary time for species diversification.
(can reproduce with limited water input). They grow in Statement 2 is correct. There is more solar energy
almost any type of soil – sandy or with varying levels available in the tropics, which contributes to higher
of acidity. They hardly need any fertilisers or irrigation. productivity; this in turn might contribute indirectly to
Millets grow well even among the sand dunes of Rajasthan, greater diversity.
where other cereals are likely to fail. Statement 3 is incorrect. Tropical regions are not subject
61. Solution (d) to frequent and major seasonal variations. Tropical
environments, unlike temperate ones, are less seasonal,
Exp) Option d is correct. relatively more constant and predictable. Such constant
Statement 1 is correct. An alien species is a species environments promote niche specialisation and lead to a
introduced outside its natural past or present distribution; greater species diversity.
if this species becomes problematic, it is termed an
invasive alien species. They are the most common 63. Solution: (d)
threat to amphibians, reptiles and mammals on The Exp) Option d is correct.
IUCN Red List; they may lead to changes in the structure Namami Gange Programme, is an Integrated Conservation
and composition of ecosystems detrimentally affecting Mission, approved as ‘Flagship Programme’ by the Union
ecosystem services, human economy and wellbeing. UN Government in June 2014 with budget outlay of Rs.20,000
SDG no.15 addresses the issues of invasive alien species. Crore to accomplish the twin objectives of effective
Statement 2 is correct. Lady Hastings, wife of the first abatement of pollution, conservation and rejuvenation of
British Governor-General who brought Water Hyacinth National River Ganga.
(native to South America) to India towards the end of Statement 1 is correct. One of NMCG’s long-term visions
the 18th century. This plant is native to Brazil’s Amazon for Ganga rejuvenation is to restore viable populations of
basin. all endemic and endangered biodiversity of the river. To
It is also referred to as the terror of Bengal given its effect address this, Wildlife Institute of India (WII), Dehradun,
on the local ecology and lives of the people. It has an effect Central Inland Fisheries Research Institute (CIFRI),
on irrigation, hydroelectric generation and navigation. It Kolkata & Uttar Pradesh State Forest Department has
also leads to a drastic reduction in fish production, aquatic been awarded projects to develop science - based aquatic
crops and an increase in diseases caused by mosquitoes. species restoration plan for Ganga River by involving

Workbook 288
.
ENVIRONMENT

multiple stakeholders along with conservation &


restoration of aquatic biodiversity.
Statement 2 is correct. Minister for Drinking Water and
Sanitation launched the ‘Ganga Gram’ project as part of
the government’s clean Ganga mission, named Namami
Gange, for holistic sanitation development in 4,470
villages on the banks of the river.
Statement 3 is correct. Industrial Effluent Monitoring is
one of the main pillars of Namami Gange programme.
Regulation and enforcement through regular and surprise
inspections of Grossly Polluting Industries (GPIs) is
carried out for compliance verification against stipulated
environmental norms. The GPIs are also inspected on
annual basis for compliance verification of the pollution
norms and process modification, wherever required
65. Solution: (c)
through third party technical institutes.
Exp) Option c is correct
Statement 4 is correct. One of the pillars of Namami Statement 1 is correct. Tropics account for their greater
Gange programme is river front development. As a part biological diversity. Higher diversity in tropical areas is
of it 58 Ghats/Crematoria projects for construction, because of various factors:
modernization, and renovation of 254 Ghats/Crematoria Tropical latitudes have remained relatively undisturbed
and Kunds/Ponds have been initiated. for millions of years.
There are no unfavourable seasons in tropics. Continued
64. Solution: (c) favourable environment has helped tropical organisms to
gain more niche specialisation and increased diversity.
Exp) Option c is correct.
More solar energy is available in tropics. This promotes
Centre of diversity refers to the geographic region in higher productivity and increased biodiversity.
which greatest variability of a crop occurs. A primary Resource availability is higher in tropics.
centre of diversity is the region of presumed origin, and There is reduced competition in tropics due to favourable
secondary centres of diversity are regions of high diversity environment.
which have developed as a result of subsequent spread of Rate of extinction is low in tropics.
a crop. Statement 2 is incorrect. Temperate areas (and not
Statement 1 is correct: Vavilovian centres of Biodiversity tropical areas) have undergone frequent glaciation in
are those geographical spots where any organism - plant the past. Since speciation is a function of time, frequent
glaciation in temperate areas killed most of the species.
or animal - whether wild or domesticated, developed No such disturbance occurred in tropics where species
its distinct characteristics first of all. These were first continued to flourish and evolve undisturbed for millions
identified by Nikolai Vavilov (Russian geneticist and of years. Thus, tropics had a long evolutionary time for
plant breeder) in 1924. An understanding of this concept species diversification.
is significant for understanding genetic diversity, which 66. Solution: (a)
in turn is important for crop improvement. Exp) Option a is correct
Statement 2 is correct: Vavilov proposed two types of Mass extinctions are deadly events caused by various
centre of diversities - Main and subsidiary. There are 8 factors and result in collapse of ecosystem and supporting
main centres and 3 subsidiary centres. India is one of the life forms. However, they open up the planet for new
forms of life to emerge.
main centres of Vavilovian Diversity.
Statement 1 is correct. More than 99 percent of all
The other main centres are - China, Central Asia, Asia organisms that have ever lived on Earth goes extinct
Minor, Mediterranean, Abyssinia (Ethiopia), Central during Mass Extinctions. As new species evolve to fit
America and South America. ever changing ecological niches, older species fade away.
But the rate of extinction is far from constant. At least a
The subsidiary centres are - Indo malaya, Chile, Brazil- handful of times in the last 500 million years, 75 to more
Paraguay. than 90 percent of all species on Earth have disappeared

289 Workbook
.
ENVIRONMENT

in a geological blink of an eye in catastrophes called mass biome has certain distinct features and characteristics that
extinctions. remain more or less constant throughout.
Statement 2 is incorrect. The single biggest driver of mass Statement 1 is correct: Tundra is the biome farthest
extinctions appears to be major changes in Earth’s carbon from the Equator, in both Northern and Southern
cycle (and not climate change) such as large igneous Hemispheres. It is the biome encompassing the Poles
province eruptions, huge volcanoes that flooded as well as the thin strip of regions just adjacent to it,
hundreds of thousands of square miles with lava. These between the latitudes of 65 and 90 degrees. On the other
eruptions ejected massive amounts of heat-trapping gases hand, Taiga biomes are found next to Tundra biomes,
such as carbon dioxide into the atmosphere, enabling between the latitudes of 55 to 65 degrees.
runaway global warming and related effects such as ocean Taiga is the biome that immediately follows the Tundra
acidification and anoxia, a loss of dissolved oxygen in biomes, in the direction of the Equator. It is found in
water. Northern parts of Europe, Asia and North America
Statement 3 is correct. The Cretaceous-Paleogene (northern Canada), Scandinavian countries (Norway,
extinction event is the most recent mass extinction and Sweden, etc).
the only one definitively connected to a major asteroid
impact. Some 76 percent of all species on the planet,
including all nonavian dinosaurs, went extinct. About
66 million years ago, an asteroid roughly 7.5 miles across
slammed into the waters off of what is now Mexico’s
Yucatán Peninsula at 45,000 miles an hour. Overnight, the
ecosystems that supported nonavian dinosaurs began to
collapse.
67. Solution: (a)
Exp) Option a is correct.
Primary production is defined as the amount of biomass
or organic matter produced per unit area over a time
period by plants during photosynthesis. The rate of
biomass production is called productivity.
Statement 1 is correct. Primary productivity depends
on the nature of the plant species inhabiting a particular
area. It also depends on a variety of environmental factors,
availability of nutrients and photosynthetic capacity of
plants. So, it varies in different types of ecosystems.
Statement 2 is incorrect. The primary productivity
in oceans is much less than primary productivity of
land. The annual net primary productivity of the whole Statement 2 is incorrect: Animals of the Tundra region
biosphere is approximately 170 billion tons (dry weight) have long (not short) lifespans. For example, Polar
of organic matter. Of this, despite occupying about 70 per bears. This is because animals in cold climates have large
cent of the surface, the productivity of the oceans is only size (small size, large surface area, more loss of heat) to
55 billion tons. prevent heat loss. Animals with bigger bodies have
Unlike terrestrial ecosystems, the majority of primary slower heart rates, and slower heart rates have been
production in the ocean is performed by phytoplankton found to be correlated with longer lifespans. Such big
that is generally confined to the littoral zone. The sunlight animals like Polar bears, walruses, etc are largely absent
zone of the ocean is called the photic zone or euphotic in Taiga, which also has an abundance of smaller animals
zone; it is a thin layer upto 10 to 100 m near the Ocean’s like rabbits, foxes, minks, etc
surface where there is sufficient light for photosynthesis Statement 3 is correct: Taiga biome is the last biome
to occur. where any trees are found. All other trees containing
biomes are closer to the Equator and hence warmer. The
68. Solution: (c) cold temperatures, snow cover, podzolized acidic soils
Exp) Option c is correct. lacking nitrogen, are all features that contribute to low
An ecosystem is the structural and functional unit of the productivity in the Taiga biome compared to other forest
environment around us. Earth has a variety of ecosystems, ecosystems. The trees produce less food due to harsh
which can be grouped into certain categories on the basis of conditions. Fewer animals are supported by them,
the abiotic factors (light & water availability. Temperature, whose numbers are also affected by the harsh climate, thus
etc) and biotic components (types of plants and animals reducing the overall productivity of this biome. However,
inhabiting them). A certain category of ecosystems spread the productivity of Taiga biomes is still greater than that
over large areas constitute a unit called a Biome. Each of Tundra biomes. Tundra has even lesser vegetation

Workbook 290
.
ENVIRONMENT

and animal population than Taiga, as its climate is even climate is cold with long winters and short summers. The
harsher. dominant trees are conifers like spruce, pines and firs
Statement 4 is correct: One of the biggest and most with needle like leaves, that can survive extremely cold
obvious differences in the Tundra and Taiga biomes is winters.
with respect to the presence of trees. The Tundra biome The predominant plants of Tundra biome are lichens,
completely lacks trees. This is due to the permafrost on mosses, grasses, sedges and dwarf woody plants.
the soil, which doesn’t let the roots of a tree stabilize. The
Taiga region is not as cold as Tundra, and while of poor
quality, the soil in Taiga biome is capable of supporting
softwood coniferous trees like Pine and Spruce, creating
what is called a Boreal Forest.
69. Solution: (b)
Exp) Option b is the correct answer.
Biomes are groups of ecosystems that have the same
climate and dominant communities. They are complex
terrestrial (mostly earth’s surface) systems of abiotic and
biotic factors that cover a large area and are characterized
by certain soil & climate characteristics and by certain
groupings of plants and animals.
Statement 1 is incorrect: Ecosystems are the interactions Major World Biomes
between biota, such as plants and animals, within the
environment, and many ecosystems can make up a single Pair 2 is correctly matched: Tropical deciduous or
biome. Nutrient and energy flow also play a critical role seasonal forests, also known as monsoon forests, are
in ecosystems. A habitat, on the other hand, is specific found in South East Asia, in Central and South America,
to a population or species; it is the area in which that Northern Australia, Western Africa and the Pacific.
group lives. Meanwhile, biomes describe life on a much Rainfall in these forest region is higher than tropical
larger scale than either habitats or ecosystems. Biomes rain forests region but are seasonal in nature. There are
can be differentiated by the organisms residing there pronounced wet and dry periods, and so are winter and
and by the climate, as well as the fact that the organisms summer seasons. The soils are brown in colour and rich
within a biome share adaptations for that particular in nutrients.
environment. Vertical stratification in the vegetation is somewhat simple
Statement 2 is correct: The distribution of biomes results with a single understory tree layer. The trees loose leaves
from insolation, precipitation and temperature. Climate, in one season but ground vegetation remains evergreen.
terrain (or geography) and ocean and wind currents ‘Teak’ and ‘sal’ forests are good examples of this type of
also play important roles. Insolation is measured by the biome. Trees are of 20-30 m height, sometimes reaching a
amount of solar energy received per square centimetre per height over 40 m in this biome. Bamboo is also one of the
minute. As the sun rotates around the sun, the position climax shrubs in these areas.
of the land masses will change resulting in various Pair 3 is correctly matched: Tropical rainforests are found
concentrations of solar radiation over the land masses. on both sides of the equator in South East Asia, Africa,
The atmosphere and its circulation systems determine South and Central America, North East Australia. Both
where moisture-carrying air masses do and do not go. temperature and humidity are very high and constant.
The energy source of those circulatory systems is the sun. Rainfall exceeds 200 cm a year and is distributed over
The sun’s energy drives atmospheric movements, sustains the year.
photosynthesis, and propels the seasons. There is a distinct stratification of vegetation in rainforests.
The dominant plants in these forests are tall, 25-30 meters
70. Solution: (c)
high with slender trunks that branch only near the tops
Exp) Option c is the correct answer. forming a dense canopy of leathery evergreen leaves.
The primary factors that influence the formation Epiphytes and liana are very common. Wherever there
of terrestrial biomes are climatic, like precipitation, is a gap in the canopy, a thick many layered ground
temperature, seasonal extremes and winds. Topography vegetation consisting of bushes, herbs, shrubs, ferns,
and light are also important factors. Of all these, mosses etc., can also be seen.
precipitation is the chief limiting factor that determines
whether a biome would be a forest or a desert or a 71. Solution: (b)
grassland. Exp) Option b is the correct answer.
Pair 1 is incorrectly matched: Coniferous Forests and Statement 1 is incorrect: The most important difference
Taiga biome stretches across North America to Eurasia. between a steppe and a savanna is where it is located.
‘ The taiga is a land of lakes, bogs and marshes. The Savannas lie closer to the equator than steppes and, thus,

291 Workbook
.
ENVIRONMENT

are warmer than steppes. Being closer to the rainforest processes that moderate natural phenomena. Regulating
means that savannas have two major seasons: a hot, wet services include pollination, decomposition, water
summer and a marginally cooler, but much drier winter. purification, erosion and flood control, and carbon
Steppes, by contrast, lie further from the equator and in storage and climate regulation.
sheltered areas. This means the precipitation they get is Supporting services are ecosystem services that are
dispersed evenly throughout the year. necessary for the production of all other ecosystem
Statement 2 is correct: The term “steppe climate” denotes services. Some examples include biomass production,
the climate encountered in regions too dry to support a production of atmospheric oxygen, soil formation and
forest but not dry enough to be a desert. Steppe soils are retention, nutrient cycling, water cycling, and provisioning
typically of the chernozem type. Steppes Grassland are of habitat.
practically treeless and the grasses are much shorter. Option 1 is correct. Ecosystems influence the local
Savannas Grassland feature widely spaced, scattered climate and air quality. For example, trees provide shade
trees. In many savannas, tree densities are higher and whilst forests influence rainfall and water availability
trees are more regularly spaced than in forests. both locally and regionally. Trees or other plants also play
an important role in regulating air quality by removing
72. Solution: (b)
pollutants from the atmosphere.
Exp) Option b is correct
Option 2 is incorrect. Providing living spaces for plants
Loss of biodiversity occurs when either a particular or animals is the ‘supporting service’ (and not regulating
species is destroyed or the habitat essential for its survival services) of the ecosystem. Ecosystems provide living
is damaged. Natural causes of biodiversity loss includes spaces for plants and animals; they also maintain a
floods, earthquakes, landslides, rivalry among species diversity of complex processes that underpin the other
and lack of pollination and diseases. ecosystem services.
Option 1 is correct. Decline in pollinators could cause Option 3 is correct. Ensuring soil fertility is the regulating
major disruptions in natural ecosystems, including loss service provided by ecosystem. Vegetation cover prevents
of biodiversity. Pollinators may decline naturally as well soil erosion through natural biological processes such
as due to human activity. As so many plants species are as nitrogen fixation. Soil erosion is a key factor in the
highly reliant on pollinators, if the numbers of pollinators process of land degradation, loss of soil fertility and
decline, or different pollinators become the most desertification, and contributes to decreased productivity
numerous, reproduction of many plant species is likely to of downstream fisheries.
be affected. This would likely lead to declines in numbers Option 4 is correct. Pollination is another regulating
of some of those plants. service of ecosystem. Insects and wind pollinate plants
Option 2 is correct. Rivalry among species is another and trees which is essential for the development of fruits,
natural cause for the loss of biodiversity. Invasive vegetables and seeds. Animal pollination is an ecosystem
species can disrupt ecosystems because they are better service mainly provided by insects but also by some birds
competitors than native species; they may consume food and bats. Pollinators such as bees, birds and bats affect
resources more rapidly or more efficiently or take over 35 percent of the world’s crop production, increasing
habitats faster than native species can adapt to the new outputs of around 75% of the leading food crops
changes. worldwide.
Option 3 is incorrect. Habitat destruction has been mostly Option 5 is incorrect. Maintaining a diversity of
caused by human activity for the purpose of development plants and animals is the ‘supporting service’ (and not
and urbanisation. The world’s forests, swamps, plains, regulating service of ecosystem). Genetic diversity (the
lakes, and other habitats continue to disappear as they are variety of genes between, and within, species populations)
harvested for human consumption and cleared to make distinguishes different breeds or races from each other,
way for agriculture, housing, roads, pipelines and the providing the basis for locally well-adapted cultivars and a
other hallmarks of industrial development. Habitat loss gene pool for developing commercial crops and livestock.
poses the greatest threat to species. It is identified as a
main threat to 85% of all species described in the IUCN’s 74. Solution: (c)
Red List (those species officially classified as “Threatened” Exp) Option c is the correct answer.
and “Endangered”). Desert environment has one of the most successful
Option 4 is incorrect. Forest loss and degradation is organisms survival and reproduction rate. These
mostly caused by the expansion of agricultural land, organisms are well-suited to their environment because
intensive harvesting of timber, wood for fuel and they have special adaptations to desert conditions.
other forest products, as well as overgrazing. They are Adaptations are features of organisms that help them
anthropogenic causes for biodiversity loss. survive and reproduce. They are the traits that result from
many generations of Natural Selection.
73. Solution: (c)
Statement 1 is incorrect: Mammals inhabiting deserts
Exp) Option c is correct show remarkable adaptive traits that have evolved
A regulating service is the benefit provided by ecosystem repeatedly and independently in different species across

Workbook 292
.
ENVIRONMENT

the globe. Genomic studies have shown that there are photosynthesis performed around the world. They are
shared patterns of adaptation at the genomic level extremely important for oxygen release. Zooplanktons
involving fat metabolism and insulin signalling among are not capable of photosynthesis. They only take in
mammals living in deserts. Many mammal species oxygen and do not produce it. Zooplanktons obtain their
have successfully adapted to the physical and climatic energy by feeding on phytoplankton.
challenges of deserts. Examples are Kangaroo rats,
fennec fox, camels etc. 76. Solution: (c)
Statement 2 is correct: Condensation is a major avenue Exp) Option c is the correct answer.
for water input to Desert ecosystems. Many plants Dissolved oxygen enters water through the air or as
in arid regions are adapted to acquire water through a plant by-product. From the air, oxygen can slowly
condensation. Some desert insects also acquire water diffuse across the water’s surface from the surrounding
through condensation on specialized hairs or body parts. atmosphere, or be mixed in quickly through aeration,
Organisms such as cacti and desert beetles can survive in whether natural or man-made. The aeration of water can
arid environments because they’ve evolved mechanisms be caused by wind (creating waves), rapids, waterfalls,
to collect water from thin air. The Namib desert beetle, ground water discharge or other forms of running water.
for example, collects water droplets on the bumps of its The actual amount of dissolved oxygen (in mg/L) depends
shell while V-shaped cactus spines guide droplets to the on temperature, pressure and salinity of the water body.
plant’s body.
Statement 1 is incorrect: Dissolved oxygen decreases
Statement 3 is correct: Desert plants have several types of exponentially as salt levels increase. That is why, at the
adaptations that help them conserve water. This includes; same pressure and temperature, saltwater holds about
a leathery or waxy coating on the leaves and stems reduces
20% less dissolved oxygen than freshwater. Therefore,
evaporation. Thick stems or other plant parts provide
decreasing amount of salinity increases the dissolved
water storage space. Small leaves or spines (modified
oxygen level in the water body.
leaves) reduce the surface area of the plant exposed to
the sun. Spines and fine hairs reflect heat and reduce Statement 2 is incorrect: The solubility of oxygen
the air flow over the plant’s surface. Many plants have decreases as temperature increases. This means that
a combination of these adaptations. For example, the warmer surface water requires less dissolved oxygen to
hedgehog cactus has enlarged stems, a thick waxy coating reach 100% air saturation than does deeper, cooler water.
and a dense cover of spines For example, at sea level (1 atm or 760 mmHg) and 4°C
(39°F), 100% air-saturated water would hold 10.92 mg/L
75. Solution: (c) of dissolved oxygen. But if the temperature were raised
Exp) Option c is the correct answer. to room temperature, 21°C (70°F), there would only be
There are two main types of plankton: phytoplankton, 8.68 mg/L DO at 100% air saturation. In summary, colder,
which are plants, and zooplankton, which are animals. deeper fresh waters have the capability to hold higher
Zooplankton and other small marine creatures eat concentrations of dissolved oxygen.
phytoplankton and then become food for fish, crustaceans, Statement 3 is correct: Dissolved oxygen will increase as
and other larger species. Phytoplankton is a group of free- pressure increases. This is true of both atmospheric and
floating microalgae that drifts with the water current and hydrostatic pressures. Water at lower altitudes can hold
forms an important part of the ocean, sea, and freshwater more dissolved oxygen than water at higher altitudes.
ecosystems. Zooplankton is a group of small and floating
organisms that form most of the heterotrophic animals in 77. Solution: (b)
oceanic environments. Exp) Option b is correct.
Statement 1 is incorrect: Zooplanktons are capable of Lakes are classified into various types based on their origin
moving with or against water currents against predators or mode of formation. These can be further classified into
or competitors. Although some phytoplankton are capable the freshwater lake, saltwater lake, Natural lake, oxbow
of slight movement, usually up and down the water lake, Artificial lake, crater lake etc.
column, most are too small and weak to do anything but
be carried passively along by the current. Pair 1 is incorrect: Pulicate lake is 2nd largest brackish
water lake in India after Chilka lake. It lies along
Statement 2 is correct: Phytoplanktons are autotrophic
Coromonadal coast.
and thus, can make their own food with sunlight
and chlorophyll. They are found mostly floating on Pair 2 is correct: Kanwar lake is Asia’s largest freshwater
the surface of water bodies as they require sunlight oxbow lake and Bihar’s only Ramsar site, located in
for photosynthesis. Zooplanktons are heterotrophic Begusarai district. It is currently facing threat of running
depending on the distribution of phytoplankton for their dry.
food and energy. They remain mostly around the dark Pair 3 is correct: The Vembanad Wetland System is the
and deeper areas of water. second-largest Ramsar site of the country. It is the second
Statement 3 is incorrect: Phytoplanktons are capable of largest lagoon in India. It is in grave danger due to the
photosynthesis, being responsible for about half of the accumulation of silt on the beds and illegal filling.

293 Workbook
.
ENVIRONMENT

78. Solution: (c) 80. Solution: (c)


Exp) Option c is correct. Exp) Option c is correct
Recently, the National Green Tribunal (NGT) ordered Genetic diversity is concerned with the variation in genes
all encroachments to be removed from Gujarat’s Banni within a particular species.
grasslands. The region’s nomadic pastoralist community, Statement 1 is correct. Genetic diversity allows species
the Maldharis, whose livelihoods are depended on this to adapt to changing environments. This diversity aims
protected shrub-savanna, welcomed the move. to ensure that some species survive drastic changes and
Statement 1 is correct. The Banni Grassland Reserve, thus carry on desirable genes. The survival of individuals
(also known as Banni Grasslands) is the largest natural ensures the survival of the population.
grassland in the Indian subcontinent. They are located Statement 2 is incorrect. Genetic diversity renders them
near the southern edge of the salt flats of Rann of Kutch in the ability to resist emerging diseases and epidemics.
Gujarat. Spread across an area of 3847 Square Kilometers, On the contrary, low genetic diversity may increase the
the Banni Grassland Reserves are known for its rich susceptibility of a species to biotic and abiotic stresses
wildlife and biodiversity. such as diseases and drought, and therefore, facing a
Statement 2 is incorrect. Mankadia tribes are the higher risk of extinction in the long run.
inhabitants from Odisha. The Maldhari community is Statement 3 is correct. The population of dogs consisting
a tribe of herdsmen in the border state of Gujarat. The of different breeds of dogs is an example of genetic
Maldharis have lived in the Gir National Park, in the diversity among species. Each species is composed
Banni Grasslands Reserve area, for the past thousand of individuals with their own set of genes. A gene is
years. the inheritance unit by which generations can inherit
They have co-existed with the lions, which the Gir the features of their parents. It refers to the nucleotide
National Park was created to preserve, for these thousand sequence in the DNA strands packaged in chromosomes.
years. These genes are responsible for the expression of their
distinct features. A population would have different
Statement 3 is correct. Prosopis juliflora — a non-native genes that are responsible for the variation among its
and invasive tree species — has encroached over half of members. For example, the population of humans consists
Banni grasslands. Prosopis juliflora (mad babool) species of individuals with different physical traits reflecting their
were introduced in the 1960s to prevent desertification. genetic diversity
The tree is harmful to ecology; yet, local communities
have over time grown dependent on it for livelihood by,
for example, making charcoal. 5.3. Animal diversity in india
81. Solution (a)
79. Solution: (a)
Exp) Option a is correct.
Exp) Option a is correct
Statement 1 is correct. Black Browed Babbler is a
A species is often defined as a group of organisms that can
songbird species in the family of Pellorneidae, recently
reproduce naturally with one another and create fertile rediscovered in south-eastern Kalimantan, the part of
offspring. Borneo that is administered by Indonesia.
Option a is correct. Endemic species are those species Features of the bird: The upper parts of the bird were rich
of plants and animals which are found exclusively in a brown while the underparts up to the breast were greyish
particular area. They are not naturally found anywhere with fine white streaking. The bird has a broad black eye
else. Endemic species are unique to a certain water body, stripe and the iris was found to be deep red. The legs are
place or region. For example: Sal and wild mango are two dark slate-grey.
examples of the endemic flora of the Pachmarhi Biosphere
Reserve. As the endemic species has very narrow ecological Statement 2 is incorrect. IUCN Status: Data Deficient.
niche and requirement for physical environment, habitat However, IUCN says that the global population size of the
bird has not been quantified, but the species is described
conservation is integral part of endemic species.
as possibly extinct.
Option b is incorrect. Endangered species includes those
Statement 3 is incorrect. Black-browed Babbler is not
species which are in danger of extinction due to decline in
found in India.
their numbers.
The single specimen of the bird was first found between
Option c is incorrect. Species which are not the natural 1843 through 1848 during an expedition in Indonesia.
inhabitants of the local habitat but are introduced into the However, after that, the species was never seen in the wild
system, are called exotic species. again. The bird is known for the longest known missing
Option d is incorrect. Vulnerable Species includes the period (170 years) for any Asian species. It is also often
species which are likely to be in danger of extinction in called ‘the biggest enigma in Indonesian ornithology’
near future if the factors threatening to their extinction
continue. Survival of these species is not assured as their 82. Solution (d)
population has reduced greatly. Exp) option d is correct.

Workbook 294
.
ENVIRONMENT

Statement 1 is correct: Monotremes are the only egg- Critically Endangered since 2002.
laying mammals. Only two kinds of egg-laying mammals
are left on the planet today: the duck-billed platypus and 84. Solution (c)
the echidna known as Monotremes. Both species are Exp) Option c is correct.
found in New Guinea, Australia, and Tasmania. Snow leopards are distributed across the 100,146 sq.
Statement 2 is correct: The only surviving examples of km of snowy forests in five Himalayan states - Jammu
monotremes are all indigenous to Australia and New & Kashmir, Uttarakhand, Himachal Pradesh, Sikkim &
Guinea although there is evidence that they were once Arunachal Pradesh. The range of snow leopard in India
more widespread including some extinct species in South is area above 3000m.
America.
Hemis National Park, Gangotri National Park,
Statement 3 is correct: Monotremes have a slightly lower
Khangchendzonga National Park and Great Himalayan
body temperature compared to ordinary mammals, a
National Park are some protected areas where snow
feature observed in reptiles. They are fairly poor at
maintaining a constant body temperature during extreme leopards are known to be found.
environmental conditions. Habitat loss, poaching and increasing conflict with
Knowledge Base: Monotremes are different from other communities have seen the world’s snow leopard
mammals because they lay eggs and have no teats. The population reduce drastically. Climate change is now
milk is provided for their young by being secreted by putting the future of their mountain home at even greater
many pores on the female’s belly. risk.
The Platypus Secure Himalaya project, launched by the Union
Environment Ministry in collaboration with United
Nations Development Program (UNDP) will cover
Himachal Pradesh, Jammu and Kashmir, Uttarakhand
and Sikkim. Among the key areas of focus, the project
will work on curbing the snow leopard smuggling and
poaching.
85. Solution (b)
Exp) Option b is correct.
Statement 1 is incorrect. Himalayan Brown Beer is
an endangered species in India but is categorised as
The Echidna LEAST CONCERN as per IUCN and is spread over
North-western and central Himalaya, including India,
Pakistan, Nepal, the Tibetan Autonomous Region of
China and Bhutan.
Statement 2 is correct. Its habitat includes high altitude
open valleys and pastures.
Statement 3 is incorrect. All beer species hibernate only
in winter and not in summers. Hibernation or “winter
sleep” is the state of inactivity or low metabolic process
performed by the animals during winters. Aestivation or
“summer sleep”, on the other hand, is the low metabolic
process by the animals during summers.
86. Solution (c)
83. Solution (a) Exp) Option c is correct.
Exp) Option a is correct. All the statements are correct
Pair 1 is correct. The Indian vulture/long billed vulture The Great Indian Bustard is one of the heaviest flying
(Gyps indicus) is an Old World vulture native to India, birds in the world.
Pakistan and Nepal. It has been listed as Critically
It inhabits is dry grasslands and scrublands on the
Endangered on the IUCN Red List since 2002.
Indian subcontinent.
Pair 2 is incorrect. The Egyptian vulture also called
the white scavenger vulture or pharaoh’s chicken is It is endemic to the Indian subcontinent. In India, the
endangered on the IUCN Red List. population confines mostly to Rajasthan and Gujarat.
Pair 3 is incorrect. The slender-billed vulture is native It is a Critically Endangered species.
to sub-Himalayan regions and Southeast Asia and is It is in Schedule I of Wildlife (Protection) Act,1972.

295 Workbook
.
ENVIRONMENT

87. Solution (b) the details have been released. The census is conducted
Exp) Option b is correct. once every five years. India uses block counting method
to estimate the numbers.
Pair 1 is correctly matched. Ganges River dolphins occur
in the Ganges-Brahmaputra River system primarily in 90. Solution (c)
India and Bangladesh. They are listed as Endangered by
Exp) Option c is correct.
the IUCN.
Statement 1 is correct. Gharial is found mostly in fresh
Pair 2 is correctly matched. Malabar Civet is a critically
water of Himalayan rivers. The population of gharial is a
endangered species which is endemic to Western Ghats,
good indicator of clean river water (and hence acts as an
Kanyakumari, Wayanad, Coorg etc. It lives in lowland
indicator species).
forest, lowland swamp and riparian forest.
Statement 2 is correct. As per the red book of IUCN
Pair 3 is incorrectly matched. Snow leopards are
gharials are critically endangered species. They are also
officially recognized as Vulnerable on the IUCN Red List
protected under Schedule I of Wildlife Protection Act,
of Threatened Species. It is found in Jammu and Kashmir,
1972.
Himachal Pradesh, Sikkim, Uttarakhand in India. It needs
habitat of alpine and sub alpine ecological zone favoring Statement 3 is incorrect. India has 3 (and not just
steep terrain. two species) species of crocodiles: mugger or marsh
Pair 4 is incorrectly matched. crocodile, saltwater crocodile and gharial.
Red Panda is listed as Endangered in the IUCN red list 91. Solution (b)
of Threatened Species and under Schedule I of the Indian Exp) Option b is correct.
Wildlife (Protection) Act, 1972.
Statement 1 is correct. Asian Elephants are included in
It is found in the forests of India, Nepal, Bhutan and the list of the Appendix I of the Convention of the Migratory
northern mountains of Myanmar and southern China. species in the recently concluded Conference of Parties
88. Solution (d) of CMS 13 at Gandhi Nagar, Gujarat in February 2020.
Under CMS COP 13 mainland Asian Elephant gets
Exp) Option d is correct.
global protection.
Statement 1 is correct. Pygmy hog is a critically
Statement 2 is incorrect. Monitoring of Illegal Killing
endangered species and one of the indicator species
for the management status of grassland habitats. The of Elephants (MIKE) Programme was established by
grasslands where the pygmy hog reside are crucial for the Convention on International Trade in Endangered
survival of other endangered species. Species of Wild Fauna and Flora (CITES) (and not by
CMS) adopted at the tenth Conference of the Parties in
Statement 2 is incorrect. Namdapha flying squirrel is
1997. It was started in South Asia in 2003.
only one of its genus and is restricted to a single valley
in the Namdapha Wildlife Sanctuary or National park 92. Solution (d)
(and not in Barak Valley) in Arunachal Pradesh.
Exp) Option d is correct.
Statement 3 is incorrect. Currently only one-horned
rhinos inhabit India. But previously India also had Option d is correct. The Ministry of Environment Forest
Sumatran and Javan rhinos which are now thought to be and Climate Change (MoEF&CC) has released ‘Marine
regionally extinct. Mega Fauna Stranding Guidelines’ and ‘National Marine
Turtle Action Plan’.
89. Solution (d)
Following are the types of seas turtle found in India:
Exp) Option d is correct. 1. Green turtle
Here the key word to identify the solution is Block 2. Olive Ridley
counting method which is used to identify large mammals
3. Loggerhead
in India like elephant, lion etc.
4. Hawksbill
Asiatic lion- India is home to the only Asiatic lion
population. 5. Flatback
It is listed under Schedule I of the Wild Life (Protection) 6. Leather back
Act, 1972. 93. Solution (d)
The International Union for the Conservation of Exp) Option d is correct.
Nature (IUCN) classified the status of Asiatic lions
as‘Endangered’. Asiatic Lion Conservation Project” The Great Indian Hornbill is the largest hornbill found in
has been recently launched by the Union Ministry of India. It lives in the broad-leaved evergreen and moist
Environment, Forests and Climate Change (MoEFCC). deciduous forests of the Western Ghats and the Nilgiris.
At present Gir National Park and Wildlife Sanctuary is It is the state bird of Kerala. It is found in the evergreen
the only home to the Asiatic lion. Census of Asiatic lion forest of Kerala and are also distributed along the foothills
was recently conducted by the Gujarat government and of the Himalayas in the North East India.

Workbook 296
.
ENVIRONMENT

In the field of ecology, classifying a species as a generalist


or a specialist is a way to identify what kinds of food and
habitat resources it relies on to survive.
Statement 1 is correct. Koala (Phascolarctos cinereus) is
an example of specialist species. Specialist species eat a
limited diet and occupy a much narrower niche. Native
to Australia, koalas are herbivorous marsupials that feed
only on the leaves of the eucalyptus tree. Therefore, their
range is restricted to habitats that support eucalyptus
trees. Within this diet, some koalas specialize even further
and eat leaves from only one or two specific trees. This can
pose a problem when environmental disruptions, like
effects from climate change or habitat loss, occur. Such
disturbances have a strong effect on specialists because
they cannot adapt to use other food sources or habitats as
94. Solution (c) quickly as generalist species.
Exp) Option c is correct. Statement 2 is incorrect. Specialist species evolved to
fit a very specific niche. Example: Canada lynx (Lynx
The Jerdon’s Courser is a nocturnal bird found only in the
canadensis) is a carnivorous specialist which preys upon
northern part of Andhra Pradesh in peninsular India.
snowshoe hare. It inhabits the forested, mountainous areas
The species was considered to be extinct until it was favoured by their prey. In contrast, generalist species can
rediscovered in 1986 and the area of rediscovery was feed on a wide variety of things and thrive in various
subsequently declared as the Sri Lankamaleswara Wild- environments. Raccoons (Procyon lotor) are an example
life Sanctuary. of a generalist species. They can live in a wide variety
Its habitat is undisturbed scrub jungle with open areas. of environments, including forests, mountains, and
large cities, which they do throughout North America.
95. Solution (b) Raccoons are omnivores and can feast on everything from
Exp) Option b is correct. fruit and nuts to insects, frogs, eggs, and human trash.
Statement 1 is incorrect. Indian Pangolin is a large Other examples of generalist species include bobcats and
anteater covered by 11-13 rows of scales on the back. It is coyotes.
widely distributed in India, except the arid region, high
98. Solution: (c)
Himalayas and the North-Eastern part of India. It can
be found at an elevation of up to 2500 m. The species also Exp) Option c is correct.
occurs in Bangladesh, Pakistan, Nepal and Sri Lanka. Bird migration is the regular seasonal movement, often
The species is understood to occur in various types of north and south along a flyway, between breeding
tropical forests as well as open land, grasslands, and and wintering grounds. Many species of bird migrate.
degraded habitats, including in close proximity to villages. Migration carries high costs in predation and mortality,
Statement 2 is correct. The Indian Pangolin is legally including from hunting by humans, and is driven
protected under Schedule I of the Indian Wildlife primarily by availability of food.
(Protection) Act, 1972 and is also categorised as Option 1 is correct: Greater Flamingo (Phoenicopterus
‘endangered’ by the International Union for Conservation roseus) is the most widespread and largest species of
of Nature (IUCN). the flamingo family. It is found in Africa, the Indian
Statement 3 is correct. Major threats to pangolins in subcontinent, the Middle East, and in southern Europe.
India are hunting and poaching for local consumptive Because of its large range and increasing population it is
use (e.g. as a protein source and traditional medicine) classified as Least Concern under IUCN Red Data List.
They migrate from Siberia to India to breed and escape
96. Solution (c) the harsh winters.
Exp) option c is correct. Option 2 is incorrect: The Lesser Florican is a type of
Heat loss or heat gain is a function of surface area. Since a bustard that is endemic to the Indian subcontinent
small animals have a larger surface area relative to their (concentrated in northwestern and western parts) and
volume, they tend to lose body heat very fast when it is does not migrate from the outside. It is found in tall
cold outside; then they have to expend much energy to grasslands. It is listed as Critically Endangered in the
generate body heat through metabolism. This is the main IUCN Red Data List. It migrates seasonally, but within
reason why very small animals are rarely found in polar India itself.
regions. Option 3 is correct: The Western European Roller
(Coracias garrulus) is a migrating bird. Its overall
97. Solution: (c) range extends into the Middle East, Central Asia and
Exp) Option c is correct Morocco. It is smaller and more blue in colour than the

297 Workbook
.
ENVIRONMENT

Indian Roller (Coracias benghalensis - also called the encourage faster decay and recycling of nutrients back
Kashmiri Roller). The Western European Roller breeds into living forest.
in the temperate zone. European Roller migrates from
Central Asia through India and across the Arabian Sea 101. Solution: (c)
to spend the winter in Africa. Exp) Option c is correct.
Option 4 is correct: Amur Falcon is a migrant bird and Statement c is correct. Net Present Value is a mandatory
the longest travelling species of the raptor family. It is one-time payment that a user has to make for diverting
classified under Least Concern in the IUCN Red List as forestland for non-forest use, under the Forest
it has huge populations. They breed in southeast Russia (Conservation) Act, 1980.This is calculated on the basis of
and northern China and winter in southern Africa, often the services and ecological value of the forests. It depends
congregating in huge roosts on passage through India. on the location and nature of the forest and the type of
Amur falcons migrate west through India and across industrial enterprise that will replace a particular parcel
the Arabian Sea to southern Africa where they spend of forest.
their winters, making a round-trip of at least 20,000 Statement a is incorrect. A Carbon Credit (and not
km every year, travelling between their breeding and Net Present Value) is a tradeable certificate or permit
wintering grounds. Because of the long trip, the stopover representing the right to emit one tonne of Carbon or
is extremely important. Doyang Lake in Nagaland is a Carbon Dioxide equivalent (tCO2e). The value of Carbon
famous stop in this route. Credit varies according to market conditions. One
99. Solution: (a) Carbon Credit is equal to one tonne of Carbon Dioxide.
The concept of Carbon Credit originated in the Kyoto
Exp) Option a is correct Protocol of UNFCCC.
In hot desert ecosystem, where environmental conditions Statement b is incorrect. Net primary productivity
are very extremes, the desert animals have physiologically (and not Net Present Value) refers to the rate of the
and behaviourally adapted themselves to desert conditions. accumulation of energy in the form of biomass. It excludes
They have developed following characteristics. energy lost in the metabolic processes.
Statement 1 is correct. They are Nocturnal in habit. Statement d is incorrect. Compensatory Afforestation
This is the primary strategy for dealing with high desert Fund (and not Net Present Value) can be used for
temperatures. It means many mammals simply avoid treatment of catchment areas, assisted natural generation,
the high daytime temperatures by being nocturnal or forest management, wildlife protection and management,
crepuscular (that is dusk- or dawn-active). For instance, relocation of villages from protected areas, managing
a javelina is never active during the day in summer, and a human-wildlife conflicts, training and awareness
bobcat is typically most active at dawn. generation, supply of wood saving devices and allied
Statement 2 is correct. Desert animals are adapted to activities.
have long legs. It is to ensure they keep their body away
from the hot ground, For instance, The kangaroo rat and 102. Solution: (c)
jerboa have long legs, which help them in jumping and Exp) Option c is correct.
swift running as well as in lifting the body above the The ten percent law of transfer of energy from one trophic
ground. Thus, it helps in reducing direct contact with the level to the next can be attributed to Raymond Lindeman.
hot sand. But Lindeman did not call it a “law” and instead called it
Statement 3 is correct. They are adapted to conserve ecological efficiencies.
water by excreting concentrated urine. For example, A The number of trophic levels in the grazing food chain
kangaroo rat produces urine twice as concentrated as sea is restricted as the transfer of energy follows 10 per cent
water and feces five times drier than a lab rat’s droppings. law – only 10 per cent of the energy is transferred to each
Statement 4 is incorrect. Camel do not store water in trophic level from the lower trophic level in a grazing
their hump. The camel in the desert ecosystem can go for food chain.
long periods without water. It is because their body tissues
have ability to tolerate elevation in body temperature and
also a degree of dehydration. It uses water for temperature
regulation. In contrary to popular belief, camels do not
store water in their hump. Their hump stores fat which
yields water after its metabolic oxidation.
100. Solution: (a)
Exp) Option a is correct.
Despite the amount of vegetation in the tropical rain
forest, the soil contains less organic matter than that of 103. Solution: (b)
temperate forests, because the warm humid conditions Exp) Option b is correct.

Workbook 298
.
ENVIRONMENT

Following is a short discussion on the comparison between 104. Solution: (d)


features of the Tropical Rainforest and the Temperate Exp) Option d is correct
Deciduous biomes.
Grassland’s biomes in the world can be categorized into
Statement 1 is incorrect: Both Tropical rainforests and temperate grassland and tropical grassland of the world.
Temperate Deciduous forests have hardwood trees. Further, following are the major temperate grassland
Tropical rainforests have trees like rosewood, ebony, found across the world.
mahogany, etc, all of which are hardwood. Temperate
Deciduous forests have broadleaf trees like oak and Pair 1 is correct. Eurasian Steppes is the largest temperate
maple which are hardwood. Temperate Deciduous grassland biome in the world. It is located from Hungary
forests also have some softwood trees like pines and to China. Its largest areal extent is found in Russia
spruce, which are absent in Tropical rainforests wherein it extends from eastern Europe to western Siberia
and between temperate coniferous forest in the north and
Statement 2 is correct: The upper layer of soils of
arid regions in the south-west.
both the biomes is deficient in nutrients, though due
to different reasons. The upper fertile layers of soil in Pair 2 is correct. South American Pampas is the
Tropical rainforests are washed away and eroded by the temperate grassland. Its most extensive cover is found in
heavy rains which occur daily. This strips the soil of a lot Argentina where these account for about 15 percent of its
of its nutrients. total geographical area. The South American Pampas are
On the other hand, in Temperate Deciduous forests, the comparatively more humid than the Eurasian Steppes
soil is highly podzolized and most of the nutrients get and North American Prairies.
leached into the lower layers where they accumulate, Pair 3 is incorrect. South African Veld is a temperate
making its upper soil layers nutrient deficient as well. grassland. It has developed on the high plateau land of
Statement 3 is correct: Tropical rainforests are mainly the South Africa. It includes the temperate grasslands of
located between the latitudes of 23.5°N (the Tropic southern Transwaal and Orange Free State of South Africa
of Cancer) and 23.5°S (the Tropic of Capricorn)—the and Some parts of Lisotho. Unlike, South American
tropics. Tropical rainforests are found in Central and Pampas which are developed on flat lowland terrain
South America, western and central Africa, western India, whereas the South African Velds have developed over
Southeast Asia, the island of New Guinea, and Australia. 1500-2000 m high plateau surface
Temperate Deciduous forests are found in eastern North Pair 4 is incorrect. Australian Downs is a temperate
America, northeastern Asia, and central and western grassland biome. It is found in the south-eastern parts
Europe. of Australia and the northern part of Tasmania. It is
characterized by relatively warmer winter season than
the temperate grasslands of the northern hemisphere and
mixture of grasses with eucalyptus trees.
105. Solution: (d)
Exp) Option d is correct.
Nilambur forests and teak plantations are the major
sources of quality teak wood in India. It is an important
timber tree in India and known worldwide for its unique
quality for shipbuilding and yacht manufacturing.
Nilambur teak is internationally known for its superior
Tropical Rainforests around the world quality and elegant appearance.
Statement 1 is correct. Nilambur teak is grown in
Nilambur region of Kerala. It is also known as Malabar
teak and the Mecca of Teak. It was accorded Geographical
Indication (GI) tag by the Geographical Indication (GI)
Registry in 2017. It is the first forest produce to get GI tag.
Statement 2 is correct. Nilambur teak is famous for its
larger size, superior colour and durability. Its durability
is attributed to the synergistic effect of total extensive
components.
Statement 3 is correct. They are highly resistive to fungal
decay. The hydrophobicity, anti-oxidant properties and
oily nature are due to a caoutchouc compound. The
heartwood of teak is extremely resistant to attack of
insects and fungi.

299 Workbook
.
ENVIRONMENT

106. Solution: (b) tree itself) to overcome the problem of germination in salt
Exp) Option b is correct. water.
Kelp forests are extensive underwater habitats that range Statement 2 is correct - Mangrove plants have (additional)
along 25% of the world’s coastlines. They grow best in cold, special roots such as prop roots, pneumatophores
nutrient-rich water, where they attain some of the highest which help to impede water flow and thereby enhance
rates of primary production of any natural ecosystem. the deposition of sediment in areas (where it is already
occurring), stabilize the coastal shores, provide breeding
Statement a is incorrect. Mangroves are the forests that ground for fishes. It prevents coastal soil erosion and
grow in sheltered low-lying coasts, estuaries, mudflats, protects coastal lands from tsunami, hurricanes and
tidal creek backwaters, marshes and lagoons of the floods. The dense roots of mangroves protect the coasts
tropical and subtropical regions. They are salt-tolerant or from tides, tsunamis etc.
halophytes and are adapted to harsh ecological conditions.
Statement 3 is correct – According to the India State
E.g., Sonneratia and Avicennia.
Forest Report 2019, there is an increase in 54 square
Statement b is correct. Kelp forests are large brown algae km. in the area under mangroves compared to 2017.
seaweeds. They grow in “underwater forests” in shallow The mangrove cover in India is 4,975 sq km, which is
oceans and nutrient-rich waters. Generally speaking, kelps 0.15% of the country’s total geographical area. Among the
live farther from the tropics than coral reefs. However, states and UTs, West Bengal has the highest percentage of
a few species have been known to occur exclusively in area under total Mangrove cover followed by Gujarat and
tropical deep waters. Andaman Nicobar Islands.
They are considered as keystone species. Statement 4 is correct - Mangroves are almost
They help reduce coastal erosion and acts as a breakwater exclusively tropical or subtropical. t is confined only
during large storms. to tropics and sub-tropics between latitudes 25 N and
25 S. They require high solar radiation to filter the saline
They are an important source of potash and iodine. water through their roots. Beyond a particular latitude the
Many kelps produce algin, a complex carbohydrate amount of sunlight decreases and hence the productivity
useful in industries such as tire manufacturing, ice-cream decreases. The number of mangrove species declines with
industry. increasing latitude, with the most northerly and southerly
Statement c is incorrect. Vallisneria is commonly called mangroves being species of Avicennia. In temperate
the eel grass or tape grass. It is the most commonly found regions, mangroves are replaced by salt marsh vegetation.
aquatic weed plant. It was introduced as an ornamental The number of mangrove species declines with increasing
aquarium plant in India. The genus is widely distributed latitude.
in tropical and subtropical regions of Asia, Africa,
108. Solution: (a)
Europe, and North America. In Malayalam, it is known as
thalayattippullu, vazhakala and mudipayal. It was recently Exp) Option a is correct
in news as it is posing a threat to paddy cultivation in the A rainforest is an area of tall, mostly evergreen trees and a
state of Tamil Nadu. high amount of rainfall.
Statement d is incorrect. Recently, the Surge in nitrogen Statement 1 is correct. More than half of the world’s
has turned sargassum into the world’s largest harmful animals live in the rainforest. Vampire bats and
algal bloom. Sargassum is a genus of large brown seaweed anacondas live in the rainforests of South America.
(a type of algae) that floats in island-like masses and never Bengal tigers and orangutans live in Asia’s rainforests,
attaches to the seafloor. The Sargasso Sea, lying within and chimpanzees live in the rainforests of Africa. Lots
the Norwegian current circuit, is full of large quantities of smaller animals live in rainforests too, including
of Sargassum seaweed and is an important geographical dragonflies, tree frogs, and at least hundreds of species of
feature. ants.
107. Solution: (d) Statement 2 is incorrect. Most rainforests are found
along or near the Equator, where it tends to be hot. The
Exp) Option d is correct equatorial latitude of tropical rainforests and tropical
Mangroves are the characteristic littoral plant formation of deciduous forests keeps day length and mean temperature
tropical and subtropical sheltered coastlines. Mangroves fairly constant throughout the year. The climate is always
are trees and bushes growing below the high-water level hot and wet in most parts of the equatorial belt.
of spring tides which exhibits remarkable capacity for salt However, some rainforests grow in temperate regions
water tolerance. where it’s cooler. Hoh Rainforest in Olympic National
Statement 1 is correct. Mangroves are salt-water Park on the Pacific Northwest coast of North America
tolerant. Mangroves are salt-tolerant variety of plants is an example of a temperate rainforest. Like tropical
that grow in warm tropical and subtropical tidal areas rainforests, temperate rainforests get lots of rain.
like estuaries and marine coastlines. They have the ability Statement 3 is correct. Canopy is a deep layer of
to absorb fresh water from saline water. Also, it exhibits vegetation roughly 6 meters (20 feet) thick in rainforest.
viviparity mode of reproduction (seeds germinate in the The canopy has a dense network of leaves and branches

Workbook 300
.
ENVIRONMENT

which forms a roof over the remaining layers. The canopy the government, and involves either prohibitions or taxes
blocks winds, rainfall, and sunlight, creating a humid, for their extraction and transit.
still, and dark environment below. Trees have adapted to Option 1 is correct: Fruits and plant parts harvested
this damp environment by producing glossy leaves with outside of forest areas are not considered forest products
pointed tips that repel water. according to the Indian Forest Act, 1927.
109. Solution: (b) However, all plants, leaves, fruits and flowers extracted
Exp) Option b is correct. from within a forest are classified as Forest Produce.
Silviculture is the art and science of controlling the Option 2 is correct: Animal parts like silk cocoons,
establishment, growth, composition, health, and quality or the silk extracted from it is not considered a Forest
of forests in order to meet the various requirements of Produce if reared outside forest.
landowners and wildlife and ecosystem. An important However, animal parts like horns, tusks, ivory, bones,
aspect of this work is regeneration of forests that have skin, etc if extracted from animals, whether wild or not,
been destroyed either due to natural reasons (like forest from those within forests is termed as Forest Produce.
fire, flood, etc) or due to human interventions (large scale
commercial purpose deforestation). There are two ways of Option 3 is incorrect: Tendu leaves, whether extracted
regenerating forests - natural and artificial. from trees inside forests or outside of it, have been
classified as Forest Produce by the Indian Forest Act,
In Natural Regeneration of Forests, the forests that
1927.
are allowed to grow from seedlings created by natural
pollination (sexual method of plant breeding), natural Tendu leaves are mostly sold by tribals to tobacco
selection and germination. companies for making bidis and their sale is an important
Option 1 is incorrect: Clear Felling System of seedling livelihood source for the tribals.
growth is the natural methods of forest regeneration. Option 4 is incorrect: Timber, whether from a tree in
In the Clear-Felling System, regeneration is allowed to forest area, or whether from outside a forest area, is
happen on an area from where the mature standing crop classified as a Forest Produce by the Indian Forest Act,
was cut and removed in a single go. 1927.
Other systems of natural seed regeneration of forest are
111. Solution: (d)
Shelterwood system and Selection system.
Option 2 is correct: Direct Sowing of seeds of varieties Exp) Option d is correct.
manually selected in the area where regeneration is to be India has a rich repository of wild and domestic plants
done, is an artificial method of forest regeneration. that have a range of medicinal uses that have been used
Option 3 is correct: Manual selection and Planting traditionally in Ayurvedic and Unani medicines to treat
involves transferring of seedlings or plants of chosen a range of diseases by the locals. Many of these belong
tree species in the area to be regenerated after they have to the biodiversity hotspot of Himalayan region. Their
successfully passed the critical phase of germination unsupervised collection by modern pharma companies
and initial development. The planting stock is generally has resulted in their depletion.
raised in a nursery. It is an artificial method of forest Pair 1 is correct: The scientific name of Indian Snakeroot
regeneration. (also known as Sarpagandha) is Rauvolfia Serpentina.
Option 4 is incorrect: Coppice regeneration is another It has been traditionally harvested and used as an anti-
Natural method of Forest Regeneration. In this method, hypertensive (lowering blood pressure) and as a mild
rather than using the natural seed formation, selection sedative. It is also used in Ayurveda to treat asthma and
and germination, there is natural growth of new shoots insomnia. It is a critically endangered species belonging
from either the base of seedlings that had been pruned/ to the family Apocynaceae. The Union Ministry of
burnt or stumps of trees and vegetation that has been Environment and Forests has red-listed this plant
harvested/ burnt (though not fully removed by roots). following its overexploitation.
In contrast to the Natural pollination and seedling
method, this is an asexual mode of reproduction in plants. Pair 2 is correct: The Cinchona is a genus of trees native
It is generally used in the case of Teaks and Sals. to the Andes of South America. The bark of this tree
yields a medicine known as Quinine. It has been used in
110. Solution: (a) the treatment of malaria.
Exp) Option a is correct. Pair 3 is correct: The scientific name of the Himalayan
In India, the forests are governed by a host of regulations, Yew is Taxus Wallichiana. It is an endangered medicinal
some of them colonial. The Indian Forest Act, 1927, is plant of the Himalayan region which has been used in
a such a colonial Act that still governs many aspects of traditional (Ayurveda and Unani) medicines by the
forests and their use by people, as well as restrictions on locals to treat cough, cold, fever and pain for centuries.
activities within it. Section 2, clause 4 of this Act defines Recently pharmaceutical research has revealed that a
what does and what does not constitute Forest Produce. derivative of this plant, named Taxol can be potentially
The extraction and use of Forest Produce is controlled by used in anti-cancer treatments.

301 Workbook
.
ENVIRONMENT

Pair 4 is correct: The scientific name of the Indian 113. Solution: (d)
Pitcher Plant is Nepenthes Khasiana. It is found in the
Exp) Option d is correct.
Garo, Khasi and Jaintia hills of Meghalaya in India. It is
an endangered species. It has been traditionally used by Statement 1 is correct: Agroforestry refers to a practice
locals for a variety of treatments. The paste of this plant of land management, wherein woody shrubs/ trees are
and its pitcher contents, is applied to the skin of leprosy grown alongside agricultural crops and livestock, on the
patients to treat their skin. The fluid of unopened pitcher same piece of land. There are 3 varieties:
are used by Khasi and Garo tribes as eye drops to cure Agrisilvicultural systems - combination of crops and
cataract and night blindness, and in treating stomach trees, such as alley cropping or homegardens.
troubles, diabetes and gynaecological problems. Silvopastoral systems - combination of forestry and
112. Solution: (b) grazing of domesticated animals on pastures, rangelands
or on-farm.
Exp) Option b is correct.
Agrosilvopastoral systems - Integration of the three
Forests store large amounts of carbon. Trees and other elements, trees, animals and crops
plants absorb carbon dioxide from the atmosphere as
Statement 2 is correct: Under agroforestry system trees
they grow. This is converted into carbon and stored in
and crops can be grown and arranged both temporally
the plant’s branches, leaves, trunks, roots and in the
and spatially. Whether the combination of trees and crops
soil. When forests are cleared or burnt, stored carbon is
is arranged spatially (simultaneous growth) or temporally
released into the atmosphere, mainly as carbon dioxide.
(sequentially, one after the other on the same piece of
Averaged over 2015—2017, global loss of tropical forests land), both the systems are deemed to be Agroforestry
contributed about 4.8 billion tonnes of carbon dioxide systems.
per year (or about 8-10% of annual human emissions of
carbon dioxide). Statement 3 is correct: In 2013, on the first day of World
Congress on Agroforestry organised in Delhi, India
Statement 1 is incorrect: Carbon stored in tree trunks, or created history by becoming the first country in the
for that matter in other parts of trees and plants like stem, world to adopt an official National Policy on Agroforestry.
leaves, roots, etc, releases carbon post deforestation,
but only when it starts to decay, not immediately. For 114. Solution: (c)
example, if the wood from a cut tree trunk is used to Exp) Option c is correct.
create furniture, the carbon stored in it will remain locked
until the furniture falls into disuse and decays naturally Climate Change has been identified as a leading global
by the action of decomposer microorganisms. issue by scientists across the globe. It is considered a
threat to the survival of main species of plants, animals,
Thus, forests act as both sink and source of carbon in and of humans themselves. It is important to understand
the atmosphere. When living, they sequester the carbon the various natural sinks of carbon like tropical rainforests
dioxide in air during photosynthesis to manufacture and coastal ecosystems in order to combat this climate
sugars that are used to create leaves, barks, roots etc. Upon change.
death, fallen leaves or rotting tree bark, etc decompose to
release back stored nutrients including carbon into the Statement 1 is correct: A study by the Yale School
atmosphere. of Forestry and Environment Studies recently has
found that Tropical Rainforests across the globe are
Statement 2 is correct: Deforestation disturbs the cumulatively responsible for storing a quarter (25%) of
hydrological cycle in a given area, by reducing the cycling the world’s carbon.
capacity of the system. Live plants and trees increase
the moisture in the atmosphere through transpiration Amazon - the biggest tropical rainforest of the earth, is
which moves water stored in vegetation roots and soil. estimated to store upto 140 billion tonnes of carbon.
This contributes to the moisture in air from other sources Approximately 2.6 billion tonnes of carbon dioxide,
and decides the quantum of rainfall in an area. Also one-third of the CO2 released from burning fossil fuels,
forests are important in ensuring that the water which is absorbed by forests every year.
comes down as precipitation gets absorbed in the soil in Statement 2 is incorrect: The oceanic planktons, and not
the area, whence it can be cycled again. Tropical rainforests are the biggest source of oxygen in
Statement 3 is correct: As mentioned above, the roots the biosphere. It is believed by the scientific community
of trees bind the soil particles together. This in turn that around 50-80% of our planet’s oxygen comes from
promotes the absorption of water that precipitates. This the photosynthesis carried out by oceanic planktons in
prevents runoff which may otherwise result in flooding the oceans. A particular species, Prochlorococcus (the
in downstream areas, as there can be a sudden rise in smallest photosynthetic organism on Earth) produces up
the volume of water flowing in the river channel beyond to 20% of the oxygen in our entire biosphere. That’s a
its capacity, as none of it was absorbed. Recent instances higher percentage than all of the tropical rainforests on
like floods in hilly states in India, can be traced back to land combined.
increased deforestation activities, which have destabilised Statement 3 is correct: Coastal ecosystems include
the soils there, while promoting run offs. ecosystems such as mangroves, tidal marshes, seagrass

Workbook 302
.
ENVIRONMENT

meadows, etc. These ecosystems, especially the seagrass


meadows, absorb twice the amount of carbon per unit
area compared to terrestrial forest ecosystems such as
tropical rainforests.
The carbon stored in coastal (as well as marine
ecosystems) is termed as Blue Carbon, and is a significant
carbon sink.

115. Solution: (c)


Exp) Option c is the correct answer.
According to Economic Survey 2021-22, India has
increased its forest area significantly over the past decade.
It ranks third globally in an average annual net gain in The top 10 countries account for 66 per cent of the world’s
forest area between 2010 to 2020, adding an average of forest area.
2,66,000 ha of additional forest area every year during the Of these countries, Brazil (59 per cent), Peru (57 per
period, or adding approximately 0.38 per cent of the 2010 cent), the Democratic Republic of Congo (56 per cent),
forest area every year between 2010 to 2020. and Russia (50 per cent) have half or more of their total
Forest area refers to the area recorded as forest in geographical area under forests.
government records and is also called recorded forest
area. 116. Solution: (c)
Statement 1 is correct. India has the tenth largest forest Exp) Option c is the correct answer.
area in the world. Russia, Brazil, Canada, the US, and India has increased its forest area significantly over the
China were the top five largest countries by forest area in past decade. India’s total forest cover was 7,13,789 sq km
2020, while India was the tenth largest. in 2021, reflecting an increase of 3.14 per cent in the
Forests covered 24 per cent of India’s total geographical forest cover over 2011, moving from 21.05 per cent of the
area [Economic Survey says 24% and India’s State of country’s geographical area in 2011 to 21.71 per cent in
Forest Report (IFOR 2021) says 24.62%] accounting for 2021.
two per cent of the world’s total forest area in 2020.

This increase in total forest cover is mainly attributed to


the rise in very dense forests (all lands with tree canopy
density of 70 per cent and above), which rose by 19.54 per
cent between 2011 and 2021.
Statement 1 and 2 are incorrect. Forest cover comprises
all lands, more than one hectare in area, with a tree
canopy density of more than 10 per cent, irrespective of
ownership and legal status.Such lands may not necessarily
be a recorded forest area, and also include orchards,
Statement 2 is correct. Russia is home to the largest area bamboo and palm plantations.
of forest – 815 million hectares. Statement 3 is correct. Amongst states, Madhya Pradesh
Brazil, the United States, Canada, China, Australia, and (11 per cent of India’s total forest cover) had the largest
the Democratic Republic of Congo also have a largest forest cover in India in 2021, followed by Arunachal
forest area – more than 100 million hectares each. Pradesh (9 per cent), Chhattisgarh (8 per cent), Odisha (7
per cent) and Maharashtra (7 per cent).

303 Workbook
.
ENVIRONMENT

elements as well as pests, and hence favoured the colonial


masters, who only wanted to profit from the resources of
India. This proved to be disastrous for the local ecosystem,
because
The dry wood of Pines is more susceptible to forest fires
causing large scale biodiversity loss and emissions due
to increased instances of forest fires.
The resin oozing from pines discouraged the growth of
other vegetation in the soil in its vicinity, causing loss of
biodiversity.
Statement 4 is correct. Mizoram (85 per cent), Arunachal 118. Solution: (d)
Pradesh (79 per cent), Meghalaya (76 per cent), Manipur Exp) Option d is correct.
(74 per cent) and Nagaland (74 per cent) were the top five India is not only famous for its diverse wildlife,
states in terms of highest per cent of forest cover w.r.t. architectural marvels and culture, but also for its dense
total geographical area of the state in 2021. and vast forest cover. Indian climate befits the variety of
flora and fauna.
Forest is the second largest land use in India next to
agriculture. The forest cover of India is assessed as 67.83
million hectares which constitute 20.64 per cent of the
country’s geographical area, ranging from the Himalayan
Temperate to Dry Zone forests.
Pair 1 is correct: Tidal Forests are the forests found on
the coasts along estuaries and bays and gulfs. They are
also known as Mangroves. These forests are populated
by trees that are adapted to tolerate high salinity. They
117. Solution: (a) have adaptations like air roots, salt secreting leaves
and viviparous seeds to survive the saline and anoxic
Exp) Option a is correct. conditions. Some examples of these trees are Sundari
The Forest department of the British government was (Heritiera Minor), Avicennia, Agar, Bhendi, Keora,
designed to maximise production of commercially Nipa, etc. India’s largest mangrove - the Sunderbans -
valuable timber such as teak, pine, etc. For this purpose, have a huge population of Sundari trees.
without understanding the indigenous soil, climate or Distribution - Sunderban Delta (West Bengal), Krishna
vegetation, they introduced exotic species which they Delta, Godavari Delta, Kaveri Delta, Gulf of Kutch,
believed were commercially profitable, without worrying Andaman & Nicobar Islands.
about the long-term consequences. Indeed, most of
these introductions led to invasions and loss of precious Pair 2 is incorrect: Tropical Deciduous Forests are those
indigenous vegetation. forests that are found in regions of moderate rainfall
(70-200 cm annually). These are also known as Monsoon
Option 1 is correct: Prosopis Juliflora, more commonly Forests. They are the most widespread in India. Their
known as the Mexican Mesquite (tree native to Mexico), most notable feature is that they shed all their leaves
was a tree alien to the Indian subcontinent. It was first during the summer or the winter season to withstand the
imported to India in 1877, and soon it spread out across heat and minimise loss of water through transpiration.
North India (as it is an invasive species). It was brought Some classic examples of the type of trees found in
in to fulfil fuelwood requirements in dry arid areas. these forests are Teak, Sal, Sheesham, Mahua, Kusum,
Option 2 is correct: The British introduced eucalyptus Sandalwood, Amaltas, Khair, tendu, Palas, Bel, etc.
(an Australian native) in the Nilgiris and the plateaus
Distribution - Foothills of Himalayas, Plains of Uttar
around it. This was because they believed the wood of
Pradesh, Bihar, eastern slopes of Western Ghats,
Eucalyptus would serve well as fuelwood. They cleared
Jharkhand, Chhattisgarh, Odisha.
the unique local Shola vegetation of the Nilgiris to make
space for this exotic species which caused significant loss Junipers are trees generally found in Alpine forests, i.e.
of unique and endemic plant and animal biodiversity, the forest found at a height between 2900m and 3500m
as well as stress on local resources like groundwater, as Pair 3 is incorrect: Khejri is mostly found in tropical
the water guzzling eucalyptus with deep roots was not thorn forests. Montane Forests are those forests that
suited to this region. are found in mountain or hilly regions with higher
Option 3 is incorrect: Oaks were the original native altitudes. Their speciality is that they have various forms
species found widespread in the Western Himalayas. of vegetation which depend on the altitude. They can
However, the Britishers undertook large scale plantation be deciduous, wet-temperate, and temperate in nature.
of Pines in this region. This was because Pine wood Some examples of trees found in these types of forests
was considered superior and more resistant to weather are pine, deodar, spruce, silver fir, oak, beech, birch,

Workbook 304
.
ENVIRONMENT

rhododendrons, chestnut, maple, etc along the Western Ghats. The species is named after M
Pair 4 is correct: Tropical Thorn Forest are those forests K Janarthanam, professor of Botany at Goa University
that are found in arid areas with scant rainfall of about for his contributions in Indian grass taxonomy and
50-100cm annually. Vegetation here is stunted and documentation of Goa’s floristic diversity. Ischaemum
scattered. The roots are long and go deep underground janarthanamii grows on low altitude lateritic plateaus in
to draw on the little moisture available at depths. Their the outskirts of Bhagwan Mahavir National Park, Goa.
leaves evolve into thorns to prevent loss of moisture by The vegetation is exposed to extreme climatic conditions
transpiration. Some other examples of trees found in this like desiccation in drier months and soils with low nutrient
type of forest are - khejri, ber, babool, neem, etc. availability. However, withstanding these, the species has
Distribution - Kutch, Saurashtra, south-western Punjab, adapted to survive harsh conditions and blossom every
western Haryana, western and northern Rajasthan, monsoon. It is also of economic significance as it is
upper Ganga plains, the Deccan Plateau and the lower commonly used as fodder.
Peninsular India.
122. Solution (d)
5.4. Plant diversity of India Exp) Option d is correct.
119. Solution (a) Statement 1 is correct. Like terrestrial plants, seagrass
also photosynthesise and manufacture their own food and
Exp) option a is correct. release oxygen.
Statement a is incorrect. Ashwaganda is globally
Statement 2 is incorrect. They grow in shallow and calm
distributed in Africa, Mediterranean to India and
SriLanka. water.
Statement b is correct. Within India, it is distributed in Statement 3 is incorrect. They are the group of higher
the sub-Himalayan tracts, ascending upto an altitude of plants adapted to life in the salt water.
1000 m. Statement 4 is correct. Though sea-grasses inhabit all
It is found throughout the drier parts (subtropical) of types of substratas (layers) from mud to rock, the lush
India like Rajasthan, Punjab, Haryana, Uttar Pradesh, green seagrass beds are found extensively in muddy and
Gujrat, Maharastra and Madhya Pradesh. sandy substratas.
This species grows in subtropical and tropical climates 123. Solution (c)
and prefers dry weather for successful growth.
Exp) Option c is correct.
Statement c is correct. Ashwagandha is one of the most
widespread tranquillisers used in India. Statement 1 is incorrect. Insectivorous plants are
successful to survive in nutrient poor habitats as they
Statement d is correct. Researchers of IIT-Delhi in supplement their photosynthesis food production by
collaboration with Japan’s National Institute of Advanced trapping insects and digesting their nitrogen rich bodies.
Industrial Science and Technology (AIST) have claimed
Statement 2 is incorrect. Nepenthes are commonly
that the properties of the ayurvedic herb Ashwagandha
known as pitcher plants because their leaves bear jar like
have “therapeutic and preventive value” against
structures. These are confined to high rainfall hills and
COVID-19 infection.
plateaus of north-eastern regions particularly in Garo,
120. Solution (c) Khasi and Jantia hills of Meghalaya (and not in A&N).
Exp) Option c is correct. 124. Solution (d)
Statement 1 is incorrect. The maximum diversity of Exp) Option d is correct.
fungi is in the Western Ghats (and not Eastern Ghats) Option a is correct. Alien species are those which occur
followed by eastern Himalaya and western Himalaya. outside their natural range. When they threaten native
Statement 2 is correct. Gymnosperms are naked seeded plants and animals or other aspects of biodiversity, they
plants. They do not have ovary, style and stigma, and are called invasive species.
naturally there is no fruit. Option b is correct. An invasion can lead to an increase
Statement 3 is incorrect. Algae are green non- in species richness, as invasive species are added to the
differentiated plants which possess chlorophyll (and not existing species pool. On the other hand, it also leads
absence of chlorophyll). They usually grow in water or in to extinction of native species resulting in decrease of
moist situations. species richness.
Option c is correct. Some of the invasive fauna found in
121. Solution (c) India are: crazy ant, giant African snail, myna, gold fish,
Exp) Option c is correct. donkey, house gecko, etc.
Researchers have recently discovered a species of Indian Option d is incorrect. About 40% of species in Indian
Muraingrass, named Ischaemum janarthanamii, flora are alien, of which 25% are invasive.

305 Workbook
.
ENVIRONMENT

125. Solution (b) Adventitious Roots which produced from the branches of
Exp) Option b is correct. the tree which remain suspended in the air till they reach
the ground. On reaching the ground they enter into the
Option a is correct. Sea Buckthorn is a shrub that
soil and get fixed. Ex: Banyan Tree.
produces an orange-yellow coloured edible berry.
Option b is incorrect. In India, it is found above the Statement 2 is incorrect. Stilt roots are types of
tree line in the Himalayan region. It is generally in dry Adventitious Roots which emerged from the butt of a tree
areas such as the cold deserts of Ladakh and Spiti. In above ground level. So that the tree appears as if supported
Himachal Pradesh, it is locally called Himalayan chharma on flying buttresses. Ex: Rhizphora species of mangroves.
and grows in the wild in Lahaul and Spiti and parts of
129. Solution (b)
Kinnaur.
Exp) Option b is correct.
Option c is correct. Medicinal Benefits of Sea Buckthorn:
1. It is used as a medicine for treating stomach, heart, Pair 1 is correctly matched. Prop roots are a type of
and skin problems. adventitious roots which are produced from the branches
2. It is rich in vitamins, carotenoids, and omega fatty of the tree and remain suspended in air till they reach
acids. Moreover, it can help troops in acclimatizing ground.
to high-altitude. Pair 2 is incorrectly matched. Tap root is the primary
Option d is correct. Commercial Benefits of Sea descending root formed by the direct prolongation of
Buckthorn: the radical of the embryo.
• It is used in making juices, jams, nutritional capsules Those are lateral roots which are spread laterally to
among others. support the tree.
It is also used in the manufacturing of cosmetics and anti- Pair 3 is correctly matched. Haustorial roots are the
ageing products. roots of parasitic plants that can absorb water from water
and nutrients from another plant.
126. Solution (b)
130. Solution (c)
Exp) Option b is correct.
Exp) Option c is correct.
Black Wattle: INVASIVE ALIEN FLORA OF INDIA
Option a is correct. Die back refers to progressive dying
Black wattle is one of the most invasive tress species in
which is usually backwards from the tip of any portion
the world and is endemic to South East Australia. It is
of the plant.
distributed in forests and grazing lands in high altitude
areas. In India, it is found in Western Ghats. Option b is correct. A dense overhead canopy leads to
an inadequate source of light thus creating conditions
It was introduced for afforestation in Western Ghats as it
adverse for the plant sustainability. As a result, it starts
regenerates rapidly after fire and forms dense thickets.
to die back. Other reasons for die back are dense week
127. Solution (a) growth, un-decomposed leaf litter on the surface, frost,
Exp) Option a is correct. drought, etc.
Option c is incorrect. In this process only the shoots of
Statement 1 is correct. The Himalayan trillium (Trillium
the plant dies immediately but roots remains alive for
govanianum), is a common herb of the Himalayas. It is a
years.
robust, trifoliate perennial herbaceous plant species with
deep red and green coloured flowers on the axis. The herb Option d is correct. Die back is a self-defense mechanism
has been used in traditional medicine to cure diseases or adaptation which is kicked in when there are adverse
like dysentery, wounds, skin boils, inflammation, sepsis conditions for plant growth.
etc.
Statement 2 is incorrect. It was declared ‘endangered’
by the International Union for Conservation of Nature
(IUCN).
There are several factors threatening the survival of the
plant such as over-exploitation, long life cycle - slow to
reach reproductive maturity - and poor capacity for
seed dispersal. The highly specific habitat requirement,
high trade value, and increasing market demand are all
causing its decline.
128. Solution (d)
Exp) Option d is correct.
Statement 1 is incorrect. Prop roots are types of

Workbook 306
.
ENVIRONMENT

The major inorganic nutrients required by phytoplankton


5.5. Marine organism for growth and reproduction are nitrogen and
131. Solution (d) phosphorus. Some phytoplankton can fix nitrogen and
can grow in areas where nitrate concentrations are low.
Exp) Option d is correct.
They also require trace amounts of iron which limits
Statement 1 is correct. Mangrove plants have special phytoplankton growth in large areas of the ocean because
roots such as prop roots, pneumatophores which help to iron concentrations are very low
impede water flow and thereby enhance the deposition
of sediment in areas. They also help in stabilizing the 134. Solution (c)
coastal shores and preventing coastal soil erosion. Exp) Option c is correct
Statement 2 is correct. Existence of extensive network
Statement 1 is correct. Seaweeds can act as a bio-
of prop roots impede the flow of water, which ensures
indicator. When waste from agriculture, industries,
the conditions for developing breeding ground for
aquaculture and households are let into the ocean, it
fishes. They provide a safe and favorable environment for
causes nutrient imbalance leading to increase in the
breeding, spawning, rearing of several fishes.
population of seaweed, which acts as a sign of marine
Statement 3 is correct. Mangroves by acting as natural chemical damage. A bio-indicator is a living organism
barriers, protect coastal lands from tsunami, hurricanes that gives us an idea of the health of an ecosystem.
and floods. They also moderate monsoon tidal floods
and reduce inundation of coastal lowlands. They protect The importance of seaweed in agriculture and animal
coastal inland from other adverse climatic impacts. husbandry is noteworthy. They can be used as fertilizers
and to increase fish production. Also, when livestock is
Statement 4 is correct. Mangroves enhance natural fed with seaweed, methane emission from cattle may be
recycling of nutrients. Mangroves are considered as reduced substantially.
one of the most productive ecosystems in the world with
significant contributions as carbon sinks in the biosphere. Seaweeds are the primitive, marine non-flowering marine
algae without root, stem and leaves, which play a major
132. Solution (c) role in marine ecosystems.
Exp) Option c is correct. Statement 2 is correct. Most heavy metals found
Statement 1 is incorrect. Phytoplankton are microscopic in marine ecosystems are trapped and removed
plant organisms that live in aquatic environments, both by seaweeds. Also, seaweeds heavily rely on iron for
salty and fresh. photosynthesis. When quantity of this mineral exceeds
healthy levels and becomes dangerous to marine life,
Statement 2 is incorrect. Some phytoplankton are seaweeds trap it and prevent damage.
bacteria, some are protists, and most are single-celled
plants. Statement 3 is correct. Large seaweeds form dense
underwater forests known as kelp forests, which act as
Statement 3 is correct. Phytoplanktons are limited to underwater nurseries for fish, snails and sea urchins.
the uppermost layers of the ocean where light intensity The herbivorous marine animals also feed on its thallus.
is sufficient for photosynthesis to take place. The The thousands of species of this organism that vary vastly
photosynthetic rate varies with light intensity. in size, shape and colour, provide habitats for marine
133. Solution (c) lifeforms and protect them from threats.
Exp) Option c is correct. Statement 4 is incorrect. The presence of seaweeds
is not essential for the survival of coral reefs. On the
Statement 1 is incorrect. Phytoplankton are limited contrary, some rare species of seaweed clash coral reefs
to the uppermost layers of the ocean where light and damage them severely. Some of the seaweed species
intensity is sufficient for photosynthesis to take place. releases hydrophobic allelochemicals (chemicals present
The photosynthetic rate varies with light intensity. These on the surface of algae that repel water) on direct contact
micro-algae are present throughout the lighted regions of with corals. These chemicals decrease photosynthesis,
all the seas and oceans including the Polar Regions. causes bleaching and thus leading to death of corals.
Statement 2 is correct. Their total biomass is many
times greater than that of the total plants on land 5.6. Protected Area network
and they serve as the “pasture grounds” in the aquatic
environment. 135. Solution (a)
Statement 3 is correct. Phytoplankton are microscopic Exp) Option a is correct.
plant organisms that live in aquatic environments, both Statement 1 is correct. Tiger Reserves are notified by
salty and fresh. State Governments as per provisions of the Wildlife
Phytoplankton produce more than 60% of oxygen (Protection) Act, 1972 on advise of the National Tiger
produced from all plants. All phytoplankton Conservation Authority.
photosynthesize, but some get additional energy by Statement 2 is incorrect. State Governments cannot de-
consuming other organisms. notify a tiger reserve, except in public interest with the

307 Workbook
.
ENVIRONMENT

approval of the National Tiger Conservation Authority foothills of Bhutan, and around Manas river, Sankosh
and the National Board for Wild Life. river, and Brahmaputra River in the North East India.
Statement 3 is incorrect. Ramgarh Vishdhari wildlife Statement 2 is correct. Bandipur National Park was
sanctuary has been recently approved as tiger reserve in established in 1973 under Project Tiger. In 1985, by
Rajasthan and not in Telangana. including adjacent areas from Venugopala Wildlife Park,
it was enlarged and named as Bandipur National Park.
136. Solution (b) It is situated in two contiguous districts (Mysore and
Exp) Option b is correct. Chamarajanagar) of Karnataka and is located at the
Statement a is incorrect. The Keibul Lamjao National tri-junction area of the States Karnataka, Tamil Nadu
Park is a national park in the Bishnupur district of the and Kerala. It forms a part of Nilgiri Biosphere Reserve.
state of Manipur in India. It is the only floating park in The park is located between the Kabini river in the north
the world, located in North East India, and an integral and the Moyar river in the south. The Nugu river runs
part of Loktak Lake. through the park. It is surrounded by Mudumalai Tiger
The national park is characterized by floating decomposed Reserve (Tamil Nadu) and Wayanad Wildlife Sanctuary
plant material locally called phumdi. It was created in (Kerala).
1966 as a wildlife sanctuary to preserve the natural habitat Statement 3 is correct. Pakke Tiger Reserve has won
of the endangered Eld’s deer. India Biodiversity Award 2016 in the category of
Statement b is correct. Bandhavgarh National Park is ‘Conservation of threatened species for its Hornbill Nest
a national park of India, located in the Umaria district Adoption Programme. Pakke TR lies in the undulating
of Madhya Pradesh. Bandhavgarh National Park resides and hilly foothills of the Eastern Himalayas in Arunachal
on the extreme north eastern border of Madhya Pradesh Pradesh. It is bounded by Bhareli or Kameng River in
and the northern edges of the Satpura mountain ranges. the west and north, and by Pakke River in the east.
Due to the tropical monsoon climatic zone, the park has Statement 4 is correct. The rampant spread of catfish,
been characterized by well-defined winters summers and known locally as African Mushi, in the waterbodies of the
rains and the sprouted weather definitely makes the whole Wayanad Wildlife Sanctuary (WWS) is posing danger
environment lusher and more unabridged. to the native aquatic species of the sanctuary, which is
It was declared a national park in 1968 and then became already facing threat from the wild growth of alien species
Tiger Reserve in 1993. This Park has a large biodiversity. of plants, including Senna spectabilis. Located in Kerala,
The density of the tiger population at Bandhavgarh (8 Wayanad Wildlife Sanctuary (WWS) is an integral part
tigers per km2) is one of the highest known in India. of the Nilgiri Biosphere Reserve. It is contiguous to the
tiger reserves of Nagerhole and Bandipur of Karnataka
Statement c is incorrect. Hemis National Park is a high-
and Mudumalai of Tamil Nadu. Kabini river (a tributary
altitude national park in Ladakh, India. Globally famous
of Cauvery River) flows through the sanctuary.
for its snow leopards, it is believed to have the highest
density of them in any protected area in the world. It is the 138. Solution (b)
only national park in India that is north of the Himalayas,
Exp) Option b is correct.
the largest notified protected area in India (largest National
Park) and is the second largest contiguous protected area, Statement 1 is incorrect. In the Independence Day Speech
after the Nanda Devi Biosphere Reserve and surrounding this year, Prime Minister announced the plan to launch a
protected areas. Project Dolphin. The proposed project is aimed at saving
Statement d is incorrect. Bandipur National Park, both river and marine dolphins. Project Dolphin will be
established in 1974 as a tiger reserve under Project Tiger, on the lines of Project Tiger, which has helped increase
is a national park located in the Indian state of Karnataka, the tiger population. Project Dolphin is expected to be
which is the state with the second highest tiger population implemented by the Ministry of Environment, Forest
in India. Along with adjacent Nagarhole National Park, and Climate Change.
it is one of the Premier Tiger Reserves in the country. It Statement 2 is correct. Aquatic life is an indicator of the
was once a private hunting reserve for the Maharaja of health of river ecosystems. As the Gangetic dolphin is
the Kingdom of Mysore but has now been upgraded to at the top of the food chain, protecting the species and
Bandipur Tiger Reserve. its habitat will ensure conservation of aquatic lives of the
river.
137. Solution (d)
Exp) Option d is correct. 139. Solution (a)
Statement 1 is incorrect. Guru Ghasidas National Exp) Option a is correct.
Park is the fourth tiger reserve situated in the state Option 1 is correct: Nilgiri Biosphere Reserve was
of Chattisgarh. It is contiguous to the Sanjay Gandhi created in 1986 and declared a part of UNESCO Man and
National Park of Madhya Pradesh. It is not home to Biosphere Reserve (MAB) network. It includes protected
Golden langurs. areas like Wayand, Mudumalai, Nagarhole, Bandipur,
The habitat of Golden langurs is restricted to the Sathyamangalam, Mukurthi, Nilambur, and Silent Valley.

Workbook 308
.
ENVIRONMENT

It was also declared a UNESCO World Heritage Site in India and Bangladesh have dispute over sharing of water
2012. of river Tessta, which is a right bank tributory of river
Option 2 is correct: Panna national park was created in Brahmaputra.
1981 and subsequently declared a Tiger Reserve in 1994. 142. Solution (d)
It has been recently included as a Biosphere Reserve in
UNESCO’s MAB network in November, 2020. Exp) Option d is correct
Option 3 is correct: Nokrek Biosphere Reserve was Statement 1 is correct. Mukurthi National Park is
established in 1986 and added to UNESCO’s MAB located in Tamil Nadu in the Western Ghats. It is also
a part of Nilgiris Biosphere Reserve. The park also has
network in 2009. It is a part of Meghalaya Plateau and
the distinction of being a UNESCO World Heritage Site
is the highest peak of Garo Hills. It is famous for its
because it is part of the Western Ghats.
population of Red Panda, and also for being the origin of
germplasm of Citrus Indica. Statement 2 is correct. The main reason behind
establishing a national park in the place was to protect
Option 4 is incorrect: Manas biosphere reserve was
the Nilgiri Tahr which is a Keystone species. Recently
established in 1990. It is also a tiger reserve and UNESCO
Tamil Nadu government is constructing fire lines’ or ‘fire
World Heritage Site. It is famous for rare and endemic breaks’, in the Mukurthi National Park ahead of summer
wildlife like Assam roofed turtle, hispid hare, golden and peak fire season.
langur and pygmy hog, Wild water buffalo, Bengal
Florican, Chinese Pangolin, etc. But Manas Biosphere 143. Solution (c)
reserve has not been declared as part of UNESCO Man Exp) Option c is correct.
and Biosphere Reserve (MAB) network.
Pair 1 is incorrectly matched. Nagaland Government
140. Solution (a) organises Amur Falcon festival to spread awareness
Exp) Option a is correct. about the need to provide safe passage to Amur falcons.
Statement 1 is correct: Diclofenac poisoning among Pair 2 is incorrectly matched. Nuakhai Juhar, also called
Nuakhai Parab or Nuakahi Bhetghat, is the agricultural
vultures is one the main reason for their decline in
festival of Western Odisha, southern Chhattisgarh and
population. Diclofenac is a veterinary nonsteroidal
adjoining areas of Simdega (Jharkhand). It is celebrated
anti-inflammatory drug (NSAID) used to domestic
to welcome the new crop of the season. Nuakhai is a
animals. When vultures feed the carcasses of such
combination of two words signifies eating of new rice as
animals then this harmful drug reaches in their body.
‘nua’ means new and ‘khai’ means eat.
Statement 2 is incorrect: Ministry for Environment,
Pair 3 is correctly matched. Hornbill Festival is one
Forests and Climate Change has launched a Vulture
of the largest indigenous festival organised by Nagaland
Action Plan (2020-25) for the conservation of vultures to preserve, protect and revive the uniqueness and
in the country. Whereas, Central Zoo Authority (CZA) richness of the Naga heritage. The festival is considered
and Bombay Natural History Society (BNHS) established the “Festival of Festivals” in Nagaland. The festival is
the Vulture Conservation Breeding Programme for tribute to Hornbill, known for its qualities of grandeur
captive breeding of vultures. and alertness.
141. Solution (c) 144. Solution (c)
Exp) Option (c) is correct Exp) Option c is correct.
Statement 1 is incorrect. Deepor Beel is a freshwater (not Statement 1 is correct. Soor Sarovar Lake is also known
Saltwater) lake located to the south-west of Guwahati, as Keetham lake and is situated alongside river Yamuna
in Kamrup district of Assam (not Meghalaya). It is in Agra, Uttar Pradesh. The lake is situated within the
considered as one of the largest beels in the Brahmaputra Soor Sarovar Bird Sanctuary which was declared as a bird
valley of Lower Assam, it is categorised as representative sanctuary in the year 1991.
of the wetland type under the Burma monsoon forest
Statement 2 is correct. Soor Sarovar Lake was declared
biogeographic region.
as a Wetland of International importance under
Statement 2 is correct. Considering the richness of the bird Ramsar Convention.
varieties found in the beel, the BirdLife International has
declared Deepor Beel as an Important Bird Area (IBA) 145. Solution (b)
with high priority for conservation. It is also Assam’s only Exp) Option b is correct.
Ramsar site designated in 2002 for sustaining a range of Statement 1 is incorrect. Govind National Park is located
aquatic life forms besides 219 species of birds. in the Uttarkashi district of the state of Uttarakhand. The
Statement 3 is incorrect. Deepor Beel lies to the North of Park lies in the higher reaches of the Garhwal Himalayas.
Meghalaya. It does not touch the border of Bangladesh The Park was established as a Wildlife Sanctuary in the
and there is no dispute regarding Deepor Beeel between year of 1955. It was declared as a National Park in the year
India and Bangladesh. of 1990.

309 Workbook
.
ENVIRONMENT

Statement 2 is correct. The climate in Govind Pashu Statement 2 is incorrect. As for region-wise distribution,
Vihar National Park have varied climatic zones starting the highest number of 8,071 leopards were found in
from sub-tropical (1200–1700m) to temperate (1700– central India and Eastern Ghats, stretching across Madhya
3500m) and alpine (>3500m) resulting in diverse flora of Pradesh, Chhattisgarh, Jharkhand, Odisha, Maharashtra
the region. and Northern Telangana. The Shivalik Hills and Gangetic
Statement 3 is correct. Recently The Uttarakhand High Plains landscape extending from Uttarakhand, Uttar
Court passed an interim order upholding the right of Pradesh to parts of Bihar 1253 leopards were recorded.
a qafila (caravan) of Van Gujjars (Nomadic Tribe) to In the Western Ghats which includes protected areas in
migrate to their summer homesteads in the bugyals Goa, Karnataka, Kerala and Tamil Nadu 3387 leopards
(Himalayan alpine meadows) located within the Govind were recorded while only 141 were estimated for north-
Pashu Vihar National Park in Uttarkashi district. Van eastern hills and Brahmaputra plains.
Gujjars pursue seasonal migration from the Terai-Bhabar Statement 3 is incorrect. Leopard population had
and Siwalik region of Uttarakhand to the higher bugyals increased in all states of central India when compared
in the Western Himalayas in summer and vice versa in to previous estimates of 2014. So it cannot be said
winter. This phenomenon of transhumance pursued by that the population of leopard has decreased in all states
the community is among the few climate-adaptive and during the last five years.
resilience strategies that ensures their pastoral livelihood Madhya Pradesh has the highest number of
remains viable and sustainable. leopards—3,421 followed by Karnataka—1783 and
Maharashtra—1690.
146. Solution (b)
Exp) Option b is correct. 149. Solution (b)
Statement 1 is incorrect. The National Board for Exp) Option b is correct.
Wildlife (and not Central Zoo Authority) adds species Statement 1 is incorrect: The term Deemed Forest has
to Recovery Programme for Critically Endangered not been properly defined in any legislation in India.
Species on the recommendation of a Standing Committee. The SC, in a 1996 judgement, had broadened the
Statement 2 is correct. The programme is one of the definition of forest to include not just land classified as
three components of the Integrated Development of forest under forest or revenue departments. It had then
Wildlife Habitats (IDWH) a centrally sponsored scheme. also asked states to form committees to identify forests,
It is meant for providing support to protected areas, irrespective of the nature of land ownership or whether
protection of wildlife outside protected areas and recovery they are notified, recognised or classified in a time-bound
programmes for saving critically endangered species and manner.
habitats. Deemed Forests are forests that are notified either with the
147. Solution (d) forest department or revenue department. Then there are
those areas that are like forests but are neither recorded,
Exp) Option d is correct. nor notified. The Supreme Court had ordered that the
Option 1 is correct. DNA finger-printing is a latest states identify and classify these as deemed forests,
technique which is used to identify tiger from their scats. The SC said Deemed forests = forest in dictionary
Option 2 is correct. In forest at various locations camera meaning regardless of ownership status + all categories of
traps are setup to count tigers. Tiger census 2018 used statutorily recognised forests under Section 2(1) of Forest
M-STrIPES that use GPS to geotag photo evidences Conservation Act, 1980.
and survey information. It used software CaTRAT to Statement 2 is correct: Deemed Forests occupy only 1%
automatically segregate camera trap photo graphs using of the total forest land in India.
artificial intelligence.
Option 3 is correct. Stripe patterns are used to finger 150. Solution (c)
print tigers which are captured by camera traps. This Exp) Option c is correct
method is used to count the number of individual tiger. In the estuarial Brahmani-Baitarani region of the north-
Option 4 is correct. Under pugmark technique the eastern Kendrapara district of Odisha, Bhitarkanika
imprints of pugmark of a tiger were recorded and used as National Park is located which is home to second largest
a basis of identification of individuals. mangrove ecosystem in the country.
It is a 145 Sq km large national park which was designated
148. Solution (d) on 16th September 1998 and obtained the status of a
Exp) Option d is correct. Ramsar site in August 2002 i.e, second Ramsar site of the
Statement 1 is incorrect. According to the Union State after the Chilika Lake.
Environment Ministry’s ‘Status of Leopards in India 2018’ The Park is home to the saltwater crocodile, Indian
report, there has been over 60% increase in population python, black ibis, wild boar, rhesus monkey, chital,
of leopards in India since 2014. There were at least darter, cobra, monitor lizard. Bhitarkanika has one of the
12,852 leopards in the country in 2018 compared to 7,910 largest populations of endangered saltwater crocodile
leopards estimated in 2014. in India.

Workbook 310
.
ENVIRONMENT

Recently the Odisha Forest department has started a two- details of access to biological resources and related
year conservation project for fishing cats in Bhitarkanika traditional knowledge among others.
National Park.
153. Solution (c)
Exp) Option c is correct.
Statement 1 is correct. Papum Reserve Forest (RF)
is an Important Bird and Biodiversity Area (IBAs) in
Arunachal Pradesh. It is located between two important
bird areas, Itanagar Wildlife Sanctuary to the east and
Pakke Wildlife Sanctuary to the west. Papum Reserve
Forest forms part of the Eastern Himalayas Endemic Bird
Area. Papum RF is a nesting habitat of three species of the
large, colourful fruit-eating hornbills: Great, Wreathed
and Oriental Pied.
Statement 2 is incorrect. Mishmi tribes reside in
the Kamlang Wildlife Sanctuary and tiger reserve of
Arunachal Pradesh. The Park is named after the Kamlang
River which flows through it.
151. Solution (d)
Dongria Kondh is a PVTG tribe living in Odisha.
Exp) Option d is correct.
Statement 3 is correct. Namdapha National Park lies at
Pair 1 is correct: The Pilibhit Tiger Reserve is situated the international border between India and Myanmar in
in the Pilibhit district of Uttar Pradesh, forming part of the state of Arunachal Pradesh. It is only parked in the
the Terai Arc Landscape, in the upper Gangetic Plain
World to have the four Feline species of big cat namely
Biogeographic Province. The habitat is characterized by
the Tiger (Panthera Tigris), Leopard (Panthera Pardus),
sal forests, tall grasslands and swamps, maintained by
Snow Leopard (Panthera Uncia) and Clouded Leopard
periodic flooding from rivers.
(Neofelis Nebulosa). Hoolock Gibbons, the only ‘ape’
Pair 2 is correct: The Pench Tiger Reserve is located at the species found in India is found in this National Park.
southern slopes of the Satpura range of Central India. It is
spread in the states of Maharashtra and Madhya Pradesh. Statement 4 is correct. Eaglenest Wildlife Sanctuary is a
Pench Tiger Reserve comprises of the Indira Priyadarshini protected area in Arunachal Pradesh. Red panda can be
Pench National Park, the Pench Mowgli Sanctuary and a commonly found here.
buffer. The Sanctuary conjoins Sessa Orchid Sanctuary to the
Pair 3 is correct: Ranthambhore is located at the junction northeast and Pakhui Tiger Reserve across the Kameng
(great boundary fault) of Aravalis and Vindhyan ranges in river to the east. It is a part of the Kameng Elephant
Rajasthan. There are many ruins of a bygone era scattered Reserve. Eaglenest is notable as a prime birding site.
all over the jungle, which give a unique, wonderful and Eaglenest derives its name from Red Eagle Division of the
mixed flavor of nature, history and wildlife. Indian army which was posted in the area in the 1950s.
Red panda can be commonly found here.
152. Solution (c)
154. Solution (c)
Exp) Option c is correct.
The Biodiversity Management Committees (BMCs) are Exp) Option c is correct.
statutory bodies that are formed by the local bodies under Pair 1 is correctly matched: Pobitora Wildlife Sanctuary
Biological Diversity Act 2002. is located in Morigaon district of Assam, with an area of
Composition: The BMC consists of a Chairperson and just 38.85 square kilometres, and yet it has about more
six persons, nominated by local bodies, including 1/3rd than 100 one-horned rhinos - making it the most densely
populated areas with one-horned rhino population in
women and 18% SC/ST.
the world.
Functions:
Pair 2 is incorrectly matched: Pakke Tiger Reserve is a
1. Prepare People’s Biodiversity Register in part of the Eastern Himalaya biodiversity hotspot (not
consultation with the local people. Eastern Ghats). It is located in Pakke Kessang district of
2. Conservation and sustainable utilization of biological Arunachal Pradesh.
resources.
Pair 3 is correctly matched: Recently in 2020, a picture of
3. Advice on any matter referred to it by the State a rare sighting of an Indian Golden Tiger near Kaziranga
Biodiversity Board or Authority for granting approval. National Park was widely circulated in the news and on
4. Biodiversity education and awareness building. social media. A Golden Tiger is a color morphism shown
5. Maintain a Register giving information about the by tigers due to the issue of inbreeding and weakened

311 Workbook
.
ENVIRONMENT

gene pool. This is often a result of land use change


combined with man-animal conflict and lack of safe
tiger corridors.

155. Solution (d)


Exp) Option d is correct.
Chief Wildlife Warden shall be the authority who shall
control, manage and maintain all protected areas. He may
on application, grant to any person a permit to entre or
reside in a sanctuary or national park for all or any of the
following purposes:
1. Investigation or study of wildlife
2. Tourism Statement 2 is incorrect. Maldhari is a nomadic tribe in
Gujarat and not in Uttarakhand. It is home to the Van
3. Photography
Gujjars tribes in the winters. The tribe is one of the few
4. Scientific research forest-dwelling nomadic communities in the country.
5. Transaction of lawful business with any person Usually, they migrate to the bugyals (grasslands) located
residing in the protected area in the upper Himalayas as part of their transhumance.
Hence all the statements are correct. 158. Solution (a)
Exp) Option a is correct.
156. Solution (c)
Project Lion: The programme has been launched by
Exp) Option c is correct Government of India for conservation of the Asiatic
Pair 1 is incorrectly matched: It is Maharashtra’s 2nd Lion. It aims to focus on habitat development, engaging
technologies in lion management, addressing the issues
marine sanctuary after Malvan Wildlife Sanctuary. It is
of disease in lions and will also be addressing the
located on the western flank of Thane creek. Human-Wildlife conflict
Pair 2 is correctly matched: Coringa Wildlife Sanctuary The motive behind finding a relocation site for the Asiatic
is located in Kakinada district of Andhra Pradesh. It is Lion species is because the population in Gir has low
home for the rare, endangered Smooth Indian Otter, genetic diversity making it vulnerable to threats from
Fishing Cat, and Estuarine Crocodile. The Sanctuary in epidemics.
the estuary of river Godavari supports rich mangrove Apart from Kuno Palpur Wildlife Sanctuary six new
sites identified for relocation:
vegetation.
1. Madhav National Park, Madhya Pradesh.
Pair 3 is correctly matched: Jaldapara National Park is 2. Mukundra Hills Tiger Reserve, Rajasthan.
located in West Bengal. The forest is mainly savannah 3. Gandhi Sagar Wildlife Sanctuary, Madhya Pradesh.
covered with tall elephant grasses. The main attraction 4. Sitamata Wildlife Sanctuary, Rajasthan.
of the sanctuary is Asiatic one-horned rhinoceros. The 5. Kumbhalgarh Wildlife Sanctuary, Rajasthan.
sanctuary holds the maximum number of rhino’s 6. Jessore-Balaram Ambaji WLS and adjoining
population in India after Kaziranga National Park. landscape, Gujarat.
Hence only statement 1 and 2 are correct.
157. Solution (b)
Exp) Option b is correct. 159. Solution (a)
Exp) Option a is correct
Statement 1 is correct. Rajaji National Park is an Indian
national park and tiger reserve that encompasses the Option a is correct: Tiger reserves are areas notified for
the protection of the tiger and also its prey. These are
Shivaliks, near the foothills of the Himalayas. The park
governed by Project Tiger which was launched in 1973.
is spread over three districts of Uttarakhand: Haridwar,
Option b is incorrect: A tiger reserve is notified by the
Dehradun and Pauri Garhwal. River Ganga and Song state government based on the recommendations of
passes through the park. The Park extends over the NTCA. Thus NTCA is not the final authority it only has
Shivalik Range from the Dehradun-Saharanpur road in a recommendatory role in the creation of a tiger reserve.
the north-west to the Rawasan River in the southeast, Option c is incorrect: No alteration in the boundaries of
with the Ganges dividing it into two parts. a tiger reserve can be made except on the recommendation

Workbook 312
.
ENVIRONMENT

of NTCA and the approval of National Board for reserve is managed by Conservation Reserve
Wildlife. Management Committee and Community Reserve
Option d is incorrect: A core zone is kept as inviolate for is managed by Community Reserve Management
the purpose of tiger conservation. But the creation of a Committee.
core zone of tiger reserve does not affect the rights of the
163. Solution (c)
Schedule Tribes or such other forest dwellers.
Exp) Option c is correct.
160. Solution (b) Sacred groves comprise of patches of forests or
Exp) Option b is correct. natural vegetation. These spaces are protected by local
Statement 1 is correct. Tiger reserves are notified by communities.
state government on recommendations of National Statement 1 is incorrect: The degree of sanctity of the
Tiger Conservation Authority. Also, a national park can sacred forests varies from one grove to another. For
be created by both central and state government. example, the Garo and the Khasi tribes of northeastern
Statement 2 is correct. The limits of buffer zone of a India completely prohibit any human interference in
tiger reserve are determined on the basis of scientific the sacred groves. However, the Gonds of central India
and objective criteria in consultation with concerned prohibit the cutting of a tree but allow fallen parts to be
gram sabha and an expert committee constituted for the used.
purpose. Statement 2 is correct: Sacred groves are communally
Statement 3 is incorrect. Protected Area has been protected forests which usually have a significant religious
defined in the Wildlife Protection Act, 1972. PAs in connotation for the protecting community. In India, there
India comprise National Parks, Sanctuaries, Conservation are over a lakh sacred grove across different states called
/ Community Reserves and Tiger Reserves. It does not by different names like Kaavu in Malayalam, Koyil kaadu
include Reserved Forests. in Tamil, Oran in Rajasthan, Devara kadu in Karnataka,
and Sernas in Madhya Pradesh.
161. Solution (d)
Statement 3 is correct: Sacred groves are important
Exp) Option d is correct. repositories of floral and faunal diversity that have been
Option a is correct. The reserved forests and protected conserved by local communities. The groves are often
forests differ in one important way: rights to all activities associated with ponds, streams or springs. The vegetative
like hunting, grazing, etc. in reserved forests are banned cover also helps in the recharging the aquifers. The
unless specific orders are issued otherwise. vegetation cover of the sacred groves improves the soil
Option b is correct. Protected areas are those in stability of the area and also prevents soil erosion.
which human occupation or at least the exploitation
of resources is limited. In protected areas, rights to 164. Solution (c)
activities like hunting and grazing are sometimes given Exp) Option c is correct.
to communities living on the fringes of the forest, who Pair A-2 is correct. Tso Kar wetland Complex- It acts
sustain their livelihood partially or wholly from forest as an important stopover ground for migratory birds
resources or products. along the Central Asian Flyway and is one of the most
Option c is correct. Reserved forests and protected forests important breeding areas in India for the black-necked
are declared by the respective state governments. crane.
Option d is incorrect. A reserved forest and protected Pair B-1 is correct. Kabartal Wetland in Bihar’s Begusarai
forest in India are terms denoting forests accorded district has been recognised as a wetland of international
a certain degree of protection. The term was first importance — the first such wetland in the state to be
introduced in the Indian Forest Act, 1927 (and not by recognized under the Ramsar Convention.
Forest Conservation Act 1980) in British Raj, to refer to Pair C-3 is correct. Sur Sarovar, also known as Keetham
certain forests granted protection under the British crown Lake, is a human-made reservoir. It was originally
in British India. created to supply water to the city of Agra in summer, the
wetland soon became an important and rich ecosystem.
162. Solution (b)
Exp) Option b is correct 165. Solution (d)
Statement 1 is correct: Community and conservation Exp) Option d is correct.
reserve were initially not part of the Wildlife Protection Statement 1 is incorrect: The PBR is the third in Madhya
Act 1972. They both came to existence as an outcome of Pradesh to be included in the list after Pachmarhi and
Amendments to Wildlife Protection Act 2003. Amarkantak.
Statement 2 is incorrect: State Board for Wildlife does Statement 2 is incorrect: It is situated in the Vindhya
not have an immediate control over them. Conservation mountain range in the northern part of Madhya Pradesh.

313 Workbook
.
ENVIRONMENT

There are 12 biosphere reserves of India which have been collectors of mahua flowers burn the forest floor in order
recognized internationally under Man and Biosphere to clear the dry leaf litter, which makes it easy to collect
(MAB) Reserve program. These are: 1) Nilgiri(First one flowers. Since these fires are unsupervised, they often
to be included) 2) Gulf of Mannar 3) Sunderban 4) Nanda spread to the adjacent forest areas. To collect fresh Kendu
Devi 5) Nokrek 6) Pachmarhi 7) Similipal 8) Achanakmar leaves used for bidis (the local cigarettes), the bushes are
– Amarkantak 9) Great Nicobar 10) Agasthyamala 11) slashed and set on fire for the new flush.
Khangchendzonga (2018) and 12) Panna(2020). Similipal has a unique microclimate, it sees occasional
afternoon showers during the summer months that act as
166. Solution (b) natural fire controllers. There are, however, certain years
Exp) Option b is correct. wherein dry spells last for more than two-three weeks
Option a is correct: To be declared as a biodiversity which spells disaster for the Similipal National Park.
hotspot a region must have a degree of threat. It has to 168. Solution: (b)
have lost at least 70% of its original habitat. Along with
this criteria there is another criteria. Exp) Option b is correct.
A National Park is a protected area constituted for the
protection and conservation of wildlife. National Parks
in India are declared under the Wildlife (Protection) Act
(WPA), 1972.
Statement 1 is incorrect. There are 104 existing national
parks in India covering an area of 43,716 km2, which is
1.33% of the geographical area of the country (National
Wildlife Database, Dec. 2020).
Statement 2 is incorrect. An area can be notified by the
state government to be constituted as a National Park, by
reason of its ecological, faunal, floral, geomorphological,
or zoological association or importance. It is done for
the purpose of protecting & propagating or developing
wildlife therein or its environment.
Option b is incorrect: India has total 4 biodiversity Whereas Biosphere Reserves are internationally
hotspots. These include: The Eastern Himalayas, Indo- recognized within the framework of UNESCO’s Man
Burma, Western Ghats and Sri Lanka, and Sundaland. and Biosphere (MAB) programme and nominated by
Option c is correct: Sundaland has the least area in national governments.
India. It is confined only till Nicobar Islands. A major Statement 3 is correct. No human activity is permitted
part of this hotspot is comprised of Thailand, Singapore, inside the national park except for the ones permitted by
Malaysia and Brunei. the Chief Wildlife Warden of the state under the conditions
Option d is correct: Other criteria to classify a region as given in CHAPTER IV, WPA 1972. Activities like grazing,
a biodiversity hotspot is it must contain 1500 species of hunting, forestry or cultivation etc. are strictly prohibited.
vascular plants as endemics.
169. Solution: (d)
167. Solution: (d) Exp) Option d is correct.
Exp) Option d is correct. WILDLIFE (PROTECTION) ACT, 1972 provides for the
The Similipal forest reserve area frequently witnesses appointment of wildlife advisory Board, Wildlife warden,
forest fires during dry weather conditions. A forest fire their powers, duties etc. It states that no person shall
which started in Similipal in February 2021, was finally destroy, exploit or remove any wild life including forest
brought under control after a week. produce from a sanctuary or destroy or damage or divert
Similipal, which derives its name from ‘Simul’ (silk the habitat of any wild animal by any act whatsoever or
cotton) tree, is a national park and a tiger reserve situated divert, stop or enhance the flow of water into or outside
in the northern part of Odisha’s Mayurbhanj district. the sanctuary, except under and in accordance with a
permit granted by the Chief Wild Life Warden. Also, no
Similipal biosphere reserve lies in the eastern end of such permit shall be granted unless the State Government
the eastern ghat. The forest fire in Similipal, Odisha, is being satisfied in consultation with the National Board
a phenomenon that has been in play for decades. The that such removal of wild life from the sanctuary or the
two main causes are well known: Poachers and non- change in the flow of water into or outside the sanctuary
timber forest product (NTFP) collectors. Dry, fallen is necessary for the improvement and better management
leaves provide a ready source of fuel for the fire and one of wild life therein, authorises the issue of such permit.
matchstick is enough to set the entire forest ablaze.
Scared animals eager to escape the leaping flames 170. Solution: (d)
flee straight into the arms of lurking poachers. NTFP Exp) Option d is correct.

Workbook 314
.
ENVIRONMENT

“Biodiversity Heritage Sites” (BHS) are well defined 18 Biosphere Reserves in India. Of these, 11 are part of the
areas that are unique, ecologically fragile ecosystems - World Network of Biosphere Reserves.
terrestrial, coastal and inland waters and, marine having Statement 1 is correct. In 1986, Nilgiri Biosphere
rich biodiversity. Reserve was established as the first Biosphere Reserve in
Statement 1 is correct. Dialong Village in Tamenglong India.
district of Manipur is a Biodiversity Heritage Site. Dialong
village, covering an area of 11.35 sq km, has been declared Statement 2 is incorrect. Gulf of Mannar is the largest
as Biodiversity Heritage Site under Section 37(1) of Biosphere Reserve in India.
Biological Diversity Act, 2002. Smallest Biosphere Reserve: Panna.
Statement 2 is correct. Ameenpur lake in Sangareddy Statement 3 is correct. Zonation of Biosphere Reserves:
district of Telangana is a Biodiversity Heritage Site. Core Areas
Ameenpur Lake has the distinction of being the first water
body in the country to be declared a Biodiversity Heritage Buffer Zone
Site. Transition Zone or Area of Cooperation.
Statement 3 is correct. Majuli in Assam is a Biodiversity Core zone is a protected under the Wildlife (Protection)
Heritage Site. It is an island situated in the Brahmaputra Act, 1972. Core Area includes protected areas that act as
River which is harboring unique Ecological and Cultural reference points on the natural state of the ecosystems
Heritage. represented by the biosphere reserves.
Statement 4 is correct. Ambaraguda in Karnataka is Activities allowed: non-destructive research and other
a revenue land located between Sharavathi Wild Life low-impact activities (such as ecotourism) are generally
Sanctuary and Someshwara Wildlife Sanctuary. It has
undertaken.
Shola vegetation which is primitive vegetation in the
Western Ghat and also has grasslands. Statement 4 is incorrect. Buffer Zone surrounds or is
contiguous to the core area. Activities are organized,
171. Solution: (a) so they do not hinder the conservation objectives of
Exp) Option a is correct. the core area, but rather help to protect it. It is used for
106 coastal and marine sites have been identified and cooperative activities compatible with sound ecological
prioritized as Important Coastal and Marine Biodiversity practices. Human activities are less intensive than that in
Areas (ICMBAs) by the Wildlife Institute of India. Sixty- the transition zone.
two ICMBAs have been identified along the west coast of Human settlement is allowed in the Transition Zone
India, and 44 have been identified along the east coast. or Area of Cooperation. Activities allowed: agricultural
Statement 1 is correct. Important Coastal and Marine activities, settlements, and other uses. The term ‘area
Biodiversity Areas (ICMBAs) were established to of cooperation’ underscores the role of cooperation as
achieve the National Biodiversity Target no 6 and Aichi the main tool to achieve the objectives of the biosphere
Biodiversity Target. They aim to conserve a substantial reserve.
portion of the coastal and marine areas in the country and
world respectively. 173. Solution: (c)
Statement 2 is incorrect. 22 ICMBAs (not all) have Exp) Option c is correct.
been prioritized for immediate conservation actions and PROJECT ELEPHANT was launched by the Government
proposed to be upgraded as Protected Areas under of India in the year 1992 as a Centrally Sponsored Scheme
categories such as Conservation or Communities Reserve.
with following objectives:
This will increase participation of the local communities
in governance. 1. To protect elephants, their habitat & corridors
2. To address issues of man-animal conflict
172. Solution: (c)
3. Welfare of captive elephants.
Exp) Option c is correct.
The Ministry of Environment, Forest and Climate Change
Biosphere reserves are areas of terrestrial and coastal/
marine ecosystems where both flora and fauna are provides the financial and technical support to major
protected. The concept of Biosphere Reserves was elephant range states in the country through Project
launched in 1971 as a, part of the United Nations Elephant.
Educational, Scientific and Cultural Organization The Project is being mainly implemented in 16 States /
(UNESCO)’s ‘Man and Biosphere Programme’. There are UTs through notification of Elephant Landscape reserves.

315 Workbook
.
ENVIRONMENT

Sl. No Elephant Range Location Elephant Reserve


Mayurjharna ER
Singhbhum ER
Mayurbhanj ER
West Bengal
Mahanadi ER
East-Central landscape (South-West Bengal- Jharkhand
1. Sambalpur ER
Jharkhand-Orissa) Odisha
Baitami ER
Chhattisgarh
South Odisha ER
Lemru ER
Badalkhol-Tamorpingla ER
Kameng-Sonitpur Landscape (Arunachal- Arunachal Pradesh Kameng ER
2.
Assam) Assam Sonitpur ER
Eastern-South Bank Landscape (Assam- Assam Dihing-Patkai ER
3.
Arunachal Pradesh) Arunachal Pradesh South Arunachal ER
Kaziranga-Karbi Anglong ER
Kaziranga-Karbi Anglong-Intanki Landscape Assam Dhansiri-Lungding ER
4.
(Assam- Nagaland) Nagaland Intanki ER
Singphan ER
North Bengal-Greater Manas Landscape Assam Chirang-Ripu ER
5
(Assam-West Bengal) West Bengal Eastern Dooars ER
Garo Hills ER
6. Meghalaya Landscape Meghalaya
Khasi Hills ER
Mysore ER
Karnataka Wayanad ER
Brahmagiri-Nilgiri-Eastern Ghat Landscape Kerala Nilambur ER
7.
(Karnataka- Kerala-Tamilnadu-Andhra) Tamil Nadu Nilgiri ER
Andhra Pradesh Coimbatore ER
Rayala ER
Anamalai-Nellianpathy-High Range Landscape Tamil Nadu Anamalai ER
8.
(Tamil Nadu-Kerala) Kerala Anamudi ER
Periyar-Agasthymalai Landscape (Kerala- Kerala Periyar ER
9.
Tamilnadu) Tamil Nadu Srivilliputhur ER
North-Western Landscape (Uttarakhand-Uttar Uttar Pradesh Uttar Pradesh ER
10.
Pradesh) Uttarakhand Shivalik ER
174. Solution: (c) Pair 2 is correct. Dandeli-Anshi Tiger reserve is located
Exp) Option c is correct. in Karnataka. It has been renamed to Kali Tiger Reserve.
The park is a habitat of Bengal tigers, black panthers
Tiger Reserves are areas of protection for tigers. They aim
and Indian elephants, amongst other distinctive fauna.
to conserve the habitat to ensure a viable population of the
The Kali River flows through the tiger reserve and is the
tigers along with their prey base in their habitat. There are
51 tigers reserves in India. The Project Tiger governs these lifeline of the ecosystem and hence the name.
reserves and the National Tiger Conservation Authority Pair 3 is correct. Satkosia Tiger reserve is located in
(NTCA) administers them. Odisha. It is located where the Mahanadi River passes
Pair 1 is incorrect. Parambikulam Tiger reserve is through a 22 km long gorge in the Eastern Ghats
located in Kerala. It is in the Sungam range of hills mountains.
between the Anaimalai Hills and Nelliampathy Hills. Pair 4 is incorrect. Kamlang Tiger reserve is located in
Parambikulam Wildlife Sanctuary is declared as part of Arunachal Pradesh. It is situated in the Lohit District
the Parambikulam Tiger Reserve. of the northeastern Indian state of Arunachal Pradesh.

Workbook 316
.
ENVIRONMENT

The park is named after the Kamlang River which flows rate of habitat loss. Although a Biodiversity Coldspot is
through it. low in species richness, it is also important to conserve, as
it may be home to rare flora and fauna. Such areas might
175. Solution: (d)
cause less conservation conflicts due to low opportunity
Exp) Option d is correct. costs.
Captive Breeding, also known as “captive propagation”,
is the process of maintaining plants or animals in 177. Solution: (d)
controlled environments, such as wildlife reserves, zoos, Exp) Option d is correct.
botanical gardens, and other conservation facilities. E.g.,
Haiderpur is located within the boundaries of Hastinapur
captive breeding of lion-tailed macaque in Arignar Anna
Wildlife Sanctuary. It is a human-made wetland covering
Zoological Park, Chennai.
an area of 6,908 hectares and was formed in 1984 after
Statement 1 is correct. Conservation Breeding the construction of the Madhya Ganga Barrage on the
Programme in India has been started under the floodplains of Ganga. It supports more than 15 globally
supervision of the Central Zoo Authority. The Central
threatened species, such as the critically endangered
Zoo Authority had already identified 73 species for
planned conservation breeding programme. 18 species gharial and the endangered hog deer, black-bellied tern,
have already been taken off for establishing off-display steppe eagle. It supports more than 25,000 waterbirds. It
conservation breeding programmes (CBP) in different serves as a breeding site for the near-threatened Indian
states. grassbird and provides refuge to the northern subspecies
population of the vulnerable swamp deer during its
Statement 2 is correct. The Central Zoo Authority would
provide 100 % financial assistance for coordinating zoos seasonal flood-driven migration.
and for improving the existing enclosures in participating Option a is incorrect. Sarsai Nawar is a small unprotected
zoos. The Coordinating Zoo shall create a housing facility wetland, enroute to Saman Wildlife Sanctuary, in Etawah
as per the design approved by CZA in off-display area of District of Uttar Pradesh. It has a large population of the
the conservation breeding center. threatened species of Sarus Cranes, the world’s tallest
Statement 3 is correct. Laboratory for Conservation flying birds. It is included in Ramsar list in early 2020.
of Endangered Species (LaCONES) is a dedicated Option b is incorrect. Gogabeel is an ox-bow lake in
laboratory of CSIR, Hyderabad. It is the only institute in Bihar’s Katihar district.It has been declared as the state’s
the country working towards conservation of endangered first ‘Community Reserve’. It is not included in Ramsar
wildlife using modern biotechnology. CCMB-LaCONES
list.
is the only laboratory in India that has developed methods
for collection and cryopreservation of semen and oocytes Option c is incorrect. Nawabganj bird sanctuay is a
from wildlife and successfully reproducing endangered haven for birds, with 25,000 waterbirds regularly recorded
blackbuck, spotted deer and Nicobar pigeons. and 220 resident and migratory species documented
Among these are globally threatened species including
176. Solution: (a) the endangered Egyptian vulture and Pallas’s fish eagle.
Exp) Option a is correct. Monsoon rains feed this diverse wetland while the Sarda
Biodiversity Coldspots are the areas that have relatively Canal supplies additional water. It is situated in Hasanganj
low biological diversity but are also experiencing a high tehsil of Unnao district of Uttar Pradesh, India

317 Workbook
.
ENVIRONMENT

ENVIRONMENT
CLIMATE CHANGE
5. The scientific view is that the increase in global
6.1. Previous Years Questions temperature should not exceed 2° C above pre-
1. The formation of ozone hole in the Antarctic industrial level. If the global temperature increases
region has been a cause of concern. What could be beyond 3° C above the pre-industrial level, what
the reason for the formation of this hole? can be its possible impact/impacts on the world?
[UPSC CSE Pre. 2011] [UPSC CSE Pre.2014]
(a) Presence of prominent tropospheric turbulence; 1. Terrestrial biosphere tends toward a net carbon
and inflow of chlorofluorocarbons source.
(b) Presence of prominent polar front and 2. Widespread coral mortality will occur.
stratospheric clouds; and inflow of 3. All the global wetlands will permanently
chlorofluorocarbons. disappear.
(c) Absence of polar front and stratospheric clouds; 4. Cultivation of cereals will not be possible
and inflow of methane and chlorofluorocarbons. anywhere in the world.
(d) Increased temperature at polar region due to
global warming Select the correct answer using the code given below:
(a) 1 only
2. Human activities in the recent past have caused (b) 1 and 2 only
the increased concentration of carbon dioxide in (c) 2, 3 and 4 only
the atmosphere, but a lot of it does not remain in
(d) 1, 2, 3 and 4
the lower atmosphere because of
[UPSC CSE Pre. 2011] 6. What is the Rio+20 Conference, often
1. Its escape into the outer stratosphere. mentioned in the news? [UPSC CSE Pre. 2015]
2. The photosynthesis by phytoplankton in the (a) It is the United Nations Conference on
oceans. Sustainable Development
3. The trapping of air in the polar ice caps. (b) It is a Ministerial Meeting of the World Trade
Which of the statements given above is/are correct? Organization
(a) 1 and 2 (c) It is a Conference of the Inter Government Panel
(b) 2 only on Climate Change.
(c) 2 and 3 (d) It is a Conference of the Member Countries of
(d) 3 only the Convention on Biological Diversity.
3. Consider the following statements: 7. Which one of the following is associated with
Chlorofluorocarbons, known as ozone-depleting the issue of control and phasing out of the use
substances, are used of ozone-depleting substances?
[UPSC CSE Pre. 2012] [UPSC CSE Pre. 2015]
1. In the production of plastic foams. (a) Bretton Woods Conference
2. In the production of tubeless tyres. (b) Montreal Protocol
3. In cleaning certain electronic components. (c) Kyoto Protocol
4. As pressurizing agents in aerosol cans. (d) Nagoya Protocol
Which of the statements given above is/are correct?
8. With reference to the International Union
(a) 1, 2 and 3 only
(b) 4 only
for Conservation of Nature and Natural
(c) 1, 3 and 4 only Resource (IUCN) and the Convention of
(d) 1, 2, 3 and 4 International Trade in Endangered Species
of Wild Fauna and Flora (CITES), which of
4. The increasing amount of carbon dioxide in
the air is slowly raising the temperature of the the following statements is/are correct ?
atmosphere, because it absorbs [UPSC CSE Pre. 2015]
[UPSC CSE Pre. 2012] 1. IUCN is an organ of the United Nations and
(a) The water vapour of the air and retains its heat. CITES is an international agreement between
(b) The ultraviolet part of the solar radiation. governments.
(c) All the solar radiations. 2. IUCN runs thousands of field projects around the
(d) The infrared part of the solar radiation. world to better manage natural environments.

Workbook 318
.
ENVIRONMENT

3. CITES is legally binding on the States that have (c) efforts put in by a climate refugee to adapt to live
joined it, but this Conventional does not take the in a new place.
place of national laws. (d) contribution of an individual person to the
Select the correct answer using the codes given carbon footprint on the planet Earth.
below. 13. Which one of the following lakes of West Africa
(a) 1 only has become dry and turned into a desert?
(b) 2 and 3 only [UPSC CSE Pre. 2022]
(c) 1 and 3 only
(a) Lake Victoria
(d) 1, 2 and 3
(b) Lake Faguibine
9. In the context of mitigating the impending (c) Lake Oguta
global warming due to anthropogenic emissions (d) Lake Volta
of carbon dioxide, which of the following can be
potential sites for carbon sequestration? 14. “Climate Action Tracker” which monitors the
[UPSC CSE Pre. 2017] emission reduction pledges of different countries
is a: [UPSC CSE Pre. 2022]
1. Abandoned and uneconomic coal seams
(a) Database created by coalition organisations of
2. Depleted oil and gas reservoirs
Research organisations
3. Subterranean deep saline formations (b) Wing of “International Panel of Climate Change”
Select the correct answer using the code given below: (c) Committee under “United Nations Framework
(a) 1 and 2 only Convention on Climate Change”
(b) 3 only (d) Agency promoted and financed by United
(c) 1 and 3 only Nations Environment Programme and World
(d) 1, 2 and 3 Bank
10. Which of the following statements best describes Practice Questions
“carbon fertilization”? [UPSC CSE Pre. 2018]
(a) Increased plant growth due to increased
concentration of carbon dioxide in the
6.2. Global warming, GHG and climate
atmosphere forcing
(b) Increased temperature of Earth due to increased 15. With reference to soil organic carbon (SOC)
concentration of carbon dioxide in the consider the following statements:
atmosphere 1. Areas of higher rainfall has higher levels of SOC.
(c) Increased acidity of oceans as a result of
2. SOC is one of the three global indicators of Land
increased concentration of carbon dioxide in the
atmosphere Degradation Neutrality (LDN).
(d) Adaptation of all living beings on Earth to the 3. The majority of world’s SOC stocks are located in
climate change brought about by the increased the temperate grasslands.
concentration of carbon dioxide in the Which of the statements given above is/are correct?
atmosphere (a) 1 and 2 only
11. In the context of which of the following do some (b) 2 and 3 only
scientists suggest the used of cirrus cloud thinning (c) 1 and 3 only
technique and the injection of sulphate aerosol (d) 1, 2 and 3
into stratosphere? [UPSC CSE Pre. 2019] 16. Which of the following statements is/are correct
(a) Creating the artificial rains in some regions with respect to greenhouse effect?
(b) Reducing the frequency and intensity of tropical 1. Green-house effect is responsible for life on the
cyclones earth.
(c) Reducing the adverse effects of solar wind on the 2. It is a naturally occurring phenomenon that
Earth shields the earth from harmful solar radiations.
(d) Reducing the global warming
3. Green-house effect is higher at higher elevations.
12. Which one of the following statements best Select the correct answer using the code given below
describes the term ‘Social Cost of Carbon’? It is a
(a) 1 only
measure, in monetary value, of the
(b) 1 and 3 only
[UPSC CSE Pre. 2020]
(c) 2 and 3 only
(a) long-term damage done by a tonne of CO2 (d) 1, 2 and 3
emissions in a given year.
(b) requirement of fossil fuels for a country to 17. Arrange the following gases in a correct order
provide goods and services to its citizens, based on the basis of their Global Warming Potential
on the burning of those fuels. (GWP) from the highest to lowest:

319 Workbook
.
ENVIRONMENT

1. HFC-23 (a) 1, 2 and 3 only


2. Nitrous oxide (b) 2 and 3 only
3. Methane (c) 1, 3 and 4 only
4. Sulfur hexafluoride (d) 1, 2, 3 and 4
Select the correct answer using the code given below: 22. With reference to the pattern of climate change in
(a) 4-1-2- 3 India, consider the following statements:
(b) 1-4-2-3 1. In India, some traditionally flood prone districts
(c) 4-3-1-2 have become drought-prone in recent years due
(d) 3-1-4-2 to climate change.
2. By 2020, India has already reduced the emission
18. With reference to Methane as a greenhouse gas,
intensity of its GDP by more than 50 per cent
consider the following statements:
over 2005 levels.
1. It is a hydrocarbon produced by breakdown of
organic materials. Which of the statements given above is/are correct?
2. Wetlands are the largest natural source of (a) 1 only
Methane. (b) 2 only
(c) Both 1 and 2
3. Methane does not have any effect on ground
(d) Neither 1 nor 2
level Ozone formation.
Which of the statements given above is/are correct? 23. Which of the following is a green house gas?
(a) 1 only (a) Nitrogen Dioxide
(b) 1 and 2 only (b) Carbon Monoxide
(c) 2 and 3 only (c) Water Vapour
(d) 1, 2 and 3 (d) Ammonia
19. Which of the following are the impacts of global 24. Arrange the following green-house gases in
warming? the descending order of their Global Warming
1. Increased events of flooding Potential?
2. Melting of glaciers 1. Methane
3. Spread of diseases 2. Hydrofluoro Carbons
4. Change in rainfall pattern 3. Carbon Dioxide
Select the correct answer using the code given below 4. Sulphur hexafluoride
(a) 1, 2 and 3 only Select the correct answer using the code given below:
(b) 2, 3 and 4 only (a) 1-3-2-4
(c) 1, 2, 3 and 4 (b) 3-1-4-2
(d) 1 and 2 only (c) 3-4-1-2
(d) 4-2-1-3
20. Which of the following are potential impacts of
global warming? 25. With reference to the Aerosol Radiative Forcing
1. Decrease in the amount of arable land in high- (ARF), which one of the following statements is
latitude region correct?
2. Decrease in arable land along coastlines (a) It is the effect of anthropogenic aerosols on the
3. Higher salinity of water table in coastal areas radiative fluxes at the top of the atmosphere.
(b) Its effects are not found at higher altitude regions
4. Reduced crop yield in tropical region
like the Himalayas.
Select the correct answer using the code given below: (c) It does not have any effect on the clouds in
(a) 1, 2 and 4 only Earth’s atmosphere.
(b) 1, 3 and 4 only (d) It refers to the forced removal of anthropogenic
(c) 2, 3 and 4 only aerosols from the atmosphere to mitigate climate
(d) 1, 2, 3 and 4 change.
21. Which of the following are the impacts of ocean 26. With reference to impact of global warming,
warming? consider the following statements:
1. Coral bleaching 1. The Himalayan region might face increased
2. Deoxygenation of ocean scarcity of water due to melting glaciers.
3. Floods in coastal areas 2. Recently the Conger ice shelf in Antarctica
4. Changed geographical distribution of organisms collapsed due to increased global warming.
Select the correct answer using the codes given Which of the following statements is/are correct?
below. (a) 1 only

Workbook 320
.
ENVIRONMENT

(b) 2 only 31. With reference to Carbon Pricing, consider the


(c) Both 1 and 2 following statements:
(d) Neither 1 nor 2 1. Currently there is no explicit Carbon tax
27. Consider the following statements: imposed in India by the government.
1. Pollen grains are male biological entities 2. Carbon credits can be traded on both private
produced by higher plant cells that are essential and public markets.
for sexual reproduction. 3. The prices of carbon credits are driven by the
2. Pollen enters the thoracic regions of the levels of supply and demand in the markets.
respiratory tract leading to poor quality of life Which of the above-mentioned statements is/are
for patients. correct?
3. Increase in global surface temperature in the (a) 1 only
atmosphere is responsible for the pollen-induced (b) 2 and 3 only
respiratory allergies. (c) 2 only
Which of the statements given above is/are correct? (d) 1, 2 and 3
(a) 1 only 32. Recently, the term ‘Wright Mons’ was seen in the
(b) 2 and 3 only news. It is related to-
(c) 1 and 3 only (a) First airplane model designed by Wright
(d) 1, 2 and 3 Brothers.
28. Which of the following methods is/are used as (b) Comet look-like asteroid discovered in Kuiper
carbon capture and storage (CCS) technology? Belt.
1. Oxyfuel method (c) Mountainous feature found on Pluto.
(d) Type of Crypto Ransomware appeared online.
2. Pre-combustion method
3. Post-combustion method 33. For which of the following purposes the
Select the correct answer using the code given below. sequestrated carbon can be used?
(a) 1 only 1. Dry Ice Production
(b) 2 and 3 only 2. Production of Plastics
(c) 2 only 3. Making of Concretes
(d) 1, 2 and 3 4. Greenhouse photosynthesis
29. With reference to Carbon sink, consider the Choose the correct answer using the options given
following statements: below:
1. The Earth’s soil absorbs roughly a quarter of all (a) 1 and 4 only
human related emissions each year. (b) 1, 3 and 4 only
2. Peatlands are the largest natural terrestrial (c) 2, 3 and 4 only
carbon store. (d) 1, 2, 3 and 4
3. Growing crops in the residue of previous crops 34. Consider the following statements with reference
degrades the carbon sequestration capacity of to different “Carbon Capture Technologies”:
the soil. 1. In post-combustion method CO2 is separated
Which of the statements given above is/are correct? from the flue gas using liquid solvents.
(a) 1 and 2 only 2. In pre-combustion method coal is gasified to
(b) 2 and 3 only produce a synthetic gas which is a mix of carbon
(c) 3 only monoxide and hydrogen.
(d) 1. 2 and 3 3. In oxy-fuel method the primary fuel is
30. Which of the following ecosystems is/are Blue combusted in oxygen instead of air, which
Carbon Ecosystems? produces a flue gas.
1. Sea Weeds Which of the above statements is/are correct?
2. Estuarine Mangroves (a) 1 and 2 only
3. Sea Grasses (b) 1 and 3 only
(c) 2 and 3 only
4. Fresh Water Ecosystem
(d) 1, 2 and 3
Choose the correct answer using the options given
below: 35. With respect to Bromine, which of the following
(a) 1 and 2 only statements is/are correct?
(b) 1 and 4 only 1. It is used in fire-retardants, fumigants and dyes.
(c) 1, 3 and 4 only 2. Bromine is more harmful in destroying
(d) 1, 2, 3 and 4 stratospheric ozone than chlorine.

321 Workbook
.
ENVIRONMENT

3. Methyl bromide is currently not on the list of (a) Sulfur hexafluoride < Nitrous Oxide < Methane
substances that deplete the ozone layer under < Carbon dioxide
Montreal Protocol. (b) Sulfur hexafluoride < Methane < Nitrous Oxide
Select the correct answer using the code given below: < Carbon dioxide
(c) Carbon dioxide < Nitrous Oxide < Methane <
(a) 1 only Sulfur hexafluoride
(b) 1 and 2 only (d) Carbon dioxide < Methane < Nitrous Oxide <
(c) 2 and 3 only Sulfur hexafluoride
(d) 1, 2 and 3
41. With reference to the impacts of climate change,
36. With reference to ozone, which among the which of the following events are most likely to
following statements is/are correct? occur?
1. Stratospheric ozone prevents all ultraviolet rays 1. Ocean deoxygenation.
from reaching to the Earth’s surface. 2. Disruption of the ocean food chain.
2. Tropospheric ozone is a primary pollutant that 3. Mass coral bleaching.
have bad impact on human health. 4. Distress in agriculture and water security.
3. Tropospheric ozone is a short-lived powerful
which of the following options is/are correct?
greenhouse gas.
(a) 1, 2 and 3 only
Select the correct answer using the code given below: (b) 2, 3 and 4 only
(a) 1 only (c) 1, 3 and 4 only
(b) 1 and 3 only (d) 1, 2, 3 and 4
(c) 2 and 3 only
(d) 3 only 6.3. Ocean acidification
37. Which among the following will not cause an 42. With reference to ocean acidification, consider the
increase in ocean acidification? following statements:
(a) Death of Sea-organisms 1. The concentration of hydrogen ions in the ocean
(b) Burning of fossil fuels increases.
(c) Cargo Shipping Operations 2. The pH of ocean increases, and the oceans
(d) Ocean Iron Fertilization become more alkaline.
38. Which of the following statement is correct with 3. The process of eutrophication leads to ocean
respect to Methane? acidification.
(a) Methane stays longer than Carbon dioxide in the Which of the statements given above is/are correct?
environment acting as powerful greenhouse gas. (a) 1 only
(b) The presence of methane in the atmosphere can (b) 2 only
also affect the abundance of other greenhouse (c) 1 and 3 only
gases. (d) 1, 2 and 3
(c) It is an important precursor to the formation of
stratospheric ozone. 43. With reference to Ocean acidification, consider
(d) Natural gas leakage from crude oil projects is the the following statements
highest source of methane emissions. 1. Ocean acidification results in the lowering of
ocean pH.
39. Consider the following statements: 2. Decrease in carbonate ions due to acidification
1. Brown carbon is mainly emitted through high- negatively impacts coral reefs.
temperature combustion of fossil fuels. 3. None of the marine organisms are positively
2. Black carbon is a PM10 aerosols whose major affected due to acidification of oceans.
source is transportation sector. Which of the statements given above is/are correct?
3. Both the black and brown carbon are known to (a) 1 and 2 only
have high light absorptive properties. (b) 1 and 3 only
Which of the following statements is/are correct? (c) 2 and 3 only
(a) 1 and 2 only (d) 1, 2 and 3
(b) 2 and 3 only 44. Which of the following statements is/are incorrect
(c) 3 only with reference to ocean acidification?
(d) 1, 2 and 3 only
1. Due to rise in ocean acidification number of
40. Which among the following is the correct sequence bicarbonate ions decreases in the oceans.
of increasing order in terms of Global Warming 2. Ocean acidification leads to fall in calcium
Potential of the Greenhouse gases? carbonate saturation horizon.

Workbook 322
.
ENVIRONMENT

Select the correct answer from the code given below: 2. The growth and survival of calcareous
(a) 1 only zooplankton is adversely affected due to ocean
(b) 2 only acidification.
(c) Both 1 and 2 3. The cloud seeding and formation of clouds is
(d) Neither 1 nor 2 hampered by ocean acidification.
4. The excess algae and plant matter in oceans
45. With respect to ocean acidification, which of the contributes to ocean acidification.
following statements is/are correct?
Which of the above statements is/are correct?
1. Ocean acidification is the process of addition
of nitrogen dioxide into water which leads to (a) 1, 2 and 3 only
(b) 2, 3 and 4 only
creation of nitric acid.
(c) 1, 2 and 4 only
2. Aragonite form of calcium carbonate is more (d) 1, 3 and 4 only
soluble than calcite form.
3. The process of eutrophication decreases the level 6.4. Ozone depletion
of acidification.
Select the correct answer using the code given below: 50. Nylon is a synthetic polymer fibre with super-long,
heavy molecules made of short, repetitive units of
(a) 1 only
diamines and dicarboxylic acids. Why there is/are
(b) 1 and 3 only
concern/concerns over the use of nylon?
(c) 2 only
(d) 2 and 3 only 1. Manufacturing and processing of nylon is
energy-intensive.
46. With reference to oceanography, the term 2. Its production process aids to the depletion of
‘lysocline’ indicates: the ozone layer.
(a) Biologically productive regions near the coasts. 3. Improper disposal of nylon products contributes
(b) The boundary below which dissolution of to marine pollution.
calcium carbonate strongly increases in the deep Select the correct answer using the code given below:
ocean. (a) 1 and 2 only
(c) Highly polluted areas due to oil spills in the (b) 2 only
ocean. (c) 1 and 3 only
(d) The boundary below which the temperature of (d) 1, 2 and 3
ocean water decreases at high rate.
51. With respect to characteristics of ozone gas in the
47. In which of the following ways, the oceans absorbs atmosphere, which of the following statements is
carbon dioxide? correct?
1. Molecular Diffusion of atmosphere with ocean. 1. In stratosphere, the concentration of ozone
2. Photosynthesis in plankton and algae in ocean above the tropics is more as compared to the
3. Steering of winds and ocean currents by Coriolis Polar Regions.
force 2. Tropospheric ozone is a major component for
Select the correct option using the code given below. smog formation.
(a) 1 and 2 only 3. Bromine is more effective than chlorine in
(b) 2 and 3 only destroying ozone molecules.
(c) 1 and 3 only Select the correct answer using the code given below
(d) 1, 2 and 3 (a) 1 and 2 only
(b) 2 and 3 only
48. With reference to Global warming effects on
(c) 3 only
oceans, consider the following statements:
(d) 1, 2 and 3
1. Global warming decreases the upwelling in oceans.
52. Which of the following are the ozone depleting
2. It can lead to forced migration of many species. substances?
Which of the statements given above is/are correct? 1. Chlorofluorocarbons
(a) 1 only 2. Halons
(b) 2 only 3. Methyl Bromide
(c) Both 1 and 2 4. Carbon Tetrachloride
(d) Neither 1 nor 2
Select the correct answer using the code given below:
49. With reference to the Ocean Acidification, (a) 1 and 2 only
consider the following statements: (b) 1, 2 and 3 only
1. Ocean acidification cause increase in the ocean’s (c) 2, 3 and 4 only
pH value. (d) 1, 2, 3 and 4

323 Workbook
.
ENVIRONMENT

53. Which of the following are Ozone Depleting 57. Which of the following pairs are correctly matched
Substances listed under Montreal Protocol? in the context of Kyoto protocol for climate
1. Halons change?
2. Chlorofluorocarbons Kyoto mechanism Provision
3. Hydrofluorocarbons
1. Clean Development An Annex B country
4. Hydrochlorofluorocarbons Mechanism can implement a
5. Methyl Chloroform project in developing
Select the correct answer using the code given below: countries
(a) 1, 2, 3 and 5 only 2. Joint An Annex B country
(b) 1, 3 and 4 only Implementation can earn emission
(c) 1, 2, 4 and 5 only reduction units from
(d) 2, 3, 4 and 5 only a project in another
Annex B country
54. With reference to “ozone depleting potential”
(ODP), which of the following statement is 3. Emissions trading Countries with spare
correct? mechanism emission units can sell
them to countries that
1. Ozone depleting potential is a measure of how are over their targets
much damage a chemical can cause to the
ozone layer compared with a similar mass of Select the correct answer using the code given below
trichlorofluoromethane (CFC-11). (a) 1 and 2 only
2. Carbon dioxide has more ozone depleting (b) 2 and 3 only
potential than Nitrous oxide (N2O). (c) 1 and 3 only
3. Hydrofluorocarbons (HFCs) have zero ozone (d) 1, 2 and 3
depleting potential.
58. In recent times, hydrogen has been increasingly
Select the correct answer using the code given below: mooted as an alternate source of energy. However,
(a) 1 and 2 only there are some limitations in using hydrogen as an
(b) 2 and 3 only energy source. What are those limitations?
(c) 1 and 3 only
1. As a fuel, hydrogen has very low energy density
(d) 1, 2 and 3
as compared to petrol.
55. What is the role of polar stratospheric clouds 2. Hydrogen cannot be generated from green
(PSC) in ozone depletion? resources.
(a) It delays chlorine activation prolonging the 3. Hydrogen is not found freely in nature and has
ozone depletion process. to be extracted from other compounds.
(b) It increases the temperature in Antarctica and
Arctic acting as catalysis for depletion. Select the correct answer using the code given below:
(c) It provides suitable sites and substrates for the (a) 1 and 3 only
depletion reaction to occur. (b) 2 and 3 only
(d) It reaches the tropics during winter further (c) 1 and 2 only
leading to depletion (d) 3 only

6.5. Mitigation strategies 59. Consider the following statements with reference
to the Polyhouse cultivation:
56. With reference to Carbon capture, utilisation 1. It uses polythene sheets as a covering material
and storage (CCUS), consider the following to grow crops in controlled climatic conditions.
statements:
2. In polyhouse cultivation, crops can be grown
1. Carbon captured need to be stored on-site as it
throughout the year.
cannot be transported to other locations.
2. Carbon captured is highly reactive and thus 3. Crops in a polyhouse are more prone to pests
cannot be used for industrial purposes. and diseases due to closed environment.
Which of the statements given above is/are correct? Which of the statements given above is/are correct?
(a) 1 only (a) 1 and 2 only
(b) 2 only (b) 2 and 3 only
(c) Both 1 and 2 (c) 1 and 3 only
(d) Neither 1 nor 2 (d) 1, 2 and 3

Workbook 324
.
ENVIRONMENT

60. With reference to carbon dioxide mitigation 65. With reference to carbon offsetting, consider the
strategies, the term terrestrial sequestration refers following statements:
to: 1. It allows a party to buy unused carbon emissions
(a) A process that captures and stores carbon units from a party that has exceeded their carbon
dioxide in vegetation and soil. emissions reduction targets.
(b) Using natural pore spaces in geologic formations 2. It has been introduced under the Clean
for carbon dioxide storage. Development Mechanism.
(c) Reaction of carbon dioxide with the minerals
and organic matter to form stable compounds. Which of the statements given above is/are correct?
(d) Trapping of carbon dioxide as a gas under low- (a) 1 only
permeability cap rock. (b) 2 only
(c) Both 1 and 2
61. Which of the following statements is/are (d) Neither 1 nor 2
correct regarding the Hydrodynamic Trapping
mechanism of Carbon Capture and Storage? 66. Which of the following is/are correct regarding
(a) Carbon dioxide is trapped as a gas under low- the Carbon Tax?
permeability cap rock. 1. It is based on the polluter pays principle.
(b) Carbon dioxide is dissolved into a liquid, such 2. It is part of National Clean Energy Fund.
as water or oil.
(c) Carbon dioxide is made to react with the Select the correct answer using the code given below:
minerals, fluids, and organic matter in a geologic (a) 1 only
formation to form stable compounds/minerals. (b) 2 only
(d) All of the above (c) Both 1 and 2
(d) Neither 1 nor 2
62. Consider the following statements:
1. Carbon net-zero is a state in which a 67. Which of the following geo-engineering
country emits no waste products or greenhouse techniques would help to counter climate change?
gases that disrupt the climate. 1. Setting up space reflectors
2. Carbon sequestration is the process of removing 2. Introducing aerosols in the stratosphere
carbon oxide from the atmosphere and then 3. Ocean fertilisation
storing it.
4. Enhancing Ocean alkalinity
3. Currently, United States of America is the world’s
largest emitter of greenhouse gases. Select the correct answer using the code given below:
Which of the statements given above is/are correct? (a) 1 and 2 only
(a) 2 only (b) 2, 3 and 4 only
(b) 1 and 3 only (c) 1 and 3 only
(c) 3 only (d) 1, 2, 3 and 4
(d) 2 and 3 only 68. With reference to Carbon offsetting and Carbon
63. Consider the following statements regarding the credits, consider the following statements:
carbon credits: 1. In the process of Carbon offsetting, there is no
1. It is a tradable emission reduction certificate net reduction in greenhouse gas emissions.
equivalent to one tonne of fossil fuel. 2. Carbon credits are traded at fixed prices at
2. It has been introduced under the Kyoto Protocol. different exchanges all over the world.
3. It is traded at a fixed price by the United Nations Which of the above-mentioned statements is/are
Environment Program (UNEP). correct?
Which of the statements given above is/are correct? (a) 1 only
(a) 1 and 3 only (b) 2 only
(b) 2 only (c) 1 and 2 only
(c) 3 only (d) d) None of the above
(d) 1, 2 and 3
69. Which of the following geo-engineering technique
64. Which state has launched the world’s first market
is not used to fight against global warming?
for trading in particulate matter emissions?
(a) Ambient Air Capture
(a) Kerala
(b) Rajasthan (b) Bioventing
(c) Uttarakhand (c) Stratospheric Aerosol Injection
(d) Gujarat (d) Ocean Fertilization

325 Workbook
.
ENVIRONMENT

70. Consider the following statements with reference 73. With reference to Climate Change Initiatives,
to steps taken by India to mitigate, prevent and which of the following statements is/are correct?
adapt to climate change: 1. National Mission on Sustainable agriculture
1. Reserve Bank of India (RBI) set up a ‘Sustainable promotes laboratory to land research by creating
Finance Group’ (SFG) to effectively counter model villages in dryland areas.
Climate change-related financial risk. 2. National Mission for Strategic Knowledge
2. Roof Top Solar programme Phase-II has set a on Climate Change aims to develop national
target of 40 GW installed capacity by December capacity for modeling the regional impact of
2022. climate change.
3. Indian Railways has set a target of Net Zero Choose the correct answer using the options given
Carbon Emission by 2050. below:
Which of the above statements is/are correct? (a) 1 only
(a) 3 only (b) 2 only
(b) 1 and 2 only (c) Both 1 and 2
(c) 1, 2 and 3 (d) Neither 1 nor 2
(d) None of the above
74. With reference to Green Grids Initiative, consider
71. Consider the following Pairs with reference to the following statements:
India’s initiatives at the international stage with 1. It has been launched by India as a part of One
their aims/purpose: Sun One World One Grid Initiative.
India’s initiatives aims/purpose/ 2. It aims to connect energy grids across borders to
definition facilitate a faster transition of renewable energy.
1. Leadership Group To take actions to Which of the above-mentioned statements is/are
for Industry implement the Paris correct?
Transition (LeadIT Agreement (a) 1 only
Group) (b) 2 only
2. Coalition for A ‘knowledge centre’ (c) Both 1 and 2
Disaster Resilient about disaster-proofing (d) Neither 1 nor 2
Infrastructure of infrastructure. 75. Consider the following statement regarding
3. Lifestyle for One-Word Movement Special Purpose Acquisition Companies (SPACs):
Environment 1. It is an entity specifically set up with the objective
Which of the above Pairs is/are correctly matched? of acquiring a firm in a particular sector.
(a) 2 only 2. SPACs are essentially shell companies without
(b) 1 and 3 only any real operations or revenues.
(c) 1, 2 and 3 3. They are prohibited from raising funds from the
(d) None of the above public through Initial Public Offering.
72. Which of the following statements are part of the Which of the statements given above is/are correct?
five-fold strategy declared by India at COP-26? (a) 1 only
1. To reduce India’s projected carbon emission by (b) 1 and 2 only
one billion tonnes by 2030 (c) 2 and 3 only
2. To reduce Economy’s carbon intensity by 45% (d) 3 only
by 2030 76. With reference to India’s efforts to counter climate
3. To achieve Net Zero emissions by 2030 change, which of the following statement is
4. To take non-fossil energy capacity to 500 GW by incorrect?
2030 (a) It has ratified Kigali Amendment to phase out
5. To meet 50% of the energy requirement from Hydrofluorocarbons.
renewables by 2070 (b) It has allocated large funds for advancing electric
Choose the correct answer using the options given mobility under FAME Scheme.
below: (c) It is likely to achieve an additional Carbon sink
(a) 1, 2 and 4 only of 2.5 to 3 billion tonnes of carbon dioxide by
(b) 1, 3 and 4 only 2030.
(c) 1, 2, 3 and 4 only (d) It is likely to meet its Cancun climate pledge
(d) 1, 2, 3, 4 and 5 made in 2010.

Workbook 326
.
ENVIRONMENT

77. With reference to India-US Climate and Clean 3. Under this, USA will help India in setting up
Energy Agenda 2030, consider the following Commission for Air Quality Management
statements: (CAQM) for cleaner air.
1. The Agenda was declared on the sidelines of
Which of the above-mentioned statements is/are
Conference of Parties 26 held in Glasgow,
correct?
Scotland.
2. India will collaborate with US under Climate (a) 1 and 2 only
Action and Finance Mobilization Dialogue (b) 2 only
(CAFMD) to deploy 450 GW of renewable (c) 2 and 3 only
energy capacity. (d) 3 only

327 Workbook
.
ENVIRONMENT

SOLUTIONS

Brazil on 20-22 June 2012. It resulted in a focused political


6.1. Previous Years Questions outcome document which contains clear and practical
1. Solution (b) measures for implementing sustainable development.
Exp) Option b is correct In Rio, Member States decided to launch a process to
develop a set of Sustainable Development Goals (SDGs),
The very low winter temperatures in the Antarctic
which will build upon the Millennium Development
stratosphere cause polar stratospheric clouds (PSCs) to
Goals and converge with the post 2015 development
form. Special reactions that occur on PSCs, combined
agenda.
with the relative isolation of polar stratospheric air, allow
chlorine and bromine reactions to produce the ozone hole 7. Solution: (b)
in Antarctic springtime.
Exp) Option b is correct.
2. Solution (b) The Montreal Protocol on Substances that Deplete the
Exp) Option b is correct Ozone Layer regulates the production and consumption
Much of the Carbon dioxide produced by human does of nearly 100 man-made chemicals referred to as ozone
not stay in the atmosphere but is stored in the oceans or depleting substances (ODS).
on land in plants and soils. Oceans are the largest carbon When released to the atmosphere, those chemicals
pools on Earth, and it also serves as a “buffer” for climate damage the stratospheric ozone layer, Earth’s protective
change. Approximately 30% of the CO2 produced by shield that protects humans and the environment from
anthropogenic activities is absorbed by oceans. Near the harmful levels of ultraviolet radiation from the sun.
surface, carbon is fixed by plant-like phytoplankton, which Adopted on 15 September 1987, the Protocol is to date the
are eaten by sea animals; some eventually rains down as only UN treaty ever that has been ratified every country
waste and dead organisms. Bacteria feed on this particulate on Earth - all 198 UN Member States.
organic carbon and produce CO2 which dissolves, while
the rest of the detritus ends on the sea floor. 8. Solution: (b)
Exp) Option b is correct.
3. Solution (c)
IUCN has observer and consultative status the United
Exp) Option c is correct Nations. Thus, clearly its not a ‘member’. It is involved
Chlorofluorocarbons (CFCs) are non-toxic, non- in data gathering and analysis, research, field projects,
flammable chemicals containing atoms of carbon, advocacy, and education. IUCN’s mission is to “influence,
chlorine, and fluorine. They are used in the manufacture encourage and assist societies throughout the world to
of aerosol sprays, blowing agents for foams and packing conserve nature and to ensure that any use of natural
materials, as solvents, and as refrigerants. CFCs are not resources is equitable and ecologically sustainable”.
used in tubeless tyres. CITES (the Convention on International Trade in
4. Solution (d) Endangered Species of Wild Fauna and Flora) is an
international agreement between governments. Its aim
Exp) Option d is correct is to ensure that international trade in specimens of wild
Molecules of carbon dioxide (CO2) can absorb energy animals and plants does not threaten the survival of the
from infrared part of solar radiations and re-emit it. The species.
re-emitted energy travels out in all directions and heats up States that have agreed to be bound by the Convention
the atmosphere.
(‘joined’ CITES) are known as Parties. Although CITES
In addition, Carbon dioxide can stay in atmosphere for is legally binding on the Parties – in other words they
centuries. have to implement the Convention – it does not take the
place of national laws. Rather it provides a framework to
5. Solution (b)
be respected by each Party, which has to adopt its own
Exp) Option b is correct domestic legislation to ensure that CITES is implemented
The last two options are highly unlikely to happen if global at the national level.
temperature increases beyond 3° C above pre-industrial
levels. 9. Solution (d)
Exp) Option d is correct
6. Solution: (a)
Carbon sequestration is the process of capturing and
Exp) Option a is correct. storing atmospheric carbon dioxide. It is one method of
The United Nations Conference on Sustainable reducing the amount of carbon dioxide in the atmosphere
Development or Rio+20, took place in Rio de Janeiro, with the goal of reducing global climate change. It involves

Workbook 328
.
ENVIRONMENT

long-term storage of carbon in plants, soils, geologic The Climate Action Tracker is an independent scientific
formations, and the ocean. analysis that tracks government climate action and
measures it against the globally agreed Paris Agreement
10. Solution (a) aim of “holding warming well below 2°C, and pursuing
Exp) Option a is correct efforts to limit warming to 1.5°C.” A collaboration of
According to Carbon Fertilization Effect, increase of two organizations, Climate Analytics and New Climate
carbon dioxide in the atmosphere increases the rate of Institute, the CAT has been providing this independent
photosynthesis in plants. Thus, increasing crop yields. analysis to policymakers since 2009.
11. Solution (d)
Exp) Option d is correct
6.2. Global warming, GHG and climate
forcing
Cirrus clouds are formed at high altitudes and trap the
long wave radiation and reflect sun light. Thinning cirrus 15. Solution (a)
clouds reduces their heat trapping capacity. Exp) Option a is correct
In Stratospheric Aerosol Injection (SAI) technique, Statement 1 is correct. SOC levels are positively related to
reflective sulfate aerosols particles are injected into the rainfall and increased biomass production. Highest SOC
stratosphere which aids in reflecting most of the sunlight levels (and variability) occur in areas of higher rainfall
back into space and creating a dimming effect and cooling that support increased biomass production, and on
the planet. soils that are unconstrained by water availability.
12. Solution (a) Statement 2 is correct. Soil organic carbon (SOC) is
Exp) Option a is correct one of the three global indicators of Land Degradation
Neutrality (LDN) because of its multifunctional roles and
The Social Cost of Carbon is usually estimated as the its sensitivity to land management and thus, predicting
net present value of climate change impacts over the and monitoring change in SOC is vital to achieving LDN
next 100 years (or longer) of one additional tonne targets.
of carbon emitted to the atmosphere today. It is the
marginal global damage costs of carbon emissions. The LDN indicators and metrics are:
The social cost of carbon (SCC) is the central concept Land cover (land cover change);
for the inclusion of climate change damages in the Cost- Land productivity (net primary productivity, NPP);
Benefit Analysis of public policy and public investments.
It measures the present value in monetary terms of the Carbon stocks (soil organic carbon, SOC).
damages incurred when an additional ton of carbon (or Statement 3 is incorrect. The magnitude of the SOC
any other Greenhouse gas) is released into the atmosphere. storage is spatially and temporally variable and determined
by different abiotic and biotic factors. Globally, the largest
13. Solution: (b) SOC stocks are located in hot-spots such as wetlands
Exp) Option b is the correct answer. and peatlands, most of which occur in regions of
Methane and nitrous oxide are important greenhouse permafrost and in the tropics. Other cases of high SOC
gases. They contribute to global warming. Important content occur as soil horizons buried by volcanic, aeolian,
anthropogenic sources of biogenic methane are wet rice alluvial, colluvial, glacial and anthropogenic processes.
fields, cattle, animal waste, landfills and biomass burning. Grasslands cover approximately 40 percent of the earth’s
Flooded rice cultivation has been identified as one of land surface; represent 70 percent of the global agricultural
the leading global agricultural sources of anthropogenic area, and contain about 20 percent of the world’s SOC
methane (CH4) emissions. Furthermore, it has been stocks (FAO and ITPS, 2015).
estimated that global rice production is responsible for
11% of total anthropogenic CH4 emissions. 16. Solution (a)
Paddies are a potential source of anthropogenic nitrous Exp) Option a is correct.
oxide (N2O) emission as well. In paddies, both the soil Statement 1 is correct. The green-house effect is a
and the rice plants emit N2O into the atmosphere. The
naturally occurring phenomenon that acts as a blanket
rice plant in the paddy is considered to act as a channel
and keeps the earth warm. It maintains the temperature
between the soil and the atmosphere for N2O emission.
suitable for living things to survive. The average
14. Solution: (a) temperature of earth’s surface would be -19°C instead
Exp) Option a is correct. of present 15°C and earth would be frozen and lifeless
planet.
Climate Action Tracker is an independent scientific
analysis produced by two research organizations Statement 2 is incorrect. It has no role in protection
tracking climate action since 2009. It monitors 32 from harmful solar radiations. Due to trapping of
countries, accounting for more than 80 percent of global longwave infra-red terrestrial radiations by greenhouse
emissions. gases there is warming of lower atmosphere.

329 Workbook
.
ENVIRONMENT

Statement 3 is incorrect. Due to lesser density and increase in the likelihood of extreme events such as
thickness of air column at higher elevation as compared heat wave, flooding, hurricanes, etc. We are now seeing
to lower region, the air temperature is generally cooler. episodic rainfall confined to only few days rather than a
As a result of the rarified atmosphere there is lesser long stretch this leads to events of flooding, e.g. Chennai
concentration of greenhouse gases to support this Floods.
effect. Due to this greenhouse effect is weaker at higher Statement 2 is correct. Global warming increases over
elevations. all temperature of the earth. This increased temperature
17. Solution (a) leads to melting of glaciers. As we saw in glacial lake
outburst in Chamoli Uttarakhand.
Exp) Option a is correct.
Statement 3 is correct. Global warming leads to
The global warming potential of different gases, as per expansion in the ecotone as warmer climates starts
IPCC Assessment Report, over a period is mentioned to expand. Due to this increase in area, range of some
below – vector borne disease like malaria, also increases.
Species Chemical Life Global Warming Statement 4 is correct. Rainfall around the world is
formula time Potential powered by wind movements, which primarily are
(years) (Time Horizon) dependent on temperature difference. With global
20 100 500 warming there is disturbance in heat budget that leads to
Years year year irregular wind patterns. This in turn affect rainfall pattern
as represented in IPCC Special Report on Climate
Carbon CO2 Variable 1 1 1 Change.
dioxide
Methane CH4 12 56 21 6.5 20. Solution (c)
Nitrous N2O  280 310 70 Exp) Option c is correct.
Option 1 is incorrect: As a result of thawing of snow, the
amount of arable land in high-latitude region is likely to
HFC-23 CHF3  9100 11700 9800 increase by reduction of the area of frozen lands.
Sulphur SF6  16300 23900 34900 Option 2 is correct: Arable land along the coast lines is
hexa bound to be reduced as a result of rising sea level.
fluroide
Option 3 is correct: Increased sea level would increase
Perfluro- CF4  4400 6500 10000 water inundation in the coastal area. Thus, salinity of the
romethane water table is likely to increase in coastal areas.
18. Solution (b) Option 4 is correct: Crop yield would be reduced in most
tropical and sub-tropical regions due to decreased water
Exp) Option b is correct. availability, and new or changed insect/ pest incidence.
Statement 1 is correct: Methane is the simplest
hydrocarbon which is made up of one carbon atom and 21. Solution (d)
four hydrogen atoms. It is produced by breakdown of Exp) Option d is correct.
organic materials. It is a flammable gas which is used as a
Option 1 is correct: Drastic changes in ocean temperature
fuel. It is a potent Greenhouse gas.
can lead to coral bleaching events, where corals expel
Statement 2 is correct: Wetlands are the largest natural the symbiotic algae living in their tissues, causing them
source, emitting CH4 (methane) from bacteria that to turn completely white. The potential recovery of
decompose organic materials in the absence of oxygen. such bleaching events is hampered due to the declining
More than 50% of Global emissions of Methane is due to calcification rates on reefs caused by ocean acidification.
human activities. The major sources of Methane emission
include fossil fuels (extraction from Oil and Gas), wastes Option 2 is correct: Warmer Ocean water holds less
(including landfills and wastewater), agriculture and oxygen and is more buoyant than cooler water. This leads
livestock. to reduced mixing of oxygenated water near the surface
with deeper waters, which naturally contain less oxygen.
Statement 3 is incorrect: Methane gas is a precursor to
Warmer water also raises oxygen demand from living
ground level Ozone which is also known as Tropospheric
organisms.
Ozone. About one third of global methane emissions,
which primarily comes from agriculture, contribute Option 3 is correct: Ocean warming leads to sea-level
significantly to background levels of tropospheric ozone rise, resulting from the thermal expansion of sea water
generation. and continental ice melting. This leads to floods in coastal
areas.
19. Solution (c)
Option 4 is correct: Increased ocean temperatures
Exp) Option c is correct. influence the geographical distribution of species. The
Statement 1 is correct. Due global warming there is an species shift in search for optimum temperatures.

Workbook 330
.
ENVIRONMENT

22. Solution (a)


Exp) Option a is correct.
Statement 1 is correct Over 75 per cent districts in India,
home to more than 63.8 crore people, are hotspots of
extreme climate events such as cyclones, floods, droughts,
heat and cold waves, according to a study by the Council
on Energy, Environment and Water (CEEW). The study
also found that, traditionally flood prone districts such as
Cuttack (Odisha), Guntur, Kurnool, Srikakulum (all AP), 25. Solution (a)
Mahbubnagar, Nalgonda (both Telangana), and Paschim Exp) Option a is correct.
Champaran (Bihar) have become drought-prone in recent Option a is correct. The effect of aerosols on climate is
years. The study found a shift in the pattern of extreme normally quantified in terms of aerosol radiative forcing.
climate events, such as flood-prone areas becoming Aerosol radiative forcing is defined as the effect of
drought-prone and vice-versa, in over 40 per cent of anthropogenic aerosols on the radiative fluxes at the top
of the atmosphere (TOA) and at the surface and on the
Indian districts. absorption of radiation within the atmosphere.
Statement 2 is incorrect According to the Ministry of Option b is incorrect. The Aryabhatta Research Institute
Environment, Forest and Climate Change (MoEFCC), of Observational Sciences (ARIES), Nainital have found
India has reduced the emission intensity of its GDP by 21 aerosol radiative forcing over the trans-Himalayas. It
found aerosol radiative forcing larger than the global
per cent over 2005 levels as per Paris Pact commitments averages, implying some amount of radiative effects over
by 2020. It is also expected that India will achieve its target the trans-Himalayas in spite of the clean atmosphere.
of reducing 35 per cent emissions intensity of its GDP well Option c is incorrect. The atmospheric aerosols play a
before the year 2030 as per the MoEFCC. key role in the regional/global climate system through
scattering and absorption of incoming solar radiation and
23. Solution (c) by modifying the cloud microphysics. Some aerosols act
as cloud condensation nuclei (CCN), thus affecting cloud
Exp) Option c is correct
albedo and lifetime. Thus, Aerosol Radiative Forcing has
Option a is incorrect. Nitrogen dioxide is not a green a significant impact on the radiation balance of the Earth.
house gas. It is nitrous oxide (N2O) which is a green Option d is incorrect Aerosol Radiative Forcing does
house gas which is naturally present as part of nitrogen not refer to the forced removal of anthropogenic aerosols
from the atmosphere to mitigate climate change. The
cycle.
effect of the total (anthropogenic + natural) aerosols on
Option b is incorrect. Carbon monoxide is harmful the atmosphere is called aerosol radiative effect or total
gas but it is not a green house gas. It is created from aerosol forcing.
incomplete combustion of organic matter. On complete 26. Solution: (c)
combustion carbon dioxide is produced which is a green Exp) Option c is the correct answer.
house gas. Global warming is the long-term heating of Earth’s climate
Option c is correct. Water vapour is the biggest system observed since the pre-industrial period (between
overall contributor to green house effect. Its amount in 1850 and 1900) due to human activities, primarily fossil
fuel burning.
atmosphere is also controlled by CO2 and other green
house gases. Human influence has been responsible for the retreat of
mountain glaciers in the 20th century.
Option d is incorrect. Ammonia is not a green house Statement 1 is correct. The Sixth Assessment Report
gas. It is generated from the animal wastes, nitrogen cycle, states scientists have ascertained that global warming
urine, etc. will have a serious impact on mountain ranges across the
world, including the Himalayas. Retreating snowlines and
24. Solution (d) melting glaciers is a cause for alarm as this can cause a
Exp) Option d is correct. change in the water cycle, the precipitation patterns,
increased floods as well as an increased scarcity of water
Global warming potential describes the impact of each in the future in the states across the Himalayas.
gas on global warming. It is a measure of the total energy Statement 2 is correct. The Conger ice shelf in East
that a gas absorbs over a period of time (usually 100 Antarctica, which had an approximate surface area of
years) compared to carbon dioxide. 1,200 sq km, collapsed after recording unusually high

331 Workbook
.
ENVIRONMENT

temperatures in March 2022. The record temperatures air. Since the nitrogen component of air is not heated, fuel
were the result of an atmospheric river that trapped heat consumption is reduced, and higher flame temperatures
over the continent. are possible.
27. Solution: (c) Statement 2 is correct. In Pre-combustion method, the
coal is gasified to produce a synthetic gas made from
Exp) Option c is the correct answer. carbon monoxide and hydrogen. The former is reacted
This question is based on the article “Indian scientists with water to produce carbon dioxide, which is captured.
propose multi-sectoral strategies for the prevention The hydrogen can be diverted to a turbine where it can be
and control of pollen allergy” published in PIB on 31st burned to produce electricity. Alternatively, some of this
March 2022. Indian scientists have recommended several gas can be bled off to feed hydrogen fuel cells for cars.
measures for the prevention and control of pollen allergy.
Statement 3 is correct. In Post-combustion method,
Statement 1 is correct: Pollen grains are microscopic CO2 is separated from the flue gas of the power station
structures, which bear androecium – a male by bubbling the gas through an absorber column packed
reproductive organ of a flower. The interior section of with liquid solvents such as ammonia. Once the chemicals
pollen grain contains cytoplasm along with the tube cell, in the absorber column become saturated, a stream of
which converts into a pollen tube and the generative cell superheated steam at around 120C is passed through
releases the sperm nuclei. They are male biological entities it. This releases the trapped CO2, which can then be
produced by higher plant cells that are essential for sexual transported for storage elsewhere.
reproduction.
Pollen themselves are immobile and dispersion is aided 29. Solution: (a)
by agents such as water, insects, birds and wind. Hence, Exp) Option a is the correct answer.
they are considered as aero planktons (floating in the air) A carbon sink is anything that absorbs more carbon from
surrounding human beings. the atmosphere than it releases – for example, plants, the
Statement 2 is incorrect: Pollen Allergy is considered a ocean and soil.
major public health problem that causes morbidity and Statement 1 is correct. The Earth’s soil absorbs roughly
subsequently affects a patient’s quality of life. Pollen due a quarter of all human emissions each year, with a
to their large size cannot enter the thoracic regions of
large portion of this stored in peatland or permafrost,
the respiratory tract but can affect the nasopharyngeal
containing approximately 75% of the carbon pool on
mucous membrane. In India, about 20-30% of the
land. Therefore, soils play a major role in maintaining a
population suffers from allergic rhinitis/hay fever and
balanced global carbon cycle.
approximately 15% develop asthma.
Statement 2 is correct. Peatlands are a type of wetland
Statement 3 is correct: There are many factors responsible
for the increase of pollen allergens in the atmosphere such which are critical for preventing and mitigating the effects
as: of climate change, preserving biodiversity, minimizing
flood risk, and ensuring safe drinking water. They are
Climate change and increase in global surface the largest natural terrestrial carbon store. They store
temperature in the atmosphere have a significant impact more carbon than all other vegetation types in the world
on plant life-cycle events (photosynthesis and plant combined.
growth) and their physiological parameters (such as
pollen production, morphology and pollen season). Statement 3 is incorrect. Conservation tillage refers to
several strategies and techniques for establishing crops in
Magnitude and concentration of pollen is the change in the residue of previous crops, which are purposely left on
meteorological conditions and geographic distribution the soil surface. Reducing tillage reduces soil disturbance
Rapid unplanned urbanization with high vehicular and helps mitigate the release of soil carbon into the
emissions was reported to be linked with rising incidences atmosphere. Conservation tillage improves the carbon
of pollen-induced respiratory allergies. sequestration capacity of the soil.
28. Solution: (d) 30. Solution: (d)
Exp) Option d is the correct answer. Exp) Option d is the correct answer.
Carbon capture and storage, also known as CCS or carbon Blue Carbon ecosystems sequester and store more carbon
sequestration, describes a family of technologies designed – often referred to as ‘blue carbon’ – per unit area than
to tackle global warming by capturing carbon dioxide. terrestrial forests. Blue Carbon refers to coastal, aquatic
There are three main types of carbon capture and storage and marine carbon sinks held by vegetation, marine
(CCS) technology that could eventually help reduce organisms and sediments.
emissions from power stations and other industrial sites: Option 1 is correct - Seaweeds, the larger and visible
pre-combustion, post-combustion and oxyfuel. marine plants are found attached to rocks, corals and
Statement 1 is correct. Oxy-fuel combustion is the other submerged strata in the intertidal and shallow sub
process of burning a fuel using pure oxygen, or a tidal zones of the sea. Seaweeds grow in shallow coastal
mixture of oxygen and recirculated flue gas, instead of waters wherever sizable substrata is available. It is a highly

Workbook 332
.
ENVIRONMENT

productive blue ecosystem. Pluto. The probe reported that icy lava flows have recently
Option 2 is correct - Coastal bays, river mouths and (no more than a billion years ago) covered substantial
tidal marshes form the estuaries. In estuaries, freshwater tracts of its surface. The findings drew particular attention
from rivers meets ocean water and the two are mixed by to a mountainous feature named Wright Mons.
the action off tides. Estuaries are highly productive as Wright Mons is a mountainous feature found on Pluto.
compared to the adjacent river or sea. The mangroves It was informally named by the New Horizons team in
present there forms the Estuarine Mangrove Ecosystem honour of the Wright brothers. It is about 150 km across
and is one of the most productive storage of Blue Carbon. its base and has a central depression (a hole) 40-50 km
Option 3 is correct - Seagrass beds are widespread wide, with a floor at least as low as the surrounding terrain.
in lagoon & in such areas, the population of fish and Scientists claim that Wright Mons is a volcano and cite the
migratory birds are also higher due to the availability of lack of impact craters as evidence that it is not likely to be
food and shelter. Sea grasses on reef flats and near estuaries older than 1-2 billion years.
are also nutrient sinks, buffering or filtering nutrient and
chemical inputs to the marine environment.
Option 4 is correct - Fresh water ecosystem is also
a blue carbon ecosystem. They are classified as lotic
(moving water) or lentic (still or stagnant water). Lotic
water system includes freshwater streams, springs,
rivulets, creeks, brooks, and rivers. Lentic water bodies
include pools, ponds, some swamps, bogs and lakes. They
vary considerably in physical, chemical and biological
characteristics.
Wright Mons
31. Solution: (b) The volume of Wright Mons exceeds 20 thousand cubic
Exp) Option b is the correct answer. kilometres. Although considerably less than the volume of
Carbon Pricing creates a negative externality and leads Mars’s biggest volcanoes, this is similar to the total volume
to reduction in emissions. It aims to shift the burden of of Hawaii’s Mauna Loa, and much greater than the volume
emission on those who are responsible for it. There are of its above sea-level portion.
two main types of carbon pricing: emissions trading 33. Solution: (d)
systems (ETS) and carbon taxes described as follows:
Exp) Option d is the correct answer.
Statement 1 is incorrect - A carbon tax is a fee for making
users of fossil fuels pay for climate damage their fuel use Carbon sequestration is the process of capturing and
imposes by releasing carbon dioxide into the atmosphere, storing atmospheric carbon dioxide. The sequestrated
and for motivating switches to clean energy. India carbon can be used in various ways like:
introduced a nationwide carbon tax in 2010, which is Option 1 is correct - Dry ice has just one ingredient:
currently Rs.400/tonne. It is based on Polluters Pay carbon dioxide. Technicians create dry ice by pumping
Principle. Some implicit taxes like high excise duties on liquid carbon dioxide into holding tanks, which reduces
fuels are also levied by the government. the temperature to -109° F and pressurizes the substance
Statement 2 is correct - Carbon credits can be traded into solid blocks or pellets. The CO2 can be taken from
on both private and public markets. Current rules of the captured carbon.
trading allow the international transfer of credits. Option 2 is correct - One of the most promising uses of
Statement 3 is correct. The prices of credits are captured CO2 is as an alternative feedstock for plastic
primarily driven by the levels of supply and demand in manufacturing, replacing crude oil, natural gas, and coal.
the markets. They are not fixed. Due to the differences in The key to making plastic out of carbon dioxide lies in
the supply and demand in different countries, the prices designing sophisticated catalysts – materials that speed up
of the credits fluctuate. One carbon credit is equal to one the rate of a chemical reaction without being used up in
metric ton of carbon dioxide, or in some markets, carbon the process - such as compounds containing metals like
dioxide equivalent gases (CO2-eq), and are bought and copper.
sold through international brokers, online retailers, and Option 3 is correct - CO2 can be added in the form
trading platforms. There is no fixed price for the carbon of aggregates – or injected during concrete mixing.
credits as it is traded on the exchanges. Multi-Commodity Carbonation curing, also known as CO2 curing, can also
Exchange of India (MCX) launched futures trading in be used after concrete has been cast. These processes turn
carbon credits in 2009. CO2 from a gas to a mineral, creating solid carbonates
that may also improve the strength of concrete. The
32. Solution: (c) captured carbon can be used and concretes can act as sink.
Exp) Option c is the correct answer. Option 4 is correct - In large glass houses with many
The National Aeronautics and Space Administration’s plants, the carbon dioxide present in the air can become
(NASA) New Horizons probe has reported new findings of depleted, effectively slowing the photosynthesis process,

333 Workbook
.
ENVIRONMENT

and consequently carbon dioxide enrichment is required. Ozone (O3) is a highly reactive gas composed of three
Systems are sometimes provided to artificially increase oxygen atoms. It is both a natural and a man-made
the level of carbon dioxide available to the plants. product that occurs in the Earth’s upper atmosphere (the
Sequestrated Carbon dioxide can be used here. stratosphere) and lower atmosphere (the troposphere).
Depending on where it is in the atmosphere, ozone affects
34. Solution: (d) life on Earth in either good or bad ways.
Exp) Option d is correct. Statement 1 is incorrect: Stratospheric ozone is known
Carbon capture and storage (CCS) is a combination to be good ozone. It acts as a protection against harmful
of technologies designed to prevent the release of CO2 ultraviolet radiation of Sun. But not all components of
generated through conventional power generation and UV rays are prevented by ozone, UVC and some parts of
industrial production processes by injecting CO2 in UVB are able to reach the earth.
suitable underground storage reservoirs.
Statement 2 is incorrect: Tropospheric ozone is
Statement 1 is correct. Post-combustion: CO2 is considered to be bad as it is created by chemical reactions
removed from the flue gas resulting from the combustion between oxides of nitrogen (NOx) and volatile organic
of a fossil fuel. Post-combustion separation involves the compounds (VOC), hence it is a secondary pollutant
use of a solvent to capture the CO2. Typical applications
(and not a primary pollutant). This happens when
for this technology include pulverized coal (PC) plants,
pollutants emitted by cars, power plants, industrial boilers,
and natural gas combined cycle plants (NGCC). This
technology is particularly suited to retrofit applications. refineries, chemical plants, and other sources chemically
react in the presence of sunlight.
Statement 2 is correct. Pre-combustion: The primary
fuel in the process is reacted with steam and air or Statement 3 is correct: Tropospheric ozone is a short-
oxygen, and is converted to a mix of carbon monoxide and lived climate pollutant. However, it is known to have
hydrogen, often called a ‘syngas’. The carbon monoxide high global warming potential and is considered third
is subsequently converted to CO2 in a ‘shift reactor’. The most important greenhouse gas after carbon dioxide
CO2 can then be separated, and the hydrogen is used to and methane.
generate power and/or heat. This technology is particularly
suitable to be applied to integrated gasification combined 37. Solution: (d)
cycle (IGCC) power plants. Exp) Option d is correct
Statement 3 is correct. Oxy-fuel combustion: The Ocean acidification is the ongoing decrease in the pH
primary fuel is combusted in oxygen instead of air, which value of the Earth’s oceans, caused by the uptake of
produces a flue gas containing mainly water vapour carbon dioxide (CO2) from the atmosphere. When carbon
and a high concentration of CO2 (80%). The flue gas is dioxide dissolves into seawater, it forms the carbonic acid
then cooled to condense the water vapour, which leaves (H2CO3).
an almost pure stream of CO2. Additional equipment is Option a is incorrect: As sea organisms die on the seabed,
required for the in-situ production of oxygen from air. their remains pile up and are consisting of carbons. Also,
35. Solution: (b) these organisms release calcium into the water. These
compounds have a far-reaching detrimental impact on
Exp) Option b is correct
the composition of the water as they add acidity.
Bromine is a dense, mobile, fuming, reddish-brown liquid
at room temperature. It has a highly unpleasant odor Option b is incorrect: Fuels such as petroleum, diesel,
resembling that of chlorine. Bromine is very harmful to and coal produce lots of carbon dioxide when burnt. This
the atmosphere. Bromine atoms are 40 to 100 times more increases the concentration of carbon dioxide gas in the
destructive in the ozone layer than chlorine atoms. atmosphere, which ultimately finds a way into the water.
Carbon and other atmospheric gases find entry into the
Statement 1 is correct: Bromine and associated
sea through acidic rainfalls or even direct dissolving
compounds are also used widely in fire-retardants,
into the water.
chemical dyes. Methyl bromide, used as a fumigant, is
the largest source of ozone-depleting bromine. Option c is incorrect: It is found that sulphur dioxide
Statement 2 is correct: Bromine is known to have emissions from shipping industry can further ocean
higher Ozone Depleting Potential than chlorine. Hence acidification with a rate that is twofold with respect to that
bromine is more effective at destroying ozone than caused by carbon dioxide emissions. Hence it increases
chlorine. the ocean acidification.
Statement 3 is incorrect: Methyl Bromide is among the Option d is correct: Ocean iron fertilization has been
list of substances that deplete the ozone layer. It is listed as proposed as a method to mitigate anthropogenic climate
an Ozone-Depleting Substances (ODS) under Montreal change. It has been further speculated that ocean iron
Protocol. fertilization could help mitigate ocean acidification. As
they would enhance the productivity of phytoplanktons
36. Solution: (d) which in turn will lead to more carbon capture, reducing
Exp) Option d is correct ocean acidification.

Workbook 334
.
ENVIRONMENT

38. Solution: (b) The larger the GWP, the more that a given gas warms the
Exp) Option b is correct Earth compared to CO2 over that time period. The time
period usually used for GWPs is 100 years. GWPs provide
Methane (CH4) is a hydrocarbon that is a primary a common unit of measure, which allows analysts to add
component of natural gas. Methane is also a greenhouse up emissions estimates of different gases (e.g., to compile
gas (GHG), so its presence in the atmosphere affects a national GHG inventory), and allows policymakers to
the earth’s temperature and climate system. Methane compare emissions reduction opportunities across sectors
is emitted from a variety of anthropogenic and natural and gases.
sources.
Carbon dioxide < Methane < Nitrous Oxide < Sulfur
Option a is incorrect: Methane is a short-lived climate hexafluoride is the correct sequence.
pollutant with an atmospheric lifetime of around 12
years. While its lifetime in the atmosphere is much 41. Solution: (d)
shorter than carbon dioxide, it is much more efficient at Exp) Option d is correct
trapping radiation.
Climate change refers to long-term shifts in temperatures
Option b is correct: Methane is generally considered and weather patterns. These shifts may be natural, such as
second to carbon dioxide in its importance to climate through variations in the solar cycle. But since the 1800s,
change. The presence of methane in the atmosphere can human activities have been the main driver of climate
also affect the abundance of other greenhouse gases, such change, primarily due to burning fossil fuels like coal, oil
as carbon dioxide, water vapor and tropospheric ozone. and gas.
Option c is incorrect: Methane is a key precursor gas of Option 1 is correct. Ocean deoxygenation is one of the
the harmful air pollutant, tropospheric ozone. Globally, most under-reported side-effects of human-induced
increased methane emissions are responsible for half of climate change. The primary causes of deoxygenation are
the observed rise in tropospheric ozone levels. eutrophication and nitrogen deposition from the burning
Option d is incorrect: Agriculture (and not due to of fossil fuels, coupled with the widespread impacts from
leakages from crude oil projects) is the key emitting ocean warming.
sector, responsible for about 40% of methane emissions. Option 2 is correct. Climate change led to disruption of
39. Solution: (c) the ocean food chain. Climate change is rapidly warming
the Earth and altering ecosystems on land and at sea that
Exp) Option c is correct produce our food. In the oceans, most added heat from
Black Carbon and Brown Carbon are two most climate warming is still near the surface and will take
important light absorbing substances in the atmospheric centuries to work down into deeper waters. But as this
aerosol. The absorptive properties of these substances can happens, it will change ocean circulation patterns and
have large impacts on radiative transfer and climate. make ocean food chains less productive.
Statement 1 is incorrect: Brown Carbon is a type Option 3 is correct. Anthropogenic greenhouse gas
of atmospheric aerosol which is a by-product of the emissions have caused an increase in global surface
combustion of organic material (and not fossil fuel). temperature of approximately 1°C since pre-industrial
This class of organic carbon, known for its light brownish times. This has led to unprecedented mass coral bleaching
color, absorbs strongly in the ultraviolet wavelengths. events which – combined with growing local pressures
Statement 2 is incorrect: Black Carbon is produced – have made coral reefs one of the most threatened
both naturally and by human activities as a result of ecosystems on Earth.
the incomplete combustion of fossil fuels, biofuels, and Option 4 is correct. The impacts of climate change on
biomass. It is PM 2.5 (and not PM 10) aerosol whose main different sectors of society are interrelated. Events of flood
source is household energy sector (and not transport). and drought have increased significantly due to climate
Statement 3 is correct: Both the black and brown carbon change. Drought can harm food production and human
are most important light absorbing substances in the health. Flooding can lead to disease spread and damages
atmosphere. The climate and radiative transfer are highly to ecosystems and infrastructure. Human health issues
impacted by the absorptive properties of these substances. can increase mortality, impact food availability, and limit
This in turn leads to global warming. worker productivity.

40. Solution: (d) 6.3. Ocean acidification


Exp) Option d is correct.
42. Solution (c)
Option d is correct.The Global Warming Potential
(GWP) was developed to allow comparisons of the global Exp) Option c is correct.
warming impacts of different gases. Specifically, it is a Statement 1 is correct. In the ocean acidification, the
measure of how much energy the emissions of 1 ton of uptake of atmospheric carbon dioxide by the ocean
a gas will absorb over a given period of time, relative to increases, this results in increase in concentration of
the emissions of 1 ton of carbon dioxide (CO2). hydrogen ions in the ocean.

335 Workbook
.
ENVIRONMENT

Statement 2 is incorrect. As the concentration of process of lowering of ocean pH driven by the uptake
hydrogen ions in the ocean increases, the concentration of carbon compounds by the ocean from the atmosphere
of carbonate ions decreases. This leads to decrease in the (and not NO2). This leads to creation of carbonic acid
pH of the oceans and the oceans become less alkaline. and ultimately increases concentration of hydrogen
Statement 3 is correct. The eutrophication of coastal ions in the ocean.
waters leads to large plankton blooms, and when these Statement 2 is correct. Aragonite form of calcium
blooms collapse and sink to the sea bed the subsequent carbonate is more soluble and is found in most of
respiration of bacteria decomposing the algae leads to a corals, mollusks and some species of algae. Whereas
decrease in sea water oxygen and an increase in CO2. This calcite is less soluble form and mainly found in shells of
leads to decline in ocean pH. planktonic algae, amoeboid protists, etc.
Statement 3 is incorrect. Eutrophication leads to large
43. Solution (a) plankton blooms, when they die and sink to sea bed, they
Exp) Option a is correct. are decomposed by bacteria. This leads to decrease in
Statement 1 is correct: Ocean acidification is the change oxygen in sea water and increase in CO2 concentrations
in ocean chemistry – lowering of ocean pH (i.e., increase which leads to decline in pH I.e increase in acidification.
in concentration of hydrogen ions) driven by the uptake Hence eutrophication enhances the ocean acidification
of carbon compounds by the ocean from the atmosphere. and does not counter it.
Oceans are an important reservoir for CO2, absorbing
a significant quantity of it (one-third) produced by
anthropogenic activities and effectively buffering climate
change.
Statement 2 is correct: The decrease in the amount of
carbonate ions in oceans makes it difficult for marine
calcifying organisms like corals to form biogenic calcium
carbonate. Thus, coral reefs erode faster than they can
rebuild.
Statement 3 is incorrect: Some algae and seagrass
sometimes benefit from ocean acidification as higher
carbon dioxide levels can increase their photosynthetic
potential and growth rates.
44. Solution (c)
Exp) Option c is correct. 46. Solution (b)
Statement 1 is incorrect: Ocean acidification leads Exp) Option b is correct.
to increase in the concentration of carbonic acid, The upper layers of the ocean tend to be supersaturated
bicarbonate ions and hydrogen ions. It leads to decrease with CaCO3 so little dissolution takes place, whilst the
in the concentration of carbonate ions. deep ocean is undersaturated and carbonate readily
dissolves. The first boundary between these two states is
known as the lysocline, the depth at which dissolution
strongly increases in the deep ocean.
Carbonate Compensation Depth (CCD) is the depth in
the sea at which the rate of dissolution of solid calcium
carbonate equals the rate of supply. Surface ocean
waters are usually saturated with calcium carbonate, so
calcareous materials are not dissolved. At mid-depths the
lower temperature and higher CO2 content of seawater
cause slow dissolution of calcareous material. Below
Statement 2 is incorrect: Saturation horizon of calcium about 4500 m waters are rich in dissolved CO2 and able to
carbonate is level below which calcium carbonate dissolve calcium carbonate readily.
minerals undergo dissolution. Due to ocean acidification 47. Solution: (a)
this horizon rises vertically in the water column. This
leads more exposure of calcifying organisms to under Exp) Option a is correct.
saturated water. The ocean absorbs carbon dioxide from the atmosphere
because as the atmospheric concentration increases more
45. Solution (c) is dissolved in the surface water. This water may then mix
Exp) Option c is correct down, or sink as it is cooled, into the deep sea where the
Statement 1 is incorrect. Ocean acidification is the absorbed carbon dioxide can stay locked up for hundreds

Workbook 336
.
ENVIRONMENT

of years as it slowly moves through the deep interior ocean shells and skeletons from calcium carbonate, such as
and back to the atmosphere. corals, oysters, clams, mussels, snails, and phytoplankton
Statement 1 is correct. The carbon dioxide moves and zooplankton, the tiny plants and animals that form
between the atmosphere and the ocean by molecular the base of the marine food web. When CO2 dissolves
diffusion: a difference between Carbon dioxide pressure in seawater, the water chemistry changes such that fewer
in the atmosphere and ocean causes Carbon dioxide to be carbonate ions, the primary building blocks for shells and
exchanged. The Carbon dioxide moves from the air to the skeletons, are available for uptake by marine organisms.
water, when the atmospheric pressure of Carbon dioxide Statement 3 is correct. The cloud seeding and formation
is higher. Hence, the Carbon dioxide is dissolved in the of clouds is hampered by ocean acidification. Due to
ocean because it is soluble. acidification in oceans, Dimethylsulphide (DMS) a
Statement 2 is correct. One of the basic properties of ocean biological sulfur compound emitted to the atmosphere.
is that it absorbs carbon dioxide is through some of its Emission occurs over the oceans by phytoplankton
lifeforms. Phytoplankton and algae both photosynthesize results in decreased cloud formation.
in the ocean. They both consume Carbon Dioxide using Statement 4 is correct. The excess algae and plant in
sunlight and release oxygen. ocean after passing of time eventually decomposes that
Statement 3 is incorrect. The Coriolis effect describes led to produce large amounts of carbon dioxide. This
how Earth’s rotation steers winds and surface ocean lowers the pH of seawater and causes ocean acidification.
currents. Coriolis causes freely moving objects to appear
to move to the right in the Northern Hemisphere and to
the left in the Southern Hemisphere. 6.4. Ozone depletion
48. Solution: (c) 50. Solution (d)
Exp) Option c is the correct answer. Exp) Option d is correct.
Climate change is having enormous impacts on ocean Statement 1 is correct. Manufacturing and processing
ecosystems. Warming trend appears to be accelerating of nylon is energy-intensive, which causes emission of
irrespective of ocean’s vast capacity to absorb heat and greenhouse gases leading to global warming. Nylon tops
carbon dioxide. the list of synthetic materials that have the highest impact
on the environment, according to the Pulse of Fashion
Statement 1 is correct. Warmer waters decrease the
Report, 2017.
upwelling causing lesser nutrients to reach the surface
of ocean water as many marine ecosystems available Statement 2 is correct. The production process of Nylon
prominently in the upwelling areas of ocean. Few releases nitrous oxide, a greenhouse gas 300 times more
prominent examples of such marine ecosystems thriving potent than carbon dioxide, which depletes the ozone.
in the upwelling areas are found near Galapagos Islands Also, Waste water generated during the production of
and along the coast of California. nylon contains the unreacted monomer, caprolactam,
Statement 2 is correct. Global warming will lead to which is polluting. Its untreated discharge through factory
increased water temperature. This increased temperature wastewater causes harm to a range of aquatic organisms.
can cause forced migration of many species in order Statement 3 is correct. Improper disposal of nylon
to reach the temperature requirements they need for products also leads to accumulation of microplastic
feeding and breeding. Warmer waters can directly impact in the aquatic ecosystem. Even if properly disposed,
hatching, growth, development, the age of sexual maturity, microscopic pieces of fiber slowly break down and
the timing of spawning and survival of marine life such as contribute to marine pollution. Conventional nylon
cephalopods. is non-biodegradable; it remains on the earth for
hundreds of years, either in a landfill or an ocean.
49. Solution: (b)
Exp) Option b is correct 51. Solution (d)
Ocean acidification is the ongoing decrease in the pH Exp) Option d is correct.
value of the Earth’s oceans, caused by the uptake of carbon Statement 1 is correct. Ozone is mainly formed at the
dioxide (CO2) from the atmosphere. The main cause tropics as oxygen rises up to stratosphere, where under
of ocean acidification are human activities, namely the the influence of ultra violet radiations ozone is formed.
burning of fossil fuels. As a result concentration of ozone in stratosphere is
Statement 1 is incorrect. Ocean acidification cause more at tropics than at polar regions. From here by
decrease in the ocean’s pH value. As carbon dioxide the Brewer Dobson Circulation this ozone moves
(CO2) dissolves in sea water, it forms carbonic acid, poleward and sink. As a result in troposphere we have
decreasing the ocean’s pH, a process collectively known as more concentration of ozone at polar regions and less
ocean acidification. at tropics.
Statement 2 is correct. Acidification can affect many Statement 2 is correct. Tropospheric or ground level
marine organisms, but especially those that build their ozone is formed by reaction between oxides of nitrogen

337 Workbook
.
ENVIRONMENT

(NOx) and volatile organic compounds (VOCs). This is the foam, refrigeration, and air conditioning sectors that
a main component of smog formation. destroy the protective ozone layer and contribute to
Statement 3 is correct. Bromine is more effective in climate change.
ozone depletion than chlorine. As each bromine atom Option 5 is correct. Methyl chloroform is identified as
destroys hundred times of more ozone molecules than a chemical substance that contributes to the degradation
what a chlorine atom does. of the stratospheric ozone layer in the Earth’s atmosphere
and listed as an ODS under Montreal Protocol.
52. Solution (d)
Exp) Option d is correct. 54. Solution: (c)
Ozone depleting substances are chemicals that destroy Exp) Option c is correct.
the earth’s protective ozone layer. Major ozone depleting The ozone-depleting potential (ODP) of a substance refers
substances (ODS) include: to the relative amount of ozone depletion caused by it. It
1) Chlorofluorocarbons (CFCs), 2) Halon, 3) is the ratio of the impact on ozone of the emission of a
Carbon tetrachloride (CCl4), 4) Methyl chloroform chemical substance to the impact of a similar emission by
(CH3CCl3), 5) Hydrobromofluorocarbons (HBFCs), 6) mass of CFC-11.
Hydrochlorofluorocarbons (HCFCs), 7) Methyl bromide
Statement 1 is correct. Ozone depleting potential is
(CH3Br), 8) Bromochloromethane (CH2BrCl) ODSs are
a measure of how much damage a chemical can cause
manufactured halogen source gases that are controlled
worldwide by the Montreal Protocol. These gases bring to the ozone layer compared with a similar mass of
chlorine and bromine atoms to the stratosphere. The gases trichlorofluoromethane (CFC-11).
that contain chlorine and bromine have long atmospheric CFC-11, with an ozone depleting potential of 1.0, is used
lifetimes. Most of these gases accumulate in the lower as the base figure for measuring ozone depleting potential.
atmosphere because they are relatively unreactive and do The higher the number, the more damage a chemical can
not dissolve readily in rain or snow. Natural air motions cause to the ozone layer.
transport these accumulated gases to the stratosphere, Statement 2 is incorrect. Nitrous oxide (N2O) has an
where they are converted to more reactive gases. Some ozone depleting potential of 0.017, while Carbon dioxide
of these gases then participate in reactions that destroy has an ozone depleting potential of 0.
ozone. Finally, when air returns to the lower atmosphere,
these reactive chlorine and bromine gases are removed Statement 3 is correct. Hydrofluorocarbons (HFCs) have
from Earth’s atmosphere by rain and snow. zero ODP because they do not contain chlorine.

53. Solution: (c) 55. Solution: (c)


Exp) Option c is correct Exp) Option c is correct
Ozone depleting substances are chemicals that destroy Polar stratospheric clouds (PSCs) are clouds in the
the earth’s protective ozone layer. The use of these winter polar stratosphere at altitudes of 15,000–25,000
chemicals is controlled by the Montreal Protocol on m (49,000–82,000 ft). Their role in ozone depletion is as
Substances that Deplete the Ozone Layer (the Montreal follows:
Protocol). There are other ozone depleting substances, but Option a is incorrect - PSCs prolong ozone depletion
their ozone depleting effects are very small, so they are not by delaying chlorine deactivation through the removal
controlled by the Montreal Protocol. of gas‐phase HNO3 and H2O by sedimentation of large
Option 1 is correct. Halons are both atmospheric ozone NAT (nitric acid trihydrate) and ice particles. The PSCs
depletes and greenhouse gases. In accordance with the not only activate chlorine, but they also absorb reactive
Montreal Protocol, their manufacture and consumption nitrogen. If nitrogen oxides were present they would
were phased out in industrialized nations by Jan. 1, 2000. combine with chlorine monoxide to form a reservoir of
Option 2 is correct. Chlorofluorocarbon (CFC) are chlorine nitrate (ClONO2).
any of several organic compounds composed of carbon, Option b is incorrect - In warmer temperatures fewer
fluorine, and chlorine. They are also listed as Ozone polar stratospheric clouds form and they don’t persist
Depleting Substances under Montreal Protocol. as long, thus limiting the ozone-depletion process.
Option 3 is incorrect. Hydrofluorocarbons (HFCs) are Increase of temperature reduces the chances of Polar
a group of industrial chemicals primarily used for cooling stratospheric clouds (PSCs) play important roles in
and refrigeration. HFCs were developed to replace stratospheric ozone depletion during winter and spring at
stratospheric ozone-depleting substances that are high latitudes (e.g., the Antarctic ozone hole). As the sun
currently being phased out under the Montreal Protocol shifts southward during spring of southern hemisphere,
on Substances that Deplete the Ozone Layer. Hence it is the polar region starts to warm. This warming causes the
not Substances that Deplete the Ozone Layer under stratospheric polar vortex and associated westerly winds
Montreal Protocol. to gradually weaken over the period of a few months. (A
Option 4 is correct. Hydrochlorofluorocarbons stratospheric polar vortex is an upper-level low-pressure
(HCFCs) are chemical compounds commonly used in area lying near one of the Earth’s poles).

Workbook 338
.
ENVIRONMENT

Option c is correct - There is a correlation exist between Pair 2 is correctly matched. The mechanism known as
the cycle of ozone depletion and the presence of polar “joint implementation,” allows a country with an emission
stratospheric clouds (PSCs) i.e., the ice particles of the reduction commitment under the Kyoto Protocol (Annex
cloud provided substrates B Party) to earn emission reduction units (ERUs)
for chemical reactions which freed chlorine from its from an emission-reduction project in another Annex
reservoirs. Usually, the reaction between HCl and B Party, each equivalent to one tonne of CO2, which
ClONO2 is very slow, but this reaction occurs at a faster can be counted towards meeting its Kyoto target. Joint
rate in the presence of a suitable substrate which is implementation offers Parties a flexible and cost-efficient
provided by the stratospheric clouds at the poles. PSC means of fulfilling a part of their Kyoto commitments,
particles provide sites for heterogeneous reactions that while the host Party benefits from foreign investment and
convert stable chlorine reservoir species to radicals that technology transfer.
destroy ozone catalytically. Pair 3 is correctly matched Emissions trading, as set out
Option d is incorrect – The clouds appear only above in Article 17 of the Kyoto Protocol, allows countries that
the Polar region. As it reaches over tropics it fizzes away. have emission units to spare - emissions permitted them
It form poleward of about 60°S latitude in the altitude but not “used” - to sell this excess capacity to countries
range 10 km to 25 km during the winter and early spring. that are over their targets. Thus, a new commodity was
The clouds are classified into Types I and II according to created in the form of emission reductions or removals.
their particle size and formation temperature.
58. Solution (d)
6.5. Mitigation strategies Exp) Option d is correct.
56. Solution (d) Statement 1 is incorrect. Hydrogen has a high energy
density as compared to other fuels, thus it produces
Exp) Option d is correct. more energy in lesser weight due to which it can prove
Statement 1 is incorrect: The carbon captured can be to be a viable option for heavy vehicles covering long
stored onsite or can also be transported to some other routes in the future. Hydrogen is two-three times more
storage locations. Once the CO2 has been captured, efficient than burning petrol. The energy in 1 kilogram
it is compressed into liquid state and transported by of hydrogen gas is about the same as the energy in 2.8
pipeline, ship or road tanker. CO2 can then be pumped kilograms of petrol.
underground, usually at depths of 1km or more, to be
The refueling time required for hydrogen is also lesser
stored into depleted oil and gas reservoirs, coalbeds or
when compared to electric vehicles and vehicles running
deep saline aquifers, where the geology is suitable.
on conventional fuels.
Statement 2 is incorrect: Captured carbon is not reactive
as such. Thus, at many places Carbon dioxide Capture and Statement 2 is incorrect. The finance minister in the
Sequestration (CCS) is an important method to decrease Union budget for 2020-21 formally announced the
the emissions from centralized large industrial plants. National Hydrogen Mission which aims for generation
Carbon dioxide is a valuable industrial gas with a large of hydrogen from green power resources.
number of uses that include production of chemicals, for Depending on the nature of the method of its extraction,
example urea, refrigeration systems, inert agent for food hydrogen is categorised into three categories, namely,
packaging, beverages, welding systems, fire extinguishers, grey, blue and green. Hydrogen produced from fossil
water treatment processes, horticulture, precipitated fuels is called grey hydrogen; this constitutes the bulk of
calcium carbonate for the paper industry and many other the hydrogen produced today. Hydrogen generated from
smaller-scale applications fossil fuels with carbon capture and storage options is
called blue hydrogen; hydrogen generated entirely from
57. Solution (d)
renewable power sources is called green hydrogen. In
Exp) Option d is correct. the last process, electricity generated from renewable
Pair 1 is correctly matched Clean Development energy is used to split water into hydrogen and oxygen.
Mechanism (CDM) allows a country with an emission- Green hydrogen has specific advantages. One, it is
reduction or emission-limitation commitment under a clean burning molecule, which can decarbonise a
the Kyoto Protocol (Annex B Party) to implement an range of sectors including iron and steel, chemicals, and
emission-reduction project in developing countries. transportation. Two, renewable energy that cannot be
Such projects can earn saleable certified emission stored or used by the grid can be channelled to produce
reduction (CER) credits, each equivalent to one tonne
hydrogen.
of CO2, which can be counted towards meeting Kyoto
targets. In simple terms Developed countries emit more Statement 3 is correct. Hydrogen, the most common
and lose carbon credits. They provide financial assistance element in nature is not found freely. Hydrogen exists
to developing and least developed countries to create only combined with other elements, and thus has to
clean energy (solar, wind energy etc.) and gain some be extracted from naturally occurring compounds like
carbon credits thereby meeting their Kyoto Quota (Kyoto water (which is a combination of two hydrogen atoms and
units) of emissions without violations. one oxygen atom).

339 Workbook
.
ENVIRONMENT

59. Solution (a) Option b is incorrect. In Solubility Trapping, Carbon


Exp) Option a is correct. dioxide can be dissolved into a liquid, such as water or oil.
Statement 1 is correct: A polyhouse is a specially Option c is incorrect. In Mineral Carbonation, Carbon
constructed structure like a building where specialised dioxide can react with the minerals, fluids, and organic
polythene sheet is used as a covering material under which matter in a geologic formation to form stable compounds/
crops can be grown in partially or fully controlled climatic minerals; largely calcium, iron, and magnesium
conditions. It is covered with a transparent material as to carbonates.
permit the entry of natural light. 62. Solution (a)
Statement 2 is correct: In a polyhouse, crops can be Exp) Option a is correct.
grown throughout the year and does not have to wait for
any particular season. Quality of produce is also found to Statement 1 is incorrect. China had recently announced
be higher in polyhouse. to become carbon net-zero by the year 2060. Net-zero is
a state in which a country’s emissions are compensated
Statement 3 is incorrect: Polyhouses are also helpful in
by absorptions and removal of greenhouse gases from
reducing threats such as extreme heat and pest attacks in
the atmosphere. Absorption can be increased by creating
crops. Along with this, fertilizer application is easier and
more carbon sinks such as forests while removal involves
is controlled automatically with the help of drip irrigation.
application of technologies such as carbon capture and
60. Solution (a) storage.
Exp) Option a is correct. Statement 2 is correct. Removing carbon oxide from
the atmosphere and then storing it is known as carbon
Carbon capture and storage, also known as CCS or
sequestration. Carbon neutrality means having a balance
carbon sequestration, describes the technologies designed
between emitting carbon and absorbing carbon from the
to tackle global warming by capturing CO2 at power
atmosphere in carbon sinks.
stations, industrial sites or even directly from the air and
permanently storing it underground. Statement 3 is incorrect. China (and not USA) is the
world’s largest emitter of greenhouse gases. It accounts for
Option a is correct. Terrestrial Sequestration: A large
almost 30% of global emissions more than the combined
amount of carbon is stored in soils and vegetation, which
emissions in the United States, the European Union and
are our natural carbon sinks. Increasing carbon fixation
India, the three next biggest emitters.
through photosynthesis, slowing down or reducing
decomposition of organic matter, and changing land use 63. Solution (b)
practices can enhance carbon uptake in these natural
Exp) Option b is correct.
sinks.
Statement 1 is incorrect: Carbon Emission Reductions
Option b is incorrect. In geologic sequestration, natural
or Carbon credit is a tradable emission reduction
pore spaces in geologic formations serve as reservoirs for
certificate. One unit of CER is one tonne equivalent of
long-term carbon dioxide storage.
carbon dioxide emission. These are financial instrument
Option c is incorrect. In mineral carbonation, carbon aimed at reducing the CO2 from atmosphere from an
dioxide can react with the minerals, fluids, and organic emission reduction project. It can be used by government,
matter in a geologic formation to form stable compounds/ industries of private individuals to offset their carbon
minerals; largely calcium, iron, and magnesium emission.
carbonates.
Statement 2 is correct: Carbon Credits system was
Option d is incorrect. In hydrodynamic trapping, carbon officially introduced in the Kyoto Protocol. These are
dioxide can be trapped as a gas under low-permeability part of clean development mechanism. CDM allows
cap rock, much like natural gas is stored in gas reservoirs. developed countries to earn certified emission reduction
credits (CERs), or ‘carbon credits’, for investment in
61. Solution (a)
emission reduction projects in developing countries.
Exp) Option a is correct. Carbon capture and storage, CERs generated from CDM projects can then be used
also known as CCS or carbon sequestration, describes by developed countries to offset their national emission
the technologies designed to tackle global warming reduction commitments under the Protocol, and similarly
by capturing CO2 at power stations, industrial sites or by private companies seeking to meet emission reduction
even directly from the air and permanently storing it obligations. These certificates are bought, on a voluntary
underground. basis, by any country or company interested in lowering
In Hydrodynamic Trapping, Carbon dioxide can be its carbon footprint.
trapped as a gas under low-permeability cap rock Statement 3 is incorrect: Carbon Credits can be traded
(much like natural gas is stored in gas reservoirs). privately or in the international market at the prevailing
Carbon sequestration describes long-term storage of market price. Each international transfer is validated by
carbon dioxide or other forms of carbon to either mitigate United Nations Framework Convention on Climate
or defer global warming. Change.

Workbook 340
.
ENVIRONMENT

64. Solution (d) Option 2 is correct. Stratospheric aerosols - Introducing


Exp) Option d is correct. small, reflective particles into the upper atmosphere to
reflect some sunlight before it reaches the surface of the
Gujarat Pollution Control Board (GPCB) has initiated Earth.
the programme for market-based trading system to
reduce air pollution in Surat, Gujarat. It is a regulatory Greenhouse gas removal (GGR) or Carbon
tool that is aimed at reducing the pollution load in an area Geoengineering techniques aim to remove carbon
and at the same time minimizing the cost of compliance dioxide or other greenhouse gases from the atmosphere.
for the industry. It is being described as the world’s It includes:
first market for trading in particulate matter emissions. Option 3 is correct. Ocean fertilisation- Adding
The government would set an emission limit and allows nutrients to the ocean in selected locations to increase
industries to buy and sell permits to stay below the primary production which draws down carbon dioxide
emission cap. from the atmosphere.
Option 4 is correct. Ocean alkalinity enhancement -
65. Solution (b) Grinding up, dispersing, and dissolving rocks such as
Exp) Option b is correct. limestone, silicates, or calcium hydroxide in the ocean to
Statement 1 is incorrect: Carbon offsetting is a variant increase its ability to store carbon and directly ameliorate
of carbon trading earned by a country by investing some ocean acidification.
amount of money in such projects, which will emit lesser 68. Solution: (a)
amount of greenhouse gas in the atmosphere. Carbon
offsets can be acquired through afforestation, renewable Exp) Option a is correct
energy and CO2 sequestration. Whereas in emissions Carbon Offsets are credits for reductions in greenhouse
trading, countries are allowed to sell unused emission gas emissions made at another location, such as wind
units to countries that have exceeded their targets. farms which create renewable energy and reduce the need
for fossil fuel powered energy. Carbon Offsets are sold in
Statement 2 is correct: Carbon offsetting is part of
metric tonnes of Carbon Dioxide equivalent. Details are
Clean Development Mechanism (CDM) and Joint
as follows:
implementation. They were launched as part of Kyoto
protocol. There are two types of Carbon trading under Statement 1 is correct – Carbon offsetting is a zero-sum
the clean development mechanism: game with no net reduction in the green-house gases.
Carbon credits are generated against the reduction made
Emission trading at one place which can be used by the person to emit more
Offset trading at another place. Offsetting refers to reducing emissions
or removing carbon dioxide from the atmosphere in
66. Solution (c) one place to make up for emissions in another. Many see
Exp) Option c is correct. it as excuse for polluting companies to delay emissions.
Statement 1 is correct: Carbon tax is based on the Statement 2 is incorrect - Carbon credits can be traded
‘polluter pay’ principle as it is charged on the consumers on both private and public markets. Current rules of
of fossil fuels which emit Green House Gases. It is a cess trading allow the international transfer of credits. The
levied on usage of fossil fuels. It puts a price on emissions, prices of credits are primarily driven by the levels of
encouraging people, businesses, and governments to supply and demand in the markets. They are not fixed.
produce less of them. A carbon tax’s burden would fall Due to the differences in the supply and demand in
most heavily on energy-intensive industries and lower- different countries, the prices of the credits fluctuate. One
income households. carbon credit is equal to one metric ton of carbon dioxide,
Statement 2 is correct: Tax collected under the carbon or in some markets, carbon dioxide equivalent gases
tax goes to National clean energy fund. This is a non- (CO2-eq), and are bought and sold through international
brokers, online retailers, and trading platforms. There is
lapsable fund set up in 2010-2011.
no fixed price for the carbon credits as it is traded on the
67. Solution (d) exchanges. Multi-Commodity Exchange of India (MCX)
launched futures trading in carbon credits in 2009.
Exp) Option d is correct.
Geoengineering is the deliberate large-scale intervention 69. Solution: (b)
in the Earth’s natural systems to counteract climate Exp) Option b is correct.
change. Geo-engineering refers to a large-scale man-made
Solar radiation management (SRM) or Solar intervention in the climate system of the earth. It is
Geoengineering techniques aim to reflect a small increasingly being discussed as a potential solution to the
proportion of the Sun’s energy back into space. They problem of climate change.
include: Option a is incorrect: Ambient air capture is a technique
Option 1 is correct. Space reflectors - Blocking a small used in solar radiation management which in turn helps
proportion of sunlight before it reaches the Earth. in fighting global warming.

341 Workbook
.
ENVIRONMENT

Option b is correct: Bioventing is a process of stimulating target of Net Zero Carbon Emission by 2030, primarily
the natural in situ biodegradation of contaminants through sourcing its energy requirements through
in soil by providing air or oxygen to existing soil renewable energy sources. Major initiatives undertaken
microorganisms. It is a bioremediation process which is for reduction of carbon emissions include 100 per cent
helps in efficient management of pollutants (and not to electrification of its network by December 2023, use of
control global warming). three phase technology for regenerative braking, “head on
Option c is incorrect: Stratospheric Aerosol Injection generation” technology eliminating the need for separate
diesel fuelled power cars, use of renewable energy source
(SAI) is a theoretical solar geoengineering proposal to
(133.26 MW solar and 103 MW wind installed capacity),
spray large quantities of tiny reflective particles into
provisioning of LED lights at all railway installations, and
the stratosphere, in order to cool the planet by reflecting
creation of additional carbon sink by afforestation.
sunlight back into space. Thus, reducing global warming.
Option d is incorrect: Ocean fertilization is done by
adding nutrients to the ocean to increase the primary
production drawing Carbon from the atmosphere. This
aids in carbon capture and reducing the global warming.
70. Solution: (b)
Exp) Option b is the correct answer.
India’s climate actions, especially the adaptation efforts are
largely financed domestically. The National Adaptation
Fund on Climate Change (NAFCC) was launched in
2015. The projects focus on climate sensitive sectors
such as agriculture, water, forestry as well as the coastal 71. Solution: (c)
and Himalayan ecosystem, and are being implemented
to enhance the adaptive capacity of the most vulnerable Exp) Option c is the correct answer.
sections of our population and ecosystems. India’s commitment to achieve Net Zero emissions by
Climate change-related financial risks pose both micro 2070 is akin to not just walking the talk on the climate
crisis, but running the talk. International Solar Alliance
and macro prudential concerns.
(ISA) and Coalition for Disaster Resilient Infrastructure
Statement 1 is correct. In May 2021, the Reserve Bank (CDRI) are an attempt by India to claim climate mission
of India (RBI) set up a new unit–‘Sustainable Finance leadership at the global level. Both have received wide
Group’ (SFG) within its Department of Regulation to support from developing as well as developed countries.
effectively counter Climate change-related financial Some more initiatives recently been taken by India at the
risks, and for leading the regulatory initiatives in the international stage.
areas of sustainable finance and climate risk. The SFG is Pair 1 is correctly matched. LeadIT was launched
coordinating with, and participating in issues relating to by India and Sweden, with the support of the World
sustainable finance or climate risk, with the international Economic Forum at the UN Climate Action Summit in
standard setting / co-operation bodies, other central banks, New York in September 2019, as one of the nine action
other financial sector regulators and the Government of tracks identified by the UN Secretary-General to boost
India. The group would also be instrumental in suggesting climate ambitions and actions to implement the Paris
strategies and evolving a regulatory framework, including Agreement. In November 2021, the Joint Ministerial
appropriate climate related disclosures, which could Statement released at the Leadership Summit called
be prescribed for banks and other regulated entities to countries and companies to come out with roadmaps for
propagate sustainable practices and mitigate climate- deep reductions in emissions across all heavy industries
related risks in the Indian context. and value chains in the coming decade.
Statement 2 is correct. Solar energy is one of the main Pair 2 is correctly matched. The Coalition for Disaster
source to accomplish the target of 40% of electric power Resilient Infrastructure (CDRI) is an international
from non-fossil-fuel. Government of India has set the partnership of governments and other agencies working
target of achieving 100 GW of solar power capacity to promote resilience to disasters, promoted by India.
in the country by the year 2022 of which 40 GW to be The effort is to climate-proof critical infrastructure in
achieved from rooftop solar (RTS). The scheme provides member countries. The coalition will not create any new
for financial assistance for upto 4 GW of solar roof top infrastructure — rather, it will serve as a ‘knowledge
capacity to residential sector and there is a provision to centre’ for member countries to share and learn best
incentivise the distribution companies for incremental practices with respect to disaster-proofing of infra.
achievement over the previous year. So far, a cumulative It will work towards making existing and upcoming
5.87 GW solar roof top projects have been set up in the infrastructure in member countries more robust and
country. resilient against climate disasters such as floods, heat,
Statement 3 is incorrect. Indian Railways has set a cyclones, forest fires, and rain.

Workbook 342
.
ENVIRONMENT

Pair 3 is correctly matched. In November 2021, Formation of knowledge networks among the existing
the Hon’ble Prime Minister proposed a One-Word knowledge institutions engaged in research and
Movement in the context of climate: LIFE - Lifestyle for development relating to climate science.
Environment, at the COP 26 in Glasgow. This movement Creating institutional capacity for research infrastructure
calls for coming together with collective participation, including access to relevant data sets.
to take lifestyle for environment forward as a campaign Development of national capacity for modeling the
and as a mass movement for environmentally conscious regional impact of climate change on different ecological
life style in a manner that revolutionizes many sectors zones.
and diverse areas such as fishing, agriculture, wellness,
dietary choices, packaging, housing, hospitality, tourism, 74. Solution: (c)
clothing, fashion, water management and energy. PM said Exp) Option c is the correct answer.
the world is recognising that lifestyles have a big role in
The Green Grids initiative is a part of Government of
climate change and proposed a “One-Word Movement”.
India’s commitment to boost renewables and especially
“This One-Word in the context of climate can become the
Solar energy across the globe. Features of it are as follows:
basic foundation of One World. This is a word LIFE…L, I,
F, E, ie, Lifestyle For Environment. What is needed today Statement 1 is correct – The program and launched in
is mindful and deliberate utilisation instead of mindless partnership with UK. India and UK launched a ‘green
and destructive consumption. grids’ initiative—the One Sun One World One Grid
(OSOWOG) project—on the sidelines of the COP26
72. Solution: (a) summit. India had first proposed connecting solar energy
Exp) Option a is the correct answer. supply across borders at the International Solar Alliance
in 2018 to allow parts of the world with excess renewable
While delivering his national statement on India’s behalf, power to send power to other countries. If the world has
the PM presented five commitments from India towards to move to a clean and green future, these interconnected
climate action at COP26. Following are the commitments: transnational grids are going to be critical solutions.
‘Panchamrita’ promises include: Statement 2 is correct - The project aims to connect energy
India will reduce its projected carbon emission by one grids across borders to facilitate a faster transition to
billion tonnes by 2030 (Statement 1 is correct) the use of renewable energy. A transnational grid would
India will reduce the carbon intensity of its economy by 45 allow countries to source solar power from regions where
per cent by 2030 (Statement 2 is correct) it is daytime to meet their green energy needs even when
their own installed solar capacity is not generating energy.
India will achieve net zero by 2070 (Statement 3 is
incorrect) 75. Solution: (b)
India will get its non-fossil energy capacity to 500 Exp) Option b is the correct answer.
gigawatts by 2030 (Statement 4 is correct) This question is based on the article “Explained: What are
India will meet 50 per cent of its energy requirements till SPACs for which a regulatory framework may be in the
2030 with renewable energy (Statement 5 is incorrect) works” published in Indian Express on 21st April 2022.
The Government of India is considering a regulatory
73. Solution: (c) framework for Special Purpose Acquisition Companies
Exp) Option c is the correct answer. (SPACs) to lay the ground for the possible listing of Indian
companies through this route in the future.
There are Eight National Missions which form the core
of the National Action Plan, representing multipronged, Statement 1 is correct: Special Purpose Acquisition
long-term and integrated strategies for achieving key Companies (SPACs) or a Blank-Cheque Company is an
goals in the context of climate change. Among these, entity specifically set up with the objective of acquiring
NMSA and NMSKCC are as follows: a firm in a particular sector.
Statement 1 is correct – Some of the Objectives of Statement 2 is correct: Special Purpose Acquisition
National Mission on Sustainable Agriculture – Companies are essentially shell companies. Shell
corporations or Shell companies are entities that do not
To devise strategic plans at the agro-climatic zone level so
have active business. They are set up to achieve specific
that action plans are contextualised to regional scales.
business objectives such as reducing tax liabilities,
To facilitate access to information and institutional shielding an entity from legal risks, raising capital. A
support by expanding Automatic Weather Station key factor that makes SPACs attractive to investors is the
networks to the panchayat level. people who sponsor them. Globally, prominent names
To promote “laboratory to land” research by creating such as tennis star Serena Williams, Dell CEO have
model villages and model farm units in rainfed and participated in SPACs.
dryland areas. Statement 3 is incorrect: A SPAC aims to raise money in
Statement 2 is correct – National Mission on Strategic an Initial Public Offering (IPO) without any operations
Knowledge on Climate Change aims to: or revenues. The money that is raised from the public is

343 Workbook
.
ENVIRONMENT

kept in an escrow account which can be accessed while pledge, also called the Cancun pledge, made in 2010. It is
making the acquisition. If the acquisition is not made also likely to meet and possibly overachieve its nationally
within two years of the IPO, the SPAC is delisted and the determined contributions (NDCs) under the Paris
money is returned to the investors. Agreement. India has also recently updated its targets in
Paris Agreement for newer contributions.
76. Solution: (c)
Exp) Option c is the correct answer. 77. Solution: (b)
Government of India has taken several steps to promote Exp) Option b is the correct answer.
environmental sustainability and reduce the impact of India is aligning with several likeminded countries to
climate change. Some of these are as follows: shift towards cleaner fuel and gather new technologies for
Option a is correct - India officially ratified the Kigali sustainable development. India-US clean energy agenda is
Amendment of the Montreal Protocol, joining 125 other one such partnership. Features are:
countries in the fight to phase out hydrofluorocarbons Statement 1 is incorrect - India-US Clean Energy
(HFCs) — harmful greenhouse gases used in refrigeration Agenda 2030 Partnership is a joint climate and clean
and air-conditioning that are known to accelerate global energy initiative of India with the US. It was launched in
warming. the Leaders’ Summit on Climate convened by the US
Option b is correct - Under Phase-II of the Faster President virtually. 40 world leaders, including the Prime
Adoption and Manufacture of (Hybrid and) Electric Minister of India, were invited to the event to underscore
Vehicles (FAME) Scheme, India allocated ₹10,000 crore the urgency of stronger climate action.
for advancing electric mobility in public transportation Statement 2 is correct – As per Climate Action and
fleets, four-wheelers, three-wheelers, private two-wheelers Finance Mobilization Dialogue (CAFMD) the US will
as well as electric vehicle charging infrastructure. The collaborate in attracting capital and enhancing the
outlay of Rs 10,000 crore is eleven times more than enabling environment to deploy 450 GW of renewable
the outlay of FAME I which was Rs. 895 crores. This energy capacity in India and demonstrate and scale
significant increase in the outlay, according to automobile innovative clean energy technologies and promote
industry players, shows that electric and hybrid vehicles bilateral clean energy investment and trade. The two
are gaining much-needed momentum in the country. countries will also collaborate in building capacities to
Option c is incorrect - India’s Intended Nationally “measure and manage climate risks”.
Determined Contribution (INDC) of creating an Statement 3 is incorrect – There is no role of USA in
additional carbon sink of 2.5-3 billion tonnes of carbon setting up of Commission for Air Quality Management.
dioxide equivalent through additional forest and tree
It was setup through an ordinance replacing the
cover by 2030, is unlikely to materialise. The current
Environmental Pollution Control Authority. CAQM is a
rate of afforestation — 35 million tonnes per year carbon
statutory mechanism to coordinate and oversee diverse
dioxide equivalent — is lower than what is needed to
efforts to improve air quality in Delhi, Punjab, Haryana,
achieve the target. “At this rate, there will be a shortfall
Rajasthan and UP, with the underlying remedial approach.
from the target pledged.
The establishment of CAQM has the potential to address
Option d is correct – As per the Emissions Gap Report the problem of air pollution but an institution by itself is
2020, India is projected to meet its pre-2020 climate not a solution.

Workbook 344
.
ENVIRONMENT

ENVIRONMENT
GOVERNMENT EFFORTS AND INITIATIVES
(c) 1, 2, 3 and 4
7.1. Previous Years Questions (d) None of the above Acts.
1. Which one of the following is not a site for in-situ 5. Consider the following protected areas:
method of conservation of flora? [UPSC CSE Pre. 2012]
[UPSC CSE Pre. 2011] 1. Bandipur
(a) Biosphere Reserve 2. Bhitarkanika
(b) Botanical Garden 3. Manas
(c) National Park 4. Sunderbans
(d) Wildlife Sanctuary
Which of the above are declared Tiger Reserves?
2. The “Red Data Books” published by the (a) 1 and 2 only
International Union for Conservation of Nature (b) 1, 3 and 4 only
and Natural Resource (IUCN) contain lists of (c) 2, 3 and 4 only
[UPSC CSE Pre. 2011] (d) 1, 2, 3 and 4
1. Endemic plant and animal species present in the 6. In which one among the following categories
biodiversity hotspots. of protected areas in India are local people not
2. Threatened plant and animal species. allowed to collect and use the biomass?
3. Protected sites for conservation of nature and [UPSC CSE Pre. 2012]
natural resources in various countries. (a) Biosphere Reserves
(b) National Parks
Select the correct answer using the codes given
(c) Wetlands declared under Ramsar Convention
below: (d) Wildlife Sanctuaries
(a) 1 and 3
(b) 2 only 7. With reference to the wetlands of India, consider
(c) 2 and 3 the following statements: [UPSC CSE Pre. 2012]
(d) 3 only 1. The country’s total geographical area under the
category of wetlands is recorded more in Gujarat
3. Regarding “carbon credits”, which one of the as compared to other States.
following statements is not correct? 2. In India, the total geographical area of coastal
[UPSC CSE Pre. 2011] wetlands is larger than that of inland wetlands.
(a) The carbon credit system was ratified in Which of the statements given above is/are correct?
conjunction with the Kyoto Protocol (a) 1 only
(b) Carbon credits are awarded to countries or (b) 2 only
groups that have reduced greenhouse gases (c) Both 1 and 2
below their emission quota (d) Neither 1 nor 2
(c) The goal of the carbon credit system is to limit
the increase of carbon dioxide emission 8. Consider the following pairs:
(d) Carbon credits are traded at a price fixed from [UPSC CSE Pre. 2013]
time to time of the United Nation Environment
National Park River flowing through
Programme
the Park
4. With reference to India, consider the following 1. Corbett National Ganga
Central Acts: [UPSC CSE Pre. 2011] Park
1. Import and Export (Control) Act, 1947
2. Kaziranga National Manas
2. Mining and Mineral development (Regulation) Park
Act, 1957
3. Customs Act, 1962 3. Silent Valley Kaveri
National Park
4. Indian Forest Act, 1927
Which of the above pairs is/are correctly matched?
Which of the above Acts have relevance to/bearing (a) 1 and 2 only
on the biodiversity conservation in the country? (b) 3 only
(a) 1 and 3 only (c) 1 and 3 only
(b) 2, 3 and 4 only (d) None

345 Workbook
.
ENVIRONMENT

9. If a wetland of international importance is brought (a) 1 and 2 only


under the ‘Montreux Record’, what does it imply? (b) 3 only
[UPSC CSE Pre. 2014] (c) 1 and 3 only
(a) Changes in ecological character have occurred, (d) 1, 2 and 3
are occurring or are likely to occur in the wetland 13. Consider the following statements regarding
as a result of human interference. ‘Earth Hour’ [UPSC CSE Pre. 2014]
(b) The country in which the wetland is located
1. It is an initiative of UNEP and UNESCO
should enact a law to prohibit any human
activity within five kilometres from the edge of 2. It is a movement in which the participants switch
the wetland. off the lights for one hour on a certain day every
(c) The survival of the wetland depends on the year.
cultural practices and traditions of certain 3. It is a movement to raise the awareness about the
communities living in its vicinity and therefore climate change and the need to save the planet.
the cultural diversity therein should not be Which of the statements given above is/are correct?
destroyed. (a) 1 and 3 only
(d) It is given the status of ‘World Heritage Site’. (b) 2 only
(c) 2 and 3 only
10. With reference to ‘Eco-Sensitive Zones’, which of
(d) 1, 2 and 3
the following statements is/are correct?
[UPSC CSE Pre. 2014] 14. With reference to Bombay Natural History Society
1. Eco-Sensitive Zones are the areas that are (BNHS), consider the following statements:
declared under the Wildlife (Protection) Act, [UPSC CSE Pre. 2014]
1972. 1. It is an autonomous organization under the
2. The purpose of the declaration of Eco-Sensitive Ministry of Environment and Forests.
Zones is to prohibit all kinds of human activities 2. It strives to conserve nature through action-
in those zones except agriculture. based research, education and public awareness.
Select the correct answer using the code given below. 3. It organizes and conducts nature trails and
camps for the general public.
(a) 1 only
(b) 2 only Which of the statements given above is/are correct?
(c) Both 1 and 2 (a) 1 and 3 only
(d) Neither 1 nor 2 (b) 2 only
(c) 2 and 3 only
11. Consider the following pairs: (d) 1, 2 and 3
[UPSC CSE Pre. 2014]
15. With reference to ‘Global Environment Facility’,
Wetlands Conference of rivers which of the following statements is/are correct?
[UPSC CSE Pre. 2014]
1. Harike Wetlands Confluence of Beas and
Satluj/ Sutlej (a) It serves as financial mechanism for ‘Convention
on Biological Diversity’ and ‘United Nations
2. Keoladeo Ghana Confluence of Banas Framework Convention on Climate Change’.
National Park and Chambal (b) It undertakes scientific research on
3. Kolleru Lake Confluence of Musi environmental issues at global level.
and Krishna (c) It is an agency under OECD to facilitate
the transfer of technology and funds to
Which of the above pairs is/are correctly matched? underdeveloped countries with specific aim to
(a) 1 only protect their environment.
(b) 2 and 3 only (d) Both (a) and (b)
(c) 1 and 3 only
(d) 1, 2 and 3 16. With reference to a conservation organization
called ‘Wetlands International’, which of the
12. Consider the following international agreements: following statements is/are correct
[UPSC CSE Pre. 2014] [UPSC CSE Pre. 2014]
1. The international Treaty on Plant Genetic 1. It is an intergovernmental organization formed
Resources for Food and Agriculture. by the countries which are signatories to Ramsar
2. The United Nations Convention to Combat Convention.
Desertification. 2. It works at the field level to develop and mobilize
3. The World Heritage Convention knowledge, and use the practical experience to
Which of the above has/have a bearing on the advocate for better policies.
biodiversity? Select the correct answer using the code given below.

Workbook 346
.
ENVIRONMENT

(a) 1 only (a) Bretton Woods Conference


(b) 2 only (b) Montreal Protocol
(c) Both 1 and 2 (c) Kyoto Protocol
(d) Neither 1 nor 2 (d) Nagoya Protocol
17. Consider the following statements: 22. What is the Rio+20 Conference, often mentioned
[UPSC CSE Pre. 2014] in the news? [UPSC CSE Pre. 2015]
1. Animal Welfare Board of India is established (a) It is the United Nations Conference on
under the Environment (Protection) Act, 1986. Sustainable Development
2. National Tiger Conservation Authority is a (b) It is a Ministerial Meeting of the World Trade
statutory body. Organization
3. National Ganga River Basin Authority is chaired (c) It is a Conference of the Inter Government Panel
by the Prime Minister. on Climate Change.
Which of the statements given above is/are correct? (d) It is a Conference of the Member Countries of
(a) 1 only the Convention on Biological Diversity.
(b) 2 and 3 only 23. Which of the following statements regarding
(c) 2 only
‘Green Climate Fund’ is/are correct?
(d) 1, 2 and 3
[UPSC CSE Pre. 2015]
18. Which of the following National Parks is unique 1. It is intended to assist the developing countries
in being a swamp with floating vegetation that in adaptation and mitigation practices to counter
supports a rich biodiversity? climate change.
[UPSC CSE Pre. 2015] 2. It is founded under the aegis of UNEP, OECD,
(a) Bhitarkanika National Park Asian Development Bank and World Bank.
(b) Keibul Lamjao National Park Select the correct answer using the codes given
(c) Keoladeo Ghana National Park
below.
(d) Sultanpur National Park
(a) 1 only
19. Which one of the following National Parks has a (b) 2 only
climate that varies from tropical to subtropical, (c) Both 1 and 2
temperate and arctic? [UPSC CSE Pre. 2015] (d) Neither 1 nor 2
(a) Khangchendzonga National Park
(b) Nandadevi National Park 24. With reference to an organization known as
(c) Neora Valley National Park ‘BirdLife International’, which of the following
(d) Namdapha National Park statements is/are correct? [UPSC CSE Pre. 2015]
1. It is a Global Partnership of Conservation
20. With reference to the International Union for Organizations.
Conservation of Nature and Natural Resource
(IUCN) and the Convention of International 2. The concept of ‘biodiversity hotspots’ originated
Trade in Endangered Species of Wild Fauna and from this organization.
Flora (CITES), which of the following statements 3. It identifies the sites known/referred to as
is/are correct ? [UPSC CSE Pre. 2015] ‘Important Bird and Biodiversity Areas’.
1. IUCN is an organ of the United Nations and Select the correct answer using the code given below.
CITES is an international agreement between (a) 1 only
governments. (b) 2 and 3 only
2. IUCN runs thousands of field projects around the (c) 1 and 3 only
world to better manage natural environments. (d) 1, 2 and 3
3. CITES is legally binding on the States that have
joined it, but this Conventional does not take the 25. BioCarbon Fund Initiative for Sustainable Forest
place of national laws. Landscapes’ is managed by the
[UPSC CSE Pre. 2015]
Select the correct answer using the codes given
below. (a) Asian Development Bank
(a) 1 only (b) International Monetary Fund
(b) 2 and 3 only (c) United Nations Environment Programme
(c) 1 and 3 only (d) World Bank
(d) 1, 2 and 3 26. With reference to ‘Forest Carbon Partnership
21. Which one of the following is associated with Facility’, which of the following statements is/are
the issue of control and phasing out of the use of correct? [UPSC CSE Pre. 2015]
ozone-depleting substances? 1. It is a global partnership of governments,
[UPSC CSE Pre. 2015] businesses, civil society and indigenous peoples.

347 Workbook
.
ENVIRONMENT

2. It provides financial aid to universities, individual 30. The term ‘Intended Nationally Determined
scientists and institutions involved in scientific Contributions’ is sometimes seen in the news in
forestry research to develop eco-friendly and the context of [UPSC CSE Pre. 2016]
climate adaptation technologies for sustainable (a) pledges made by the European countries to
forest management. rehabilitate refugees from the war-affected
3. It assists the countries in their ‘REDD+ Middle East
(Reducing Emissions from Deforestation +)’ (b) plan of action outlined by the countries of the
efforts by providing them which financial and world to combat climate change
technical assistance.
(c) capital contributed by the member countries
Select the correct answer using the code given below. in the establishment of Asian Infrastructure
(a) 1 only Investment Bank
(b) 2 and 3 only (d) plan of action outlined by the countries of the
(c) 1 and 3 only world regarding Sustainable Development Goals
(d) 1, 2 and 3
31. What is/are the importance/ importances of the
27. The Genetic Engineering Appraisal Committee is ‘United Convention to Combat Desertification’?
constituted under the [UPSC CSE Pre. 2015] [UPSC CSE Pre. 2016]
(a) Food Safety and Standards Act, 2006
1. It aims to promote effective action through
(b) Geographical Indications of Goods (Registration
innovative national programmes and supportive
and Protection) Act, 1999
(c) Environment (Protection) Act, 1986 international partnerships
(d) Wildlife (Protection) Act, 1972 2. It has a special/particular focus on South Asia
and North Africa regions, and its Secretariat
28. With reference to the Agreement at the UNFCCC facilitates the allocation of major portion of
Meeting in Paris in 2015, which of the following financial resources to these regions.
statements is/are correct? [UPSC CSE Pre. 2016]
3. It is committed to bottom-up approach,
1. The Agreement was signed by all the member
encouraging the participation of local people in
countries of the UN and it will go into effect in
combating the desertification.
2017.
2. The Agreement aims to limit the greenhouse Select the correct answer using the code given below.
gas emissions so that the rise in average global (a) 1 only
temperature by the end of this century does not (b) 2 and 3 only
exceed 2° C or even 1.5° C above pre-industrial (c) 1 and 3 only
levels. (d) 1, 2 and 3
3. Developed countries acknowledged their
historical responsibility in global warming and 32. The FAO accords the status of ‘Globally Important
committed to donate $ 1000 billion a year from Agricultural Heritage System (GIAHS)’ to
2020 to help developing countries to cope with traditional agricultural systems. What is the
climate change. overall goal of this initiative?
Select the correct answer using the code given below. [UPSC CSE Pre. 2016]
(a) 1 and 3 only 1. To provide modern technology, training in
(b) 2 only modern farming methods and financial support
(c) 2 and 3 only to local communities of identified GIAHS so as
(d) 1, 2 and 3 to greatly enhance their agricultural productivity.
2. To identify and safeguard eco-friendly traditional
29. Consider the following statements:
[UPSC CSE Pre. 2016] farm practices and their associated landscapes,
agricultural biodiversity and knowledge systems
1. The Sustainable Development Goals were first
of the local communities
proposed in 1972 by a global think tank called
the ‘Club of Rome’. 3. The provide Geographical Indication status to
2. The Sustainable Development Goals have to be all the varieties of agricultural produce in such
achieved by 2030. identifies GIAHS
Which of the statements given above is/are correct? Select the correct answer using the code given below.
(a) 1 only (a) 1 and 3 only
(b) 2 only (b) 2 only
(c) Both 1 and 2 (c) 2 and 3 only
(d) Neither 1 nor 2 (d) 1, 2 and 3

Workbook 348
.
ENVIRONMENT

33. Consider the following pairs: to reduce greenhouse gas emissions to specified
[UPSC CSE Pre. 2016] levels by the year 2022
(d) It is one of the multilateral REDD+ initiatives
Terms sometimes seen Their origin hosted by the World Bank
in the news
37. With reference to ‘Agenda 21’, sometimes seen in
1. Annex-I Countries Cartagena Protocol the news, consider the following statements:
2. Certified Emissions Nagoya Protocol [UPSC CSE Pre. 2016]
Reductions 1. It is a global action plan for sustainable
3. Clean Development Kyoto Protocol development.
Mechanisms 2. It originated in the World Summit on Sustainable
Development held in Johannesburg in 2002.
Which of the pairs given above is/are correctly
matched? Which of the statements given above is/are correct?
(a) 1 and 2 only (a) 1 only
(b) 2 and 3 only (b) 2 only
(c) 3 only (c) Both 1 and 2
(d) 1, 2 and 3 (d) Neither 1 nor 2
34. With reference to an initiative called ‘The 38. In the cities of our country, which among the
Economics of Ecosystems and Biodiversity following atmospheric gases are normally
(TEEB)’, which of the following statements is/are considered in calculating the value of Air Quality
correct? [UPSC CSE Pre. 2016] Index? [UPSC CSE Pre. 2016]
1. It is an initiative hosted by UNEP, IMF and 1. Carbon dioxide
World Economic Forum. 2. Carbon monoxide
2. It is a global initiative that focuses on drawing 3. Nitrogen dioxide
attention to the economic benefits of biodiversity. 4. Sulfur dioxide
3. It presents an approach that can help 5. Methane
decisionmakers recognize, demonstrate and Select the correct answer using the code given below.
capture the value of ecosystems and biodiversity. (a) 1, 2 and 3 only
Select the correct answer using the code given below. (b) 2, 3 and 4 only
(a) 1 and 2 only (c) 1, 4 and 5 only
(b) 3 only (d) 1, 2, 3, 4 and 5
(c) 2 and 3 only
39. Which of the following best describes/ describe the
(d) 1, 2 and 3
aim of ‘Green India Mission’ of the Government of
35. Proper design and effective implementation India? [UPSC CSE Pre. 2016]
of UNREDD+ Programme can significantly 1. Incorporating environmental benefits and
contribute to [UPSC CSE Pre. 2016] costs into the Union and State Budgets thereby
1. protection of biodiversity implementing the ‘green accounting’
2. resilience of forest ecosystems 2. Launching the second green revolution to
3. poverty reduction enhance agricultural output so as to ensure food
security to one and all in the future
Select the correct answer using the code given below.
3. Restoring and enhancing forest cover and
(a) 1 and 2 only
responding to climate change by a combination
(b) 3 only
of adaptation and mitigation measures
(c) 2 and 3 only
(d) 1, 2 and 3 Select the correct answer using the code given below.
(a) 1 only
36. What is ‘Greenhouse Gas Protocol’? (b) 2 and 3 only
[UPSC CSE Pre. 2016] (c) 3 only
(a) It is an international accounting tool for (d) 1, 2 and 3
government and business leaders to understand,
quantify and manage greenhouse gas emissions. 40. Consider the following statements:
(b) It is an initiative of the United Nations to offer [UPSC CSE Pre. 2017]
financial incentives to developing countries to 1. Climate and Clean Air Coalition (CCAC) to
reduce greenhouse gas emissions and to adopt reduce Short Lived Climate Pollutants is a
eco-friendly technologies unique initiative of G20 group of countries.
(c) It is an intergovernmental agreement ratified by 2. The CCAC focuses on methane, black carbon
all the member countries of the United Nations and hydrofluorocarbons.

349 Workbook
.
ENVIRONMENT

Which of the statements given above is/are correct? (a) It enjoys the same level of protection as the tiger.
(a) 1 only (b) It no longer exists in the wild, a few individuals
(b) 2 only are under captive protection; and not it is
(c) Both 1 and 2 impossible to prevent its extinction.
(d) Neither 1 nor 2 (c) It is endemic to a particular region of India.
(d) Both (b) and (c) stated above are correct in this
41. With reference to ‘Global Climate Change context.
Alliance’, which of the following statements is/are
correct? [UPSC CSE Pre. 2017] 46. According to the Wildlife (Protection) Act, 1972,
1. It is an initiative of the European Union. which of the following animals cannot be hunted
by any person except under some provisions
2. It provides technical and financial support
provided by law? [UPSC CSE Pre. 2017]
to targeted developing countries to integrate
climate change into their development policies 1. Gharial
and budgets. 2. Indian wild ass
3. It is coordinated by World Resources Institute 3. Wild buffalo
(WRI) and World Business Council for Select the correct answer using the code given below:
Sustainable Development (WBCSD). (a) 1 only
Select the correct answer using the code given below: (b) 2 and 3 only
(a) 1 and 2 only (c) 1 and 3 only
(b) 3 only (d) 1, 2 and 3
(c) 2 and 3 only 47. How is the National Green Tribunal (NGT)
(d) 1, 2 and 3 different from the Central Pollution Control
42. Consider the following statements: Board (CPCB)? [UPSC CSE Pre. 2018]
[UPSC CSE Pre. 2017] 1. The NGT has been established by an Act whereas
1. Climate and Clean Air Coalition (CCAC) to the CPCB has been created by an executive order
Reduce Short Lived Climate Pollutants is a of the Government.
unique initiative of G2O group of countries. 2. The NGT provides environmental justice and
2. The CCAC focuses on methane, black carbon helps reduce the burden of litigation in the
and hydrofluorocarbons. higher courts whereas the CPCB promotes
cleanliness of streams and wells, and aims to
Which of the statements given above is/are correct? improve the quality of air in the country.
(a) 1 only
Which of the statements given above is/are correct?
(b) 2 only
(c) Both 1 and 2 (a) 1 only
(d) Neither 1 nor 2 (b) 2 only
(c) Both 1 and 2
43. Consider the following statements in respect (d) Neither 1 nor 2
of Trade Related Analysis of Fauna and Flora in
Commerce (TRAFFIC): [UPSC CSE Pre. 2017] 48. In which one of the following States is Pakhui
Wildlife Sanctuary located? [UPSC CSE Pre. 2018]
1. TRAFFIC is a bureau under United Nations
Environment Programme (UNEP). (a) Arunachal Pradesh
(b) Manipur
2. The mission of TRAFFIC is to ensure that trade (c) Meghalaya
in wild plants and animals is not a threat to the (d) Nagaland
conservation of nature.
Which of the above statements is/are correct? 49. With reference to the ‘Global Alliance for Climate-
Smart Agriculture (GACSA)’, which of the
(a) 1 only
following statements is/are correct?
(b) 2 only
[UPSC CSE Pre. 2018]
(c) Both 1 and 2
(d) Neither 1 nor 2 1. GACSA is an outcome of the Climate Summit
help in Paris in 2015.
44. The term ‘M-STrIPES’ is sometimes seen in the 2. Membership of GACSA does not create any
news in the context of [UPSC CSE Pre. 2017] binding obligations.
(a) Captive breeding of Wild Fauna 3. India was instrumental in the creation of
(b) Maintenance of Tiger Reserves GACSA.
(c) Indigenous Satellite Navigation System
Select the correct answer using the code given below:
(d) Security of National Highways
(a) 1 and 3 only
45. In India, if a species of tortoise is declared protected (b) 2 only
under Schedule I of the Wildlife (Protection) Act, (c) 2 and 3 only
1972, what does it imply? [UPSC CSE Pre. 2017] (d) 1, 2 and 3

Workbook 350
.
ENVIRONMENT

50. The Partnership for Action on Green Economy (c) Kaundinya, Gundla Brahmeswaram and
(PAGE), a UN mechanism to assist countries Papikonda Wildlife Sanctuaries; and Mukurthi
transition towards greener and more inclusive National Park
economies, emerged at [UPSC CSE Pre. 2018] (d) Kawal and Sri Venkateswara Wildlife Sanctuaries;
(a) The Earth Summit on Sustainable Development and Nagarjunasagar -Srisailam Tiger Reserve
2002, Johannesburg
(b) The United Nations Conference on Sustainable 55. Which one of the following National Parks lies
Development 2012, Rio de Janeiro completely in the temperate alpine
(c) The United Nations Framework Convention on [UPSC CSE Pre. 2019]
Climate change 2015, Paris (a) Manas National Park
(d) The World Sustainable Development Summit (b) Namdahpa National Park
2016, New Delhi (c) Neora Valley National Park
51. “Momentum for Change : Climate Neutral Now” (d) Valley of Flowers National Park
is an initiative launched by [UPSC CSE Pre. 2018] 56. Consider the following statements:
(a) The Intergovernmental Panel on Climate Change [UPSC CSE Pre. 2019]
(b) The UNEP Secretariat 1. Under Ramsar Convention, it is mandatory on
(c) The UNFCCC Secretariat the part of the Government of India to protect
(d) The World Meteorological Organization
and conserve all the wetlands in the territory of
52. Consider the following statements: India.
[UPSC CSE Pre. 2018] 2. The Wetlands (Conservation and Management)
1. The definition of “Critical Wildlife Habitat” is Rules, 2010 were framed by the Government of
incorporated in the Forest Rights Act, 2006. India based on the recommendation of Ramsar
2. For the first time in India, Baigas have been Convention.
given Habitat Rights. 3. The Wetlands (Conservation and Management)
3. Union Ministry of Environment, Forest and Rules, 2010 also encompass the drainage area or
Climate Change officially decides and declares catchment regions of the wetlands as determined
Habitat Rights for Primitive and Vulnerable by the authority.
Tribal Groups in any part of India.
Which of the statements given above is/are correct?
Which of the statements given above is/are correct?
(a) 1 and 2 only
(a) 1 and 2 only (b) 2 and 3 only
(b) 2 and 3 only
(c) 3 only
(c) 3 only
(d) 1, 2 and 3
(d) 1, 2 and 3
53. Consider the following statements: 57. Consider the following statements:
[UPSC CSE Pre. 2019] [UPSC CSE Pre. 2020]
1. As per law, the Compensatory Afforestation 1. 36% of India’s districts are classified as
Fund Management and Planning Authority “overexploited” or “critical” by the Central
exists at both National and State levels. Ground Water Authority (CGWA).
2. People’s participation is mandatory in the 2. CGWA was formed under the Environment
compensatory afforestation programmes carried (Protection) Act.
out under the Compensatory Afforestation Fund 3. India has the largest area under groundwater
Act, 2016. irrigation in the world.
Which of the statements given above is/are correct? Which of the statements given above is/are correct?
(a) 1 only (a) 1 only
(b) 2 only (b) 2 and 3 only
(c) Both 1 and 2 (c) 2 only
(d) Neither 1 nor 2
(d) 1 and 3 only
54. Which of the following are in Agasthyamala
58. Among the following Tiger Reserves, which one
Biosphere Reserve? [UPSC CSE Pre. 2019]
has the largest area under “Critical Tiger Habitat”?
(a) Neyyar, Peppara and Shendurney Wildlife
Sanctuaries; and Kalakad Mundanthurai Tiger [UPSC CSE Pre. 2020]
Reserve (a) Corbett
(b) Mudumalai, Sathyamangalam and Wayanand (b) Ranthambore
Wildlife Sanctuaries; and Silent Valley National (c) Nagarjunsagar-Srisailam
Park (d) Sunderbans

351 Workbook
.
ENVIRONMENT

59. Which one of the following protected areas is well 3. EP100 brings together leading companies
known for the conservation of a sub-species of the committed to driving innovation in energy
Indian swamp deer (Barasingha) that thrives well efficiency and increasing competitiveness while
on hard ground and is exclusively graminivorous? delivering on emission reduction goals.
[UPSC CSE Pre. 2020] 4. Some Indian companies are members of EP100.
(a) Kanha National Park 5. The International Energy Agency is the
(b) Manas National Park Secretariat to the “Under2 Coalition”.
(c) Mudumalai Wildlife Sanctuary Which of the statements given above are correct?
(d) Tal Chhapar Wildlife Sanctuary
(a) 1, 2, 4 and 5
60. Which of the following are the most likely places (b) 1, 3 and 4 only
to find the musk deer in its natural habitat? (c) 2, 3 and 5 only
[UPSC CSE Pre. 2020] (d) 1, 2, 3, 4 and 5
1. Askot Wildlife Sanctuary 64. In the context of WHO Air Quality Guidelines,
2. Gangotri National Park consider the following statements:
3. Kishanpur Wildlife Sanctuary [UPSC CSE Pre. 2022]
4. Manas National Park 1. The 24-hour mean of PM5 should not exceed
Select the correct answer using the code given below: 15 ug/m³ and annual mean of PM5 should not
(a) 1 and 2 only exceed 5 μg/m³.
(b) 2 and 3 only 2. In a year, the highest levels of ozone pollution
(c) 3 and 4 only occur during the periods of inclement weather.
(d) 1 and 4 only 3. PM10 can penetrate the lung barrier and enter
61. If a particular plant species is placed under the bloodstream.
Schedule VI of The Wildlife Protection Act, 1972, 4. Excessive ozone in the air can trigger asthma.
what is the implication? [UPSC CSE Pre. 2020] Which of the statements given above are correct?
(a) a license is required to cultivate that plant. (a) 1, 3 and 4
(b) Such a plant cannot be cultivated under any (b) 1 and 4 only
circumstances. (c) 2, 3 and 4
(c) It is a Genetically Modified crop plant. (d) 1 and 2 only
(d) Such a plant is invasive and harmful to the
65. Consider the following statements
ecosystem.
[UPSC CSE Pre. 2022]
62. According to India’s National Policy on Biofuels, 1. The India Sanitation Coalition is a platform to
which of the following can be used as raw materials promote sustainable sanitation and is funded by
for the production of biofuels? the Government of India and the World Health
[UPSC CSE Pre. 2020] Organization.
1. Cassava 2. The National Institute of Urban Affairs is an
2. Damaged wheat grains apex body of the Ministry of Housing and Urban
3. Groundnut seeds Affairs in Government of India and provides
4. Horse grams innovative solutions to address the challenges of
5. Rotten potatoes Urban India.
6. Sugar beet Which of the statements given above is/are correct?
Select the correct answer using the code given below: (a) 1 only
(b) 2 only
(a) 1, 2, 5 and 6 only
(c) Both 1 and 2
(b) 1, 3, 4 and 6 only
(d) Neither 1 nor 2
(c) 2, 3, 4 and 5 only
(d) 1, 2, 3, 4, 5 and 6 66. Which one of the following has been constituted
under the Environment (Protection) Act, 1986?
63. Consider the following statements:
[UPSC CSE Pre. 2022]
[UPSC CSE Pre. 2022]
(a) Central water Commission
1. “The Climate Group” is an international non-
(b) Central Ground Water Board
profit organization that drives climate action by
(c) Central Ground Water Authority
building large networks and runs them.
(d) National Water Development Agency
2. The International Energy Agency in partnership
with the Climate Group launched a global 67. Which one of the following statements best
initiative “EP100”. describes the ‘Polar Code’? [UPSC CSE Pre. 2022]

Workbook 352
.
ENVIRONMENT

(a) It is the international code of safety for ships (c) 2 and 4 only
operating in polar waters. (d) 1, 3 and 4 only
(b) It is the agreement of the countries around the
North Pole regarding the demarcation of their 71. Climate Smart Cities Assessment Framework has
territories in the polar region. been launched by:
(c) It is a set of norms to be followed by the countries (a) C40 Cities Climate Leadership Group
whose scientists undertake research studies In (b) NITI Aayog
the North Pole and South Pole. (c) Ministry of Housing and Urban Affairs
(d) It is a trade and security agreement of the (d) UN-Habitat
member countries of the Arctic Council.
72. With reference to Smog towers recently installed
68. Which one of the following best describes the term in Indian cities, consider the following statements:
“greenwashing:”? [UPSC CSE Pre. 2022] 1. The filters installed in the tower mainly use
(a) Conveying a false impression that a company’s carbon nanofibers.
products are eco-friendly and environmentally 2. They have the capacity to trap particulate matter
sound of all sizes suspended in the air.
(b) Non-Inclusion of ecological/ environmental 3. Polluted air is re-circulated into the atmosphere
costs in the Annual Financial Statements of a after its purification in towers.
country
(c) Ignoring the disastrous ecological consequences Which of the statements given above is/are correct?
while undertaking infrastructure development (a) 1 only
(d) Making mandatory provisions for environmental (b) 2 and 3 only
costs in a government project/programme (c) 1 and 3 only
(d) 1, 2 and 3
69. With reference to Indian laws about wildlife
protection, consider the following statements: 73. Consider the following statements about
[UPSC CSE Pre. 2022] Seabuckthorn which was recently seen in the news:
1. Wild animals are the sole property of the 1. Seabuckthorn helps in controlling soil erosion
government. and prevents desertification.
2. When a wild animal is declared protected, such 2. In India, it grows in the Himalayan region above
animal is entitled for equal protection whether it the tree line.
is found in protected areas or outside. 3. It is also known for its medicinal properties.
3. Apprehension of a protected wild animal Which of the statements given above is/are correct?
becoming a danger to human life is sufficient
ground for its capture or killing. (a) 1 only
(b) 1 and 3 only
Which of the statements given above is/are correct? (c) 2 and 3 only
(a) 1 and 2 (d) 1, 2 and 3
(b) 2 only
(c) 1 and 3 74. With reference to BS-VI fuel norms, consider the
(d) 3 only following:
1. These norms are applicable to only two wheelers,
Practice Questions three wheelers and four wheelers.
2. These aim to reduce the air pollution in India
7.2. Schemes, Projects and Measures due to emissions by automobiles.
70. With reference to recently launched Drinking 3. The norms reduce the sulphur emission from 50
Water Supply Quality Management System in BS-IV to 10 ppm.
consider the following statements: Which of the statements given above is/are correct?
1. The system outlines the process of water supply (a) 1 and 2 only
from raw water sources to household taps. (b) 2 and 3 only
2. Its main focus is on ensuring uniform water (c) 1 and 3 only
supply in metro cities only. (d) 1, 2 and 3
3. It does not mention how water utilities should 75. With reference to the ‘’Gross Environment
treat water. Product (GEP)’’, consider the following statements
4. This system is launched by Ministry of Jal Shakti. 1. It assesses the economic value of the products
Which of the statements given above is/are correct? and services provided by an ecosystem.
(a) 1 and 3 only 2. Uttarakhand has recently decided to introduce
(b) 2 and 3 only GEP in the state.

353 Workbook
.
ENVIRONMENT

Which of the statements given above is/are correct? 79. The Government has launched the Repurpose Used
(a) 1 only Cooking Oil initiative. Which of the following are
(b) 2 only the benefits of this programme?
(c) Both 1 and 2 1. This will limit the Total Polar Compounds in
(d) Neither 1 nor 2 food.
76. Green Economy as a concept, has been promoted 2. This can replace diesel in vehicles and plant
directly/indirectly through which of the following machinery.
initiatives by India? 3. This can help in reducing current account deficit.
1. Decarbonization of the economy through pan- Select the correct code from below:
India shift towards Bharat Stage-VI vehicles. (a) 1 and 2 only
2. Extended Producer Responsibility (EPR) under (b) 1 and 3 only
Plastic Waste Management Rules, 2016 (c) 2 and 3 only
3. Initiatives like Swachh Bharat Abhiyaan and (d) 1, 2 and 3
Namami Gange. 80. Consider the following statements:
4. International Solar Alliance (ISA) to promote 1. The global-level afforestation is a geo-
use of solar energy engineering technique to counteract climate
Select the correct answer using the code given below: change.
(a) 1, 2 and 3 Only 2. Ocean fertilisation increases the level of carbon
(b) 1, 3 and 4 Only dioxide in the atmosphere.
(c) 2, 3 and 4 Only 3. Dissolving rocks such as limestone in the ocean
(d) 1, 2, 3 and 4 increases its ability to store carbon.
77. Consider the following statements regarding the Which of the statements given above is/are correct?
Green Term Ahead Market (GTAM) in electricity (a) 1 and 2 only
sector: (b) 1 and 3 only
1. It will create a Pan India Market for selling and (c) 2 and 3 only
buying of renewable energy. (d) 1, 2 and 3
2. GTAM contracts will be segregated into Solar 81. With reference to National Clean Air Programme,
and Non-Solar Renewable Purchase Obligations. consider the following statements:
Which of the statements given above is/are correct? 1. It has target of reducing the PM2.5 and PM10
(a) 1 only concentration by 20% to 30% by 2024.
(b) 2 only 2. It is implemented by Central Pollution Control
(c) Both 1 and 2 Board.
(d) Neither 1 nor 2 3. Currently, it includes 122 non-attainment cities
78. Consider the following pairs: across 23 states and Union territories.
Which of the statements given above is/are correct?
Index Objective (a) 1 only
1. Environment ranks 180 countries on (b) 3 only
Performance Index environmental health (c) 2 and 3 only
2020 and ecosystem vitality. (d) 1, 2 and 3
2. Global Climate Risk analyses human 82. Consider the following initiatives related to Green
Index impacts and direct building and the agency who developed it.
economic losses due to
weather-related events. Initiatives Agency
3. Climate Change to enhance 1. LEED (Leadership A. TERI
Performance Index transparency in in Energy and
international climate environmental
politics and compare design)
progress made by 2. GRIHA (Green B. Ministry of Power
individual countries. rating for
Which of the pair/s given above is/are correctly integrated habitat
matched? assessment)
(a) 1 only
(b) 1 and 2 only 3. Eco-Niwas Samhita C. USGBC (United
(c) 2 and 3 only States green building
(d) 1, 2 and 3 building Council)

Workbook 354
.
ENVIRONMENT

Which of the following pairs above are correctly 1. It will strengthen the community led initiatives
matched? to support conservation of species such as tiger
(a) 1-A, 2-B, 3-C and elephant.
(b) 1-B, 2-A, 3-C 2. It is financed by the Global Environment Facility.
(c) 1-C, 2-A, 3-B 3. It is being implemented in all states of India.
(d) 1-A, 2-C, 3-B Select the correct option using the code given below:
83. The ‘’Anamaya’’ initiative recently launched by the (a) 1 and 2 only
Government of India aims to: (b) 2 and 3 only
(a) Promote regional arts and handicrafts on (c) 1 and 3 only
e-commerce platforms. (d) 1, 2 and 3
(b) Improve the health and nutrition status of the 88. Which of the following statements is/are correct
tribal communities. regarding Namami Gange?
(c) Promote research-based education in schools 1. NMCG (National mission for clean Ganga)
and universities. is the implementing wing of NGC (National
(d) Transform tourist places that are listed as World Ganga council) and is headed by Prime minister
Heritage Sites. of India.
84. What is the aim of the recently launched RAISE 2. NGC is a statutory body created under water
initiative? pollution act, 1974.
(a) To create Responsible Artificial Intelligence for 3. Ganga Gram project is one of the components of
Social Empowerment. Namami Gange program.
(b) To improve indoor air quality in workspaces in Select the correct answer using the code given below:
India. (a) 1, 2 and 3
(c) To campaign for elimination of open defecation (b) 3 only
and improve solid waste management. (c) 1 and 2 only
(d) To raise awareness about safe, healthy and (d) 1 and 3 only
sustainable dietary habits.
89. With reference to the National Ganga Council,
85. With reference to the Plastic Parks scheme, consider the following:
consider the following: 1. It was set up in 2016 and replaced the National
1. It aims to consolidate and synergize the River Ganga Basin Authority (NRGBA).
capacities of the domestic downstream plastic 2. The National Ganga Council is formed under
processing industry. the Environment (Protection) Act (EPA),1986.
2. The scheme is being implemented by the 3. It is chaired by Environment minister
Ministry for Chemicals & Fertilizers. Which of the statements given above is/are correct?
3. A Plastic Park is an industrial zone devoted to (a) 1 and 2 only
plastic enterprises and its allied industries. (b) 1 only
Which of the statements given above is/are correct? (c) 2 and 3 only
(a) 1 and 2 only (d) 1, 2 and 3
(b) 2 and 3 only
90. Which of the following is incorrect regarding the
(c) 1 and 3 only National Action Plan on Climate Change?
(d) 1, 2 and 3
(a) Promotion of sustainable habitat is one of its key
86. Green Bonus demand, recently seen in the news, dimensions.
refers to? (b) It comprises a set of 8 national missions.
(a) It refers to the extended funds demanded by hilly (c) ‘Perform, Achieve and Trade’ (PAT) scheme is
region administrations in lieu of the economic implemented under this plan.
loss due to environmental conservation. (d) NITI Aayog is responsible for overall
(b) It refers to the payments under the Kyoto implementation of the plan.
protocol to the Annexure 1 countries for their 91. reference to the concept of “Zero Budget Natural
contribution to reducing greenhouse emissions. Farming (ZBNF)”, consider the following
(c) It refers to demand by the countries containing statements:
tropical rainforest to compensate them for 1. ZNBF does not involve the use of chemical
not taking to deforestation for economic pesticides and chemical fertilizers.
development.
2. ZNBF discourages intensive irrigation and deep
(d) It is bonus demanded by the forest dwellers for
ploughing.
preserving the forest resources.
3. Due to lack of expertise and technical know-
87. Which of the following statements regarding the how, ZBNF has not been adopted anywhere in
Green-Ag Project is/are correct? India.

355 Workbook
.
ENVIRONMENT

Which of the statements given above are correct? 95. With reference to the National Clean Air
(a) 1 and 2 only Programme, consider the following:
(b) 2 and 3 only 1. It targets to achieve 20% to 30% reduction in
(c) 1 and 3 only Particulate Matter concentrations by the year
(d) 1, 2 and 3 2024.
92. With reference to recently launched Drinking 2. The year 2017 has been kept as base year for
Water Supply Quality Management System measuring the pollution changes.
consider the following statements: 3. It will be implemented in all metro cities and
1. The system outlines the process of water supply state capitals.
from raw water sources to household taps. 4. NCAP will be a long-term, time-bound, national
2. Its main focus is on ensuring uniform water level strategy.
supply in metro cities only. Which of the statements given above is/are correct?
3. It does not mention how water utilities should (a) 1 and 2 only
treat water. (b) 1, 2 and 3 only
4. This system is launched by Ministry of Jal Shakti. (c) 1, 2 and 4 only
Which of the statements given above is/are correct? (d) 1, 2, 3 and 4
(a) 1 and 3 only 96. With reference to the Graded Response Action
(b) 2 and 3 only Plan, consider the following:
(c) 2 and 4 only
(d) 1, 3 and 4 only 1. The plan was formulated in 2016 for Delhi and
NCR region to reduce pollution.
93. With reference to National Action Plan for Climate 2. GRAP includes the measures to prevent PM10
Change (NAPCC), consider the following pairs: and PM2.5 levels to go beyond the ‘moderate’
national Air Quality Index (AQI) category.
Mission Nodal Ministry
3. It has six stages of implementation as per the
1. National Solar Ministry of New and pollution load.
Mission Renewable Energy
Which of the statements given above is/are correct?
2. National Mission Ministry of Power (a) 1 and 2 only
for Enhanced (b) 2 and 3 only
Energy Efficiency (c) 1 and 3 only
3. National Mission Ministry of (d) 1, 2 and 3
for Sustaining Environment, Forests 97. Consider the following statements with respect to
Himalayan and Climate Indian Cooling Action Plan:
Ecosystem Change
1. It aims at protecting ozone layer by providing
4. National Mission Ministry of Science and sustainable cooling.
on Strategic Technology 2. It can lead to doubling of farmers’ income.
Knowledge for
Climate Change Which of the statements given above is/are correct?
(a) 1 only
Which of the pairs given above are correctly
(b) 2 only
matched?
(c) Both 1 and 2
(a) 1 and 2 only (d) Neither 1 nor 2
(b) 1 and 3 only
(c) 1, 2 and 4 only 98. SATAT initiative, recently seen in news is related
(d) 1, 2, 3 and 4 to
94. Which of the following is/are objectives of the (a) To boost availability of affordable and clean
National Mission for a Green India? transport fuels
1. To increase forest-based livelihood income for 3 (b) Child immunization
million forest dependent households. (c) promoting private investment in space
technology
2. To enhance annual carbon dioxide sequestration.
(d) facilitating credit to pharmaceutical industries
3. Increase Forest/tree cover on 5 million hectare of
forest/non-forest lands 99. With what purpose is the Government of India
Select the correct answer using the code given below: promoting the establishment of Compressed Bio
(a) 1 only Gas (CBG) plants?
(b) 2 only 1. Efficient management of municipal waste
(c) 2 and 3 only 2. Provide a clean and green alternative as transport
(d) 1, 2 and 3 fuel

Workbook 356
.
ENVIRONMENT

3. Provide a boost to farmers’ income 1. These norms are applicable to only four wheelers.
Select the correct answer using the code given below: 2. These aim to reduce the air pollution in India
(a) 2 only due to emissions by automobiles.
(b) 2 and 3 only 3. The norms reduce the sulphur content from 50
(c) 1 and 3 only ppm in BS-IV to 10 ppm.
(d) 1, 2 and 3 Which of the statements given above is/are correct?
(a) 1 and 2 only
100. Consider the following statements with reference
(b) 2 and 3 only
to the Faster Adoption and Manufacturing of
(c) 1 and 3 only
(Hybrid &) Electric Vehicles in India (FAME
(d) 1, 2 and 3
India-2) Scheme:
1. It focuses on supporting electrification of public 104. With reference to the National Initiative on
transportation. Climate Resilient Agriculture, consider the
2. Electric two wheelers are not covered under the following statements:
scheme. 1. The initiative was launched by The Energy and
3. Vehicles fitted with only advanced battery are Resource Institute.
eligible for incentive under the scheme. 2. It aims to enhance the resilience of crops,
livestock and fisheries.
Which of the statements given above is/are correct?
Which of the statements given above is/are correct?
(a) 1 and 2 only
(b) 2 and 3 only (a) 1 only
(c) 1 and 3 only (b) 2 only
(c) Both 1 and 2
(d) 1, 2 and 3
(d) Neither 1 nor 2
101. With reference to System of Air Quality and
105. Consider the following statements regarding the
Weather Forecasting and Research (SAFAR),
National Adaptation Fund for Climate Change
consider the following: (NAFCC):
1. The system is indigenously developed by the 1. It funds the projects in the States and Union
Indian Institute of Tropical Meteorology (IITM), Territories of India that are particularly
Pune. vulnerable to the adverse effects of climate
2. It aims to measure the air quality of a change.
metropolitan city. 2. NABARD has been designated as National
3. It monitors all weather parameters like Implementing Entity (NIE) for implementation
temperature, rainfall, humidity, wind speed etc. of projects under NAFCC.
4. It does not monitor pollutants like benzene and Which of the statements given above is/are correct?
mercury. (a) 1 only
Which of the statements given above is/are correct? (b) 2 only
(a) 1 and 2 only (c) Both 1 and 2
(b) 1, 2 and 3 only (d) Neither 1 nor 2
(c) 1, 3 and 4 only 106. With reference to FAME II Scheme launched by
(d) 1, 2, 3 and 4 the Government of India, consider the following
102. Which of the following pollutants are measured statements:
under the National Air Quality Index? 1. It provides demand incentives for promotion of
1. PM10 electric vehicles in the country.
2. PM2.5 2. The scheme will be applicable only to vehicles
used for public transport or those registered for
3. Carbon monoxide
commercial purposes.
4. Ozone
Which of the statements given above is/are correct?
5. Lead
(a) 1 only
6. Ammonia (b) 2 only
Select the correct answer using the code given below: (c) Both 1 and 2
(a) 1, 2, 3 and 5 only (d) Neither 1 nor 2
(b) 1, 3, 4 and 6 only
107. Delhi declaration is associated with
(c) 2, 3, 5 and 6 only
(d) 1, 2, 3, 4, 5 and 6 (a) Sustainable management of E-waste
(b) Combat air pollution in NCR
103. With reference to BS-VI fuel norms, consider the (c) Achieving land degradation neutrality
following: (d) Tackle the menace of microbeads

357 Workbook
.
ENVIRONMENT

108. With reference to Environmental Impact Which of the statements given above is/are correct?
Assessment (EIA) in India, consider the following (a) 1 only
statements: (b) 1 and 3 only
1. EIA notifications are formulated under the (c) 1 and 2 only
Environment Protection Act, 1986. (d) 2 and 3 only
2. It is the responsibility of Gram Panchayats to 112. Which of the following is incorrect with regards
provide notice for public hearing under EIA. to the Hazardous and Other Wastes (Management
3. EIA is funded by an agency or individual whose & Transboundary Movement) Amendment Rules,
interest is in clearance for the project. 2019?
4. Since 2006, all the projects must mandatorily go (a) Exporters of silk waste have now been given
through the complete process of EIA. exemption from requiring permission from the
Ministry of Environment, Forest and Climate
Which of the above-mentioned statements is/are
Change.
correct? (b) Electrical components manufactured in and
(a) 1 and 3 only exported from India, if found defective can now
(b) 1, 2 and 4 only be imported back into the country, within a year
(c) 2 and 3 only of export, without obtaining permission from
(d) 3 and 4 only the Ministry of Environment.
(c) Solid plastic waste has been allowed for import
7.3. Acts and Policies into the country for Special Economic Zones
(SEZ) and by Export Oriented Units (EOU).
109. Which of the following activities are prohibited (d) Industries which do not require consent under
in various coastal zones according to Coastal Water (Prevention and Control of Pollution)
Regulation Zone Rules, 2019? Act 1974 are now exempted from requiring
1. Non-Conventional Power sources in CRZ I-A authorization.
zone. 113. Which of the following statements is incorrect
2. Tourism facilities like hotels in CRZ II zone. with regards to the Solid Waste Management
3. Agriculture in CRZ III zone. Rules 2016?
4. Construction of monuments in the CRZ IV (a) They focus on segregation of waste at source.
zone. (b) The Rules are applicable even beyond Municipal
Select the correct option using the code given below: areas.
(c) Responsibilities of Generators have been
(a) 1 only introduced to segregate waste in to five streams.
(b) 1, 3 and 4 only (d) The bio-degradable waste should be processed,
(c) 1, 2 and 4 only treated and disposed within the premises as far
(d) 1, 2, 3 and 4 as possible.
110. Which of the following statements regarding the 114. Which of the following statements is/are correct
Biomedical Waste Management Rules, 2016 is/ are regarding E-waste management rules 2016?
correct? 1. Deposit refund scheme has been introduced in
1. Healthcare facilities are exempted from phasing these rules.
out of chlorinated products like gloves. 2. It is not applicable to micro enterprises, but
2. Healthcare facilities have to pretreat biomedical small industries are brought under the purview.
waste before transportation from the premises. 3. Pan India authorization of EPR (Extended
Select the correct option using the code given below: producer responsibility) by CPCB (central
pollution control board) has been introduced.
(a) 1 only
(b) 2 only Select the correct answer using the code given below:
(c) Both 1 and 2 (a) 1 only
(d) Neither 1 nor 2 (b) 1 and 2 only
(c) 2 and 3 only
111. Consider the following statements with reference (d) 1, 2 and 3
to ‘GRAM UJALA’ programme:
115. Which of the following statements is/are correct
1. Under the programme, LED bulbs will be
regarding Plastic waste management rules, 2016?
distributed at an affordable cost in rural areas.
1. Shopkeepers and street vendors are not allowed
2. The programme will have a significant impact on to sell commodities in plastic bags.
India’s fight against climate change.
2. These rules shall apply to every waste generator,
3. The programme is launched by the Ministry of local body, Gram Panchayat, manufacturer,
Rural Development. Importers and producer

Workbook 358
.
ENVIRONMENT

3. There is no provision for thermosetting plastic (d) It seeks to establish a Bar-Code System for bags
in these rules. or containers containing bio-medical waste for
Select the correct answer using the code given below: disposal.
(a) 1 and 2 only 120. Consider the following statements with reference
(b) 2 only to the Compensatory Afforestation Fund
(c) 2 and 3 only (CAMPA) Act, 2016:
(d) 3 only 1. It sets up a National Compensatory Afforestation
116. With reference to the ‘’Biotech-PRIDE Guidelines’’ Fund under the Consolidated Fund of India.
released by the Government of India recently, 2. The Prime Minister is the chairperson of the
consider the following statements: National Compensatory Afforestation Fund
1. They provide framework to facilitate sharing and Management and Planning Authority.
exchange of biological knowledge in India. Which of the statements given above is/are correct?
2. They provide rules for the generation of new (a) 1 only
biological data in Indian universities. (b) 2 only
3. The guidelines seeks to ensure data sharing (c) Both 1 and 2
benefits and equity of access. (d) Neither 1 nor 2
4. They will be implemented through Indian 121. Consider the following statements regarding the
Biological Data Centre. Coastal Regulation Zones (CRZ) in India:
Which of the statements given above are correct? 1. The construction of any new projects in CRZ-I is
(a) 1 and 2 only totally prohibited.
(b) 3 and 4 only 2. Traditional fishing and allied activities by local
(c) 1, 3 and 4 only communities are permitted in CRZ-IV.
(d) 1, 2, 3 and 4
Which of the statements given above is/are correct?
117. With reference to the Plastic Waste Management (a) 1 only
Rules, 2016, which of the following statements is (b) 2 only
incorrect? (c) Both 1 and 2
(a) It aims to increase minimum thickness of plastic (d) Neither 1 nor 2
carry bags from 40 to 50 microns.
(b) It has expanded the jurisdiction of applicability 122. With reference to the Scheduled Tribes and Other
from the municipal area to rural areas. Traditional Forest Dwellers (Recognition of
(c) The persons organising public events have been Forest Rights) Act, 2006, consider the following
made responsible for management of waste statements:
generated from these events. 1. It applies to only individual rights and not
(d) A penalty of Rs 1 lakh rupees has been introduced community rights.
for non-compliance of these rules. 2. The Ministry of Tribal Affairs is the nodal agency
for its implementation.
118. In the context of Biomedical Waste Management
Rules, 2016, consider the following statements: 3. National Parks and Sanctuaries have been
excluded from the purview of this Act.
1. It aims to phase out the use of chlorinated plastic
bags, gloves and blood bags within two years. Which of the statements given above is/are correct?
2. State Government is to provide land for setting (a) 1 and 2 only
up common bio-medical waste treatment facility. (b) 2 only
(c) 3 only
Which of the statements given above is/are correct?
(d) 1 and 3 only
(a) 1 only
(b) 2 only 123. Consider the following statements, regarding the
(c) Both 1 and 2 Biological Diversity Act, 2002:
(d) Neither 1 nor 2 1. It makes provisions for fair and equitable sharing
119. Which of the following is not a salient feature of of the benefits arising out of the use of biological
the biomedical waste management rules 2016? resources.
(a) It provides a comprehensive definition of what 2. It envisages Biodiversity Management
includes biomedical waste. Committees (BMC) at the local level to regulate
(b) Bio-medical waste has been classified into 10 access to the biological resources.
categories instead of the earlier 4 categories to Which of the statements given above is/are incorrect?
improve the segregation of waste at source. (a) 1 only
(c) The State Government will provide the land for (b) 2 only
setting up common bio-medical waste treatment (c) Both 1 and 2
and disposal facilities. (d) Neither 1 nor 2

359 Workbook
.
ENVIRONMENT

124. Consider the following statements regarding the (a) 1 only


Environment (Protection) Act, 1986: (b) 2 only
1. The provisions of this Act have supremacy (c) Both1 and 2
over anything inconsistent contained in any (d) Neither 1 nor 2
enactment other than this Act.
128. Government of India has introduced separate
2. It grants immunity to the officers of the specific environmental impact assessment laws for
Government for any act done under the which of the following geographical regions?
provisions of this Act.
1. Doon Valley in Dehradun
3. Only a person affected by the alleged offences
2. Aravalli ranges in Gurgaon and Alwar
under the Act, can approach the courts for
remedies. 3. Numaligarh in Assam
4. Sundarbans of West Bengal
Which of the statements given above is/are correct?
(a) 1 only Choose the correct answer using the options given
(b) 1 and 2 only below:
(c) 2 and 3 only (a) 1, 2 and 3 only
(d) 1, 2 and 3 (b) 2, 3 and 4 only
(c) 1, 2 and 4 only
125. With reference to the Wildlife (Protection) Act, (d) 1, 2, 3 and 4 only
1972, which of the statements given below is/are
correct? 129. With reference to disadvantages of Environmental
1. The State Governments are not empowered to Impact Assessment, which of the following
declare any wild animal as vermin. statements is/are correct?
2. A separate Schedule VI is added to protect some 1. The Comprehensive EIA report takes the data
plant species. for one season only to prepare the report.
Select the correct answer using the code given below: 2. It is funded by an agency or individual whose
(a) 1 only interest is in clearance for the project.
(b) 2 only 3. The EIA consultants are not held liable for
(c) Both 1 and 2 discrepancies in the report.
(d) Neither 1 nor 2 Choose the correct answer using the options given
below:
126. Consider the following statements with respect to
Wildlife Protection Act 1972: (a) 1 and 2 only
(b) 2 and 3 only
1. Under Schedule 5 of the act, a state government
(c) 1 and 3 only
on its own can declare an animal as vermin.
(d) 1, 2 and 3
2. The act has made it compulsory to immunize the
livestock within a radius of 5 km from national 130. With reference to Environmental Impact
park or wildlife sanctuary. Assessment, consider the following statements:
3. Schedule 6 lists plant species which cannot be 1. State Level Impact Assessment Authority
cultivated, collected, extracted or traded. (SEIAA) is constituted by central government
Which of the statements given above are correct? under Wildlife Protection Act, 1972.
(a) 1 and 2 only 2. The State Expert Appraisal Committee is
(b) 1 and 3 only constituted by respective state governments in
(c) 2 and 3 only consultation with governor.
(d) 1, 2 and 3 3. Ministry of environment has introduced new
rating system with negative markings for SEIAAs
127. With reference to Convention on Environmental
to increase efficiency.
Impact Assessment in a Transboundary Context,
Consider the following statements: Which of the above-mentioned statements is/are
1. The convention was signed post the horrific correct?
destruction of trans-boundary rivers due to (a) 1 and 2 only
World War II. (b) 3 only
2. The Environmental Impact Assessment laws in (c) 1 and 3 only
India were adopted under the obligation of this (d) None of the above
convention. 131. With reference to ‘ClimateHazards and
Which of the above-mentioned statements is/are Vulnerability Atlas of India’, consider the following
correct? statements:

Workbook 360
.
ENVIRONMENT

1. The atlas provides a range of vulnerability in 135. With reference to the National Air Quality Index
various categories with risks for every district in (AQI), consider the following statements:
India. 1. The index covers all air pollutants.
2. According to Atlas, South 24 Parganas district 2. The index is based on ambient concentration
in West Bengal had the highest cyclone values of air pollutants and their likely health
vulnerability in the country for cyclones of all impacts.
intensities. 3. The formulation of the index was an initiative
3. According to the atlas, all of the coastal districts under Swachh Bharat Mission.
of the western coast are highly vulnerable to
cyclones. Which of the above statements is/are correct?
(a) 1 and 2 only
Which of the statements given above is/are correct?
(b) 2 and 3 only
(a) 1 only (c) 1 and 3 only
(b) 2 and 3 only
(d) 1, 2 and 3
(c) 1 and 2 only
(d) 1, 2 and 3 136. Which of the following are some important
pollutants released by steel industry in India?
132. With reference to Environmental Impact
Assessment, consider the following statements: 1. Oxides of sulphur
1. The Central Pollution Control Board issues the 2. Oxide of nitrogen
notice for environmental public hearing for 3. Carbon monoxide
projects. 4. Carbon dioxide
2. All the projects considered for EIA mandatorily Select the correct answer using the code given below.
go for public hearings (a) 1, 3 and 4 only
Which of the above-mentioned statements is/are (b) 2 and 3 only
correct? (c) 1 and 4 only
(a) 1 only (d) 1, 2, 3 and 4
(b) 2 only
(c) Both 1 and 2 137. With reference to Central Pollution Control Board
(d) Neither 1 nor 2 (CPCB), consider the following statements:
1. It is under the administrative control of Ministry
133. Which of the following statements is/are correct of Environment, Forests and Climate Change.
with reference to the Environment Impact
2. It is a statutory body established under the
Assessment (EIA) in India?
Environment (Protection) Act, 1986.
1. The EIA is statutorily backed by the Environment
3. It derives all its powers and functions under the
Protection Act, 1986.
Air (Prevention and Control of Pollution) Act,
2. Under it, all projects categorized into Category 1981.
A and Category B requires clearance by the
Ministry of Environment, Forest and Climate Which of the statements given above is/are correct?
Change. (a) 1 only
Select the correct answer using the code given below: (b) 1 and 3 only
(a) 1 only (c) 2 and 3 only
(b) 2 only (d) 1, 2 and 3
(c) Both 1 and 2 138. With reference to Environmental Protection Act,
(d) Neither 1 nor 2 1986, which of the following statements is/are
134. With reference to the Nagar Van Scheme, consider correct?
the following statements: 1. The Environment Impact Assessment in India is
1. The scheme aims to develop 200 Urban Forests statutorily backed under this act.
across the country by 2025. 2. The act defines Critical Wildlife Habitat for
2. These urban forests will be created only on land forest dwellers.
other than existing forest land in the city. 3. The Coastal Regulation zone is notified under
3. The finances for the scheme are paid from the the Act.
Compensatory Afforestation Fund. Choose the correct answer using the options given
Which of the above statements is/are correct? below:
(a) 1 only (a) 1 only
(b) 2 only (b) 1 and 3 only
(c) 1 and 3 only (c) 3 only
(d) 2 and 3 only (d) 2 and 3 only

361 Workbook
.
ENVIRONMENT

139. Consider the following statements: 2. The process for determining the individual forest
1. All States in India have raised the State Disaster rights is delegated to Gram Sabhas.
Response Force. 3. The Act is applicable in National Parks, Wildlife
2. Disaster Management Division under the Sanctuaries as well as Tiger Reserves.
Ministry of Home Affairs is responsible for Select the correct answer using the code given below:
response and preparedness for natural calamities (a) 1 and 2 only
except drought and epidemics. (b) 2 only
3. The Disaster Management Act, 2005 defines the (c) 3 only
term ‘disaster’. (d) 1, 2 and 3
Which of the above statement is/are correct?
143. Under Joint Forest management (JFM) in India,
(a) 1 only which of the following body is entrusted with the
(b) 2 and 3 only protection and management of nearby forests?
(c) 1 and 3 only
(a) Village community
(d) 1 and 2 only
(b) District Planning Committee
140. With reference to various climate change (c) Nagar Panchayat
initiatives, consider the following statements: (d) Zilla Parishad
1. The Bureau of Indian Standards (BIS) has been 144. With reference to Wetland (Conservation and
formulating Star Labelling Program to improve Management) Rules, 2017 which of the following
energy efficiency of appliances. statements is/are incorrect?
2. Indian Network on Climate Change Assessment 1. It mandates the National Wetlands Committee
(INCCA) was launched by Ministry of to prepare a comprehensive digital inventory of
Environment and Forest to promote domestic all wetlands in the country.
research on Climate Change.
2. Under the rules, “Salt pans” have been newly
Which of the above-mentioned statements is/are included in the category of wetlands.
correct?
Select the correct answer using the code given below:
(a) 1 only
(a) 1 only
(b) 2 only
(b) 2 only
(c) Both 1 and 2
(c) Both 1 and 2
(d) Neither 1 nor 2
(d) Neither 1 nor 2
141. In context of Hazardous and Other Wastes
145. Which of the following conditions must be met by
(Management & Trans boundary Movement)
the country to be called ‘megadiverse’?
Rules, 2016 consider the following statements:
1. Existence of a minimum 5000 endemic plants
1. Import of critical care medical equipment falls
under the prohibited category as hazardous 2. Must have a minimum of 5 biodiversity hotspots
waste. 3. Country must border an ocean
2. It does not cover the waste generated during Select the correct answer using the code given below:
the manufacturing processes of the commercial (a) 2 only
products. (b) 1 and 3 only
3. It is mandatory State Pollution Control Board (c) 2 and 3 only
(SPCB) to prepare an annual inventory of the (d) 1, 2 and 3
waste generated under these rules.
146. Consider the following statements about various
Which of the above statements given above is/are environmental organizations.
correct? 1. Wildlife Trust of India is a statutory body set up
(a) 1 and 2 only after an amendment to the Wild life (Protection)
(b) 1 and 3 only Act in 1991.
(c) 2 and 3 only 2. Wildlife Crime Control Bureau (WCCB) is a
(d) 1, 2 and 3 statutory body set up under the Prevention of
142. With reference to Scheduled Tribes and Other Cruelty to Animals Act of 1960.
Traditional Forest Dwellers (Recognition of Which of the statements given above is/are correct?
Forest Rights) Act, 2006, which of the following (a) 1 only
statements is/are correct? (b) 2 only
1. The Act provides rights to allocation of forest (c) ) Both 1 and 2
land for developmental purposes. (d) Neither 1 nor 2

Workbook 362
.
ENVIRONMENT

151. With reference to the Central Zoo Authority,


7.4. Indian Environmental bodies, which of the following statements is/are correct?
organisations and reports 1. Every zoo in the country is required to obtain
147. Which of the following organisations were recognition from the Central Zoo Authority for
established under the Wildlife Protection Act, its operation.
1972? 2. It identifies endangered species for the purpose
1. National Tiger Conservation Authority of captive breeding.
2. Animal Welfare Board of India 3. It regulates the exchange of animals under
3. Wildlife Crime Control Bureau Schedule-I and II of the Wildlife Protection Act
among zoos in India.
4. National Board for Wildlife
Select the correct answer using the code given below
Select the correct answer using the code given below:
(a) 1 and 2 only
(a) 2 and 4 only (b) 1 and 3 only
(b) 1, 2 and 3 only (c) 2 and 3 only
(c) 1, 3 and 4 only (d) 1, 2 and 3
(d) 1, 2, 3 and 4
152. Which of the following matters can be entertained
148. Consider the following statements regarding the by NGT?
National Board for wildlife: 1. It can regulate social and commercial functions
1. It is a statutory body established under the in public parks.
Wildlife (Protection) Act, 1972. 2. It can prescribe the use of treated waste water
2. The Prime Minister is the Chairperson of the from STP (Sewage treatment plants) at nearby
Board. parks for its effective utilization.
Which of the statements given above is/are incorrect? 3. It can hear matters relating to protection of
(a) 1 only National parks and sanctuaries.
(b) 2 only Select the correct answer using the code given below
(c) Both 1 and 2 (a) 1 and 2 only
(d) Neither 1 nor 2 (b) 2 and 3 only
149. Consider the following statements with respect to (c) 1 and 3 only
the India State of Forest Report (ISFR) 2019: (d) 1, 2 and 3
1. The total forest and tree cover of India comprises 153. Which of the following statements is/are correct
33 percent of the geographical area of the regarding NGT (National green Tribunal)?
country. 1. It is a quasi-judicial body.
2. Mangrove cover in India is assessed as a part of 2. NGT can exercise suo-motu jurisdiction.
the report. 3. NGT can apply the polluter pays principles while
3. All North-Eastern states has witnessed an passing judgements.
increase in forest cover. 4. Environmental relief fund has been established
Which of the statements given above is/are correct? under the National Green Tribunal Act, 2010.
(a) 2 only Select the correct answer using the code given below:
(b) 1 and 2 only (a) 1 and 4 only
(c) 1 and 3 only (b) 1, 2 and 3 only
(d) 2 and 3 only (c) 1 and 3 only
150. With reference to the Animal Welfare Board of (d) 2, 3 and 4 only
India, which of the following statements is/are 154. Which of the following statements is/are correct
incorrect? regarding EPCA (Environmental pollution control
1. It is a non-statutory advisory body on Animal authority) which has been recently dissolved?
Welfare laws. 1. It was constituted under the Environment
2. It also advises on the improvement in the design (Protection) Act, 1986.
of vehicles. 2. Its chairman was to be a retired Supreme Court
3. The board was established due to the efforts of judge.
Menaka Gandhi. 3. It had power to file criminal complaints before
Select the correct answer using the code given below: the court for not following his directions.
(a) 1 and 3 only 4. It has been replaced by the Commission for Air
(b) 2 and 3 only Quality Management in National Capital Region
(c) 2 only (NCR) and Adjoining Areas.
(d) 1, 2 and 3 Select the correct answer using the code given below:

363 Workbook
.
ENVIRONMENT

(a) 1, 2 and 3 only (c) Both 1 and 2


(b) 2 and 4 only (d) Neither 1 nor 2
(c) 1, 3 and 4 only
159. With reference to the National Green Tribunal,
(d) 2, 3 and 4 only
consider the following:
155. With reference to Society of Integrated Coastal 1. NGT deals with civil cases under the
Management (SICOM), consider the following Environment (Protection) Act, 1986 only.
statements: 2. NGT is mandated to make disposal of
1. It is the nodal agency for implementation of applications or appeals finally within 6 months
Integrated Coastal Zone Management (ICZM) of filing of the same.
practices. 3. The NGT has five places of sittings with New
2. It has launched Beach Environment & Delhi as the Principal place of sitting.
Aesthetic Management Services (BEAMS) for Which of the statements given above is/are correct?
development of world class beaches in India. (a) 1 and 2 only
3. It works for livelihood security of coastal and (b) 2 and 3 only
island communities. (c) 1 and 3 only
Which of the statements given above are correct? (d) 1, 2 and 3
(a) 1 and 2 only 160. With reference to the Tree Transplantation policy
(b) 2 and 3 only recently seen in news, consider the following
(c) 1 and 3 only statements:
(d) 1, 2 and 3 1. It has been launched in India recently by the
156. Consider the following statements: Ministry of Environment, Forest and Climate
1. The National Biodiversity Authority (NBA) was Change.
established under the Biological Diversity Act, 2. A minimum of 80% of trees affected by a
2002. development or construction project will be
2. Biodiversity Management Committees, at local transplanted.
levels, are responsible for documentation of 3. The cost for transplantation of trees is borne by
knowledge relating to biological diversity. the Centre and states in the ratio of 50:50.
Which of the statements given above is/are correct? Which of the statements given above is/are correct?
(a) 1 only (a) 1 and 2 only
(b) 2 only (b) 2 and 3 only
(c) Both 1 and 2 (c) 2 only
(d) Neither 1 nor 2 (d) 1, 2 and 3

157. With reference to Wildlife Crime Control Bureau, 161. With reference to the Central Pollution Control
which of the following statement(s) is/are correct? Board (CPCB), consider the following:
1. CPCB is a statutory organisation which was
1. It is a statutory body established under the
constituted under the Water (Prevention and
Wildlife (Protection) Act, 1972.
Control of Pollution) Act, 1974.
2. It is mandated to collect and collate intelligence
2. It is also entrusted with the powers and functions
related to organized wildlife crime activities.
under the Air (Prevention and Control of
3. It works under the Ministry of Home Affairs. Pollution) Act, 1981.
Select the correct answer using the code given below: 3. It aims to improve the quality of air and to
(a) 1 only prevent, control or abate air pollution in the
(b) 1 and 2 only country.
(c) 2 and 3 only Which of the statements given above is/are correct?
(d) 1, 2 and 3 (a) 1 only
158. With reference to the Environment Pollution (b) 2 only
(Prevention and Control) Authority (EPCA), (c) 3 only
consider the following: (d) 1, 2 and 3
1. It was constituted with the objective of ‘protecting 162. Consider the following statements with reference
and improving’ the quality of the environment in to steps taken by India to combat plastic pollution:
the National Capital Region. 1. India piloted a resolution on ‘’Addressing Single
2. It is a statutory body constituted under the Air Use Plastic Product Pollution’’ in the United
Pollution Act, 1981. Nations Environment Assembly.
Which of the statements given above is/are correct? 2. Plastic Waste Management Amendment Rules,
(a) 1 only 2021 prohibits all single use plastic items by
(b) 2 only 2022.

Workbook 364
.
ENVIRONMENT

3. The thickness of plastic carry bags has been 2. Animal Welfare Board of India does not have
increased from 5 microns to 25 microns recently. any power to provide financial assistance for
4. Extended producer responsibility (EPR) is now animal welfare.
applicable to both pre-consumer and post- Which of the statements given above is/are correct?
consumer plastic packaging waste. (a) 1 only
Which of the above statements is/are correct? (b) 2 only
(a) 1 and 4 only (c) Both 1 and 2
(b) 3 only (d) Neither 1 nor 2
(c) 2 and 3 only
(d) 1, 2 and 4 only 167. Consider the following statements:
1. Under the National Disaster Management Plan
163. “ Recently this wetland has been recognised 2016, India has aligned its National Plan with the
under the 1971 Ramsar Convention on Sendai Framework for Disaster Risk Reduction.
Wetlands. statement(s) is/are correct? 2. A National Executive Committee is constituted
1. NDMA can provide support to other countries under the chairmanship of the Prime Minister
affected by major disasters. to oversee the functions of National Disaster
2. NDMA is responsible for approving the plans Management Authority.
prepared by the Ministries or Departments of Which of the statement(s) given above is/are correct?
the Government of India.
(a) 1 only
3. Home Minister is the ex-officio chairman of the
(b) 2 only
NDMA.
(c) Both 1 and 2
Select the correct answer using the codes given (d) Neither 1 nor 2
below:
(a) 1 and 3 only 168. The PRAIVESH Portal, sometimes seen in the
(b) 1 and 2 only news, is
(c) 3 only (a) A single-window system for getting
(d) 1, 2 and 3 environmental clearances.
(b) A platform aimed at starting a culture of Pro-
164. It is a human made wetland. It is formed on a
Active Governance and Timely Implementation.
floodplain of the River Ganga on the Madhya
(c) An integrated public platform that provides
Ganga Barrage. Critically endangered gharial and
the endangered hog deer are found here. It serves single-point availability of key healthcare goods.
as a breeding site for the near-threatened Indian (d) A technology driven Control Room-Cum-
grassbird. It is located within the boundaries of Management Information System.
Hastinapur Wildlife Sanctuary.” 169. With reference to E-Waste Management Rules,
The wetland mentioned here is 2016 consider the following statements:
(a) Sarsai Nawar 1. Deposit Refund Scheme has been introduced
(b) Gogabeel under which the producer charges an additional
(c) Nawabganj bird sanctuay amount as a deposit at the time of sale of the
(d) Haiderpur electrical and electronic equipment.
165. Which of the following activities is/are prohibited 2. The Central Pollution Control Board (CPCB)
inside Eco-Sensitive Zones (ESZs)? is provided with the power to levy fines for
1. Agriculture practices by local communities noncompliance on companies.
2. Rainwater harvesting Which of the statements given above is/are correct?
3. Commercial mining (a) 1 only
4. Hydroelectric projects (b) 2 only
5. Tourism activities like Hot air balloon (c) Both 1 and 2
(d) Neither 1 nor 2
Select the correct answer using the code given below:
(a) 1, 2 and 3 only 170. With reference to the National Building Code of
(b) 3, 4 and 5 only India (NBC), consider the following statements:
(c) 1, 3 and 5 only 1. It is a set of model guidelines for building
(d) 2, 3, 4 and 5 only construction for government as well as the
166. Consider the following statements. private sector construction companies.
1. Zoological survey of India prepares Red Data 2. It is published by the Bureau of Energy Efficiency.
Book of fauna of India. Which of the statements given above is/are correct?

365 Workbook
.
ENVIRONMENT

(a) 1 only 1. It is an inter-governmental wildlife law


(b) 2 only enforcement support body of South Asian
(c) Both 1 and 2 countries.
(d) Neither 1 nor 2 2. It operates from its secretariat located in
Kathmandu.
171. Consider the following statements about
‘Decarbonising Transport in Emerging Select the correct answer using the code given below
Economies’ (DTEE) project: (a) 1 only
(b) 2 only
1. It focusses on reducing transport CO2 emissions
(c) Both 1 and 2
and help nations meet their climate goals and
(d) Neither 1 nor 2
NDCs (Nationally Determined Contributions).
2. It is being implemented in all the nations 175. Which of the following countries is/are members
signatory to the United Nations Framework of the South Asia Co-operative Environment
Convention on Climate Change. Programme (SACEP)?
1. Bangladesh
Which of the statements given above is/are correct?
2. India
(a) 1 only
(b) 2 only 3. Myanmar
(c) Both 1 and 2 4. Pakistan
(d) Neither 1 nor 2 Select the correct answer using the code given below:
(a) 1, 2 and 3 only
172. Which among the following technologies have (b) 1, 2 and 4 only
been introduced to reduce stubble burning in (c) 2, 3 and 4 only
farmlands in India? (d) 1, 2, 3 and 4
1. Turbo Happy Seeder (THS) machine.
2. Pusa bio-decomposer tablets. 176. With reference to the United Nations Environment
Programme (UNEP), which of the statements
3. Super Straw Management System. given below is/are correct?
4. Sowing of Genetically engineered crops. 1. Resource efficiency, disasters and conflicts are
Select the correct answer using the code given below: some of the areas which come under the purview
(a) 1 and 2 only of the UNEP.
(b) 2, 3 and 4 only 2. Global Environmental Facility operates under
(c) 1, 2 and 3 only the UNEP.
(d) 1, 2, 3 and 4 3. It is headquartered at Nairobi, Kenya.
173. Consider the following statements about the Select the correct answer using the code given below:
National Disaster Response Force (NDRF): (a) 1 and 2 only
(b) 2 only
1. NDRF was constituted in 2006 under the
(c) 1 and 3 only
Environment Protection Act, 1986.
(d) 2 and 3 only
2. The general superintendence, direction and
control of NDRF is vested in the National 177. With reference to United Nations Environment
Disaster Management Authority (NDMA). Programme Finance Initiative, which of the
3. NDRF battalions are trained and equipped statements given below is/are correct?
for response during chemical, biological, 1. The initiative aims to mobilize private sector
radiological and nuclear (CBRN) emergencies. finance for sustainable development.
2. It was created by the provisions of the Paris
Which of the above statements given above is/are Agreement of 2015.
correct?
Select the correct answer using the code given below:
(a) 1 and 2 only
(b) 2 and 3 only (a) 1 only
(c) 1 and 3 only (b) 2 only
(c) Both 1 and 2
(d) 1,2 and 3
(d) Neither 1 nor 2
7.5. International Environmental 178. Consider the following statements:
Organisations, Conventions And 1. United Nations General Assembly (UNGA)
Initiatives proclaimed 2021–2030 as the Decade on
Ecosystem Restoration.
174. With respect to the South Asia Wildlife 2. It will be implemented by United Nations
Enforcement Network (SAWEN), which of the Environment Programme (UNEP) and Food
following statements is/are correct? and Agriculture Organization (FAO).

Workbook 366
.
ENVIRONMENT

Which of the statements given above is/are correct? (a) 1, 3 and 4 only
(a) 1 only (b) 2, 3 and 4 only
(b) 2 only (c) 3 and 4 only
(c) Both 1 and 2 (d) 1, 2, 3 and 4
(d) Neither 1 nor 2 183. Consider the following statements regarding the
179. With respect to International Solar Alliance, Powering Past Coal Alliance (PPCA)?
which of the following statements is incorrect? 1. It encourages all members to commit to phasing
(a) It was launched at CoP 21 of United Nations out coal from the world by 2050.
Framework Convention on Climate Change. 2. India is a founding-member of the Alliance.
(b) Membership is limited only to countries lying Which of the statements given above is/are correct?
fully or partially in between Tropic of Cancer (a) 1 only
and Tropic of Capricorn. (b) 2 only
(c) Headquarter of the alliance is in Gurugram. (c) Both 1 and 2
(d) Alliance aims for 1000GW installed solar (d) Neither 1 nor 2
capacity globally by 2030.
184. Which of the following is/are the main purpose(s)
180. How is the Commission for Air Quality behind the launch of the ‘Extinction Rebellion’, a
Management (CAQM) different from the Central decentralised, international and politically non-
Pollution Control Board (CPCB)? partisan movement?
1. The CAQM has been established by an Act of 1. To prevent the international trade of endangered
Parliament whereas the CPCB has been created species of wild fauna and flora.
by an executive order of the Government. 2. To persuade the governments to act justly on the
2. The CAQM aims to tackle air pollution in the climate emergency.
entire country whereas the CPCB addresses the 3. To preserve and protect the rich diversity of
quality of air in the National Capital Region and world’s cultural expressions.
adjoining areas only.
Select the correct answer using the code given below:
Select the correct answer using the code given below: (a) 1 and 3 only
(a) 1 only (b) 2 only
(b) 2 only (c) 2 and 3 only
(c) Both 1 and 2 (d) 1 only
(d) Neither 1 nor 2
185. Which of the following deals with plastic pollution
181. With reference to the Blue Carbon Initiative, in Ocean?
consider the following statements: 1. The TARA Ocean foundation
1. It aims to mitigate climate change through 2. Glowing Glowing Gone campaign
the restoration and sustainable use of coastal
Select the correct answer using the code given below:
ecosystems.
(a) 1 only
2. The Initiative is launched by Ministry of (b) 2 only
Environment, Forests and Climate Change. (c) Both of the above
Which of the statements given above is/are correct? (d) None of the above
(a) 1 only
186. Blue Dot Network, sometimes mentioned in the
(b) 2 only
news, is related to
(c) Both 1 and 2
(d) Neither 1 nor 2 (a) Deep sea mining for undiscovered minerals and
stones
182. With respect to the Global Methane Assessment (b) Exploration of polymetallic nodules in Indian
Report released recently, consider the following Ocean region
statements: (c) Fishing and aquaculture industry
1. It has been released by the World Meteorological (d) Global Infrastructure projects
Organization. 187. Consider the following statements regarding
2. Methane concentration in the atmosphere have United Nations Framework Convention on
dropped drastically in 2020 over the past decade. Climate Change (UNFCCC):
3. Agriculture sector is the largest emitter of the 1. It was adopted at the “Rio Earth Summit” in
methane. 1992.
4. Cutting methane emissions can rapidly reduce 2. Its ultimate objective is to eliminate greenhouse
the rate of global warming. gases from the atmosphere.
Which of the statements given above are correct? 3. It is legally binding.

367 Workbook
.
ENVIRONMENT

Which of the statements given above is/are correct? 1. The initiative will be implemented in all South
(a) 1 Only Asian countries.
(b) 2 Only 2. The programme intends to promote widescale
(c) 3 Only adoption of electric vehicles in India.
(d) None of the above.
Which of the statements given above is/are correct?
188. With reference to the UN Framework Convention (a) 1 only
on Climate Change (UNFCCC), consider the (b) 2 only
following statements: (c) Both 1 and 2
1. The UNFCCC was adopted at the United Nations (d) Neither 1 nor 2
Conference on the Environment 1972.
2. Under the Convention, Annex II grouping 192. With respect to India’s Intended Nationally
include the countries with economies in Determined Contribution (INDC) targets, which
transition and OECD member countries. of the following statements is correct?
3. The UNFCCC Secretariat maintains the registry (a) It was launched as part of UNDP’s Sustainable
for Nationally Determined Contributions Development Goals.
(NDC) established under the Paris Agreement. (b) India has pledged to improve emission intensity
Which of the statements given above is/are correct? of its GDP by 33 to 35 percent by 2030, below
2015 levels.
(a) 1 only
(b) 1 and 2 only (c) It has pledged to increase non-fossil fuels-based
(c) 3 only electricity capacity to 50 percent by 2030.
(d) 2 and 3 only (d) It has pledged to create an additional carbon
sink of 2.5 to 3 billion tonnes of CO2 equivalent
189. With reference to landmark developments that through additional forest and tree cover by 2030.
have taken place at UNFCCC Conferences,
consider the following statements: 193. Which of the following is/are correct with
1. Green Climate Fund was an outcome of reference to the Climate Technology Centre and
Conference of Parties (COP)-16 held at Cancun. Network (CTCN)?
2. The Paris Agreement requires each Party to 1. It is the operational arm of the UNFCCC
prepare, communicate and maintain successive Technology Mechanism, hosted by the UN
Nationally Determined Contributions (NDCs). Environment Programme.
3. Powering Past Coal Alliance, launched at COP- 2. It promotes transfer of technologies for low
23, is led by US and Canada. carbon and climate resilient development.
Which of the statements given above are correct? Select the correct answer using the code given below:
(a) 1 and 2 only (a) 1 Only
(b) 2 and 3 only (b) 2 Only
(c) 1 and 3 only (c) Both 1 and 2
(d) 1, 2 and 3 (d) Neither 1 nor 2
190. With reference to the Paris agreement, which of 194. Consider the following statements regarding the
the statements given below are correct? Climate Technology Centre and Network:
1. It is a non-binding international treaty on
1. It provides technical assistance to developing
climate change.
countries for the transfer of climate technologies.
2. It invites countries to submit long-term low
greenhouse gas emission development strategies. 2. It is hosted by the UN Environment Programme
in collaboration with the United Nations
3. Under enhanced transparency framework,
Industrial Development Organization.
countries will report transparently on action
taken. Which of the statements given above is/are correct?
4. The nationally determined contributions (a) 1 only
(NDCs) are mandatory for the countries. (b) 2 only
Select the correct answer using the code given below: (c) Both 1 and 2
(a) 1 and 2 only (d) Neither 1 nor 2
(b) 1, 2 and 3 only 195. the following statements regarding the Climate
(c) 1, 3 and 4 only Neutral Now Initiative:
(d) 2, 3 and 4 only
1. It was launched to promote the voluntary use of
191. Consider the following statements regarding the carbon market mechanisms recognized under
Nationally Determined Contributions (NDC)– the United Nations Framework Convention on
Transport Initiative for Asia (TIA): Climate Change.

Workbook 368
.
ENVIRONMENT

2. It is an intergovernmental initiative to achieve a 200. Consider the following statements regarding the
climate neutral world by 2030. UN-REDD programme:
Which of the statements given above is/are incorrect? 1. The programme assists countries to develop
(a) 1 only capacities to reduce forest carbon emissions.
(b) 2 only 2. The programme offers developing countries
(c) Both 1 and 2 results-based payments for results-based actions.
(d) Neither 1 nor 2 3. The UN-REDD programme is a collaboration
196. Race To Zero, sometimes seen in the news, is a between Food and Agriculture Organisation,
global campaign related to: United Nations Development Programme and
(a) Achieving net zero carbon emissions by 2050. United Nations Environment Programme.
(b) Reducing global hunger by half in next ten years. Which of the statements given above is/are correct?
(c) Reducing road accident deaths by 90 per cent till (a) 1 and 3 only
2050.
(b) 1 and 2 only
(d) Resolving global refugee crisis by involving all
stakeholders. (c) 2 and 3 only
(d) 1, 2 and 3
197. With reference to the Montreal Protocol, which of
the following statements is/are incorrect? 201. The Global Environment Facility (GEF) serves as
1. It aims to keep earth’s temperature rise to below a formal financial mechanism for which of the
1.5 degree Celsius. following conventions/Protocols?
2. It aims to reduce the production and 1. Convention on Biological Diversity
consumption of ozone depleting substances. 2. Minamata Convention on Mercury
3. It includes legally binding reduction goals for the 3. Stockholm Convention on Persistent Organic
use of hydrofluorocarbons. Pollutants
4. India is a member to the Montreal Protocol. 4. United Nations Convention to Combat
Select the correct answer using the code given below: Desertification
(a) 1 only Select the correct answer using the code given below.
(b) 3 only (a) 2 and 3 only
(c) 1 and 4 only (b) 1 and 4 only
(d) 2 and 4 only (c) 1, 2 and 4 only
198. The agreement refers to the Hydrofluorocarbon (d) 1, 2, 3 and 4
(HFC) Amendment to the Montreal Protocol.
It aims to reduce the emissions of powerful 202. The Poznan strategic program, sometimes seen in
greenhouse gases, hydrofluorocarbons (HFCs), in the news, is:
a move that could prevent up to 0.5 degrees Celsius (a) A defence coalition of countries around South
of global warming by the end of this century. It is a China Sea.
legally binding agreement, which came into force (b) A program for climate technology development
from January 1, 2019. The agreement is: and transfer activities.
(a) Vienna Convention (c) An outer space initiative by European Space
(b) Cancun Agreement Agency.
(c) Marrakesh Partnership (d) A strategic partnership for free navigation in
(d) Kigali Agreement Indian Ocean.
199. With reference to REDD+ (Reducing Emissions 203. With reference to Clean Technology Fund (CTF),
from Deforestation and Forest Degradation-Plus), consider the following statements:
consider the following statements:
1. World Bank is the Trustee of Clean Technology
1. It is a climate change mitigation mechanism Fund (CTF).
developed by the United Nations Environment
Programme. 2. The CTF provides large-scale financial resources
to invest in clean technology projects in
2. It aims to enhance forest carbon stocks along
with sustainable management of forests. developing countries.
3. The objective of this mechanism is to create a 3. Rewa Solar Plant is the first project in India to
financial value for the carbon stored in forests. get funding from CTF.
Which of the statements given above are correct? Which of the statements given above is/are correct?
(a) 1 and 2 Only (a) 1 and 2 only
(b) 1 and 3 Only (b) 2 and 3 only
(c) 2 and 3 Only (c) 3 only
(d) 1, 2 and 3 (d) 1, 2 and 3

369 Workbook
.
ENVIRONMENT

204. With reference to Climate Smart Agriculture 208. Consider the following statements:
(CSA), which of the following statements is/are 1. Climate and Clean Air Coalition (CCAC),
correct? an initiative of G20 group of countries, is a
1. It focuses solely on adaptation to the changing mechanism to reduce Long Lived Climate
climate. Pollutants.
2. It aims at increased productivity, enhanced 2. The CCAC focuses on reducing the
resilience and reduced emissions. concentration of methane, black carbon and
3. It has decentralization at its heart with attention hydrofluorocarbons from the atmosphere.
to country/locality specific landscape approach. Which of the statements given above is/are correct?
Select the correct answer using the code given below: (a) 1 only
(b) 2 only
(a) Only 1 and 2
(c) Both 1 and 2
(b) Only 2 and 3
(d) Neither 1 nor 2
(c) Only 1 and 3
(d) 1, 2 and 3 209. With reference to ‘Agenda 21’, sometimes seen in
the news, consider the following statements:
205. With reference to Intergovernmental Panel on
1. It is a global action plan for sustainable
Climate Change (IPCC), which of the following
development.
statements is/are correct?
2. It originated in the World Summit on Sustainable
1. It is a non-UN body for assessing the science
Development held in Johannesburg in 2002.
related to climate change.
Which of the statements given above is/are correct?
2. It does not conduct any research nor does it
monitor climate-related data or parameters. (a) 1 only
(b) 2 only
Select the correct answer using the code given below: (c) Both 1 and 2
(a) Only 1 (d) Neither 1 nor 2
(b) Only 2
(c) Both 1 and 2 210. Consider the following statements regarding the
(d) Neither 1 nor 2 Convention on Biological Diversity (CBD):
1. It was the result of the United Nations Conference
206. Q.175) With reference to the Intergovernmental on the Environment, Stockholm in 1972.
Panel on Climate Change (IPCC), which of the 2. The Cartagena Protocol to the CBD deals
statements given below is incorrect? with the safe use of living modified organisms
(a) The IPCC is an organization of governments that resulting from modern biotechnology.
are members of the United Nations or WMO. 3. The Nagoya Protocol to the CBD deals with fair
(b) It was created by World Meteorological and equitable sharing of benefits arising from
Organization and United Nations Environment the utilization of genetic resources.
Program. Which of the statements given above is/are correct?
(c) The IPCC does not conduct its own research.
(a) 1 only
(d) The recommendations made by IPCC
(b) 2 only
Assessment Reports are binding on the countries.
(c) 2 and 3 only
207. In the context of National Green-House Gas (d) 1 and 3 only
Inventories Programme (NGGIP), consider the 211. Consider the following statements with reference
following statements: to the Convention on International Trade in
1. The Intergovernmental Panel on Climate Endangered Species of Wild Fauna and Flora
Change (IPCC) Task Force develops and refines (CITES):
a methodology for the calculation and reporting 1. Trade in species listed in Appendix I of CITES is
of national Green-House Gases (GHG) completely prohibited.
emissions and removals. 2. Parties have to adopt their own domestic
2. India is yet to establish its GHG inventories legislations to ensure that CITES is implemented
program. at the national level.
Which of the statements given above is/are incorrect? Which of the statements given above is/are correct?
(a) Only 1 (a) 1 only
(b) Only 2 (b) 2 only
(c) Both 1 and 2 (c) Both 1 and 2
(d) Neither 1 nor 2 (d) Neither 1 nor 2

Workbook 370
.
ENVIRONMENT

212. Which of the following statements is/are correct 216. Match the following pairs of Ramsar Sites in India
regarding the Coalition Against Wildlife and the States/Union Territory in which they are
Trafficking (CAWT)? located:
1. Membership of CAWT is open to governments,
NGOs and corporations. Ramsar site State
2. CAWT was established by United Kingdom. 1. Tso Moriri Lake A. Tripura
3. The CAWT organisation is not directly involved 2. Rudrasagar Lake B. Himachal Pradesh
in any enforcement activities.
3. Deepor Beel C. Ladakh
Select the correct answer using the code given below:
(a) 1 only 4. Pong Dam Lake D. Assam
(b) 1 and 2 only Select the correct answer using the code given below:
(c) 2 and 3 only (a) a) 1-C, 2-A, 3-D, 4-B
(d) 1 and 3 only (b) b) 1-B, 2-A, 3-D, 4-C
(c) c) 1-D, 2-A, 3-C, 4-B
213. Consider the following statements with respect to
TRAFFIC: (d) d) 1-C, 2-B, 3-A, 4-D
1. The program works to stop the trade in wild 217. With reference to Bonn Convention, which of the
animals and plants for the conservation of statements given below is/are incorrect?
wildlife. 1. It is an intergovernmental treaty, concluded
2. This is a joint conservation programme of IUCN under the aegis of the International Union for
and WWF. Conservation of Nature.
Which of the statements given above is/are correct? 2. It is the only global convention specializing in
(a) 1 only the conservation of migratory species.
(b) 2 only Select the correct answer using the code given below:
(c) Both 1 and 2 (a) 1 only
(d) Neither 1 nor 2 (b) 2 only
214. With respect to the Ramsar Convention on (c) Both 1 and 2
wetlands, which of the following statement(s) is/ (d) Neither 1 nor 2
are correct?
218. Consider the following statements:
1. As per the convention only natural wetlands are
1. Bonn Convention deals with the conservation
selected as Ramsar sites.
and sustainable use of migratory animals and
2. ‘Montreux Record’ highlights the sites needing their habitats.
priority conservation attention.
2. 13th Conference of Parties (COP) of Bonn
3. It issued the first-ever Global Wetland Outlook
Convention was hosted by India in 2020.
Report in 2018.
3. The Indian sub-continent is a part of the Central
Select the correct answer using the code given below: Asian Flyway (CAF).
(a) 2 only
(b) 3 only Which of the statements given above is/are correct?
(c) 1 and 2 only (a) 1 only
(d) 2 and 3 only (b) 1 and 2 only
(c) 2 and 3 only
215. Consider the following: (d) 1, 2 and 3
1. Montreux Record is a register of wetland sites
where changes in ecological character have 219. Consider the following statements:
occurred due to human interference. 1. Basel Convention is an international treaty
2. Montreux Record is maintained as part of that was designed to reduce the movements of
Ramsar List. hazardous waste between nations.
3. Chilika Lake and Keoladeo National Park are 2. Rotterdam Convention is a multilateral treaty
two wetland sites in India that are currently to promote shared responsibilities in relation to
included in the Montreux record. importation of hazardous chemicals.
Which of the statements given above is/are correct? Which of the statements given above is/ are correct?
(a) 1 only (a) 1 only
(b) 1 and 2 only (b) 2 only
(c) 2 and 3 only (c) Both 1 and 2
(d) 1, 2 and 3 (d) Neither 1 nor 2

371 Workbook
.
ENVIRONMENT

220. Consider the following pairs: 2. Forest Landscape Restoration is the underlying
approach of the challenge.
Convention Objective
Which of the statements given above is/are correct?
1. Stockholm reduction in Persistent (a) 1 only
Convention Organic Pollutants (b) 2 only
2. Basel Convention Prior Informed (c) Both 1 and 2
Consent (PIC) (d) Neither 1 nor 2
procedure covering 224. With reference to the Bonn Challenge, consider
pesticides the following statements:
3. Rotterdam control of 1. Bonn Challenge is launched by the United
Convention Transboundary Nations Environment Programme.
Movements of 2. It aims to restore degraded and deforested lands.
Hazardous Wastes
3. India is not a part of the Bonn Challenge.
4. Gothenburg to Abate Acidification, Which of the statements given above is/are correct?
Protocol Eutrophication and
(a) 1 and 2 only
Ground-level Ozone
(b) 1 and 3 only
Which of the pairs given above are correctly (c) 2 only
matched? (d) 1, 2 and 3
(a) 1 and 2 only
(b) 1 and 4 only 225. The Global Initiative on reducing Land
(c) 2 and 3 only Degradation and Enhancing Conservation of
(d) 3 and 4 only Terrestrial Habitats has been launched by-
(a) G20
221. With reference to United Nations Convention to (b) World Economic Forum (WEF).
Combat Desertification (UNCCD), consider the (c) United Nations Environment Programme
following statements: (UNEP)
1. It is the sole legally binding international (d) United Nations Convention to Combat
agreement for sustainable land practices. Desertification (UNCCD)
2. Its member countries have agreed to make the
Sustainable Development Goal (SDG) target of 226. Consider the following statements with reference
achieving land degradation neutrality (LDN) by to TX2 seen in news:
2025, a national target for action. 1. TX2 means to double population of wild tigers
Which of the statements given above is/are correct? by 2030.
(a) 1 only 2. At St. Petersburg Summit (2010) all the tiger
(b) 2 only range countries committed themselves to the
(c) Both 1 and 2 goal of doubling tiger population.
(d) Neither 1 nor 2 3. Recently Pilibhit Tiger Reserve has won the
inaugural TX2 Award.
222. Consider the following statements about
the United Nations Convention to Combat Which of the statements given above is /are correct?
Desertification: (a) 2 and 3 only
1. It is the only legally binding international (b) 1 and 2 only
agreement linking environment to sustainable (c) 1 and 3 only
land management. (d) 1, 2 and 3
2. Increase in the area of deserts is included in its 227. Consider the following statements with reference
criteria of desertification. to supplementary agreements on Convention on
3. India has not ratified the convention. Biological Diversity:
Which of the statements given above is/are correct? 1. Nagoya Protocol governs the movements of
(a) 1 only Living Modified Organisms.
(b) 1 and 3 only 2. Cartagena Protocol on Biosafety governs Access
(c) 2 and 3 only to Genetic Resources and the Fair and Equitable
(d) 1, 2 and 3 Sharing of Benefits.
223. With reference to the Bonn Challenge, consider 3. The Nagoya-Kuala Lumpur Supplementary
the following statements: Protocol deals with liability and redress in the
1. It is a global effort to bring 50 million hectares event of damage resulting from living modified
of deforested and degraded land into restoration organisms (LMOs).
by 2020. Which of the statements given above is/are incorrect?

Workbook 372
.
ENVIRONMENT

(a) 1 and 2 only (c) Food and Agriculture Organization


(b) 3 only (d) Climate and Clean Air Coalition
(c) 1, 2 and 3
232. Which of the following statements regarding
(d) 1 and 3 only
Operation Thunder, seen in news recently, is/ are
228. Which of the following statements regarding the incorrect?
key findings of the Living Planet Report, 2020 is/ 1. It was organised by the TRAFFIC and WWF.
are correct? 2. Its objective is to stop smuggling of wildlife and
1. Southeast Asia is the worst hit region with forest products.
maximum biodiversity loss. 3. India had recently withdrawn from the
2. More than 50% of the vertebrate species have operation.
gone extinct in the past 5 decades. Select the correct option using the code given below:
3. The report is published by UNEP (United (a) 1 and 2 only
Nations Environment Programme). (b) 2 and 3 only
Select the correct answer using the code given below: (c) 1 and 3 only
(a) 1 only (d) 1, 2 and 3
(b) 2 only
233. The ‘Enhanced Transparency Framework’,
(c) 3 only
sometimes seen in news, is related to which one of
(d) 1, 2 and 3
the following agreement/plans?
229. Consider the following statements with respect (a) Atlantic Charter
to the Globally Important Agricultural Heritage (b) Joint Comprehensive Plan of Action
(GIAHS) systems: (c) Paris Agreement
1. It is an initiative of the UNESCO. (d) Regional Comprehensive Economic Partnership
2. Below Sea Level Farming System in Kuttanad, 234. The “Fit for 55” plan calls cutting emissions of
Kerala has been designated as a GIAHS site. greenhouse gases by at least 55% by 2030. The plan
Which of the statements given above is/are correct? is launched by the:
(a) 1 only (a) World Economic Forum
(b) 2 only (b) European Union
(c) Both 1 and 2 (c) Organization for Economic Cooperation and
(d) Neither 1 nor 2 Development
(d) United Nations Commission on Sustainable
230. Consider the following pairs:
Development
CITES Appendix Protection 235. Consider the following statements regarding
1. Appendix I Trade is completely Asian Waterbird Census:
prohibited 1. It is conducted annually by the World Wide
2. Appendix II Species which are Fund for Nature.
not facing imminent 2. It aims to monitor on an annual basis the status
extinction but need and condition of wetlands.
monitoring so that Which of the statements given above is/are correct?
trade doesn’t become a
(a) 1 only
threat.
(b) 2 only
3. Appendix III Species that are (c) Both 1 and 2
protected in at least one (d) Neither 1 nor 2
country.
Which of the pairs given above is/are correctly 236. Which one of the following statements correctly
matched? describes the ‘’Friends of the Earth International’’?
(a) 1 only (a) It is an international network to create
(b) 2 and 3 only environmentally sustainable and socially just
(c) 1 and 3 only societies.
(d) 1, 2 and 3 (b) It is the world’s largest wildlife conservation
organization.
231. Global Methane Assessment is released by which (c) It is an organization working in conflict ridden
of the following environmental organisations? regions of the world.
(a) World Meteorological Organization (d) It is the global not-for-profit organization
(b) Global Alliance on Health and Pollution dedicated to healthcare sector improvement.

373 Workbook
.
ENVIRONMENT

237. Which of the following funds are administered by 3. It finances biodiversity conservation programmes
the Global Environment Facility? in developing and emerging countries.
1. Green Climate Fund Select the correct answers from the codes given
2. BioCarbon Fund below:
3. Special Climate Change Fund (a) 1 and 2 only
4. Least Developed Countries Fund (b) 2 only
(c) 1 and 3 only
Select the correct answer using the code given below: (d) 2 and 3 only
(a) 1, 2 and 3 only
(b) 1, 3 and 4 only 242. Consider the following statements with reference
(c) 3 and 4 only to the Coalition for Sustainable Climate Action:
(d) 1, 2, 3 and 4 1. It is an initiative launched by the International
Solar Alliance.
238. The Ministry of Finance, Government of India has
2. It will bring together global public and private
launched the ‘’Sustainable Finance Collaborative’’
corporates to fund renewable energy generation.
programme in partnership with the
(a) World Bank Which of the statements given above is/are correct?
(b) United Nations Development Programme (a) 1 only
(c) International Solar Alliance (b) 2 only
(d) Asian Development Bank (c) Both 1 and 2
(d) Neither 1 nor 2
239. Consider the following statements:
1. The Minamata Convention aims to control the 243. The Global Climate Risk Index is annually
anthropogenic release of mercury. published by the
2. The Rotterdam Convention aims to promote (a) United Nations Environment Programme
shared responsibilities in relation to the imports (b) Germanwatch
of hazardous chemicals. (c) Greenpeace
3. Both the Minamata Convention and Rotterdam (d) World Meteorological Organisation
Convention are funded by the World Bank. 244. With reference to the Warsaw International
Which of the statements given above are correct? Mechanism, sometimes seen in news, consider the
(a) 1 and 2 only following statements
(b) 1 and 3 only 1. It addresses loss and damage associated with the
(c) 2 and 3 only adverse effects of climate change.
(d) 1, 2 and 3 2. It was established by the Intergovernmental
240. Which of the following are the targets set by Panel on Climate Change (IPCC) in 2015 to help
India with respect to its Nationally Determined in the implementation of the Paris Agreement.
Contributions (NDCs) under the Paris Which of the statements given above is/are correct?
Agreement? (a) 1 only
1. Reducing the emissions intensity of its GDP by (b) 2 only
33 to 35 per cent by 2030 from 2005 level. (c) Both 1 and 2
2. 50 per cent electricity generation from non- (d) Neither 1 nor 2
fossil fuel-based energy resources by 2050. 245. With reference to Climate change, which one of
3. Creation of additional carbon sink of 2.5 to 3 the following organizations has recently published
billion Tonnes of CO2 equivalent by 2030. the Sixth Assessment Report?
Select the correct answer using the code given below: (a) Intergovernmental Panel on Climate Change
(a) 1 only (IPCC)
(b) 1 and 3 only (b) United Nations Environment Programme
(c) 2 and 3 only (c) World Meteorological Organization
(d) 1, 2 and 3 (d) UNFCCC Council
241. Which one of the following statements is/ 246. The London Convention and MARPOL, frequently
are correct about the ‘International Climate seen in news, are related to
Initiative’? (a) They aim to reduce and mitigate the marine
1. It is an initiative launched by the Government pollution caused due to ships and other sources.
of India. (b) They seek to stop the illegal international trade
2. It operates within the framework of the United of threatened wildlife animals around the world.
Nations Framework Convention on Climate (c) They aim to reduce and stop financial market
Change (UNFCCC). frauds and money laundering related activities.

Workbook 374
.
ENVIRONMENT

(d) They promote arbitration mechanism between Which of the statements given above is/are correct?
governments and private entities. (a) 1 only
247. Consider the following statements regarding the (b) 2 only
Kigali Agreement: (c) Both 1 and 2
1. It is a non-legally binding voluntary initiative. (d) Neither 1 nor 2
2. It aims to phase out Hydrofluorocarbons which 252. Consider the following statements with reference
are ozone depleting substances. to Climate Adaptation and Resilience (CARE) for
3. The Kigali Agreement is an amendment to the South Asia project:
Montreal Protocol. 1. The project focuses on reducing socio-economic
4. Recently, India ratified the Kigali Amendment. impact of climate-induced natural disasters.
Which of the statements given above is/are correct? 2. An effective implementation of the project will
(a) 1 and 2 only help reduce the impact of future pandemics like
(b) 2 and 3 only COVID-19.
(c) 1 and 2 only Which of the statements given above is/are correct?
(d) 3 and 4 only (a) 1 only
248. Talanoa Dialogue is often seen in news. It is (b) 2 only
related to (c) Both 1 and 2
(a) World Trade Organisation dialogue (d) Neither 1 nor 2
(b) Afghanistan conflict resolution process 253. With reference to the Loss and Damage
(c) UN climate talks
Mechanism, Consider the following statements:
(d) A dialogue to create Free Trade Agreement in
South America. 1. The Loss and Damage Mechanism was first
established at the Paris Climate Agreement in
249. With reference to the Coalition for Disaster 2015.
Resilient Infrastructure (CDRI), consider the 2. It will help in climatic adaptation process through
following statements: finance, technology and capacity-building.
1. It is an inter-governmental alliance consisting of 3. It redresses the losses due to adverse climatic
all member nations of the United Nations.
events in all vulnerable countries.
2. India is the permanent co-chair of the Governing
Council. Which of the statements given above is/are correct?
3. Each member country must contribute funds to (a) 1 only
a CDRI pool through a quota system. (b) 2 only
(c) 2 and 3 only
Which of the statements given above is/are correct?
(d) 1, 2 and 3
(a) 1 only
(b) 2 only 254. Which of the following sub – indices are part of
(c) 1 and 2 only the Food and Agriculture Organization Food
(d) 1, 2 and 3 Price Index (FFPI)?
250. Which of the following statements is correct with 1. The FAO Cereal Price Index
reference to the Beach Environment & Aesthetics 2. The FAO Pulses Price Index
Management Services (BEAMS) programme? 3. The FAO Vegetable Oil Price Index
(a) It is a joint initiative by the Ministry of 4. The FAO Dairy Price Index
Environment, Forest and Climate Change and 5. The FAO Meat Price Index
UN Environment (UNEP).
(b) It has been launched on the lines of the Blue Flag Select the correct answer using the codes given
beach certification programme. below:
(c) The program awards “National Heritage (a) 1, 2, 3 and 4 only
beaches” as part of the project. (b) 1, 3, 4 and 5 only
(d) The program involves training of student (c) 2, 3, 4 and 5 only
volunteers as “Bal Mitra” for creating awareness. (d) 1, 2, 3, 4 and 5
251. Consider the following statements: 255. The ‘Green Strategic Partnership’ was recently
1. Coalition for Sustainable Climate Action (CSCA) seen in news in the context of India’s relations
is an initiative of the United Nations Framework with:
Convention on Climate Change (UNFCCC). (a) Denmark
2. CSCA aims to raise awareness of short-lived (b) Ireland
climate pollutant impacts and mitigation (c) Japan
strategies. (d) Australia

375 Workbook
.
ENVIRONMENT

256. Consider the following statements: Select the correct answer using the code given below:
1. TRAFFIC is a joint programme of World Wide (a) 1 only
Fund for Nature and IUCN. (b) 2 only
2. Its aims to ensure that international trade of (c) Both 1 and 2
wild animals and plants does not threaten their (d) Neither 1 nor 2
survival.
3. ‘Not All Animals Migrate by Choice’ is a joint 261. Consider the following pairs:
initiative of TRAFFIC and Wildlife Crime
Control Bureau (WCCB) of India. Programs Significance
Select the correct answer using the code given below: 1. Climate and Clean Presents awards to best
(a) 1 and 2 only Air Coalition examples of climate
(b) 2 and 3 only solutions in world
(c) 1 and 3 only 2. Climate Provides direct
(d) 1, 2 and 3 Technology Centre funding to countries
257. With reference to the Blue Flag Programme, and Network for technology
consider the following statements: upgradation
1. The programme is operated by the Ministry of 3. Green Climate Operational Arm of
Environment, Forest and Climate Change in Fund UNFCCC Technology
India. Mechanism
2. Its main aim is to protect marine flora and fauna Which of the above-mentioned pairs is/are correct?
in the ocean waters. (a) 1 and 2 only
Which of the statements given above is/are correct? (b) 3 only
(a) 1 only (c) 1 and 3 only
(b) 2 only (d) None of the above
(c) Both 1 and 2
(d) Neither 1 nor 2 262. Which of the following statements correctly
describes the difference between Kyoto Protocol
258. Which of the following statements is/are incorrect and Paris Agreement?
regarding the Climate Smart Cities Assessment
(a) While the Kyoto protocol exempted
Framework (CSCAF) 2.0?
industrialized countries, Paris agreement
1. It assesses the performance of the major cities
required them to reduce greenhouse gases.
in using innovative solutions to combat climate
(b) While Kyoto protocol focused on 6 major
change.
greenhouse gases, Paris agreement focused on
2. It is implemented by the Ministry of Housing
all major greenhouse gases.
and Urban Affairs.
(c) While Kyoto protocol aimed to reduce the
3. It aims to inculcate a climate-sensitive approach
temperature by 2°C, Paris agreement aimed to
to urban planning and development in India.
limit the increase to 2°C above pre-industrial
Select the correct answer using the code given below: levels.
(a) 1 only (d) While the Kyoto protocol was legally non-
(b) 1 and 2 only binding, Paris agreement is a legally binding
(c) 2 only commitment.
(d) 2 and 3 only
263. With reference to Funds related to climate and
259. Recently, the Global Initiative on Reducing Land
environment, consider the following statements:
Degradation was launched by
(a) United Nations Environment Programme 1. Green Climate Fund was established with a
(b) G20 Forum mandatory requirement to invest 50% of its
(c) International Union for Conservation of Nature resources tomitigationand 50% toadaptation.
(d) United Nations Convention to Combat 2. Global Environment Facility is formally
Desertification established as the permanent financing arm by
UNFCCC.
260. With reference to the United in Science Report
2020, which of the statements given below is/are Which of the above-mentioned statements is/are
correct? correct?
1. It was compiled by the World Meteorological (a) 1 only
Organization (WMO). (b) 2 only
2. It is primarily concerned with new technologies (c) Both 1 and 2
in weather forecasting. (d) Neither 1 nor 2

Workbook 376
.
ENVIRONMENT

264. Consider the following statements about the Which of the above statements are correct?
United Nations Office for Disaster Risk Reduction (a) 1, 2 and 3 only
(UNDRR):
(b) 2, 3 and 4 only
1. UNDRR oversees the implementation of the
Sendai Framework for Disaster Risk Reduction (c) 1, 3 and 4 only
2015-2030. (d) 1, 2 and 4 only
2. The Gender Dimensions of Disaster Risk and
Resilience (2021) report is an annual publication 268. With reference to Reducing Emissions from
of UNDRR. Deforestation and Forest Degradation (REDD)
3. The Global Facility for Disaster Reduction and and REDD+, consider the following statements:
Recovery (GFDRR) is a global partnership 1. Reducing Emissions from Deforestation and
managed by UNDRR.
Forest Degradation (REDD) is a global initiative
Which of the above statements is/are correct?
negotiated under United Nations Convention to
(a) 1 only
(b) 1 and 2 only Combat Desertification (UNCCD).
(c) 3 only 2. REDD+ is a voluntary climate change mitigation
(d) 2 and 3 only approach that aims to incentivize developing
265. With reference to the ‘ecosystem approach’ as countries to reduce emissions from deforestation.
per Convention on Biological Diversity (CBD),
consider the following statements: Which of the statements given above is/are correct?
1. Ecosystem approach was recently adopted for (a) 1 only
the first-time in the Conference of the Parties (b) 2 only
(COP) 2021 held at Kunming, China. (c) Both 1 and 2
2. The ecosystem approach excludes human beings
for the integrated study and management of (d) Neither 1 nor 2
land, water and living resources. 269. Which of the following statement is correct with
Which of the statements(s) given above is/are reference to Warsaw International Mechanism for
correct?
Loss and Damage?
(a) 1 only
(b) 2 only (a) It is a convention for the unification of different
(c) Both 1 and 2 rules relating to International Carriage by air.
(d) Neither 1 nor 2 (b) It is a convention associated with Climate
266. With reference to the natural and artificial Change Impacts.
environment, consider the following statements: (c) It is a mutual defence treaty between Communist
1. Both natural ecosystem and artificial ecosystem
states.
requires human intervention to sustain.
2. Natural ecosystems generally contain more biotic (d) It is an InternationalConventionon the
and abiotic factors than artificial ecosystems. Protection of the Rights of All Migrant Workers.
Which of the statement(s) given above is/are correct? 270. Consider the following pairs:
(a) 1 only
(b) 2 only Kyoto Protocol Functions
(c) Both 1 and 2 Mechanisms
(d) Neither 1 nor 2
267. Consider the following statements about the 1. Emissions Trading advanced country A
‘Sendai Framework for Disaster Risk Reduction’: can invest in a project
1. It is a binding agreement which recognizes that to reduce greenhouse
the State has the primary role to reduce disaster gas emissions in
risk. any other advanced
country B.
2. It aims for a substantial reduction in disaster
losses resulting from both man-made and 2. Clean Development An advanced country
natural hazards. Mechanism can implement
3. It marks a shift in emphasis from disaster emission reduction
management to disaster risk management. projects in a
4. The goals are defined in terms of outcome-based developing country to
targets instead of focusing on sets of activities earn certified emission
and actions. reduction (CER).

377 Workbook
.
ENVIRONMENT

3. Joint An advanced country (c) Both 1 and 2


Implementation “A” can acquire (d) Neither 1 nor 2
emission units from 274. Consider the following statements:
an advanced country 1. The Johannesburg Declaration was the first
“B” for meeting a major international framework for disaster
part of their emission risk reduction that recognized the interrelation
reduction target. between sustainable development and Disaster
Which of the above pairs is/are correctly matched? Risk Reduction.
(a) 2 only 2. The Sustainable Development Goals (SDGs)
(b) 1 and 3 only under the 2030 Agenda adopted by the UN
(c) 1 only General Assembly recognizes disaster risk
(d) 2 and 3 only reduction.
271. With reference to ‘International Customary Law’, Which of the statement(s) given above is/are correct?
consider the following statements: (a) 1 only
1. It is an international law derived from a general (b) 2 only
practice accepted as law based on customs. (c) Both 1 and 2
2. These laws are not binding legal rules as they are (d) Neither 1 nor 2
not supported through international agreements 275. Consider the following statements:
or treaties.
1. Hyogo framework was adopted in 2005 as a 10-
3. Indian Judicial and legal system has not year plan to make the world safer from natural
recognized the validity of International hazards.
customary law.
2. The Sendai Framework is the successor
Which of the statements given above is/are correct? instrument to the Hyogo Framework .
(a) 1 only 3. UNDP is tasked to support the implementation,
(b) 1 and 3 only follow-up and review of the Sendai Framework
(c) 2 only
(d) 2 and 3 only Which of the statement(s) given above is/are correct?
(a) 1 and 2 only
272. With reference to the Intergovernmental Panel (b) 1 and 3 only
on Climate Change (IPCC) Report, consider the (c) 1, 2 and 3
following statements: (d) 3 only
1. The high uptake of anthropogenic carbon by
oceans results in increased pH level of water. 276. Consider the following statements with reference
2. The range of kelp forests in the Northern to Plastic Waste Management in India:
Hemisphere may expand poleward due to 1. The Plastic Waste Management (Amendment)
climate change. Rules, 2022 prohibit the sale and purchase
3. The global mean Sea Surface Temperature has of surplus extended producer responsibility
on average increased since the beginning of the certificates.
20th century. 2. The Hazardous and Other Wastes (Management
Which of the statements given above is/are correct? and Transboundary Movement) Amendment
(a) 1 and 2 only Rules 2019, regulate the import of plastic waste
(b) 2 and 3 only into the country.
(c) 1 and 3 only 3. The above rules are framed under the provisions
(d) 1, 2 and 3 of the Environment (Protection) Act, 1986.
273. Consider the following statements: Which of the above statements is/are correct?
1. World Conservation Monitoring Centre is a (a) 1 only
non-profit organisation with IUCN as one of the (b) 2 only
founder organisation. (c) 2 and 3 only
(d) 1, 2 and 3
2. United Nations Environment Programme
Finance Initiative is a partnership between 277. The ‘FASTER (Fast and Secured Transmission of
UNEP and the global financial sector to Electronic Records)’ Platform was launched to-
mobilize private sector finance for sustainable (a) accessibility of police records to the central
development. agencies through encrypted and secure platform
Which of the statement(s) given above is/are correct? (b) communicate interim orders, stay orders,
(a) 1 only and bail orders of the Supreme Court to the
(b) 2 only concerned authorities.

Workbook 378
.
ENVIRONMENT

(c) faster redressal of corporate grievances through ranks the districts of the eight States of North
inter-departmental coordination across different East.
ministries.
(d) information regarding financial frauds for Which of the statement(s) given above is/are correct?
effective forensic audit and investigation. (a) 1 only
(b) 2 only
278. With reference to various initiatives of Indian
government on Sustainable Development Goal (c) 1 and 3 only
(SDG), consider the following statements: (d) 1, 2 and 3
1. The SDG India Index tracks goal-wise scores on 279. Which one of the following international
the 16 SDGs for all States and Union Territories.
agencies/organizations has recently released
2. The National Indicator Framework (NIF) of
SDGs was developed to assist the Government Global Wind Report 2022?
in monitoring the progress of SDGs at national (a) Global Wind Energy Council
level. (b) World Economic Forum
3. Recently the North Eastern Region (NER) (c) International Renewable Energy Agency
District SDG Index Report was launched, which (d) World Wind Energy Association

379 Workbook
.
ENVIRONMENT

SOLUTIONS

Also, they cannot be downgraded to the status of a


7.1. Previous Years Questions ‘sanctuary’. Both the Central Government and State
1. Solution (b) governments can declare National Parks.
Exp) Option b is correct 7. Solution (a)
In situ conservation involves the maintenance of genetic Exp) Option a is correct
variation at the location where it is encountered, either in A wetland is a place where the land is covered by water,
the wild or in traditional farming systems. either salt, fresh or somewhere in between. Marshes and
Botanical Garden in not a site for in-situ method of ponds, the edge of a lake or ocean, the delta at the mouth
conservation of flora. of a river, low-lying areas that frequently flood—all of
these are wetlands.
2. Solution (b)
Wetlands in India account for 4.7% of the total
Exp) Option b is correct geographical area of the country. Gujarat has the highest
Red data book is the document established by IUCN for proportion (22.8%) and UT of Chandigarh has nearly
documenting the rare and endangered species of plants, negligible part of the total wetland area in the country.
animals, fungi and also a few local species that exist India has totally 27, 403 wetlands, of which 23,444 are
within a state or country. inland wetlands and 3,959 are coastal wetlands.
3. Solution (d) 8. Solution (d)
Exp) option d is correct Exp) Option d is correct
Carbon credits allow companies to compensate for their
greenhouse gas emissions. The mechanism of Carbon National Park River flowing through
Credits was formulized the Kyoto Protocol, and the the Park
market mechanism was agreed through the subsequent Corbett National Park Ramganga
Marrakesh Accords.
Kaziranga National Park Dhiplu
4. Solution (a) Silent Valley National Park Kunthipuzha
Exp) Option a is correct
9. Solution (c)
The Montreux Record is a register of wetland sites on
the List of Ramsar wetlands of international importance Exp) Option c is correct
where changes in ecological character have occurred, All of the above have a bearing on the biodiversity
are occurring, or are likely to occur as a result of conservation in the country.
technological developments, pollution or other human
interference. 10. Solution (d)
Exp) Option d is correct
5. Solution (b)
Eco-Sensitive Zones are declared under Environment
Exp) Option b is correct (Protection) Act, 1986.
Bandipur Tiger Reserve is situated in Karnataka, it was Eco-tourism activities are not prohibited in most of the
established in 1974. Eco-Sensitive Zones. The activities which are prohibited
Manas Tiger Reserve is situated in Assam, it was or allowed could be different for each Eco-Sensitive Zone.
established in 1973.
11. Solution (a)
Sunderban Tiger Reserve is also one amongst the initial
Exp) Option a is correct
nine Tiger Reserves constituted at the time of inception of
the Project Tiger scheme in the year 1973. Wetlands Conference of rivers
6. Solution (b) Harike Wetlands Confluence of Beas and
Exp) Option b is correct Satluj/ Sutlej
In National Parks, human activity is not allowed. Also, Keoladeo Ghana Confluence of Gambhi
the grazing of livestock and private tenurial rights are not National Park and Banganga
permitted. The species that are mentioned in the Schedule
of the Wildlife Act are also not allowed for hunting or Kolleru Lake Confluence of Krishna
capturing purposes. and Godavari delta

Workbook 380
.
ENVIRONMENT

12. Solution (d) 18. Solution (b)


Exp) Option d is correct Exp) Option b is correct
All of the above international agreements have a bearing Keibul Lamjao is a swamp with floating mass of vegetation
on the biodiversity. created by accrual of organic garbage and biomass with
soil particles that has been thickened into a solid form
13. Solution (c) called phumdis, at the south–eastern side of the Loktak
Exp) Option c is correct Lake, which has been declared a Ramsar site. The swamp
encompasses three hills, namely, Pabot, Toya and Chingjao
Launched in 2007, Earth Hour is organized by the World that provide a refuge for the large mammals during the
Wide Fund for Nature. It is held every year on the last monsoon season. The distinctive nature of the park is that
Saturday of March in which the participants switch off it is “too deep to be marsh, too shallow to be a lake”.
the lights for one hour on a certain day every year to call
attention to climate change. 19. Solution (d)
Exp) Option d is correct
14. Solution (c)
Namdapha National Park is a protected area and a
Exp) Option c is correct biodiversity hotspot located in Arunachal Pradesh. The
BNHS is one of the oldest scientific organizations in India, national park harbours the northern most low land
working for nature conservation since 1883. The primary evergreen rainforests in the world at 27°N latitude. It
goal of BNHS is to spread awareness about nature through is the fourth largest national park in India. The Biomes
science-based research, conservation advocacy, education, recognised are evergreen Forests, Moist deciduous forests,
sub-tropical forests, Temperate Forests and Alpine.
scientific publications, nature tours, and other programs.
It is not a part of Ministry of Environment and Forests. 20. Solution (b)
15. Solution (a) Exp) Option b is correct
Exp) Option a is correct IUCN has observer and consultative status the United
Nations. Thus, clearly its not a ‘member’. It is involved
The Global Environment Facility (GEF) was established in data gathering and analysis, research, field projects,
on the eve of the 1992 Rio Earth Summit to help tackle advocacy, and education. IUCN’s mission is to “influence,
our planet’s most pressing environmental problems. encourage and assist societies throughout the world to
The GEF serves as a “financial mechanism” to five conserve nature and to ensure that any use of natural
conventions: Convention on Biological Diversity (CBD), resources is equitable and ecologically sustainable”.
United Nations Framework Convention on Climate CITES (the Convention on International Trade in
Change (UNFCCC), Stockholm Convention on Persistent Endangered Species of Wild Fauna and Flora) is an
Organic Pollutants (POPs), UN Convention to Combat international agreement between governments. Its aim
Desertification (UNCCD), and Minamata Convention on is to ensure that international trade in specimens of wild
Mercury. animals and plants does not threaten the survival of the
species.
The GEF is not an agency of OECD and does not undertake
States that have agreed to be bound by the Convention
scientific research on environmental issues.
(‘joined’ CITES) are known as Parties. Although CITES
16. Solution (b) is legally binding on the Parties – in other words they
have to implement the Convention – it does not take the
Exp) Option b is correct place of national laws. Rather it provides a framework to
Wetlands International is an independent, global not- be respected by each Party, which has to adopt its own
for-profit organisation dedicated to the conservation and domestic legislation to ensure that CITES is implemented
restoration of wetlands. It is not an intergovernmental at the national level.
organization.
21. Solution (b)
17. Solution (b) Exp) Option b is correct
Exp) Option b is correct The Montreal Protocol on Substances that Deplete the
Animal Welfare Board is established under Prevention of Ozone Layer regulates the production and consumption
of nearly 100 man-made chemicals referred to as ozone
Cruelty to Animals Act, 1960.
depleting substances (ODS).
National Tiger Conservation Authority (NTCA) is a When released to the atmosphere, those chemicals
statutory body under the Ministry of Environment, damage the stratospheric ozone layer, Earth’s protective
Forests and Climate Change. The Wildlife Protection Act shield that protects humans and the environment from
of 1972 was amended in 2006 to provide for constituting harmful levels of ultraviolet radiation from the sun.
the National Tiger Conservation Authority responsible Adopted on 15 September 1987, the Protocol is to date the
for implementation of the Project Tiger plan to protect only UN treaty ever that has been ratified every country
endangered tigers. on Earth - all 198 UN Member States.

381 Workbook
.
ENVIRONMENT

22. Solution (a) Indigenous Peoples focused on reducing emissions from


Exp) Option a is correct deforestation and forest degradation, forest carbon stock
conservation, the sustainable management of forests, and
The United Nations Conference on Sustainable the enhancement of forest carbon stocks in developing
Development or Rio+20, took place in Rio de Janeiro, countries (activities commonly referred to as REDD+).
Brazil on 20-22 June 2012. It resulted in a focused political
outcome document which contains clear and practical The four strategic objectives of the FCPF:
measures for implementing sustainable development. • To assist countries in their REDD+ efforts by
In Rio, Member States decided to launch a process to providing them with financial and technical assistance
develop a set of Sustainable Development Goals (SDGs), in building their capacity to benefit from possible
which will build upon the Millennium Development future systems of positive incentives for REDD+.
Goals and converge with the post 2015 development • To pilot a performance-based payment system for
agenda. REDD+ activities, with a view to ensuring equitable
benefit sharing and promoting future large-scale
23. Solution (a) positive incentives for REDD+.
Exp) Option a is correct • Within the approach to REDD+, to test ways to
The Green Climate Fund (GCF) is a fund established sustain or enhance livelihoods of local communities
within the framework of the UNFCCC as an operating and to conserve biodiversity.
entity of the Financial Mechanism to assist developing • To disseminate broadly the knowledge gained in the
countries in adaptation and mitigation practices to development of the Facility and the implementation
counter climate change. of Readiness Preparation Proposals (RPPs) and
The GCF Board is charged with the governance and Emission Reductions Programs (ERPs).
oversight of the Fund’s management. It was established by 27. Solution (c)
194 sovereign governments party to the UN Framework
Convention on Climate Change (UNFCCC). The Board is Exp) Option c is correct
independent and guided by the Conference of the Parties The Genetic Engineering Appraisal Committee is
(COP) to the Convention. constituted in under the Environment (Protection) Act,
1986.
24. Solution (c)
The Genetic Engineering Appraisal Committee (GEAC)
Exp) Option c is correct functions in the Ministry of Environment, Forest and
BirdLife International is a global partnership of Climate Change (MoEF&CC).
conservation organisations that strives to conserve birds, It is responsible for appraisal of activities involving
their habitats and global biodiversity, working with people large scale use of hazardous microorganisms and
towards sustainability in the use of natural resources. recombinants in research and industrial production
In 1988, British ecologist Norman Myers published a from the environmental angle. The committee is also
seminal paper identifying 10 tropical forest “hotspots.” responsible for appraisal of proposals relating to release of
These regions were characterized both by exceptional genetically engineered (GE) organisms and products into
levels of plant endemism and serious levels of habitat loss. the environment including experimental field trials.
The Important Bird and Biodiversity Areas (IBA) 28. Solution (b)
programme of Birdlife International aims to identify, Exp) Option b is correct
monitor and protect a global network of IBAs for
conservation of the world’s birds and associated Paris Agreement is an international agreement to combat
biodiversity. climate change.
The agreement aims to keep the global temperature rise
25. Solution (d) this century well below 2 degrees Celsius above the pre-
Exp) Option d is correct industrial level and pursue efforts to limit the temperature
The BioCarbon Fund was set up as a public-private increase even further to 1.5 degrees Celsius.
sector initiative managed by the World Bank. The fund Not all members have signed this agreement and the
supports projects that generate “multiple revenue streams, agreement is scheduled to go into effect in 2020.
combining financial returns from the sale of emission COP21 agreed to extend the current goal of mobilizing
reductions (i.e., carbon credits) with increased local $100 billion a year in support by 2020 through 2025, with
incomes and other indirect benefits from sustainable land a higher goal for period after 2025 to strengthen the ability
management practices”. of countries to deal with the impacts of climate change.
26. Solution (c) 29. Solution (b)
Exp) Option c is correct Exp) Option b is correct
The Forest Carbon Partnership Facility is a global The Sustainable Development Goals (SDGs) are a
partnership of governments, businesses, civil society, and collection of 17 interlinked global goals designed to be

Workbook 382
.
ENVIRONMENT

a “blueprint to achieve a better and more sustainable recognize the wide range of benefits provided by
future for all”. The SDGs were set up in 2015 by the ecosystems and biodiversity, demonstrate their values in
United Nations General Assembly and are intended to economic terms and, where appropriate, capture those
be achieved by the year 2030. The 1972 United Nations values in decision-making.
Conference on the Environment in Stockholm was the
first world conference to make the environment a major 35. Solution (d)
issue. However, the SDGs were not proposed in 1972. Exp) Option d is correct
The UNREDD+ Programme can significantly contribute
30. Solution (b)
to all of the above.
Exp) Option b is correct
Reducing emissions from deforestation and forest
The Paris Agreement requests each country to outline degradation (REDD+) is a mechanism developed by
and communicate their post-2020 climate actions, known Parties to the United Nations Framework Convention on
as their Intended Nationally Determined Contributions Climate Change (UNFCCC). It creates a financial value
(INDCs). These are non-binding in nature. for the carbon stored in forests by offering incentives for
INDCs pair national policy setting in which countries developing countries to reduce emissions from forested
determine their contributions in the context of their lands and invest in low-carbon paths to sustainable
national priorities, circumstances and capabilities with a development.
global framework under the Paris Agreement that drives
36. Solution (a)
collective action toward a zero-carbon, climate-resilient
future. Exp) Option a is correct
GHG Protocol standards help countries, cities and
31. Solution (c) companies design climate mitigation goals, assess and
Exp) Option c is correct report progress toward goal achievement, and estimate
The United Nations Convention to Combat Desertification the greenhouse gas effects of policies and actions. It is
(UNCCD) is the only legally binding international developed by World Resources Institute (WRI) and World
agreement to address the problem of desertification. The Business Council on Sustainable Development (WBCSD).
Convention is based on the principles of participation, 37. Solution (a)
partnership and decentralization. The UNCCD was
signed in 1994 and came into force in 1996. Exp) Option a is correct
Agenda 21 is a non-binding, voluntarily implemented
32. Solution (b) action plan of the United Nations with regard to sustainable
Exp) Option b is correct development. It is a product of the Earth Summit (UN
The Food and Agriculture Organization of the United Conference on Environment and Development) held in
Nations (FAO) started an initiative in 2002 for Rio de Janeiro, Brazil, in 1992.
conservation of Globally Important Agricultural Heritage 38. Solution (b)
Systems (GIAHS). A GIAHS is a living, evolving system
of human communities in an intricate relationship with Exp) Option b is correct
their territory, cultural or agricultural landscape or Under Ramsar Convention, it is not mandatory to protect
biophysical and wider social environment. Therefore, it and conserve all the wetlands in the territory of India.
is about traditional agricultural systems and not modern According to Wetlands (Conservation and Management)
practices. Rules, 2010 , “wetland” means an area or of marsh, fen,
peatland or water; natural or artificial, permanent or
33. Solution (c) temporary, with water that is static or flowing, fresh,
Exp) Option c is correct brackish or salt, including areas of marine water, the
depth of which at low tide does not exceed six metres
Annex-I Countries Kyoto Protocol and includes all inland waters such as lakes, reservoir,
Certified Emissions Reductions Kyoto Protocol tanks, backwaters, lagoon, creeks, estuaries and manmade
wetland and the zone of direct influence on wetlands
Clean Development Mechanisms Kyoto Protocol that is to say the drainage area or catchment region of
34. Solution (c) the wetlands as determined by the authority but does not
include main river channels, paddy fields and the coastal
Exp) Option c is correct wetland.
The Economics of Ecosystems and Biodiversity (TEEB)
is a global initiative focused on “making nature’s values 39. Solution (b)
visible”. It strives to mainstream the values of biodiversity Exp) Option b is correct
and ecosystem services into decision-making at all levels. In India the AQI is based upon eight chief pollutants:
It aims to achieve this goal by following a structured Particulate Matter less than 10 micrometres (PM 10),
approach to valuation that helps decision-makers Particulate Matter less than 2.5 micrometres (PM 2.5),

383 Workbook
.
ENVIRONMENT

Ozone (O3), Nitrogen Dioxide (NO2), Carbon Monoxide TRAFFIC, the wildlife trade monitoring network, is the
(CO), Sulphur Dioxide (SO2), Ammonia (NH3) and Lead leading non-governmental organization working globally
(Pb). on trade in wild animals and plants in the context of both
biodiversity conservation and sustainable development.
40. Solution (b) TRAFFIC was established in 1976 by IUCN and WWF
Exp) Option b is correct. to respond to the growing threats posed by illegal wildlife
Statement 1 is incorrect. The Climate and Clean Air trade and overexploitation.
Coalition to Reduce Short-Lived Climate Pollutants
(CCAC) was launched by the United Nations 44. Solution (b)
Environment Programme (UNEP) and six countries — Exp) Option b is correct
Bangladesh, Canada, Ghana, Mexico, Sweden, and the MSTrIPES (Monitoring System for Tigers: Intensive
United States in February 2012. Protection and Ecological Status).
Statement 2 is correct. The Climate and Clean Air The MSTrIPES program uses Global Positioning System
Coalition is a voluntary partnership of governments, (GPS), General Packet Radio Services (GPRS), and
intergovernmental organizations, businesses, scientific remote sensing, to collect information from the field,
institutions and civil society organizations committed create a database using modern Information Technology
to improving air quality and protecting the climate (IT) based tools, analyses the information using GIS
through actions to reduce short-lived climate and statistical tools to provide inferences that allow
pollutants. tiger reserve managers to better manage their wildlife
The Coalition’s initial focus is on methane, black carbon, resources.
and HFCs. At the same time, partners recognize that
action on short-lived climate pollutants must complement 45. Solution (a)
and supplement, not replace, global action to reduce Exp) Option a is correct
carbon dioxide, in particular efforts under the UNFCCC Wildlife (Protection) Act, 1972 created six schedules
41. Solution (a) which gave varying degrees of protection to classes of
flora and fauna.
Exp) Option a is correct
Schedule I and Schedule II (Part II) get absolute
The Global Climate Change Alliance (GCCA) is an
protection, and offences under these schedules attract the
initiative of the European Union (EU) to strengthen
maximum penalties.
dialogue and cooperation on climate change with
developing countries most vulnerable to climate change. It consists of over 50 animals like Indian Lion, Leopard,
The GCCA focuses on the Least Developed Countries Indian Elephant and Tiger among others.
(LDCs) and the Small Island Developing States (SIDS), 46. Solution (d)
which are often the most affected by climate change but
have the fewest resources to tackle it. Exp) Option d is correct
The initiative was launched in 2007 and is coordinated by All the above mention animals are listed under Schedule
the European Commission (EC). I of Wildlife (Protection) Act, 1972 and thus they are
accorded maximum protection under law.
42. Solution (b)
47. Solution (b)
Exp) Option b is correct
Exp) Option b is correct.
The Climate and Clean Air Coalition (CCAC) to Reduce
Short Lived Climate Pollutants was launched by the Statement 1 is incorrect. Central Pollution Control
United Nations Environment Programme (UNEP) and 6 Board (CPCB), is statutory organization, was constituted
countries – Bangladesh, Canada, Ghana, Mexico, Sweden in September, 1974 under the Water (Prevention and
and the United States. Control of Pollution) Act, 1974.
CCAC is focused on reducing Short-lived climate Statement 2 is correct. National Green Tribunal (NGT)
pollutants (SLCPs). SLCPs are powerful climate forcers was set up in 2010 under the NGT Act, 2010, for the
that remain in the atmosphere for a much shorter period purpose of effective and expeditious disposal of cases
of time than carbon dioxide (CO2), yet their potential to relating to environmental protection. The Tribunal shall
warm the atmosphere can be many times greater. not be bound by the procedure laid down under the Code
The short-lived climate pollutants black carbon, methane, of Civil Procedure, 1908, but shall be guided by principles
tropospheric ozone, and hydrofluorocarbons are the of natural justice.
most important contributors to the man-made global CPCB serves as a field formation and also provides
greenhouse effect after carbon dioxide, responsible for up technical services to the Ministry of Environment and
to 45% of current global warming. Forests of the provisions of the Environment (Protection)
Act, 1986. Principal Functions of the CPCB, as spelt out
43. Solution (b) in the Water (Prevention and Control of Pollution) Act,
Exp) Option b is correct 1974, and the Air (Prevention and Control of Pollution)

Workbook 384
.
ENVIRONMENT

Act, 1981, (i) to promote cleanliness of streams and wells Under the Scheduled Tribes and Other Traditional Forest
in different areas of the States by prevention, control and Dwellers (Recognition of Forest Rights) Act, 2006 the
abatement of water pollution, and (ii) to improve the District Level Committee shall ensure that all Particularly
quality of air and to prevent, control or abate air pollution Vulnerable Tribal Groups receive habitat rights, in
in the country. consultation with the concerned traditional institutions of
Particularly Vulnerable Tribal Groups and their claims for
48. Solution (a) habitat rights are filed before the concerned Gram Sabhas.
Exp) Option a is correct
53. Solution (a)
Pakhui Wildlife Sanctuary lies in the foothills of the
Eastern Himalaya in the East Kameng district of Exp) Option a is correct.
Arunachal Pradesh. It was declared a tiger reserve in 2002. Statement 1 is correct. The central government constituted
It is bordered by the Kameng River and Pakke River. the Compensatory Afforestation Fund Management and
Planning Authority (CAMPA) for the management of
49. Solution (b) money towards compensatory afforestation, and other
Exp) Option b is correct money recoverable, in compliance of the conditions
GACSA is an inclusive, voluntary and action-oriented stipulated by the central government and in accordance
multi-stakeholder platform on Climate-Smart Agriculture with the Forest (Conservation) Act.
(CSA). GACSA was launched in 2014 during UN Climate The Compensatory Afforestation Fund Act 2016,
Summit. establishes the National and State Compensatory
India is not a member of GACSA, though few NGOs from Afforestation Fund Management and Planning
India are members of the alliance. Authorities to manage the National and State Funds.
Statement 2 is incorrect. There is no mention in the
50. Solution (b) Compensatory Afforestation Fund Act, 2016 that
Exp) Option b is correct People’s participation is mandatory in the compensatory
The Partnership for Action on Green Economy (PAGE) afforestation programmes carried out under the act.
was launched in 2013 as a response to the call at United
54. Solution (a)
Nations Conference on Sustainable Development 2012
Rio+20 to support those countries wishing to embark on Exp) Option a is correct
greener and more inclusive growth trajectories. PAGE The Agasthyamalai Biosphere Reserve comprises of
seeks to put sustainability at the heart of economic policies Neyyar, Peppara and Shendurney Wildlife Sanctuaries
and practices to advance the 2030 Agenda for Sustainable and Kalakad Mundanthurai Tiger Reserve.
Development. This Biosphere Reserve located in Western ghats became
It brings together the expertise of five UN agencies – part of World Network of Biosphere Reserve in 2016.
UNEP, ILO, UNIDO, UNDP and UNITAR – and working
closely with national governments. 55. Solution (d)
Exp) Option d is correct
51. Solution (c)
Valley of Flowers National Park is located in Uttarakhand.
Exp) Option c is correct The valley is situated at a very remarkable area, which is
The UNFCCC secretariat launched its Climate Neutral the conversion point of Himalayan ranges, Zanskar and
Now initiative in 2015. Western and Eastern Himalayas.
According to this initiative, Climate neutrality is a three
56. Solution (c)
step process, which requires individuals, companies and
governments to: Exp) Option c is correct
• Measure their climate footprint; The National Mission for Green India (GIM) is one of
the eight Missions outlined under the National Action
• Reduce their emissions as much as possible;
Plan on Climate Change (NAPCC). It aims at protecting,
• Offset what they cannot reduce with UN certified restoring and enhancing India’s diminishing forest cover
emission reductions. and responding to climate change by a combination of
52. Solution (a) adaptation and mitigation measures.
Exp) Option a is correct 57. Solution (b)
The Baigas were discriminated against and often evicted Exp) Option b is correct.
from forest areas by government agencies. They became Statement 1 is incorrect. Based on groundwater levels,
India’s first community to get habitat rights. This tribe is areas across the country are split into three categories:
found in Madhya Pradesh and Chhattisgarh. Over-exploited, Critical and Semi critical. ‘Overexploited’
The Ministry of Tribal Affairs is implementing the Scheme refers to groundwater being extracted more than what’s
for Development of Particularly Vulnerable Tribal Groups recharged, critical—where the groundwater taken out is
(PVTGs) 90-100% of what’s recharged—and semi-critical where

385 Workbook
.
ENVIRONMENT

extraction rate is 70%-90%. Places with lower extraction • Placement in Schedule VI provides for regulation in
rates are deemed ‘safe’ blocks. Almost 22% of groundwater the cultivation of a specified plant and restricts its
(assessed un- its) in the country has either dried up or possession, sale and transportation.
is in the ‘critical’ and ‘over-exploited’ categories. Out of • Both cultivation and trade of specified plants can only
the total 6881 assessment units in the country, 1186 be carried out with prior permission of the competent
units (17%) have been categorized as ‘Over-Exploited’, authority.
313units (5%) are ‘Critical’, 972 units (14%) are ‘semi-
• For cultivation of Schedule VI species, a license would
critical’. be required.
Statement 2 is correct. Central Ground Water
Authority has been constituted under Section 3 (3) of 62. Solution (a)
the Environment (Protection) Act, 1986 to regulate and Exp) Option a is correct
control development and management of ground water According to India’s National Policy on Biofuels the
resources in the country. following can be used as raw materials for the production
Statement 3 is correct. India has the largest area under of biofuels:
groundwater irrigation in the world. About 39 million sugar cane, sugar beet, sweet sorghum, corn, cassava,
hectares (67% of its total irrigation) of land is under algae, bagasse, wood waste, agricultural and forestry
groundwater irrigation. India is followed by China residues, edible and non-edible vegetable oils, and animal
(19 million ha) and the USA (17 million ha). India is fat of diesel quality.
the largest user of groundwater in the world. More than
60% of irrigated agriculture and 85% of drinking water 63. Solution: (b)
supplies are dependent on groundwater. Groundwater Exp) Option b is correct.
abstracted in the Indo-Gangetic basin is about one-fourth
Statement 1 is correct:Climate group is an international
of the global total.
non-profit founded in 2003, with offices in London,
58. Solution (c) New York, New Delhi, Amsterdam and Beijing. It has
grown its network to include over 500 multinational
Exp) Option c is correct businesses in 175 markets worldwide.
Area under “Critical Tiger Habitat”/ Area of the core: Statement 2 is incorrect:EP100 is a global initiative led
Nagarjunsagar-Srisailam (2595 sq. km.) > Sunderbans by The Climate Group and the Alliance to Save Energy.
(1699.62 sq. km.) > Ranthambore (1113.36 sq. km) > ( and not by International Energy Agency)
Corbett (821.99 sq. km.). Statement 3 is correct:The Climate Group’s global
59. Solution (a) EP100 initiative brings together a growing group of
energy-smart companies committed to improving their
Exp) Option a is correct energy productivity and doing more with less green-
Kanha Tiger Reserve is the only place in the wild where house gas emission. By integrating ambitious energy
the near-extinct Hard-ground Barasingha can be found. targets into business strategy, leading companies are
driving innovation in energy efficiency and increasing
60. Solution (a) competitiveness while delivering on emissions
Exp) Option a is correct reduction goals.
Musk deer live mainly in forested and alpine scrub habitats Statement 4 is correct: EP100 has a global reach, with
in the mountains of southern Asia, notably the Himalayas. member companies operating in more than 130 markets
Askot Wildlife Sanctuary is setup for conservation of Musk around the world. India’s own Mahindra group is one of the
Deer in its natural habitat. It is located near Pithoragarh leaders in the ongoing battle to cut energy consumption
in Uttarakhand. and carbon dioxide emissions by increasingly switching
to renewable energy.
Musk deer are also found in Gangotri National Park.
Statement 5 is incorrect:Climate Group is the Secretariat
61. Solution (a) to the Under2 Coalition and not the International Energy
Exp) Option a is correct Agency and works with governments to accelerate climate
action through four work streams:
• Wildlife Protection Act, 1972 is an Act passed by
the Parliament of India which came into force on 9th • Pathways
September 1972. • Policy action
• It was enacted for the protection of plants, birds and • Transparency
animal species. It has six schedules that extend to the • Diplomacy
whole of India.
64. Solution: (b)
• Schedule I to V deals with animals, whereas Schedule
VI regulates and restricts cultivation, possession and Exp) Option b is correct.
sale of a rare plant species. Statement 1 is correct:The World Health

Workbook 386
.
ENVIRONMENT

Organization(WHO) has released the Global Air Quality Chamber Commerce and Industry (FICCI), enables
Guidelines(AQGs).The new guidelines recommend air and supports safe and sustainable sanitation by bringing
quality levels for six pollutants particulate matter (PM) multiple organizations on a common platform through a
2.5 and PM 10, ozone (O3), nitrogen dioxide (NO2), range of catalytic actions. These include supporting the
sulfur dioxide (SO2) and carbon monoxide (CO). unlocking of WASH financing with focus on the private
sector, forging partnerships with allied organizations
for leading the discourse on sustainable sanitation;
convening, curating and disseminating best practices in
the sanitation advocacy — space and providing inputs
into the policy aspects of sanitation through participation
at allied forums.
It is not funded by WHO.
Statement 2 is correct. The National Institute of Urban
Affairs (NIUA) was appointed as an apex body to
support and guide the Government of India in its urban
development plans. Since then, it has worked closely with
the Ministry of Housing and Urban Affairs. NIUA seeks to
provide innovative solutions to address the challenges
of a fast-urbanising India, and pave the way for more
inclusive and sustainable cities of the future.
66. Solution: (c)
Exp) Option c is the correct answer.
Central Ground Water Authority has been constituted
under Section 3 (3) of the Environment (Protection)
Act, 1986 to regulate and control development and
Statement 2 is incorrect: Ozone at ground level is formed management of groundwater resources in the country.
by the reaction with sunlight (photochemical reaction) 67. Solution: (a)
of pollutants such as nitrogen oxides (NOx) from vehicle
and industry emissions and volatile organic compounds Exp) Option a is the correct answer.
(VOCs) emitted by vehicles, solvents and industry. As The International Code for Ships Operating in Polar
a result, the highest levels of ozone pollution occur Waters (the Polar Code) is a new code adopted by the IMO.
during periods of sunny weather and not during The Code acknowledges that polar waters may impose
inclement weather. additional demands on ships beyond those normally
Statement 3 is incorrect: PM is a common proxy encountered. It provides a mandatory framework for
indicator for air pollution. It affects more people than any ships operating in polar waters.
other pollutant. The major components of PM are sulfate, It is mandatory under both the International Convention
nitrates, ammonia, sodium chloride, black carbon, mineral for the Safety of Life at Sea (SOLAS) and the International
dust and water. It consists of a complex mixture of solid Convention for the Prevention of Pollution from Ships
and liquid particles of organic and inorganic substances (MARPOL). The Polar Code covers the full range of
suspended in the air. While particles with a diameter of design, construction, equipment, operational, training,
10 microns or less, (≤ PM10) can penetrate and lodge search and rescue and environmental protection matters
deep inside the lungs, the even more health-damaging relevant to ships operating in the inhospitable waters
particles are those with a diameter of 2.5 microns or less, surrounding the two poles. The Polar Code entered into
(≤ PM2.5). PM2.5 can penetrate the lung barrier and force on 1 January 2017.
enter the blood system. Chronic exposure to particles
contributes to the risk of developing cardiovascular and 68. Solution: (a)
respiratory diseases, as well as of lung cancer. Exp) Option a is the correct answer.
Statement 4 is correct: Excessive ozone in the air can Greenwashing is the process of conveying a false
have a marked effect on human health. It causes breathing impression or providing misleading information about
problems, trigger asthma, reduce lung function and cause how a company’s products are more environmentally
lung diseases. Ozone triggers asthma because it is very sound. Greenwashing is considered an unsubstantiated
irritating to the lungs and airways. claim to deceive consumers into believing that a
company’s products are environmentally friendly.
65. Solution: (b)
Exp) Option b is the correct answer. 69. Solution: (a)
Statement 1 is incorrect. India Sanitation Coalition Exp) Option a is the correct answer
(ISC), launched in June 2015, at Federation of Indian Statement 1 is correct: According to Section 39 of Wild

387 Workbook
.
ENVIRONMENT

Life (Protection) Act, 1972, Every Wild Animal shall actions within the city including investments.
be the property of the State Government, and, where The framework has 28 indicators across five categories
such animal is hunted in a sanctuary or National Park namely
declared by the Central Government, such animal or any
animal article, trophy, uncured trophy or meat  [derived (1) Energy and Green Buildings;
from such animal, or any vehicle, vessel, weapon, trap or (2) Urban Planning, Biodiversity and Green Cover;
tool used in such hunting] shall be the property of the (3) Mobility and Air;
Central Government. (4) Water Resource Management and
Statement 2 is correct: The law governing the subject (5) Waste Management.
of wildlife, the Wildlife (Protection) Act, 1972, does not
discriminate between animals found in protected areas Knowledge Base:
and outside. It provides for equal protection for wild C40 Cities Climate Leadership Group is a network of
animals irrespective of where they are found. the world’s 97 megacities committed to achieve the goals
Statement 3 is incorrect: According to Wildlife of Paris Agreement. C40 supports cities to collaborate
Protection Act, 1972, only if the wild animal becomes a effectively, share knowledge and drive meaningful,
danger to human life or is diseased or disabled beyond measurable and sustainable action on climate change.
recovery can it be allowed to be captured or killed by Indian cities in this group – Delhi, Mumbai, Bengaluru,
the competent authority, the Chief Wildlife Warden of the Chennai and Kolkata.
State. This provision is applicable to wild animals listed in
Schedule I of the Wildlife (Protection) Act, 1972, which 72. Solution (d)
includes leopards. Mere apprehension or fear that a wild Exp) Option d is correct.
animal could endanger human life is not a ground for Statement 1 is correct: Smog towers have been installed
capture or killing. in New Delhi recently to tackle the problem of high air
pollution. The filters installed in the tower use carbon
7.2. Schemes, Projects and Measures nanofibers as a major component and are fitted along its
peripheries. The tower focuses on reducing particulate
70. Solution (a) matter load.
Exp) Option a is correct. Statement 2 is correct: Smog towers can trap particulate
Statement 1 is correct. It outlines the process of water matter of all sizes suspended in the air. They can collect
supply, from raw water sources to household taps, and more than 75 per cent of particulate matters (PM) 2.5 and
has been developed keeping in view the Centre’s Jal Jeevan 10.
Mission. Statement 3 is correct: After the polluted air enters
Statement 2 is incorrect. It is expected that the system the smog tower, it is purified and re-circulated into the
would make the process of piped water supply more atmosphere. Large-scale air filters draw in the air through
uniform, especially in rural and underdeveloped areas fans installed at the top before passing it through the
(and not in metro cities only) of the country where the filters and releasing it near the ground.
system runs on various government orders and circulars.
73. Solution (d)
Statement 3 is correct. The system doesn’t mention how
water utilities should treat the water. But states that the Exp) Option d is correct.
process should be planned in such a manner that after Statement 1 is correct: Seabuckthorn is a soil-binding
treatment the drinking water should conform to the plant which prevents soil-erosion, checks siltation in
Indian Standard (IS) 10500 developed by the BIS. rivers and helps preserve floral biodiversity.
Statement 4 is incorrect. Drinking water supply quality Statement 2 is correct: In India, it is found above the
management system — requirements for piped drinking tree line in the Himalayan region, generally in dry areas
water supply service, the draft has been prepared by the such as the cold deserts of Ladakh and Spiti. According
BIS’ Public Drinking Water Supply Services Sectional to the Seabuckthorn Association of India, around 15,000
Committee (and not by Ministry of Jal Shakti). hectares in Himachal, Ladakh, Uttarakhand, Sikkim and
Arunachal Pradesh are covered by this plant.
71. Solution (c)
Statement 3 is correct: Seabuckthorn is widely used for
Exp) Option c is correct. treating stomach, heart and skin problems.
Climate Smart Cities Assessment Framework (CSCAF) It was seen in the news because recently the Himachal
is initiated and implemented by the Ministry of Housing Pradesh government decided to start planting
and Urban Affairs and is supported by the Climate seabuckthorn in the cold desert areas of the state to
Centre for Cities under National Institute of Urban prevent erosion.
Affairs (NIUA).
It aims to provide a clear roadmap for Indian cities 74. Solution (b)
towards combating climate change while planning their Exp) Option b is correct.

Workbook 388
.
ENVIRONMENT

To curb growing menace of air pollution through the thereby, significantly reducing the harmful effect of the
vehicle’s emission, the Government of India has decided plastic on the environment.
to leapfrog from the exiting BS – IV norms to the BS- VI, Statement 3 is correct: Swachh Bharat Abhiyaan directly
thereby skipping the BS – V norms, and to implement the generates employment and Namami Gange programme
BS – VI norms with effect from 1st April 2020. contributes towards lesser pollution cleaner environment.
Statement 1 is incorrect. These norms are applicable Statement 4 is correct: Clean and Affordable Energy
to all two wheelers, three wheelers, four wheelers and generated under International Solar Alliance (ISA) can
construction equipment vehicles. help developing countries in their transition to a low-
Statement 2 is correct. Upgrading to stricter fuel carbon economy.
standards helps tackle air pollution by reducing the
amount of sulphur present in the fuels and hence reducing 77. Solution (c)
the amount of NOx (nitrogen oxides) released from the Exp) Option c is correct.
vehicles.
Statement 1 is correct. Union Power Ministry has
Statement 3 is correct. The newly introduced BS-VI fuel launched Green Term Ahead Market (GTAM) in
is estimated to reduce the amount of sulphur released by electricity. The market is first Exclusive product for
80%, from 50 parts per million to 10 ppm. renewable energy sector in the world. GTAM would
Knowledge Base: BS fuel norms are based on the European benefit buyers of renewable energy through competitive
EURO fuel standards. prices and transparent and flexible procurement. It will
also benefit sellers as they will get access to Pan India
75. Solution (c)
Market.
Exp) Option c is correct.
Statement 2 is correct. Transactions through GTAM
Statement 1 is correct: Gross Environment Product will be bilateral in nature with clear identification
(GEP) is an assessment method to measure ecological of corresponding buyers and sellers, there will not be
status. It can be considered as the valuation of products any difficulty in accounting for Renewable Purchase
and services that the ecosystem provides for Welfare Options (RPO). GTAM contracts will be segregated into
of human beings, economic and social development and Solar RPO & Non-Solar RPO as RPO targets are also
sustainable development. segregated.
Statement 2 is correct: In India, Uttarakhand government
has recently announced to initiate valuation of its natural 78. Solution (d)
resources in the form of GEP, along the lines of Gross Exp) Option d is correct.
Domestic Product.
Pair 1 is correctly matched. Environment Performance
Knowledge Base: GEP is one of the components of green Index has been released biennially by Yale and Columbia
GDP. Green GDP acts as an indicator of economic growth universities in collaboration with the World Economic
while keeping environmental factors in mind along with Forum (WEF).
calculation of the GDP of a nation.
The 2020 Environmental Performance Index (EPI)
76. Solution (d) provides a data-driven summary of the state of
Exp) Option d is correct. sustainability around the world. Using 32 performance
indicators across 11 issue categories, the EPI ranks
United Nations Environment Programme (UNEP) defines
Green Economy as one that results in improved human 180 countries on environmental health and ecosystem
well-being and social equity while significantly reducing vitality.
environmental risks and ecological scarcity. It provides Denmark was ranked first in 2020.
for Sustainable Development and poverty eradication. India was ranked 168 in 2020. In 2018, it was ranked 177.
There is no one defined way to achieve it. United Nation All South Asian countries except Afghanistan are ahead of
acknowledges it through its Partnership for Action on India in the ranking.
Green Economy (PAGE) programme.
Pair 2 is correctly matched. Global Climate Risk Index
Statement 1 is correct: Transition to Green Economy is published by the German watch, the Germany-based
prioritizes decarbonizing the economy. It can be done think tank.
through initiatives like promotion of Bharat Stage-VI
vehicles in India. Additionally, the transition needs The Global Climate Risk Index 2021 analyses to what
committing the environmental community to justice extent countries and regions have been affected by
and equity: conserve the biosphere. In the process, The impacts of weather-related loss events (storms, floods,
Economics of Environment and Biodiversity (TEEB) and heat waves etc.). Human impacts (fatalities) and direct
Ecological Footprint calculation are of significant help. economic losses were analysed. The most recent data
Statement 2 is correct: Extended Producer Responsibility available — for 2019 and from 2000 to 2019 — were taken
(EPR) under Plastic Waste Management Rules, 2016 has into account.
the responsibility of a producer for the environmentally India ranked at 7th Position in Climate Risk Index 2021.
sound management of the product until the end of its life In 2020, India ranked 5th on the index.

389 Workbook
.
ENVIRONMENT

Mozambique, Zimbabwe, and the Bahamas were Statement 3 is correct. Ocean Alkalinity Enhancement
respectively the top three countries most affected in 2019. is the process of grinding up, dispersing, and dissolving
Pair 3 is correctly matched. Climate Change Performance rocks such as limestone, silicates, or calcium hydroxide
Index has been developed by not-for-profit organizations in the ocean to increase its ability to store carbon and
Germanwatch and New Climate Institute (Germany) directly ameliorate ocean acidification.
together with the Climate Action Network (CAN
International).
The Climate Change Performance Index (CCPI)
tracks countries’ efforts to combat climate change.
It aims to enhance transparency in international climate
politics and enables comparison of climate protection
efforts and progress made by individual countries.
The Parameters used are GHG emissions (40%), renewable
energy (20%), energy use (20%) and climate policy (20%).
Sweden leads in climate protection.
India has dropped by one position from ninth in 2019 to
10th in 2020.
79. Solution (d)
Exp) Option d is correct.
The Food Safety and Standards Authority of India (FSSAI)
had launched RUCO (Repurpose Used Cooking Oil), an
initiative that will enable collection and conversion of
used cooking oil to bio-diesel.
Statement 1 is correct. During frying, several properties of
oil are altered, Total Polar Compounds (TPC) are formed
on repeated frying. The toxicity of these compounds is
associated with several diseases such as hypertension,
atherosclerosis, Alzheimer’s disease, liver diseases.
FSSAI has fixed a limit for Total Polar Compounds at 25 81. Solution (d)
percent beyond which the vegetable oil shall not be used.
From 1st July, 2018 onwards, all Food Business Operators Exp) Option d is correct.
(FBOs) are required to monitor the quality of oil during Statement 1 is correct: National Clean Air Programme
frying by complying with the said regulations. was launched by Ministry of Environment, Forest
Statement 2 is correct. Biodiesel from used cooking oil and Climate Change in 2019. It seeks to cut the
can easily replace diesel in vehicles and plant machinery. concentration of coarse (particulate matter of diameter
After processing, the resulting biodiesel can easily replace 10 micrometer or less, or PM10) and fine particles
diesel in numerous existing machines and vehicles. Many (particulate matter of diameter 2.5 micrometer or less,
of these machines will not need any re calibration for or PM2.5) by 20% to 30% in the next five years, with
them to use this fuel. 2017 as the base year for comparison.
Statement 3 is correct. It substitutes some amount of Statement 2 is correct: National Clean Air Programme
petrochemical oil import. This can help in reducing is implemented by Central Pollution Control Board.
current account deficit. It is a time bound national level strategy for pan India
implemented to tackle the increasing air pollution problem
80. Solution (b) across the country. Implementation of the city specific
Exp) Option b is correct. action plans are regularly monitored by Committees at
Statement 1 is correct. The global-level afforestation Central and State level namely Steering Committee and
is considered as a Geo-engineering technique. Implementation Committee.
Geoengineering is the deliberate large-scale intervention Statement 3 is correct: Currently, the plan includes
in the Earth’s natural systems to counteract climate 122 non-attainment cities, across the different states
change. and Union territories, which were identified by the
Statement 2 is incorrect. Ocean Fertilisation is the Central Pollution Control Board (CPCB) on the basis
adding of nutrients to the ocean in selected locations to of their ambient air quality data between 2011 and
increase primary production. This results in decrease in 2015. Recently, NGT has slammed the Ministry of
the carbon dioxide in the atmosphere. It is a part of the Environment, Forest and Climate Change over its
geoengineering. report on National Clean Air Programme.

Workbook 390
.
ENVIRONMENT

82. Solution (c) Statement 1 is correct. The scheme aims to increase the
Exp) Option c is correct. competitiveness, polymer absorption capacity and value
addition in the domestic downstream plastic processing
Leadership in Energy & Environmental Design (LEED): industry through adaptation of modern, research and
It is an international recognized certification system for development led measurers.
the green buildings developed by the U.S. Green Building
Council Statement 2 is correct. The Department of Chemicals
and Petrochemicals under the Ministry for Chemicals &
GRIHA-It is jointly developed by The Energy and Fertilizers has approved setting up of 10 Plastic Parks in
Resources Institute (TERI) and the Ministry of New the country, out of which 6 parks have been given final
and Renewable Energy. It is rating tool evaluates the approval in the States of Assam, Madhya Pradesh (two
environmental performance of a building holistically over parks), Odisha, Tamil Nadu and Jharkhand.
its entire life cycle
Statement 3 is correct. A plastic park is an industrial zone
Eco niwas samhita- Ministry of Power has launched the devoted to plastic enterprises and its allied industries.
Energy Conservation- New Indian Way for Affordable It includes a whole range of companies required by the
& Sustainable Homes (ECO Niwas) Samhita 2018, plastics processing.
an Energy Conservation Building Code for Residential
Buildings (ECBC-R) to give a further fillip to India’s community from material and machinery suppliers,
energy conservation efforts. plastics processing companies, plastic recycling
companies including waste management system.
83. Solution (b)
86. Solution (a)
Exp) Option b is correct.
Exp) Option a is correct.
Anamaya, the Tribal Health Collaborative, was launched
to enhance the health and nutrition status of the tribal The Green Bonus Demand is the demand of Himalayan
communities of India. The Collaborative is a multi- States to the Government to frame policies on the basis
stakeholder initiative of Tribal Affairs Ministry supported of the social, political and economic specificity of the
by Piramal Foundation and Bill and Melinda Gates Mountain regions and not based on the riverine plains.
Foundation (BMGF). It will converge efforts of various These hilly states want additional budgetary provisions
Government agencies and organisations. for their ecological services — green cover belt, fresh air
and water of rivers — to the country.
This Collaborative is a unique initiative bringing together
governments, philanthropists, national and international Recently, 11 Himalayan States of India met in Dehradun,
foundations, NGOs/CBOs to end all preventable deaths Uttarakhand, demanding a “green bonus”, or a payment
among the tribal communities of India. It aims to build for environmental services they provide to the nation.
a sustainable, high-performing health eco-system to A “Mussoorie resolution” was passed at the conclave
address the key health challenges faced by the tribal making a collective pledge to conserve and protect
population of India. It will begin its operations with 50 their rich cultural heritage, bio-diversity, glaciers, rivers
tribal, Aspirational Districts (with more than 20% ST and lakes besides making their own contribution to the
population) across 6 high tribal population states. Over a nation’s prosperity.
10-year period, the work of the THC will be extended to
87. Solution (a)
177 tribal Districts as recognised by the Ministry of Tribal
Affairs. Exp) Option a is correct.
The Green-Ag Project aims to catalyse transformative
84. Solution (b) change of India’s agricultural sector to support
Exp) option b is correct. achievement of national and global environmental
RAISE initiative stands for “Retrofit of Air-conditioning benefits and conservation of critical biodiversity and
to improve Indoor Air Quality for Safety and Efficiency”. forest landscapes.
This was launched to alleviate the issue of bad air quality Statement 1 is correct: Key missions that will be targeted
in workspaces across the nation. for strengthening include community led initiatives to
RAISE 2020 mega summit was organized recently for support conservation of globally important species such
exchange of ideas and chart a course for using AI for as the tigers, elephants and the Great Indian Bustard.
social transformation, inclusion and empowerment in Such initiatives could include community led actions such
areas like Healthcare, Agriculture, Education and Smart as community anti-poaching patrolling, community led
Mobility, among other sectors. communication/ awareness activities, habitat and species
Here we are talking about RAISE initiative which monitoring activities. These will be strongly linked to
focuses upon improving air quality in workplaces Tiger Reserve and Elephant reserve management plans at
across India. the landscapes that the project will be working.
Statement 2 is correct: This project has been implemented
85. Solution (d) with the help of funding from Global Environment
Exp) Option d is correct. Facility (GEF).

391 Workbook
.
ENVIRONMENT

Statement 3 is incorrect: It is being implemented only in • National Mission for Enhanced Energy Efficiency
high conservation value landscapes in 5 chosen states - • National Mission on Sustainable Habitat
(i) Madhya Pradesh: Chambal Landscape, (ii) Mizoram:
Dampa Landscape, (iii) Odisha: Similipal Landscape, • National Water Mission
(iv) Rajasthan: Desert National Park Landscape and v) • National Mission for Sustaining the Himalayan
Uttarakhand: Corbett-Rajaji Landscape. Ecosystem
• National Mission for A Green India
88. Solution (b)
Exp) Option b is correct • National Mission for Sustainable Agriculture
Statement 1 is incorrect. NMCG (National mission for • National Mission on Strategic Knowledge for Climate
clean Ganga) is the implementing wing of NGC (National Change
Ganga council) and is not headed by Prime minister of Option (c) is correct: Perform, Achieve and Trade’
India. NGC is headed by Prime minister. NMCG has scheme has been launched under the National Mission
a two-tier management structure and comprises of for Enhanced Energy Efficiency, one of the eight
Governing Council and Executive Committee. Both of missions under the umbrella National Action Plan on
them are headed by Director General, NMCG. Climate Change. It was launched by Bureau of Energy
Statement 2 is incorrect. The National Ganga Council Efficiency (BEE) with the aim to make the industrial
(NGC) is an authority created in October 2016 under the sector energy efficient.
River Ganga (Rejuvenation, Protection and Management) Option (d) is incorrect: Prime Minister’s Council on
Authorities Order, 2016, dissolving the National Ganga Climate Change is in charge of the overall implementation
River Basin Authority. of the plan. Funding to this plan is done by National
Statement 3 is correct. 8 pillars of Namami Gange Adaptation Fund for Climate Change (NAFCC), a fund
program are- 1. Sewerage Treatment Infrastructure 2. managed by the NABARD.
River-Surface Cleaning 3. Afforestation 4. Industrial
Effluent Monitoring 5. River-Front Development 6. 91. Solution (a)
Bio-Diversity 7. Public Awareness 8. Ganga Gram. Exp) Option a is correct.
89. Solution (a) Statement 1 is correct. Zero budget natural farming
(ZBNF) is a method of chemical-free agriculture
Exp) Option a is correct. drawing from traditional Indian practices.
Statement 1 is correct. The National Ganga Council, Instead of commercially produced chemical inputs,
also known as the National Council for Rejuvenation, the ZBNF promotes the application of jeevamrutha — a
Protection, and Management of River Ganga was set up mixture of fresh desi cow dung and aged desi cow urine,
in 2016. It replaced the National River Ganga Basin jaggery, pulse flour, water and soil — on farmland. This is
Authority (NRGBA). a fermented microbial culture that adds nutrients to the
Statement 2 is correct. The National Ganga Council is soil, and acts as a catalytic agent to promote the activity of
formed under the Environment (Protection) Act (EPA), microorganisms and earthworms in the soil
1986.
A similar mixture, called bijamrita, is used to treat
Statement 3 is incorrect. The National Ganga Council is seeds, while concoctions using neem leaves and pulp,
chaired by Prime Minister. tobacco and green chillis are prepared for insect and pest
90. Solution (d) management.
Exp) Option d is correct. Statement 2 is correct. The ZBNF method also promotes
soil aeration, minimal watering, intercropping, bunds and
Option (a) is correct: National Action Plan on Climate topsoil mulching and discourages intensive irrigation
Change was launched on 30th June 2008. It is a multi- and deep ploughing.
pronged long-term strategy for promoting understanding
of climate change, adaptation and mitigation, energy Statement 3 is incorrect. Karnataka recently initiated
efficiency and natural resource conservation. “National implementation of zero budget natural farming
Mission on Sustainable Habitat”, that seeks to promote (ZBNF) on a pilot basis in 2,000 hectares in each of the
sustainability of habitats is one of the eight missions 10 agro-climatic zones of the state.
under the plan. According to the Economic Survey 2019, more than 1.6
Option (b) is correct: National Action Plan on lakh farmers were practising the ZBNF in almost 1,000
Climate Change effectively pulls together a number villages using some form of state support.
of the government’s existing national plans on water, ZBNF was originally promoted by Maharashtrian
renewable energy, energy efficiency agriculture and others agriculturist and Padma Shri recipient Subhash Palekar,
under the umbrella of eight missions. These missions are who developed it in the mid-1990s as an alternative to
as follows: the Green Revolution’s methods driven by chemical
• National Solar Mission fertilizers, pesticides and intensive irrigation.

Workbook 392
.
ENVIRONMENT

92. Solution (a) forest lands and improved quality of forest cover on
Exp) Option a is correct. another 5 m ha (a total of 10 m ha).
Statement 1 is correct. It outlines the process of water 2. Improved ecosystem services including biodiversity,
supply, from raw water sources to household taps, and hydrological services and carbon sequestration as a
has been developed keeping in view the Centre’s Jal Jeevan result of treatment of 10 m ha.
Mission. 3. Increased forest-based livelihood income for 3
Statement 2 is incorrect. It is expected that the system million forest dependent households.
would make the process of piped water supply more 4. Enhanced annual CO2 sequestration of 50-60 million
uniform, especially in rural and underdeveloped areas tonnes by the year 2020.
(and not in metro cities only) of the country where the
system runs on various government orders and circulars. The Mission also aims to build a skilled cadre of young
community foresters from scheduled tribes and other
Statement 3 is correct. The system doesn’t mention how
forest dwelling communities.
water utilities should treat the water. But states that the
process should be planned in such a manner that after 95. Solution (c)
treatment the drinking water should conform to the
Indian Standard (IS) 10500 developed by the BIS. Exp) Option c is correct.
Statement 4 is incorrect. Drinking water supply quality National Clear Air Programme was launched by the
management system — requirements for piped drinking MoEFCC in January 2019. It is the first-ever effort in the
water supply service, the draft has been prepared by the country to frame a national framework for air quality
BIS’ Public Drinking Water Supply Services Sectional management with a time-bound reduction target.
Committee (and not by Ministry of Jal Shakti).
Statements 1 and 2 are correct. It seeks to reduce PM2.5
93. Solution (c) and PM10 concentration by 20%–30% by 2024, taking
Exp) Option c is correct 2017 as the base year for the comparison of concentration.
Pair 1 is correctly matched: National Solar Mission aims Statement 3 is incorrect. It will be implemented in 124
to increase share of solar power in India’s energy mix. non-attainment cities (102 initially). Under NCAP, 124
Initial target was 20 GW by 2022 which has been revised non-attainment cities have been identified across the
to 100 GW. Ministry of New and Renewable Energy is the country based on the Air Quality data from 2014-2018.
nodal ministry. Non-attainment cities: These are those that have fallen
Pair 2 is correctly matched: NMEEE aims to strengthen short of the National Ambient Air Quality Standards
the market mechanisms that exist for energy efficiency (NAAQS) for over five years.
by creating a conducive regulatory and policy regime. Statement 4 is correct. The NCAP will be a long-term,
It includes promotion of innovative business models,
time-bound, national level strategy, five-year action
financial mechanisms that will help create and sustain
markets for energy efficiency. Perform, Achieve & Trade plan with 2019 as the first year.
(PAT) is the most important component under the 96. Solution (a)
mission. Ministry of Power is the implementing ministry.
Exp) Option a is correct.
Pair 3 is incorrectly matched: NMSHE aims to protect
and sustain the fragile Himalayan ecosystem by building Statement 1 is correct. Approved by the Supreme Court
knowledge, capacity and sustainable development models in 2016, the plan was formulated after several meetings
for the region. Because it involves intensive research and that the Environment Pollution (Prevention and
data analysis on the possible impacts on the region, it is Control) Authority (EPCA) held with state government
implemented by the Ministry of Science and Technology. representatives and experts. The result was a plan that
Pair 4 is correctly matched: NMSKCC seeks to build institutionalised measures to be taken when air quality
knowledge networks among the existing institutions deteriorates.
engaged in R&D relating to climate change. It is
Statement 2 is correct. GRAP includes the measures
implemented by the Ministry of Science and Technology.
which will be taken by different government agencies
94. Solution (d) to prevent worsening of Air Quality of Delhi-NCR
Exp) Option d is correct. and prevent PM10 and PM2.5 levels to go beyond the
All the statements are correct. ‘moderate’ national Air Quality Index (AQI) category. If
air quality reaches the severe+ stage, GRAP talks about
The National Mission for a Green India (also known as
Green India Mission) is one of the eight Missions under shutting down schools and implementing the odd-even
the National Action Plan on Climate Change (NAPCC). road-space rationing scheme.
The objectives of the Mission are - Statement 3 is incorrect. It has only four stages of
1. Increased forest/tree cover on 5 m ha of forest/non- implementation as per the pollution load.

393 Workbook
.
ENVIRONMENT

Compressed Bio-Gas CBG) plants across India.


The scheme envisages setting up of 5000 CBG plants by
FY 2023-24. Signing of the recent MoUs will give a big
fillip to the clean energy initiative of the Government.
99. Solution (d)
Exp) Option d is correct.
All the statements are correct.
Government of India has launched Sustainable Alternative
Towards Affordable Transportation (SATAT) initiative
on 1.10.2018 to promote Compressed Bio Gas (CBG)
as an alternative, green transport fuel for efficient
management of biomass and organic waste including
municipal waste as well as from forest and agricultural
waste including animal-husbandry and marine waste.
CBG can be generated form agricultural wastes, thus
will provide an alternate source of income to farmers.
97. Solution (c) Farmer can make income by converting CBG into ethanol.
Exp) Option c is correct. 100. Solution (c)
Statement 1 is correct. India is one of the first countries Exp) Option c is correct.
in the world to develop a comprehensive Cooling Action Faster Adoption and Manufacturing of (Hybrid &)
plan which has a long term vision to address the cooling Electric Vehicles in India (FAME India-2) Scheme was
requirement across sectors and lists out actions which launched on April 1, 2019 with an outlay of Rs. 10,000
can help reduce the cooling demand. It was launched by Crore. It is to be completed by March 31, 2022. The scheme
the Ozone Cell of the Union Ministry of Environment, is implemented by the Department of Heavy Industry,
Forest and Climate Change (MoEFCC). ICAP aims to Ministry of Heavy Industries & Public Enterprises.
provide sustainable cooling (reducing cooling demand,
refrigerant demand) while keeping in mind, at the same Statement 1 is correct. FAME India-2 scheme puts
time, the need to protect the ozone layer. emphasis on electrification of the public transportation
including shared transport.
Statement 2 is correct. Following are the benefits of
ICAP: Statement 2 is incorrect. It aims to generate demand by
way of supporting 7000 Electric Buses (e-bus), 5 lakh
• Thermal comfort for all – provision for cooling for Electric Three Wheelers (e-3W), 55000 Electric Four-
EWS and LIG housing Wheeler Passenger Cars (including Strong Hybrid) (e-
• Sustainable cooling – low GHG emissions related to 4W) and 10 lakh Electric Two Wheelers (e-2W).
cooling Scheme will be applicable mainly to vehicles used for
• Doubling Farmers Income – better cold chain public transport or those registered for commercial
infrastructure – better value of produce to farmers, purposes in e-3W, e-4W and e-bus segments. However,
less wastage of produce privately owned registered e-2Ws are also covered under
• Skilled workforce for better livelihoods and the scheme as a mass segment.
environmental protection Statement 3 is correct. Vehicles, fitted with only advanced
• Make in India – domestic manufacturing of air- chemistry battery, meeting with minimum Techanical
conditioning and related cooling equipment’s Criteria and registered as “Motor Vehicle” as per CMVR
shall be eligible for incentive under the scheme.
• Robust R&D on alternative cooling technologies – to
provides push to innovation in cooling sector. 101. Solution (b)
98. Solution (a) Exp) Option b is correct.
Exp) Option a is correct. The System of Air Quality and Weather Forecasting And
Research (SAFAR) is a national initiative introduced by
Sustainable Alternative Towards Affordable the Ministry of Earth Sciences (MoES).
Transportation (SATAT) initiative has been launched
to boost availability of affordable and clean transport Statement 1 is correct. The system is indigenously
fuels. It envisages boosting production and availability developed by the Indian Institute of Tropical
of CBG as an alternative and affordable clean fuel for Meteorology (IITM), Pune and is operationalized by the
transportation sector. Under this, recently an MoU has India Meteorological Department (IMD).
been signed between MoPNG and leading oil & gas Statement 2 is correct. It measures the air quality of a
marketing companies & technology providers to establish metropolitan city, by measuring the overall pollution

Workbook 394
.
ENVIRONMENT

level and the location-specific air quality of the city. It Statement 2 is correct. Upgrading to stricter fuel
gives out real-time air quality index on a 24x7 basis with standards helps tackle air pollution by reducing the
color-coding (along with 72 hours advance forecast). amount of sulphur present in the fuels and hence reducing
Statement 3 is correct. It monitors all weather parameters the amount of NOx (nitrogen oxides) released from the
like temperature, rainfall, humidity, wind speed, and wind vehicles.
direction, UV radiation, and solar radiation. Statement 3 is correct. The newly introduced BS-VI fuel
Statement 4 is incorrect. Pollutants monitored: PM2.5, is estimated to reduce the amount of sulphur released by
PM10, Ozone, Carbon Monoxide (CO), Nitrogen Oxides 80%, from 50 parts per million to 10 ppm.
(NOx), Sulfur Dioxide (SO2), Benzene, Toluene, Xylene, 104. Solution (b)
and Mercury.
Exp) Option b is correct.
102. Solution (d) Statement 1 is incorrect. The Indian Council of
Exp) Option d is correct. Agriculture Research (ICAR) has launched National
National Air Quality Index: Initiative on Climate Resilient Agriculture (NICRA)
(and not The Energy and Resource Initiative) during
Launched in 2014 with outline ‘One Number - One 2010-11.
Color - One Description’ for the common man to
judge the air quality within his vicinity. It was launched Statement 2 is correct. This initiative will primarily
for monitoring the quality of air in major urban centres enhance the resilience of Indian Agriculture covering
across the country on a real-time basis and enhancing crops, livestock and fisheries.
public awareness for taking mitigative action. 105. Solution (c)
The measurement of air quality is based on eight pollutants, Exp) Option c is correct.
namely: Particulate Matter (PM10), Particulate Matter
(PM2.5), Nitrogen Dioxide (NO2), Sulphur Dioxide Statement 1 is correct. The National Adaptation Fund
(SO2), Carbon Monoxide (CO), Ozone (O3), Ammonia for Climate Change (NAFCC) was established in August,
(NH3), and Lead (Pb). 2015 to meet the cost of adaptation to climate change
for the State and Union Territories of India that are
AQI has six categories of air quality. These are: Good, particularly vulnerable to the adverse effects of climate
Satisfactory, Moderately Polluted, Poor, Very Poor and change. The projects under NAFCC prioritizes the needs
Severe. that builds climate resilience in the areas identified under
It has been developed by the CPCB in consultation with the SAPCC (State Action Plan on Climate Change) and
IIT-Kanpur and an expert group comprising medical and the relevant Missions under NAPCC (National Action
air-quality professionals. Plan on Climate Change).
Statement 2 is correct. NABARD has been designated as
National Implementing Entity (NIE) for implementation
of adaptation projects under NAFCC by Govt. of India.
NABARD is also National Implementing Entity (NIE)
for Adaptation Fund (AF) under Kyoto Protocol and its
presence across the country.
106. Solution (a)
Exp) Option a is correct.
Statement 1 is correct. Government has approved Phase-
II of FAME Scheme with an outlay of Rs. 10,000 Crore
for a period of 3 years commencing from 1st April 2019.
Out of total budgetary support, about 86 percent of fund
has been allocated for Demand Incentive so as to create
demand for Electric Vehicles (EVs) in the country.
103. Solution (b)
Statement 2 is incorrect. Vehicles, fitted with only
Exp) Option b is correct. advanced chemistry battery, meeting with minimum
To curb growing menace of air pollution through the Technical Criteria and registered as “Motor Vehicle”
vehicle’s emission, the Government of India has decided as per CMVR shall be eligible for incentive under the
to leapfrog from the exiting BS – IV norms to the BS- VI, scheme. With greater emphasis on providing affordable
thereby skipping the BS – V norms, and to implement the & environment friendly public transportation options for
BS – VI norms with effect from 1st April 2020. the masses, scheme will be applicable mainly to vehicles
Statement 1 is incorrect. These norms are applicable used for
to all two wheelers, three wheelers, four wheelers and public transport or those registered for commercial
construction equipment vehicles. purposes in e-3W, e-4W and e-bus segments. However,

395 Workbook
.
ENVIRONMENT

privately owned registered e-2Ws are also covered under categorizing the developmental projects in two categories,
the scheme as a mass segment. i.e., Category A and Category B. Category A projects
require mandatory environmental clearance and thus we
107. Solution (c) do not undergo the screening process. Category B projects
Exp) Option c is correct. undergoes screening process and they are classified into
Option c is correct. The Delhi Declaration is a consensus two types. 1. Category B1 projects (Mandatory requires
document on the proposed adoption of a “voluntary” EIA). 2. Category B2 projects (Do not require EIA). Thus,
land degradation neutrality target by India. It includes Category A projects and Category B1 projects undergo
a commitment on a range of issues, including gender and the complete EIA process whereas Category B2 projects
health, ecosystem restoration, taking action on climate are excluded from complete EIA process.
change, private sector engagement, Peace Forest Initiative
and recovery of five million hectares of degraded land 7.3. Acts and Policies
in India.
109. Solution (a)
108. Solution: (a) Exp) Option a is correct.
Exp) Option a is correct Statement 1 is correct: Non-conventional power and
Environmental Impact Assessment (EIA) is a process associated structures are prohibited in CRZ I-A zone
of evaluating the likely environmental impacts of a (maximum protected zone). These activities are allowed
proposed project or development. Environmental impact from CRZ I-B zone onwards (CRZ II, CRZ III and CRZ
assessment (EIA) is one of the tools available with the IV).
planners to achieve the goal of harmonising development Statement 2 is incorrect: Construction activities related
activities with the environmental concerns. Features of to tourism like building resorts, hotels, etc in vacant
EIA in India are: plots have now been allowed in CRZ II zone and onwards
Statement 1 is correct - The Environment Impact in order to boost coastal economy.
Assessment in India is statutorily backed by the Statement 3 is incorrect: Activities related to agriculture,
Environment Protection Act, 1986. It contains various horticulture, pasture, gardens, playfields, and forestry are
provisions on EIA methodology and process. The first allowed in CRZ III zone and onwards.
EIA notification was promulgated in 1994 by the then
Ministry of Environment and Forests (now the Ministry Statement 4 is incorrect: Construction of monuments
of Environment, Forests and Climate Change). The and memorials by the concerned state government
notification mandated Environmental Clearance (EC) for is allowed only in the CRZ IV zone, but with adequate
expansion or modernisation of any activity or for setting safeguards.
up new projects listed in Schedule 1 of the same. Although 110. Solution (b)
the EIA in India was started in 1976-77, these were
administrative decisions and lacked the legislative support Exp) Option b is correct.
until government enacted Environmental Protection Act Statement 1 is incorrect: All healthcare facilities are
of 1986. expected to phase out use of chlorinated products and
Statement 2 is incorrect – To provide notice for gloves (except blood bags) as per the 2018 amendment
public hearing under EIA is the responsibility of State to the Rules.
Pollution Control Board and not Gram Panchayats. Statement 2 is correct: Although there was no such
The State Pollution Control Board shall cause a notice for obligation earlier, now healthcare facilities have
environmental public hearing which shall be published to designate a clean, ventilated space for packed
in at least two newspapers widely circulated in the region biomedical waste as it awaits transportation to common
around the project. State Pollution Control Board shall treatment facilities. This packed waste should be properly
mention the date, time and place of public hearing. segregated into 4 categories and pretreated with
Suggestions, views, comments and objections of the prescribed chemicals as per WHO guidelines to reduce
public shall be invited within thirty days from the date of their lethality.
publication of the notification.
111. Solution (c)
Statement 3 is correct - It is the responsibility of the
project proponent to commission the preparation of Exp) Option c is correct.
the EIA for its project. The EIA is actually funded by an Statement 1 is correct: Under the GRAM UJALA
agency or individual whose primary interest is to procure programme, high quality LED bulbs will be distributed, at
clearance for the project proposed. This is a major an affordable cost of 10 rupees per bulb in rural areas. The
drawback in EIA as there is a chance of biasedness in the programme is expected to usher in a better standard of
reports or the consultancy might have little expertise in life, financial savings, more economic activity, and better
the area. safety for rural citizens.
Statement 4 is incorrect – The 2006 notification of EIA Statement 2 is correct: Under phase 1 of GRAM UJALA
has decentralised the environmental clearance projects by programme 1 crore 50 lakh LED bulbs will be distributed

Workbook 396
.
ENVIRONMENT

which will have a significant impact on India’s climate to authorized rag-pickers or waste collectors or local
change action energy savings of 2025 million kWh/year bodies.
and CO2 reductions of 1.65 million T CO2/year. The Rules are now applicable beyond Municipal areas
Statement 3 is incorrect: The programme is launched and extend to urban agglomerations, census towns,
by the Ministry Power & New and Renewable Energy notified industrial townships, areas under the control
(and not the Monistry of Rural Development. It will of Indian Railways, airports, airbase, Port and harbour,
be implemented by the Convergence Energy Services defence establishments, special economic zones, State and
Limited (CESL), a wholly owned subsidiary of Energy Central government organizations, places of pilgrims,
Efficiency Services Limited (EESL). religious & historical importance. (Statement b is correct)
112. Solution (c) The bio-degradable waste should be processed,
Exp) Option c is correct. treated and disposed of through composting or bio-
In order to strengthen the implementation of methanation within the premises as far as possible
environmentally sound management of hazardous (Statement d is correct). The residual waste shall be given
waste in the country Hazardous and Other Wastes to the waste collectors or agency as directed by the local
(Management& Transboundary Movement) authority.
Amendment Rules, 2019 were released. The source segregation of waste has been mandated
The amendment has been done keeping into consideration (Statement a is correct) to channelize the waste to wealth
the “Ease of Doing Business” and boosting “Make in by recovery, reuse and recycle.
India” initiative by simplifying the procedures under the Integration of waste pickers/ rag pickers and waste
Rules, while at the same time upholding the principles of dealers/ Kabadiwalas in the formal system should be
sustainable development and ensuring minimal impact done by State Governments, and Self Help Group, or any
on the environment. other group to be formed.
Some of its salient features are: No person should throw, burn, or bury the solid waste
Option a is correct. Exporters of silk waste have now generated by him, on streets, open public spaces outside
been given exemption from requiring permission from his premises, or in the drain, or water bodies.
the Ministry of Environment, Forest and Climate Change.
Generator will have to pay ‘User Fee’ to waste collector
Option b is correct. Electrical and electronic assemblies and for ‘Spot Fine’ for Littering and Non-segregation.
and components manufactured in and exported from
India, if found defective can now be imported back into 114. Solution (d)
the country, within a year of export, without obtaining
Exp) Option d is correct
permission from the Ministry of Environment, Forest and
Climate Change. Statement 1 is correct. A deposit refunds scheme
Option c is incorrect. Solid plastic waste has been (DRS) works by asking for a payment when something
prohibited from import into the country including in is purchased or used, which is then paid back upon its
Special Economic Zones (SEZ) and by Export Oriented return. It has been introduced in the scheme.
Units (EOU). Statement 2 is correct. It is not applicable for micro
Option d is correct. Industries which do not require enterprises, but it is applicable for small enterprises as
consent under Water (Prevention and Control of they continue to be a major source to e-waste pollution.
Pollution) Act 1974 and Air (Prevention and Control of Statement 3 is correct. Provision for Pan India EPR
Pollution) Act 1981, are now exempted from requiring Authorization by CPCB has been introduced replacing
authorization also under the Hazardous and Other Wastes the state wise EPR authorization.
(Management &
Transboundary Movement) Rules, 2016, provided that 115. Solution (b)
hazardous and other wastes generated by such industries Exp) Option b is correct.
are handed over to the authorized actual users, waste
Statement 1 is incorrect. Shopkeepers and street vendors
collectors or disposal facilities.
are allowed to sell commodities in plastic bags but they
113. Solution (c) should register themselves.
Exp) Option c is correct. Statement 2 is correct. These rules shall apply to
Salient features of Solid Waste Management Rules, 2016 every waste generator, local body, Gram Panchayat,
manufacturer, Importers and producer. In 2011 rules,
Responsibilities of Generators have been introduced
these rules applied only to municipal area.
to segregate waste in to three streams (Statement c is
therefore incorrect), Wet (Biodegradable), Dry (Plastic, Statement 3 is incorrect. CPCB has been mandated to
Paper, metal, wood, etc.) and domestic hazardous wastes formulate the guidelines for thermosetting plastic (the
(diapers, napkins, empty containers of cleaning agents, plastic which are difficult to recycle). In the 2011 rules,
mosquito repellents, etc.) and handover segregated wastes there was no specific provision for such type of plastic.

397 Workbook
.
ENVIRONMENT

116. Solution (c) 119. Solution (b)


Exp) Option c is correct. Exp) Option b is correct.
Statement 1 is correct: The Biotech-PRIDE (Biotech Bio-Medical Waste Management Rules, 2016- The
Promotion of Research and Innovation through Data objective of the rules is to properly manage the per day
Exchange) guidelines aim at providing a well-defined bio-medical waste from Healthcare Facilities (HCFs)
framework and guiding principle to facilitate and across the country.
enable sharing and exchange of biological knowledge, Statement in Option a is correct. It defines biomedical
information and data. It is specifically applicable to high- waste as human and animal anatomical waste, treatment
throughput, high-volume data generated by research apparatus like needles, syringes and other materials used
groups across the country. in health care facilities in the process of treatment and
Statement 2 is incorrect: These guidelines do not research.
deal with generation of biological data per se but is an Statement in Option b is incorrect. Bio-medical waste
enabling mechanism to share and exchange information has been classified into 4 categories instead of the
and knowledge generated as per the existing laws, rules, earlier 10 categories to improve the segregation of waste
regulations and guidelines of the country. at source.
Statement 3 is correct: These guidelines will ensure data Statement in Option c is correct. The State Government
sharing benefits viz. maximizing use, avoiding duplication, will provide the land for setting up common bio-medical
maximized integration, ownership information, better waste treatment and disposal facilities.
decision-making and equity of access. They will act as Statement in Option d is correct. It seeks to establish a
the enabling mechanism for sharing the data publicly and Bar-Code System for bags or containers containing bio-
within a reasonable period of time after data-generation, medical waste for disposal.
thus the utility of the data will be maximal.
Statement 4 is correct: These Guidelines will be
implemented through Indian Biological Data Centre
(IBDC) at Regional Centre for Biotechnology supported
by Department of Biotechnology.
117. Solution (d)
Exp) Option d is correct.
Plastic Waste Management Rules, 2016
Option a is correct. Increase minimum thickness of
plastic carry bags from 40 to 50 microns and stipulate
minimum thickness of 50 micron for plastic sheets also to
facilitate collection and recycle of plastic waste.
Option b is correct. Expand the jurisdiction of
applicability from the municipal area to rural areas,
because plastic has reached rural areas also.
Option c is correct. To bring in the responsibilities 120. Solution (d)
of producers and generators, both in plastic waste Exp) Option d is correct.
management system and to introduce collect back system Statement 1 is incorrect. The Compensatory
of plastic waste by the producers/brand owners, as per Afforestation Fund (CAMPA) Act, 2016, sets up a national
extended producers’ responsibility. Compensatory Afforestation Fund (CAF) under the
Option d is incorrect. Nothing as such has been non-lapsable Public Account of India at the Central
stipulated in the rules. level. All money collected by the State Governments and
Union territory Administrations is transferred to the
118. Solution (c) National Compensatory Afforestation Fund.
Exp) Option c is correct. Statement 2 is incorrect. The governing body of the
Statement 1 is correct. Biomedical Waste Management National Compensatory Afforestation Fund Management
Rules 2016 aims to phase out the use of chlorinated and Planning Authority is headed by the Minister
plastic bags, gloves and blood bags within two years. for Environment, Forest and Climate Change. The
Statement 2 is correct. According to the rules, State governing body of the National Authority is assisted by
Government is to provide land for setting up common the executive committee and the monitoring group.
bio-medical waste treatment and disposal facility. Also, Knowledge Base: Forests are routinely diverted for non-
no occupier shall establish on-site treatment and disposal forest and developmental purposes. In such cases, the
facility, if a service of common bio-medical waste Forest (Conservation) Act of 1980 requires that non-
treatment facility is available at a distance of seventy-five forest land, equal to the size of the forest being diverted,
kilometer. is afforested.

Workbook 398
.
ENVIRONMENT

121. Solution (b) for any act done under the provisions of this Act or under
Exp) Option b is correct. the powers vested in them or functions assigned to them
under this Act.
Statement 1 is incorrect. CRZ-I includes the area
between Low Tide Line and High Tide Line. The 2011 Statement 3 is incorrect. The Environment (Protection)
CRZ notification allows projects (in CRZ-I) relating to Act, 1986 has relaxed the rule of “Locus Standi” and
the Department of Atomic Energy; pipelines, conveying because of such relaxation even a common citizen can
systems including transmission; weather systems; approach the court for the alleged offences under this Act.
exploration and extraction of natural gas; construction of 125. Solution (c)
dispensaries, schools etc.
Exp) Option c is correct.
Statement 2 is correct. The CRZ-IV areas include the
aquatic area from low tide line up to territorial limits Statement 1 is correct. The Wildlife (Protection) Act,
including the area of the tidal influenced water body. 1972, was amended in 1991 to transfer sections 61 and
There is no restriction on the traditional fishing and allied 62 to the Centre for declaring certain wild animals to be
activities undertaken by local communities in CRZ-IV. vermin. Now the State Governments are not empowered
However, no untreated sewage, effluents or solid waste to declare any wild animal vermin.
shall be let off or dumped in these areas. Statement 2 is correct. The schedule VI has been added
to include the specified plant species to be protected by
122. Solution (b) the Wildlife (Protection) Amendment Act of 1991. The
Exp) Option b is correct. rating of the Schedules I to V is in accordance with the
Statement 1 is incorrect. The Scheduled Tribes and Other risk of survival of the wildlife (fauna) enlisted in them.
Traditional Forest Dwellers (Recognition of Forest Rights) Animals included in these schedules are provided for total
Act, 2006, provides for the restitution of deprived forest protection from hunting and the trade and commerce
rights for both individual rights to cultivated land in related to such animals are strictly regulated.
forestland and community rights over common property 126. Solution (c)
resources.
Exp) Option c is correct.
Statement 2 is correct. The Ministry of Tribal Affairs is
the nodal agency for its implementation. Statement 1 is incorrect. With the amendment of the
act in 1991, the power of state government has been
Statement 3 is incorrect. National Parks and Sanctuaries withdrawn. State governments are not empowered
have been included along with Reserve Forest, Protected to declare an animal vermin and include it into the
Forests for the recognition of Rights. Schedule 5 of the act. The animal is declared vermin by
123. Solution (d) the central government only which can be hunted for a
certain period of time.
Exp) Option d is correct.
Statement 2 is correct. The act has also made provisions
Statement 1 is correct. The Biological Diversity Act, 2002 for security of wildlife from diseases. As per the act it has
provides for: been made compulsory to immunize the livestock living
1. Conservation of biological diversity, within 5km radius of national park or wildlife sanctuary.
2. Sustainable use of its components, Statement 3 is correct. Plant species which are listed
3. Fair and equitable sharing of the benefits arising out in Schedule 6 of the act cannot be cultivated, collected,
of the use of biological resources, knowledge and for extracted or traded.
matters connected therewith or incidental thereto.
Statement 2 is correct. The Act envisages a three-tier 127. Solution: (d)
structure to regulate access to the biological resources Exp) Option d is the correct answer
National Biodiversity Authority (NBA) at central level Many developments today are occurring on the border
State Biodiversity Boards (SBB) at state level. areas in the rivers or transboundary natural creations. To
make this development sustainable, following convention
Biodiversity Management Committees (BMC) at the has been signed:
local level.
Statement 1 is incorrect – It is not related specifically
124. Solution (b) to World War II. It was signed in 1991 in Espoo,
Exp) Option b is correct. Finland and came into force in 1997. The Convention on
Environmental Impact Assessment in a Transboundary
Statement 1 is correct. Under the Act, there will be Context (informally called the Espoo Convention) is
supremacy of provision. In other words, the provisions of a United Nations Economic Commission for Europe
this Act and the rules or orders made under this Act shall (UNECE) convention. The Convention sets out the
have effect and supremacy over anything inconsistent obligations of Parties—that is States that have agreed to be
contained in any enactment other than this Act. bound by the Convention—to carry out an environmental
Statement 2 is correct. The Environment (Protection) Act, impact assessment of certain activities at an early stage of
1986 grants immunity to the officers of the Government planning. It also lays down the general obligation of States

399 Workbook
.
ENVIRONMENT

to notify and consult each other on all major projects Statement 2 is correct - It is the responsibility of the
under consideration that are likely to have a significant project proponent to commission the preparation
adverse environmental impact across boundaries. of the EIA for its project. The EIA is actually funded
Statement 2 is incorrect – India has not yet signed or by an agency or individual whose primary interest is to
ratified the convention. As of April 2014, the treaty had procure clearance for the project proposed. This is a major
been ratified by 44 states and the European Union. Also drawback in EIA as there is a chance of biasedness in the
the Environmental Impact Assessment came in 1976- reports or the consultancy might have little expertise in
77 and was later notified in India under Environmental the area.
Protection Act, 1986. Statement 3 is correct - There is no accreditation of
EIA consultants, therefore any such consultant with a
128. Solution: (a)
track record of fraudulent cases cannot be held liable
Exp) Option a is the correct answer for discrepancies. It is hard to imagine any consultant
Besides EIA, the Government of India under Environment after being paid lakh of rupees, preparing a report for
(Protection) Act 1986 issued a number of other the project proponents, indicating that the project is not
notifications, which are related to environmental impact viable.
assessment. These are limited to specific geographical
areas. They are 130. Solution: (d)
• Prohibiting location of industries except those related Exp) Option d is the correct answer
to Tourism in a belt of 1 km from high tide mark The Environmental Impact Assessment under
from the Revdanda Creek up to Devgarh Point (near Environmental Protection Act, 1986 has created several
Shrivardhan) as well as in 1 km belt along the banks bodies and committees for efficient assessment. Details
of Rajpuri Creek in Murud Janjira area in the Raigarh are as follows:
district of Maharashtra (1989)
Statement 1 is incorrect - A State Level Environment
• Restricting location of industries, mining Impact Assessment Authority (SEIAA) shall be
operations and regulating other activities in Doon constituted by the Central Government under sub-
Valley (1989) (Statement 1 is correct) section (3) of section 3 of the Environment (Protection)
• Regulating activities in the coastal stretches of the Act, 1986 comprising of three Members including a
country by classifying them as coastal regulation zone Chairman and a Member – Secretary to be nominated
and prohibiting certain activities (1991) by the State Government or the Union territory
• Restricting location of industries and regulating other Administration concerned. The Member-Secretary shall
activities in Dahanu Taluka in Maharashtra (1991) be a serving officer of the concerned State Government.
• Restricting certain activities in specified areas of The names are forwarded by the state governments to
Aravalli Range in the Gurgaon district of Haryana centre which henceforth constitutes the committee.
and Alwar district of Rajasthan (1992) (Statement Statement 2 is incorrect - The SEAC shall be constituted
2 is correct) by the Central Government in consultation with
• Regulating industrial and other activities, which the concerned State Government. The SEAC shall
could lead to pollution and congestion in an be reconstituted after every three years. SEACs shall
area north west of Numaligarh in Assam (1996) function on the principle of collective responsibility. The
(Statement 3 is correct) Chairperson shall endeavour to reach a consensus in each
Statement 4 is incorrect – There is no such specific EIA case, and if consensus cannot be reached, the view of the
for Sundarbans of West Bengal. It is covered by the overall majority shall prevail.
Environmental Impact Assessment . Statement 3 is incorrect – New rating of SEIAAs
has been introduced for encouraging the efficiency,
129. Solution: (b) transparency and accountability in the functioning of
Exp) Option b is the correct answer SEIAAs. The ranking system is based on the provisions
The Environmental Impact assessment suffers from of EIA Notification 2006 and various guidelines issued by
certain drawbacks in its implementation. These are as Ministry from time to time and designed to encourage the
follows: SEIAAs to increase their efficiency. However, there is no
negative marking proposed for not meeting the criteria
Statement 1 is incorrect - Rapid EIA takes data of only
for ranking.
one season and not comprehensive EIA. The difference
between Comprehensive EIA and Rapid EIA is in the 131. Solution: (a)
time-scale of the data supplied. Rapid EIA is for speedier
appraisal process. While both types of EIA require Exp) Option a is correct.
inclusion/ coverage of all significant environmental Statement 1 is correct: The Climate Hazards and
impacts and their mitigation, Rapid EIA achieves Vulnerability Atlas of India, developed by the scientists
this through the collection of one season (other than at Climate Research and Services (CRS) office of the
monsoon) data only to reduce the time required where India Meteorological Department (IMD), Pune. The atlas
comprehensive EIA collects data from all four seasons. features extreme rainfall, drought, cold wave, heatwave,

Workbook 400
.
ENVIRONMENT

dust storms, hail storm, thunderstorm, cyclones, snowfall, and hydropower projects, real estate and other industrial
lightning, winds and fog. The atlas provides a range of projects. The projects are assessed based on their potential
vulnerability with risks ranging from nil, low, moderate, impact on the environment. Based on the assessments,
high and very high categories for every Indian district. they are granted or denied environmental clearance by a
Statement 2 is incorrect: Purba Medinipur in West panel of experts.
Bengal had the highest cyclone vulnerability in the Statement 1 is correct. The Environment Impact
country for cyclones of all intensities. Baleshwar in Assessment in India is statutorily backed by the
Orissa topped the list in terms of vulnerability to severe Environment Protection Act, 1986 which contains various
cyclonic storms. South 24 Parganas was the second-most provisions on EIA methodology and process.
vulnerable to all cyclones, and ranked third in terms of Statement 2 is incorrect. Under the existing rules,
severe cyclonic storms. projects are categorized into Category A and Category
Statement 3 is incorrect: Around 4% of the districts and B. EIA for Category A project requires clearance by
7% of the population in India are highly vulnerable to Environment Ministry and for Category B project, State
cyclones. Most of the coastal districts of the East Coast Environment Impact Assessment Authority (SEIAA)
(not west coast) are highly vulnerable to cyclones. clearance is required. All Category A project needs to
mandatorily undergo the EIA process.
132. Solution: (d)
However, Category-B project are further sub-classified
Exp) Option d is the correct answer into Category B1 and Category B2 projects, depending on
The projects under environmental impact assessment their scope and environmental impact. Amongst the two,
have to go through the public hearing process as people Category-B2 projects have been exempted from the EIA
are the important stakeholder. Following is the method process.
for public hearing:
134. Solution: (c)
Statement 1 is incorrect - The State Pollution Control
Board shall issue a notice for environmental public Exp) Option c is correct.
hearing. Notice shall be published in at least two On the occasion of World Environment Day, the
newspapers widely circulated in the region around the government had announced implementation of the Nagar
project, one of which shall be in the vernacular language van scheme in 2020.
of the locality concerned. State Pollution Control Board Statement 1 is correct. Nagar van scheme aims to develop
shall mention the date, time and place of public hearing. 200 Urban Forests across the country in next five years i.e.,
Suggestions, views, comments and objections of the by 2025 with a renewed focus on people’s participation
public shall be invited within thirty days from the date of and collaboration between Forest Department, Municipal
publication of the notification. bodies, NGOs, Corporates and local citizens.
Statement 2 is incorrect - Category B2 projects doesn’t Statement 2 is incorrect. These urban forests will
require to go for mandatory environmental clearance. primarily be on the existing forest land in the city as well
After 2006 Amendment the EIA, cycle comprises of as on any other vacant land offered by local urban local
four stages 1. Screening 2. Scoping 3. Public hearing 4. bodies.
Appraisal Statement 3 is correct. The finances for the scheme will
Category A projects require mandatory environmental be paid for by the CAMPA (Compensatory Afforestation
clearance and thus we do not undergo the screening Fund (CAF) Act, 2016) funds. It was enacted to manage
process. the funds collected for compensatory afforestation.
Category B projects undergoes screening process and 135. Solution: (b)
they are classified into two types. 1. Category B1 projects
Exp) Option b is correct.
(Mandatory requires EIA). 2. Category B2 projects (Do
not require EIA). Thus, Category A projects and Category National Air Quality Index (AQI) was launched in 2014
B1 projects undergo the complete EIA process whereas with outline ‘One Number – One Color -One Description’
Category B2 projects are excluded from complete EIA for the common man to judge the air quality within his
process. vicinity.
Statement 1 is incorrect. There are six AQI categories,
133. Solution: (a) namely Good, Satisfactory, Moderately polluted, Poor,
Exp) Option a is correct. Very Poor, and Severe. The proposed AQI will consider
Environment impact assessment is a process which eight pollutants (PM10, PM2.5, NO2, SO2, CO, O3, NH3,
prevents industrial and infrastructural projects from and Pb) for which short-term (up to 24-hourly averaging
being approved without proper oversight. This process period) National Ambient Air Quality Standards are
ensures that every project should go through the EIA prescribed.
process for obtaining prior environmental clearance. Statement 2 is correct. The AQI values and corresponding
EIA covers projects such as mining of coal or other ambient concentrations (health breakpoints) as well
minerals, infrastructure development, thermal, nuclear as associated likely health impacts for the identified

401 Workbook
.
ENVIRONMENT

eight pollutants. It transforms complex air quality data water pollution, and (ii) to improve the quality of air and
of eight pollutants into a single number (index value), to prevent, control or abate air pollution in the country.
nomenclature and color. It also provides background air quality data needed for
Statement 3 is correct. The formulation of the index was industrial siting and towns planning.
a continuation of the initiatives under Swachh Bharat
138. Solution: (b)
Mission.
Exp) Option b is the correct answer.
136. Solution: (d) Government of India has enacted the comprehensive
Exp) Option d is correct Environmental Protection Act, 1986 to deal with the
Steel is produced from iron ore or scrap. Iron ore is a environmental degradation and pollution. Some details
mineral aggregate that can be converted economically into are as follows:
iron. The quality of the iron ore is mainly determined by Statement 1 is correct - The environmental impact
its composition; a high iron content and low Sulphur assessment in India was started in 1976-77 when the
and phosphorus contents are favorable. Planning Commission asked the then Department of
Statement 1 is correct: Emissions of Sulphur dioxide Science and Technology to examine the river-valley
are directly related to the combustion of oil in reheating projects from environmental angle. The Government of
furnaces and for coke production. Both are an important India enacted the Environment (Protection) Act on 1986.
process in the production of steel. To achieve the objectives of the Act, one of the decisions
Statement 2 is correct: Furnaces in steel production that were taken is to make environmental impact
require high temperatures which that mean that the assessment statutory.
formation of nitrogen oxides is hard to avoid in Statement 2 is incorrect - The Critical Wildlife Habitats
connection with the combustion of fuel since the nitrogen have been envisaged in Scheduled Tribes and Other
is in the air. Traditional Forest Dwellers (Recognition of Forest
Statement 3 is correct: A very important chemical Rights) Act, 2006. The power to notify the rules to
reaction during steelmaking is the oxidation of carbon. designate a CWH rests with Ministry of Environment and
The by-product of this process is carbon monoxide, a Forests.
phenomenon common to all steelmaking processes and Statement 3 is correct - In India, the Coastal Regulation
very important for mixing. Zone (CRZ) Rules govern human and industrial activity
Statement 4 is correct: The main process steps that close to the coastline, in order to protect the fragile
generate carbon dioxide in iron and steelmaking are the ecosystems near the sea. The coastal regulation zones
production of coke, and the process of melting metal in have been declared by the Ministry of Environment,
the blast furnace. Forest and Climate change under the Environment
Protection Act 1986. While the CRZ Rules are made by
137. Solution: (b) the Union environment ministry, implementation is to be
Exp) Option b is correct. ensured by state governments through their Coastal Zone
Central Pollution Control Board as a regulatory authority Management Authorities.
to environmental protection and regulations in India. The 139. Solution: (b)
enactment of the Environment (Protection) Act, 1986,
widened the scope of activities of the CPCB Exp) Option b is correct.
Statement 1 is correct. Statement 1 is incorrect. 24 State/UTs have raised their
SDRF. These SDRF are placed strategically at suitable
It is under the administrative control of Ministry of Forest, locations well connected to the airport, rail heads and
Ministry of Environment, Forests and Climate Change. roads for their immediate deployment at the disaster sites.
Statement 2 is incorrect. Statement 2 is correct. The Disaster Management Division
The Central Pollution Control Board (CPCB), statutory under the Ministry of Home Affairs is responsible for
organisation, was constituted in September, 1974 under response, relief and preparedness for natural calamities
the Water (Prevention and Control of Pollution) Act, and man-made disasters (except drought and epidemics).
1974. The Division is also responsible for legislation, policy,
Statement 3 is correct. capacity building, prevention, mitigation and long-term
Further, CPCB was entrusted with the powers and rehabilitation.
functions under the Air (Prevention and Control of Statement 3 is correct. Disaster Management Act,
Pollution) Act, 1981. 2005 aims to manage disasters, including preparation of
Principal Functions of the CPCB, as spelt out in the Water mitigation strategies, capacity-building and more.
(Prevention and Control of Pollution) Act, 1974, and the Definition of a “disaster” in Section 2 (d) of the DM
Air (Prevention and Control of Pollution) Act, 1981, (i) Act states that a disaster means a “catastrophe, mishap,
to promote cleanliness of streams and wells in different calamity or grave occurrence in any area, arising from
areas of the States by prevention, control and abatement of natural or man-made causes.

Workbook 402
.
ENVIRONMENT

140. Solution: (b) For the first time, Rules have been made to distinguish
Exp) Option b is the correct answer. between Hazardous Waste and other wastes. Other
wastes include: Waste tyre, paper waste, metal scrap, used
India has taken steps like Star rating and Network electronic items, etc. and are recognized as a resource
on Climate change assessment for promoting energy for recycling and reuse. These resources supplement the
efficiency and climate research respectively. Details of industrial processes and reduce the load on the virgin
these are as follows: resource of the country.
Statement 1 is incorrect - The Star Labelling Programme It comprises the waste generated during the
has been formulated by the Bureau of Energy Efficiency manufacturing processes of the commercial products
(not by Bureau of Indian Standards) under the Energy such as industries involved in petroleum refining,
Conservation Act, 2001. A key objective of this scheme production of pharmaceuticals, petroleum, paint,
is to provide the consumer an informed choice about the aluminium, electronic products etc.
energy saving and thereby the cost saving potential of the
relevant marketed product. The scheme targets display Statement 3 is correct.
of energy performance labels on high-energy end-use State Pollution Control Board (SPCB) is mandated to
equipment & appliances and lays down minimum energy prepare an annual inventory of the waste generated;
performance standards. waste recycled, recovered, utilised including co-processed;
Statement 2 is correct - The Indian Network on Climate waste re-exported and waste disposed and submit to
Change Assessment (INCCA) was launched in October the Central Pollution Control Board by the 30th day of
2009 by the Ministry of Environment and Forests September every year.
(MoEF) in an effort to promote domestic research on 142. Solution: (d)
climate change, and build on the country’s climate change
expertise. INCCA is a network-based programme of the Exp) Option d is correct.
MoEF, which consists of over 120 institutions and over The Scheduled Tribes and Other Traditional Forest
250 scientists country wide is aimed at bringing in more Dwellers (Recognition of Forest Rights) Act, 2006, (FRA)
science-based policy-making, based on measurements, is a result of the protracted struggle by the marginal and
monitoring and modelling. tribal communities of our country to assert their rights
over the forestland over which they were traditionally
141. Solution: (b) dependent.
Exp) Option b is correct. Statement 1 is correct. The Act provides rights to
Hazardous waste means any waste, which by reason of allocation of forest land for developmental purposes to
characteristics, such as physical, chemical, biological, fulfil basic infrastructural needs of the community. In
reactive, toxic, flammable, explosive or corrosive, causes conjunction with the Right to Fair Compensation and
danger to health, or environment. Transparency in Land Acquisition, Rehabilitation and
Scientific disposal of hazardous waste through collection, Settlement Act, 2013, FRA protects the tribal population
storage, packaging, transportation and treatment, in an from eviction without rehabilitation and settlement.
environmentally sound manner minimises the adverse Statement 2 is correct. The process for determining the
impact on human health and on the environment. nature and extent of both individual forest rights and
Statement 1 is correct. community forest rights is delegated to Gram Sabhas.
The Act further enjoins upon the Gram Sabha and rights
The following items have been prohibited for import: holders the responsibility of conservation and protection
Waste edible fats and oil of animals, or vegetable origin; of bio-diversity, wildlife, forests, adjoining catchment
Household waste; areas, water sources and other ecologically sensitive areas.
It can also stop any destructive practices affecting these
Critical Care Medical equipment;
resources or cultural and natural heritage of the tribals.
Tyres for direct re-use purpose;
Statement 3 is correct. The FRA is applicable in National
Solid Plastic wastes including Pet bottles; Parks, Wildlife Sanctuaries, and Tiger Reserves. This
Waste electrical and electronic assemblies scrap; act defines ‘forest land’ under Section 2 (d) as “land of any
Other chemical wastes especially in solvent form. description falling within any forest area and includes
sanctuaries and National Parks”. FRA only recognises
Scarcity of critical care medical equipment has forced India pre-existing rights which are already being exercised by
to allow entry of imported used life-saving equipment for the eligible persons in the National Parks and Sanctuaries.
re-use due to the prevailing Covid-19 situation even as it Other than securing the tenure of the existing forest
falls under the prohibited category as ‘hazardous waste. dwellers on the land, no new rights are being created
Considering possible use of such equipment during the which might potentially impact the ecological balance
pandemic, the environment ministry has given “one-time inside the protected areas.
permission” to allow such used critical care equipment
for re use. 143. Solution: (a)
Statement 2 is incorrect. Exp) Option a is correct.

403 Workbook
.
ENVIRONMENT

Joint Forest Management (JFM) is partnership involving Mega-diverse countries thus have a high biodiversity
both the forest departments and local communities in within their borders. This means there are many more
natural forest management. The concept was introduced animal and plant species living in these countries than
by Government of India through the National Forest in other parts of the world. However, there is no criteria
Policy of 1988. regarding the existence of 5 biodiversity hotspots.
Under JFM, village communities are entrusted with 146. Solution: (d)
the protection and management of nearby forests.
The communities are required to organize forest Exp) Option d is correct.
protection committees, village forest committees, village The Wildlife Trust of India (WTI) is committed to the
forest conservation and development societies, etc. protection of India’s wildlife; it achieves this by working
Each of these bodies has an executive in partnership with local communities and governments
committee that manages its day-to-day affairs. on a range of projects, from species rehabilitation to the
In return of their services to the forests, the communities prevention of the illegal wildlife trade.
get the benefit of using minor non-timber forest produce. Statement 1 is incorrect. Wildlife Trust of India is not a
As a result, the forest can be conserved in a sustainable statutory body, rather an NGO founded in 1998 with the
manner. aim of conserving nature, especially endangered species
144. Solution: (c) and threatened habitats, in partnership with communities
and governments.
Exp) Option c is correct.
Statement 2 is incorrect. Wildlife Crime Control Bureau
Wetland (Conservation and Management) Rules, 2017, (WCCB) is a statutory body but not set up under the
notified by the environment ministry, decentralise Prevention of Cruelty to Animals Act of 1960, but by
wetlands management by giving states powers to not only
amending the the Wild life (Protection) Act of 1972 in
identify and notify wetlands within their jurisdictions but
2007. It complements the efforts of state government,
also keep a watch on prohibited activities.
primary enforcers of the wildlife (protection) Act 1972
Statement 1 is incorrect. It provides for the State Wetlands and other enforcement agencies of the country.
Authority or Union Territory Wetlands Authority to
prepare a comprehensive digital inventory of all wetlands
within a specified period. The State Wetlands Authority or 7.4. Indian Environmental bodies,
Union Territory Wetlands Authority has to prepare a list organisations and reports
of all wetlands of the State or Union Territory within three 147. Solution (c)
months from the date of publication of these rules.
Exp) Option c is correct.
Statement 2 is incorrect. Salt pans as ‘wetlands’ have
been omitted from the Wetland (Conservation and Option c is correct. The Wildlife (Protection) Act of 1972
Management) Rules, 2017. They were identified as provides the basic framework to ensure the protection and
wetlands in the 2010 Rules, as they are often important management of wildlife. It provides for the appointment
sites of migratory birds and other forms of biodiversity. of wildlife advisory Board, Wildlife warden, their powers,
The omission in the 2017 Rules suggests that while duties etc.
saltpans do exist as wetlands, they do not require any Following organisations were established under the act:
conservation or ecological balance. National Parks, Wildlife Sanctuaries etc.
145. Solution: (b) Constitution of Central Zoo Authority.
Exp) Option b is correct Constitution of National Board for Wildlife
‘Megadiversity’ means there is extreme high biodiversity. National Tiger Conservation Authority
All the countries that are called ‘megadiverse’ have a very Wildlife Crime Control Bureau
high biodiversity, that means many different species of The Animal Welfare Board of India was established in
animals and plants exist in the country. This concept was
1962 under Section 4 of the Prevention of Cruelty to
first proposed in 1988 by Russell Mittermeier.
Animals Act, 1960. It is a statutory advisory body on
Option b is correct. Megadiverse countries are all the Animal Welfare Laws and promotes animal welfare in the
countries that house a large number of endemic species. country.
‘Endemic’ means species that only exist in this country.
There are 17 megadiverse countries in the world. There 148. Solution (d)
are two basic conditions to be met by country for it to be Exp) Option d is correct.
called megadiverse:
Relevance) The National Board for Wildlife has not met
The country must have a minimum of 5,000 endemic even once in the past five years with all its 47 members
plants in attendance, the Union ministry of environment, forest
The country must also have a marine ecosystem within its and climate change ministry informed the Rajya Sabha
borders which means the country must border an ocean. recently.

Workbook 404
.
ENVIRONMENT

Statement 1 is correct. The National Board is a statutory 151. Solution (d)


body under the Wildlife (Protection) Act, 1972. An Exp) Option d is correct
amendment was made to the Wildlife (Protection) Act,
1972, in 2002. By which the National Board for wildlife Statement 1 is correct: Central Zoo Authority was
established in 1992 by Amendment of Wildlife
was established in 2003 by replacing Indian Board for
Protection Act 1991. The act authorizes this body to give
Wildlife.
recognition to zoos. Every zoo in the country needs to
Statement 2 is correct. The National Board for Wildlife get recognition from the Authority for its operations.
has 47 members with the Prime Minister in the Chair.
Statement 2 is correct: It also identifies endangered
The Minister in charge of the Ministry of Environment
wildlife species for purpose of captive breeding and
& Forests in the Central Government is the Vice-
assigning responsibility in this regard to a zoo.
Chairperson. The Additional Director General of Forests
(WL) & Director, Wildlife Preservation is the Member- Statement 3 is correct: Central Zoo Authority also
Secretary to the Board. The Board is responsible for regulates the exchange of endangered species, listed in
promotion of conservation and development of wildlife Schedule-I and II of the Wildlife Protection Act 1972,
and forests. among the zoos in India. It also gives permission for
exchange of animals between India and foreign zoos
149. Solution (a) before clearance under EXIM Policy and CITES permits
Exp) Option a is correct. are issued.
The India State of Forest Report 2019, a biennial 152. Solution (a)
exercise, is prepared by Forest Survey of India (FSI).
Exp) Option a is correct.
It assesses the forest and tree cover, bamboo resources,
carbon stock and forest fires. Statement 1 is correct. Organizing social and commercial
functions in park can lead to pollution due to use of diesel
Statement 1 is incorrect. The total forest and tree cover
generator sets , also lead to non -compliance of solid waste
in India is 80.73 million hectare which is 24.56 percent of
management rules. Thus NGT can hear these matters and
the geographical area of the country. regulate these activities.
India has set a target of bringing 33% of its geographical Statement 2 is correct. Not utilising treated water
area under the forest cover. results in wasteful expenditure besides wasting a
Statement 2 is correct. Mangrove cover has been scarce natural resource treated at high cost. So NGT
separately reported in the ISFR 2019 and the total can prescribed the use of treated waste water in parks.
mangrove cover in the country is 4,975 sq km. An increase Statement 3 is incorrect. NGT is not empowered to
of 54 sq Km in mangrove cover has been observed as hear matters pertaining to issues coming under the
compared to the previous assessment of 2017. Top three ambit of the Wildlife (Protection) Act, 1972, which
states showing mangrove cover increase are Gujarat (37 is applicable in case of National Parks, Sanctuaries
sq km) followed by Maharashtra (16 sq km) and Odisha and Tiger Reserves. So NGT can’t hear matters relating
(8 sq km). to protection of National marks and sanctuaries from
Statement 3 is incorrect. The sharpest decline in forest proposed Dam construction and widening of highways.
cover was seen in the north-eastern States of Arunachal
153. Solution (b)
Pradesh, Manipur and Mizoram. There has been a
decrease of forest cover to the extent of 765 sq km (0.45%) Exp) Option b is correct
in the region. Except Assam and Tripura, all the States Statement 1 is correct. National Green Tribunal,
in the region show decrease in forest cover. The loss established in 2010, as per the National Green Tribunal
is attributed primarily due to the traditional farming Act is a statutory and quasi judicial body.
practice of shifting cultivation. Statement 2 is correct. NGT can exercise suo-
150. Solution (a) motu jurisdiction. In cases of drastic environmental
damage like the gas leak at LG Polymers India plant
Exp) Option a is correct in Visakhapatnam, NGT said it has wide jurisdiction
Statement 1 is incorrect: Animal Welfare Board of India including the power to institute proceedings suo motu (on
is a statutory advisory body on Animal Welfare laws. its own) as it cannot keep its hands tied in cases of drastic
It was set up in 1962 in accordance with Prevention of environmental damage
Cruelty Act 1962. Statement 3 is correct While passing Orders/decisions/
Statement 2 is correct: As part of its functions, it advises awards, the NGT will apply the principles of sustainable
the government or any local authority on improvements development, the precautionary principle and the
in the design of vehicles so as to lessen the burden on polluter pays principles.
draught animals. Statement 4 is incorrect. Any amount of compensation
Statement 3 is incorrect: Shrimati Rukmani Devi or relief under the order made by NGT shall be remitted
Arundale pioneered the setting up of the board with its to Environmental relief fund. This was created under
headquarters at Chennai and not Menaka Gandhi. Public liability insurance act, 1991

405 Workbook
.
ENVIRONMENT

154. Solution (c) Statement 2 is correct. Biological Diversity Act (2002),


Exp) Option c is correct. also establishes Biodiversity Management Committees
(BMCs) at the local level. They are responsible
Statement 1 is correct. The EPCA was constituted under for promoting conservation, sustainable use and
the provisions of the Environment (Protection) Act by the documentation of biological diversity and knowledge
central government in 1998 for the national capital region relating to biological diversity.
Statement 2 is incorrect. Its former chairman was
a former IAS officer. So retired SC judge is not its 157. Solution (b)
chairman. Exp) Option b is correct.
Statement 3 is correct. EPCA can issue directions in Statement 1 is correct. Wildlife Crime Control Bureau
writing to any person, officer or authority. If its directions is a statutory body established under the Wildlife
are not followed, it has powers to file criminal complaints (Protection) Act, 1972. The bureau complements the
(under section 19 of the Act) before courts. efforts of the state governments, primary enforcers of the
Statement 4 is correct. It has been replaced by the Wildlife (Protection) Act, 1972, and other enforcement
Commission for Air Quality Management in National agencies of the country.
Capital Region (NCR) and Adjoining Areas (CAQM). In Statement 2 is correct. The Bureau is mandated to collect
the CAQM, there will be at least six permanent members and collate intelligence related to organized wildlife
and it will be headed by a former or incumbent Secretary crime activities and to disseminate the same to State and
to the Government of India, or a Chief Secretary to a other enforcement agencies for immediate action so as to
State government. It would have members from several apprehend the criminals.
Ministries as well as representatives from the states. Statement 3 is incorrect. Wildlife Crime Control
155. Solution (d) Bureau is multi-disciplinary body established under
the Ministry of Environment and Forests and Climate
Exp) Option d is correct. Change. It is headquartered in New Delhi.
Relevance) SICOM launched program BEAMS in
September, 2020. 158. Solution (a)
Statement 1 is correct. SICOM is the nodal agency for Exp) Option a is correct.
strategic planning, management, execution, monitoring Statement 1 is correct. EPCA was constituted with the
and successful implementation of Integrated Coastal objective of ‘protecting and improving’ the quality
Zone Management (ICZM) practices in across the of the environment and ‘controlling environmental
country and National Project Management Unit pollution’ in the National Capital Region. The EPCA
(NPMU) for the ICZM Project and ENCORE (Enhancing also assists the apex court in various environment-related
Coastal and Ocean Resource Efficiency) project in all the matters in the region.
13 Coastal States/UTs. Statement 2 is incorrect. EPCA was constituted under
Statement 2 is correct. SICOM in pursuit of promoting section 3 of the Environment (Protection) Act, 1986 by a
its policies for sustainable development in coastal notification of MoEFCC for the National Capital Region
regions have embarked upon a highly acclaimed in compliance with the Supreme Court order dated
program BEAMS (Beach Environment & Aesthetics January 1998. Hence, it is not a statutory body.
Management Services) under its ICZM project. This is
one of the several projects of ICZM that Govt of India is 159. Solution (b)
undertaking for the sustainable development of coastal Exp) Option b is correct.
regions, striving for globally recognized and the coveted NGT is a specialised body set up under the National
eco-label ‘Blue flag’. Green Tribunal Act (2010) for effective and expeditious
Statement 3 is correct. SICOM works for vibrant, disposal of cases relating to environmental protection
healthy and resilient Coastal and Marine Environment and conservation of forests and other natural resources.
for continuous and enhanced outflow of benefits to The Tribunal comprises of the Chairperson, the Judicial
the Country and the Coastal Community. It provides Members and Expert Members. They shall hold office for
guidelines for coastal and marine ecological security and term of five years and are not eligible for reappointment.
livelihood security of coastal and island communities. The Chairperson is appointed by the Central Government
in consultation with Chief Justice of India (CJI).
156. Solution (c)
Statement 1 is incorrect.
Exp) Option c is correct.
The NGT deals with civil cases under the seven laws
Statement 1 is correct. The National Biodiversity related to the environment, these include:
Authority (NBA) was established in 2003 to implement
India’s Biological Diversity Act (2002). The NBA The Water (Prevention and Control of Pollution) Act,
is a Statutory, Autonomous Body and it performs 1974,
facilitative, regulatory and advisory function for the The Water (Prevention and Control of Pollution) Cess
Government of India. Act, 1977,

Workbook 406
.
ENVIRONMENT

The Forest (Conservation) Act, 1980, Statement 1 is correct. India piloted a resolution on
The Air (Prevention and Control of Pollution) Act, 1981, ‘’Addressing Single Use Plastic Product Pollution’’ which
was adopted by the Fourth United Nations Environment
The Environment (Protection) Act, 1986, Assembly held in 2019. The resolution recognizes the
The Public Liability Insurance Act, 1991 and need for the global community to act on the single use
The Biological Diversity Act, 2002. plastic products pollution. The adoption of this resolution
was a significant step.
Statement 2 is correct. NGT is mandated to make
disposal of applications or appeals finally within 6 Statement 2 is incorrect. In August 2021, the Ministry of
months of filing of the same. The Tribunal is not bound Environment, Forest and Climate Change, Government of
by the procedure laid down under the Code of Civil India, notified the Plastic Waste Management Amendment
Procedure 1908, but shall be guided by principles of Rules, 2021 prohibiting identified single use plastic
‘natural justice’. items, which have low utility and high littering potential,
by 2022. Hence, only specific single use plastics are
Statement 3 is correct. The NGT has five places of banned. The manufacture, import, stocking, distribution,
sittings, New Delhi is the Principal place of sitting and sale and use of identified single-use plastic, including
Bhopal, Pune, Kolkata and Chennai are the other four. polystyrene and expanded polystyrene, commodities
shall be prohibited with effect from the July 1, 2022.
160. Solution (c)
Examples of single use plastic includes Plastic bottles,
Exp) Option c is correct.
Plastic packaging materials, Plastic shopping bags.
Statement 1 is incorrect: Delhi Cabinet has approved the etc.,. Examples of polystyrene include rigid trays and
‘Tree Transplantation Policy’ for the preservation of trees containers, disposable eating utensils, and foamed
in the National Capital. cups, plates, and bowls.
Statement 2 is correct: A minimum of 80% of trees Statement 3 is incorrect. In order to stop littering
affected by a development or construction project will due to light-weight plastic carry bags, the thickness of
be transplanted and as much 80% of the transplanted plastic carry bags has been increased from fifty microns
trees must survive after transplantation. Under this policy, to seventy-five microns with effect from September 30,
10 saplings will be planted in addition to the tree being 2021 and to one hundred and twenty microns with effect
dug up with the root intact and scientifically transplanted from December 31, 2022. Increased thickness of plastic
at another location instead of being felled. bags will also allow reuse.
Statement 3 is incorrect: No such provision is made. It Statement 4 is correct. The Ministry of Environment has
costs around Rs 1 lakh to transplant an average-sized tree. launched new rules called the Plastic Waste Management
(Amendment) Rules, 2022.According to the new
161. Solution (d) rule, Extended producer responsibility (EPR) is now
Exp) Option d is correct. applicable to both pre-consumer and post-consumer
plastic packaging waste. Provisions and targets for
Statement 1 is correct. CPCB is a statutory organisation
collection, re-use (by brand owners), recycling (by PIBOs)
which was constituted under the Water (Prevention and
and use of recycled plastic (by PIBOs) have been laid out.
Control of Pollution) Act, 1974.
Statement 2 is correct. It is also entrusted with the powers 163. Solution: (b)
and functions under the Air (Prevention and Control of Exp) Option b is correct.
Pollution) Act, 1981. The National Disaster Management Authority (NDMA) is
Statement 3 is correct. Principal Functions of the CPCB, the apex statutory body for disaster management in India.
as spelt out in the Water (Prevention and Control of Setting up of NDMA and the creation of an enabling
Pollution) Act, 1974, and the Air (Prevention and Control environment for institutional mechanisms at the State and
of Pollution) Act, 1981, District levels is mandated by the Disaster Management
to promote cleanliness of streams and wells in different Act, 2005. NDMA is mandated to lay down the policies,
areas of the States by prevention, control and abatement plans and guidelines for Disaster Management. India
of water pollution, envisions the development of an ethos of Prevention,
to improve the quality of air and to prevent, control or Mitigation, Preparedness and Response.
abate air pollution in the country. Statement 1 is correct. NDMA provide such support
to other countries affected by major disasters as may be
162. Solution: (a) determined by the Central Government
Exp) Option a is the correct answer. Statement 2 is correct. NDMA goes through the plans
India is committed to mitigate pollution caused by littered prepared by the Ministries or Departments of the
single use plastics. In 2018, the Hon’ble Prime Minister Government of India in accordance with the National
announced that India would phase-out single use plastic Plan and is responsible for approving them.
by 2022. Statement 3 is incorrect. The National Disaster

407 Workbook
.
ENVIRONMENT

Management Authority (NDMA), headed by the Activities permitted in ESZs:


Prime Minister of India, is the apex body for Disaster Ongoing agriculture and horticulture practices by local
Management in India. communities,
164. Solution: (d) Rainwater harvesting,
Exp) Option d is correct. Organic farming,
Haiderpur is located within the boundaries of Hastinapur Adoption of green technology and
Wildlife Sanctuary. It is a human-made wetland covering Use of renewable energy sources.
an area of 6,908 hectares and was formed in 1984 after
the construction of the Madhya Ganga Barrage on the 166. Solution: (a)
floodplains of Ganga. It supports more than 15 globally Exp) Option a is correct.
threatened species, such as the critically endangered The Zoological Survey of India (ZSI) was established on
gharial and the endangered hog deer, black-bellied tern, 1st July, 1916 to promote survey, exploration and research
steppe eagle. It supports more than 25,000 waterbirds. It leading to the advancement in our knowledge of various
serves as a breeding site for the near-threatened Indian aspects of exceptionally rich life of India.
grassbird and provides refuge to the northern subspecies
population of the vulnerable swamp deer during its • Statement 1 is correct. Following are the primary
seasonal flood-driven migration. objectives of Zoological survey of India –
Taxonomic studies of all faunal components collected.
Option a is incorrect. Sarsai Nawar is a small unprotected
wetland, enroute to Saman Wildlife Sanctuary, in Etawah • Periodic review of the Status of Threatened and
District of Uttar Pradesh. It has a large population of the Endemic species.
threatened species of Sarus Cranes, the world’s tallest • Preparation of Red Data Book, Fauna of India and
flying birds. It is included in Ramsar list in early 2020. Fauna of States.
Option b is incorrect. Gogabeel is an ox-bow lake in • Bioecological studies on selected important
Bihar’s Katihar district.It has been declared as the state’s communities/species.
first ‘Community Reserve’. It is not included in Ramsar • Preparation of databases for the recorded species of
list. the country.
Option c is incorrect. Nawabganj bird sanctuay is a • Maintenance & Development of National Zoological
haven for birds, with 25,000 waterbirds regularly recorded Collections.
and 220 resident and migratory species documented
• Training, Capacity Building and Human Resource
Among these are globally threatened species including
Development.
the endangered Egyptian vulture and Pallas’s fish eagle.
Monsoon rains feed this diverse wetland while the Sarda • Faunal Identification, Advisory services and Library
Canal supplies additional water. It is situated in Hasanganj Services.
tehsil of Unnao district of Uttar Pradesh, India • Publication of results including Fauna of India and
Fauna of States.
165. Solution: (b)
Statement 2 is incorrect. Animal welfare board of India is
Exp) Option b is correct. a statutory body set up under the Prevention of Cruelty to
Eco-sensitive zone (ESZ) is an area notified by the Animals Act of 1960. It also has been given some financial
Ministry of Environment, Forests and Climate Change powers. It can encourage the formation of Pinjara-poles,
(MoEFCC), around Protected Areas, National Parks rescue homes, animal shelters, sanctuaries by grant of
and Wildlife Sanctuaries. ESZ is declared under the financial assistance. It can also give financial assistance
Environment (Protection) Act 1986. The main aim behind and other assistance to Animal Welfare Organisations,
ESZs is to regulate certain activities and thus minimise the functioning in any local area or to encourage the
negative impacts of such activities on the fragile ecosystem formation of Animal Welfare Organisations in any local
surrounding the protected areas. area which shall work under the general supervision and
Statement 3,4 and 5 is correct. Prohibited activities guidance of the Board.
under the ESZs: 167. Solution: (a)
Commercial mining, Exp) Option a is correct
Saw mills, The National Disaster Management Plan (NDMP)
Industries causing pollution (air, water, soil, noise etc), provides a framework and direction to the government
Establishment of major hydroelectric projects (HEP), agencies for all phases of disaster management cycle.
Commercial use of wood, The NDMP is a “dynamic document” in the sense that
it will be periodically improved keeping up with the
Tourism activities like hot-air balloons, emerging global best practices and knowledge base
Discharge of effluents or any solid waste or production of in disaster management. It is in accordance with the
hazardous substances. provisions of the Disaster Management Act, 2005, the

Workbook 408
.
ENVIRONMENT

guidance given in the National Policy on Disaster The e-waste rules, formerly the E-waste (Management and
Management (NPDM) 2009, and the established Handling) Rules, provide guidelines for the transportation,
national practices. storage and recycling of waste, and also introduced the
Statement 1 is correct. With National Disaster concept of extended producer responsibility (EPR).
Management Plan 2016 India has aligned its National Plan Statement 1 is incorrect.
with the Sendai Framework for Disaster Risk Reduction Deposit Refund Scheme has been introduced as an
2015-2030, to which India is a signatory. additional economic instrument under which the
Statement 2 is incorrect. A National Executive producer charges an additional amount as a deposit at
Committee is constituted under Section 8 of Disaster the time of sale of the electrical and electronic equipment
Management Act, 2005 to assist the National Authority and returns it to the consumer along with interest when
(NDMA) in the performance of its functions. the end-of-life electrical and electronic equipment is
Union Home secretary is its ex-officio chairperson. returned.
NEC has been given the responsibility to act as Statement 2 is correct.
the coordinating and monitoring body for disaster As per the new rules, the Central Pollution Control Board
management, to prepare a National Plan, monitor the (CPCB) was given the overall responsibility of ensuring
implementation of National Policy etc. that firms complied with the power to levy fines for non-
compliance.
168. Solution: (a) The new rules specified that producers would have
Exp) Option a is correct. to ensure collection of 10% of the quantity of e-waste
PRAIVESH Portal is a web based, role-based workflow generation during 2017-18, with a 10% increase every
application which has been developed for online year until 2023 and 70% from 2023 onwards.
submission and monitoring of the proposals submitted by Knowledge Base: For the first time, the Rules will bring
the proponents for seeking Environment, Forest, Wildlife the producers under Extended Producer Responsibility
and CRZ Clearances from Central, State and district level (EPR), along with targets.
authorities. EPR is a known policy tool that requires makers of
Statement a is correct. PARIVESH (Pro-Active and electronics to take financial and/or physical responsibility
Responsive facilitation by Interactive, Virtuous and for managing the disposal of their products after the end
Environmental Single-window Hub) is a Single-Window of their lives.
Integrated Environmental Management System. In 2016, the rules were broadened to introduce a ‘Producer
Statement b is incorrect. PRAGATI (Pro-Active Responsibility Organisation’ (PRO) to help collect and
Governance And Timely Implementation), as the name recycle e-waste, and brought buy-back, deposit refund
suggests, is aimed at starting a culture of Pro-Active and exchange schemes under the EPR.
Governance and Timely Implementation. It is also a robust
system for bringing e-transparency and e-accountability 170. Solution: (a)
with real-time presence and exchange among the key Exp) Option a is correct.
stakeholders. The National Building Code of India (NBC), a
Statement c is incorrect. AarogyaPath Portal is an comprehensive building Code, is a national instrument
integrated public platform that provides single-point providing guidelines for regulating the building
availability of key healthcare goods such as medical construction activities across the country. The Bureau of
equipment, drugs, apparel, etc. It is a Council of Scientific Indian Standards has published it, as a recommendatory
& Industrial Research (CSIR) National Healthcare Supply document. However, the Home Ministry asked States to
Chain Portal. CSIR expects AarogyaPath to become the incorporate it into their local building bylaws. It makes
national healthcare information platform of choice in the the recommendations a “mandatory requirement”.
years to come. Statement 1 is correct.
Statement d is incorrect. It is an overview of the It serves as a Model Code for adoption by all agencies
CHAMPIONS Portal. The CHAMPIONS stands here involved in building construction works be they Public
for Creation and Harmonious Application of Modern Works Departments, other government construction
Processes for Increasing the Output and National departments, local bodies or private construction
Strength. It is a technology driven Control Room-Cum- agencies. The Code mainly contains administrative
Management Information System which utilizes modern regulations, development control rules and general
information and communication technology (ICT) tools. building requirements; fire safety requirements;
stipulations regarding materials, structural design and
169. Solution: (b) construction (including safety); building and plumbing
Exp) Option b is correct. services; approach to sustainability; and asset and facility
The Ministry of Environment, Forest and Climate Change management.
notified the E-Waste Management Rules, 2016. Statement 2 is incorrect.

409 Workbook
.
ENVIRONMENT

It is published by the Bureau of Indian Standards and it is Some of the technological interventions to control stubble
a recommendatory document. burning in India are as under:
Knowledge Base: The National Building Code of India Happy seeder is a machine which does not require a
broadly covers the following areas: stubble-free land to plant wheat. It is being made available
Fire prevention: This covers aspects of fire prevention to farmers at a subsidized cost so that farmers don’t resort
pertaining to the design and construction of buildings. It to stubble burning.
also describes the various types of buildings materials and Pusa bio-decomposer is a solution developed by the
their fire rating. scientists at the Indian Agricultural Research Institute,
Life Safety: This covers life safety provisions in the Pusa, which can turn crop residue into manure in 15 to 20
event of fire and similar emergencies, also addressing days and therefore, can prevent stubble burning.
construction and occupancy features that are necessary to Super SMS (Straw Management System): It cuts and
minimize danger to life from fire, smoke, fumes or panic. spreads the straw in uniform manner in the field at the
Fire Protection: Covers significant appurtenances time of harvesting of paddy.
(accessories) and their related components and Statement 4 is incorrect.
guidelines for selecting the correct type of equipment Genetically engineered (GE) crops provide protection
and installations meant for fire protection of the building, against insects and diseases, or tolerance to herbicides
depending upon the classifications and type of building. and pesticides apart from giving increased yield to the
farmers. They have no role in producing less stubble
171. Solution: (a) after harvesting.
Exp) Option a is correct.
173. Solution: (b)
NITI Aayog in collaboration with International Transport
Forum (ITF) will launch the “Decarbonising Transport in Exp) Option b is correct.
India” project on 24 June, with the intention to develop The NDRF was constituted under Section 44 of the
a pathway towards a low-carbon transport system for Disaster Management Act, 2005 for a specialized response
India. Decarbonising Transport in Emerging Economies to natural and man-made disasters.
(DTEE) supports transport decarbonisation across Statement 1 is incorrect. NDRF was constituted in 2006
different world regions. under the Disaster Management Act for the purpose of
Statement 1 is correct. specialized response to natural and man-made disasters.
The Decarbonising Transport in Emerging Economies As per National Policy on Disaster Management 2009, the
(DTEE) project helps national governments and other State Governments are also required to raise their own
stakeholders to identify transport measures and establish SDRF for quickly responding to disasters.
pathways to reduce transport CO2 emissions and meet Statement 2 is correct. The general superintendence,
their climate goals and NDCs. direction and control of the National Disaster Response
Statement 2 is incorrect. Force (NDRF) is vested in the National Disaster
Management Authority (NDMA). The NDMA, as the
The DTEE is collaboration between the International apex body for disaster management, is headed by the
Transport Forum (ITF) and the Wuppertal Institute, Prime Minister.
supported by the International Climate Initiative (IKI)
of the German Federal Ministry for the Environment, Statement 3 is correct. Battalions are also trained and
Nature Conservation and Nuclear Safety. equipped for response during chemical, biological,
radiological and nuclear (CBRN) emergencies. In the
Current participants: India, Argentina, Azerbaijan, and beginning, the personnel of NDRF were deployed for
Morocco. routine law and order duties also but in 2007 it has been
In India, this ambitious five-year project is being made a dedicated force for disaster response related
implemented with the help of NITI Aayog to develop a duties. It functions under the Ministry of Home Affairs.
pathway towards a low-carbon transport system through
the development of modelling tools and policy scenarios.
7.5. International Environmental
172. Solution: (c) Organisations, Conventions And
Exp) Option c is correct. Initiatives
Stubble burning by farmers in Punjab, Haryana and Uttar 174. Solution (c)
Pradesh contributes anywhere from 17% to 78% to the Exp) Option c is correct.
particulate matter emission load in Delhi during winter.
Statement 1 is correct: SAWEN is an inter-governmental
Agricultural fires, in which farmers set fire to their wildlife law enforcement support body of South Asian
fields after harvesting paddy, tend to begin around late countries namely - Afghanistan, Bangladesh, Bhutan,
September and peak around the last week of October by India, Maldives, Nepal, Pakistan and Sri Lanka. It
which time farmers have harvested most of their paddy. promotes regional cooperation to combat wildlife crime
Statement 1,2 and 3 are correct. in South Asia.

Workbook 410
.
ENVIRONMENT

Statement 2 is correct: SAWEN operates its activities 179. Solution (b)


from the Secretariat based in Kathmandu. Though Exp) Option b is correct.
SAWEN was officially launched in 2011 in Paro, Bhutan.
Option a is correct: ISA was launched at CoP 21 of
175. Solution (b) UNFCCC at Paris in 2015. The purpose of the International
Solar Alliance is to bring countries on a platform that
Exp) Option b is correct.
supports clean energy, sustainable environment, clean
Option b is correct. South Asia Co-operative public transport and clean climate.
Environment Programme (SACEP), established in 1982,
Option b is incorrect: Initially the membership of ISA
is an intergovernmental organization to promote and
was restricted to only those countries which lied partially
support protection, management and enhancement of
or fully in the tropics. But recently India has expanded
the environment in South Asia. The members of SACEP
it to all members of United Nation even beyond the
include Afghanistan, Bangladesh, Bhutan, India,
tropics.
Maldives, Nepal, Pakistan and Sri Lanka.
Option c is correct: Headquarter of ISA is set up at
Headquarters: Colombo, Sri Lanka National Institute of Solar Energy, Gurugram.
176. Solution (c) Option d is correct: ISA has set a target of 1000 GW of
Exp) Option c is correct. installed solar capacity globally by 2030. For which it
has projected the need of 1 trillion dollars.
Statement 1 is correct. UNEP categorizes its work into
seven broad thematic areas – climate change, disasters 180. Solution (d)
and conflict, ecosystem management, environmental Exp) Option d is correct.
governance, chemicals and waste, resource efficiency and
environment under review. Statement 1 is incorrect. The “Commission for Air
Quality Management (CAQM)” is a statutory authority
Statement 2 is incorrect. The Global Environment established through an ordinance. This statutory
Facility (GEF) was established on the eve of the 1992 Rio authority will act as an apex body that will supersede the
Earth Summit to help tackle our planet’s most pressing existing central pollution body and Uttar Pradesh, Delhi,
environmental problems. It is one of the main fund Rajasthan, Haryana, and Punjab state pollution control
providers for the UNEP. It does not operate under UNEP. boards. The statutory authority has been empowered to
Statement 3 is correct. UNEP, established in 1972, is send directives to the aforementioned state government
headquartered at Nairobi, Kenya. regarding air pollution.
177. Solution (a) The Central Pollution Control Board (CPCB) is statutory
organisation constituted in September, 1974 under the
Exp) Option a is correct. Water (Prevention and Control of Pollution) Act, 1974.
Statement 1 is correct. United Nations Environment Further, CPCB is entrusted with the powers and functions
Programme Finance Initiative (UNEP FI) is a partnership under the Air (Prevention and Control of Pollution) Act,
between UNEP and the global financial sector to mobilize 1981.
private sector finance for sustainable development. UNEP Statement 2 is incorrect. CAQM has been established to
FI works with more than 350 members – banks, insurers, tackle the air pollution menace in the National Capital
and investors – and over 100 supporting institutions – to Region and Adjoining areas.
help create a financial sector that helps in achieving the
Principal Functions of the CPCB, as spelt out in the Water
2030 Agenda for Sustainable Development and Paris
(Prevention and Control of Pollution) Act, 1974, and the
Agreement on Climate Change. Air (Prevention and Control of Pollution) Act, 1981, (i)
Statement 2 is incorrect. The UNEP Statement of to promote cleanliness of streams and wells in different
Commitment by Financial Institutions on Sustainable areas of the States by prevention, control and abatement
Development was the original backbone of the Finance of water pollution, and (ii) to improve the quality of air
Initiative when it was created in the wake of the Rio Earth and to prevent, control or abate air pollution in the entire
Summit in 1992. country.
178. Solution (c) 181. Solution (a)
Exp) Option c is correct. Exp) Option a is correct.
Statement 1 is correct: The United Nations General Blue Carbon refers to coastal, aquatic and marine carbon
Assembly declared 2021 – 2030 the UN Decade on sinks held by the indicative vegetation, marine organism
Ecosystem Restoration. It aims to scale up the restoration and sediments.
of degraded and destroyed ecosystems as a proven Statement 1 is correct. The Blue Carbon Initiative is
measure to fight the climate crisis and enhance food a global program working to mitigate climate change
security, water supply and biodiversity. through the restoration and sustainable use of coastal
Statement 2 is correct: UN Environment and FAO will and marine ecosystems. The Initiative currently focuses
lead the implementation. on mangroves, tidal marshes and seagrasses. The Blue

411 Workbook
.
ENVIRONMENT

Carbon Initiative brings together governments, research 183. Solution (a)


institutions, non-governmental organizations and Exp) Option a is correct.
communities from around the world.
Statement 1 is correct. The PPCA aims to encourage a
Statement 2 is incorrect. The Initiative is coordinated global moratorium on the construction of new unabated
by Conservation International (CI), the International coal-fired power plants. The PPCA also encourages all
Union for Conservation of Nature (IUCN), and the members to endorse the PPCA Declaration - including
Intergovernmental Oceanographic Commission of the a commitment to phase out coal by 2030 in the OECD
United Nations Educational, Scientific, and Cultural and EU countries, and by no later than 2050 in the rest
Organization (IOC-UNESCO). of the world.
182. Solution (c) Statement 2 is incorrect. India is not a founder-member
of the PPCA. It was launched by the UK and Canadian
Exp) Option c is correct.
governments at COP23 in 2017. It is a coalition of
Statement 1 is incorrect: Climate and Clean Air national and sub-national governments, businesses and
Coalition (CCAC) and the United Nations Environment organisations working to advance the transition from
Programme (UNEP) have released a report titled “Global unabated coal power generation to clean energy.
Methane Assessment: Benefits and Costs of Mitigating
Methane Emissions”. 184. Solution (b)
Statement 2 is incorrect: Methane in the atmosphere Exp) Option b is correct.
reached record levels in 2020. Statement 2 is correct whereas Statements 1 and 3 are
The annual increase in atmospheric methane for 2020 was incorrect.
14.7 parts per billion (ppb), which is the largest annual Extinction Rebellion, also referred to as ‘XR’, is a global
increase recorded since 1983. movement which describes itself as a “decentralised,
international and politically non-partisan movement
using non-violent direct action and civil disobedience to
persuade governments to act justly on the Climate and
Ecological Emergency”.
XR was launched in the United Kingdom on October
31, 2018, as a response to a report by the United Nations
Intergovernmental Panel on Climate Change (IPCC),
which declared that “we only have 12 years to stop
catastrophic climate change and our understanding that
we have entered the 6th mass extinction event”. The
movement now has a presence in 75 countries, including
India.
The group has “three core demands” of governments
around the world. It wants governments to “Tell the
Truth”, to “Act Now”, and to “Go Beyond Politics” in order
to confront the climate and ecological emergency that the
world is faced with.
Statement 3 is correct: Source of Methane Emissions: It wants them to communicate the urgency to bring
More than half of global methane emissions stem from change, and reduce greenhouse gas emissions to net zero
human activities in three sectors: fossil fuels (35%), waste by 2025.
(20%) and agriculture (40%).
185. Solution (a)
Fossil fuel sector: Oil and gas extraction, processing and
Exp) Option a is correct
distribution account for 23%. Coal mining alone accounts
for 12% of emissions. Statement 1 is correct. The TARA Ocean foundation
conducted the scientific expedition in 2019 under the
Waste sector: Landfills and wastewater make up about
mission named as Mission microplastic 2019. It will trace
20% of global anthropogenic emissions.
the origin of plastic from land to sea with the help of its
Agricultural sector: Livestock emissions from manure boat schooner Tara.
and fermentation represent roughly 32%. Further, rice
Statement 2 is incorrect. The Ocean agency has launched
cultivation accounts for 8% of global anthropogenic
this campaign in partnership with UNEP. It is a creative
emissions.
awareness campaign that draws attention to coral
Statement 4 is correct: Cutting methane emissions can fluorescence due to climate change. Coral fluorescence,
rapidly reduce the rate of warming in the near-term as or “glowing” coral, is a last line of defence before coral
the gas broke down quickly. dies and bleaches. Through Glowing Glowing Gone, The
Ocean Agency hopes to garner public support to inspire

Workbook 412
.
ENVIRONMENT

policy and funding to conserve coral reefs and save an Statement 3 is correct. The secretariat provides technical
ecosystem on which our entire planet depends. So it does expertise and assists in the analysis and review of
not deal with plastic pollution. climate change information reported by Parties and
in the implementation of the Kyoto mechanisms. It
186. Solution (d) also maintains the registry for Nationally Determined
Exp) Option d is correct. Contributions (NDC) established under the Paris
Agreement, a key aspect of implementation of the Paris
Option d is correct. The Blue Dot Network (BDN) was
Agreement.
launched in 2019 by the U.S., Japan and Australia. It is
an initiative that will bring together governments, the 189. Solution (a)
private sector and civil society to certify projects that Exp) Option a is correct
uphold global infrastructure principles. Under it,
infrastructure projects within the Indian Ocean Region Statement 1 is correct: The mechanisms that were an
will be certified in keeping with globally-recognised outcome of COP 16: Green Climate Fund (GCF),
standards that uphold principles of transparency and Adaptation Fund, Technology mechanism. The GCF
financial viability. It does not offer public funds or loans supports projects, programs and policies in developing
country parties.
for the project.
Statement 2 is correct: Nationally determined
187. Solution (a) contributions (NDCs) are the core of the Paris Agreement
Exp) Option a is correct. and the achievement of the long-term goals. The Paris
Agreement requires each Party to prepare, communicate
Statement 1 is correct: The UNFCCC is a “Rio and maintain successive nationally determined
Convention”, one of three adopted at the “Rio Earth contributions (NDCs) that it intends to achieve. Parties
Summit” in 1992. Its sister Rio Conventions are the UN shall pursue domestic mitigation measures, with the aim
Convention on Biological Diversity and the Convention of achieving the objectives of such contributions.
to Combat Desertification.
Statement 3 is incorrect: The “Powering Past Coal”
Statement 2 is incorrect: The ultimate objective of alliance was initiated by the UK, Canada and Marshall
the Convention is to “stabilize (and not eliminate) Islands at the annual Conference of Parties (COP23)
greenhouse gas concentrations in the atmosphere at a meeting in Bonn, Germany. The aim is to phase out
level that will prevent dangerous human interference coal-based power by 2030 and to accelerate the transition
with the climate system.” It states that “such a level from coal to clean energy.
should be achieved within a time-frame sufficient to Major coal users like China, the US, Russia have not
allow ecosystems to adapt naturally to climate change, joined.
to ensure that food production is not threatened, and to
enable economic development to proceed in a sustainable 190. Solution (d)
manner.” Exp) Option d is correct.
Statement 3 is incorrect: The treaty itself is not Statement 1 is incorrect. The Paris Agreement is a legally
legally binding as it does not set mandatory limits on binding international treaty on climate change. It was
greenhouse gas emissions for individual countries and adopted by 196 Parties at COP 21 in Paris, on 12 December
doesn’t contain any enforcement mechanisms. 2015 and entered into force on 4 November 2016. Its goal
is to limit global warming to well below 2, preferably to 1.5
188. Solution (c)
degrees Celsius, compared to pre-industrial levels.
Exp) Option c is correct. Statement 2 is correct. To better frame the efforts
Statement 1 is incorrect. The United Nations Framework towards the long-term goal, the Paris Agreement invites
Convention on Climate Change is an international countries to formulate and submit by 2020 long-term
environmental treaty signed at the United Nations low greenhouse gas emission development strategies (LT-
Conference on Environment and Development (the Earth LEDS). LT-LEDS provide the long-term horizon to the
Summit) held in Rio de Janeiro in 1992. The Convention NDCs. Unlike NDCs, they are not mandatory.
presently has 197 parties. Statement 3 is correct. With the Paris Agreement,
Statement 2 is incorrect. Under the UNFCCC, Annex II countries established an enhanced transparency
includes the Organization for Economic Cooperation and framework (ETF). Under ETF, starting in 2024, countries
Development (OECD) members of Annex I, except the will report transparently on actions taken and progress
economies in transition (EIT) Parties. They are required to in climate change mitigation, adaptation measures
provide financial resources to enable developing countries and support provided or received. It also provides for
to undertake emissions reduction activities under the international procedures for the review of the submitted
Convention and to help them adapt to adverse effects of reports.
climate change. In addition, they are expected to promote Statement 4 is correct. In NDCs, countries communicate
the development and transfer of environmentally friendly actions they will take to reduce their Greenhouse
technologies to EIT Parties and developing countries. Gas emissions in order to reach the goals of the Paris

413 Workbook
.
ENVIRONMENT

Agreement. Countries also communicate in the NDCs CTCN provides technology solutions, capacity building
actions they will take to build resilience to adapt to the and advice on policy, legal and regulatory frameworks
impacts of rising temperatures. The NDCs are mandatory tailored to the needs of individual countries.
for the countries. Hence, both the statements are correct.
191. Solution (b) 194. Solution (c)
Exp) Option b is correct. Exp) Option c is correct.
Statement 1 is incorrect. NDC Transport Initiative Statement 1 is correct. The Climate Technology Centre &
for Asia (NDC-TIA): The initiative aims to promote a Network (CTCN) facilitates the transfer of technologies
comprehensive approach to decarbonize transport in through three core services:
India, Vietnam and China over the period 2020-24. It
is supported by the International Climate Initiative (IKI) 1. Providing technical assistance at the request of
of the German Ministry for the Environment and Nature developing countries to accelerate the transfer of
Conservation and Nuclear Safety (BMU). In India, NITI climate technologies.
Aayog will be its implementing partner. 2. Creating access to information and knowledge on
climate technologies.
Statement 2 is correct. The NDC–TIA India Component
will focus on establishing a multi-stakeholder dialogue 3. Fostering collaboration among climate technology
platform for decarbonizing transport in India, stakeholders via the Centre’s network of regional and
strengthening GHG and transport modelling capacities. It sectoral experts from academia, the private sector,
also aims to provide technical support on GHG emission and public and research institutions.
reduction measures, finance climate actions in transport, Statement 2 is correct. The United Nations Environment
offer policy recommendations on electric vehicle (EV) Programme (UN Environment) hosts the CTCN
demand and supply policies and evaluate business models in collaboration with the United Nations Industrial
through cost-benefit analysis. The programme intends to Development Organization (UNIDO) and the support
support the development of policies and regulations to of a consortium of partners that are engaged in some
promote electric vehicle charging infrastructure uptake 1,500 activities related to climate technologies in over 150
and smooth widescale adoption of EVs in India. countries.
192. Solution (d) 195. Solution (b)
Exp) Option d is correct. Exp) Option b is correct.
Option a is incorrect. INDC’s are launched as part Statement 1 is correct. The Climate Neutral Now
of United Nations Convention on Climate Change Initiative was launched in 2015 by the UNFCCC
(UNFCCC) in accordance to Paris climate deal (and not secretariat, based on a mandate to promote the voluntary
UNPD’s SDG 2030). use of carbon market mechanisms recognized under the
Option b is incorrect. India has pledged to reduce the Convention. It has evolved to become a much wider tool
emissions intensity of its GDP by 33-35 per cent by for awareness-raising, capacity building, partnership
2030, below 2005 levels (and not 2015 levels). developing, promoting and facilitating the estimation of
Option c is incorrect. India’s pledge to install 40 per cent carbon footprints, the reduction of those footprints, and
(and not 50 percent) of its total electricity capacity from voluntary compensation (offsetting).
non-fossil fuel-based energy by 2030. Statement 2 is incorrect. It aims to increase climate
Option d is correct. India as per its INDC’s intends to action by engaging non-Party stakeholders (sub-national
create an additional carbon sink of 2.5 to 3 billion governments, companies, organizations, individuals). The
tonnes of CO2 through additional forests by 2030. Climate Neutral Now Initiative encourages and supports
organizations and other interested stakeholders to act
193. Solution (c) now in order to achieve a climate neutral world by 2050,
Exp) Option c is correct. as enshrined in the Paris Agreement.
Statement 1 is correct: The Climate Technology Centre 196. Solution (a)
and Network (CTCN) is the operational arm of the
UNFCCC Technology Mechanism, hosted by the UN Exp) Option a is correct.
Environment Programme and the UN Industrial Race To Zero is a global campaign, launched by UNFCCC,
Development Organization (UNIDO). Serving as the to rally leadership and support from businesses, cities,
core of the CTCN, network members respond to climate regions, investors for a healthy, resilient, zero carbon
technology requests from developing country Parties to recovery that prevents future threats, creates decent jobs,
the UNFCCC. and unlocks inclusive, sustainable growth.
Statement 2 is correct: The Centre promotes the Race To Zero mobilizes actors outside of national
accelerated transfer of environmentally sound governments to join the Climate Ambition Alliance,
technologies for low carbon and climate resilient which was launched at the Secretary-General of the
development at the request of developing countries. United Nations (UNSG’s) Climate Action Summit 2019.

Workbook 414
.
ENVIRONMENT

These ‘real economy’ actors join 120 countries in the Statement 3 is correct: Its objective is to create a financial
largest ever alliance committed to achieving net zero value for the carbon stored in forests by offering
carbon emissions by 2050 at the latest. incentives for developing countries to reduce emissions
from forested lands and invest in low-carbon paths to
197. Solution (a) sustainable development. Developing countries would
Exp) Option a is correct. receive results-based payments for results-based actions.
Statement 1 is incorrect. Paris Climate Change Apart from this, it promotes the informed and meaningful
involvement of all the stakeholders, including indigenous
Agreement (and not Montreal Protocol) aims to limit
people and other forest dependent communities.
global warming to well below 2 degrees Celsius preferably
to 1.5 degree Celsius, compared to pre-industrial levels. 200. Solution (a)
Statement 2 is correct. The Vienna Convention for the Exp) Option a is correct.
Protection of the Ozone Layer and its Montreal Protocol Statement 1 is correct. The UN-REDD programme
on Substances that Deplete the Ozone Layer was designed assists countries to develop the capacities needed to
to reduce the production and consumption of ozone meet the UNFCCC’s REDD+ requirements, so that they
depleting substances in order to reduce their abundance can qualify to receive result-based payments under the
in the atmosphere, and thereby protect the earth’s fragile convention.
ozone Layer.
Statement 2 is incorrect. Reducing emissions from
Statement 3 is correct. The Kigali Amendment to the deforestation and forest degradation (REDD+)
Montreal Protocol is legally binding and came into programme, (not REDD), offers developing countries
force from January 1, 2019. Under the amendment, the results-based payments for results-based actions. REDD+
developed countries will reduce hydrofluorocarbons is a mechanism developed by Parties to the United Nations
(HFCs) consumption beginning in 2019. Most developing Framework Convention on Climate Change (UNFCCC).
countries will freeze consumption in 2024 and some It creates a financial value for the carbon stored in forests
developing countries including India with unique by offering incentives for developing countries to reduce
circumstances will freeze consumption in 2028. emissions from forested lands and invest in low-carbon
Statement 4 is correct. India became a Party to the paths to sustainable development. Developing countries
Vienna Convention for the Protection of Ozone Layer would receive results-based payments for results-based
actions. REDD+ goes beyond simply deforestation and
in 1991 and the Montreal Protocol on substances that
forest.
deplete the ozone layer in 1992.
degradation and includes the role of conservation,
198. Solution (d) sustainable management of forests and enhancement of
Exp) Option d is correct. forest carbon stocks.
In 2016, the Parties to the Montreal Protocol adopted the Statement 3 is correct. The UN-REDD programme
agreement on HFCs at the close of the 28th Meeting of is a collaboration between Food and Agriculture
the Parties (MOP 28) in Kigali, Rwanda. Governments Organisation of the United Nations (FAO), United
agreed that it would enter into force on 1 January 2019, Nations Development Programme (UNDP) and United
Nations Environment Programme (UNEP).
provided that at least 20 Parties to the Montreal Protocol
had ratified it. 201. Solution (d)
Under this agreement developed countries will start to Exp) Option d is correct.
phase down HFCs by 2019. Developing countries will The Global Environment Facility (GEF) was established
follow with a freeze of HFCs consumption levels in 2024, on the eve of the 1992 Rio Earth Summit to help tackle
with some countries freezing consumption in 2028. By the our planet’s most pressing environmental problems.
late 2040s, all countries are expected to consume no more
The GEF serves as financial mechanism for the
than 15-20 per cent of their respective baselines.
following conventions:
199. Solution (c) Convention on Biological Diversity (CBD),
Exp) Option c is correct. United Nations Framework Convention on Climate
Statement 1 is incorrect: Reducing emissions from Change (UNFCCC)
deforestation and forest degradation (REDD+) is a United Nations Convention to Combat Desertification
climate change mitigation mechanism being developed (UNCCD),
by Parties to the United Nations Framework Convention Stockholm Convention on Persistent Organic Pollutants
on Climate Change (UNFCCC). (POPs)
Statement 2 is correct: REDD+ goes beyond simply Minamata Convention on Mercury
reducing deforestation and forest degradation. It also
includes the role of conservation, sustainable management 202. Solution (b)
of forests and enhancement of forest carbon stocks. Exp) Option b is correct.

415 Workbook
.
ENVIRONMENT

The Poznan strategic program (PSP), initiated by the Statement 2 is correct: IPCC does not conduct any
Global Environment Facility (GEF), provides funding research nor does it monitor climate-related data or
to climate technology development and transfer activities. parameters. However, being a scientific body, it reviews
The program has supported countries to undertake and assesses the most recent scientific, technical and
technology needs assessments (TNAs), develop technology socio-economic information produced worldwide
pilot projects and implement hundreds of climate projects relevant to the understanding of climate change. It has
with objectives related to climate technologies. regularly delivered the Assessment Reports in a non-
Countries created the PSP in 2007, when the Conference technical style suitable for policymakers.
of the Parties (COP) requested the GEF to elaborate a
206. Solution (d)
strategic programme for scaling up the level of investment
for technology transfer. This was undertaken with the aim Exp) Option d is correct.
of helping developing countries to address their needs for Option a is correct. The IPCC is an organization of
environmentally sound technologies. governments that are members of the United Nations or
WMO. The IPCC currently has 195 members.
203. Solution (d)
Option b is correct. IPCC was created in 1988 by the
Exp) Option d is correct. World Meteorological Organization(WMO) and the
Clean Technology Fund is a part of Climate Investment United Nations Environment Programme (UNEP).
Fund and promotes financing for demonstration,
Option c is correct. The IPCC does not conduct its own
deployment and transfer of low carbon technologies
research. For the assessment reports, IPCC scientists
with a significant potential for long-term greenhouse gas
volunteer their time to assess the thousands of scientific
emissions savings. The Climate Investment Funds (CIFs)
papers published each year to provide a comprehensive
are designed by developed and developing countries and
summary of what is known about the drivers of climate
are implemented with the multilateral development banks
change, its impacts and future risks, and how adaptation
(MDBs) to bridge the financing and learning gap between
and mitigation can reduce those risks.
now and the next international climate change agreement.
CIFs are two distinct funds: the Clean Technology Fund Option d is incorrect. The IPCC Assessment Reports
and the Strategic Climate Fund. are policy-relevant but not policy-prescriptive. They may
present projections of future climate change based on
Statement 1 is correct: World Bank acts as Trustee for all
Climate Investment Funds (CIF), including the CTF. different scenarios and the risks that climate change poses
and discuss the implications of response options, but they
Statement 2 is correct: The Clean Technology Fund do not tell policymakers what actions to take, i.e., they are
provides new large-scale financial resources to invest in not binding on countries.
clean technology projects in developing countries with
a significant potential for long-term greenhouse gas 207. Solution (b)
emissions savings. Exp) Option b is correct.
Statement 3 is correct: Rewa solar plant is the first project Statement 1 is correct: The IPCC established the NGGIP
to get funding from the Clean Technology Fund in India. and its Task Force develops and refines a methodology for
204. Solution (b) the calculation and reporting of national GHG emissions
and removals.
Exp) Option b is correct
Statement 2 is incorrect: India established its ‘India
Statement 1 is incorrect – Climate Smart Agriculture GHG Program’ in 2008 led by World Resources Insitute-
focuses on both Adaptation and Mitigation (Can sequester India (WRI-India), The Energy and Resource Institute
carbon into the soil) approaches against Climate Change. (TERI) and Confederation of Indian Industry (CII).
Statement 2 is correct - By Increased productivity, It is an industry-led voluntary framework to measure
enhanced resilience (Adaptation) and reduced emissions and manage greenhouse gas emissions. The programme
(Mitigation), it achieves a triple win situation making builds comprehensive measurement and management
farming a profitable business for farmers. strategies to reduce emissions and drive more profitable,
Statement 3 is correct- With decentralization at its heart, competitive and sustainable businesses and organisations
Climate Smart Agriculture gives attention to Landscape in India.
approach, it promotes country and locality specific
solutions to Agriculture. 208. Solution (b)
Exp) Option b is correct.
205. Solution (b)
Statement 1 is incorrect. The Climate and Clean Air
Exp) Option b is correct. Coalition to Reduce Short-Lived Climate Pollutants
Statement 1 is incorrect: Intergovernmental Panel on (CCAC) was launched by the United Nations
Climate Change (IPCC) was established by the UNEP Environment Programme (UNEP) and six countries —
and the WMO in 1988. It is a UN body for assessing the Bangladesh, Canada, Ghana, Mexico, Sweden, and the
science related to climate change. United States in February 2012.

Workbook 416
.
ENVIRONMENT

Statement 2 is correct. The Climate and Clean Air to be respected by each Party, which has to adopt its
Coalition is a voluntary partnership of governments, own domestic legislations to ensure that CITES is
intergovernmental organizations, businesses, scientific implemented at the national level.
institutions and civil society organizations committed
to improving air quality and protecting the climate 212. Solution (d)
through actions to reduce short-lived climate Exp) Option d is correct.
pollutants.
Statement 1 is correct. Membership of CAWT is open
The Coalition’s initial focus is on methane, black to governments, NGOs and corporations committed
carbon, and HFCs. At the same time, partners recognize to combat wildlife trafficking. The CAWT is a unique
that action on short-lived climate pollutants must voluntary public-private
complement and supplement, not replace, global action
to reduce carbon dioxide, in particular efforts under the coalition of like-minded governments and organizations.
UNFCCC Statement 2 is incorrect. The Coalition Against Wildlife
Trafficking (CAWT) was established in September 2005
209. Solution (a)
by the U.S. State Department, it focuses on political
Exp) Option a is correct. and public attention on growing threats to wildlife from
Statement 1 is correct. Agenda 21 is a non-binding poaching and illegal trade. India joined CAWT in 2006.
action plan of the United Nations with regard to Statement 3 is correct. The Coalition Against Wildlife
sustainable development. Trafficking aims to improve wildlife law enforcement.
Statement 2 is incorrect. Agenda 21 is a product of the It promotes enforcement training and information
Earth Summit (UN Conference on Environment and sharing and strengthening regional cooperative networks.
Development) held in Rio de Janeiro, Brazil, in 1992. But the CAWT organization is not directly involved in
210. Solution (c) any enforcement activities.
Exp) Option c is correct. 213. Solution (b)
Statement 1 is incorrect. Convention on Biological Exp) Option b is correct.
Diversity was the result of United Nations Conference on
Environment and Development (UNCED), also known Statement 1 is incorrect. The program works to manage
as the Earth Summit in 1992. It aims at conservation of (and not stop) the trade in wild animals and plants at
biological diversity, the sustainable use of its components a sustainable level without damaging the integrity of
and the fair and equitable sharing of the benefits arising ecological systems.
out of the utilization of genetic resources. Statement 2 is correct. TRAFFIC, the wildlife trade
Statement 2 is correct. The Cartagena Protocol monitoring network, is a joint program of WWF
(2000) on Biosafety to the Convention on Biological and IUCN. It is an International non-governmental
Diversity is an international agreement which aims to organization founded in 1976 with headquarters at
ensure the safe handling, transport and use of living Cambridge, United Kingdom.
modified organisms (LMOs) resulting from modern
biotechnology. The protocol also deals with any adverse 214. Solution (d)
effects of modern biotechnology on biological diversity. Exp) Option d is correct.
Statement 3 is correct. The Nagoya Protocol is a Statement 1 is incorrect: As per the criteria for
supplementary agreement to the Convention on Biological identification of Wetlands of National Importance,
Diversity. The main objective of the protocol is the access a site which is a representative, rare or unique example
to Genetic Resources and the Fair and Equitable Sharing of natural or near-natural wetland can be selected.
of Benefits arising from their utilization.
Ex - Harike is a man-made wetland which came into
211. Solution (b) existence in 1952 after the construction of barrage near
Exp) Option b is correct. the confluence of Sutlej and Beas.
CITES is an international agreement between Statement 2 is correct: Montreux Record is a register
governments to ensure that international trade in plants of wetland sites where adverse changes in ecological
and animals does not threaten their survival in the character have occurred, are occurring, or are likely to
wild. It is administered through the United Nations occur and which are in need of priority conservation
Environment Programme (UNEP). attention. It is maintained as part of Ramsar List.
Statement 1 is incorrect. Appendix I includes species Currently India has two sites in Montreux Record i.e.,
threatened with extinction. Trade in specimens of Keoladeo National Park and Loktak Lake.
these species is permitted only in exceptional licensed Statement 3 is correct: For the first time in 2018
circumstances. Ex- Red Panda, Asiatic Lion. convention issued report on state of World’s wetlands. The
Statement 2 is correct. CITES does not take the place report was titled Global Wetland Outlook. As per the
of national laws. Rather it provides a framework report from 1975 to 2015 around 35% wetlands were lost.

417 Workbook
.
ENVIRONMENT

215. Solution (b) Statement 1 is correct. The Convention on the


Exp) Option b is correct. Conservation of Migratory Species of Wild Animals
(CMS), also known as Bonn Convention, provides a
Statement 1 is correct: The Montreux Record is a register
global platform for the conservation and sustainable
of wetland sites on the List of Wetlands of International
use of migratory animals and their habitats. It brings
Importance where changes in ecological character have
together the States through which migratory animals
occurred, are occurring, or are likely to occur as a result
pass, called as the Range States. It is an intergovernmental
of technological developments, pollution or other human
treaty, concluded under the aegis of the United Nations
interference.
Environment Programme.
Statement 2 is correct: Montreux record is the principal
Statement 2 is correct. The 13th Conference of Parties
tool of the Convention and is
(COP) of the Convention on the conservation of
maintained as part of the Ramsar List. migratory species of wild animals (CMS) was hosted by
Statement 3 is incorrect: Keoladeo National Park, India in February 2020 at Gandhinagar in Gujarat.
Rajasthan and Loktak Lake, Manipur have been included Statement 3 is correct. The Indian sub-continent is
in Montreux Record in 1990 and in 1993 respectively. also a part of the major bird flyway network, i.e., the
Chilika Lake, Orissa included in Montreux Record in Central Asian Flyway (CAF) that covers areas between
1993 has been removed in November 2002. the Arctic and Indian Oceans, and covers migratory
216. Solution (a) waterbird species, including globally threatened species.
India has also launched the National Action Plan for the
Exp) Option a is correct. conservation of migratory species under the Central
Pair 1-C – Tso Moriri Lake is located in Ladakh. It is Asian Flyway.
a freshwater to brackish lake with wet meadows and
borax-laden wetlands along the shores. The site is said to 219. Solution (c)
represent the only breeding ground outside of China for Exp) Option c is correct.
the Black-necked crane and the only breeding ground for Statement 1 is correct. The Basel Convention on the
Bar-headed geese in India.
Control of Transboundary Movements of Hazardous
Pair 2-A – Rudrasagar Lake is located in Tripura. It is Wastes and Their Disposal, usually known as the Basel
a lowland sedimentation reservoir in the northeast hills, Convention, is an international treaty that was designed
fed by three perennial streams discharging to the River to reduce the movements of hazardous waste between
Gomti. nations, and specifically to prevent transfer of hazardous
Pair 3-D – Deepor Beel is located in Assam. A permanent waste from developed to less developed countries
freshwater lake in a former channel of the Brahmaputra (LDCs). It does not, however, address the movement of
river. It supports some threatened species like Spot billed radioactive waste. The Convention is also intended to
Pelican, Lesser and Greater Adjutant Stork and Baer’s minimize the amount and toxicity of wastes generated,
Pochard. to ensure their environmentally sound management as
Pair 4-B - Pong Dam Lake is situated in Himachal closely as possible to the source of generation, and to
Pradesh. It is a water storage reservoir created in 1975 assist LDCs in environmentally sound management of the
on the Beas River in the low foothills of the Himalaya on hazardous and other wastes they generate.
the northern edge of the Indo-Gangetic plain. Statement 2 is correct. The Rotterdam Convention
(formally, the Rotterdam Convention on the Prior
217. Solution (a) Informed Consent Procedure for Certain Hazardous
Exp) Option a is correct. Chemicals and Pesticides in International Trade) is a
Statement 1 is incorrect. The Convention on the multilateral treaty to promote shared responsibilities
Conservation of Migratory Species of Wild Animals or in relation to importation of hazardous chemicals.
Bonn Convention aims to conserve terrestrial, aquatic The convention promotes open exchange of information
and avian migratory species throughout their range. It and calls on exporters of hazardous chemicals to use
is an intergovernmental treaty, concluded under the aegis proper labelling, include directions on safe handling, and
of the United Nations Environment Programme and not inform purchasers of any known restrictions or bans.
International Union for Conservation of Nature. Signatory nations can decide whether to allow or ban the
Statement 2 is correct. It is the only global convention importation of chemicals listed in the treaty, and exporting
specializing in the conservation of migratory species, countries are obliged to make sure that producers within
their habitats and migration routes. The Convention their jurisdiction comply.
complements and co-operates with a number of other 220. Solution (b)
international organizations, NGOs and partners in the
media as well as in the corporate sector. Exp) Option b is correct.
Pair 1 is correctly matched. The Stockholm Convention
218. Solution (d) on Persistent Organic Pollutants was adopted on 22 May
Exp) Option d is correct. 2001 in Stockholm, Sweden and was entered into force

Workbook 418
.
ENVIRONMENT

on 17 May 2004. Persistent Organic Pollutants (POPs) Statement 2 is incorrect. ‘Desertification’ as defined in
are organic chemical substances which are carbon- the UNCCD refers to land degradation in the drylands
based. (arid, semi-arid and dry sub humid regions) resulting
Pair 2 is incorrectly matched. The Basel Convention from various factors and does not connote spread or
on the Control of Transboundary Movements of expansion of deserts.
Hazardous Wastes and their Disposal was adopted on Statement 3 is incorrect. India became a signatory to
22 March 1989 by the Conference of parties in Basel, UNCCD in 1994, and ratified it in 1996. The Ministry of
Switzerland. Environment, Forest and Climate Change is the nodal
Pair 3 is incorrectly matched. Rotterdam Convention was Ministry for the Convention.
adopted in 1998 and entered into force on 24 February
2004. The Convention creates legally binding obligations 223. Solution (b)
for the implementation of the Prior Informed Consent Exp) Option b correct.
(PIC) procedure. The Convention covers pesticides and Statement 1 is incorrect. The Bonn Challenge is a global
industrial chemicals that have been banned or severely effort to bring 150 million hectares of deforested and
restricted. It also promotes shared responsibility and degraded land into restoration by 2020 and 350 million
cooperative efforts among Parties in the international hectares by 2030. It is an implementation vehicle for
trade of certain hazardous chemicals. national priorities such as water and food security
Pair 4 is correctly matched. Gothenburg Protocol, while simultaneously helping countries contribute to the
adopted in 1999, aims to Abate Acidification, achievement of international climate change, biodiversity
Eutrophication and Ground-level Ozone. The Protocol and land degradation commitments.
sets national emission ceilings for 2010 up to 2020 for
Statement 2 is correct. Forest landscape restoration
four pollutants: sulphur (SO2), nitrogen oxides (NOx),
(FLR) is the underlying approach of Bonn challenge. FLR
volatile organic compounds (VOCs) and ammonia
(NH3). aims to restore ecological integrity at the same time as
improving human well-being through multifunctional
221. Solution (a) landscapes.
Exp) Option a correct. 224. Solution (c)
Statement 1 is correct: United Nations Convention to Exp) Option c is correct.
Combat Desertification (UNCCD) is the sole legally
binding international agreement linking environment Bonn Challenge is a global effort to bring 150 million
and development to sustainable land management. hectares of the world’s deforested and degraded land into
This convention was the outcome of Rio earth summit restoration by 2020, and 350 million hectares by 2030. It
organized in 1992. This convention addresses specifically brings together countries that share borders, ecosystems
the arid, semi-arid and dry sub-humid areas, known as the and economies in regional platforms to collaborate on
drylands, where some of the most vulnerable ecosystems common goals and challenges. These platforms help
and peoples can be found. catalyse political will and ambition in restoring degraded
Statement 2 is incorrect: Land Degradation Neutrality lands.
(LDN) is defined as the state whereby the amount and Statement 1 is incorrect: Bonn Challenge is launched by
quality of land resources, necessary to support ecosystem the Government of Germany and IUCN.
functions and services and enhance food security, Statement 2 is correct: It aims to restore 150 million
remains stable or increases within specified temporal hectares of degraded and deforested by 2020 and 350
and spatial scales and ecosystems. LDN (SDG 15.3) is a million hectares by 2030. The Challenge surpassed the
target under the Sustainable Development Goals (SDGs) 150-million-hectare milestone for pledges in 2017.
to be achieved by the nations. In 14th Conference of the
Parties (COP14) to UNCCD, the member countries Statement 3 is incorrect: India became part of the “Bonn
have agreed to make the Sustainable Development Goal Challenge” in January 2019.
target of achieving land degradation neutrality by 2030, a 225. Solution (a)
national target for action.
Exp) Option a is correct.
222. Solution (a) The world’s 20 most powerful economies have launched
Exp) Option a correct. an initiative to protect land. The Leaders’ Declaration
Statement 1 is correct. The United Nations Convention issued on 22 November 2020, states that the G-20 leaders
to Combat Desertification (UNCCD) is the sole launched “the Global Initiative on Reducing Land
legally binding international agreement linking Degradation and Enhancing Conservation of Terrestrial
environment and development to sustainable land Habitats to prevent, halt, and reverse land degradation.
management. UNCCD, established in 1994, is one of Building on existing initiatives, we share the ambition to
the Rio Conventions that focuses on desertification, land achieve a 50 percent reduction of degraded land by 2040,
degradation and drought (DLDD). on a voluntary basis.”

419 Workbook
.
ENVIRONMENT

226. Solution (a) initiative by Food and Agricultural Organisation


Exp) Option a is correct. (FAO) and not by UNESCO. It seeks to safeguard
and support the world’s agricultural heritage systems.
Statement 1 is incorrect. TX2 stands for “Tigers times
These traditional agricultural systems represent models
two”, signalling the goal to double the population of wild
tigers by 2022. of sustainable agricultural production. GIAHS are
outstanding landscapes of aesthetic beauty that combine
Statement 2 is correct. Global Tiger Summit was held in agricultural biodiversity, resilient ecosystems and a
St Petersburg in 2010, where 13 Heads of State signed the valuable cultural heritage.
Declaration on Tiger Conservation. The goal adopted at
St. Petersburg is to double the population of wild tigers Statement 2 is correct. Kuttanad below Sea Level
by 2022. Farming System of Kerala has been designated as a
GIAHS site in India. Apart from this, there are two more
Statement 3 is correct. Pilibhit Tiger Reserve has won the
recognized GIAHS sites in India:
inaugural TX2 Award, presented by CATS, Global Tiger
Forum, IUCN, UNDP, The Lion’s Share and WWF, for its Koraput Traditional Agriculture of Odisha.
remarkable contribution to tiger conservation. The awards Pampore Saffron Heritage of Kashmir.
celebrate the 10-year anniversary of all 13 Tiger Range
Countries committing to double the global population of 230. Solution (b)
wild tigers by 2022 - a goal known as TX2. Exp) option b is correct.
227. Solution (a) Pair 1 is incorrect: CITES appendix I provides protection
to species which are in danger of extinction. Under this
Exp) Option a is correct.
commercial trade is prohibited, but trade for research
Statement 1 is incorrect The Cartagena Protocol on purpose is permitted only if the origin country ensures
Biosafety to the Convention on Biological Diversity the trade won’t harm the species’ chance of survival.
is an international treaty governing the movements
of living modified organisms (LMOs) resulting from Pair 2 is correct: CITES Appendix II provides protection
modern biotechnology from one country to another. to the species which are not facing imminent threat of
extinction but they need monitoring so that trade does
Statement 2 is incorrect. Nagoya Protocol governs the not become a threat.
access to Genetic Resources and the Fair and Equitable
Sharing of Benefits Arising from their Utilization Pair 3 is correct: CITES appendix III provides protection
(ABS). to species that are protected in at least one country.
Countries protection animal under this, issues export
Statement 3 is correct. The Nagoya-Kuala Lumpur
Supplementary Protocol on Liability and Redress is a permits for trade of such animals.
supplementary protocol to the Cartagena protocol on 231. Solution (d)
Biosafety.
Exp) Option d is correct.
The Supplementary Protocol fulfils the commitment set
forth in Article 27 of the Cartagena Protocol to elaborate Option d is correct. Global Methane Assessment
international rules and procedures on liability and report is published by Climate and Clean Air Coalition
redress for damage to biodiversity resulting from with the help of the United Nations Environment
transboundary movements of LMOs. Programme (UNEP). Report suggested that the world
needs to dramatically cut methane emissions to avoid the
228. Solution (b) worst of climate change. According to the report more
Exp) Option b is correct. than half of global methane emissions stem from human
Statement 1 is incorrect: According to Living Planet activities in three sectors: fossil fuels (35%), waste (20%)
Report,2020 the region most hit by biodiversity loss is and agriculture (40%).
Latin America (94%) (not south east Asia). Climate and Clean Air Coalition is a voluntary partnership
Statement 2 is correct: The report says that 68% of of governments, intergovernmental organizations,
vertebrate species - birds, reptiles, amphibians, mammals businesses, scientific institutions and civil society
have gone extinct between 1970 and 2014 due to human organizations. It aims to protect the climate and improve
activity. air quality through actions to reduce short-lived climate
pollutants. India is a member of the coalition.
Statement 3 is incorrect: The Living Planet Report,
WWF’s flagship publication released every two years, is a 232. Solution (c)
comprehensive study of trends in global biodiversity and
the health of the planet. Exp) Option c is correct.
Statement 1 is incorrect: It is coordinated by the World
229. Solution (b) Customs Organisation (WCO) and INTERPOL.
Exp) Option b is correct Statement 2 is correct: The objective of this exercise is
Statement 1 is incorrect. Globally Important Agricultural to disrupt networks of transnational smugglers trading
Heritage systems (GIAHS), started in 2002, is an in wildlife (live/ products) and forest products whose

Workbook 420
.
ENVIRONMENT

trade is banned under environmental laws (domestic and To encourage greater interest in waterbirds and wetlands
international like CITES) to ensure their conservation. amongst citizen
Statement 3 is incorrect: India has not withdrawn from
236. Solution (a)
Operation Thunder. In India, the India Customs along
with INTERPOL and WCO seized red sandalwood that Exp) Option a is correct.
were destined for UAE recently. Friends of the Earth International is the world’s
largest grassroots environmental network to create
233. Solution (c) environmentally sustainable and socially just societies.
Exp) Option c is correct. It was established with a vision of a peaceful and
In 2015, at COP 21, Parties, aiming to strengthen the sustainable world based on societies living in harmony
global response to the threat of climate change, adopted with nature. It aims to build a society built upon peoples’
the Paris Agreement and through it established an sovereignty and participation. It will be founded on social,
enhanced transparency framework. The enhanced economic, gender and environmental justice and be free
transparency framework is expected to build mutual trust from all forms of domination and exploitation, such as
and confidence, and promote effective implementation by neoliberalism, corporate globalization, neo-colonialism
providing: and militarism. It primarily works -
Clarity and tracking of progress towards achieving the To collectively ensure environmental and social justice,
countries’ individual nationally determined contributions. human dignity, and respect for human rights and peoples’
rights so as to secure sustainable societies.
Clarity on support provided and received by relevant
individual Parties in the context of climate change actions. To halt and reverse environmental degradation and
depletion of natural resources, nurture the earth’s
By design, the enhanced transparency framework covers
ecological and cultural diversity, and secure sustainable
all substantive aspects of the Paris Agreement, including
livelihoods.
tracking progress of implementation and achievement of
nationally determined contributions. 237. Solution (c)
234. Solution (b) Exp) Option c is correct.
Exp) Option b is correct. Statement 1 is incorrect: At COP 16, 2010, held in
Cancun, Parties established the Green Climate Fund
‘’Fit for 55’’ plan is launched by the European Union (EU).
(GCF) as an operating entity of the Financial Mechanism
As part of the European Green Deal, with the European
of the UNFCCC. The Fund is governed by the GCF Board
Climate Law, the EU has set itself a binding target of
and it is accountable to and functions under the guidance
achieving climate neutrality by 2050. This requires current
of the COP to support projects, programmes, policies
greenhouse gas emission levels to drop substantially in
and other activities in developing country Parties using
the next decades. As an intermediate step towards climate
thematic funding windows. The GCF also serves the Paris
neutrality, the EU has raised its 2030 climate ambition,
Agreement.
committing to cutting emissions by at least 55% by 2030.
Statement 2 is incorrect: The BioCarbon Fund Initiative
235. Solution (b) for Sustainable Forest Landscapes (ISFL) is a multilateral
Exp) Option b is correct. fund, supported by donor governments. It promotes
Statement 1 is incorrect. Asian Waterbird Census (AWC) reducing greenhouse gas emissions from the land sector,
was started in the year 1987. It is an annual event in which including efforts to reduce deforestation and forest
thousands of volunteers across Asia and Australasia count degradation in developing countries (REDD+). The fund
waterbirds in the wetlands of their country. This event is is managed by the World Bank.
coordinated by Wetlands International and forms part Statement 3 is correct. The Special Climate Change
of global waterbird monitoring programme called the Fund (SCCF) supports adaptation and technology transfer
International Waterbird Census (IWC). in all developing country parties to the United Nations
In India, the AWC is annually coordinated by the Framework Convention on Climate Change (UNFCCC).
Bombay Natural history Society(BNHS) and Wetlands It is administered by the Global Environment Facility.
International. It is not conducted by the World Wide Statement 4 is correct. The Least Developed Countries
Fund for Nature. Fund (LDCF), established under the United Nations
Statement 2 is correct. Objectives of the Census: Framework Convention on Climate Change (UNFCCC).
It addresses the special needs of the Least Developed
To obtain information on an annual basis of waterbird Countries (LDCs) that are especially vulnerable to the
populations at wetlands in the region during the non- adverse impacts of climate change. It is administered by
breeding period of most species (January), as a basis for the Global Environment Facility.
evaluation of sites and monitoring of populations.
To monitor on an annual basis the status and condition 238. Solution (b)
of wetlands. Exp) Option b is correct.

421 Workbook
.
ENVIRONMENT

The Department of Economic Affairs, Ministry Statement 3 is correct: It helps in financing climate
of Finance has launched the Sustainable Finance change mitigation and biodiversity conservation
Collaborative (SFC) in partnership with the United programmes in developing, emerging and transition
Nations Development Programme (UNDP). It will boost countries.
Inda’s efforts on mobilizing sustainable and green finance.
The Collaborative provides valuable insights and tangible 242. Solution (c)
recommendations that would be helpful in developing a Exp) Option c is correct.
Sustainable Finance framework/roadmap for India. Statement 1 is correct: The Coalition for Sustainable
Climate Action is launched by the International Solar
239. Solution (a) Alliance (ISA) Secretariat in 2020.
Exp) Option a is correct. Statement 2 is correct: Coalition for Sustainable Climate
Statement 1 is correct: The Minamata Convention on Action comprises of global public and private corporates.
Mercury aims to Controlling the anthropogenic release They will contribute towards ISA’s corpus fund. Recently
of mercury throughout its lifecycle. The Convention five Public Sector Undertakings (PSUs) under Petroleum
addresses the interim storage of mercury and its disposal and Natural Gas Ministry joined Coalition for Sustainable
once it becomes a waste, site contaminated with mercury Climate Action (ISA-CSCA) as Corporate Partners.
as well as health issues. The Minamata Convention was
agreed in the fifth session of the Intergovernmental 243. Solution (b)
Negotiating Committee in Geneva, Switzerland and Exp) Option b is correct.
entered into force on August 2017. More than 140 The Global Climate Risk Index is an annual publication
countries including India have ratified the Convention. of the global think tank Germanwatch. Germanwatch
Statement 2 is correct: The Rotterdam Convention works for environment related issues and sustainable
aims to promote shared responsibilities in relation to development.
importation of hazardous chemicals. The Rotterdam The report takes into account the effects of weather-
Convention was adopted in September 1998 and came related events on Nations and upholds the need to support
into force in 2004. India is a party to the Rotterdam developing countries to deal with climate change effects.
convention with 161 other parties. As per the latest report published in 2021
Statement 3 is incorrect: The Minamata Convention is 1. Mozambique and Bahamas were the worst affected
funded by the Global Environment Facility. The Rotterdam countries in 2019.
Convention is funded by the member countries. 2. India was placed better than Afghanistan and Japan
in the index
240. Solution (b) 3. In India, cyclone Fani, which hit the country in 2019
Exp) Option b is correct. was the worst disaster affecting around 28 million
people and killing approximately 90 people in India
Statement 1 and 3 are correct: India’s Intended Nationally
and Bangladesh.
Determined Contributions (INDCs) targets include
4. Also, floods caused by heavy rains and intense
• To reduce the emissions intensity of its GDP by 33 to monsoon affected millions of people in India causing
35 per cent by 2030 from 2005 level. enormous economic damage.
• To create an additional carbon sink of 2.5 to 3 billion
Tonnes of CO2 equivalent through additional forest 244. Solution (a)
and tree cover by 2030. Exp) Option a is correct.
Statement 2 is incorrect: Other important target is - to Statement 1 is correct. The purpose of the Warsaw
achieve about 40 per cent cumulative electric power International Mechanism is to implement appropriate
installed capacity from non-fossil fuel-based energy approaches to address loss and damage—particularly
resources by 2030 with the help of transfer of technology where loss and damage pushes society to reexamine
and low-cost international finance including from Green current ways of thinking and managing climate risks.
Climate Fund. It focuses on enhancing action and support, including
finance, technology and capacity-building, to address loss
241. Solution (d) and damage associated with the adverse effects of climate
change and to enable countries to undertake actions.
Exp) Option d is correct.
Statement 2 is incorrect Warsaw International Mechanism
Statement 1 is incorrect: International Climate Initiative
for Loss and Damage associated with Climate Change
is an important instrument of the German Government
Impacts (WIM) is an initiative of UNFCCC. The COP
(German Federal Ministry for the Environment). of UNFCCC established it to address loss and damage
Statement 2 is correct: International Climate Initiative associated with impacts of climate change, in developing
operates within the framework of the United Nations countries that are particularly vulnerable to the adverse
Framework Convention on Climate Change (UNFCCC) effects of climate change. It was established at COP19
and the Convention on Biological Diversity (CBD). (November 2013) in Warsaw, Poland.

Workbook 422
.
ENVIRONMENT

245. Solution (a) adoption and to inform the next round of Nationally
Exp) Option a is correct. Determined Contributions (NDCs) with a view to raise
climate ambition. Under the Talanoa process, stakeholders
The Sixth Assessment Report (AR 6) is released by the talked about the three agreed guiding questions on the
Intergovernmental Panel on Climate Change (IPCC). climate crisis:
The report, Climate Change 2021: the Physical Science
Basis, for the first time gave cognisance to the role of 249. Solution (b)
compound extremes and multiple climate change drivers Exp) Option b is correct.
operating in tandem in maximising disaster impacts in
India and elsewhere. Statement 1 is incorrect. Coalition for Disaster Resilient
Infrastructure (CDRI) Initiative has 22 member
The IPCC report also spelt out with “moderate confidence” countries and 7 organisations.
how urbanisation has pushed up intense rainfall in cities
across South Asia using several scientific evidences It is a multi-stakeholder global partnership of national
generated on India cities. It also noticed that, burgeoning governments, UN agencies, multilateral development
air pollution has reduced the intensity as well as frequency banks, private sector, academic and knowledge
of monsoon rains in India and the rest of south Asia. institutions. It aims to promote the resilience of new and
existing infrastructure systems to climate and disaster
246. Solution (a) risks.
Exp) Option a is correct. Statement 2 is correct. The Governing Council shall
The London Convention on the Prevention of Marine be co-chaired by representatives of two national
Pollution by Dumping of Wastes and Other Matter governments. India shall be permanent co-chair of
(1972) aims to promote the effective control of all sources the Governing Council. The other co-chair shall be
of marine pollution and to take all practicable steps to nominated by rotation by the members of the Governing
prevent pollution of the sea by dumping of wastes and Council, every two years.
other matter. Statement 3 is incorrect. There are no obligations on
MARPOL convention (The International Convention the part of members to make financial contributions to
for the Prevention of Pollution from Ships) of 1973 - It CDRI. However, at any point, members of the CDRI may
covers pollution of the marine environment by ships from make voluntary contributions.
operational or accidental causes. It lists various forms of
250. Solution (b)
marine pollution caused by oil, noxious liquid substances,
and harmful substances in packaged form, sewage and Exp) Option b is correct.
garbage from ships, etc. Option a is incorrect. Beach Environment & Aesthetics
Management Services (BEAMS) that comes under
247. Solution (d) ICZM (Integrated Coastal Zone Management) project.
Exp) Option d is correct. This was launched by the Society of Integrated Coastal
Statement 1 is incorrect. The Kigali Amendment to the Management (SICOM) and the Union Ministry of
Montreal Protocol is legally binding with mandatory Environment, Forest and Climate Change (MoEFCC).
targets for countries. UNEP is not involved.
Statement 2 is incorrect. The Kigali Amendment aims Option b is correct. On the lines of Blue Flag
to phase down hydrofluorocarbons (HFCs) as they are certification, India has also launched its own eco-label
potent greenhouse gases with global warming potential BEAMS (Beach Environment & Aesthetics Management
(GWP) thousand times more than carbon dioxide (CO2). Services). It will help India strive for globally recognized
They were developed to replace stratospheric ozone- and the coveted eco-label ‘Blue flag”.
depleting substances (ODS). So they are not ozone- Option c is incorrect. No such award is provided. The
depleting substances. objectives of BEAMS program is to:
Statement 3 is correct. The Kigali agreement is an Abate pollution in coastal waters,
amendment to the Montreal Protocol. Promote sustainable development of beach facilities,
Statement 4 is correct. In August 2021, the Union Protect & conserve coastal ecosystems & natural resources,
Government approved the ratification of the Kigali
Amendment to the Montreal Protocol on phasing challenge local authorities & stakeholders to strive and
down climate-damaging refrigerant Hydrofluorocarbons maintain high standards of cleanliness, hygiene & safety
(HFCs). for beachgoers in accordance with coastal environment &
regulations.
248. Solution (c) The program is based on stakeholder approach and
Exp) Option c is correct. hence also involves local communities in beach
The Talanoa Dialogue was convened as part of the management program.
UN climate talks. Talanoa Dialogue takes stock of Option d is incorrect. The program involves no such
the progress of climate action since Paris Agreement provision.

423 Workbook
.
ENVIRONMENT

251. Solution (d) climate change, to enable countries to undertake actions.


Exp) Option d is correct. Statement 3 is incorrect. It to address losses and damage
Statement 1 is incorrect. Coalition for Sustainable in developing countries (not developed countries) that
Climate Action (CSCA) is an initiative of the are particularly vulnerable to the adverse effects of climate
International Solar Alliance (ISA). change.
Statement 2 is incorrect. CSCA will take measures to 254. Solution (b)
attract corporate members who operate in member Exp) Option b is correct
countries and can help fight climate challenges in all the
four regions of the ISA. The FAO Food Price Index (FFPI) is a measure of the
monthly change in international prices of a basket of food
252. Solution (c) commodities. It consists of the average of five commodity
Exp) Option c is correct. group price indices weighted by the average export shares
of each of the groups over 2014-2016.
Regional Integrated Multi-Hazard Early Warning System
(RIMES) has partnered with the World Bank Group to The five commodity groups are:
implement the Climate Adaptation and Resilience Project • The FAO Cereal Price Index
(CARE). • The FAO Vegetable Oil Price Index
Statement 1 is correct: It aims to provide access to • The FAO Dairy Price Index
detailed weather, water, and climate data and decision-
support systems (DSS). This will assist policymakers and • The FAO Meat Price Index
sectorial users enhance climate resilience and reduce the • The FAO Sugar price index
socio-economic impacts of natural disasters.
255. Solution (a)
Statement 2 is correct: As hazards and their impacts
are interconnected, efforts towards enhancing climate Exp) Option a is correct.
adaptation and resilience could also help reduce India and Denmark have launched the Green Strategic
the impact of future pandemics like COVID-19. Partnership for delivering sustainable solutions to
Example: Some states like West Bengal, Maharashtra India. The Partnership will focus on expanding economic
faced issues like cyclones and flood hazards recently ties, green growth, and cooperation on global challenges
which negatively impacted their efforts to control the such as climate change. It will give Denmark a special
pandemic. place in delivering sustainable solutions to India.
CARE is initially implemented in three countries that The Green Strategic Partnership is a mutually beneficial
face the highest climate risk in the South Asian region arrangement to advance political cooperation, expand
– Bangladesh, Nepal, and Pakistan. Under the project, economic relations and green growth, create jobs
RIMES will help develop an open Regional Resilience and and strengthen cooperation on addressing global
Data Analytics Platform, and various DSSs for agriculture, challenges and opportunities; with focus on an ambitious
transport, water resources, finance and planning sectors. implementation of the Paris Agreement and the UN
Sustainable Development Goals.
253. Solution (b)
Exp) Option b is correct. 256. Solution (a)
Statement 1 is incorrect. The Warsaw International Exp) Option a is correct.
Mechanism for Loss and Damage associated with Statement 1 is correct: The Wildlife Trade Monitoring
Climate Change Impacts was established at the COP19 Network (TRAFFIC) is an NGO jointly formed by
(Warsaw, Poland) UNFCCC in November 2013. the World Wide Fund for Nature (WWF) and IUCN
The Mechanism’s role was recognized in 2015 in Article for monitoring the trade in wild animals and plants
8 of the Paris Agreement and it was reviewed in 2019 at in the context of both biodiversity conservation and
COP25. sustainable development. CITES is a convention
between countries and TRAFFIC compliments the
Statement 2 is correct. Functions of the Loss and Damage CITES in the task of monitoring the international trade
Mechanism in wild animals and plants.
Pillar 1. Enhancing knowledge and understanding of Statement 2 is correct: TRAFFIC aims to ensure that
comprehensive risk management approaches to address international trade of wild animals and plants does not
loss and damage associated with the adverse effects of threaten their survival and conservation. It investigates
climate change, including slow onset impacts. and analyses wildlife trade trends, patterns, impacts and
Pillar 2. Strengthening dialogue, coordination, coherence drivers to provide the leading knowledge base on trade in
and synergies among relevant stakeholders. wild animals and plants.
Pillar 3. Enhancing action and support, including Statement 3 is incorrect: ‘Not All Animals Migrate
finance, technology and capacity-building, to address by Choice’ is a joint initiative of United Nation
loss and damage associated with the adverse effects of Environment Programme and Wildlife Crime Control

Workbook 424
.
ENVIRONMENT

Bureau (WCCB) of India. It is an awareness campaign The Global Initiative on Reducing Land Degradation
launched at major airports across the country. It aims aims to strengthen the implementation of existing
at creating awareness and garnering public support for frameworks to prevent, halt, and reverse land
the protection and conservation of wildlife, prevention degradation within G20 member states and globally.
of smuggling and reduction in demand for wildlife It also takes into account possible implications on the
products. achievement of other SDGs and adhering to the principle
of doing no harm.
257. Solution (a)
Exp) Option a is correct. 260. Solution (a)
Relevance) Environment Minister, 0n 18th September, Exp) Option a is correct.
2020, announced that eight Indian beaches have Statement 1 is correct. The United in Science Report
been recommended for the international Blue Flag 2020 is a multi-agency report from leading science
certification. organizations. The report has been compiled by the
Statement 1 is correct. The Blue Flag Programme for World Meteorological Organization (WMO) under
beaches and marinas is operated by the Foundation for the direction of the United Nations Secretary-General.
Environmental Education (FEE), an international, The report also got inputs from the Global Carbon
non-governmental, non-profit organisation. It is Project, the Intergovernmental Panel on Climate Change,
headquartered in Copenhagen, Denmark. the Intergovernmental Oceanographic Commission of
In India, the national operating agency is CEE(Centre for UNESCO, the UN Environment Programme and the UK
Environmental Education) under MoEFCC. Met Office.
Statement 2 is incorrect. The ‘Blue Flag’ is a certification Statement 2 is incorrect. The United in Science Report
that can be obtained by a beach, marina, or sustainable shows the increasing and irreversible impacts of
boating tourism operator, and serves as an eco-label. climate change, which affects glaciers, oceans, nature,
The programme promotes sustainable development economies and living conditions. The United in Science
in freshwater and marine areas through four main 2020 report, the second in a series, presents the very latest
criteria: water quality, environmental management, scientific data and findings related to climate change to
environmental education and safety. The programme inform global policy and action.
identifies beaches for providing tourists a clean and
hygienic bathing water, facilities/amenities, a safe and 261. Solution: (d)
healthy environment, and sustainable development of the Exp) Option d is the correct answer.
area.
There has been creation of several institutions and
258. Solution (a) programs across the globe to deal with the issue of
environment. Some of the programs and their aims are
Exp) Option a is correct.
as follows:
Statement 1 is incorrect. The objective of CSCAF (2.0) is
to provide a clear roadmap for cities towards combating Pair 1 is incorrect – It is United Nation Momentum
Climate Change while planning and implementing for Change Award and not Climate and Clean Air
their actions, including investments. The framework has Coalition. The Climate & Clean Air Coalition is the only
five categories namely; 1. Energy and Green Buildings, global effort that unites governments, civil society and
2. Urban Planning, Green Cover and Biodiversity, 3. private sector, committed to improving air quality and
Mobility and Air Quality, 4. Water Management, 5. Waste protecting the climate in next few decades by reducing
Management. short-lived climate pollutants across sectors.
Statement 2 is correct. The programme is implemented by Pair 2 is incorrect - Climate Technology Centre and
the Ministry of Housing and Urban Affairs (MoHUA). Network is the operational arm of UNFCCC technology
The Climate Centre for Cities under National Institute Mechanism and it is hosted and managed by UNEP in
of Urban Affairs (NIUA) is supporting MoHUA in collaboration with UNIDO and with the support of 11
implementation of CSCAF. Centers of Excellence located in developing and developed
Statement 3 is correct. CSCAF initiative intends to countries. It is located in Copenhagen, Denmark. It does
inculcate a climate-sensitive approach to urban not provide funding directly to countries but instead
planning and development in India. It will help to reduce supports the provision of technical assistance provided
the impacts of disasters like cyclones, floods, heat waves, by experts on specific climate technology sectors.
water scarcity and drought-like conditions, on many of Pair 3 is incorrect – The aim of Green Climate Fund is to
our cities. gather finances for the developing and least developed
countries for adaptation and mitigation measures. It is
259. Solution (b)
the Climate Technology Centre and Network which is
Exp) Option b is correct. operational arm of UNFCCC technology Mechanism.
The Global Initiative on Reducing Land Degradation
was launched at the Environment Ministerial Meeting 262. Solution: (b)
(EMM) of the G20 countries. Exp) Option b is the correct answer.

425 Workbook
.
ENVIRONMENT

Kyoto Protocol and Paris Agreement were the two major Fund has been adopted as the permanent financing
milestones in the history of emission reduction for mechanism of UNFCCC. Over time GCF is expected to
greenhouse gases. The differences between the two are: become the main multilateral financing mechanism to
Option a is incorrect – Kyoto Protocol, sets binding support climate action in developing countries.
emission reduction targets for 37 industrialized
264. Solution: (a)
countries and the European community in its first
commitment period. It didn’t exempt industrialized Exp) Option a is correct.
countries. Paris agreement on the other hand, included United Nations Office for Disaster Risk Reduction
both developed and developing countries diluting the (UNDRR) brings governments, partners and communities
principle of Common but differentiated responsibility. together to reduce disaster risk and losses and to ensure a
Option b is correct – The Kyoto Protocol focused only safer, sustainable future.
on major 6 Greenhouse gas emissions. They are Carbon UNISDR’s Strategic Framework 2016-2021 guides the
dioxide methane, sulfur hexafluoride, HFCs, PFCs and organization to achieve substantial reduction of disaster
nitrous oxide. While the Paris agreement rather focussed risk and losses for a sustainable future.
on reducing the emissions of all the major anthropogenic
Statement 1 is correct. UNDRR oversees the
greenhouse gases. There was no specific GHGs taken
implementation of the Sendai Framework for Disaster
and rather generally all the greenhouse gases were to be
included in calculation as per the agreement. Risk Reduction 2015-2030, supporting countries in its
implementation, monitoring and sharing what works in
Option c is incorrect - The specific goal of the Paris climate reducing existing risk and preventing the creation of new
agreement was to reduce greenhouse gas emissions so that risk.
the global average temperature would not rise more than 2
degrees Celsius above pre-industrial levels with an earnest Statement 2 is incorrect. The Gender Dimensions of
effort to not allow it to rise 1.5 degrees Celsius above Disaster Risk and Resilience (2021) report is an annual
pre-industrial levels. The goal in Kyoto Protocol was to publication of World Bank.
reduce greenhouse gas emissions to 5.2% below 1990 Statement 3 is incorrect. The Global Facility for Disaster
levels by 2012. There was no consideration to control Reduction and Recovery (GFDRR) is a global partnership
specific amount of temperature rise in the Kyoto Protocol. managed by the World Bank.
Option d is incorrect – Kyoto Protocol binds developed
countries because it recognizes that they are largely 265. Solution: (d)
responsible for the current high levels of GHG emissions Exp) Option d is the correct answer.
in the atmosphere, which are the result of more than 150 The ecosystem approach is a strategy for the integrated
years of industrial activity. On the other hand, while Paris management of land, water and living resources that
Agreement increased accountability through National
promotes conservation and sustainable use in an equitable
Determined Contributions, it was voluntary and there
way.
was no legally binding provisions.
Statement 1 is incorrect. At its second meeting,
263. Solution: (a) held in Jakarta, November 1995, the Conference of
Exp) Option a is the correct answer. the Parties (COP) of the Convention on Biological
UNFCCC has established mechanisms like Green Climate Diversity (CBD) adopted the ecosystem approach as
Fund (GCF), Global Environment Facility (GEF) for the primary framework for action under the Convention,
channeling finances to mitigate climate disasters. Features and subsequently has referred to the ecosystem approach
of these are: in the elaboration and implementation of the various
thematic and cross-cutting issues work programmes
Statement 1 is correct - GCF was established under the under the Convention.
Cancún Agreements in 2010 as a dedicated financing
vehicle for developing countries within the global climate Statement 2 is incorrect. An ecosystem approach is based
architecture, serving the Financial Mechanism of the on the application of appropriate scientific methodologies
UNFCCC and the Paris Agreement. GCF is mandated focused on levels of biological organization. This
to invest 50% of its resources to mitigation and 50% encompasses the essential structure, processes, functions
to adaptation in grant equivalent. At least half of its and interactions among organisms and their environment.
adaptation resources must be invested in the most climate It recognizes that humans, with their cultural diversity,
vulnerable countries (SIDS, LDCs, and African States). are an integral component of many ecosystems.
The GCF programming strategy recognizes that we must
scale up both mitigation and adaptation efforts. GCF aims 266. Solution: (b)
to leverage synergies and minimize potential trade-offs Option b is the correct answer.
between adaptation and mitigation. A natural ecosystem is the interaction between the
Statement 2 is incorrect - Article 21 Of UNFCCC names environment and the organisms living in the environment.
the GEF to serve as the financial mechanism on an It occurs naturally in nature and requires no human
interim basis (not on permanent basis). Green Climate intervention for survival. Examples: rivers, forests, etc.

Workbook 426
.
ENVIRONMENT

Artificial ecosystems are human made structures where Statement 2 is correct: REDD+ is an extension of the
biotic and abiotic components are made to interact with REDD initiative. The “+” here includes, “sustainable
each other for survival. It is not self-sustaining. Examples: management of forests, conservation, and enhancement
aquariums, agriculture farmlands, zoological parks etc of forest carbon stocks”. It creates a financial value for
Statement 1 is incorrect. A natural ecosystem is self- the carbon stored in forests by offering incentives for
sustaining and self-perpetuating. This means that it developing countries to reduce emissions from forested
does not need any outside help to sustain itself. But, an lands and invest in low-carbon paths to sustainable
artificial ecosystem, on the other hand, cannot sustain development. REDD+ is a more comprehensive forest
itself without the help of humans. management initiative which includes incentives for
conservation and sustainable management of forests,
Statement 2 is correct. Natural ecosystems are more beyond a mere reduction in deforestation.
diverse than artificial ecosystems. This is because natural
ecosystems generally contain more biotic and abiotic 269. Solution: (b)
factors than artificial ecosystems. Exp) Option b is correct.
267. Solution: (b) In Warsaw in 2013, all parties agreed to set up a new
Exp) Option b is correct. mechanism for Loss and Damage.
The Sendai Framework is the successor instrument to the Option b is correct. The 19th session of the Conference
Hyogo Framework for Action (HFA) 2005-2015: Building of Parties (COP19) established the Warsaw International
the Resilience of Nations and Communities to Disasters. Mechanism for Loss and Damage to address loss and
The Sendai Framework for Disaster Risk Reduction 2015- damage associated with impacts of climate change,
2030 (Sendai Framework) was the first major agreement of including extreme events and slow onset events, in
the post-2015 development agenda and provides Member developing countries that are particularly vulnerable to
States with concrete actions to protect development gains the adverse effects of climate change.
from the risk of disaster. The issue is important because even after GHG emissions
are reduced and communities adapt to climate change,
Statement 1 is incorrect. The Sendai Framework is
there would still be loss and damage to people, livelihoods
a 15-year, voluntary, non-binding agreement which
and infrastructure as a result of their inability to cope
recognizes that the State has the primary role to reduce
with climate change. Loss generally refers to the complete
disaster risk, but that responsibility should be shared
forfeiture of items like land, ecosystems, or of human
with other stakeholders including local governments, the
lives, while damage refers to the harm to infrastructure
private sector, the scientific community and NGOs.
and property that could be repaired. The term includes
Statement 2 is correct. It aims for a substantial reduction both economic and non-economic losses.
in disaster losses resulting from both man-made and
natural hazards. It places almost equal importance on 270. Solution: (a)
all kinds of disasters and not only on those arising from Exp) Option a is correct.
natural hazards.
The Kyoto Protocol aims to limit or reduce the greenhouse
Statement 3 is correct. There is significant shift from gas emissions by three market-based mechanisms –
earlier emphasis on disaster management to addressing emissions trading, clean development mechanism and
disaster risk management itself by focusing on the joint implementation.
underlying drivers of risk. Pair 1 is incorrect. Emissions trading mentioned in
Statement 4 is correct. For the first time the goals Article 17 of the Kyoto Protocol, where an advanced
are defined in terms of outcome-based targets instead country “A” can acquire emission units from an advanced
of focusing on sets of activities and actions. It places country “B” for meeting a part of their emission reduction
governments at the center of disaster risk reduction with target.
the framework emphasizing the need to strengthen the Pair 2 is correct. Clean development mechanism defined
disaster risk governance. in Article 12 of the Kyoto Protocol, where an advanced
268. Solution: (b) country can implement emission reduction projects in a
developing country to earn certified emission reduction
Exp) Option b is correct. (CER) credits which can be traded to achieve their
REDD and REDD+ are a climate change mitigation reduction targets.
mechanism that works to reduce greenhouse gas Pair 3 is incorrect. Joint implementation defined in
emissions by paying developing countries to stop cutting Article 6 of the Kyoto Protocol, is where an advanced
down their forests. country “A” and an advanced country “B” may take part
Statement 1 is incorrect: REDD stands for Reducing in an emission-reduction project and count the resulting
Emissions from Deforestation and Forest Degradation, emission units towards meeting its Kyoto target.
and it is a global initiative negotiated under the United
Nations Framework Convention on Climate Change 271. Solution: (a)
(UNFCCC) in 2005 (and not UNCCD). Exp) Option a is correct.

427 Workbook
.
ENVIRONMENT

Statement 1 is correct: International customary law refers 273. Solution: (c)


to international law norms derived from a custom. It Exp) Option c is correct.
derives from “a general practice accepted as law”. Such
practice can be found in official accounts of military Statement 1 is correct. World Conservation Monitoring
Centre (WCMC) is a UK non-profit organisation dealing
operations but is also reflected in a variety of other
with the handling and management of information
official documents, including military manuals, national
on the conservation of biodiversity. Its three founder
legislation and case law.
organisations were IUCN - The World Conservation
Statement 2 is incorrect: International customary laws Union, the World Wide Fund for Nature (WWF) and the
are binding legal rules that have developed on global or United Nations Environment Programme (UNEP).
region levels through continued practice. States recognize
Statement 2 is correct. United Nations Environment
that treaties and customary international law are sources
Programme Finance Initiative (UNEP FI) is a
of international law and, as such, are binding. This is set
partnership between UNEP and the global financial
forth, for example, in the Statute of the International
sector to mobilize private sector finance for sustainable
Court of Justice. One example of the binding nature
development. UNEP FI works with more than 400
of customary international law is its application by
banks, insurers, and investors and over 100 supporting
national and international courts and tribunals.
institutions – to help create a financial sector that serves
Statement 3 is incorrect: Supreme Court of India has people and planet while delivering positive impacts. We
been of the view that customary international law (CIL), aim to inspire, inform and enable financial institutions
unless contradictory to domestic law, is part of the Indian to improve people’s quality of life without compromising
legal regime even if the parliament has not enacted a law that of future generations. By leveraging the UN’s role,
for that. In Vellore Citizens Welfare Forum v. Union UNEP FI accelerates sustainable finance.
of India it held that CIL which is not contrary to the
municipal law shall be deemed to have been incorporated 274. Solution: (b)
in India’s domestic law. This principle has been affirmed Exp) Option b is correct.
in subsequent decisions like in Research Foundation Statement 1 is incorrect. The Yokohama Strategy and
for Science v. Union of India. Here, the SC, relying on plan of Action for a Safer World (1994), as the first
the Vellore Citizen case, declared that the precautionary major international framework for disaster risk reduction,
principle, an environmental law concept, is part of CIL recognized the interrelation between sustainable
and thus part of Indian law. development and DRR.
272. Solution: (b) The Johannesburg Declaration, 2002 on Sustainable
Exp) Option b is correct. Development was adopted at the World Summit on
Sustainable Development (WSSD). Here the Plan of
The Intergovernmental Panel on Climate Change (IPCC) Implementation of the World Summit on Sustainable
is the United Nations body for assessing the science Development was agreed upon
related to climate change.
Statement 2 is correct. In September 2015, the UN
Statement 1 is incorrect. The ocean’s uptake of General Assembly adopted the 2030 Agenda for
anthropogenic carbon affects its chemistry in a process Sustainable Development that includes 17 Sustainable
referred to as ocean acidification, which increases the Development Goals (SDGs). The 2030 Agenda for
concentrations of aqueous CO2, bicarbonate and hydrogen Sustainable Development recognizes and reaffirms the
ions, and decreases pH, carbonate ion concentrations and urgent need to reduce the risk of disasters. In addition
calcium carbonate mineral saturation states. The pH of to direct references to the outcomes of the Third UN
seawater water gets lower as it becomes more acidic. Conference on DRR (Sendai Framework), there are
Statement 2 is correct. Kelp ecosystems will decline with specific opportunities to achieve SDGs through reducing
the increased frequency of heatwaves and sea temperature disaster risk. For example, by reducing exposure and
extremes as well as through the impact of invasive vulnerability of the poor to disasters or building resilient
subtropical species. The Climate change will contribute infrastructure.
to the continued decline in the extent of kelps in the
temperate zone and the range of kelp in the Northern 275. Solution: (a)
Hemisphere will expand poleward. Exp) Option a is correct.
Statement 3 is correct. The Global mean Sea Surface Statement 1 is correct. In January 2005, 168
Temperature has increased since the beginning of the Governments adopted a 10-year plan to make the world
20th century from 1850–1900 to 2011–2020 by 0.88°C safer from natural hazards at the World Conference
(very likely range: 0.68– 1.01°C). The key characteristic on Disaster Reduction, held in Kobe, Hyogo, Japan.
of ocean temperature change relevant for ecosystems is The Hyogo Framework is a global blueprint for disaster
climate velocity, a measure of the speed and direction risk reduction efforts during the next decade. Its goal is
at which isotherms move under climate change which to substantially reduce disaster losses by 2015 - in lives,
gives the rate at which species must migrate to maintain and in the social, economic, and environmental assets
constant climate conditions. of communities and countries. The Framework offers

Workbook 428
.
ENVIRONMENT

guiding principles, priorities for action, and practical Waste Management Amendment Rules, 2022 under the
means for achieving disaster resilience for vulnerable provision of the Environment Protection Act, 1986.
communities.
277. Solution: (b)
Statement 2 is correct. The Sendai Framework is the
successor instrument to the Hyogo Framework for Exp) Option b is the correct answer.
Action (HFA) 2005-2015: Building the Resilience of FASTER (Fast and Secured Transmission of Electronic
Nations and Communities to Disasters. It is the outcome Records) Platform is a digital platform to communicate
of stakeholder consultations initiated in March 2012 and interim orders, stay orders, and bail orders of the
inter-governmental negotiations held from July 2014 to Supreme Court to authorities concerned through a
March 2015, which were supported by the UNDRR upon secured electronic communication channel. It is developed
the request of the UN General Assembly. by Registry of the Supreme Court in collaboration with
Statement 3 is incorrect. United Nations Office for the National Informatics Centre (NIC).
Disaster Risk Reduction, is tasked to support the FASTER Cell has been established in the Registry of the
implementation, follow-up and review of the Sendai Supreme Court. The cell will transmit digitally signed
Framework. records of proceedings or orders related to bail and release
276. Solution: (c) passed by the court to the nodal officers and duty holders
concerned through email.
Exp) Option c is the correct answer.
To reach all districts of India, 73 nodal officers have been
Plastic Waste Management refers to managing the plastic nominated at various levels. All nodal officers have been
waste generated and processing it to make it reusable. connected through a specific Judicial Communication
India has been actively taking steps to effectively manage Network (JCN) by creating a secured pathway.
the plastic waste as seen by frequent amendments to
the Plastic Waste Management Rules. The new rules are 278. Solution: (d)
formed in order to achieve broader goal of sustainable Exp) Option d is correct.
development through circular economy. Circular
Economy focuses on increasing productivity in terms of NITI Aayog has constructed the SDG India Index
more efficient utilization of resources. spanning across 16 out of 17 SDGs (leaving out Goals 12,
13, 14 and 17). The Index tracks the progress of all the
Statement 1 is incorrect.
States and Union Territories (UTs) on a set of 62 National
The Ministry of Environment has launched new rules Indicators, measuring their progress on the outcomes
called the Plastic Waste Management (Amendment) Rules, of the interventions and schemes of the Government of
2022. For the first time, its guidelines allow for the sale India.
and purchase of surplus extended producer responsibility
(EPR) certificates. This is a system whereby makers and Statement 1 is correct. The SDG India Index computes
users of plastic packaging can collect certificates — called goal-wise scores on the 16 SDGs for each State and Union
Extended Producer Responsibility (EPR) certificates — Territory.
and trade in them. (Extended Producer Responsibility Statement 2 is correct. The Ministry of Statistics and
(EPR) means the responsibility of a producer for the Programme Implementation has developed the National
environmentally sound management of the product Indicator Framework (NIF) of SDGs to assist the
(plastic packaging) until the end of its life.) Government in monitoring and tracking the progress of
The certificates will help organisations to make up SDGs at national level, based on the official data provided
for their shortfall from other organisations that have by Ministries, Departments and State Governments.
used recycled content in excess of their obligation. Statement 3 is correct. The first edition of the North
Thus, setting up a market mechanism for plastic waste Eastern Region (NER) District SDG Index Report was
management. released by NITI Aayog and Ministry of Development of
Statement 2 is correct. The Hazardous and Other North Eastern Region in 2021. It is a collaborative effort
Wastes (Management and Transboundary Movement) by NITI Aayog and Ministry of DoNER, with technical
Rules, 2016 as amended in 2019 regulate the import support from UNDP. The Index measures the performance
of identified plastic waste into the country by SEZ and of the districts of the eight States of Arunachal Pradesh,
EOUs. It prohibits import of Solid plastic waste into the Assam, Manipur, Meghalaya, Mizoram, Nagaland, Sikkim
country from Special Economic Zones (SEZ) and by and Tripura on the Sustainable Development Goals. It
Export Oriented Units (EOU). The regulation of import also measures their corresponding targets and ranks
of plastic waste prevents dumping of plastic waste by the districts based on the same.
other countries in the country and allows for recycling of
plastic waste generated in the country. 279. Solution: (a)
Statement 3 is correct. Ministry of Environment, Exp) Option a is the correct answer.
Forest and climate Change (MoEF & CC) framed both The Global Wind Energy Council (GWEC) was established
The Hazardous and Other Wastes (Management and in 2005 to provide a credible and representative forum for
Transboundary Movement) Rules, 2016 and Plastic the entire wind energy sector at an international level.

429 Workbook
.
ENVIRONMENT

Key Findings of the Global Wind Report 2022: The total onshore wind installations in 2021 were still
18% lower than the previous year. The decline was driven
The global wind industry had its second-best year in 2021
primarily by the slow-down of onshore wind growth in
with almost 94 GW of capacity added globally, trailing the world’s two largest wind power markets, China and
behind the 2020s record growth by only 1.8%. the US.

Workbook 430

You might also like